Está en la página 1de 341

AM

PLIA
CIONES
AM
manual de
AMPLIACIONES

1
MANUAL APIR
AMPLIACIONES
1º Edición - Marzo 2020

ISBN
978-84-120385-1-4

Academia de preparación PIR S.L.


www.academiapir.com
admin@academiapir.com

APIR es una marca registrada de la Academia de Preparación PIR S.L.

DISEÑO,MAQUETACIÓN E ILUSTRACIONES
Laura González García, Academia de preparación PIR
COORDINACIÓN EDITORIAL:
Kazuhiro Tajima Pozo.
Alicia López Frutos.
Juana Portillo Abellán.

PSICOPATOLOGÍA
- Reseñas: Raquel Zuñiga Costa, Gessamí Vilarrubias Gutiérrez y Kazuhiro Tajima Pozo........... Pág 8

Ampliaciones:

Tema 7: PSICOPATOLOGÍA DEL PENSAMIENTO....................................................................... Pág 11


Raquel Zuñiga Costa y Kazuhiro Tajima Pozo.

PSICOLOGÍA CLÍNICA
- Reseñas: Pablo Rodríguez López, Laura Quiles Higuero, Anna Rodríguez Romero y Alberto Montero
Rosa......................................................................................................................................... Pág 14

Ampliaciones:

Tema 1: TRASTORNOS NEUROCOGNITIVOS............................................................................ Pág 22


María Soriano Medina, Laura Quiles Higuero, Pablo Rodríguez López.

Tema 3: TRASTORNOS ADICTIVOS Y RELACIONADOS CON SUSTANCIAS................................ Pág 27


Ana María Rodríguez Romero, Ana Amado Mera, Laura Quiles Higuero

Tema 4: ESPECTRO DE LA ESQUIZOFRENIA Y OTROS TRASTORNOS PSICÓTICOS. ....................Pág 36


Beatriz Torres Pardo, Pablo Rodríguez López, María Soriano Medina

Tema 5: TRASTORNOS DEPRESIVOS Y TRASTORNOS BIPOLARES Y RELACIONADOS

Tema 5.1: TRASTORNOS DEPRESIVOS......................................................................................Pág 39


Pablo Rodríguez López, Beatriz Torres Pardo, Ana María Rodríguez Romero

Tema 5.2: TRASTORNO BIPOLAR Y TRASTOR NOS RELACIONADOS..........................................Pág 47


Pablo Rodríguez López, Beatriz Torres Pardo, Ana María Rodríguez Romero

Tema 6: TRASTORNOS DE ANSIEDAD..................................................................................... Pág 48


Laura Quiles Higuero, Pablo Rodríguez López, María Soriano Medina.

Tema 7: TRASTORNO OBSESIVO-COMPUSIVO Y RELACIONADOS........................................... Pág 53


Laura Quiles Higuero, María Soriano Medina, Raquel Zúñiga Costa

Tema 8: TRASTORNOS RELACIONADOS CON EL TRAUMA Y EL ESTRÉS...................................Pág 56


María Soriano Medina, Laura Quiles Higuero,Pablo Rodríguez López

2
Tema 9: TRASTORNOS DE SÍNTOMAS SOMÁTICOS Y RELACIONADOS.....................................Pág 59
Pablo Rodríguez López, Beatriz Torres Pardo, Ana María Rodríguez Romero

Tema 10: TRASTORNOS FACTICIOS..........................................................................................Pág 61


Pablo Rodríguez López, Beatriz Torres Pardo, Ana María Rodríguez Romero

Tema 11: TRASTORNOS DISOCIATIVOS....................................................................................Pág 62


Beatriz Torres Pardo, Ana María Rodríguez Romero, Ana Amado Mera

Tema 12: TRASTORNOS ALIMENTARIOS Y DE LA INGESTIÓN....................................................Pág 65


María Soriano Medina, Laura Quiles Higuero,Pablo Rodríguez López

Tema 14: DISFUNCIONES SEXUALES.........................................................................................Pág 70


Raquel Zúñiga Costa, Pablo Rodríguez López,Beatriz Torres Pardo

Tema 15: TRASTORNOS PARAFÍLICOS. ....................................................................................Pág 75


Raquel Zúñiga Costa, Pablo Rodríguez López, Beatriz Torres Pardo.

Tema 16: DISFORIA DE GÉNERO. .............................................................................................Pág 79


Raquel Zúñiga Costa, Pablo Rodríguez López, Beatriz Torres Pardo.

Tema 17: TRASTORNOS DEL SUEÑO-VIGILIA.............................................................................Pág 81


Laura Quiles Higuero, María Soriano Medina, Raquel Zúñiga Costa.

Tema 18: TRASTORNOS DISRUPTIVOS, DEL CONTROL DE LOS IMPULSOS Y DE LA CONDUCTA............Pág 91


Ana Amado Mera, Laura Quiles Higuero, María Soriano Medina

Tema 19: TRASTORNOS DE PERSONALIDAD............................................................................Pág 94


Ana Amado Mera, Laura Quiles Higuero, María Soriano Medina

AMPLIACIONES MANUALES 3º EDICIÓN...............................................................................Pág 99


Laura Quiles Higuero, Ana María Rodríguez Romero, Alberto Montero Rosa.

CLÍNICA INFANTIL
- Reseñas: Sara Gallardo Gómez,Andere Maguregui García de la Torre y Adela Lozano García ..........Pág 102

Ampliaciones:

Tema 2. DISCAPACIDAD INTELECTUAL...................................................................................Pág 104


Montse Gallart Aliu, Andere Maguregui García de la Torre, Sara Gallardo Gómez.

Tema 3. TRASTORNO DEL ESPECTRO AUTISTA........................................................................Pág 105


Sara Gallardo Gómez, Montse Gallart Aliu, Andere Maguregui García de la Torre.

Tema 4. TRASTORNOS DE LA COMUNICACIÓN......................................................................Pág 108


Sara Gallardo Gómez, Montse Gallart Aliu, Andere Maguregui García de la Torre.

Tema 5. TRASTORNOS DEL APRENDIZAJE. .............................................................................Pág 111


Montse Gallart Aliu, Andere Maguregui García de la Torre, Sara Gallardo Gómez.

Tema 6. TRASTORNOS MOTORES...........................................................................................Pág 113


Andere Maguregui García de la Torre, Sara Gallardo Gómez, Montse Gallart Aliu.

3
Tema 10. TRASTORNOS DE LA EXCRECIÓN..............................................................................Pág 116
Juana Portillo Abellán, Beatriz Torres Pardo, Montse Gallart Aliu.

Tema 11. OTROS TRASTORNOS DE INICIO EN LA INFANCIA Y LA ADOLESCENCIA: TRASTORNO DE


ANSIEDAD POR SEPARACIÓN..................................................................................................Pág 118
Andere Maguregui García de la Torre, Beatriz Torres Pardo, Juana Portillo Abellán.

Tema 11. OTROS TRASTORNOS DE INICIO EN LA INFANCIA Y LA ADOLESCENCIA:MUTISMO SELECTI-


VO..........................................................................................................................................Pág 118
Beatriz Torres Pardo, Juana Portillo Abellán, Sara Gallardo Gómez.

TRATAMIENTOS PSICOLÓGICOS
- Reseñas: Marta López-Botet de Juan, Salvador Moreno Albert y Miriam Bersabé Pérez.........Pág 128

Ampliaciones: .

Tema 0: INTRODUCCIÓN....................................................................................................... Pág 136


Marta López-Botet de Juan

Tema 2: TRATAMIENTO DE LAS ADICCIONES.........................................................................Pág 137


Paula Rancaño Vázquez

Tema 3: TRATAMIENTO DE LA ESQUIZOFRENIA......................................................................Pág 137


Andrés Caballo López

Tema 4: TRATAMIENTO DE LOS TRASTORNOS DEL ESTADO DE ÁNIMO.................................Pág 140


Lara Jiménez Martín

Tema 7: TRATAMIENTO POR ESTRES POSTRAUMATICO..........................................................Pág 141


Marta López-Botet de Juan

Tema 15: TRATAMIENTO DE LOS TRASTORNOS DE PERSONALIDAD.......................................Pág 141


Andrés Caballo López

Tema 27: TRATAMIENTO DE LOS TRASTORNOS EMOCIONALES EN LA INFANCIA: ANSIEDAD Y DE-


PRESIÓN..................................................................................................................................Pág 146
Marta López-Botet de Juan

Tema 36: ASMA.....................................................................................................................Pág 147


Paula Rancaño Vázquez

PSICOLOGÍA DIFERENCIAL Y DE LA PERSONALIDAD


- Reseñas: María Soriano Medina y Nicolás Gómez Calmaestra. .............................................Pág 150

Ampliaciones:

Tema 14: LA IDENTIDAD PERSONAL. ....................................................................................Pág 154


Nicolás Gómez Calmaestra y María Soriano Medina.

4
PSICOTERAPIAS
- Reseñas: Joan Pau Marí Molina y Ángeles Fernández Sigüenza ............................................Pág 158

Ampliaciones:

Tema 2: LA APROXIMACIÓN HISTÓRICA A LAS PSICOTERAPIAS.............................................. Pág162


Sonia Salvador Coscujuela,Joan Pau Marí Molina,Helena Painceira Domínguez.

Tema 3: PSICOANÁLISIS I: FREUD..............................................................................................Pág162


Miriam Jiménez Buña, Joan Pau Marí Molina,Helena Painceira Domínguez.

Pág162
Tema 6: TERAPIAS DE FAMILIA Y MODELOS SISTÉMICOS.........................................................
Joan Pau Marí Molina,Sonia Salvador Coscujuela,Miriam Jiménez Bruña.

Tema 9: TERAPIAS COGNITIVAS...............................................................................................Pág173


Helena Painceira Domínguez,Amelia Villena Jimena, Sonia Salvador Coscujuela.

Tema 10: TERAPIAS CONDUCTUALES DE TERCERA GENERACIÓN............................................Pág174


Sonia Salvador Coscujuela, Amelia Villena Jimena,Joan Pau Marí Molina,Miriam Jiménez Bruña,Helena
Painceira Domínguez

Tema 11: MODELOS INTEGRADORES.......................................................................................Pág181


Joan Pau Marí Molina, Miriam Jiménez Bruña, Amelia Villena Jimena

ESTADÍSTICA, MÉTODO EXPERIMENTAL Y PSICOMETRÍA


- Reseñas: Mar Marañón Murcia. ..........................................................................................Pág 184

Ampliaciones: -

Tema 1: ESTADÍSTICA. ..........................................................................................................Pág 186


Eduardo Fernández Jiménez

Tema 2: MÉTODO CIENTÍFICO Y EXPERIMENTAL. ..................................................................Pág 187


Eduardo Fernández Jiménez.

Tema 3: PSICOMETRÍA. ..........................................................................................................Pág 188


Francisco Antonio García Gullón.

EVALUACIÓN PSICOLÓGICA
- Reseñas: Alicia López Frutos y Carmen Barreiro González. ..................................................Pág 190

Ampliaciones:

Tema 3: TÉCNICAS DE OBSERVACIÓN. ..................................................................................Pág 194


Alicia López Frutos y Belén Gago Velasco.

Tema 5: LA ENTREVISTA. .......................................................................................................Pág 197


Belén Gago Velasco y Alicia López Frutos.

Tema 6: TÉCNICAS OBJETIVAS. ..............................................................................................Pág 198


Alicia López Frutos y Belén Gago Velasco.
5
PSICOBIOLOGÍA
- Reseñas: Salvador Vizcaíno Montes de Oca, Ana carrasco Flores y Beatriz Gracia Biarge. .....Pág 202

Ampliaciones:

Tema 1: NIVEL CELULAR: La neurona.....................................................................................Pág 205


Gabriel Ródenas Perea.

Tema 14: FUNCIONES COGNITIVAS SUPERIORES. .................................................................Pág 205


Berta Pinilla Santos y Irene Alonso Fernández.

Tema 16: EMOCIÓN...............................................................................................................Pág 206


Iñigo Alberdi Páramo.

DESARROLLO PSICOLÓGICO
- Reseñas: Ana carrasco Flores................................................................................................Pág 208

Ampliaciones:

Tema 4: DESARROLLO COGNITIVO EN LA SEGUNDA INFANCIA. .............................................Pág 210


Javier Martín Ruiz

PSICOLOGÍA BÁSICA
- Reseñas: Beatriz Gracia Biarge.............................................................................................Pág 212

Ampliaciones:

Tema 2: ATENCIÓN................................................................................................................Pág 214


Javier Martín Ruiz.

Tema 3: MEMORIA. ..............................................................................................................Pág 214


Javier Martín Ruiz.

Tema 5: APRENDIZAJE............................................................................................................Pág 215


Francisco Antonio García Gullón.
.
PSICOLOGÍA DE LA SALUD
- Reseñas: Beatriz Torres Pardo y Marta Rosado García. .........................................................Pág 216

Ampliaciones:

Tema 1: INTRODUCCIÓN A LA PSICOSOMÁTICA. ..................................................................Pág 217


Beatriz Torres Pardo

PSICOLOGÍA SOCIAL
- Reseñas: Salvador Vizcaíno Montes de Oca. ........................................................................Pág 218

Ampliaciones:

Tema 4: PROCESOS DE INTERACCIÓN SOCIAL. .....................................................................Pág 220


Lorena Cabello Chillón.

6
ANEXOS................................................................................................................................Pág 222

ANEXO I :EXAMEN PIR 2020 (CONVOCATORIA 2019)

ANEXO II :EXAMEN PIR POR ÁREAS (CONVOCATORIA 2019)

7
PSICOPATOLOGÍA
RESEÑAS

RESEÑAS sujeto), tienen gran plasticidad y el sujeto no les otorga valor


de realidad.
• Aparecen en:
PÁG COLUMNA PÁRRAFO Consumo de sustancias (se llaman entonces fantasiopsias).

45 1 Incluir tabla 2

PÁG COLUMNA PÁRRAFO


TEXTO AMPLIADO
43 2 7
Paralogismos: a una palabra existente se le da un significado
diferente al que tiene en realidad (PIR19, 124)
TEXTO DEL MANUAL

6.8. Alteraciones mnésicas en trastornos mentales

PÁG COLUMNA PÁRRAFO Tabla 6. Anomalías en el recuerdo

20 1 Tras tabla 8 PSEUDOMEMORIAS Y FALSIFICACIÓN DE LA MEMORIA

Hacen referencia a la elaboración de recuerdos para rellenar


TEXTO AMPLIADO “vacíos” de memoria.
• Tres fenómenos:
Trastornos delirantes de Falsa Identidad. Munro incluye ade-
más de los Síndromes de Capgras y y Fregoli (tabla 8): - Confabulación: el paciente inventa recuerdos sin intención
de mentir (PIR 99, 235; PIR 02, 130; PIR 05, 241; PIR 15,
- Intermetamorfosis: creencia de que personas de su 157). Aparece en el síndrome amnésico, Korsakoff y demen-
ambiente han intercambiado sus identidades. cias, con estado de conciencia lúcida, y sirve para dar res-
- Síndrome de los dobles subjetivos: creencia de que puesta a cosas que no recuerda y así mantener una continui-
existen dobles exactos del propio paciente (doppelgänger) dad mnésica (PIR19, 140).
(PIR19, 125).

La mayoría de los investigadores han ligado los síndromes


de falsa identificación con algún tipo de disfunción cerebral PÁG COLUMNA PÁRRAFO
orgánica (hemisferio derecho). Se ha hablado de atrofia tem-
poral bilateral de desconexión interhemisférica (delirio de 35 2 2
dobles subjetivos)
TEXTO DEL MANUAL

• Alucinaciones somáticas, corporales, viscerales o cenesté-


PÁG COLUMNA PÁRRAFO sicas.
36 2 2
Son percepciones particulares del interior del propio cuerpo,
de todos los órganos o de las extremidades (p. ej., sentir que
tu corazón no late o tus intestinos han dejado de funcionar
TEXTO DEL MANUAL (PIR 00, 40; PIR 09, 48; PIR19, 141).

En psiquiatría se suelen observar en esquizofrenia general-


Imágenes alucinoides
mente acompañando a delirios de contenido bizarro o en
• Definición:
depresiones graves, habitualmente depresión con síntomas
Se producen en ausencia de estímulos desencadenantes
psicóticos de tipo hipocondríacos, en forma de sensación de
(PIR19, 133) y aparecen en el espacio negro de los ojos ce-
rrados (imágenes de fiebre) o en el espacio físico externo, a podredumbre o mal funcionamiento de órganos internos.
causa de intoxicaciones o uso de drogas (PIR 95, 23; PIR 03,
103; PIR 05, 104).
• Características:
Son subjetivas, autónomas (sin control por parte del

8
PÁG COLUMNA PÁRRAFO PÁG COLUMNA PÁRRAFO
36 2 1 57 2 6

TEXTO DEL MANUAL TEXTO DEL MANUAL


Alucinaciones fisiológicas: imágenes hipnagógicas e hipno- 8.5. Disfemia o tartamudez
pómpicas
Definición
• Definición:
Son imágenes (normalmente poco elaboradas: luces, soni- Trastorno prosódico (PIR) que produce un patrón de habla
dos bruscos, etc.) que se dan cuando el individuo está en alterado en el que predomina la falta de ritmo y fluidez, que
estado de semiconsciencia entre el sueño y la vigilia. se acompaña de particulares formas de acentuación y en-
tonación,de una duración excesiva de tensión silábica y de
• Subtipos: una distribución anómala de las pausas, produciendo como
- Hipnagógicas: consecuencia una producción del habla más lenta (PIR 01;
En fase de adormecimiento. (PIR19, 142) 47; PIR 01, 80).
- Hipnopómpicas:
En el paso del sueño a la vigilia (PIR 95, 21; PIR 96, 177; PIR Es más frecuente en niños que en adultos, existiendo dife-
17, 210). Suelen ser visuales o auditivas. rencias en función del sexo, siendo la proporción entre va-
rones y mujeres de 5/1. Con gran variabilidad interindividual
(PIR 93, 160).

PÁG COLUMNA PÁRRAFO Conceptos relacionados


• Taquifemia o farfulleo:
46 2 10 Rapidez excesiva de la palabra, omisión de sílabas y sonidos
y articulación imprecisa de fonemas.
TEXTO DEL MANUAL • Disartria (PIR 15,150):
Trastorno de la articulación debido a una lesión en el sistema
7.3. Trastornos del contenido del pensamiento: delirios nervioso periférico o central, que provoca anomalías en el
tono o movimiento de los músculos fonadores) (PIR19, 144).
Tabla 1. Escala de trastornos del pensamiento, el lenguaje y
la comunicación (continúa).
PÁG COLUMNA PÁRRAFO
NEOLOGISMO
Elaboración de nuevas palabras cuyo significado y derivación 48 1 3
es incapaz de comprender el otro (PIR19, 143).

TEXTO DEL MANUAL


PÁG COLUMNA PÁRRAFO
7.3. Trastornos del contenido del pensamiento:
57 1 4
Tabla 7. Suptipos de delirios primarios según Jaspers (1975).

TEXTO DEL MANUAL PERCEPCIÓN DELIRANTE (PIR19, 145).

8.4. Dislalias infatiles y retraso simple del lenguaje

Dislalias infantiles
PÁG COLUMNA PÁRRAFO
Definición 31 2 3
Dificultad para articular fonemas, sílabas o palabras, como
consecuencia de alteraciones orgánicas (p. ej., malformacio-
nes orgánicas como labio leporino, o por déficit auditivos) o TEXTO DEL MANUAL
funcionales (las más frecuentes) (PIR 97, 79; PIR 18, 77; PIR
18, 88). Distorsiones en la percepción del tamaño y forma de los es-
tímulos: metamorfopsias
Se diferencia de la disartria en que, en ésta, la dificultad de la
articulación y expresión del habla debida a trastornos Subtipos
del tono y del movimiento de los músculos que controlan la • Dismegalopsias (PIR 18, 87):
articulación es consecuencia de lesiones del sistema nervioso Distorsiones en la percepción visual del tamaño (PIR 16, 67).
central y/o periférico (PIR 15,150) (PIR19, 144). - Macropsias o megalopsias:

9
A escala aumentada. Actitud inmóvil junto a rigidez muscular que permite colocar
- Micropsias: al sujeto en una posición forzada sin que éste vuelva a su
A pequeña escala. posición original durante un tiempo indefinido (PIR 97, 84;
• Dismorfopsias: PIR19, 149).
Distorsiones en la percepción visual de la forma (PIR 97, 86; Con este trastorno también se relaciona la flexibilidad cérea
PIR 02, 148; PIR19, 146). que supone el mantenimiento de posturas forzadas y anti-
gravitatorias, sin que el sujeto intente recuperar la postura
original (PIR 99, 230), y que se denomina catalepsia rígida
cuando va acompañada de rigidez muscular.
PÁG COLUMNA PÁRRAFO
37 1 2

PÁG COLUMNA PÁRRAFO


TEXTO DEL MANUAL
43 2 7
Imágenes parásitas

• Definición: TEXTO DEL MANUAL


Se producen cuando el sujeto no fija la atención en ellas y
desaparecen cuando se concentra en ellas. Son consecuen- 6.8. Alteraciones mnésicas en trastornos mentales
tes a un estímulo que ya no está presente y son más frecuen-
tes cuando existe un cansancio extremo (PIR19, 147). Tabla 6. Anomalías en el recuerdo
• Se distinguen de las imágenes mnémicas por su autono-
mía, y de las imágenes consecutivas por su subjetividad. PSEUDOMEMORIAS Y FALSIFICACIÓN DE LA MEMORIA
• Variante:
Imágenes obsesivas. Son un caso especial porque se mantie- Hacen referencia a la elaboración de recuerdos para rellenar
nen aunque el individuo fije su atención en ellas. Es el caso, “vacíos” de memoria.
por ejemplo, de un paciente que refiere “llevo todo el día • Tres fenómenos:
con esa cancioncilla metida en la cabeza, no consigo desha- - Confabulación: el paciente inventa recuerdos sin intención
cerme de ella por más que lo intento” (PIR 07, 72). de mentir (PIR 99, 235; PIR 02, 130; PIR 05, 241; PIR 15,
157). Aparece en el síndrome amnésico, Korsakoff y demen-
cias, con estado de conciencia lúcida, y sirve para dar res-
puesta a cosas que no recuerda y así mantener una continui-
dad mnésica (PIR19, 140; PIR19, 150).
PÁG COLUMNA PÁRRAFO
46 2 4

TEXTO DEL MANUAL

7.3. Trastornos del contenido del pensamiento: delirios

Tabla 1. Escala de trastornos del pensamiento, el lenguaje y


la comunicación (continúa).

DISCURSO DIVERGENTE
También llamado “habla distraída”.
• Interrupción del discurso como respuesta a estímulos in-
mediatos y el paciente cambia de tema (PIR 02, 142; PIR 04,
259; PIR19,148).

PÁG COLUMNA PÁRRAFO


66 1 2

TEXTO DEL MANUAL

Síntomas que componen el síndrome catatónico (PIR 03,


115)
• Catalepsia:

10
PSICOPATOLOGÍA
AMPLIACIONES

AMPLIACIONES
PÁG COLUMNA PÁRRAFO PÁG COLUMNA PÁRRAFO

45 1 2 45 1 Incluir tabla 2

CAMBIO DE ALGUNA FRASE DEL TEXTO QUE AMPLIACIÓN


YA HAY PARA QUE TENGA SENTIDO.
Otros trastornos formales o del curso del pensamiento:

Eliminar: Disgregación: pérdida de la idea directriz como consecuencia


Clasificación (Fish) de la rotura de las asociaciones normales. Hay un desliza-
• Trastornos formales o del curso del pensamiento. miento de un tema a otro, sin necesidad de conexión. Puede
• Trastornos del contenido del pensamiento. aparecer deformación en la sintaxis con palabras y sílabas sin
conexión, o mantenerse una construcción gramatical correc-
AMPLIACIÓN ta aunque sin haber coherencia en el discurso, como resulta-
do el contenido se vuelve absurdo e incomprensible.
En su lugar:
El pensamiento no está influido por estímulos externos, se
Los trastornos del pensamiento se han dividido en trastornos construye de forma autónoma.
del curso del pensamiento o formales y trastornos del con-
tenido del pensamiento (Fish). Algunos autores añaden un Paralogismos: a una palabra existente se le da un significado
tercer grupo: los trastornos de la vivencia del pens miento. diferente al que tiene en realidad.
Glosomanía: palabras neoformadas que aparecen acumula-
das sin una sintaxis concreta.
Glosolalia: lenguaje personal del enfermo que utiliza un vo-
PÁG COLUMNA PÁRRAFO cabulario y una sintaxis de su propia invención, pero con un
sentido propio. También se conoce como “criptolalia”.
46 tabla 6

AMPLIACIÓN PÁG COLUMNA PÁRRAFO

También llamado “fuga de ideas”, “pensamiento ideofugiti- 46 tabla 10


vo” y “pérdida de asociaciones”.

CAMBIO DE ALGUNA FRASE DEL TEXTO QUE


YA HAY PARA QUE TENGA SENTIDO.
PÁG COLUMNA PÁRRAFO Eliminar:
46 tabla 5
Elaboración de nuevas palabras cuyo significado y deriva-
ción es incapaz de comprender el otro.

AMPLIACIÓN
AMPLIACIÓN
• Cuando se le realiza una pregunta al paciente, éste res-
ponde oblicuamente, es decir, con respuestas tangenciales o En su lugar:
incluso irrelevantes (PIR 97, 76).
Creación y so de palabras o expresiones sin un significado
Diferenciar de las pararrespuestas. En el pensamiento tan- generalmente aceptado. El paciente crea nuevas palabras
gencial las repuestas son involuntarias. a partir de la combinación o condensación de otras, siendo
estas incomprensibles para los otros.

11
PÁG COLUMNA PÁRRAFO PÁG COLUMNA PÁRRAFO

48 tabla 7 2 50 1 Tras tabla 8

AMPLIACIÓN AMPLIACIÓN

En intuición delirante añadir: Trastornos delirantes de Falsa Identidad. Munro incluye ade-
También se conoce como ocurrencia o inspiración delirante. más de los Síndromes de Capgras y y Fregoli (tabla 8):
- Intermetamorfosis: creencia de que personas de su
ambiente han intercambiado sus identidades.
- Síndrome de los dobles subjetivos: creencia de que
PÁG COLUMNA PÁRRAFO existen dobles exactos del propio paciente (doppelgänger).
La mayoría de los investigadores han ligado los síndromes
48 tabla 7 4 de falsa identificación con algún tipo de disfunción cerebral
orgánica (hemisferio derecho). Se ha hablado de atrofia tem-
poral bilateral de desconexión interhemisférica (delirio de
AMPLIACIÓN dobles subjetivos)

En recuerdo delirante añadir:


También representación delirante.

PÁG COLUMNA PÁRRAFO


48 tabla 8

AMPLIACIÓN

Añadir otra fila:

DELIRIO DE SIGNIFICACIÓN
El paciente encuentra significados especiales en los hechos
cotidianos, recibe información a través de ellos que sólo él
puede descifrar, todo sería “pruebas” que el paciente ha de
ir superando.

DELIRIOS SOMÁTICOS-HIPOCONDRÍACOS
Delirios relacionados con el funcionamiento del propio cuer-
po. Creencia de que el cuerpo o alguna parte de él no están
funcionando bien.

PÁG COLUMNA PÁRRAFO


49 tabla 8 8

AMPLIACIÓN

IDEA DELIRAINTE DE GRANDEZA O MEGALOMANÍACA

12
13
PSICOLOGÍA CLÍNICA
RESEÑAS

RESEÑAS Propensión que tiene la sustancia de crear adicción/depen-


dencia (PIR 17, 41; PIR 18, 110; PIR 19, 122).

PÁG COLUMNA PÁRRAFO


33 TOMO I 1 4 PÁG COLUMNA PÁRRAFO
49 TOMO I 1 13

TEXTO DEL MANUAL

1.4. Trastornos amnésicos


TEXTO DEL MANUAL
• Pueden aparecen confabulaciones (PIR 19, 140).
3.2. Enfoques desde la psicología

- Teoría de los procesos oponentes o teoría de la motivación


adquirida (Solomon y Corbit).
PÁG COLUMNA PÁRRAFO Tolerancia y abstinencia como procesos opuestos que bus-
can la homeostasis del organismo. Explicaría el fenómeno
48 TOMO I 2 3 de tolerancia condicionada descrito por Siegel (PIR 19, 164).

TEXTO DEL MANUAL PÁG COLUMNA PÁRRAFO


3.1 Introducción y conceptos básicos 59 TOMO I 2 2

Droga es toda sustancia que, introducida en un organismo


vivo, puede modificar una o más funciones del mismo (OMS,
1975). TEXTO AMPLIADO SI LA FUENTE ES OTRA

Clasificación de las sustancias 3.9. Trastornos relacionados con opiáceos


(Ver tabla 1)
El primer consumo produce náuseas, vómitos y disforia,
Glosario de conceptos de uso cotidiano en el área de las posteriormente el consumo produce placer, euforia, reduc-
adicciones ción de la ansiedad... lo que lleva a un consumo habitual
• Existen distintos tipos de patrones de consumo y diferen- para encontrarse bien y evitar así el síndrome de abstinen-
ciarlos es importante, de cara a establecer un posible diag- cia.
nóstico. Los sujetos que realizan un consumo regular de heroína y
Así podemos diferenciar: morfina desarrollan rápidamente tolerancia y dependencia
- Uso. (PIR 19, 121).
El consumo de una sustancia no acarrea consecuencias ne-
gativas en el individuo.
- Abuso.
El consumo de la sustancia se produce de forma continuada PÁG COLUMNA PÁRRAFO
a pesar de las consecuencias negativas.
Dependencia. 61 TOMO I 1 2
El uso excesivo de la sustancia produce consecuencias nega-
tivas significativas a lo largo de un amplio periodo de tiempo
(PIR 07, 170). TEXTO DEL MANUAL
• Vías de administración de las sustancias.
- Oral, ingesta mascada o sublingual. 3.11. Trastornos relacionados con estimulantes
- Pulmonar, inhalada o fumada (es la vía más rápida en
llegar al cerebro). El DSM 5 incluye una categoría genérica de Trastornos rela-
- Nasal, esnifada. cionados con los estimulantes, donde incluye los trastornos
- Intravenosa. relacionados con la cocaína, las anfetaminas y otros estimu-
- Intramuscular o subcutánea. lantes.
- Rectal. En el DSM-IV-TR, se recogían de forma independiente cate-
• Potencial adictivo de una droga (PIR 16, 88). gorías para:

14
• Trastornos relacionados con la cocaína:
PÁG COLUMNA PÁRRAFO
Dependencia, abuso, intoxicación abstinencia y trastorno
mental inducido. 79 TOMO I 1 Último párrafo,
• Trastornos relacionados con las anfetaminas o sustancias en la Tabla 19.
de acción similar:
Dependencia, abuso, intoxicación abstinencia y trastorno
mental inducido. TEXTO DEL MANUAL
Aunque los criterios diagnósticos para ambas categorías
eran prácticamente idénticos. 4.3 Trastorno esquizofreniforme
La cocaína es un estimulante del SNC. La forma de presenta-
ción de esta sustancia es variada: e hojas masticadas, polvos Criterios diagnósticos
o
TRASTORNO ESQUIZOFRENIFORME
La cocaína impide la recaptación neuronal de dopamina, (DSM-IV-TR Y DSM 5) (PIR 12, 85)
serotonina y noradrenalina, y facilita la liberación de estos
neurotransmisores en el espacio sináptico (PIR 94, 120). La A. Se cumplen los Criterios A, D y E para la esquizofrenia.
cocaína produce un elevado grado de dependencia psicoló- B. Un episodio del trastorno (incluidas las fases prodrómica,
gica y poca dependencia física (PIR 18, 111; PIR 19, 120). activa y residual) dura al menos 1 mes, pero menos de 6
meses (cuando el diagnóstico debe hacerse sin esperar a la
remisión, se calificará como “provisional”).

PÁG COLUMNA PÁRRAFO Especificar si:


• Sin características de buen pronóstico.
61 TOMO I 1 3 • Con características de buen pronóstico (PIR 19, 123): indi-
cadas por dos (o más) de los siguientes ítems:
- Inicio de síntomas psicóticos acusados dentro de las prime-
TEXTO DEL MANUAL ras 4 semanas del primer cambio importante en el compor-
tamiento
3.11. Trastornos relacionados con estimulantes o en la actividad habitual.
- Confusión o perplejidad a lo largo del episodio psicótico.
La cocaína es un estimulante del SNC. La forma de presenta- - Buena actividad social y laboral premórbida.
ción de esta sustancia es variada: e hojas masticadas, polvos - Ausencia de aplanamiento o embotamiento afectivos.
o *mezclada con heroína (speed ball), mezclando un alcaloi-
de de la cocaína con bicarbonato sódico (crack), separada de
su base de hidrocloruro por calentamiento (base libre).
PÁG COLUMNA PÁRRAFO

PÁG COLUMNA PÁRRAFO 92 TOMO I 1 3

68 TOMO I 2 Tabla 3
TEXTO DEL MANUAL
TEXTO DEL MANUAL 5.4. Trastornos depresivos DSM 5

4.1. Clasificaciones internacionales Trastorno de Disregulación Destructiva del Estado de Ánimo


(DSM 5) (PIR 19, 86)
CATATONIA ASOCIADA A OTRO TRASTORNO MENTAL Se trata de una categoría de nueva aparición en el DSM 5,
(ESPECIFICADOR CON CATATONIA) (DSM 5) (PIR 19, 149) que queda definida por los siguientes criterios diagnósticos.
(Ver tabla 10 en las páginas siguientes)

PÁG COLUMNA PÁRRAFO


PÁG COLUMNA PÁRRAFO
77 TOMO I Tabla 17. Altera- Pie de tabla 17
ciones neurológi- 97 TOMO I Tabla 14. Especi- Fila 3
cas en la esquizo- ficadores para los
frenia. Trastornos Depre-
sivos (DSM 5)
TEXTO DEL MANUAL
TEXTO DEL MANUAL
4.2. Esquizofrenia
Tabla 17. Alteraciones neurológicas en la esquizofrenia (PIR 5.4. Trastornos depresivos DSM 5
19, 152).

15
Especificadores para trastornos depresivos (DSM 5) bajos de serotonina están en la etiología de los trastornos
del estado de ánimo. La serotonina regula nuestras reaccio-
Con características melancólicas: nes emocionales, es decir, actúa de una manera inhibitoria
A. Una de las características siguientes está presente durante sobre nuestra conducta.
el período más grave del episodio actual: Cuando los niveles de serotonina están bajos el comporta-
1. Pérdida de placer por todas o casi todas las actividades miento es más impulsivo y los estados de ánimo fluctúan
(PIR 16, 117; PIR 17, 50; PIR 19, 130). ampliamente.
La hipótesis permisiva de la depresión (PIR 02, 44; PIR 15,
173; PIR 19, 131) postula que el déficit funcional de sero-
PÁG COLUMNA PÁRRAFO tonina favorece que otros neurotransmisores oscilen más
ampliamente favoreciendo la aparición de alteraciones del
108 TOMO I Tabla 28. Datos 26 estado de ánimo:
epidemiológicos
de los trastornos (fila RIESGO DE
afectivos. SUICIDIO)
Basado en BE- PÁG COLUMNA PÁRRAFO
LLOCH, A., SAN- 117 TOMO I 2 1
DÍN, B. y RAMOS,
F. (2008): Manual
de Psicopatolo- TEXTO DEL MANUAL
gía. Volumen II.
Página 238. 5.8 Modelos explicativos de los trastornos depresivos

Columna 3 Teoría de los estilos de respuesta (Nolen-Hoeksema (1991).


(TRASTORNOS Esta teoría se centra en los factores que intervienen en el
BIPOLARES). curso de la depresión. Un estilo de respuesta que tiende a la
rumiación mantendría y agravaría los síntomas del trastorno
depresivo (PIR 18, 129; PIR 19, 129), en contraposición a un
estilo más activo que tiende a la distracción o a la solución
TEXTO DEL MANUAL de problemas.

5.6. Epidemiología de los trastornos afectivos Según esta autora, el origen del estilo rumiativo estaría en
los primeros años de crianza del sujeto (aprendizaje por mo-
HISTORIA NATURAL delado y prácticas de socialización).
Mayor (PIR 05, 116; PIR 19, 136)

PÁG COLUMNA PÁRRAFO


PÁG COLUMNA PÁRRAFO
EN AMPLIACIO-
112 TOMO I 1 7 NES DSM-5
(TEMA TRASTOR-
NOS DE ANSIE-
DAD)
TEXTO DEL MANUAL

5.8 Modelos explicativos de los trastornos depresivos


TEXTO AMPLIADO
• Hipótesis noradrenérgica.
Es la teoría más aceptada. La depresión vendría asociada a Desarrollo y curso
un déficit funcional de la noradrenalina (NA). Esta afirmación
se basa en la psicofarmacología, los primeros antidepresivos Generalmente se desarrolla en la primera infancia, antes de
(IMAO) impiden la degradación de la NA. También se com- los 10 años, entre los 7 y los 11 (siendo la edad media de
probó que los tricíclicos (otro grupo de antidepresivos) impe- unos 10 años). Las situacionales presentan una edad de ini-
dían que las moléculas de noradrenalina fueran capturadas cio más tardía (PIR 19, 180). Las que se inician en la infancia
por la neurona que las libera al espacio presináptico. Ambos y la adolescencia suelen sufrir altibajos durante ese periodo.
tipos de fármacos provocan el aumento de la cantidad de Las que persisten en la edad adulta no suelen remitir.
neurotransmisores libres en el espacio sináptico lo que facili-
ta la transmisión de la señal eléctrica (PIR).
La privación del sueño, concretamente la fase REM, tiene
efectos antidepresivos y esto es debido a un aumento de la
sensibilidad de los receptores de la noradrenalina (PIR 96,
87).
• Hipótesis serotoninérgica.
La serotonina también juega un papel importante, niveles

16
Study del 2000 apuntó que las personas con TOC presenta-
PÁG COLUMNA PÁRRAFO
ban gran interferencia a nivel social y laboral. Los síntomas
EN AMPLIA- específicos pueden originar obstáculos específicos, pudiendo
CIONES DSM-5 generar problemas en las relaciones interpersonales, fracaso
(TEMA TRASTOR- escolar o pérdida de empleo. Es por ello por lo que podría
NOS DE ANSIE- decirse que, en comparación con trastornos de ansiedad y
DAD) depresión, los pacientes con TOC tienen mayor probabilidad
de ser solteros y estar en paro (PIR 19, 155).
TEXTO AMPLIADO SI LA FUENTE ES OTRA
Son poco frecuentes las remisiones completas y después de
una remisión puede existir recaídas producidas por ansiedad,
Factores de riesgo y pronóstico depresión (gran comorbilidad (PIR 15, 226)), fatiga u otros
factores. El deterioro puede fluctuar de moderado a muy
Temperamentales: Afectividad negativa (neuroticismo), inhi- grave y en algunos casos incapacita totalmente al sujeto.
bición conductual.
Ambientales: Sobreprotección o pérdida de los padres, sepa-
ración, maltrato físico y abusos sexuales. Exposiciones nega-
tivas o traumáticas al estímulo fóbico.
Genéticos y fisiológicos: Susceptibilidad genética a una de- PÁG COLUMNA PÁRRAFO
terminada categoría de fobia específica (PIR 19, 137). Las EN AMPLIA-
personas con fobia a la sangre-inyección-herida suelen pre- CIONES DSM-5
sentar una respuesta vasovagal. (TEMA TRAS-
TORNO OBSESI-
VO-COMPULSIVO
PÁG COLUMNA PÁRRAFO Y TRASTORNOS
RELACIONADOS)
147 TOMO I 2 3
TEXTO AMPLIADO
TEXTO DEL MANUAL Factores de riesgo y pronóstico

7.2. Trastorno obsesivo-compulsivo Temperamentales: Síntomas de interiorización, emotividad


negativa, inhibición comportamental en infancia (PIR 19,
Epidemiología 109).

Prevalencia vital Ambientales: Maltrato físico y sexual en infancia, sucesos


2,5%. traumáticos, agentes infecciosos.
En el DSM-IV-TR y hasta hace relativamente poco se consi- Genéticos y fisiológicos: Tasa en familiares de primer grado
deraba que el TOC afectaba por igual a hombres y mujeres, dos veces mayor, también aumento más precoz. Disfunción
sin embargo, los últimos estudios y el DSM-5 señalan que las de corteza orbitofrontal, cingulada anterior y cuerpo estria-
mujeres se ven afectadas con una tasa ligeramente más alta do.
que los varones en la edad adulta. Además, es importante
señalar que en los varones es frecuente que el trastorno se
inicie antes (PIR 19, 111), es por ello que el TOC, durante las
primeras etapas del desarrollo, es más frecuente en varones. PÁG COLUMNA PÁRRAFO
Seria en la adolescencia cuando la prevalencia se igualaría
entre hombres o mujeres, o incluso llegaría afectar ligera- 157 TOMO I 4 Especificador de
mente más a estas. la tabla 4.

TEXTO DEL MANUAL


PÁG COLUMNA PÁRRAFO
8.2. Trastorno de estrés postraumático
148 TOMO I 1 6.
2º párrafo del TRASTORNO DE ESTRÉS POSTRAUMÁTICO (MAYORES DE 6
apartado “Cur- AÑOS) (DSM 5)
so” modificado y
ampliado Especificar si:
• Con expresión retardada: si la totalidad de los criterios
diagnósticos no se cumplen hasta al menos seis meses des-
pués del acontecimiento (aunque el inicio y la expresión de
TEXTO AMPLIADO algunos síntomas puedan ser inmediatos) (PIR 19, 132).

La OMS sitúa el TOC dentro de una de las diez condiciones


más incapacitantes. El British National Psychiatric Morbidity

17
PÁG COLUMNA PÁRRAFO PÁG COLUMNA PÁRRAFO
159 TOMO I 2 2 42 TOMO II 2 8

TEXTO DEL MANUAL TEXTO DEL MANUAL

8.2. Trastorno de estrés postraumático 11.6. Trastorno de identidad disociativo (antes trastorno de
personalidad múltiple)
• Modelos basados en el procesamiento de la información.
Los modelos anteriores no son capaces de explicar la reex- Criterios diagnósticos (PIR19, 116)
perimentación continua del evento traumático, que unido a Los criterios diagnósticos del Trastorno de Identidad Diso-
la amplia generalización de los miedos son los síntomas que ciativo, como se aprecia en las tablas 12, 13 y 14 (ver en la
diferencian al TEPT de otros trastornos. página siguiente), presentan bastantes cambios en la actual
La teoría del procesamiento de la información propuesta por versión DSM 5 con respecto de la anterior DSM-IV-TR.
Foa y Kozak (1986) trata de explicar las estructuras
cognitivas del miedo para entender la reexperimentación de
la sintomatología del TEPT (PIR 07, 191; PIR 19, 135). Esta
teoría es una aplicación del modelo de Lang. PÁG COLUMNA PÁRRAFO
EN AMPLICIONES
DSM-5 (TEMA
TRASTORNOS
PÁG COLUMNA PÁRRAFO
DISOCIATIVOS)
35 TOMO II 2 4

TEXTO AMPLIADO
10.2 Trastorno facticio Características que apoyan el diagnóstico

Epidemiologia y curso Muchos individuos tienen dificultades para describir sus sín-
Existen pocos datos de prevalencia. En hospitales generales tomas. Pueden sentir que están “volviéndose locos” o temer
suponen el 1% de los pacientes visitados por profesionales un daño cerebral irremediable. Alteración subjetiva de la per-
de la salud mental. cepción del paso del tiempo.
El Trastorno facticio es más frecuente en mujeres que en Síntomas somáticos tenues como la saturación de la cabeza,
hombres. No obstante, la variante más crónica y grave (co- hormigueo, sensación de desmayo… Preocupaciones obse-
nocida como Síndrome de Münchausen) parece ser más fre- sivas y rumiaciones marcadas (PIR 19, 154). Hiporreactividad
cuente en hombres. fisiológica ante estímulos emocionales. Sustratos neuronales
Mayor predominio de los cuadros con síntomas físicos. en el eje hipotálamo-hipofiso-suprarrenal, lóbulo parietal in-
El curso del Trastorno facticio suele ser en forma de episo- ferior y circuito prefrontal-límbico.
dios intermitentes (PIR 19, 126) Los episodios únicos y los
que se caracterizan por ser persistentes y sin remisiones son
menos frecuentes.
PÁG COLUMNA PÁRRAFO
57 TOMO II 1 2
PÁG COLUMNA PÁRRAFO (Debajo de la Ta-
bla 11)
41 TOMO II 2 3

TEXTO AMPLIADO
TEXTO DEL MANUAL
12.5 Bulimia nerviosa
11.4. Amnesia disociativa
Epidemiología y curso
Algunas personas pueden recordar gradualmente los recuer- Es un trastorno que no se detecta con facilidad porque los
dos disociados años más tarde. Las capacidades disociativas pacientes intentan ocultarlo y porque además el peso puede
pueden disminuir con la edad, pero no siempre. A medida encontrarse dentro de unos valores normales (PIR 99, 124).
que la amnesia remite, puede haber una considerable angus- La incidencia es mayor en el sexo femenino (90% de los ca-
tia, conducta suicida (PIR19, 117) y síntomas de trastorno de sos). La prevalencia en mujeres está en torno a 1-3%.
estrés postraumático (TEPT). Aparece entre los 18 y 25 años y está más distribuido social-
mente que la anorexia nerviosa, siendo más prevalente que
la anorexia en mujeres jóvenes (PIR 19, 127).

18
PÁG COLUMNA PÁRRAFO TEXTO DEL MANUAL

91 TOMO II 1 Tabla 7 17. 4. Disomnias


Criterio C
NARCOLEPSIA (DSM 5)

B. Presencia de al menos una de las características siguientes:


TEXTO DEL MANUAL 1. Episodios de cataplejía, definida por (a) o (b), que se pro-
ducen como mínimo algunas veces al mes:
C. El episodio no se puede atribuir a los efectos fisiológicos a. En los individuos con enfermedad de larga duración, epi-
de una sustancia o de otra afección médica (PIR 19, 128). sodios breves (segundos o minutos) de pérdida brusca bilate-
ral del tono muscular con conservación de la consciencia que
se desencadenan con la risa o las bromas.
b. En los niños o en otros individuos en los seis meses poste-
PÁG COLUMNA PÁRRAFO riores al inicio, episodios espontáneos de muecas o de abrir
la boca y sacar la lengua, o hipotonía general sin un desen-
92 TOMO II Tabla 4. Disforia 2
cadenante emocional evidente.
de género (DSM
2. Deficiencia de hipocretina, según el valor de inmunorreac-
5).
tividad de hipocretina-1 en el líquido cefalorraquídeo (LCR)
(inferior o igual a un tercio del valor en individuos sanos ana-
TEXTO DEL MANUAL lizados con la misma prueba, o inferior o igual a 110 pg/ml).
La concentración baja de hipocretina- 1 en el LCR no se ha
16.2. Trastorno de la identidad sexual (DSM-IV-TR) - de observar en el contexto de lesión, inflamación o infección
Disforia de género (DSM 5) cerebral aguda.
3. Polisomnografía nocturna con latencia del sueño REM
1. Un poderoso deseo de ser del otro sexo o una insistencia (movimientos oculares rápidos) inferior o igual a 15 minutos
de que él o ella es del sexo opuesto (o de un sexo alternativo (PIR 18, 101), o una prueba de latencia múltiple del sueño
distinto del que se le asigna) (PIR 19, 178). con un valor medio inferior o igual a 8 minutos y dos o más
períodos REM al inicio del sueño (PIR 19, 113).

PÁG COLUMNA PÁRRAFO PÁG COLUMNA PÁRRAFO


98 TOMO II Tabla 2. Principa- 3 106 TOMO II 2 9
les cambios DSM
5.
TEXTO DEL MANUAL

TEXTO DEL MANUAL 17. 4. Disomnias

17.3. Clasificaciones internacionales Trastorno del ritmo circadiano (DSM-IV-TR) - Trastorno del
ritmo circadiano sueño-vigilia (DSM 5)
PRINCIPALES CAMBIOS DSM 5
Epidemiología
• Parasomnias: se mantiene el término parasomnia, pero
cambia la denominación de algunos trastornos y aparecen • Tipo de fases de sueño retrasadas (PIR 19, 114):
nuevos diagnósticos con entidad propia: La prevalencia entre la población general es de aproxima-
- El Trastorno del despertar del sueño no REM engloba Terro- damente un 0,17%, pero parece ser mayor del 7% en ado-
res nocturnos y Sonambulismo (PIR 19, 112). lescentes. Los síntomas suelen aparecer en la adolescencia y
- Aparece el diagnóstico de Trastorno del comportamiento principios de la edad adulta y la gravedad tiende a disminuir
del sueño REM (antes en Parasomnia no especificada). con la edad.
- Aparece el diagnóstico de Síndrome de Piernas Inquietas
(antes en Disomnia no especificada).

PÁG COLUMNA PÁRRAFO


103 TOMO II Tabla 13. Criterio B3
Narcolepsia (DSM
5).

19
PÁG COLUMNA PÁRRAFO
Epidemiología
109 TOMO II Tabla 21. Para- Encabezado de
somnias (DSM-IV- la Tabla 21, de La prevalencia es desconocida, aunque algunos estudios es-
TR y DSM 5). la parte derecha timan un 40% entre la población psiquiátrica diagnostica-
(DSM 5) da con otros trastornos, 30-60% en poblaciones clínicas de
trastornos de personalidad
TEXTO DEL MANUAL (el más frecuente) y en un 2% de la población general (PIR
19, 134).
17.5 Parasomnias

DSM 5 PÁG COLUMNA PÁRRAFO


PARASOMNIAS (PIR 19, 84)
135 TOMO II 2 2
• Trastorno por pesadillas.
• Trastorno del despertar en sueño no-REM (sonambulismo,
terrores
nocturnos). TEXTO DEL MANUAL
• Trastorno comportamental del sueño en fase REM.
• Síndrome de las piernas inquietas. 19.10. Trastorno narcisista de la personalidad

Prevalencia del 2-16% en la población clínica y menos del


1% en población general (PIR 19, 153).
PÁG COLUMNA PÁRRAFO
119 TOMO II 1 1
PÁG COLUMNA PÁRRAFO
138 TOMO II 1 3
TEXTO DEL MANUAL

18.4. Trastorno explosivo intermitente


TEXTO DEL MANUAL
Criterios diagnósticos (PIR 19, 85)
Los criterios diagnósticos del Trastorno explosivo intermiten- 19.13. Trastorno obsesivo-compulsivo de la personalidad
te se amplían en el DSM 5, como vemos en la tabla 4 y tabla
5 (ver en la página siguiente). Dos veces más frecuente en varones (PIR 19, 151).

PÁG COLUMNA PÁRRAFO


122 TOMO II 2 5

18.8. Juego patológico o ludopatía

Lo que determina que un juego sea patológico es la capa-


cidad de la persona para controlar voluntariamente su im-
plicación en el juego, pero generalmente la percepción de
la persona sobre su propia capacidad puede estar alterada,
sobre todo en los jugadores más patológicos (ilusión de con-
trol; PIR 19, 118).

PÁG COLUMNA PÁRRAFO


133 TOMO II 2 4

TEXTO DEL MANUAL

19.8 Trastorno límite de la personalidad

20
21
trasladados a una residencia y tres veces más declive funcio
PSICOLOGÍA CLÍNICA
AMPLIACIONES

AMPLIACIONES nal, tanto al alta como tres meses después.

DIAGNÓSTICO DIFERENCIAL
TRASTORNOS NEUROCOGNITIVOS Trastornos Psicóticos, Trastornos Bipolares y Depresivos con
características psicóticas, Trastorno de Estrés Agudo, Simula-
DELIRIUM ción y Trastorno Facticio, otros Trastornos Neurocognitivos.

CARACTERÍSTICAS QUE APOYAN EL DIAGNÓSTICO


Alteración de la conciencia y cambio cognitivo respecto a si- TRASTORNO NEUROCOGNITIVO MAYOR O LEVE DEBIDO A
tuación basal. Dificultades en mantener la atención, distrac- ENFERMEDAD DE ALZHEIMER
ción, perseveración, disminución espacial e incluso personal.
Aparece en horas o pocos días, es fluctuante, y empeora en CARACTERÍSTICAS QUE APOYAN EL DIAGNÓSTICO
tardes/noches. Aproximadamente el 80% de los individuos con Trastorno
Presentar un Trastorno Neurocognitivo hace a los individuos Neurocognitivo Mayor debido a Enfermedad de Alzheimer
más vulnerables al Delirium. tiene síntomas conductuales y psicológicos. Estos síntomas
A menudo comienza por alteraciones en el ciclo sueño-vigi- provocan tanto sufrimiento o más que las manifestaciones
lia. Puede haber alteraciones emocionales como ansiedad, cognitivas, y a menudo son el motivo de consulta. En la fase
temor, depresión, irritabilidad, enfado, euforia o apatía. La más leve, a menudo aparecen depresión y apatía. En la fase
alteración del estado de ánimo puede hacerse evidente me- más grave, son comunes los síntomas psicóticos, la irritabili-
diante gritos, chillidos, insultos, murmullos, gemidos u otros dad, la agitación, la agresividad y la deambulación sin objeti-
ruidos; especialmente por la noche o en ausencia de estimu- vo. En fases avanzadas, pueden observarse alteraciones de la
lación ambiental. marcha, disfagia, incontinencia, mioclonías y convulsiones.

PREVALENCIA PREVALENCIA
Mayor en ancianos hospitalizados. Prevalencia global en la Aumenta marcadamente con la edad: entre el 5-10% en la
comunidad baja (1-2%), pero aumenta con la edad, llegan- séptima década y al menos el 25% a partir de entonces.
do al 14% en individuos >85 años. Prevalencia 10-30% de El porcentaje de demencias atribuible a la Enfermedad de
ancianos que acuden a urgencias. En ingresados: prevalencia Alzheimer oscila entre el 60 y más del 90%, así como el
entre 14-24%, incidencia 6-56%; hasta 60% en individuos Trastorno Neurocognitivo Leve representa una proporción
en residencia y hasta 83% en individuos al final de la vida. importante de los casos de Deterioro Cognitivo Ligero.

DESARROLLO Y CURSO DESARROLLO Y CURSO


La mayoría tiene recuperación completa con o sin tratamien- Progresión gradual, en ocasiones con breves mesetas, hacia
to. La intervención temprana reduce la duración. Puede pro- la demencia grave y la muerte. La supervivencia media tras el
gresar a estupor, coma, convulsiones y muerte. La mortali- diagnóstico es de 10 años. Los individuos en fases avanzadas
dad entre los hospitalizados con Delirium es elevada (hasta quedan mutistas y encamados. Los síntomas habitualmente
el 40% en el primer año tras el dx). se inician en la octava y la novena décadas de la vida; las for-
mas de inicio temprano (quinta y sexta década) se relacionan
FACTORES DE RIESGO Y PRONÓSTICO con mutaciones causales conocidas.
Ambientales: Aumenta en contextos de deterioro funcional,
inmovilidad, antecedentes de caídas, niveles de actividad ba- FACTORES DE RIESGO Y PRONÓSTICO
jos, uso de drogas y medicamentos psicoactivos. Ambientales: Los traumatismos craneoencefálicos aumentan
Genéticos y fisiológicos: Tanto el Trastorno Neurocognitivo el riesgo.
Mayor como el Leve aumentan el riesgo de Delirium y com- Genéticos y fisiológicos: La edad es el mayor factor de riesgo.
plican su curso. Los ancianos con especialmente vulnerables. El polimorfismo de susceptibilidad genética relacionado con
La susceptibilidad en la primera infancia y la niñez es mayor la apolipoproteína E4, aumenta el riesgo y reduce la edad de
que en la edad adulta. En infancia, está relacionado con en- inicio. Los sujetos con síndrome de Down pueden desarro-
fermedades febriles y medicamentos como anticolinérgicos. llar Enfermedad de Alzheimer si sobreviven hasta la mediana
edad. Los factores de riesgo vascular aumentan la patología
MARCADORES DIAGNÓSTICOS cerebrovascular.
A menudo hay una ralentización generalizada del EEG y a
veces actividad rápida; sin embargo, la EEG no es suficiente ASPECTOS RELACIONADOS CON LA CULTURA
para el diagnóstico. La detección puede ser más difícil en los entornos cultura-
les y socioeconómicos en los que la pérdida de memoria se
REPERCUSIONES FUNCIONALES considera normal a edades avanzadas, los ancianos tienen
Asociado a mayor declive funcional y mayor riesgo de institu- menos exigencias cognitivas en la vida diaria y los niveles
cionalización. Tres veces más riesgo en hospitalizados de ser educativos muy bajos suponen un mayor reto para realizar

22
una evaluación cognitiva objetiva. de la motoneurona se asocia a un deterioro más rápido.

MARCADORES DIAGNÓSTICOS MARCADORES DIAGNÓSTICOS


Atrofia cortical, placas neuríticas predominantemente ami- Atrofia, en la variante frontotemporal, de los lóbulos fronta-
loides y ovillos neurofibrilares de proteína tau (examen his- les y temporales anteriores. En la variante semántica, los tem-
topatológico post mortem). En los casos de inicio temprano porales medio, inferior y anterior atrofiados bilateralmente,
con herencia autosómica dominante puede haber mutación de forma asimétrica, más afectado el hemisferio izquierdo.
en uno de los genes conocidos como causantes de la En-
fermedad de Alzheimer: la proteína precursora del amiloide REPERCUSIONES FUNCIONALES
(PPA), la presenilina 1 (PSEN1) o la presenilina 2 (PSEN2). La Al tener inicio relativamente temprano, a menudo afecta a la
apolipoproteína E4 no puede utilizarse como marcador diag- vida laboral y familiar. El lenguaje y el comportamiento están
nóstico ya que solo es factor de riesgo. alterados. Puede haber perturbación importante del ámbito
familiar, implicaciones legales y problemas en el trabajo por
REPERCUSIONES FUNCIONALES comportamientos socialmente inapropiados. Puede aparecer
Dificultades derivadas de la pérdida de memoria. La cogni- hiperoralidad, deambulación impulsiva y otros comporta-
ción social y la memoria procedimental pueden estar relati- mientos desinhibidos.
vamente preservadas durante periodos prolongados.
DIAGNÓSTICO DIFERENCIAL
DIAGNÓSTICO DIFERENCIAL Otros Trastornos Neurocognitivos, otras afecciones neuroló-
Otros Trastornos Neurocognitivos, otras enfermedades neu- gicas, otros trastornos mentales y afecciones médicas.
rológicas o sistémicas concurrentes activas y Trastorno De-
presivo Mayor.
TRASTORNO NEUROCOGNITIVO MAYOR O LEVE CON
COMORBILIDAD CUERPOS DE LEWY
La mayoría son ancianos y tienen múltiples afecciones mé-
dicas. A menudo concurre con enfermedad cerebrovascular, CARACTERÍSTICAS QUE APOYAN EL DIAGNÓSTICO
que contribuye al cuadro clínico. A menudo presentan caídas repetidas y síncopes, y episodios
transitorios de pérdida de consciencia inexplicados. Puede
haber disfunción autonómica como hipotensión ortostática
TRASTORNO NEUROCOGNITIVO MAYOR O LEVE FRONTO- e incontinencia urinaria. Las alucinaciones auditivas y otras
TEMPORAL alucinaciones visuales son comunes, al igual que los delirios
sistematizados, los falsos reconocimientos delirantes y la de-
CARACTERÍSTICAS QUE APOYAN EL DIAGNÓSTICO presión.
Síntomas extrapiramidales (solapamiento con parálisis su-
pranuclear progresiva o degeneración corticobasal). Puede PREVALENCIA
haber características de una enfermedad de la motoneurona Entre 0,1-5% en la población general anciana. Entre el 1,7-
(atrofia muscular, debilidad). Puede haber alucinaciones vi- 30,5% de todos los casos de demencia. 1,5 hombres por
suales. cada mujer.

PREVALENCIA DESARROLLO Y CURSO


Es una causa común de Trastorno Neurocognitivo de inicio Gradualmente progresivo, de inicio insidioso. A menudo hay
temprano en sujetos de menos de 65 años. La prevalencia pródromos de episodios confusionales de inicio agudo, pre-
poblacional está en el rango de 2-10 por 100.000. Un 20- cipitados con frecuencia por enfermedades o cirugías. Puede
25% de los casos aparece en mayores de 65 años. Supone el haber ocasionales mesetas, pero finalmente progresa hacia
5% de todos los casos de demencia. La variante del compor- la demencia grave y la muerte. La supervivencia media es
tamiento y semántica es mayor en varones; la variante con de 5-7 años. El inicio va típicamente desde la sexta hasta la
lenguaje no fluido es mayor en mujeres. novena década; la mayoría debutan en torno a los 70 años.

DESARROLLO Y CURSO MARCADORES DIAGNÓSTICOS


Comúnmente debutan en la sexta década de la vida, aunque Sinucleinopatía debida al pliegue anormal y el depósito de
la edad de inicio varía entre la tercera y la novena década. La la alfa-sinucleína. Puede asociarse al trastorno del compor-
enfermedad es gradualmente progresiva. La edad media de tamiento del sueño REM. La prueba de sensibilidad neuro-
supervivencia es de 6-11 años tras el inicio de los síntomas y léptica puede hacer sospechar. Baja captación de dopamina
de 3-4 años tras el diagnóstico. La supervivencia es menor y por el estriado, observada en SPECT o PET. Conservación de
el declive es más rápido en Frontotemporal que en Enferme- estructuras temporales mediales, captación baja generali-
dad de Alzheimer. zada con actividad occipital reducida, actividad prominente
de ondas lentas en el EEG con ondas transitorias del lóbulo
FACTORES DE RIESGO Y PRONÓSTICO temporal.
Genéticos y fisiológicos: Aproximadamente el 40% tiene an-
tecedentes familiares de Trastorno Neurocognitivo de inicio REPERCUSIONES FUNCIONALES
temprano; el 10% muestra un patrón de herencia autosómi- Suelen estar más afectados que otros individuos con otros
ca dominante. Se han identificado mutaciones en el gen que trastornos neurodegenerativos como la Enfermedad de Al-
codifica la proteína tau asociada a los microtúbulos (MAPT), zheimer. La disfunción motora y autonómica produce difi-
el gen de la granulina (GRN) y el gen C9ORF72. La patología cultades en el aseo, los traslados y la ingesta. Los trastornos

23
del sueño y los síntomas psiquiátricos prominentes también DIAGNÓSTICO DIFERENCIAL
aumentan la afectación. La calidad de vida es peor que indi- Otros Trastornos Neurocognitivos, otras afecciones médicas,
viduos con Enfermedad de Alzheimer. otros trastornos mentales.

DIAGNÓSTICO DIFERENCIAL COMORBILIDAD


Trastorno Neurocognitivo mayor o leve debido a Enferme- Comúnmente concurre con Trastorno Neurocognitivo mayor
dad de Parkinson. o leve debido a Enfermedad de Alzheimer y con depresión.

COMORBILIDAD
A menudo coexiste con la patología de la Enfermedad de TRASTORNO NEUROCOGNITIVO MAYOR O LEVE DEBIDO A
Alzheimer y de la enfermedad cerebrovascular, en especial UN TRAUMATISMO CEREBRAL
entre los grupos de más edad. En la Enfermedad de Alzhei-
mer hay patología de la sinucleína concomitante en el 60% CARACTERÍSTICAS QUE APOYAN EL DIAGNÓSTICO
de los casos. Puede estar acompañado por trastornos emocionales (irrita-
bilidad, facilidad para frustrarse, tensión y ansiedad, labilidad
afectiva), cambios de personalidad (desinhibición, apatía,
TRASTORNO NEUROCOGNITIVO MAYOR O LEVE VASCU- suspicacia, agresividad), alteraciones físicas (cefalea, fatiga,
LAR trastornos de sueño, vértigo o mareo, acúfenos o hipera-
cusia, fotosensibilidad, anosmia) y en especial en los trau-
CARACTERÍSTICAS QUE APOYAN EL DIAGNÓSTICO matismos craneoencefálicos más graves pueden presentarse
Antecedentes de ictus, episodios isquémicos transitorios y signos neurológicos (convulsiones, hemiparesia, alteraciones
signos de infartos cerebrales. Cambios de personalidad y áni- visuales, déficits de pares craneales) y lesiones ortopédicas.
mo, abulia, depresión, labilidad emocional. El desarrollo de
síntomas depresivos de inicio tardío acompañado de lentitud PREVALENCIA
psicomotora y la disfunción ejecutiva es una presentación Alrededor del 2% de la población tiene una discapacidad
común entre los ancianos con enfermedad isquémica pro- asociada a un traumatismo craneoencefálico. Las causas más
gresiva de pequeños vasos (cambios isquémicos subcortica- frecuentes son caídas, accidentes de tráfico y golpes en la
les) (“depresión vascular”). cabeza (por ejemplo, por deportes de contacto). El trauma-
tismo craneoencefálico leve repetido podría tener secuelas
PREVALENCIA persistentes acumulativas.
Es la segunda causa más frecuente de Trastorno Neurocog-
nitivo después de la Enfermedad de Alzheimer. Oscila entre DESARROLLO Y CURSO
el 0,2% en individuos de 65-70 años hasta el 16% en los La gravedad se clasifica en el momento de la lesión. El curso
de 80 años o más. En los tres meses siguientes a un ictus, el de la recuperación es variable y no solo depende de la lesión
20-30% es diagnosticado de demencia. Mayor prevalencia si no de la edad, los antecedentes y el abuso de sustancias,
en varones que en mujeres. que pueden dificultar la respiración. Salvo en traumatismos
craneoencefálicos graves, el curso típico es una mejoría
DESARROLLO Y CURSO completa o sustancial de los síntomas neurológicos, neuro-
Puede aparecer a cualquier edad, aunque aumenta de ma- cognitivos y psiquiátricos en días o semanas, observándose
nera exponencial después de los 65 años. El curso puede normalmente la recuperación completa a los tres meses. Los
variar desde inicio agudo con mejoría parcial a un declive traumatismos craneoencefálicos moderados y graves se han
escalonado o un declive progresivo con fluctuaciones y me- asociado a mayor riesgo de depresión y agresividad y trastor-
setas de diferentes duraciones. nos neurodegenerativos como la Enfermedad de Alzheimer.

FACTORES DE RIESGO Y PRONÓSTICO FACTORES DE RIESGO Y PRONÓSTICO


Ambientales: Las consecuencias de la lesión vascular cerebral Para el traumatismo craneoencefálico: Prevalencia mayor
están influenciadas por la neuroplasticidad (educación, ejer- en niños menores de 4 años, en adolescentes mayores y los
cicio físico y actividad mental). mayores de 65 años. Las caídas son la causa más común,
Genéticos y fisiológicos: Mismos factores de riesgo que para seguida de los antecedentes de tráfico.
la enfermedad cerebrovascular: hipertensión, diabetes, taba- Para los Trastornos Neurocognitivos tras un traumatismo cra-
quismo, obesidad, colesterol elevado, niveles de homocisteí- neoencefálico: Traumatismos repetidos, intoxicación concu-
na elevados, aterosclerosis y arteriosclerosis, fibrilación auri- rrente por una sustancia que aumente la gravedad del trau-
cular y otras situaciones que aumenten el riesgo de embolias matismo craneoencefálico en un accidente de tráfico.
cerebrales. Factores clave también son la angiopatía amiloide Modificadores del curso: Ser mayor de 40 años, baja puntua-
cerebral y el trastorno hereditario denominado arteriopatía ción inicial en la Escala de Coma de Glasgow, peor función
cerebral autosómica dominante con infartos subcorticales y motora, ausencia de reactividad pupilar y signos de lesión
leucoencefalopatía (CADASIL). cerebral.

MARCADORES DIAGNÓSTICOS MARCADORES DIAGNÓSTICOS


No hay biomarcadores establecidos. Pruebas neuropsicológicas, hemorragias y signos de contu-
sión.
REPERCUSIONES FUNCIONALES
Déficits físicos que producen discapacidad adicional. REPERCUSIONES FUNCIONALES
Menor eficiencia cognitiva, dificultades de concentración,

24
capacidades disminuidas, dificultades en el autocuidado, Consumo de Sustancias).
descoordinación, ataxia, lentitud, síntomas depresivos, pér- TRASTORNO NEUROCOGNITIVO MAYOR O LEVE DEBIDO A
dida de control emocional, apatía. INFECCIÓN POR VIH

DIAGNÓSTICO DIFERENCIAL CARACTERÍSTICAS QUE APOYAN EL DIAGNÓSTICO


El Trastorno de Estrés Postraumático puede concurrir y tener Episodios previos de inmunosupresión grave, cargas virales
solapamiento de síntomas. elevadas en líquido cefalorraquídeo e indicadores de enfer-
medad avanzada por el VIH como anemia e hipoalbumine-
COMORBILIDAD mia. En Trastorno Neurocognitivo avanzado puede haber
Trastorno por consumo de sustancias, Trastorno de Estrés síntomas neuromotores prominentes.
Postraumático (especialmente en poblaciones militares).
PREVALENCIA
Entre 1/3 y más del 50% de los infectados por el VIH pre-
TRASTORNO NEUROCOGNITIVO MAYOR O LEVE INDUCIDO sentan alteraciones cognitivas al menos leve; 25% criterios
POR SUSTANCIAS/MEDICAMENTOS Trastorno Neurocognitivo leve, menos del 5% Trastorno
Neurocognitivo mayor.
CARACTERÍSTICAS QUE APOYAN EL DIAGNÓSTICO
El inducido por sustancias depresoras puede manifestarse DESARROLLO Y CURSO
con síntomas añadidos de mayor irritabilidad, ansiedad, al- Puede resolverse, mejorar, empeorar lentamente o presentar
teraciones de sueño y disforia. El inducido por estimulan- un curso fluctuante. La rápida progresión hacia una disfun-
tes puede manifestarse mediante depresión, hipersomnia y ción profunda es poco habitual en el contexto del tratamien-
apatía de rebote. En las formas graves puede haber rasgos to antirretroviral combinado disponible en la actualidad. La
neuromotores prominentes como descoordinación, ataxia y progresión sigue un patrón “subcortical”. El curso global
lentitud motora, pérdida de control emocional y apatía. presenta una heterogeneidad considerable.

PREVALENCIA FACTORES DE RIESGO Y PRONÓSTICO
Desconocida. Más probable en individuos mayores, con con- El Trastorno Neurocognitivo debido a VIH no se ha reduci-
sumo crónico y otros factores de riesgo como déficits nutri- do con la terapia antirretroviral combinada, aunque sí las
cionales. Para el abuso de alcohol, un 30-40% en los dos pri- presentaciones más graves. Factores contribuyentes: control
meros meses de abstinencia; datos controvertidos en otras inadecuado en el Sistema Nervioso Central, aparición de ce-
sustancias como estimulantes, opiáceos y cannabis. pas resistentes a fármacos, inflamación crónica sistémica y
cerebral, envejecimiento, abuso de drogas, infecciones con-
DESARROLLO Y CURSO comitantes.
Tienden a comenzar durante la adolescencia y alcanzan el
pico en la veintena y treintena. Es común la recuperación MARCADORES DIAGNÓSTICOS
completa de las funciones neurocognitivas en los individuos Se requieren pruebas séricas de VIH para el diagnóstico. Útil
que logran una abstinencia estable antes de los 50 años; en líquido cefalorraquídeo. Pruebas de neuroimagen: dismi-
es probable que el Trastorno Neurocognitivo se mantenga nución del volumen cerebral total, adelgazamiento cortical,
persistente en quien no logra la abstinencia, por menor plas- reducción del volumen de SB e hiperintensidades de áreas
ticidad neuronal y el comienzo de otros cambios cerebrales parcheadas.
asociados a la edad.
REPERCUSIONES FUNCIONALES
FACTORES DE RIESGO Y PRONÓSTICO Variables. Afectación de funciones ejecutivas y lentificación
Mayor edad, tiempo de consumo prolongado y persistencia del procesamiento de la información.
de consumo pasados los 50 años. Además, para el Trastorno
Neurocognitivo inducido por alcohol: déficits nutricionales DIAGNÓSTICO DIFERENCIAL
de larga evolución, enfermedad hepática, factores de riesgo Otras enfermedades cerebrovasculares o neurodegenerati-
y enfermedad cardio y cerebrovascular. vas.

MARCADORES DIAGNÓSTICOS COMORBILIDAD


En abuso crónico de alcohol: adelgazamiento cortical, pérdi- Inflamación crónica sistémica y neurológica que puede aso-
da de sustancia blanca y aumento de los surcos y ventrículos. ciarse a enfermedad cerebrovascular y síndrome metabólico.
Trastorno por Consumo de sustancias inyectables y otras en-
REPERCUSIONES FUNCIONALES fermedades de transmisión sexual.
Disminución de eficiencia cognitiva, dificultad de concentra-
ción, síndromes motores.
TRASTORNO NEUROCOGNITIVO MAYOR O LEVE DEBIDO A
DIAGNÓSTICO DIFERENCIAL ENFERMEDAD POR PRIONES
Otros Trastornos Neurocognitivos fuera del contexto del uso,
intoxicación y abstinencia de sustancias. CARACTERÍSTICAS QUE APOYAN EL DIAGNÓSTICO
Incluye los Trastornos Neurocognitivos secundarios a un gru-
COMORBILIDAD po de encefalopatías espongiformes subagudas (como la
Con otros trastornos psiquiátrico (Trastorno de Estrés Pos- enfermedad de Creutzfeldt-Jakobs, la variante de la enfer-
traumático, Trastornos Psicóticos, Depresivos, Bipolares, por medad de Creutzfeldt-Jakob, el Kuru, el síndrome de Gerst-

25
mann-Stráussler-Scheinker y el insomnio letal) causadas por Ambientales: Exposición e herbicidas y pesticidas.
unos agentes transmisibles denominados priones. La historia Genéticos y fisiológicos: Edad más avanzada al comienzo del
típicamente muestra una rápida progresión hacia el Trastor- trastorno y mayor duración de la enfermedad.
no Neurocognitivo mayor a lo largo de un periodo de tan
solo 6 meses, por lo que el trastorno normalmente sólo se MARCADORES DIAGNÓSTICOS
observa en su forma mayor. Pruebas neuropsicológicas. Neuroimágenes estructurales
y escáneres del transportador de dopamina diferencian las
PREVALENCIA demencias relacionadas con cuerpos de Lewy (DemCLw y
Incidencia 1 o 2 casos por millón de habitantes. Prevalencia Enfermedad de Parkinson –EP-) de las no relacionadas con
desconocida, pero muy baja dada la corta supervivencia. cuerpos de Lewy (Enfermedad de Alzheimer –EA-).

DESARROLLO Y CURSO DIAGNÓSTICO DIFERENCIAL


Puede desarrollarse a cualquier edad (pico aproximadamen- Trastorno Neurocognitivo con cuerpos de Lewy, debido a En-
te a los 67 años). Los síntomas prodrómicos incluyen fati- fermedad de Alzheimer, vascular, debido a otra enfermedad
ga, ansiedad, problemas de apetito y sueño y dificultades médica, Parkinsonismo inducido por neuroléptico y otras
para concentrarse; tras varias semanas, les siguen descoor- afecciones médicas.
dinación, alteraciones de la visión, anomalías de la marcha,
movimientos que pueden ser mioclónicos, coreoatetoides o COMORBILIDAD
balísticos, junto con una demencia rápidamente progresiva. Enfermedad de Parkinson y Enfermedad de Alzheimer pue-
El trastorno típicamente progresa de forma muy rápida hasta den coexistir. También con enfermedades cerebrovascula-
alcanzar el grado máximo de discapacidad en unos meses; res, especialmente en ancianos. La combinación de múltiples
más raramente progresa a lo largo de dos años con un curso patologías puede reducir las capacidades funcionales de los
similar a otros Trastornos Neurocognitivos. individuos con Enfermedad de Parkinson. Los síntomas mo-
tores y la depresión o apatía pueden empeorar la discapaci-
FACTORES DE RIESGO Y PRONÓSTICO dad funcional.
Ambientales: Transmisión entre especies (encefalopatía es-
pongiforme bovina); inyección de factor de crecimiento hu-
mano mediante trasplante de córnea. TRASTORNO NEUROCOGNITIVO MAYOR O LEVE DEBIDO A
Genéticos y fisiológicos: Componente genético en hasta el ENFERMEDAD DE HUNTINGTON
15% de los casos (mutación autosómica dominante).
CARACTERÍSTICAS QUE APOYAN EL DIAGNÓSTICO
MARCADORES DIAGNÓSTICOS La depresión, la irritabilidad, la ansiedad, los síntomas obse-
Solo confirmada mediante biopsia o autopsia. Proteína 14- sivo-compulsivos y la apatía se asocian con frecuencia a la
3-3 y proteína tau. EEG con descargas agudas, trifásicas y enfermedad de Huntington, mientras que la psicosis lo hace
sincrónicas con una frecuencia de 0,5-2Hz. de forma más rara, y a menudo estos síntomas preceden al
inicio de los síntomas motores.
DIAGNÓSTICO DIFERENCIAL
Otros Trastornos Neurocognitivos mayores. PREVALENCIA
Los déficits neurocognitivos son el final de la Enfermedad de
Huntington. Prevalencia mundial de la Enfermedad de Hun-
TRASTORNO NEUROCOGNITIVO MAYOR O LEVE DEBIDO A tington: 2,7 por 100.000.
ENFERMEDAD DE PARKINSON
DESARROLLO Y CURSO
CARACTERÍSTICAS QUE APOYAN EL DIAGNÓSTICO La edad media del diagnóstico de la enfermedad es a los
Apatía, estado de ánimo deprimido, ansiedad, alucinaciones, 40 años. El curso es gradualmente progresivo, con una su-
delirios, cambios en la personalidad, trastorno del comporta- pervivencia media de aproximadamente 15 años a partir del
miento del sueño REM y somnolencia excesiva diurna. diagnóstico de los síntomas motores. Los síntomas iniciales
suelen ser irritabilidad, ansiedad y estado de ánimo deprimi-
PREVALENCIA do. Los síntomas motores tempranos pueden incluir la apari-
En EEUU: 0,5% entre los 65 y los 69 años; hasta un 3% a ción de inquietud en las extremidades y leve apraxia. Según
partir de los 85 años. Más común en hombres que en mu- progresa, aparece ataxia e inestabilidad postural, hasta apa-
jeres. Hasta el 75% de los individuos con Enfermedad de recer disartria, imposibilidad de deambulación, masticación
Parkinson desarrollará un Trastorno Neurocognitivo mayor y deglución.
en algún momento. La prevalencia del Trastorno Neurocog-
nitivo leve en enfermos de Parkinson es del 27%. FACTORES DE RIESGO Y PRONÓSTICO
Genéticos y fisiológicos: La base genética de la Enfermedad
DESARROLLO Y CURSO de Huntington es una expansión autosómica dominante de
Se inicia típicamente entre la sexta y la novena década de la penetrancia completa del trinucleótido CAG (“repetición
vida; la mayoría de los casos, a principios de los 60 años. El CAG”) en el gen de la hungtingtina. Una longitud de la re-
Trastorno Neurocognitivo leve a menudo se desarrolla tem- petición de 36 o más se asocia invariablemente con Enferme-
pranamente en el curso de la Enfermedad de Parkinson; el dad de Huntington, asociándose longitudes mayores a una
deterioro mayor aparece más tarde. edad de inicio más temprana.

FACTORES DE RIESGO Y PRONÓSTICO MARCADORES DIAGNÓSTICOS

26
Pruebas genéticas. Observación de la expansión autosómica ducta psicomotora y del comportamiento interpersonal. La
dominante de penetrancia completa del trinucleótido CAG intoxicación puede mantenerse, aunque la droga no sea de-
(“repetición CAG”) en el gen de la hungtingtina, en el cro- tectable en el cuerpo debido a los efectos permanentes en el
mosoma 4. La Enfermedad de Huntington se diagnostica por SNC. La intoxicación es el primer trastorno relacionado con
la aparición de síntomas. Hay pérdida de volumen de gan- sustancias que aparece, generalmente en la adolescencia.
glios basales, en especial caudado y putamen. Vía de administración: Las vías de administración más rápi-
das y eficientes tienden a desembocar en intoxicaciones más
REPERCUSIONES FUNCIONALES intensas y un patrón de consumo en escalada.
En la fase prodrómica, lo más común es el declive laboral, re- Duración de los efectos: Drogas de acción más rápida pro-
firiendo pérdida en la habilidad para realizar el trabajo habi- vocan intoxicaciones más inmediatas en comparación con
tual. Los aspectos emocionales, comportamentales y cogniti- drogas de acción lenta. Drogas de duración corta son más
vos están muy asociados con el declive funcional, en especial adictivas que drogas de duración larga. Sin embargo, drogas
las alteraciones en velocidad de procesamiento, iniciación y de duración larga provocan abstinencias de mayor duración.
atención en lugar de la memoria. A medida que aumenta la
enfermedad, empeora la funcionalidad por alteración de la
marcha, disartria, comportamientos irritables o impulsivos y TRASTORNOS MENTALES INDUCIDOS POR SUSTANCIAS Y
los movimientos coreicos graves que interfieren en el auto- MEDICAMENTOS
cuidado.
Características: En general drogas sedativas pueden causar
DIAGNÓSTICO DIFERENCIAL trastornos depresivos durante la intoxicación, mientras que
Otros trastornos mentales, otros Trastornos Neurocogniti- en la abstinencia se suelen observar alteraciones ansiosas.
vos, otros trastornos del movimiento (como la enfermedad Durante la intoxicación de sustancias estimulantes suelen
de Wilson, la discinesia tardía inducida por medicamentos, asociarse a trastornos psicóticos y de ansiedad, en la absti-
el corea de Sydenham, el lupus eritematoso sistémico o el nencia suelen aparecer trastornos depresivos. Tanto las dro-
corea senil. gas sedantes como estimulantes producen alteraciones en el
  sueño y alteraciones sexuales.
Desarrollo y curso: Los trastornos mentales inducidos suelen
desaparecer un mes después del cesar la abstinencia aguda,
la intoxicación grave o la administración del medicamento.
TRASTORNOS ADICTIVOS A excepción de: TNC inducido por alcohol, TNC inducido
por sedantes, hipnóticos y ansiolíticos y trastorno perceptivo
persistente por alucinógenos (flashbacks). No está claro si
TRASTORNOS RELACIONADOS CON SUSTANCIAS
personas con antecedentes familiares o personales de tras-
tornos mentales son más vulnerables a padecer un síndrome
La característica esencial del trastorno por consumo de sus-
inducido.
tancias es la asociación de síntomas cognitivos, comporta-
mentales y fisiológicos que indican que una persona con-
tinúa consumiendo la sustancia a pesar de los problemas
significativos relacionados con dicha sustancia. Se puede ALCOHOL
diagnosticar en 10 tipos de sustancias. No se incluye entre
ellas la cafeína. TRASTORNO RELACIONADO CON EL ALCOHOL
Los criterios de trastorno por consumo de sustancias se po-
drían agrupar en 4 áreas: CARACTERÍSTICAS CLÍNICAS QUE APOYAN EL DIAGNÓSTI-
1) Control deficitario: consumo en mayor cantidad o durante CO
mayor tiempo, deseos y esfuerzos fallidos por dejar de con- El consumo de alcohol afecta al tracto gastrointestinal, car-
sumir, inversión de tiempo en el consumo y la obtención de diovascular, SNC y periférico. Puede provocar problemas de
la droga y craving. conducta, de insomnio, depresión y ansiedad. Un efecto
2) Deterioro social: incumplimiento de deberes por el con- poco frecuente pero devastador es el trastorno amnésico
sumo, continuar con el consumo a pesar de los problemas persistente inducido por alcohol (Wernicke-Korsakoff) que
que conlleva. se recogería en el DSM 5 en TNC inducido por sustancias/
3) Riesgo de consumo: consumo de la sustancia incluso en medicamentos. El riesgo de suicidio es importante en casos
situaciones que comportan un riesgo físico y consumo de la de intoxicación grave o trastorno depresivo mayor inducido
sustancia a pesar de saber que se padece un problema físico por alcohol.
o psicológico que se produce o se exacerba con el consumo
4) Aspectos farmacológico: tolerancia y abstinencia PREVALENCIA
No es necesario que se cumplan los criterios de abstinencia Frecuente (3.6% población mundial). Más en hombres adul-
o tolerancia para establecer el diagnóstico de trastorno por tos que en mujeres. Tasa más elevada entre los 18 y 29 años,
consumo de sustancias. Sin embargo, si hay abstinencia se- la tasa disminuye con la madurez, la mínima tasa es en per-
guramente el curso clínico será más grave. sonas de +65 años.

TRASTORNOS INDUCIDOS POR SUSTANCIAS DESARROLLO Y CURSO


La mayoría que desarrolla un trastorno por consumo de
Intoxicación: Los cambios más frecuentes en la intoxicación alcohol lo hace a los 30 años. El alcoholismo temprano se
conllevan alteraciones de la percepción, de la vigilia, de la asocia a jóvenes con trastornos de conducta e intoxicacio-
atención, del pensamiento, del razonamiento, de la con- nes precoces. En jóvenes es frecuente la comorbilidad con

27
trastorno de conducta y trastorno de personalidad antisocial. Es frecuente que haya una intoxicación para otra sustancia,
Los cambios físicos derivados de la edad hacen que personas sobre todo si la persona presenta un trastorno de conducta
mayores que consumen sean más susceptibles a trastornos o un trastorno antisocial de la personalidad.
depresivos, intoxicaciones más graves, complicaciones médi-
cas y complicaciones derivadas de menores ingestas. El curso ABSTINENCIA A ALCOHOL
es variable, con remisiones y recaídas.
CARACTERÍSTICAS DIAGNÓSTICAS
FACTORES DE RIESGO Y DE PRONÓSTICO Se inicia la abstinencia tras un cese de alcohol en sangre de
4-12 horas. El pico de abstinencia es al segundo día. Empie-
Ambientales: Actitudes culturales hacia la bebida, disponi- zan a mejorar al 4rto-5to día. Síntomas de abstinencia como
bilidad y precio, estrés y experiencias previas. Una variable la ansiedad, el insomnio o la disfunción autonómica pueden
mediadora potencial en el desarrollo de problemas con el persistir hasta 3-6 meses. El 3% presentan convulsiones.
alcohol en personas predispuestas: consumo grave entre los
pares, expectativas positivas exageradas sobre efectos de al- FACTORES DE RIESGO Y PRONÓSTICO
cohol y malas habilidades de gestión estrés. La mayoría bebe diariamente y consume grandes cantidades
Genético y fisiológico: 40-60% de la varianza del riesgo se (+ 8 bebidas por día) durante varios años. Hay más riesgo si
explica por influencia genética. Las personas con anteceden- presentan enfermedades médicas comórbidas, componente
tes se multiplica el riesgo por 3-4 veces. La concordancia de genético (familiares que hayan padecido síndrome de absti-
gemelos monocigóticos es superior a la concordancia de nencia), consumo comórbido de sedantes o historia previa
gemelos dicigóticos (GMZ> GDZ). Además, tasa elevada de de abstinencia.
concordancia. Los fenotipos de bajo riesgo son aquellos que
sufren un enrojecimiento agudo de la piel con el consumo de MARCADORES DIAGNÓSTICOS
OH (típico de asiáticos). Hiperactividad autonómica y consumo de alcohol prolonga-
Modificadores del curso: La impulsividad se asocia con un do.
inicio precoz y una mayor gravedad.

ASPECTOS DIAGNÓSTICOS RELACIONADOS CON EL GÉNE- CAFEÍNA


RO
INTOXICACIÓN DE CAFEÍNA
Hombres consumen más y tienen más a presentar trastornos
por consumo de alcohol. Mujeres tienden a presentar mayo- PREVALENCIA
res niveles de alcohol en sangre que hombres (pesan menos En la población general no está clara.
en general y metabolizan menos).
Marcadores diagnósticos: Prueba más directa para medir el DESARROLLO Y CURSO
consumo es concentración de alcohol en sangre. También se La vida media de la cafeína es de aproximadamente 4-6h.
utiliza para determinar la tolerancia al alcohol. Los síntomas de intoxicación de la cafeína duran unos días,
Comorbilidad: Trastorno bipolar, esquizofrenia, trastorno no se conocen consecuencias a largo plazo. Altas dosis (p.
antisocial de la personalidad, trastorno depresivo mayor y ej. 5g-10g) pueden ser letales y requieren atención médica.
trastorno de ansiedad. También puede suprimir sistema in- A mayor edad, reacciones a la cafeína más intensas. Niños y
mune y predisponer a infecciones y cáncer. adolescentes, por bajo peso, tienen mayor riesgo de intoxi-
cación.
INTOXICACIÓN ALCOHOL
FACTORES DE RIESGO Y PRONÓSTICO
DESARROLLO Y CURSO Ambientales: Anticonceptivos orales (disminuyen elimina-
Puede durar desde minutos a horas. La edad promedio de ción de la cafeína)
la primera intoxicación es 15 años. Prevalencia más alta: 18- Comorbilidad: No se han descrito problemas médicos rela-
25 años. La frecuencia e intensidad disminuye con la edad. cionados. Si puede causar o exacerbar síntomas somáticos
Cuanto más pronto se produzca el inicio de intoxicaciones de ansiedad y trastornos gastrointestinales. Altas dosis pue-
habituales mayor será la probabilidad de desarrollar un tras- den provocar gran mal, fallos respiratorios y muerte. El con-
torno por consumo de alcohol. sumo excesivo se asocia con trastornos afectivos, trastornos
de conducta alimentaria, trastornos psicóticos, trastornos del
FACTORES DE RIESGO Y PRONÓSTICO sueño y trastornos por consumo de sustancias. No suele ser
Temperamentales (búsqueda de sensaciones e impulsividad) comórbido a trastornos de ansiedad.
y ambientales (tras ingestas intensas).

ASPECTOS DIAGNÓSTICOS RELACIONADOS CON EL GÉNE- ABSTINENCIA A LA CAFEÍNA


RO
Actualmente mayor tolerancia a la intoxicación en el género CARACTERÍSTICAS DIAGNÓSTICAS
femenino. El dolor de cabeza es el rasgo distintivo de la abstinencia a la
cafeína. Es un dolor de desarrollo difuso y gradual, palpitan-
MARCADORES DIAGNÓSTICOS te, intenso y sensible al movimiento. La cafeína es la droga
Comportamiento y fetor alcohólico. activa conductual más ampliamente utilizada. La probabili-
dad y gravedad de la abstinencia aumentan en función de
COMORBILIDAD la dosis diaria. Hay gran variabilidad interindividual e intra-

28
individual entre episodio y otro episodio. Se recomienda la de sustancias y otros trastornos mentales en la adultez. El
reducción gradual de la cafeína para disminuir la gravedad e inicio temprano se relaciona con problemas externalizantes
incidencia del síndrome de abstinencia. tipo trastornos de conducta y problemas internalizantes. Es
un factor de riesgo en general para la salud mental y en con-
CARACTERÍSTICAS ASOCIADAS QUE APOYAN EL DIAG- sumidores adolescentes en general favorece inestabilidad
NÓSTICO del estado de ánimo, del nivel de energía y cambios en los
Estudios de electroencefalograma (EEG) muestran que la patrones de alimentación. Las personas que desarrollan un
abstinencia aumenta ondas theta (lentas) y reducen ondas trastorno establecen un patrón de consumo que aumenta
beta 2 (rápidas). También se produce un deterioro en el ren- gradualmente de frecuencia y cantidad. Junto con el alco-
dimiento, menos motivación laboral, sociabilidad y mayor hol y el tabaco suelen ser de las primeras sustancias que se
uso de analgésicos. consumen, a veces es percibida como menos nociva que al-
cohol y tabaco. La intoxicación de cannabis no provoca una
DESARROLLO Y CURSO disfunción tan marcada como la que provocaría el consumo
Los síntomas aparecen generalmente después de 12-24 ho- intensivo de alcohol.
ras de la última dosis de cafeína. Pico máximo de la abstinen-
cia 1-2 días después. Los síntomas duran 2-9 días. El dolor de FACTORES DE RIESGO Y PRONÓSTICO
cabeza puede mantenerse hasta 21 días. La abstinencia cesa
30-60 min. después de la ingesta de café. La tasa de consu- Temperamentales: antecedentes de trastornos de conducta
mo y la cantidad consumida aumenta con la edad hasta que y trastorno antisocial de la personalidad, trastornos internali-
tienes 30 años, luego desciende. zantes y externalizantes. Altas puntuaciones en desinhibición
del comportamiento.
Factores de riesgo y pronóstico: Ambientales: fracaso escolar, consumo de tabaco, inestabi-
Temperamentales: Personas con otro TM (p.ej. TCAS), pre- lidad familiar, consumo de familiares, fácil acceso y mayor
sos, fumadores, personas que consumen OJ. disponibilidad de la sustancia.
Ambientales: Falta de disponibilidad (p.ej. en procedimientos Genéticos: Los factores hereditarios explican entre el 30-
médicos, embarazo, prácticas religiosas…) 80% de la varianza total de riesgo del trastorno. Se hipo-
F. fisiológicos y genéticos: No identificados genes específi- tetiza que hay una base genética común para problemas de
cos. consumo de sustancias y de la conducta en adolescentes.

COMORBILIDAD MARCADORES DIAGNÓSTICOS


Trastorno depresivo mayor, trastorno de ansiedad genera- Pruebas biológicas de metabolitos de cannabinoides. Estos
lizada, trastorno de pánico, trastorno antisocial de la per- se secretan lentamente y persisten durante largos periodos
sonalidad, trastorno por consumo de sustancias (alcohol o de tiempo en el cuerpo.
cocaína o cannabis).
CONSECUENCIAS FUNCIONALES DEL TRASTORNO POR
CONSUMO DE CANNABIS
Alteraciones en la función cognitiva, funciones ejecutivas
superiores, reducción de la actividad prosocial, síndrome
CANNABIS desmotivación, problemática en las relaciones sociales, par-
ticipación en conductas de riesgo y afecciones respiratorias.
TRASTORNO POR CONSUMO DE CANNABIS El uso crónico se relaciona con muchos trastornos mentales,
en particular el consumo de cannabis se hipotetiza que es un
CARACTERÍSTICAS DIAGNÓSTICAS factor causal de trastornos psicóticos, exacerba síntomas e
La novedad DSM5 es reconocer la existencia de la abstinen- interfiere en el tratamiento.
cia al cannabis. Los síntomas más comunes incluyen: irrita-
bilidad, ira o agresividad, ansiedad, depresión, inquietud, COMORBILIDAD
dificultad para dormir y disminución del apetito o pérdida de - Trastornos por consumo de sustancias: Un 53%
peso. Si se prescribe el cannabis con una finalidad terapéuti- presentaran un trastorno por consumo de nicotina y un 50%
ca, y éste genera de forma natural síndrome de abstinencia presentaran un trastorno por consumo de alcohol. Además
y tolerancia, no se debería de utilizar estos criterios como de los que buscan tratamiento ¾ consumen otras sustancias.
para establecer un diagnóstico de trastorno por consumo de - Trastornos mentales: Trastorno antisocial de la per-
sustancias. sonalidad (30%), Trastorno de ansiedad (24%), trastorno
obsesivo compulsivo (19%), trastorno paranoide (19%),
PREVALENCIA trastorno bipolar 1 (13%). Un 60% de los adolescentes con
Sustancia ilícita más ampliamente utilizada. Más en varones. trastorno de consumo de cannabis presentan trastornos ex-
La mayor tasa se sitúa en edades comprendidas entre los 18 ternalizantes (trastornos de conducta y TDAH), mientras que
y 29 años y las más bajas en +65 años. el 33% presentan trastornos internalizantes (ansiedad, de-
presión y trastorno de estrés postraumático).
DESARROLLO Y CURSO - Los efectos de salud más importantes del cannabis
Inicio más común en adolescencia o adultos jóvenes. Infre- son los que afectan al sistema respiratorio.
cuente en la preadolescencia. Además, progresión más rá-
pida en adolescentes. El inicio temprano del consumo de
cannabis (p. ej. antes de los 15 años) es un predictor robusto INTOXICACIÓN POR CANNABIS
del desarrollo del trastorno, otros trastornos por consumo

29
CARACTERÍSTICAS DIAGNÓSTICAS
La intoxicación se desarrolla en minutos si la vía es fumada INTOXICACIÓN FENICICLIDINA
y puede tardar 2 horas si se ingiere por vía oral. Los efectos
duran 3-4 horas, durando más si se usa vía oral. Los efectos CARACTERÍSTICAS DIAGNÓSTICAS
pueden reaparecer durante 12-24horas debido a la lenta li- Las manifestaciones clínicas más comunes de la intoxicación
beración. son desorientación, confusión sin alucinaciones, alucinacio-
nes o delirios, un síndrome de tipo catatónico y un coma de
PREVALENCIA gravedad variable.
Desconocida.
MARCADORES DIAGNÓSTICOS
Detectable en la orina durante un máximo de 8 días después
ABSTINENCIA CANNABIS de su consumo.

CARACTERÍSTICAS CLÍNICAS CONSECUENCIAS FUNCIONALES INTOXICACIÓN FENICICLI-


Un 50-95% de grandes consumidores de cannabis refiere DINA
abstinencia. La abstinencia no es de suficiente gravedad Gran toxicidad cardiovascular y neurológica (p. ej., convul-
como para requerir atención médica. siones, distonías, discinesias, catalepsia, hipotermia o hiper-
termia).

DESARROLLO Y CURSO
La mayoría de los síntomas se inician 24-72 horas después
del cese. El pico máximo es a la semana. La duración aproxi- INHALANTES
mada es de 1 a 2 semanas. Las dificultades en el sueño pue-
den durar un mes. Presentan una mayor duración y gravedad TRASTORNOS RELACIONADOS CON LOS INHALANTES
en adultos.
CARACTERÍSTICAS DIAGNÓSTICAS
Alrededor de un 10% de los consumidores de inhalantes ha
ALUCINÓGENOS referido experimentar tolerancia y abstinencia leve, aunque
algunas personas siguen consumiendo para evitar el síndro-
me de abstinencia. Sin embargo, debido a que los síntomas
TRASTORNOS RELACIONADOS CON LOS ALUCINÓGENOS de abstinencia son leves, el DSM-5 no reconoce un diag-
nóstico de abstinencia de inhalantes ni incorpora las quejas
CARACTERÍSTICAS DIAGNÓSTICAS por abstinencia como criterio diagnóstico del trastorno por
Se incluye dentro de esta categoría las fenciclidinas (o sus- consumo de inhalantes.
tancias similares a la fenciclidina p. ej., la PCP, “polvo de Los inhalantes mas consumidos son el pegamento, el betún
ángel”) y otros compuestos como la ketamina, la ciclohexa- o el tolueno, la gasolina para los encendedores, o las pintu-
mina y la dizocilpina. Estas sustancias se desarrollaron como ras en aerosol.
anestésicos disociativos y se convirtieron en drogas de uso Características asociadas al diagnóstico: la presencia de com-
recreativo. Los efectos alucinógenos en las personas vulnera- plicaciones médicas características del trastorno (p. ej., pa-
bles pueden durar semanas y pueden precipitar un episodio tología cerebral de la sustancia blanca, rabdomiólisis) y la
parecido a la esquizofrenia. Los efectos residuales de la dro- existencia de un “sarpullido del esnifador de pegamento” en
ga se asemejan también a la esquizofrenia. Se ha observado la zona perioral o perinasal.
que la ketamina tiene utilidad en el tratamiento del trastorno
depresivo mayor. No se han descrito con claridad los efectos PREVALENCIA
de abstinencia en humanos. Desconocida.

CARACTERÍSTICAS ASOCIADAS QUE APOYAN EL DIAG- FACTORES DE RIESGO Y DE PRONÓSTICO


NÓSTICO Temperamentales: Trastornos por uso de sustancias no inha-
Detectable en sangre hasta 8 días o más tiempo después del lantes comórbidos y un trastorno de conducta o un trastorno
consumo. de la personalidad antisocial. Otros predictores son el inicio
precoz de su consumo y la asistencia previa a servicios de
EPIDEMIOLOGÍA salud mental.
Se desconoce. Ambientales: Disponibilidad de inhalantes, aumentando el
riesgo del mal uso. El maltrato o el trauma infantil.
FACTORES DE RIESGO Y DE PRONÓSTICO Genéticos y fisiológicos: La desinhibición del comportamien-
Poca información. Entre las personas admitidas a tratamien- to. Debido a que la desinhibición del comportamiento está
to por abuso de sustancias, las que consumían fenciclidina sometida a una fuerte influencia genética, los jóvenes de
como sustancia primaria, en comparación con los admitidos familias con problemas de sustancias y de comportamiento
por el consumo de otras sustancias, eran más jóvenes y te- antisocial están en especial riesgo de trastorno por consumo
nían niveles educativos más bajos. de inhalantes.

ASPECTOS DIAGNÓSTICOS RELACIONADOS CON EL GÉNE- ASPECTOS DIAGNÓSTICOS RELACIONADOS CON LA CUL-
RO TURA
Tres cuartas partes son hombres. Ciertas comunidades aborígenes, pandillas de niños.

30
opiáceos durante el despliegue en Vietnam logro la absti-
ASPECTOS DIAGNÓSTICOS RELACIONADOS CON A1 GENE- nencia tras su regreso, pero experimentaron un aumento en
RO las tasas de trastorno por consumo de anfetaminas y de alco-
Aunque la prevalencia del trastorno por consumo de inha- hol, además de una mayor tendencia al suicidio. El aumento
lantes es casi idéntica en los varones y las mujeres adoles- de la edad se asocia con una disminución en la prevalencia,
centes, el trastorno es muy poco frecuente entre las mujeres como resultado de la mortalidad inicial y de la remisión de
adultas. los síntomas después de los 40 años.

COMORBILIDAD FACTORES DE RIESGO Y PRONÓSTICO


Trastornos de conducta en la adolescencia, trastorno antiso- Sobre todo a destacar factores géneticos y fisiológicos. Los
cial de personalidad en la adultez. El uso de inhalantes en los factores genéticos desempeñan un papel particularmen-
adultos y el trastorno por consumo de inhalantes también te importante tanto de forma directa como indirecta. Por
están estrechamente relacionados con los intentos de suici- ejemplo, la impulsividad y la búsqueda de novedades son
dio y la ideación suicida. los rasgos temperamentales individuales que se relacionan
con la tendencia a desarrollar un trastorno por consumo de
sustancias
INTOXICACIÓN POR INHALANTES
ASPECTOS DIAGNÓSTICOS RELACIONADOS CON LA CUL-
CARACTERÍSTICAS DIAGNÓSTICAS TURA
La intoxicación desaparece en cuestión de pocos minutos u Minorías étnicas que viven en zonas desfavorecidas han teni-
horas tras la exposición. do una incidencia muy elevada del trastorno. También per-
sonas blancas de clase media, especialmente en las mujeres
PREVALENCIA y personal médico.
Desconocida.
MARCADORES DIAGNÓSTICOS
ASPECTOS DIAGNÓSTICOS RELACIONADOS CON EL GÉNE- Los resultados de las pruebas de toxicología en orina rutina-
RO ria detectan la mayoría de los opiáceos (p.ej. heroína, morfi-
Se desconocen las diferencias de género en la prevalencia de na, codeína, oxicodona, propoxifeno) hasta 12-36 horas des-
intoxicación por inhalantes en la población general. pués de la administración. El fentanilo no se detecta en los
análisis de orina rutinarios, pero se puede identificar durante
CONSECUENCIAS FUNCIONALES DE LA INTOXICACIÓN POR varios días mediante procedimientos más especializados. La
INHALANTES metadona y la buprenorfina y el LAAM se tienen que ana-
Podría ocasionar inconsciencia, anoxia y muerte (“muerte lizar específicamente con otras pruebas, detectándose des-
súbita por inhalación” probablemente por arritmia o paro pués de varios días o hasta más de una semana después del
cardíaco) y consecuencias neurológicas y problemas médicos consumo. Es frecuente encontrar evidencia del consumo de
persistentes. otras sustancias en las pruebas de laboratorio (p. ej., cocaína,
marihuana, alcohol, anfetaminas, benzodiazepinas).
Los resultados de la prueba para la detección del virus de
las hepatitis A, B y C son positivos hasta en el 80- 90 % de
OPIÁCEOS los consumidores que se inyectan opiáceos. También es fre-
cuente el VIH. Además, se han observado cambios sutiles en
TRASTORNO POR CONSUMO DE OPIÁCEOS los patrones de secreción de cortisol y en la regulación de la
temperatura corporal hasta 6 meses después de la desintoxi-
PREVALENCIA cación de opiáceos.
0,37 % entre los adultos de más de 18 años. Esto puede
ser una subestimación debido a la gran cantidad de presos RIESGO DE SUICIDIO
con este trastorno. Las tasas son más altas en los hombres Al igual que el riesgo general observado en todos los trastor-
que en las mujeres. Las adolescentes pueden tener una ma- nos por consumo de sustancias, el trastorno por consumo de
yor probabilidad de desarrollar este trastorno. La prevalencia opiáceos se asocia con un mayor riesgo de intentos de suici-
disminuye con la edad, situándose el pico más alto entre los dio y de suicidios consumados. Sobre todo, por sobredosis.
adultos de 29 años o más jóvenes. El trastorno por consumo También es más frecuente que las intoxicaciones y abstinen-
de analgésicos es frecuente. cias induzcan depresiones más graves.

DESARROLLO Y CURSO CONSECUENCIAS FUNCIONALES DEL TRASTORNO POR


El trastorno por consumo de opiáceos puede comenzar a CONSUMO DE OPIÁCEOS
cualquier edad, pero los problemas asociados con el uso de El consumo de opiáceos se asocia con la falta de secreciones
opiáceos se observan con mayor frecuencia en la adolescen- de las mucosas, causando sequedad en la boca y la nariz.
cia tardía o en los primeros años de la veintena. El curso es Estreñimiento. Venas esclerosadas “venas en forma de hi-
abrupto con remisiones y recaídas. Las tasas de mortalidad a lera”. Cuando no pueden usar estas venas, con frecuencia
largo plazo pueden ser de hasta un 2 % por año, alrededor se inyectan directamente en el tejido subcutáneo (skin-po-
del 20-30 % de las personas con este trastorno logra la abs- pping), provocando celulitis y abscesos. También hepatitis,
tinencia a largo plazo. Una excepción es el personal militar VIH. Las personas que aspiran nasalmente (esnifan) heroína
que desarrollo dependencia a los opiáceos en Vietnam. Mas u otros opiáceos suelen desarrollar irritación de la mucosa
del 90 % de la población que había sido dependiente de los nasal, a veces acompañada de perforación del tabique nasal.

31
También son comunes las dificultades en el funcionamien- ANSIOLÍTICOS
to sexual. Los hombres a menudo experimentan disfunción
eréctil durante la intoxicación o con el uso crónico. Las mu- CARACTERÍSTICAS DIAGNÓSTICAS
jeres suelen tener alteraciones de la función reproductora Esta clase de sustancias incluye todos los medicamentos
y menstruaciones irregulares. La dependencia fisiológica a prescritos para dormir y casi todos los medicamentos an-
los opiáceos puede aparecer en alrededor de la mitad de ti-ansiedad (ansiolíticos). No se incluyen dentro de esta clase
los bebes nacidos de mujeres con trastorno por consumo de los agentes ansiolíticos no benzodiazepínicos (p. ej., buspiro-
opiáceos, lo que puede producir un síndrome de abstinencia na, gepirona), ya que no se asocian significativamente con el
grave que requiere tratamiento médico. Los bebes también abuso. se obtienen de forma legal e ilegal.
suelen presentar bajo peso al nacer. Los sedantes, hipnóticos o ansiolíticos pueden estar rece-
tados con propósitos médicos apropiados y, en función de
COMORBILIDAD la dosis, pueden producir tolerancia y abstinencia. Si estos
Las afecciones medicas más comunes asociadas con el tras- fármacos se prescriben o recomiendan para fines médicos
torno por consumo de opiáceos son infecciones víricas (p. apropiados y se usan según las indicaciones, la tolerancia o
ej., el VIH, el virus de la hepatitis C) y las bacterianas. la abstinencia resultantes no cumplen con los criterios para
Este trastorno se asocia a otros trastornos por consumo de el diagnóstico de un trastorno por consumo de sustancias.
sustancias, especialmente los relacionados con el tabaco, el
alcohol, el cannabis, los estimulantes y las benzodiacepinas. CARACTERÍSTICAS ASOCIADAS QUE APOYAN EL DIAG-
Las personas con trastorno por consumo de opiáceos están NÓSTICO
en riesgo de desarrollar distimia, depresiones leves o mode- El trastorno por consumo de sedantes, hipnóticos o ansiolí-
radas. El insomnio es frecuente, especialmente durante la ticos se asocia a menudo con otros trastornos por consumo.
abstinencia. Además, el trastorno de la personalidad antiso- La intoxicación intensa y repetida con sedantes, hipnóticos
cial es más frecuente en las personas con trastorno por con- o ansiolíticos puede estar asociada con una depresión grave
sumo de opiáceos que en la población general y el trastorno que, aunque temporal, puede conllevar tentativas de suicidio
de estrés postraumático también se ve con mayor frecuen- y suicidio consumado.
cia. Los antecedentes de trastorno de conducta en la infancia
o en la adolescencia se han identificado como un factor de PREVALENCIA
riesgo significativo para las adicciones, especialmente para el Las tasas del DSM-IV del trastorno son ligeramente mayores
trastorno por consumo de opiáceos. entre los hombres adultos (el 0,3 %) que, entre las mujeres
adultas, pero en el rango de 12 a 17 años de edad, la tasa de
las mujeres (el 0,4 %) supera a la de los hombres (el 0,2 %).
ABSTINENCIA POR OPIÁCEOS La prevalencia a los 12 meses del DSM-IV del trastorno por
consumo de sedantes, hipnóticos o ansiolíticos disminuye en
CARACTERÍSTICAS DIAGNÓSTICAS función de la edad y es mayor entre los 18 y los 29 años (el
La piloerección y la fiebre están asociadas con una abstinen- 0,5 %). Se produce un segundo pico de prevalencia entre las
cia más grave y no se ven a menudo en la práctica clínica personas mayores de 65 años.
habitual. El cumplimiento de criterios diagnósticos de abs-
tinencia de opiáceos no es suficiente para un diagnóstico DESARROLLO Y CURSO
de trastorno por consumo de opiáceos, pero los síntomas Inicio en la adolescencia- adultez temprana. El curso normal
concurrentes, como la conducta de búsqueda de drogas y suele pasar de un consumo ocasional a uno problemático.
las ansias de consumo, son sugestivos de comorbilidad con Otro curso clínico observado, aunque con menos frecuencia,
un trastorno por consumo de opiáceos. Los códigos de la es el que afecta a un individuo que originalmente obtuvo el
CIE-10-MC solo permiten un diagnóstico de abstinencia de medicamento por prescripción de un médico y que finalmen-
opiáceos si existe la presencia de una comorbilidad modera- te también hace un uso problemático.
da o grave con un trastorno por consumo de opiáceos. La El efecto adverso más común es déficit cognoscitivo, los pro-
gravedad y la velocidad de la abstinencia asociadas con los blemas de memoria y en la coordinación motora.
opiáceos depende de la vida media del opiáceo utilizado. En
el caso de la heroína los síntomas empiezan 4-12h después ASPECTOS DIAGNÓSTICOS RELACIONADOS CON EL GÉNE-
de la última dosis, los síntomas presentan su máximo nivel RO
a los 1-3 días y disminuyen gradualmente en un periodo de Las mujeres parecen tener un mayor riesgo que los hombres
tiempo de 5 a 7 días. Hay síntomas persistentes (ansiedad, de abusar de los medicamentos sedantes, hipnóticos o an-
disforia, anhedonia e insomnio) que duran semanas y meses. siolíticos recetados.
En cambio, en el caso del LAAM o metadona los síntomas
aparecen 2-4 días después de la última dosis. COMORBILIDAD
El trastorno de personalidad antisocial y el comportamiento
PREVALENCIA antisocial se asocian especialmente con el trastorno por con-
Al menos en un 60 % de los consumidores de 1 vez en los sumo de sedantes, hipnóticos o ansiolíticos. También asociar
12 meses anteriores. a trastorno bipolar, trastornos de ansiedad, depresión y otros
trastornos de consumo de sustancias.

SEDANTES, HIPNÓTICOS O ANSIOLÍTICOS ABSTINENCIA A SEDANTES, HIPNÓTICOS O ANSIOLÍTICOS

TRASTORNO POR CONSUMO DE SEDANTES, HIPNÓTICOS O CARACTERÍSTICAS ASOCIADAS QUE APOYAN EL DIAG-

32
NÓSTICO el peso o para mejorar el rendimiento. Entre las personas
La abstinencia de sustancias de acción más corta, que se admitidas a tratamiento por el uso estimulantes de tipo an-
absorben rápidamente y que no tienen metabolitos activos fetamínico como principal droga problema, el 66 % afirma-
(p. ej., triazolam), puede empezar horas después de cesar el ba que la consumía fumada, el 18 % inyectada y el 10 %
consumo de la sustancia; la abstinencia de sustancias con esnifada. El estimulante consumido por vía fumada o intra-
metabolitos de acción prolongada (p. ej., diazepam) puede venosa se asocia con una progresión rápida al trastorno por
no comenzar hasta después de 1-2 días o más. El síndrome consumo de estimulantes de alta gravedad/ de desarrollo en
de abstinencia producido por las sustancias de esta clase se cuestión de semanas o meses. El uso intranasal de cocaína
caracteriza por el desarrollo de un delirium potencialmen- y oral de estimulantes de tipo anfetamínico produce un pa-
te mortal. El curso evolutivo del síndrome de abstinencia se trón de progresión más gradual que ocurre durante meses o
predice en general por la vida media de la sustancia. Los me- años. Con el consumo continuo, hay una disminución de los
dicamentos cuya acción suele durar alrededor de 10 horas o efectos placenteros debida a la tolerancia y un aumento de
menos (p. ej., Lorazepam, oxazepam, temazepam) producen los efectos disfóricos.
síntomas de abstinencia a las 6 - 8 horas de descender los
niveles en sangre, con un pico de intensidad que se sitúa CONSECUENCIAS FUNCIONALES DEL TRASTORNO
en el segundo día y una mejoría notable al cuarto o quinto Se pueden desarrollar varias afecciones médicas dependien-
día. Para las sustancias con una vida media más larga (p. ej., do de la vía de administración. Los usuarios por vía intrana-
diazepam), los síntomas pueden no presentarse hasta más sal a menudo padecen sinusitis, irritación, sangrado de la
de una semana después de la interrupción, con un pico de mucosa nasal y perforación del tabique nasal. Las personas
intensidad en la segunda semana y una reducción notable que fuman las drogas tienen mayor riesgo de sufrir proble-
durante la tercera o cuarta semana. Además, pueden apa- mas respiratorios. Los que utilizan inyecciones intravenosas
recer síntomas adicionales a más largo plazo que persistan presentan riesgo de contracción de infecciones tipo VIH, he-
durante varios meses, pero con menor intensidad. patitis etc.
El dolor en el pecho es un síntoma común durante la intoxi-
cación por estimulantes. El infarto de miocardio, las palpita-
ESTIMULANTES ciones, las arritmias, la muerte súbita por paro respiratorio
o cardíaco y el accidente cerebrovascular se han asociado
TRASTORNOS RELACIONADOS CON LOS ESTIMULANTES con el consumo de estimulantes. El deterioro neurocognitivo
es común entre los usuarios de metanfetamina. Los proble-
CARACTERÍSTICAS DIAGNÓSTICAS mas de salud oral incluyen “boca de metanfetamina”, con
Estas sustancias generalmente se toman por vía oral o por afección de las encías, caries y llagas en la boca relacionadas
vía intravenosa, aunque la metanfetamina también se toma con los efectos tóxicos de fumar la droga y el bruxismo en el
por vía nasal. Los efectos de las anfetaminas y de las drogas estado de intoxicación.
similares a las anfetaminas son parecidos a los de la cocaína.
La cocaína se puede consumir en diferentes preparaciones COMORBILIDAD
(p. ej., hojas de coca, pasta de coca, clorhidrato de cocaína, Se asocian con otros trastornos por consumo de sustancias
y alcaloides de la cocaína, como base libre y crack) que di- especialmente, con sustancias con propiedades sedantes
fieren en cuanto a su potencia debido a los diversos niveles para paliar efectos adversos de los estimulantes. Los usua-
de pureza y velocidad de inicio. La cocaína, se esnifa a o se rios de cocaína suelen ingerir alcohol, mientras que los usua-
disuelve en agua y se inyecta por vía intravenosa. Las perso- rios de estimulantes de tipo anfetamínico a menudo toman
nas expuestas a los estimulantes de tipo anfetamínico o a la cannabis. El trastorno por consumo de estimulantes puede
cocaína pueden desarrollar un trastorno por consumo de es- estar asociado con el trastorno de estrés postraumático, el
timulantes en una semana. Pueden presentarse síntomas de trastorno de la personalidad antisocial, el TDAH y el juego
abstinencia, especialmente hipersomnia, aumento del apeti- patológico. A menudo hay problemas cardiopulmonares. La
to y disforia, los cuales intensifican el deseo de la sustancia o queja más frecuente es dolor en el pecho.
craving. Los estados de abstinencia se asocian con síntomas
depresivos temporales pero intensos que pueden parecerse
a un episodio depresivo mayor; los síntomas depresivos ge- INTOXICACIÓN A ESTIMULANTES
neralmente se resuelven en una semana.
CARACTERÍSTICAS ASOCIADAS QUE APOYAN EL DIAG-
PREVALENCIA NÓSTICO
La estimación de la prevalencia a los 12 meses de estimulan- Los efectos estimulantes más frecuentes son la euforia, el
tes de tipo anfetamínico es de 0.2%, siendo similar en hom- incremento de la frecuencia cardiaca y de la presión arterial,
bres y en mujeres en la edad adulta y ligeramente superior y el incremento de la actividad psicomotora. Los efectos de-
en mujeres en la adolescencia. El consumo de estimulantes presores tales como la tristeza, la bradicardia, la disminución
por vía intravenosa es ligeramente superior en hombres. de la presión arterial y la disminución de la actividad psico-
motora son menos frecuentes y generalmente solo aparecen
DESARROLLO Y CURSO con un uso crónico a dosis altas.
Los trastornos por consumo de estimulantes se producen en
todos los estratos de la sociedad y son más comunes en-
tre los individuos de 12 a 25 años en comparación con los ABSTINENCIA DE ESTIMULANTES
mayores de 26 años. El consumo regular de los individuos
en tratamiento se produce alrededor de los 23 años. Algu- CARACTERÍSTICAS DIAGNÓSTICAS
nas personas comienzan a usar estimulantes para controlar El síndrome de abstinencia se caracteriza por el desarrollo

33
de un ánimo disfórico acompañado de dos o más de los mentos de nicotina no parece causar daño médico.
siguientes cambios fisiológicos: fatiga, sueños vividos des- COMORBILIDAD
agradables, insomnio o hipersomnia, aumento del apetito y Enfermedades cardiovasculares, EPOC y cáncer. Fumar tam-
retraso psicomotor o agitación (Criterio B). A menudo existe bién aumenta los problemas perinatales, como bajo peso al
bradicardia y es una medida fiable de la abstinencia de es- nacer y el aborto involuntario. Las comorbilidades psiquiátri-
timulantes. A menudo se pueden desarrollar ansia de con- cas más comunes son los trastornos por consumo de alcohol
sumo (craving) y anhedonia, pero no forman parte de los y otras sustancias, el trastorno depresivo, bipolar, la ansiedad
criterios diagnósticos. y el trastorno de personalidad y TDAH. Fumar incrementa el
riesgo de padecer un TM, así mismo, se ha visto que el taba-
quismo es más frecuente en población psiquiátrica.

TABACO
ABSTINENCIA AL TABACO
TRASTORNOS RELACIONADOS CON EL CONSUMO DE TA-
BACO CARACTERÍSTICAS DIAGNÓSTICAS
Por lo general, la frecuencia cardíaca disminuye en 5-12 la-
PREVALENCIA tidos por minuto en los primeros días después de dejar de
Las tasas son similares entre los hombres (el 14 %) y las mu- fumar y el peso aumenta un promedio de 4 a 7 libras (2-3
jeres adultas (el 12 %), y disminuyen con la edad desde un kg) durante el primer año después de dejar de fumar. La
17 % entre los 18 y 29 años a un 4 % entre las personas ma- abstinencia de tabaco puede producir cambios de humor clí-
yores de 65 años. Aunque se ha observado que, en muchos nicamente significativos y deterioro funcional.
países en desarrollo, la prevalencia del consumo de tabaco
es mucho mayor en los hombres que en las mujeres, esto CARACTERÍSTICAS ASOCIADAS QUE APOYAN EL DIAG-
no pasaría sobre todo en naciones desarrolladas. A menudo NÓSTICO
existe un retraso en la transición demográfica, de tal manera Es frecuente el deseo intenso de alimentos dulces o azuca-
que el consumo de tabaco aumenta en las mujeres poste- rados y los problemas en el funcionamiento para realizar las
riormente. tareas que requieren vigilancia. Además, fumar aumenta el
metabolismo de muchos medicamentos utilizados para tra-
DESARROLLO Y CURSO tar los trastornos mentales, por lo que dejar de fumar puede
El inicio del consumo de tabaco después de los 21 años es aumentar los niveles en sangre de estos medicamentos, y
poco común. Más del 80% de las personas que fuman ta- esto puede producir resultados clínicamente significativos
baco intenta dejar de fumar en algún momento, pero el 60 (Este efecto parece no ser debido a la nicotina, sino más bien
% recae la primera semana y menos de un 5 % permanece a otros compuestos del tabaco).
en abstinencia de por vida. La mayoría de las personas que
consumen tabaco hace múltiples intentos, de manera que la PREVALENCIA
mitad de los consumidores de tabaco se abstiene finalmente. Los signos y síntomas más comunes son la ansiedad, la irrita-
Las personas que logran abandonar el consumo, por lo ge- bilidad y la dificultad para concentrarse. Los síntomas menos
neral no lo hacen hasta después de los 30 años. comunes son la depresión y el insomnio.

ASPECTOS RELACIONADOS CON LA CULTURA


La prevalencia del consumo de tabaco disminuyo en Estados
Unidos desde la década de los sesenta hasta la década de los TRASTORNO RELACIONADO CON OTRAS SUSTANCIAS (O
noventa, no notándose esta disminución en poblaciones his- SUSTANCIAS DESCONOCIDAS)
panas y afroamericanas. Además, es más frecuente fumar en
los países en desarrollo que en los países desarrollados. Los CARACTERÍSTICAS DIAGNÓSTICAS
hallazgos de estudios sugieren que algunas de las diferencias Se incluyen en este apartado los esteroides anabólicos, los
étnicas pueden tener una base biológica. fármacos antiinflamatorios no esteroideos, el cortisol, los
medicamentos antiparkinsonianos, los antihistamínicos, el
MARCADORES DIAGNÓSTICOS óxido nitroso, los nitritos de amilo, butilo o isobutilo, la nuez
El monóxido de carbono en el aire, la nicotina y su metaboli- de betel (produce euforia y sensación de flotar), el kava (que
to cotinina en la sangre, la saliva o la orina se pueden utilizar produce sedación, falta de coordinación, pérdida de peso,
para medir la intensidad del consumo. Sin embargo, están leve hepatitis y anomalías pulmonares) o las catinonas (que
débilmente relacionados con el trastorno por consumo de producen efectos estimulantes).
tabaco. Los trastornos relacionados con sustancias desconocidas es-
tán asociados con las sustancias no identificadas, tales como
CONSECUENCIAS FUNCIONALES DEL TRASTORNO POR las intoxicaciones en las que el individuo no puede identificar
CONSUMO DE SUSTANCIAS la droga ingerida, o los trastornos por consumo de sustan-
La 1/2 de los fumadores que no dejan de consumir tabaco cias que implican nuevas drogas del mercado negro aun no
morirá temprano por una afección relacionada con el taba- identificadas, o drogas conocidas que se venden ilegalmente
co, y la morbilidad relacionada con el tabaco se produce en bajo nombres falsos.
más de la 1/2 de los consumidores de tabaco. El principal
predictor de reversibilidad es la duración del tabaquismo. El PREVALENCIA
fumador pasivo tiene un riesgo de padecer afecciones del Los datos muy limitados indican que la prevalencia sería infe-
corazón y cáncer del 30 %. El uso a largo plazo de medica- rior a cualquiera de los trastornos por consumo de sustancias

34
identificadas. de un billete de rascar cada día podría no ser problemático,
ASPECTOS DIAGNÓSTICOS RELACIONADOS CON LA CUL- mientras que jugar menos frecuentemente en el casino, en
TURA: acontecimientos deportivos o a las cartas podría indicar un
Ciertas culturas pueden tener relación con este trastorno, trastorno. Del mismo modo, las cantidades de dinero que se
sobre todo relacionar con sustancias indígenas específicas. gastan en las apuestas no son en si mismas indicativas de
juego patológico. Algunos individuos pueden apostar miles
de euros al mes y no tienen problemas con el juego, mientras
que otros pueden apostar cantidades mucho más pequeñas,
JUEGO PATOLÓGICO pero pueden experimentar dificultades sustanciales relacio-
nadas con el juego.
CARACTERÍSTICAS ASOCIADAS QUE APOYAN EL DIAG-
NÓSTICO DIAGNÓSTICO DIFERENCIAL
El juego patológico debe distinguirse del juego social y pro-
Casi la 1/2 de las personas en tratamiento por juego patoló- fesional, es decir, jugadores no problemáticos. En el jue-
gico tiene ideación suicida, y alrededor del 17 % ha intenta- go profesional, los riesgos son limitados y la disciplina es
do suicidarse. fundamental. El juego social se produce normalmente con
amigos y tiene una duración de tiempo limitado y pérdidas
PREVALENCIA aceptables. Además, algunas personas pueden experimentar
En las mujeres, la tasa de prevalencia vital es de aproximada- problemas asociados con el juego (p. ej., el comportamiento
mente un 0,2 %, y en los hombres es del 0,6 %. En general de recuperación de las pérdidas y la falta de control a corto
el trastorno afecta más a varones que a mujeres si bien esta plazo) que no cumplen con todos los criterios para el juego
brecha de género podría estar estrechándose, así mismo, la patológico
prevalencia disminuye con la edad, siendo más frecuente en-
tre los jóvenes y en las personas de mediana edad que entre COMORBILIDAD
adultos mayores. Entre los adultos jóvenes y los adolescen- El juego patológico se asocia con una mala salud general
tes, las diferencias por género son más marcadas a favor del (sobre todo taquicardia y la angina de pecho son patologías
género masculino. Esto concuerda con el dato del DSM5 que más frecuentes entre las personas con juego patológico que
refiere que en mujeres el trastorno por juego se inicia de en la población general). También es frecuente que presen-
forma más tardía. ten otros TM como los trastornos por consumo de sustan-
cias, los trastornos depresivos, los trastornos de ansiedad y
DESARROLLO, CURSO Y GÉNERO los trastornos de la personalidad o que el trastorno por juego
El inicio del juego patológico puede aparecer durante la ado- preceda alguno de éstos.
lescencia o en los adultos jóvenes, pudiendo aparecer tam-
bién más tarde.
El curso es insidioso, si bien en mujeres la progresión suele
ser más rápida que en los hombres. La aparición del tras-
torno en las etapas intermedias o tardías de la vida es más
frecuente en las mujeres que entre los hombres.
-Tipos de juego por edad: Las personas más jóvenes prefie-
ren diferentes formas de juego (p. ej. apuestas deportivas),
mientras que los adultos mayores prefieren las máquinas tra-
gaperras y el bingo.
- Tipos de juego por género: Los varones tienden a apostar
en los juegos de cartas, los deportes y las carreras de caba-
llos, mientras que las mujeres prefieren las máquinas traga-
perras y el bingo. (Esta preferencia de juego concuerdan con
el hecho de que las mujeres debutan con el trastorno por
juego de forma más tardía, siendo adultas mayores).
En general la proporción de individuos que buscan trata-
miento para el juego patológico es baja en todos les grupos
de edad, siendo especialmente poco probable que individuos
más jóvenes acudan a tratamiento. En el caso de las mujeres
éstas parecen buscar tratamiento antes que los hombres, si
bien las tasas de búsqueda de tratamiento continúan man-
teniéndose bajas (<10%). Además, las mujeres con juego
patológico tienen más probabilidades que los varones de pa-
decer trastornos depresivos, bipolares y ansiosos.
Los individuos con juego patológico tienen mayor tendencia
a participar en algunos tipos de juegos de azar (p. ej., la
compra diaria de boletos de rascar) con más frecuencia que
en otros (p. ej., jugar semanalmente a las máquinas tragape-
rras o al blackjack en-el casino). La frecuencia de los juegos
de azar puede estar más relacionada con el tipo de juego
que con la gravedad del trastorno. Por ejemplo, la compra

35
ESPECTRO DE LA ESQUIZOFRENIA Y diagnosticarse Trastorno Delirante si la duración total de los
episodios afectivos es breve en comparación con la dura-
OTROS TRASTORNOS PSICÓTICOS ción total del cuadro delirante.

TRASTORNO DELIRANTE REPERCUSIONES FUNCIONALES


Aparente normalidad del comportamiento si no se habla
CARACTERÍSTICAS DIAGNÓSTICAS de las ideas delirantes. Deterioro funcional más circunscrito
Presencia de delirios durante más de 1 mes. Nunca criterio que en otros trastornos psicóticos. Pueden existir dificulta-
A de esquizofrenia. Salvo por el impacto del delirio, el fun- des en el rendimiento laboral y aislamiento social.
cionamiento no está afectado. Si existen episodios afectivos
concurrentes, su duración sobre el total del cuadro es breve COMORBILIDAD
(diagnóstico diferencial con esquizoafectivo). No se explica No se recoge en DSM 5.
por organicidad ni por otro trastorno mental (por ejemplo,
dismórfico corporal o TOC). Subtipos por contenido deliran-
te: erotomaníaco, celotípico, persecutorio, somático, mixto
y no especificado (ver por psicopatología). TRASTORNO PSICÓTICO BREVE
CARACTERÍSTICAS QUE APOYAN EL DIAGNÓSTICO
Puede haber introspección fáctica (reconocer que otros CARACTERÍSTICAS DIAGNÓSTICAS
consideran sus creencias delirantes como irracionales) pero 1 síntoma psicótico (delirios, alucinaciones, lenguaje des-
no introspección verdadera. Puede haber afecto irritable o organizado o comportamiento desorganizado/catatónico).
disfórico reactivo a la creencia delirante. Comportamientos Dura entre 1 día y 1 mes. Retorno completo al nivel pre-
litigantes o antagónicos. Pueden existir problemas legales mórbido. Inicio brusco: si claro cambio de no psicótico a
(especialmente en los subtipos celotípico y erotomaníaco), psicótico en un periodo de 2 semanas, sin pródromos. No
sociales, conyugales y laborales. se explica por organicidad ni por otro trastorno mental.
CARACTERÍSTICAS QUE APOYAN EL DIAGNÓSTICO
PREVALENCIA Suele acompañarse de agitación emocional o gran confu-
Se estima 0.2% a lo largo de la vida. H = M. Tipo celotípico sión. Puede acompañarse de cambios bruscos en el afecto.
más frecuente en varones que en mujeres. Puede haber aumento del riesgo de suicidio durante el
episodio agudo. Aunque le cuadro sea breve, puede ser
DESARROLLO Y CURSO necesaria la supervisión del paciente durante su curso para
Funcionamiento mejor que el de la esquizofrenia. Desarrollo garantizar que se cubren sus necesidades nutricionales e hi-
estable, tendencia a la cronicidad del cuadro. Aunque algu- giénicas y que está protegido de su temporal falta de juicio.
nos casos evolucionan a esquizofrenia. Relación familiar con
esquizofrenia y TP esquizotípico. Aunque puede aparecer PREVALENCIA
en adultos jóvenes, suele ser más prevalente en individuos Es el 9% de los casos de primer episodio psicótico en EE.
mayores. UU. 2M > 1 H

FACTORES DE RIESGO Y PRONÓSTICO DESARROLLO Y CURSO


No se recogen en DSM 5. Inicio en cualquier momento de la vida, sobre todo en ado-
lescencia o principio de edad adulta. Edad media de inicio:
ASPECTOS RELACIONADOS CON LA CULTURA 30 años. Por definición, dura menos de 1 mes, con retorno
El contenido delirante varía a través de los distintos contex- completo al nivel premórbido. Es habitual que la duración
tos culturales. Importancia del contexto cultura y religioso sea inferior (pocos días).
para poder valorar una creencia como delirante o no.
ASPECTOS RELACIONADOS CON EL GÉNERO FACTORES DE RIESGO Y PRONÓSTICO
No se recogen en DSM 5. Temperamento: rasgos preexistentes de personalidad pue-
den predisponer al desarrollo del Trastorno Psicótico Breve.
DIAGNÓSTICO DIFERENCIAL Por ejemplo, rasgos esquizotípicos, límites o rasgos del
- TOC y trastornos relacionados: si el sujeto está dominio psicótico (desregulación perceptiva) o del dominio
convencido de que sus creencias patológicas con ciertas, se de afectividad negativa (suspicacia).
debe hacer uso del especificador de “ausencia de intros-
pección/creencias delirantes” (no hacer el diagnóstico de ASPECTOS RELACIONADOS CON LA CULTURA
Trastorno delirante). Diferenciar el trastorno de patrones de respuesta cultural-
- Afecciones médicas: delirium, TNC, trastorno psi- mente aceptados. Importancia del contexto cultural y reli-
cótico debido a afección médica o inducido por sustancia/ gioso para poder valorar una creencia como delirante o no.
medicación. Base orgánica de estos cuadros.
- Esquizofrenia y esquizofreniforme: tienen otros ASPECTOS RELACIONADOS CON EL GÉNERO
síntomas además de las creencias delirantes, que no están No se recogen en DSM 5.
presentes en el Trastorno Delirante.
- Trastornos depresivos, bipolares y esquizoafectivo: DIAGNÓSTICO DIFERENCIAL
se distinguen del Trastorno Delirante por la relación tempo- - Afecciones médicas: síndrome de Cushing, tumor
ral entre episodios afectivos y los delirios, así como por la cerebral… diagnosticar trastorno psicótico debido a otra
gravedad de los síntomas del estado de ánimo. Sólo puede afección médica. Los cuadros de delirium también pueden

36
incluir sintomatología psicótica de breve duración.
- Trastornos relacionados con sustancias: diagnosti- ASPECTOS RELACIONADOS CON EL GÉNERO
car trastorno psicótico inducido por sustancia/medicamento No se recogen en DSM 5.
o valorar sui se cumplen criterios para delirium inducido por
sustancias/medicamentos o intoxicación de alguna sustan- DIAGNÓSTICO DIFERENCIAL
cia. Las pruebas de laboratorio para detectar tóxicos en - Otros trastornos mentales y afecciones médicas: no
sangre/orina pueden ayudar al diagnóstico diferencial. deben explicarse los síntomas por organicidad.
- Trastornos depresivos y bipolares: no se diagnostica - Esquizofrenia: duración y repercusión funcional.
psicótico breve si los síntomas se explican mejor por algo - Trastorno psicótico breve: duración.
afectivo con síntomas psicóticos.
- Otros trastornos psicóticos: duración, sintomatolo- REPERCUSIONES FUNCIONALES
gía. Atención en los casos recurrentes. Disfunciones en varias áreas de funcionamiento diario (estu-
- Simulación y facticios: en estos casos los síntomas dios, trabajo, relaciones sociales y familiares…). Consecuen-
se fingen (simulación) o de producen intencionalmente cias funcionales peores para los que evolucionan a esqui-
(facticio). zofrenia o esquizoafectivo. Los que se recuperan tienen
- Trastornos de personalidad: en algunos sujetos con mejores resultados funcionales.
TP, el estrés puede precipitar breves periodos de síntomas
psicóticos. Si duran más de 1 día, diagnóstico adicional de COMORBILIDAD
psicótico breve. No se recoge en DSM 5.
-
REPERCUSIONES FUNCIONALES
Pronóstico excelente en cuanto a funcionamiento social y
sintomatología para la mayoría de los individuos, a pesar de
las altas tasas de recaída. ESQUIZOFRENIA
COMORBILIDAD
No se recoge en DSM 5. CARACTERÍSTICAS DIAGNÓSTICAS
2/5 síntomas (delirios, alucinaciones, lenguaje desorganiza-
do, comportamiento desorganizado/catatónico y síntomas
residuales) durante al menos un mes. Duración total mayor
TRASTORNO ESQUIZOFRENIFORME de 6 meses. Deterioro del funcionamiento. No se explica
por organicidad. Presencia habitual de síntomas anímicos y
episodios de estado de ánimo completos de forma concu-
CARACTERÍSTICAS DIAGNÓSTICAS rrente a la fase activa (necesidad de diagnóstico diferencial
Síntomas idénticos a los de la esquizofrenia, pero de más con respecto a trastornos afectivos con síntomas psicóti-
breve duración (más de 1 mes, pero menos de 6 meses). cos).
Si los síntomas duran más de 6 meses, debe cambiarse el
diagnóstico a Esquizofrenia. Ausencia de criterio que exija CARACTERÍSTICAS QUE APOYAN EL DIAGNÓSTICO
deterioro funcional. Afecto inapropiado, ánimo disfórico, alteraciones del sue-
ño, alteraciones de la alimentación. Síntomas disociativos,
CARACTERÍSTICAS QUE APOYAN EL DIAGNÓSTICO preocupaciones somáticas, ansiedad, fobias…) La hostilidad
No se disponen de pruebas de laboratorio o psicométricas. y la agresión pueden asociarse a la esquizofrenia. Aunque
Se han hallado múltiples anomalías en pruebas de neuroi- la mayoría de las personas con diagnóstico de esquizofre-
magen, pero ninguna es diagnóstica. nia no son más agresivas que la población general y, en
cambio, sí son víctimas de agresiones con mayor frecuencia
PREVALENCIA que los sujetos de la población general. Déficits cognitivos
En países desarrollados, la incidencia es hasta 5 veces más (memoria, atención, lenguaje, procesamiento sensorial,
baja que esquizofrenia. En países en vías de desarrollo, cognición social…). Déficits laborales y vocacionales.
la incidencia puede ser igual a la de la esquizofrenia, en Ausencia de introspección (similar a anosognosia en cua-
especial para el especificador “con características de buen dros orgánicos). La ausencia de insight correlaciona con
pronóstico”. Incidencia similar a esquizofrenia y similar en peor adherencia al tratamiento, mayores tasas de recaída,
distintos contextos. mayor número de ingresos involuntarios, peor funciona-
miento psicosocial, agresiones y peor curso de enfermedad.
DESARROLLO Y CURSO No se disponen de pruebas de laboratorio, radiológicas o
Similar al de la esquizofrenia. Un tercio de los pacientes se psicométricas.
recuperan y su diagnóstico se queda como esquizofrenifor- Datos de neuroimagen: diferencias con controles sanos en
me. Dos tercios evolucionan a esquizofrenia o esquizoafec- arquitectura celular, conectividad, sustancia blanca, volu-
tivo men de sustancia gris, volumen cerebral total, índices de
FACTORES DE RIESGO Y PRONÓSTICO seguimiento ocular y electrofisiológicos.
Genéticos y fisiológicos: los familiares tienen un riesgo Signos neurológicos comunes: déficits en coordinación
aumentado de padecer esquizofrenia. motora, integración sensorial, secuenciación motora de
movimientos complejos, confusión izquierda-derecha.
ASPECTOS RELACIONADOS CON LA CULTURA
No se recogen en DSM 5. PREVALENCIA

37
Prevalencia a lo largo de la vida del 0.3-07%. Variaciones ausentes en la esquizofrenia.
según raza, etnia, país y origen geográfico en inmigrantes e - TEPT: presencia de suceso traumático que explica la
hijos de inmigrantes. H: más síntomas negativos y duración sintomatología (por ejemplo, las alucinaciones como sínto-
del trastorno. M: más síntomas afectivos y cuadros breves. ma de intrusión).
- TEA y trastornos de la comunicación: déficits en la
DESARROLLO Y CURSO interacción social con comportamientos repetitivos y restrin-
Inicio en adolescencia tardía-mitad de la treintena. El inicio gidos. Edad de inicio más temprana en estos cuadros.
antes de la adolescencia es raro. - Otros TM asociados con un episodio psicótico:
H: 20-25 años, peor pronóstico. M: 26-30 años. Inicio brus- delirium y trastornos neurocognitivos pueden presentar
co o insidioso. 20% pronóstico favorable (algunos incluso sintomatología psicótica, pero ésta está en relación con un
recuperación total). Síntomas positivos mejoran más con el deterioro más amplio de las funciones cognitivas, de base
tiempo que los síntomas negativos. orgánica.
En infancia, delirios y alucinaciones menos elaborados y las
alucinaciones visuales más frecuentes que en adultos. En REPERCUSIONES FUNCIONALES
niños también es frecuente la desorganización del lenguaje Disfunción social y laboral significativa. Empleos de catego-
y la conducta. ría inferior a sus progenitores, no se casan, tienen escasos
Inicio tardío (+40 años): más mujeres. contactos sociales.

FACTORES DE RIESGO Y PRONÓSTICO COMORBILIDAD


Ambientales: efecto estacional (nacidos a finales de invier- - Trastornos relacionados con sustancias: +50%
no/principios de primavera), más en niños que crecen en tabaco.
medio urbano y en algunos grupos étnicos minoritarios. - Trastornos de ansiedad: prevalencia de TOC y tras-
Genéticos y fisiológicos: predisposición genética (amplio torno de pánico superiores a población general.
abanico de alelos, aunque la mayoría de los pacientes - TP paranoide o esquizotípico premórbidos.
diagnosticados de esquizofrenia no presentan antecedentes - Enfermedades médicas asociadas que acortan la
familiares de psicosis), complicaciones del embarazo y parto esperanza de vida: síndrome metabólico, enfermedades
(hipoxia), mayor edad paterna, otras condiciones pre y peri- cardiovasculares y pulmonares, sobrepeso, diabetes, menos
natales adversas (estrés, infecciones, malnutrición, diabetes compromiso con el cuidado de la salud física, efectos se-
materna…). cundarios de medicación antipsicótica… Podría haber una
vulnerabilidad compartida entre la psicosis y los trastornos
ASPECTOS RELACIONADOS CON LA CULTURA médicos que explicaría parte de la comorbilidad médica de
Necesidad de valorar la adecuación a la cultura de referen- la esquizofrenia.
cia para valorar la sintomatología (delirios, alucinaciones,
lenguaje, sobre todo).
TRASTORNO ESQUIZOAFECTIVO
ASPECTOS RELACIONADOS CON EL GÉNERO
M: inicio más tardío, más carga afectiva, más síntomas po-
sitivos que empeoran en etapas posteriores de la vida, fun- CARACTERÍSTICAS DIAGNÓSTICAS
cionamiento social más preservado. H: inicio más temprano, Criterio A de esquizofrenia + episodio depresivo o maniaco
más síntomas negativos y desorganización, peor pronóstico. concurrentes. Al menos 2 semanas de síntomas psicóticos
en ausencia de alteración afectiva. Los síntomas afectivos
RIESGO DE SUICIDIO forman parte sustancial del cuadro. No se explica por orga-
5-6% fallece por suicidio. 20% intentan suicidarse alguna nicidad.
vez. Riesgo de suicidio especialmente alto entre los varones
con consumo de sustancias comórbidos. Otros factores CARACTERÍSTICAS QUE APOYAN EL DIAGNÓSTICO
de riesgo de suicidio: síntomas depresivos, desesperanza, Funcionamiento afectado (aunque no es criterio diagnósti-
situación de desempleo, periodo posterior a episodio agudo co). Disminución del contacto social y del autocuidado. Sín-
o tras alta hospitalaria. tomas negativos menos graves que en esquizofrenia. Mayor
riesgo de desarrollar trastornos depresivos y bipolares. No
DIAGNÓSTICO DIFERENCIAL pruebas ni medidas biológicas para el diagnóstico.
- Trastorno depresivo y bipolar con características
psicóticas: los síntomas psicóticos aparecen únicamente en PREVALENCIA
el contexto del episodio afectivo. 0.3% a lo largo de la vida. M > H. Más subtipo depresivo
- Trastorno esquizoafectivo: relación temporal entre entre las mujeres.
episodio afectivo y criterio A de esquizofrenia.
- Trastorno esquizofreniforme y psicótico breve: du- DESARROLLO Y CURSO
ración inferior a esquizofrenia y menor repercusión funcio- Inicio a cualquier edad. Inicio típico: edad adulta temprana.
nal. Diversos patrones temporales.
- Trastorno delirante: ausencia de otros síntomas Progresión mejor que en esquizofrenia, pero peor que en
psicóticos aparte de las ideas delirantes. los cuadros afectivos. Tipo bipolar más frecuente en adultos
- TP esquizotípico: síntomas bajo umbral asociados a jóvenes. Tipo depresivo más común entre adultos mayores.
rasgos de personalidad persistentes.
- TOC y dismórfico corporal: presencia de obse- FACTORES DE RIESGO Y PRONÓSTICO
siones, compulsiones y/o preocupaciones por el aspecto, Genéticos y fisiológicos: mayor riesgo entre familiares de

38
primer grado de pacientes con esquizofrenia, trastorno TRASTORNOS DEPRESIVOS
bipolar o trastorno esquizoafectivo.

TRASTORNO DE DESREGULACIÓN DISRUPTI-


ASPECTOS RELACIONADOS CON LA CULTURA VA DEL ESTADO DE ÁNIMO
Deben valorarse factores culturales y socioeconómicos, en
especial cuando el individuo y el clínico no compartan el CARACTERÍSTICAS DIAGNÓSTICAS
mismo entorno cultural y económico. Sobrediagnóstico Se añadió al DSM-5 para abordar las grandes dudas sobre
de esquizofrenia frente a esquizoafectivo en poblaciones la inapropiada clasificación y tratamiento de los niños que
afroamericanas e hispanoamericanas. presentan irritabilidad crónica persistente frente a los niños
que presentan un trastorno bipolar clásico (p. ej., episódi-
RIESGO DE SUICIDIO co).
Riesgo de suicidio del 5%. Factores de riesgo para el
suicidio: síntomas depresivos. Mayores tasas de suicidio en PREVALENCIA
poblaciones norteamericanas que en europeas, sudamerica- Es frecuente entre los niños que acuden a las consultas de
nas e indias (pasa lo mismo con la esquizofrenia). salud mental infantil.
2-5%. Se esperan tasas mayores en los hombres y los niños
ASPECTOS RELACIONADOS CON EL GÉNERO en edad escolar que en las mujeres y los adolescentes.
No se recogen en DSM 5.
DESARROLLO Y CURSO
DIAGNÓSTICO DIFERENCIAL Aproximadamente, la mitad de los niños con irritabilidad
- Otros TM y afecciones médicas. grave crónica tendrá una presentación que va a continuar
- Trastorno psicótico debido a otra afección médica: cumpliendo los criterios para un año después. Las tasas de
debido a organicidad. conversión de la irritabilidad grave no episódica en trastor-
- Esquizofrenia, trastornos depresivos y bipolares: no bipolar son muy bajas. Sin embargo, los niños con irrita-
relación temporal entre sintomatología psicótica y afectiva. bilidad crónica tienen mayor riesgo de desarrollar trastornos
depresivos unipolares y trastornos de ansiedad en la edad
REPERCUSIONES FUNCIONALES adulta.
Consecuencias funcionales muy variables. Funcionamiento
afectado, aunque no es criterio diagnóstico). FACTORES DE RIESGO Y PRONÓSTICO
COMORBILIDAD Temperamentales: muchos niños presentan síntomas que
- Trastornos relacionados con sustancias. también cumplen los criterios del TDAH y de un trastorno
- Trastornos de ansiedad. de ansiedad, siendo estos diagnósticos frecuentes a una
- Enfermedades médicas que acortan la esperanza edad temprana. En algunos niños también se cumplen los
de vida. criterios de trastorno depresivo mayor.
  Genéticos y fisiológicos: se ha sugerido que los niños que
presentan irritabilidad crónica no episódica pueden diferen-
  ciarse de los niños con trastorno bipolar basándose en el
riesgo familiar.

ASPECTOS DIAGNÓSTICOS RELACIONADOS CON EL GÉNE-


RO
Los niños que acuden a consulta son sobre todo varones.
En las muestras comunitarias parece existir un predominio
de varones.

CONSECUENCIAS FUNCIONALES
Provoca una grave perturbación en las vidas de los afecta-
dos y en sus familias. Tanto en el trastorno de desregula-
ción disruptiva del estado de ánimo como en el trastorno
bipolar son frecuentes las conductas peligrosas, la ideación
o los intentos de suicidio, la agresividad intensa y la hospi-
talización psiquiátrica.

COMORBILIDAD
Las tasas de comorbilidad son extremadamente altas.
En caso de que cumplan criterios de trastorno bipolar, sólo
se haría el diagnóstico de éste.
Si los niños presentan síntomas que cumplen los criterios
del trastorno negativista desafiante o del trastorno explo-
sivo intermitente y del trastorno de desregulación disrup-
tiva del estado de ánimo, sólo se deberían diagnosticar de
trastorno de desregulación disruptiva del estado de ánimo.
Además, no se debería hacer un diagnóstico de trastorno

39
de desregulación disruptiva del estado de ánimo si los sín- modo que el paciente deprimido va a ser incapaz de
tomas aparecen solo en un contexto que provoca ansiedad, atender a sus necesidades básicas de autocuidado, o sufre
cuando se alteran las rutinas de un niño con trastorno del mutismo, o está catatónico.
espectro autista o trastorno obsesivo-compulsivo, ni en el
contexto de un episodio depresivo mayor. COMORBILIDAD
Se asocia con frecuencia con los trastornos relacionados
con sustancias, el trastorno de pánico, el trastorno obsesi-
vo-compulsivo, la anorexia nerviosa, la bulimia nerviosa y el
TRASTORNO DEPRESIVO MAYOR trastorno límite de personalidad.

CARACTERÍSTICAS DIAGNÓSTICAS
Los síntomas de los criterios diagnósticos del trastorno TRASTORNO DEPRESIVO-PERSISTENTE (DISTI-
depresivo mayor deberían aparecer casi cada día para po- MIA)
derlos considerar, con la excepción del cambio de peso y la
ideación suicida. Los pacientes cuyos síntomas cumplan los criterios del
trastorno depresivo mayor durante dos años se deberían
CARACTERÍSTICAS QUE APOYAN EL DIAGNÓSTICO diagnosticar de trastorno depresivo persistente, además del
Los pacientes deprimidos que ingresan en residencias de trastorno depresivo mayor.
ancianos tienen una probabilidad notablemente incremen-
tada de fallecer en el primer año. PREVALENCIA
La hiperactividad del eje hipofisario-hipotalámico-adrenal 0,5 % para el trastorno depresivo persistente y del 1,5 %
parece relacionarse con la melancolía, los rasgos psicóticos para el trastorno depresivo mayor crónico.
y el riesgo de un posible suicidio.
DESARROLLO Y CURSO
PREVALENCIA Entre los individuos con trastorno depresivo persistente
7 %. En los sujetos de entre 18 y 29 años es tres veces ma- y trastorno límite de la personalidad, la covarianza de los
yor que en los pacientes de 60 años o mayores. Las mujeres correspondientes rasgos a lo largo del tiempo sugiere la
presentan tasas que llegan a ser 1,5-3 veces mayores. existencia de un mecanismo común. El comienzo temprano
(p. ej., antes de los 21 años) se asocia a una mayor proba-
DESARROLLO Y CURSO bilidad de trastornos comórbidos de la personalidad y de
La probabilidad de que se inicie se incrementa notablemen- abuso de sustancias.
te en la pubertad.
El riesgo es mayor en los pacientes cuyo episodio anterior COMORBILIDAD
fue grave, en los sujetos jóvenes y en las personas que han En comparación con los pacientes con trastorno depresivo
presentado múltiples episodios. La persistencia de síntomas mayor, los que tienen trastorno depresivo persistente tienen
depresivos durante la remisión, aunque sean leves, es un mayor riesgo de comorbilidad psiquiátrica en general y de
factor que predice claramente la recurrencia. trastorno de ansiedad y abuso de sustancias en particular.

FACTORES DE RIESGO Y PRONÓSTICOS


Los familiares de primer grado de los pacientes con tras-
torno depresivo mayor tienen un riesgo dos a cuatro veces TRASTORNO DISFÓRICO PREMENSTRUAL
mayor que el de la población general. La heredabilidad es
de aproximadamente el 40%, y los rasgos neuróticos de la
personalidad cuentan en proporción considerable para esta CARACTERÍSTICAS QUE APOYAN EL DIAGNÓSTICO
asociación genética. Se han descrito delirios y alucinaciones en la fase lútea tar-
día del ciclo menstrual, aunque son raros. Algunos conside-
ASPECTOS RELACIONADOS CON EL GÉNERO ran la fase premenstrual un período con riesgo de suicidio.
En las mujeres hay mayor riesgo de intentos de suicidio y el
riesgo de suicidio consumado es menor. DESARROLLO Y CURSO
Muchas pacientes refieren que los síntomas empeoran al
RIESGO DE SUICIDIO acercarse la menopausia.
El factor de riesgo que se ha descrito de manera más sis-
temática son los antecedentes de intentos o amenazas de FACTORES DE RIESGO Y PRONÓSTICO
suicidio, pero habría que recordar que la mayoría de los sui- Es posible que las mujeres que usan anticonceptivos orales
cidios consumados no se han precedido de intentos fallido. tengan menos síntomas premenstruales que las mujeres
Otras características asociadas a un aumento del riesgo de que no los usan.
suicidio consumado son: ser varón, estar soltero o vivir solo
y tener intensos sentimientos de desesperanza. La presencia ASPECTOS RELACIONADOS CON LA CULTURA
de trastorno límite de personalidad aumenta notablemente El trastorno disfórico premenstrual no es un síndrome aso-
el riesgo de futuros intentos de suicidio. ciado a una cultura.
Sin embargo, los factores culturales pueden influir signifi-
CONSECUENCIAS FUNCIONALES cativamente sobre la frecuencia, la intensidad y la expresi-
El deterioro puede llegar a la incapacidad completa, de

40
vidad de los síntomas, y sobre los patrones de búsqueda de un año después. Las tasas de conversión de la irritabilidad
ayuda. grave no episódica en trastorno bipolar son muy bajas. Sin
embargo, los niños con irritabilidad crónica tienen mayor
MARCADORES DIAGNÓSTICOS riesgo de desarrollar trastornos depresivos unipolares y tras-
El diagnóstico se confirma tras dos meses de evaluación tornos de ansiedad en la edad adulta.
prospectiva de los síntomas. Las variaciones relacionadas con la edad también diferecian
el trastorno bipolar clásico del trastorno de desregulación
COMORBILIDAD disruptiva del estado de ánimo. Las tasas del trastorno bi-
El trastorno previo que con más frecuencia se refiere en las polar son por lo general muy bajas antes de la adolescencia
pacientes con trastorno disfórico premenstrual es el episo- (&lt; 1 %), con un aumento estable a comienzos de la edad
dio depresivo mayor. Existe un amplio espectro de trastor- adulta (prevalencia del 1-2 %). El trastorno de desregulción
nos médicos que pueden empeorar en la fase premenstrual; disruptiva del estado de ánimo es más frecuente que el tras-
sin embargo, la ausencia de un período libre de síntomas torno bipolar antes de la adolescencia y los síntomas de este
durante el intervalo posmenstrual obvia el diagnóstico de trastorno, por lo general, son menos frecuentes conforme el
trastorno disfórico premenstrual. niño evoluciona hacia la edad adulta.

TRASTORNO DE DESREGULACIÓN DISRUPTIVA FACTORES DE RIESGO Y PRONÓSTICO


DEL ESTADO DE ÁNIMO Temperamentales: Los niños con irritabilidad crónica nor-
malmente presentan historias psiquiátricas complicadas.
CARACTERÍSTICAS DIAGNÓSTICAS En estos niños son frecuentes los antecedentes amplios de
Se debe distinguir cuidadosamente la presentación clínica irritabilidad crónica, que se manifiesta característicamente
del trastorno de desregulación disruptiva del estado de áni- antes de que se cumplan todos los criterios del trastorno.
mo de las presentaciones de otras afecciones relacionadas, Estas presentaciones prediagnósticas pueden haber reunido
en particular el trastorno bipolar pediátrico. De hecho, el las condiciones para el diagnóstico de trastorno negativista
trastorno de desregulación disruptiva del estado de ánimo desafiante. Muchos niños con trastorno de desregulación
se añadió al DSM-5 para abordar las grandes dudas sobre disruptiva del estado de ánimo presentan síntomas que tam-
la inapropiada clasificación y tratamiento de los niños que bién cumplen los criterios del trastorno por déficit de aten-
presentan irritabilidad crónica persistente frente a los niños ción/ hiperactividad (TDAH) y de un trastorno de ansiedad,
que presentan un trastorno bipolar clásico (p. ej., episódico). siendo estos diagnósticos frecuentes a una edad temprana.
En los niños, algunos investigadores consideran la irritabili- En algunos niños también se cumplen los criterios del tras-
dad grave no episódica como una característica del trastor- torno depresivo mayor.
no bipolar, aunque tanto el DSM-IV como el DSM-5 exigen, Genéticos y fisiológicos: En términos de agregación familiar y
para reunir las condiciones para el diagnóstico de trastorno genética, se ha sugerido que los niños que presentan irritabi-
bipolar I, que tanto los niños como los adultos presenten lidad crónica no episódica pueden diferenciarse de los niños
episodios delimitados de manía o hipomanía. Durante las úl- con trastorno bipolar basándose en el riesgo familiar.
timas décadas del siglo XX, esta discusión entre los investiga- Sin embargo, estos dos grupos no difieren en las tasas fami-
dores, de que la irritabilidad grave y no episódica es una ma- liares de trastornos de ansiedad, trastornos depresivos uni-
nifestación de manía pediátrica, coincidió con un aumento polares o abuso de sustancias. En comparación con los ni-
significativo de las tasas con que los clínicos diagnosticaban ños con trastorno bipolar infantil o con otras enfermedades
el trastorno bipolar a sus pacientes pediátricos. Este aumen- mentales, los que presentan un trastorno de desregulación
to abrupto en las tasas de diagnóstico parece atribuirse a disruptiva del estado de ánimo tienen similitudes y diferen-
que los clínicos combinaban al menos dos presentaciones cias en los déficits de procesamiento de la información. Por
clínicas en una sola categoría. Es decir, que se habían etique- ejemplo, en los niños con trastorno bipolar y en los niños
tado como trastorno bipolar en los niños tanto las presenta- irritables crónicos, así como en los niños con otros trastor-
ciones clásicas episódicas de manía como las presentaciones nos psiquiátricos, existen déficits en el reconocimiento de
no episódicas de irritabilidad grave. En el DSM-5, el término las emociones faciales, así como alteraciones en la toma de
trastorno bipolar se reserva explícitamente para las presenta- decisiones y en el control cognitivo. Existen también pruebas
ciones episódicas de los síntomas bipolares. de una disfunción específica del trastorno que aparece en
las tareas que evalúan el uso de la atención en respuesta a
PREVALENCIA estímulos emocionales y que muestra signos típicos de dis-
Es frecuente entre los niños que acuden a las consultas de función en los niños con irritabilidad crónica.
salud mental infantil. 2-5%. Se esperan tasas mayores en los
hombres y los niños en edad escolar que en las mujeres y los ASPECTOS DIAGNÓSTICOS RELACIONADOS CON EL GÉNERO
adolescentes. Los niños que acuden a consulta son sobre todo varones. En
las muestras comunitarias parece existir un predominio de
DESARROLLO Y CURSO varones.
Como es probable que los síntomas del trastorno de desr
gulación disruptiva del estado de ánimo cambien al madurar CONSECUENCIAS FUNCIONALES
el niño, el uso de este diagnóstico se debería restringir a los La irritabilidad grave crónica, como la que aparece en el tras-
grupos de edad similares a aquellos en que se ha establecido torno de desregulación disruptiva del estado de ánimo, se
su validez (7-18 años). Aproximadamente, la mitad de los asocia a una alteración importante de la familia del niño y
niños con irritabilidad grave crónica tendrá una presentación de las relaciones con los compañeros, así como también del
que va a continuar cumpliendo los criterios para el trastorno rendimiento académico.

41
Como consecuencia de su extremadamente baja tolerancia a depresivos acompañantes. Al principio, el paciente quizá
la frustración, estos niños tienen por lo general dificultades niegue que está triste, pero la tristeza se podría reconocer
para progresar adecuadamente en el colegio, son a menudo a través de la entrevista o deducir de la expresión facial o la
incapaces de participar en las actividades que normalmente conducta.
disfrutan los niños, su vida familiar se altera gravemente En los pacientes que se centran en una queja somática, los
a causa de sus crisis y su irritabilidad, y tienen problemas clínicos deberían determinar si el malestar de esa queja se
para hacer o conservar amigos. Por lo general, los niveles asocia a síntomas depresivos específicos. Aparecen fatiga
de disfunción de los niños con trastorno bipolar y de los ni- y alteración del sueño en una alta proporción de casos, y
ños con trastorno de desregulación disruptiva del estado de las alteraciones psicomotoras son mucho menos frecuentes
ánimo son parecidos. Ambos trastornos provocan una grave pero son indicativas de una mayor gravedad global, al igual
perturbación en las vidas de los afectados y en sus familias. que la presencia de una culpa delirante o casi delirante.
Tanto en el trastorno de desregulación disruptiva del estado En algunos pacientes con episodios leves, el funcionamiento
de ánimo como en el trastorno bipolar son frecuentes las puede parecer normal, pero requiere un esfuerzo notable-
conductas peligrosas, la ideación o los intentos de suicidio, la mente elevado.
agresividad intensa y la hospitalización psiquiátrica. Casi siempre se produce una pérdida del interés o del placer,
al menos en algún grado. Los pacientes pueden referir que
COMORBILIDAD se sienten menos interesados por los hobbies, “que no les
Las tasas de comorbilidad son extremadamente altas. importan”, o que no sienten placer en las actividades que
En caso de que cumplan criterios de trastorno bipolar, sólo se antes consideraban placenteras (Criterio A2). Los familiares
haría el diagnóstico de éste. Si los niños presentan síntomas notan a menudo un mayor aislamiento social o el abandono
que cumplen los criterios del trastorno negativista desafiante de los pasatiempos con los que disfrutaban (p. ej., el que
o del trastorno explosivo intermitente y del trastorno de des- había sido un ávido jugador de golf deja de jugar, o un niño
regulación disruptiva del estado de ánimo, sólo se deberían que solía disfrutar del fútbol se excusa para no practicarlo).
diagnosticar de trastorno de desregulación disruptiva del es- En algunos pacientes, en comparación con los niveles pre-
tado de ánimo. Además, no se debería hacer un diagnóstico vios, existe una reducción significativa del interés o del deseo
de trastorno de desregulación disruptiva del estado de áni- sexual.
mo si los síntomas aparecen solo en un contexto que pro- El sentimiento de inutilidad o de culpa que se presenta en el
voca ansiedad, cuando se alteran las rutinas de un niño con episodio depresivo mayor puede incluir evaluaciones negati-
trastorno del espectro autista o trastorno obsesivo-compulsi- vas, fuera de la realidad, de la propia valía, preocupaciones
vo, ni en el contexto de un episodio depresivo mayor. de culpa y rumiaciones sobre pequeños errores del pasado
(Criterio A7). Estos pacientes a menudo malinterpretan los
DIAGNÓSTICO DIFERENCIAL acontecimientos diarios neutros o triviales como si proba-
Trastornos bipolares: El rasgo central que diferencia el tras- sen los propios defectos personales y tienen un exagerado
torno de desregulación disruptiva del estado de ánimo y los sentido de la responsabilidad hacia los acontecimientos in-
trastornos bipolares en los niños está relacionado con el cur- apropiados.
so longitudinal de los síntomas nucleares. Es muy frecuente que el paciente se culpe a sí mismo de es-
Trastorno negativista desafiante: Aunque los síntomas del tar enfermo y de fracasar en sus responsabilidades laborales
trastorno negativista desafiante se producen normalmente o interpersonales como consecuencia de la depresión, pero
en los niños con trastorno de desregulación disruptiva del esta actitud, a menos que sea delirante, no se considera su-
estado de ánimo, los síntomas afectivos del trastorno de des- ficiente para cumplir este criterio.
regulación disruptiva del estado de ánimo son relativamente En los pacientes ancianos, las dificultades de memoria pue-
raros en los niños con trastorno negativista desafiante. den ser la queja principal y pueden confundirse con los sig-
Trastorno explosivo intermitente: Los niños con síntomas nos precoces de una demencia (“pseudodemencia”). Cuan-
sugestivos de un trastorno explosivo intermitente presentan do el episodio depresivo
accesos de ira graves muy parecidos a los de los niños con se trata con éxito, los problemas de memoria a menudo des-
trastorno de desregulación disruptiva del estado de ánimo. aparecen por completo. Sin embargo, en algunos pacientes,
Sin embargo, al contrario que en el trastorno de desregula- especialmente en los ancianos, un episodio depresivo mayor
ción disruptiva del estado de ánimo, en el trastorno explo- puede ser a veces la presentación inicial de una demencia
sivo intermitente no se requiere una alteración persistente irreversible.
del estado de ánimo entre los accesos. Además, el trastorno Los pacientes con mayor riesgo de suicidio pueden haber
explosivo intermitente requiere sólo 3 meses de síntomas ac- puesto sus asuntos en orden (p. ej., testamento actualizado,
tivos, a diferencia del trastorno de desregulación disruptiva deudas pagadas), haber adquirido los materiales necesarios
del estado de ánimo, que requiere 12 meses. (p. ej., una cuerda o una pistola) y haber elegido un lugar
y una fecha para consumar el suicidio. Las motivaciones del
TRASTORNO DEPRESIVO MAYOR suicidio pueden ser el deseo de rendirse a la vista de obstá-
culos que se perciben como insuperables, un intenso deseo
CARACTERÍSTICAS DIAGNÓSTICAS de terminar con lo que se percibe como un estado de dolor
Los síntomas de los criterios diagnósticos del trastorno de- emocional interminable y terriblemente doloroso, la incapa-
presivo mayor deberían aparecer casi cada día para poderlos cidad de imaginar ningún disfrute en la vida o el deseo de no
considerar, con la excepción del cambio de peso y la idea- ser una carga para los demás. La resolución de tales pensa-
ción suicida. Debería haber ánimo deprimido la mayor parte mientos puede servir mejor que la negación de dichos planes
del día, además de casi cada día. A menudo los síntomas para medir si disminuye el riesgo de suicidio.
de presentación son el insomnio y la fatiga, y el trastorno
se infradiagnostica si no se reconocen los otros síntomas

42
CARACTERÍSTICAS QUE APOYAN EL DIAGNÓSTICO La probabilidad de los intentos de suicidio disminuye duran-
Los pacientes deprimidos que ingresan en residencias de an- te la época adulta media y tardía, aunque no así el riesgo
cianos tienen una probabilidad notablemente incrementada de suicidio consumado. Las depresiones de inicio en edades
de fallecer en el primer año. La hiperactividad del eje hipofi- tempranas son más familiares y es más probable que conlle-
sario-hipotalámico-adrenal parece relacionarse con la melan- ven asociadas alteraciones de la personalidad. El curso del
colía, los rasgos psicóticos y el riesgo de un posible suicidio. trastorno depresivo mayor no suele cambiar por lo general
con la edad.
PREVALENCIA
7 %. En los sujetos de entre 18 y 29 años es tres veces mayor FACTORES DE RIESGO Y PRONÓSTICOS
que en los pacientes de 60 años o mayores. Las mujeres pre- Temperamentales: El neuroticismo (la afectividad negativa)
sentan tasas que llegan a ser 1,5-3 veces mayores. es un factor de riesgo bien establecido para el comienzo del
trastorno depresivo mayor, y parece que unos niveles altos
DESARROLLO Y CURSO vuelven a los sujetos más proclives a desarrollar episodios
El trastorno depresivo mayor puede aparecer por primera vez depresivos en respuesta a los acontecimientos vitales estre-
a cualquier edad, pero la probabilidad de que se inicie se santes.
incrementa notablemente en la pubertad. En Estados Uni- Ambientales: Los acontecimientos adversos en la infancia,
dos, la incidencia parece tener un pico hacia los 20 años; sin especialmente cuando son múltiples experiencias de diver-
embargo, no es infrecuente que se inicie por primera vez en sos tipos, constituyen un potente conjunto de factores de
la ancianidad. El curso del trastorno depresivo mayor es bas- riesgo de contraer un trastorno depresivo mayor. Los acon-
tante variable, de modo que algunos pacientes apenas tecimientos vitales estresantes están bien reconocidos como
presentan remisiones (períodos de 2 o más meses sin sín- precipitantes de los episodios de depresión mayor, pero la
tomas o con sólo uno o dos síntomas leves), si es que las presencia o ausencia de acontecimientos vitales adversos
presentan, mientras que otros permanecen durante muchos próximos al comienzo de los episodios no parece ser útil
años con pocos o ningún síntoma entre los episodios. para el pronóstico ni la elección del tratamiento.
La recuperación se inicia normalmente dentro de los tres me- Genéticos y fisiológicos: Los familiares de primer grado de
ses siguientes al comienzo en dos de cada cinco pacientes los pacientes con trastorno depresivo mayor tienen un riesgo
con depresión mayor y en el plazo de un año en cuatro de dos a cuatro veces mayor que el de la población general.
cada cinco pacientes. El riesgo relativo parece ser mayor en las formas de inicio
El carácter reciente del inicio es un determinante fuerte de la temprano y recurrentes. La heredabilidad es de aproximada-
probabilidad de recuperación a corto plazo y se puede espe- mente el 40 %, y los rasgos neuróticos de la personalidad
rar que muchos pacientes que sólo han estado deprimidos cuentan en proporción considerable para esta asociación
durante varios meses se recuperen de modo espontáneo. Las genética.
características que se asocian a unas tasas de recuperación Modificadores del curso: Esencialmente, todos los trastornos
menores, además de la duración del episodio actual, son los no afectivos mayores aumentan el riesgo de que un sujeto
rasgos psicóticos, la ansiedad intensa, los trastornos de per- desarrolle depresión. Los episodios de depresión mayor que
sonalidad y la gravedad de los síntomas. se desarrollan en el contexto de otro trastorno suelen seguir
El riesgo de recurrencia se vuelve progresivamente menor con con frecuencia un curso más refractario.
el tiempo conforme se incrementa la duración de la remisión. Las enfermedades médicas crónicas o discapacitantes tam-
El riesgo es mayor en los pacientes cuyo episodio anterior bién aumentan el riesgo de episodios de depresión mayor.
fue grave, en los sujetos jóvenes y en las personas que han Estas enfermedades prevalentes, como la diabetes, la obe-
presentado múltiples episodios. La persistencia de síntomas sidad mórbida y la patología cardiovascular, se complican a
depresivos durante la remisión, aunque sean leves, es un fac- menudo con episodios depresivos, y es más probable que
tor que predice claramente la recurrencia. estos episodios se vuelvan crónicos si se comparan con los
Muchas enfermedades bipolares comienzan con uno o más episodios depresivos de los pacientes sin enfermedades mé-
episodios depresivos, y en una proporción considerable de dicas.
pacientes que inicialmente parecen tener un trastorno depre-
sivo mayor se acaba probando con el tiempo que en realidad ASPECTOS RELACIONADOS CON LA CULTURA
padecen un trastorno bipolar. Esto es más probable en los Los estudios sobre el trastorno depresivo mayor en las dife-
pacientes en que la enfermedad se inicia en la adolescencia, rentes culturas han mostrado diferencias de hasta siete ve-
en los que tienen rasgos psicóticos y en los que tienen ante- ces en las tasas de prevalencia a los doce meses, pero existe
cedentes familiares de trastorno bipolar. La presencia del es- mucha más uniformidad en la proporción mujeres/hombres,
pecificador “con rasgos mixtos” también aumenta el riesgo en las edades medias de inicio y en el grado en que la pre-
de un futuro diagnóstico de manía o hipomanía. sencia del trastorno aumenta la probabilidad de un trastorno
A pesar de que las tasas de prevalencia de los trastornos de- por abuso de sustancias comórbido.
presivos presentan diferencias claras entre ambos sexos, di- La mayoría de los casos de depresión no se reconocen en
chas diferencias entre sexos no parecen existir en relación los centros de atención primaria en la mayor parte de los
con la fenomenología, el curso y la respuesta al tratamiento. países, y de que es muy probable que en muchas culturas los
Asimismo, no se aprecian efectos claros de la edad actual síntomas somáticos sean la queja de presentación. Entre los
sobre el curso o la respuesta al tratamiento en el trastorno síntomas del Criterio A, el insomnio y la pérdida de energía
depresivo mayor. Sin embargo, existen algunas diferencias en son los que se presentan de modo más uniforme.
cuanto a los síntomas: la hipersomnia y la hiperfagia son más
probables en los sujetos jóvenes, y los síntomas melancólicos,
en particular las alteraciones psicomotoras, son más frecuen- ASPECTOS RELACIONADOS CON EL GÉNERO
tes en los sujetos mayores. En las mujeres hay mayor riesgo de intentos de suicidio y el

43
riesgo de suicidio consumado es menor. rrir durante el trastorno depresivo persistente. Los pacientes
cuyos síntomas cumplan los criterios del trastorno depresivo
RIESGO DE SUICIDIO mayor durante dos años se deberían diagnosticar de tras-
El factor de riesgo que se ha descrito de manera más sis- torno depresivo persistente además del trastorno depresivo
temática son los antecedentes de intentos o amenazas de mayor. Los pacientes con trastorno depresivo persistente
suicidio, pero habría que recordar que la mayoría de suici- describen su estado de ánimo como triste o con el ánimo
dios consumados no se han precedido de intentos fallido. “por los suelos”. Durante los períodos de ánimo deprimido
Otras características asociadas a un aumento del riesgo de deben presentarse al menos dos de los seis síntomas del Cri-
suicidio consumado son: ser varón, estar soltero o vivir solo terio B. Como estos síntomas forman parte de la experiencia
y tener intensos sentimientos de desesperanza. La presencia diaria del paciente, sobre todo en los casos de inicio tem-
de trastorno límite de personalidad aumenta notablemente prano (p. ej., “siempre he sido así”), es posible que no se
el riesgo de futuros intentos de suicidio. mencionen a menos que se le pregunte al individuo.

CONSECUENCIAS FUNCIONALES PREVALENCIA


El deterioro puede ser muy ligero, de modo que muchas 0,5 % para el trastorno depresivo persistente y del 1,5 %
personas que interactúan con los individuos afectos no son para el trastorno depresivo mayor crónico.
conscientes de los síntomas depresivos. Sin embargo, el
deterioro puede llegar hasta a la incapacidad completa, de DESARROLLO Y CURSO
modo que el paciente deprimido va a ser incapaz de atender El trastorno depresivo persistente tiene a menudo un inicio
sus necesidades básicas de autocuidado, o sufre mutismo, temprano e insidioso (p. ej., en la infancia, la adolescencia o
o está catatónico. Entre los pacientes que se atienden en la juventud) y, por definición, tiene un curso crónico. Entre
los centros médicos generales, los pacientes con trastorno los individuos con trastorno depresivo persistente y trastorno
depresivo mayor tienen más dolor, más enfermedades físicas límite de la personalidad, la covarianza de los correspondien-
y más disminución del funcionamiento en las áreas física, tes rasgos a lo largo del tiempo sugiere la existencia de un
social y personal. mecanismo común. El comienzo temprano (p. ej., antes de
los 21 años) se asocia a una mayor probabilidad de trastor-
COMORBILIDAD nos comórbidos de la personalidad y de abuso de sustancias.
Se asocia con frecuencia con los trastornos relacionados Cuando los síntomas alcanzan el nivel de un episodio de-
con sustancias, el trastorno de pánico, el trastorno obsesi- presivo mayor, es probable que desciendan posteriormente
vo-compulsivo, la anorexia nerviosa, la bulimia nerviosa y el a un nivel inferior. Sin embargo, los síntomas depresivos re-
trastorno límite de personalidad. miten menos probablemente en un determinado período de
tiempo, en el contexto de un trastorno depresivo recurrente
DIAGNÓSTICO DIFERENCIAL que en el de un episodio depresivo mayor.
Episodios maníacos con ánimo irritable o episodios mixtos:
Los episodios de depresión mayor Con marcado ánimo irrita- FACTORES DE RIESGO Y PRONÓSTICO
ble pueden ser difíciles de diferenciar de los episodios manía- Temperamentales: Los factores que predicen un peor pro-
cos con ánimo irritable o de los episodios mixtos. nóstico a largo plazo son los mayores niveles de neuroticismo
Trastorno del humor debido a otra afección médica. (afectividad negativa), la mayor gravedad de los síntomas,
Trastorno depresivo o bipolar inducido por sustancias/medi- un peor funcionamiento global y la presencia de trastornos
camentos. de ansiedad o de conducta.
Trastorno por déficit de atención/hiperactividad: Si se cum- Ambientales: Los factores de riesgo en la infancia son la pér-
plen los criterios de ambos, se debería diagnosticar el trastor- dida de los padres y su separación.
no por déficit de atención/hiperactividad además del trastor- Genéticos y fisiológicos: No hay diferencias claras entre el
no del ánimo. Sin embargo, el clínico debe tener precaución desarrollo, el curso y la historia familiar de la enfermedad
para no sobrediagnosticar un episodio depresivo mayor en del trastorno distímico y del trastorno depresivo mayor cró-
los niños con TDAH cuya alteración del ánimo se caracterice nico del DSM-IV. Es, por tanto, probable que los pacientes
por irritabilidad. con trastorno depresivo persistente tengan una mayor pro-
Trastorno de adaptación con ánimo deprimido: El episodio porción de familiares de primer grado con dicho trastorno.
depresivo mayor que ocurre en respuesta a un factor estre- Se han implicado varias regiones cerebrales (p. ej., el córtex
sante psicosocial se distingue del trastorno de adaptación prefrontal, el cíngulo anterior, la amígdala, el hipocampo)
con ánimo deprimido por el hecho de que los criterios diag- en los trastornos depresivos persistentes. Además, es posible
nósticos completos del episodio depresivo mayor no se cum- que existan también alteraciones polisomnográficas.
plen en el trastorno de adaptación.
Tristeza. No debería diagnosticarse episodio depresivo ma- CONSECUENCIAS FUNCIONALES
yor a menos que se cumplan los criterios de gravedad. Pueden ser incluyo mayores que las del trastorno depresivo
mayor.

DIAGNÓSTICO DIFERENCIAL
TRASTORNO DEPRESIVO-PERSISTENTE (DISTI- Trastorno depresivo mayor: Si existe ánimo deprimido ade-
MIA) más de dos o más síntomas de los criterios del episodio de-
presivo persistente durante dos años o más, entonces se rea-
CARACTERÍSTICAS DIAGNÓSTICAS liza el diagnóstico de trastorno depresivo persistente.
La depresión mayor puede preceder al trastorno depresivo
persistente, y los episodios de depresión mayor pueden ocu-

44
El diagnóstico depende de la duración de dos años, que lo CARACTERÍSTICAS QUE APOYAN EL DIAGNÓSTICO
distingue de los episodios de depresión que duran menos de Se han descrito delirios y alucinaciones en la fase lútea tardía
dos años. Si los síntomas cumplen criterios suficientes para el del ciclo menstrual, aunque son raros. Algunos consideran la
diagnóstico de un episodio depresivo mayor en cualquier mo- fase premenstrual un período con riesgo de suicidio.
mento de este período, entonces se debería hacer el diagnós-
tico de depresión mayor, pero no se codifica como un diag- PREVALENCIA
nóstico separado, sino como un especificador del trastorno La prevalencia a los doce meses del trastorno disfórico pre-
depresivo persistente. Si los síntomas del paciente cumplen menstrual está entre el 1,8 y el 5,8 % de las mujeres con
en el momento actual todos los criterios del episodio depresi- menstruación. Hay una sobrestimación de la prevalencia si
vo mayor, entonces se debería usar el especificador “con epi- las cifras se basan en informes retrospectivos más que en
sodios intermitentes de depresión mayor con el episodio ac- evaluaciones diarias prospectivas. Sin embargo, la prevalen-
tual”. Si el episodio depresivo mayor ha tenido una duración cia estimada, basada en un registro diario de los síntomas
de al menos dos años y sigue presentándose, entonces se usa durante 1-2 meses, puede ser menos representativa, ya que
el especificador “con episodio de depresión mayor persisten- las pacientes con síntomas más graves quizá no sean capa-
te”. Cuando no se cumplen todos los criterios del episodio ces de mantener el proceso de registro. La estimación más
depresivo mayor en el momento actual, pero ha existido al rigurosa del trastorno disfórico premenstrual es del 1,8 %
menos un episodio de depresión mayor en el contexto de al en las mujeres que cumplen todos los criterios sin deterioro
menos dos años de síntomas depresivos persistentes, enton- funcional, y del 1,3 % en las mujeres que cumplen los cri-
ces se usa el especificador “con episodios intermitentes de terios con deterioro funcional y síntomas concomitantes de
depresión mayor, sin episodio actual”. Si el paciente no ha otro trastorno mental.
presentado un episodio de depresión mayor en los últimos
dos años, entonces se usa el especificador “con síndrome DESARROLLO Y CURSO
distímico puro”. El inicio del trastorno disfórico premenstrual puede aparecer
en cualquier momento tras la menarquia. Muchas pacientes
Trastornos psicóticos: Los síntomas depresivos son un rasgo refieren que los síntomas empeoran al acercarse la meno-
común que se asocia a los trastornos psicóticos crónicos. No pausia. Los síntomas cesan tras la menopausia, aunque el
se hace un diagnóstico distinto de trastorno depresivo persis- tratamiento hormonal sustitutivo cíclico puede precipitar de
tente si los síntomas ocurren durante el curso del trastorno nuevo la presentación de síntomas.
psicótico (incluidas las fases residuales).
FACTORES DE RIESGO Y PRONÓSTICO
Trastorno depresivo o bipolar debido a otra afección médica. Ambientales: Los factores ambientales que se asocian a
la presentación del trastorno disfórico premenstrual son
Trastorno depresivo o bipolar inducido por sustancias/medi- el estrés, los antecedentes de traumas interpersonales, los
camentos. cambios estacionales y a los aspectos socioculturales de la
conducta sexual femenina en general y del papel del género
Trastornos de la personalidad. A menudo hay signos de la femenino en particular.
coexistencia de alteraciones de la personalidad. Cuando la Genéticos y fisiológicos: No se conoce la heredabilidad del
presentación en un individuo cumple los criterios tanto del trastorno disfórico premenstrual. Sin embargo, se estima
trastorno depresivo persistente como de un trastorno de la que el rango de heredabilidad para los síntomas premens-
personalidad, se deben realizar ambos diagnósticos. truales oscila entre un 30 y un 80 %, siendo la estimación
de la heredabilidad de un 50 % para el componente más
COMORBILIDAD estable de los síntomas premenstruales.
En comparación con los pacientes con trastorno depresivo Modificadores de curso: Es posible que las mujeres que usan
mayor, los que tienen trastorno depresivo persistente tienen anticonceptivos orales tengan menos síntomas premens-
mayor riesgo de comorbilidad psiquiátrica en general y de truales que las mujeres que no los usan.
trastorno de ansiedad y abuso de sustancias en particular.
ASPECTOS RELACIONADOS CON LA CULTURA
El trastorno disfórico premenstrual no es un síndrome aso-
TRASTORNO DISFÓRICO PREMENSTRUAL ciado a una cultura.
Sin embargo, los factores culturales pueden influir significa-
tivamente sobre la frecuencia, la intensidad y la expresividad
de los síntomas, y sobre los patrones de búsqueda de ayuda.
CARACTERÍSTICAS DIAGNÓSTICAS
Típicamente, los síntomas alcanzan su cumbre alrededor del MARCADORES DIAGNÓSTICOS
comienzo de la menstruación. Aunque no es infrecuente que El diagnóstico se confirma tras dos meses de evaluación
los síntomas permanezcan durante los primeros días de la prospectiva de los síntomas. Se han validado varias escalas,
menstruación, la paciente debe tener un período libre de sín- incluidas la Evaluación diaria de la gravedad de los proble-
tomas en la fase folicular después de que comience el perío- mas y la Escala analógica visual para los síntomas anímicos
do menstrual. Para confirmar el diagnóstico provisional se premenstruales. Se ha validado la Escala de evaluación de la
requieren evaluaciones diarias prospectivas de los síntomas tensión premenstrual, que tiene una versión autoaplicada y
durante al menos dos ciclos sintomáticos. otra para un observador, y ambas versiones se usan mucho
para medir la gravedad de la enfermedad en las mujeres con
trastorno disfórico premenstrual

45
CONSECUENCIAS FUNCIONALES
El deterioro del funcionamiento social puede manifestarse a
través de desavenencias matrimoniales y de problemas con
los niños, con otros miembros de la familia o con los amigos.

COMORBILIDAD
El trastorno previo que con más frecuencia se refiere en las
pacientes con trastorno disfórico premenstrual es el episo-
dio depresivo mayor. Existe un amplio espectro de trastornos
médicos que pueden empeorar en la fase premenstrual; sin
embargo, la ausencia de un período libre de síntomas duran-
te el intervalo posmenstrual obvia el diagnóstico de trastorno
disfórico premenstrual.

DIAGNÓSTICO DIFERENCIAL
Síndrome premenstrual. El síndrome premenstrual difiere del
trastorno disfórico premenstrual en que no se requiere un
mínimo de cinco síntomas y en que no existe un convenio
de síntomas afectivos en las mujeres con síndrome premens-
trual. La presencia de síntomas físicos o conductuales en la
premenstruación, sin los síntomas afectivos requeridos, pro-
bablemente cumpla los criterios del síndrome premenstrual.

Dismenorrea: La dismenorrea es un síndrome de menstrua-


ciones dolorosas que difiere del caracterizado por cambios
afectivos. Además, los síntomas de la dismenorrea comien-
zan al inicio de la menstruación.

Trastorno bipolar, trastorno depresivo mayor y trastorno de-


presivo persistente (distimia): Muchas mujeres con trastorno
bipolar o trastorno depresivo mayor o trastorno depresivo
persistente (distimia) (natural o inducido por sustancias/
medicamentos) creen que tienen un trastorno disfórico pre-
menstrual. Sin embargo, cuando exponen los síntomas se
dan cuenta de que éstos no siguen un patrón premenstrual.
El solapamiento de síntomas es particularmente relevante
para diferenciar el trastorno disfórico premenstrual del tras-
torno depresivo mayor, del trastorno depresivo persistente,
de los trastornos bipolares y del trastorno límite de la perso-
nalidad. Sin embargo, la tasa de trastornos de la persona-
lidad no es más alta en las mujeres con trastorno disfórico
premenstrual que en las que no lo presentan.

Uso de tratamientos hormonales: Si la mujer suspende las


hormonas y los síntomas desaparecen, se puede deducir que
se trata de un trastorno depresivo inducido por sustancias/
medicamentos.

46
TRASTORNO BIPOLAR Y TRASTOR- CONSECUENCIAS FUNCIONALES
Aunque muchos individuos con trastorno bipolar recuperan
NOS RELACIONADOS un nivel funcional completo entre los episodios, aproxima-
damente un 30 % presenta una alteración grave de la fun-
ción en el ámbito laboral.
TRASTORNO BIPOLAR I La recuperación funcional es más tardía que la sintomática,
especialmente la ocupacional, lo que hace que, a igualdad
CARACTERÍSTICAS ASOCIADAS QUE APOYAN EL DIAG-
de nivel educativo, el nivel socioeconómico sea menor que
NÓSTICO
entre la población general.
Los episodios maníacos pueden acompañarse de juego pa-
tológico o conductas antisociales. Algunos pacientes pue-
COMORBILIDAD
den ponerse agresivos; si están delirantes, pueden atacar fí-
La concurrencia de trastornos mentales es frecuente, siendo
sicamente a otras personas o suicidarse. El estado de ánimo
los trastornos más frecuentes los de ansiedad (ej. ataques
puede variar muy rápidamente hacia el enfado o hacia la
de pánico, trastorno de ansiedad social, fobia específica), y
depresión. Durante los episodios maníacos pueden aparecer
afecta aproximadamente a las tres cuartas partes de estos
síntomas depresivos que duran momentos, horas o, más ra-
pacientes, más de la mitad de los pacientes con trastorno
ramente días.
bipolar I presenta TDAH, trastorno de conducta destructiva
PREVALENCIA
o impulsiva, cualquier trastorno de conducta, o cualquiera
0,6 %. La tasa de prevalencia a lo largo de la vida de los
de los trastornos por uso de sustancias. Los adultos con tras-
hombres respecto a las mujeres es de aproximadamente
torno bipolar I tienen tasas más altas de afecciones médicas
1,1:1.
concomitantes graves o sin tratar.
DESARROLLO Y CURSO
La edad de comienzo del primer episodio maníaco, hipoma-
níaco o depresivo mayor es aproximadamente de 18 años TRASTORNO BIPOLAR II
para el trastorno bipolar I, aunque puede darse a lo largo de
todo el ciclo vital, incluso con cuadros iniciales a los 60 o 70
años. Si el comienzo de los síntomas maníacos se produce CARACTERÍSTICAS ASOCIADAS QUE APOYAN EL DIAG-
en la edad adulta tardía o en la tercera edad, se debería NÓSTICO
considerar la posibilidad de una enfermedad orgánica (p. ej., Un rasgo común del trastorno bipolar II es la impulsividad,
trastorno neurocognitivo frontotemporal) o el consumo o que puede contribuir a los intentos de suicidio y a los tras-
abstinencia de alguna sustancia. tornos por consumo de sustancias.
Más del 90% de los pacientes que tienen un único episodio
maníaco vuelve a tener episodios recurrentes. Aproximada- PREVALENCIA
mente el 60% de los episodios maníacos ocurre inmediata- La prevalencia durante 12 meses del trastorno bipolar II, a
mente antes de un episodio depresivo mayor. nivel internacional, es del 0,3%.
FACTORES DE RIESGO Y PRONÓSTICO
Ambientales: más frecuente en los países ricos que en los DESARROLLO Y CURSO
países pobres (1,4 vs.0,7 %). Existen mayores tasas de tras- La edad media de comienzo es hacia la mitad de la veintena,
torno bipolar I entre los separados, divorciados y viudos que lo que significa que es ligeramente más tardío que el trastor-
entre las personas casadas y las que nunca lo han estado. no bipolar I, pero más temprano que el trastorno depresivo
Genéticos y fisiológicos: existe un riesgo promedio 10 ve- mayor. La enfermedad suele iniciarse con un episodio de-
ces superior entre los familiares adultos de los pacientes con presivo y no se reconoce como trastorno bipolar II hasta que
trastorno bipolar I y II. La esquizofrenia y el trastorno bipolar aparece un episodio hipomaníaco; esto ocurre en alrededor
probablemente comparten un origen genético. del 12% de los pacientes con diagnóstico inicial de trastor-
Modificadores del curso: si un individuo ha tenido un episo- no depresivo mayor. Pueden preceder al diagnóstico la an-
dio maníaco con rasgos psicóticos, es más probable que los siedad, el abuso de sustancias y los trastornos de conducta
siguientes episodios incluyan rasgos psicóticos. La recupera- alimentaria, lo que complica su detección.
ción interepisódica incompleta es más frecuente si el episo- El intervalo interepisódico tiende a disminuir con la edad.
dio actual se acompaña de rasgos psicóticos incongruentes Aproximadamente un 5-15% de los pacientes con trastorno
con el estado de ánimo. bipolar II va a desarrollar un episodio maníaco, debiéndose
cambiar el diagnóstico a trastorno bipolar I con independen-
ASPECTOS RELACIONADOS CON EL GÉNERO cia del curso posterior.
Las mujeres presentan con más probabilidad ciclos rápidos y
estados mixtos. Las mujeres con trastorno bipolar I o II pre-
sentan síntomas depresivos con mayor probabilidad que los FACTORES DE RIESGO Y PRONÓSTICO
hombres. También tienen mayor riesgo que los hombres de Genéticos y fisiológicos: parecen existir factores genéticos
presentar a lo largo de la vida un trastorno por consumo de que influyen sobre la edad de inicio de los trastornos bipo-
alcohol. lares.
Modificadores del curso: el patrón de ciclos rápidos se aso-
RIESGO DE SUICIDIO cia a un peor pronóstico. En los pacientes con trastorno bi-
Se estima que el riesgo de suicidio es 15 veces superior al de polar, una mayor educación, menos años de enfermedad y
la población general. El trastorno bipolar supone un cuarto estar casado son factores que se asocian de forma indepen-
de todos los suicidios consumados. diente con la recuperación funcional.

47
ASPECTOS RELACIONADOS CON EL GÉNERO TRASTORNOS DE ANSIEDAD
Es más frecuente en las mujeres, y presentan con mayor pro-
babilidad hipomanía con rasgos depresivos mixtos y un curso
de ciclos rápidos. TRASTORNO DE PÁNICO
RIESGO DE SUICIDIO CARACTERÍSTICAS DIAGNÓSTICAS
Es mayor en el trastorno bipolar II. Aproximadamente, un Ataques de pánico inesperados y recurrentes (4/13 sínto-
tercio de los pacientes con trastorno bipolar II refiere antece- mas), seguidos durante 1 mes o más de inquietud o preocu-
dentes de intento de suicidio. pación acerca de los ataques de pánico o sus consecuencias
y/o un cambio significativo en el comportamiento relaciona-
CONSECUENCIAS FUNCIONALES do con los ataques (evitar esfuerzo físico, reorganizar vida
Aunque muchos pacientes vuelven a un nivel completo de diaria para garantizar la disponibilidad de ayuda en caso de
funcionamiento entre los episodios afectivos, al menos un crisis, evitar situaciones agorafóbicas). No se explica por or-
15% continúa con algún tipo de disfunción entre los episo- ganicidad ni por otro trastorno mental.
dios y un 20% entra directamente en otro episodio afectivo
sin recuperación interepisódica. CARACTERÍSTICAS QUE APOYAN EL DIAGNÓSTICO
Ataque de pánico nocturno (despertar del sueño en un es-
COMORBILIDAD tado de pánico): al menos 1 vez en 1/4 - 1/3 de las personas
Se presenta habitualmente asociado a uno o más trastornos con trastorno de pánico (que además suelen tener ataques
mentales, siendo los trastornos de ansiedad los más frecuen- durante el día).
tes. Aproximadamente el 60% de los pacientes con trastor- Sensaciones de ansiedad que están más relacionadas con
no bipolar II presenta al mismo tiempo tres o más trastornos problemas de salud física o mental y creencia de que un sín-
mentales, el 75% tiene un trastorno de ansiedad y el 37% toma físico leve o el efecto adverso de un medicamento ten-
tiene un trastorno por abuso de sustancias. El 14% de los drá un resultado catastrófico. Preocupaciones sobre si será
pacientes con trastorno bipolar II tiene a lo largo de su vida posible completar tareas diarias o sobrellevar el estrés diario,
al menos un trastorno de la conducta alimentaria, siendo el uso excesivo de drogas/medicamentos o comportamientos
trastorno por atracones más frecuente que la bulimia y la extremos para controlar ataques.
anorexia nerviosa.
PREVALENCIA
CICLOTIMIA 2-3% en adultos y adolescentes. Es baja antes de los 14 años
(< 0,4%) y en personas mayores (0,7%).
PREVALENCIA Más prevalente en mujeres que en hombres (2:1). La diferen-
La prevalencia a lo largo de la vida es de aproximadamente ciación se produce en la adolescencia (es observable antes de
0,4-1%. La prevalencia dentro de los trastornos del humor los 14 años), donde se produce un aumento gradual, espe-
oscila entre el 3 y el 5%. En la población general, el trastorno cialmente en las mujeres, alcanzando un máximo durante la
ciclotímico es igualmente frecuente en los hombres que en edad adulta.
las mujeres. En los ambientes clínicos es más probable que
acudan a tratamiento las mujeres con trastorno ciclotímico DESARROLLO Y CURSO
que los hombres. La edad media de inicio es de 20 a 24 años. Inicio poco fre-
cuente en la infancia y después de los 45 años.
DESARROLLO Y CURSO Curso crónico con oscilaciones. Sólo una minoría alcanza
Comienza normalmente en la adolescencia o en la edad una remisión completa durante años sin ninguna recaída.
adulta temprana. El comienzo del trastorno ciclotímico suele Habitualmente el curso se complica por trastornos de ansie-
ser insidioso, y el curso es persistente. Existe un riesgo del dad, depresión o uso de sustancias.
15-50% de que un paciente con trastorno ciclotímico desa- Los adolescentes pueden están menos preocupados por los
rrolle posteriormente un trastorno bipolar I o II. En los niños ataques que los adultos.
con trastorno ciclotímico, la edad promedio de inicio son los En adultos mayores: baja prevalencia relacionada con una
6 años y medio. respuesta más débil del sistema nervioso autónomo a los
estados emocionales; cuadro híbrido de ataques de pánico
FACTORES DE RIESGO Y PRONÓSTICO con síntomas limitados y trastorno de ansiedad generalizada;
Genéticos y fisiológicos: trastorno depresivo mayor, el tras- atribución de los síntomas a ciertas circunstancias estresan-
torno bipolar I y el trastorno bipolar II son más frecuentes tes (incluso si ocurrieron de manera inesperada), impidiendo
entre los familiares biológicos de primer grado de los pacien- el diagnóstico de trastorno de pánico.
tes con trastorno ciclotímico que entre la población general.

COMORBILIDAD FACTORES DE RIESGO Y PRONÓSTICO


Pueden presentar trastornos relacionados con el uso de sus- Temperamentales: afectividad negativa (neuroticismo) y
tancias y trastornos del sueño. La mayoría de los niños con sensibilidad a la ansiedad. Historia de “episodios de miedo”
trastorno ciclotímico que son atendidos en las consultas de (ataques con síntomas limitados). La ansiedad por separa-
psiquiatría tiene trastornos mentales comórbidos; es más ción grave en la infancia puede preceder.
probable que tengan comorbilidad con el TDAH que otros Ambientales: abusos sexuales y malos tratos en la infancia.
pacientes pediátricos con trastornos mentales. Fumar. Factores de estés identificables.
Genéticos y fisiológicos. vulnerabilidad genética. Importan-
cia de la amígdala y estructuras relacionadas. Mayor riesgo

48
en hijos de padres con ansiedad, depresión y trastornos bi- ESPECIFICADOR DEL ATAQUE DE PÁNICO
polares. Algunas dificultades respiratorias (asma).
CARACTERÍSTICAS DIAGNÓSTICAS
ASPECTOS RELACIONADOS CON LA CULTURA Aparición súbita de miedo o malestar intenso que alcanza su
Las preocupaciones específicas sobre los ataques o sus con- máximo en cuestión de minutos, tiempo durante el cual se
secuencias tienden a cambiar de una cultura a otra. En indi- presentan 4/13 síntomas.
viduos de ascendencia africana los criterios del trastorno de Ataques de “síntomas limitados”: ataques que cumplen to-
pánico sólo se cumplen cuando el trastorno es grave y existe dos los demás criterios, pero que tienen menos de 4 sínto-
un deterioro importante. mas.
Pueden ser “esperados” o “inesperados”. Se requieren ata-
ASPECTOS RELACIONADOS CON EL GÉNERO ques inesperados y recurrentes para el diagnóstico de Tras-
En mujeres, asociación entre trastorno de pánico y gen de la torno de pánico.
catecol-O-metiltransferasa (COMT). Se pueden objetivar síntomas específicos de la cultura (p. ej.,
zumbido de oídos, dolor de cuello, dolor de cabeza, gritos
MARCADORES DIAGNÓSTICOS incontrolables); pero no deben contar como uno de los 4
El lactato de sodio, la cafeína, el isoproterenol, la yohimbina, síntomas requeridos para el diagnóstico.
el dióxido de carbono y la colecistocinina, pueden provocar
ataques de pánico en individuos con el trastorno, en un gra- CARACTERÍSTICAS QUE APOYAN EL DIAGNÓSTICO
do mucho mayor que en controles sanos. Ataque de pánico nocturno (despertarse del sueño en un
estado de pánico).
RIESGO DE SUICIDIO Los ataques de pánico están relacionados con mayor tasa de
Los ataques de pánico y la presencia de trastorno de pánico intentos e ideación suicida.
en los últimos 12 meses están relacionados con mayor tasa
de intentos e ideación suicida en los últimos 12 meses. PREVALENCIA
11,2% en adultos (en población general). Baja prevalencia
CONSECUENCIAS FUNCIONALES en niños y personas mayores.
Altos niveles de interferencia en el funcionamiento social, Mayor prevalencia en mujeres que en hombres (diferencia
ocupacional y físico, costes económicos y mayor número de todavía más pronunciada en el trastorno de pánico).
visitas médicas. En adultos mayores: deterioro en tareas de
cuidado y actividades de voluntariado. Los ataques de pánico DESARROLLO Y CURSO
con síntomas completos se asocian a mayor morbilidad que La edad media de inicio es de 22-23 años. Influenciado por
los ataques de síntomas limitados. el curso de cualquier otro trastorno mental concurrente y de
eventos estresantes.
DIAGNÓSTICO DIFERENCIAL Los adolescentes parecen estar menos dispuestos que los
- Otros trastornos de ansiedad especificados o tras- adultos a hablar de los ataques de pánico.
torno de ansiedad no especificado (ej. “síntomas limita-
dos”): no hay síntomas completos e inesperados. FACTORES DE RIESGO Y PRONÓSTICO
- Trastorno de ansiedad debido a otra condición mé- Temperamentales: afectividad negativa (neuroticismo) y
dica: organicidad (p. ej., hipertiroidismo, hiperparatiroidis- sensibilidad a la ansiedad. Historia de “episodios de miedo”
mo, feocromocitoma, etc.). (ataques con síntomas limitados).
- Trastorno de ansiedad inducido por sustancias/me- Ambientales: fumar, factores de estrés.
dicamentos: efecto fisiológico de una sustancia (p. ej., psi-
coestimulantes). ASPECTOS RELACIONADOS CON LA CULTURA
- Otros trastornos mentales con ataques de páni- La frecuencia de cada uno de los 13 síntomas del ataque de
co como característica asociada (p. ej., otros trastornos de pánico varía interculturalmente.
ansiedad y trastornos psicóticos): los ataques de pánico en Los síndromes culturales influyen en la presentación de los
otros trastornos de ansiedad son esperados y se producen ataques de pánico (síntomas, frecuencia, atribución indivi-
sólo en respuesta a desencadenantes específicos. dual de lo inesperado) entre diferentes culturas, pudiendo
originar miedo a ciertas situaciones que van desde discusio-
COMORBILIDAD nes personales (ataque de nervios) hasta algunos tipos de
Poca frecuencia en ausencia de otra psicopatología. Tras- ejercicio (ataques khyál), pasando por el viento atmosférico
tornos de ansiedad (especialmente agorafobia), depresión (ataques trúng gió). La aclaración de las atribuciones cultu-
mayor (10-65%; en 1/3 la depresión precede al pánico), rales puede ayudar a distinguir los ataques inesperados de
trastorno bipolar y trastorno por consumo moderado de al- los esperados.
cohol. El trastorno de pánico a menudo tiene una edad de
inicio anterior a la del trastorno comórbido. Si se produce ASPECTOS RELACIONADOS CON EL GÉNERO
después, puede ser indicador de gravedad de la enfermedad Más comunes en mujeres. Las características clínicas no di-
comórbida. Síntomas y afecciones médicas generales (ma- fieren entre mujeres y hombres.
reos, arritmias, hipertiroidismo, asma, EPOC, síndrome de
intestino irritable, etc.). MARCADORES DIAGNÓSTICOS
Aumento brusco de la excitación, frecuencia cardíaca e ines-
tabilidad cardiorrespiratoria.

CONSECUENCIAS FUNCIONALES

49
Los ataques de pánico asociados a otros trastornos mentales 10% remisión completa.
pueden tener una mayor gravedad, tasas más altas de co- A largo plazo se asocia con riesgo de trastorno depresivo
morbilidad, tendencia suicida y peor respuesta al tratamien- mayor secundario, trastorno depresivo persistente y trastor-
to. no por consumo de sustancias.
Los ataques de pánico completos (vs. síntomas limitados) El tipo de situaciones agorafóbicas temidas puede cambiar
presentan mayor morbilidad. (en niños: estar solo fuera de casa; en adultos mayores: estar
en tiendas, haciendo cola y en espacios abiertos).
Cogniciones:
DIAGNÓSTICO DIFERENCIAL - En niños: perderse.
- Otros episodios paroxísticos (p. ej., “ataques de - En adultos: experimentar síntomas de ansiedad.
ira”): estados emocionales diferentes del miedo o malestar - En ancianos: caerse.
intenso.
- Trastorno de ansiedad debido a otra condición mé- FACTORES DE RIESGO Y PRONÓSTICO
dica: organicidad (p. ej., hipertiroidismo, hiperparatiroidis- Temperamentales: inhibición conductual, neuroticismo,
mo, feocromocitoma). sensibilidad a la ansiedad.
- Trastorno de ansiedad inducido por sustancias/me- Ambientales. Sucesos negativos y/o estresantes en la infan-
dicamentos: efecto fisiológico de una sustancia (p. ej., psi- cia, familia con escasa calidez y excesiva sobreprotección.
coestimulantes). Genéticos y fisiológicos: heredabilidad del 61%. Fobia con
- Trastorno de pánico: ataques de pánico inesperados mayor contribución genética.
y recidivantes, seguidos durante 1 mes o más de preocupa-
ción por las crisis y/o cambios en el comportamiento. ASPECTOS RELACIONADOS CON EL GÉNERO
Los hombres presentan mayor comorbilidad con trastornos
COMORBILIDAD por consumo de sustancias.
Trastornos de ansiedad, depresivos, bipolares, de control de
impulsos y por consumo de sustancias. CONSECUENCIAS FUNCIONALES
Deficiencia y discapacidad funcional, menos productividad
laboral, más días de incapacidad. 1/3 confinados en casa.

AGORAFOBIA DIAGNÓSTICO DIFERENCIAL


- Fobia específica situacional: si el miedo, la ansiedad y la
CARACTERÍSTICAS DIAGNÓSTICAS evitación se limitan a una de las situaciones de agorafobia, se
Miedo o ansiedad intensa, desproporcionada al peligro real, diagnosticaría fobia específica situacional (vs agorafobia). Se
provocada por exposición real o anticipada a una amplia diagnosticaría agorafobia (vs fobia específica situacional) si
gama de situaciones (2/5), debido a la idea de que escapar hay temor a dos o más de las situaciones de agorafobia. Si
podría ser difícil o en el caso de experimentar ansiedad o la situación se teme por razones distintas a experimentar sín-
síntomas incapacitantes o embarazosos (vómitos, síntomas tomas similares a la ansiedad o síntomas incapacitantes o
infamatorios del intestino) podría no disponer de ayuda. En embarazosos, se diagnosticaría fobia específica.
personas mayores: miedo a caerse. En niños: desorientación, - Trastorno de ansiedad por separación: miedo al desapego
encontrarse perdidos. Las situaciones casi siempre provocan de las figuras de apego y el hogar.
miedo o ansiedad, se evitan activamente o se resisten con - Trastorno de ansiedad social: miedo a ser evaluado nega-
miedo o ansiedad. El cuadro dura al menos 6 meses, general tivamente.
malestar, y no se explica mejor por organicidad ni por otro - Trastorno de pánico: los ataques de pánico no se ex-
trastorno mental. tienden a la evitación de dos o más situaciones agorafóbicas.
- Trastorno de estrés agudo y trastorno de estrés postraumá-
CARACTERÍSTICAS QUE APOYAN EL DIAGNÓSTICO tico: acontecimiento traumático.
Confinamiento en casa, dependencia de los demás para - Trastorno depresivo mayor: evitar dejar la casa porapatía,
servicios, asistencia o necesidades básicas. Desmoralización, anergia, baja autoestima y anhedonia.
síntomas depresivos, abuso de alcohol y sedantes, autome- - Otras afecciones médicas: organicidad (p. ej., trastornos
dicación. neurodegenerativos con alteraciones motoras asociadas,
trastornos cardiovasculares). Evitación de situaciones por
PREVALENCIA preocupación realista por su incapacidad o vergüenza.
1,7% de los adolescentes y adultos son diagnosticados
anualmente. En mayores de 65 años, la prevalencia anual es COMORBILIDAD
0,4%. Máxima incidencia en adolescencia tardía y adultez Otros trastornos de ansiedad (que suelen preceder a la ago-
temprana. Doble de probabilidad de experimentar agorafo- rafobia) y trastornos depresivos y por consumo de alcohol
bia en mujeres que en hombres. (que suelen ser secundarios), y trastorno de estrés postrau-
mático.
DESARROLLO Y CURSO
En población general, el 30% tienen ataques de pánico o
trastorno de pánico antes (vs. el 50% en población clínica).
2/3 agorafobia: inicio antes de los 35 años. 2 picos: ado-
lescencia tardía/adultez temprana; después de los 40 años.
Edad media de inicio: 17 años. Edad de inicio sin ataques
de pánico previos: 25-29 años. Curso persistente y crónico.

50
FOBIA ESPECÍFICA RIESGO DE SUICIDIO
60% más de probabilidad de intentos de suicidio. Podría
CARACTERÍSTICAS DIAGNÓSTICAS deberse a la comorbilidad con trastornos de personalidad y
Miedo o ansiedad intensa por un objeto o situación especí- otros trastornos de ansiedad.
fica. La exposición al estímulo fóbico provoca casi invariable-
mente una respuesta de ansiedad inmediata. El sujeto evita
activamente la situación o la resiste con elevado miedo o CONSECUENCIAS FUNCIONALES
ansiedad. El miedo o ansiedad son desproporcionados en Deterioro del funcionamiento psicosocial y disminución de la
relación con el peligro real. El cuadro dura 6 meses o más, calidad de vida. La angustia y el deterioro tienden a aumen-
genera malestar, y no se explica por otro trastorno mental. tar con el número de estímulos fóbicos.

ESPECIFICADORES DIAGNÓSTICO DIFERENCIAL


Temor a un promedio de 3 objetos/situaciones, y en un 75% - Agorafobia: miedo a dos o más situaciones agorafó-
temor a más de una situación y objeto. bicas (vs. una para la fobia específica situacional). Las situa-
ciones son temidas o evitadas debido a tener pensamientos
CARACTERÍSTICAS QUE APOYAN EL DIAGNÓSTICO sobre el desarrollo de un síntoma de un ataque de pánico u
Suelen experimentar mayor activación fisiológica cuando an- otros incapacitantes o humillantes en situaciones en las que
ticipan o se exponen al estímulo fóbico. La respuesta fisioló- sería difícil escapar o disponer de ayuda (vs. otros motivos,
gica varía. Fobias específicas situacionales, al entorno natural como miedo a ser dañado por el estímulo fóbico).
y a animales: activación del sistema nervioso simpático. Fobia - Trastorno de ansiedad social: miedo a la evaluación
específica a la sangre-inyección-herida: respuesta vasovagal social negativa.
de desmayo. - Trastorno de ansiedad por separación: situaciones
temidas debido a la separación del cuidador principal/figura
PREVALENCIA de apego.
7-9% en población general. 5% en niños. 16% en adoles- - Trastorno de pánico: crisis de pánico de forma ines-
centes (13-17 años). 3-5% en personas mayores. perada, no sólo en respuesta al estímulo fóbico.
Mayor prevalencia en mujeres que en hombres (2:1), aunque - Trastorno obsesivo-compulsivo: el miedo o ansiedad
las tasas varían en función del estímulo fóbico: es hacia un objeto o situación como consecuencia de ideas
- Fobias específicas a los animales, al entorno natural obsesivas.
y las situacionales: predominio en mujeres. - Trauma y trastornos relacionados con estresantes: si
- Fobia específica a la sangre-inyección-herida: similar se desarrolla la fobia después de un evento traumático hay
en ambos sexos. que considerar el diagnóstico de trastorno de estrés postrau-
mático.
DESARROLLO Y CURSO - Trastornos de la alimentación: el comportamiento
Tras un acontecimiento traumático, la observación de otros de evitación se limita exclusivamente a los alimentos y aspec-
individuos que sufren traumatismos, una crisis de pánico in- tos relacionados.
esperada en una situación que se convertirá en temida o la - Trastornos del espectro de la esquizofrenia y otros
transmisión de información. trastornos psicóticos: el miedo y la evitación se deben a
Generalmente se desarrolla en la primera infancia, antes de creencias delirantes.
los 10 años, entre los 7 y los 11 (siendo la edad media de
unos 10 años). Las situacionales presentan una edad de ini- COMORBILIDAD
cio más tardía. Las que se inician en la infancia y la adoles- Poco frecuente en ausencia de otra psicopatología. Se asocia
cencia suelen sufrir altibajos durante ese periodo. Las que a depresión en personas mayores.
persisten en la edad adulta no suelen remitir. Mayor riesgo de desarrollar otros trastornos: de ansiedad,
A pesar de que se suelen desarrollar en la infancia y la ado- depresivos y bipolares, por consumo de sustancias, síntomas
lescencia, pueden iniciarse a cualquier edad, normalmente somáticos y relacionados y trastornos de personalidad (espe-
tras experiencias traumáticas. cialmente el trastorno dependiente de la personalidad).

FACTORES DE RIESGO Y PRONÓSTICO


Temperamentales: afectividad negativa (neuroticismo), inhi- TRASTORNO DE ANSIEDAD SOCIAL (FOBIA
bición conductual. SOCIAL)
Ambientales: sobreprotección o pérdida de los padres, sepa-
ración, maltrato físico y abusos sexuales. Exposiciones nega- CARACTERÍSTICAS DIAGNÓSTICAS
tivas o traumáticas al estímulo fóbico. Miedo intenso o ansiedad ante situaciones sociales en las
Genéticos y fisiológicos: susceptibilidad genética a una de- que el sujeto puede ser analizado por los demás. Miedo a
terminada categoría de fobia específica. Las personas con actuar de una cierta manera o de mostrar síntomas de ansie-
fobia a la sangre-inyección-herida suelen presentar una res- dad que se valoren negativamente. Algunas personas tienen
puesta vasovagal. miedo a ofender a los demás y ser rechazados. Hay quienes
temen y evitan orinar en los baños públicos cuando están
ASPECTOS RELACIONADOS CON LA CULTURA presentes otras personas (paruresis o “síndrome de la ve-
Diferencias interculturales en prevalencia, contenido de la jiga tímida”). Las situaciones sociales casi siempre generan
fobia, edad de inicio y frecuencias por género. miedo o ansiedad, desproporcionada a la amenaza real, y se
evitan o se resisten con elevada ansiedad. El cuadro dura 6
meses o más, genera malestar, y no se explica por organici-

51
dad ni por otro trastorno mental. das. A veces intensidad delirante.

ESPECIFICADORES ASPECTOS RELACIONADOS CON EL GÉNERO


Los miedos de actuación de los sujetos con trastorno de an- Mujeres: mayor número de miedos sociales y trastornos de-
siedad social “sólo de actuación” son normalmente los más presivos, bipolares y de ansiedad comórbidos.
incapacitantes en su vida profesional o en las situaciones que Hombres: miedo a citas, comorbilidad con trastorno nega-
requieren hablar en público habitualmente. tivista desafiante, trastorno de conducta y uso de alcohol y
drogas para aliviar síntomas.
CARACTERÍSTICAS QUE APOYAN EL DIAGNÓSTICO Paruresis: mayor prevalencia en hombres.
Pueden ser inadecuadamente asertivos, sumisos, ejercer
gran control sobre las conversaciones, adoptar posturas cor- CONSECUENCIAS FUNCIONALES
porales rígidas, tener contacto ocular inadecuado y/o voz Tasas elevadas de abandono escolar. Alteraciones del bien-
muy suave. Pueden ser tímidos o retraídos. Rubor. Pueden estar, empleo, productividad laboral, nivel socioeconómico,
buscar empleo en puestos que no requieran contacto social calidad de vida. Interferencia en actividades de ocio. Se aso-
(no en “solo actuación”), y vivir en casa familiar más tiempo. cia a estar solo, soltero o divorciado y no tener hijos (so-
Hombres: pueden demorarse a la hora de casarse y formar bre todo en hombres). En personas mayores, deterioro en
una familia. Mujeres: pueden acabar viviendo como amas de labores de cuidado y actividades de voluntariado. No tener
casa y madres. empleo predice la persistencia del trastorno.
Es frecuente la automedicación (ej. alcohol). En adultos ma-
yores puede haber una exacerbación de los síntomas de en- DIAGNÓSTICO DIFERENCIAL
fermedades médicas (temblor, taquicardia). - Timidez normal: no patológica.
- Agorafobia: miedo y evitación de situaciones socia-
PREVALENCIA les cuando perciben que podría ser difícil escapar o disponer
En Estados Unidos: 7%. En Europa: 2,3%. Disminuye con la de ayuda si experimentan síntomas de ansiedad (vs. miedo a
edad. En adultos mayores varía del 2 al 5 %. la evaluación negativa).
En población general, la prevalencia es mayor en mujeres - Trastorno de pánico: la preocupación se centra en
que en hombres (2,2:1,5). La diferencia de género es supe- las crisis de ansiedad.
rior en adolescentes y adultos jóvenes. En población clínica, - Trastorno de ansiedad generalizada: la atención se
la prevalencia en hombres es similar o ligeramente más alta centra más en la naturaleza de las relaciones existentes (vs.
que la de mujeres. miedo a la evaluación negativa).
- Trastorno de ansiedad por separación: pueden evi-
DESARROLLO Y CURSO tar entornos sociales porque les preocupa la separación de
Edad media de inicio: 13 años (el 75% tiene una edad en- las figuras de apego o porque necesitan su presencia. Si sus
tre 8-15 años). A veces aparece después de una historia de figuras de apego están presentes están cómodos en situacio-
inhibición social o timidez en la infancia. Inicio brusco o in- nes sociales.
sidioso. - Fobias específicas: pueden temer la vergüenza o hu-
Puede disminuir cuando el sujeto, que siente temor por citas millación, pero no la evaluación negativa.
con otras chicas, se casa; y puede volver a aparecer después - Mutismo selectivo: no sienten miedo a la evaluación
de divorciarse. negativa en situaciones sociales en las que no se requiere
Adolescentes: patrón más amplio de miedo y evitación. hablar.
Adultos jóvenes: más ansiedad social en situaciones especí- - Trastorno depresivo mayor: les puede preocupar
ficas. que les evalúen porque sienten que son malos o no son dig-
Adultos mayores: menos nivel de ansiedad social, pero en nos de ser queridos.
una amplia gama de situaciones. - Trastorno dismórfico corporal: preocupación por
En personas mayores, puede deberse a discapacidad por dé- uno o más defectos físicos no observables.
ficits sensoriales, vergüenza por el aspecto, afecciones médi- - Trastorno delirante: ideas delirantes (las personas
cas, incontinencia o deterioro cognitivo. con trastorno de ansiedad social son conscientes de que sus
30% remiten en 1 año. 50% remiten en unos pocos años. creencias son desproporcionadas).
60% (sin tratamiento específico), curso de varios años. - Trastorno del espectro autista: los sujetos con tras-
torno de ansiedad social pueden tener relaciones adecuadas
FACTORES DE RIESGO Y PRONÓSTICO para la edad y suficiente capacidad de comunicación social.
Temperamentales: inhibición del comportamiento, miedo a - Trastorno de personalidad evitativa: patrón de evita-
la evaluación negativa. ción más amplio.
Ambientales: maltrato infantil y adversidad. - Otros trastornos mentales: en esquizofrenia hay sín-
Genéticos y fisiológicos: inhibición conductual. El Trastor- tomas psicóticos; en trastornos de conducta alimentaria el
no de ansiedad social es hereditario (lo es menos el de tipo miedo a la evaluación negativa de los síntomas es la única
“sólo de actuación”). 2-6 veces más probabilidades en fami- forma de ansiedad social; en trastorno obsesivo-compulsivo
liares de primer grado (interacción entre factores genéticos no hay miedo social independiente de los focos de las obse-
específicos, como el miedo a la evaluación negativa, e ines- siones y compulsiones.
pecíficos, como neuroticismo). - Otras condiciones médicas: organicidad (síntomas
embarazosos).
ASPECTOS RELACIONADOS CON LA CULTURA - Trastorno negativista desafiante: negativa a hablar
Síndrome de Taijin kyofusho: preocupación por la evaluación por oposición hacia las figuras de autoridad.
social y miedo a hacer que otras personas se sientan incómo-

52
COMORBILIDAD sobre todo trastorno depresivo mayor).
Otros trastornos de ansiedad, depresivo mayor, por consu-
mo de sustancias (el trastorno de ansiedad social general- ASPECTOS RELACIONADOS CON LA CULTURA
mente precede a los anteriores, excepto a la fobia específica La expresión del trastorno varía en diferentes culturas: en
y el trastorno de ansiedad por separación), trastorno dismór- algunas predominan los síntomas somáticos y en otras los
fico corporal, bipolar, evitativo de la personalidad (la forma cognitivos. Importante tener en cuenta el contexto social y
más generalizada de trastorno de ansiedad social, no “sólo cultural al evaluar si las preocupaciones son o no excesivas.
actuación”).
En niños: autismo de alto funcionamiento y mutismo selec- ASPECTOS RELACIONADOS CON EL GÉNERO
tivo. Más frecuente en mujeres. En población clínica: 55-60% son
TRASTORNO DE ANSIEDAD GENERALIZADA mujeres. En población general: 2/3 son mujeres. Síntomas
similares, pero diferencias en comorbilidad:
CARACTERÍSTICAS DIAGNÓSTICAS - Mujeres: trastornos de ansiedad, depresión.
Ansiedad y preocupación (anticipación aprensiva) excesiva - Hombres: trastornos por uso de sustancias.
sobre diversos sucesos o actividades (el centro de las preocu-
paciones puede trasladarse de una situación a otra durante CONSECUENCIAS FUNCIONALES
el curso del trastorno), difícil de controlar, y que incluye 3/6 Dificultad para hacer las cosas de manera rápida y eficien-
síntomas. Dura al menos 6 meses, genera malestar, y no se te. Consumo de tiempo y energía. La preocupación excesiva
explica por organicidad ni por otro trastorno mental. puede afectar a la capacidad para fomentar la confianza en
Las preocupaciones del trastorno de ansiedad generalizada sus hijos. Discapacidad significativa (moderada-grave).
(vs. las preocupaciones de la ansiedad no patológica) son
excesivas, interfieren con el funcionamiento, son más pe- DIAGNÓSTICO DIFERENCIAL
netrantes y angustiosas, tienen mayor duración, se suelen - Trastorno de ansiedad debido a otra afección médi-
producir sin desencadenantes y se asocian con más síntomas ca: organicidad (p. ej., feocromocitoma, hipertiroidismo).
físicos. - Trastorno de ansiedad inducido por sustancias/me-
dicamentos: efecto fisiológico de una sustancia (p. ej., eleva-
CARACTERÍSTICAS QUE APOYAN EL DIAGNÓSTICO do consumo de café).
Tensión muscular (temblores, contracciones nerviosas, ines- - Trastorno de ansiedad social: ansiedad ante situa-
tabilidad, molestias musculares, dolor). Síntomas somáticos. ciones sociales o de evaluación por terceros (vs. trastorno
Respuesta de sobresalto exagerada. Afecciones relacionadas de ansiedad generalizada: preocupación estén o no siendo
con el estrés: síndrome de intestino irritable, dolores de ca- evaluados).
beza, etc. - Trastorno obsesivo-compulsivo: obsesiones como
pensamientos, impulsos o imágenes intrusivos no deseados.
PREVALENCIA - Trastorno de estrés postraumático y trastornos de
Adolescentes: 0,9%. Adultos: 2,9%. Riesgo de por vida: adaptación: en el trastorno de estrés postraumático, la an-
9,0%. Mayor prevalencia en mujeres que en hombres (2:1). siedad se explica como síntoma del trastorno; el trastorno
Mayor prevalencia en la edad media, disminuyendo en eda- adaptativo se diagnostica sólo cuando no se cumplen los cri-
des más avanzadas. Más prevalencia en países desarrollados. terios de otro trastorno.
DESARROLLO Y CURSO - Trastornos depresivos, bipolares, psicóticos: la preo-
La edad media de inicio es de 30 años. Es más tardía que la cupación sólo aparece durante el curso de esos trastornos.
de los otros trastornos de ansiedad.
Curso crónico y oscilante. Bajas tasas de remisión completa. COMORBILIDAD
Diferencias entre grupos de edad en el contenido de las pre- Otros trastornos de ansiedad y trastornos depresivos.
ocupaciones: Menor comorbilidad con trastornos por consumo de sustan-
- Niños y adolescentes: escuela, rendimiento deporti- cias, de conducta, psicóticos, del neurodesarrollo y neuro-
vo, puntualidad, sucesos catastróficos. cognitivos.
cognitivos.
- Adultos: bienestar de la familia, salud física propia.
- Ancianos: seguridad, caídas, enfermedad física cró-  
nica. TRASTORNO OBSESIVO-COMPULSI-
Inicio precoz: mayor comorbilidad y deterioro.
Podría sobrediagnosticarse en los niños (importante hacer un VO Y RELACIONADOS
buen diagnóstico diferencial con otros trastornos de ansie-
dad en la infancia y otros trastornos mentales, para descartar TRASTORNO OBSESIVO-COMPULSIVO
que las preocupaciones se puedan explicar mejor por alguno
de ellos). CARACTERÍSTICAS DIAGNÓSTICAS
Presencia de obsesiones y compulsiones. Las compulsiones
no se hacen por placer, aunque algunos individuos experi-
FACTORES DE RIESGO Y PRONÓSTICO mentan alivio. Ambas deben consumir mucho tiempo (más
Temperamentales: inhibición conductual, afectividad negati- de una hora al día) o provocar malestar. La frecuencia y gra-
va (neuroticismo), evitación del daño. vedad varía entre los individuos; se considera de 1 a 3 horas
Ambientales: adversidades en la infancia, sobreprotección para síntomas de leves a moderados y casi constantemente
de los padres. para síntomas incapacitantes).
Genéticos y fisiológicos: 1/3 del riesgo es genético (compar-
tido con otros trastornos de ansiedad y estado de ánimo, CARACTERÍSTICAS QUE APOYAN EL DIAGNÓSTICO

53
El contenido varía entre los individuos. Limpieza, simetría, re- COMORBILIDAD
petición, orden, pensamientos prohibidos o tabú y daño son Mayor comorbilidad con otro Trastorno de Ansiedad y Tras-
aspectos frecuentes. A menudo se tienen síntomas en más tornos Depresivos. Menor comorbilidad con Trastornos por
de una dimensión. Hay gran variedad de respuestas afectivas Consumo de Sustancias, Trastornos de Conducta, Trastornos
asociadas (ansiedad, ataques de pánico, disgusto, evitación Psicóticos, Trastornos del Neurodesarrollo y Trastornos Neu-
de lugares, personas y objetos). rocognitivos.

PREVALENCIA
1,2% en EEUU. 1,1-1,8% a nivel internacional. Las mujeres
están ligeramente más afectadas en la edad adulta; los varo- TRASTORNO DISMÓRFICO CORPORAL
nes más afectados en la infancia.
DESARROLLO Y CURSO CARACTERÍSTICAS DIAGNÓSTICAS
La edad media de inicio es de 19,5 años (un 25% antes de Preocupación por uno o más defectos percibidos en su apa-
los 14, raro después de los 35). Los varones presentan un ini- riencia física, la cual no es observable o solo levemente apre-
cio más temprano que las mujeres. El inicio suele ser gradual ciada por los demás. Consumen mucho tiempo (3-8 horas
y si no se trata suele ser crónico, con síntomas oscilantes. al día). Actos mentales excesivos y repetitivos en respuesta a
la preocupación (comprobaciones, comparaciones, excesiva
FACTORES DE RIESGO Y PRONÓSTICO preparación, rascado). La dismorfia muscular es una forma
Temperamentales: Síntomas de interiorización, emotividad de Trastorno Dismórfico Corporal casi exclusiva de varones
negativa, inhibición comportamental en infancia. (preocupación por cuerpo demasiado pequeño o insuficien-
Ambientales: Maltrato físico y sexual en infancia, sucesos temente magro o musculado).
traumáticos, agentes infecciosos.
Genéticos y fisiológicos: Tasa en familiares de primer grado CARACTERÍSTICAS QUE APOYAN EL DIAGNÓSTICO
dos veces mayor, también aumento más precoz. Disfunción Ideas delirantes de referencia, ansiedad, evitación social, de-
de corteza orbitofrontal, cingulada anterior y cuerpo estria- presión, neuroticismo y perfeccionismo, baja extroversión y
do. autoestima. Recibir tratamientos cosméticos y estéticos, ope-
raciones.
ASPECTOS RELACIONADOS CON LA CULTURA
Se diagnostica en todo el mundo, hay similitud en distribu- PREVALENCIA
ción por sexos, en edad de inicio y comorbilidad en todas las En EEUU 2,4% (2,5% en mujeres y 2,2% en varones). Fue-
culturas. Pueden existir variaciones regionales. ra de EEUU, 1,7-1,8% con prevalencia por género similar.
9-15% en pacientes de dermatología, 3-16% pacientes de
ASPECTOS RELACIONADOS CON EL GÉNERO cirugía estética, 8% de los ortodoncistas, 10% pacientes de
Los hombres tienen una edad más temprana de inicio y ma- cirugía oral o maxilofacial.
yor probabilidad de comorbilidad con tics. Las mujeres son
más propensas en la dimensión de limpieza y los varones en DESARROLLO Y CURSO
pensamientos prohibidos y simetría. Puede haber comienzo Edad media de inicio del trastorno 16-17 años; edad más
o exacerbación en periparto (síntomas en relación a la rela- frecuente de aparición de síntomas 12-13 años. Dos terceras
ción madre-bebé). partes tienen un inicio antes de los 18 años (más riesgo de
intentos de suicidio y más comorbilidad). Parece ser cróni-
DIAGNÓSTICO DIFERENCIAL co, aunque puede mejorar con tratamiento basado en datos
Trastornos de Ansiedad (preocupaciones de la vida real en científicos.
Trastorno de Ansiedad Generalizada, temores más circuns-
critos en la Fobia Específica o la Fobia Social); Trastorno De- FACTORES DE RIESGO Y PRONÓSTICO
presivo Mayor (pensamientos congruentes con el estado de Ambientales: Altas tasas de abandono y abuso infantil.
ánimo); otros Trastornos relacionados con el Obsesivo-Com- Genéticos y fisiológicos: Tasas altas en familiares de primer
pulsivo (preocupaciones limitadas al aspecto físico en Tras- grado de personas con Trastorno Obsesivo Compulsivo.
torno Dismórfico Corporal y comportamientos compulsivos
de arranque de pelo o rascado excesivo en Tricotilomanía o ASPECTOS RELACIONADOS CON LA CULTURA
trastorno de Excoriación); Trastornos de la Alimentación (no El Taijin kyofusho (del tradicional sistema diagnóstico japo-
limitado a peso y comida); Tics y movimientos estereotipados nés) tiene el subtipo shubo-kyofu (fobia a un cuerpo defor-
(no dirigidos a neutralizar obsesiones); Trastornos psicóticos mado) parecido al Trastorno Dismórfico Corporal.
(puede haber escasa conciencia de enfermedad e incluso
ser delirante pero no hay otros síntomas de esquizofrenia u ASPECTOS RELACIONADOS CON EL GÉNERO
otros); Trastorno de Personalidad Obsesivo-Compulsivo (no En los varones son más frecuentes las preocupaciones geni-
aparecen obsesiones ni compulsiones); otros comportamien- tales y las mujeres son más propensas a tener un Trastorno
tos impulsivos (parafilias, juego, sustancias, etc.). de la Conducta Alimentaria comórbido. La dismorfia muscu-
REPERCUSIONES FUNCIONALES lar es casi exclusiva en hombres.
Dificultad para hacer las cosas de manera rápida y eficien-
te. Consumo de tiempo y energía. La preocupación excesiva DIAGNÓSTICO DIFERENCIAL
puede afectar a la capacidad de los pacientes de fomentar Preocupaciones normales sobre la apariencia y defectos fí-
la confianza en sus hijos. Discapacidad significativa (mode- sicos claramente perceptibles (exceso o conductas compro-
rada-grave). batorias, deterioro); Trastornos de la Conducta Alimentaria
(preocupación por el peso; pueden ser comórbidos); Otros

54
Trastornos Obsesivo-Compulsivo y relacionados (solo centra-
dos en la apariencia); Trastorno de Ansiedad por Enfermar ASPECTOS RELACIONADOS CON LA CULTURA
(no preocupación por enfermar ni niveles altos de somati- Parece ser un fenómeno universal, aunque la mayoría de los
zación); Trastorno Depresivo Mayor (síntomas depresivos estudios son con población occidental.
secundarios); Trastornos de Ansiedad (ansiedad social y
evitación son comunes); Trastornos Psicóticos (puede haber ASPECTOS RELACIONADOS CON EL GÉNERO
creencias delirantes, pero no se diagnostica Trastorno De- Las características generales son comparables entre varones
lirante si no que se usa el especificador); otros trastornos y mujeres, pero las mujeres muestran más una adquisición
y síntomas (características sexuales en malestar de género, excesiva (en particular, compra).
olor fétido o repugnante -síndrome de referencia olfativo).
DIAGNÓSTICO DIFERENCIAL
REPERCUSIONES FUNCIONALES Otras afecciones médicas (lesiones cerebrales, tumores, in-
Tasas de ideación suicida altas en adultos y niños y adoles- fecciones); Trastornos del Neurodesarrollo (espectro autista
centes. Afectación de funcionamiento psicosocial, que varía o discapacidad intelectual); Trastornos del Espectro de la Es-
en el grado de incapacitación, sobre todo en relación a la quizofrenia y otros Trastornos Psicóticos (consecuencia de
evitación. delirios o síntomas negativos); Episodio Depresivo Mayor (re-
traso psicomotor, fatiga o anergia); Trastorno Obsesivo-Com-
COMORBILIDAD pulsivo (consecuencia de las obsesiones o compulsiones);
El Trastorno Depresivo Mayor es el trastorno comórbido más Trastornos Neurocognitivos (enfermedades degenerativas).
común, normalmente inicia después del Trastorno Dismórfi-
co Corporal. También son habituales el Trastorno de Ansie- REPERCUSIONES FUNCIONALES
dad Social, el Trastorno Obsesivo Compulsivo y los Trastor- El desorden afecta a las actividades básicas de casa, incluso
nos Relacionados con Sustancias. al sueño. La calidad de vida se deteriora considerablemente.
Aumenta el riesgo de incendio y caídas. Son comunes los
conflictos con vecinos y autoridades.
TRASTORNO DE ACUMULACIÓN
COMORBILIDAD
CARACTERÍSTICAS DIAGNÓSTICAS Un 75% tiene un Trastorno del Estado de Ánimo o de Ansie-
Dificultad persistente para deshacerse o separarse de las po- dad comórbido. Un 20% también puede cumplir criterios de
sesiones, independiente de su valor real. Experimentan an- Trastorno Obsesivo-Compulsivo. A menudo las comorbilida-
gustia cuando se enfrentan a la opción de desecharlas. Los des son el principal motivo de consulta.
artículos abarrotan y desordenan las zonas de vida activa im-
posibilitándolas para su uso o si están disponibles es gracias
a la acción de un tercero. Los síntomas causan malestar y/o
deterioro. TRICOTILIMANÍA (TRASTORNO DE ARRAN-
CARSE EL PELO)
CARACTERÍSTICAS QUE APOYAN EL DIAGNÓSTICO
Indecisión, perfeccionismo, evitación, dilación, dificultad en CARACTERÍSTICAS DIAGNÓSTICAS
la planificación, organización y pérdida de atención. La acu- Acto recurrente de arrancarse el propio pelo, en cualquier
mulación de animales es una forma especial de trastorno de región (frecuentemente cuero cabelludo, cejas y párpados;
acumulación, donde hay mayor falta de conciencia de en- menos frecuentes el vello axilar, facial, púbico y regiones pe-
fermedad. rirrectales). Los sitios pueden variar con el tiempo. El acto de
arrancarse pelo tiene que dar lugar a una pérdida de este; ha
PREVALENCIA habido intentos para disminuir el acto. Malestar o deterioro.
No se dispone de estudios, se estima aproximadamente en-
tre el 2 y el 6%. Algunos estudios concluyen que es signifi- CARACTERÍSTICAS QUE APOYAN EL DIAGNÓSTICO
cativamente mayor en los varones en población general, lo Pueden aparecer rituales en torno al arranque. Puede ir pre-
que contrasta con las muestras clínicas, predominantemente cedido de distintos estados emocionales (ansiedad, aburri-
formadas por mujeres. Síntomas casi tres veces más frecuen- miento, tensión). Suele haber tensión previa y alivio poste-
te en adultos mayores (55-94 años) que en jóvenes. rior, aunque puede ser un acto más automático. No suele
producirse en presencia de otras personas.
DESARROLLO Y CURSO
Parece comenzar en etapas tempranas y se extiende hasta las PREVALENCIA
últimas etapas. Los primeros síntomas aparecen a los 11-15 En población general, 1-2%.
años, interfieren a los 20 y el deterioro se hace clínicamente
significativo a mediados de los 30. El curso es habitualmente DESARROLLO Y CURSO
crónico. El comienzo normalmente coincide con la pubertad. Normal-
FACTORES DE RIESGO Y PRONÓSTICO mente curso crónico y fluctuante. Frecuente empeoramiento
Temperamentales: Indecisión. en mujeres acompañando a los cambios hormonales.
Ambientales: Acontecimientos vitales estresantes traumáti-
cos. FACTORES DE RIESGO Y PRONÓSTICO
Genéticos y fisiológicos: El 50% de los individuos que acu- Genéticos y fisiológicos: Prueba de vulnerabilidad genética.
mulan informan de que tiene un familiar que también lo Más frecuente en individuos con Trastorno Obsesivo-Com-
hace.

55
pulsivo y en sus familiares de primer grado.
DIAGNÓSTICO DIFERENCIAL
ASPECTOS RELACIONADOS CON LA CULTURA Trastornos Psicóticos (delirio de parasitosis, hormigueos);
Puede ser similar en todas las culturas, aunque los datos son otros Trastornos Obsesivo-Compulsivos y relacionados (com-
de regiones occidentales. pulsiones de lavado); Trastornos del Neurodesarrollo (movi-
mientos estereotipados); Trastornos de Síntomas Somáticos y
ASPECTOS RELACIONADOS CON EL GÉNERO relacionados (conductas fraudulentas del trastorno facticio);
10 mujeres por cada hombre. Puede reflejar diferencias en otros trastornos y afecciones médicas (sarna, acné); Trastor-
la búsqueda de tratamiento según el género. En los niños, nos inducidos por sustancias/medicamentos (cocaína).
varones y mujeres están representados por igual.
REPERCUSIONES FUNCIONALES
DIAGNÓSTICO DIFERENCIAL Angustia, discapacidad social y ocupacional. Al menos 1
Depilación o manipulación normal (razones estéticas); otros hora rascándose, pensándolo o resistiéndolo. Evitación.
Trastornos Obsesivo-Compulsivos y relacionados (rituales Complicaciones médicas (daño de tejidos, cicatrización, in-
de simetría); Trastornos del Neurodesarrollo (estereotipias); fección, cirugía).
Trastornos Psicóticos (como reacción ante un delirio o aluci-
nación); otra afección médica (inflamación u otras afeccio- COMORBILIDAD
nes dermatológicas); Trastornos relacionados con Sustancias Trastorno Obsesivo-Compulsivo y Tricotilomanía, Trastorno
(estimulantes). Depresivo Mayor, Trastorno de conductas repetitivas centra-
das en el cuerpo.
REPERCUSIONES FUNCIONALES
Asociada a angustia, discapacidad social y ocupacional. Con-  
secuencias médicas infrecuentes. La ingestión del pelo (tri-
cofagia) puede llevar a dolores, anemia, náuseas, vómitos,
obstrucción intestinal… TRASTORNOS RELACIONADOS CON
COMORBILIDAD TRAUMAS Y FACTORES DE ESTRÉS
Suele ir acompañada de otros trastornos mentales, general-
mente Trastorno Depresivo Mayor y Trastorno de Excoria-
ción.
TRASTORNO DE APEGO REACTIVO

TRASTORNO DE EXCORIACIÓN (RASCARSE LA CARACTERÍSTICAS DIAGNÓSTICAS


Ausencia de apego o apego muy poco desarrollado entre
PIEL) el niño y sus presuntos cuidadores adultos. Pueden tener la
capacidad de crear apegos selectivos, aunque debido a las
CARACTERÍSTICAS DIAGNÓSTICAS pocas oportunidades no los creen. Ausencia de búsqueda
Rascado frecuente de la propia piel (normalmente en cara, del consuelo que sería de esperar y falta de respuesta a las
brazos, manos). Además del rascado puede haber frota- conductas de consuelo. Está comprometida su regulación
miento, prensión, punción y mordedura. El rascado tiene emocional, tienen emociones negativas de miedo, tristeza
que conducir a lesiones en la piel, aunque intenten ocultar- o irritabilidad. No se puede hacer el diagnóstico hasta los 9
lo, e intentos de disminuir el rascado. Malestar o deterioro. meses de vida.

CARACTERÍSTICAS QUE APOYAN EL DIAGNÓSTICO CARACTERÍSTICAS QUE APOYAN EL DIAGNÓSTICO


Puede involucrar la piel o las costras, rituales, diferentes esta- Relación etiológica compartida con el abandono social, por
dos emocionales (ansiedad, aburrimiento, tensión, satisfac- lo que a menudo concurre con retrasos en el desarrollo (es-
ción o placer). El dolor no suele acompañar al rascado. Puede pecialmente en cognición y lenguaje). Estereotipias y otros
haber tensión anterior y alivio posterior o ser automático. signos de negligencia grave (por ejemplo, desnutrición).

PREVALENCIA PREVALENCIA
En población general, 1,4%. Desconocida. Se observa con poca frecuencia en la clínica. Sí
se ha observado en niños expuestos a negligencia, en hoga-
DESARROLLO Y CURSO res de acogida e instituciones, aunque aún en estas pobla-
Inicio a menudo en adolescencia, coincidiendo con el inicio ciones es poco frecuente (menos del 10%).
de la pubertad. Comienza con frecuencia con el acné. Los DESARROLLO Y CURSO
sitios de rascado pueden variar. El curso es habitualmente Las características se manifiestan de manera similar entre
crónico y fluctuante. los 9 meses y los 5 años sin tratamiento y sin recuperación
FACTORES DE RIESGO Y PRONÓSTICO de entornos de cuidado el trastorno persiste durante varios
Genéticos y fisiológicos: Más frecuente en individuos con años.
Trastorno Obsesivo-Compulsivo y familiares de primer grado.
FACTORES DE RIESGO Y PRONÓSTICO
ASPECTOS RELACIONADOS CON EL GÉNERO Ambientales: La negligencia social grave es un requisito para
Tres cuartas partes son mujeres, aunque este dato puede el diagnóstico, aunque la mayoría de los niños gravemente
reflejar diferencias en búsqueda de tratamiento. descuidados no desarrolla la enfermedad. El pronóstico de-

56
pende de la calidad del ambiente tras la negligencia.
DIAGNÓSTICO DIFERENCIAL
ASPECTOS RELACIONADOS CON LA CULTURA Trastorno por Déficit de Atención/Hiperactividad (impulsivi-
Se debe tener precaución al realizar este diagnóstico en las dad social).
culturas en las que no se ha estudiado el apego.
REPERCUSIONES FUNCIONALES
DIAGNÓSTICO DIFERENCIAL Dificulta la capacidad de los niños pequeños para relacio-
Trastorno del Espectro Autista (historia de abandono social, narse de forma interpersonal con los adultos y compañeros.
intereses restringidos y comportamientos repetitivos); Dis-
capacidad Intelectual (habilidades sociales y emocionales COMORBILIDAD
comparables a sus habilidades cognitivas, sin reducción del Las investigaciones son limitadas. Se asocian los retrasos
afecto positivo); Trastornos Depresivos (deberían buscar y cognitivos, retrasos en el lenguaje, estereotipias y Trastorno
responder a intentos de consuelo). por Déficit de Atención/Hiperactividad.

REPERCUSIONES FUNCIONALES
Dificultades interpersonales con adultos o compañeros, de- TRASTORNO DE ESTRÉS POSTRAUMÁTICO
terioro funcional en múltiples dominios durante la primera
infancia. CARACTERÍSTICAS DIAGNÓSTICAS
Desarrollo de síntomas específicos tras la exposición a uno
COMORBILIDAD o más eventos traumáticos. Las reacciones emocionales al
Retrasos cognitivos, del lenguaje, estereotipias, desnutrición evento traumático (miedo, impotencia, horror) ya no forman
severa, síntomas depresivos. parte del criterio A. Se definen síntomas de Reexperimenta-
ción (criterio B), evitación (criterio C), alteraciones cognitivas
negativas o en los estados de ánimo, incluyendo amnesia
disociativa (criterio D), irritabilidad, sensibilidad, reactividad
TRASTORNO DE LA RELACIÓN SOCIAL DESIN- (criterio E), despersonalización o desrealización.
HIBIDA
CARACTERÍSTICAS QUE APOYAN EL DIAGNÓSTICO
CARACTERÍSTICAS DIAGNÓSTICAS Regresión en el desarrollo como pérdida de lenguaje en los
Comportamiento culturalmente inapropiado, demasiado fa- niños pequeños, pseudoalucinaciones auditivas, ideación pa-
miliar con personas relativamente extrañas, traspasando los ranoide. En sucesos traumáticos prolongados puede haber
límites sociales de la cultura. No puede hacerse el diagnósti- problemas para regular las emociones o mantener relaciones
co hasta los 9 meses de vida. personales estables. Puede presentarse duelo complicado o
síntomas de Trastorno de Estrés Postraumático si hay muer-
CARACTERÍSTICAS QUE APOYAN EL DIAGNÓSTICO tes violentas.
Puede coexistir con retrasos de desarrollo (sobre todo cog-
nitivos y del lenguaje), estereotipias y otras señales de ne- PREVALENCIA
gligencia grave (por ejemplo, desnutrición). Puede verse en En EEUU la prevalencia es de 3,5%, estimándose más baja
niños sin apego normal o con un apego que oscile entre en Europa y otros continentes, en torno al 0,5-1%. Los índi-
seguro y alterado. ces más altos entre veteranos y otras personas con profesión
de riesgo (policías, bomberos, médicos de emergencia), su-
PREVALENCIA pervivientes de violación, combate militar, cautiverio e inter-
Desconocida, parece ser poco frecuente. En poblaciones de namiento y genocidio. Los niños y adolescentes generalmen-
alto riesgo (instituciones, hogares de acogida) solo aparece te muestran una prevalencia menor, al igual que personas
en alrededor del 20%. Rara vez se observa en otros contex- mayores presentan tasas más bajas.
tos clínicos.
DESARROLLO Y CURSO
DESARROLLO Y CURSO Puede ocurrir a cualquier edad. Los síntomas generalmente
No hay pruebas de que el abandono después de los 2 años comienzan dentro de los 3 primeros meses después del trau-
se asocie con manifestaciones del trastorno. En la adolescen- ma, aunque puede haber retraso incluso de años (“expresión
cia se hacen más evidentes el comportamiento desinhibido retrasada”). Hay una recuperación completa a los 3 meses
y los conflictos. El trastorno no se ha descrito en los adultos. en la mitad de los adultos, mientras que otras personas per-
manecen sintomáticas durante más de 12 meses y a veces
FACTORES DE RIESGO Y PRONÓSTICO más de 50 años.
Ambientales: La negligencia social grave es un requisito
diagnóstico para el trastorno, aunque la mayoría de los niños FACTORES DE RIESGO Y PRONÓSTICO
no desarrolle la enfermedad. No se ha identificado el tras- 1. Pretraumáticos:
torno en los niños abandonados después de los dos años. Temperamentales: Problemas emocionales en la infancia y
El pronóstico de la calidad del entorno cuidador posterior al trastornos mentales previos.
abandono. Ambientales: Bajo nivel socio económico, educación escasa,
Modificadores del curso: la calidad del cuidado puede mo- exposición a trauma previo, adversidad en la niñez, carac-
derar el curso, aunque algunos niños pueden mostrar signos terísticas culturales (estrategias de afrontamiento), deficien-
persistentes al menos hasta la adolescencia. cia de inteligencia, condiciones raciales minoritarias, historia
psiquiátrica familiar. El apoyo social antes de la exposición al

57
suceso es protector. mujeres. En niños predomina el Trastorno Negativista Desa-
Genéticos y fisiológicos: Sexo femenino y edad más joven en fiante y la Ansiedad por Separación. Existe una comorbilidad
el momento del trauma. considerable entre el Trastorno de Estrés Postraumático y un
2. Peritraumáticos: Trastorno Neurocognitivo importante.
Ambientales: Gravedad (dosis) del trauma (a mayor magni-
tud, más probabilidad de Trastorno de Estrés Postraumático),
peligro vital percibido, lesiones personales, violencia inter- TRASTORNO DE ESTRÉS AGUDO
personal (especialmente trauma perpetrado por un cuidador
o al presenciar cómo se amenaza a un cuidador). En militares CARACTERÍSTICAS DIAGNÓSTICAS
el ser autor o testigo de atrocidades o matar al enemigo. La Desarrollo de síntomas característicos con una duración de
disociación que se produce durante el trauma y persiste des- 3 días a 1 mes tras la exposición a uno o más eventos trau-
pués es factor de riesgo. máticos. Las presentaciones clínicas varían según la persona,
3. Postraumáticos: pero incluyen reexperimentaciones, disociaciones, respues-
Temperamentales: Evaluaciones negativas, inadecuadas es- tas reactivas de ira o irritabilidad, incapacidad persistente
trategias de afrontamiento. para sentir emociones positivas, despersonalización o des-
Ambientales: Posterior exposición a recuerdos repetitivos, realización, amnesia disociativa, evitación La probabilidad
pérdidas relacionadas, acontecimientos adversos posterio- de desarrollar este trastorno puede aumentar a medida que
res. El apoyo social es un factor protector que modera el aumente la intensidad del estresor y la proximidad física.
resultado después de un trauma.
CARACTERÍSTICAS QUE APOYAN EL DIAGNÓSTICO
ASPECTOS RELACIONADOS CON LA CULTURA Pensamientos catastróficos o muy negativos acerca de su
El riesgo varía en función de los distintos grupos culturales papel en el evento traumática o la probabilidad de un daño
(p.ej. genocidios, estrés de aculturación entre los inmigran- futuro (culpa), comportamientos caóticos o impulsivos.
tes). La evaluación completa de las expresiones del Trastorno
de Estrés Postraumático en cada zona debe incluir una valo- PREVALENCIA
ración de la diversidad cultural de los conceptos de peligro. Varía según la naturaleza del evento y el contexto. Menos
del 20% de los casos después de sucesos traumáticos sin
ASPECTOS RELACIONADOS CON EL GÉNERO asalto interpersonal, entre 13-21% de los accidentes de trá-
Más frecuente en mujeres durante toda la vida, también ma- fico, 14% de las lesiones cerebrales traumáticas leves, 19%
yor duración. Se asocia a mayor probabilidad de exposición de los asaltos, 10% quemaduras graves, entre 6-12% de
a eventos traumáticos (violación, violencia interpersonal). En los accidentes industriales. Tasas más altas (entre 20-50%)
poblaciones expuestas específicamente a estos tipos de es- tras eventos traumáticos interpersonales (asalto, violación,
trés las diferencias de género no son significativas. tiroteo masivo)

DIAGNÓSTICO DIFERENCIAL DESARROLLO Y CURSO


Trastornos de adaptación (factor estresante de cualquier gra- No se puede diagnosticar hasta 3 días después del evento.
vedad); otros trastornos y afecciones traumáticas (se pueden Puede evolucionar a Trastorno de Estrés Postraumático des-
cumplir criterios de otro trastorno mental además de Tras- pués de un mes o puede remitir. La mitad de las personas
torno de Estrés Postraumático); Trastorno de Estrés Agudo que finalmente desarrollan Trastorno de Estrés Postraumáti-
(duración entre 3 días y 1 mes); Trastornos de Ansiedad y co presentan inicialmente Trastorno por Estrés Agudo.
Trastorno Obsesivo-Compulsivo (síntomas obsesivos o de
ansiedad no asociados a un evento traumático); Trastorno FACTORES DE RIESGO Y PRONÓSTICO
Depresivo Mayor (puede diagnosticarse aparte); Trastornos Temperamentales: Trastorno mental previo, altos niveles de
de la Personalidad (manifestaciones independientes de la afecto negativo (neuroticismo), mayor gravedad percibida
exposición traumática); Trastornos Disociativos (manifesta- del evento, estilo de afrontamiento evitativo.
ciones del subtipo Trastorno de Estrés Postraumático “con Ambientales: Historia de trauma anterior.
síntomas disociativos”); Trastorno de Síntomas Neurológicos Genéticos y fisiológicos: Las mujeres tienen mayor riesgo. La
Funcionales; Trastornos Psicóticos (distinguir flashbacks de reactividad elevada antes del trauma (se refleja en la respues-
alucinaciones o ilusiones); lesión cerebral traumática (sínto- ta de sobresalto acústico) aumenta el riesgo.
mas posteriores a una conmoción).
ASPECTOS RELACIONADOS CON LA CULTURA
REPERCUSIONES FUNCIONALES El perfil de los síntomas varía en las diferentes culturas, en
El trauma aumenta el riesgo de suicidio (por ejemplo, abu- particular con respecto a los síntomas disociativos (por ejem-
so infantil). Altos niveles de discapacidad social, ocupacional plo, posesión o trance), pesadillas, evitación y síntomas psi-
y física, costes económicos considerables y altos niveles de cosomáticos.
utilización de los servicios médicos. Relaciones sociales y fa-
miliares pobres, absentismo laboral, menos ingresos y menor ASPECTOS RELACIONADOS CON EL GÉNERO
nivel educativo y de éxito laboral. Más frecuente entre las mujeres; las diferencias neurobioló-
COMORBILIDAD gicas ligadas al sexo en la respuesta de estrés pueden con-
Son un 80% más propensos a presentar síntomas que cum- tribuir a ello. Mayor probabilidad de exposición a eventos
plen criterios para al menos otro trastorno mental (Depre- traumáticos como violación y otras formas de violencia inter-
sión, Ansiedad, Consumo de Sustancias, Trastorno Bipolar). personal en mujeres.
Los Trastornos por Uso de Sustancias y los Trastornos de
Conducta son más comunes entre los varones que entre las DIAGNÓSTICO DIFERENCIAL

58
Trastornos de Adaptación (factor estresante de cualquier
gravedad); Trastorno de Pánico (solo si inesperados y anti- COMORBILIDAD
cipación futura); Trastornos Disociativos (si persiste la amne- Se puede diagnosticar Trastorno de Adaptación además de
sia); Trastorno de Estrés Postraumático (síntomas después del otro trastorno mental solo si este último no explica los sínto-
primer mes); Trastorno Obsesivo-Compulsivo (obsesiones, mas particulares que se producen al reaccionar ante el factor
no pensamientos intrusivos); Trastornos Psicóticos (distinguir estresante. Frecuentemente acompañan a las enfermedades
flashbacks de alucinaciones o ilusiones); lesión cerebral trau- médicas.
mática (síntomas posteriores a una conmoción).
REPERCUSIONES FUNCIONALES
Deterioro del funcionamiento social, interpersonal y laboral. TRASTORNOS DE SÍNTOMAS SOMÁ-
Elevados niveles de ansiedad que interfieren en el sueño, ni-
vel de energía y capacidad de concentración. La evitación
TICOS Y TRASTORNOS RELACIONA-
puede originar falta de asistencia a las citas médicas, evita- DOS
ción de la conducción o absentismo laboral.
TRASTORNO DE SÍNTOMAS SOMÁTICOS

TRASTORNO DE ADAPTACIÓN CARACTERÍSTICAS DIAGNÓSTICAS


Los síntomas pueden estar o no asociados con otra afección
CARACTERÍSTICAS DIAGNÓSTICAS médica. Los diagnósticos de trastorno de síntomas somáticos
Presencia de síntomas emocionales o comportamentales en y de una enfermedad médica conjunta no son mutuamente
respuesta a un factor de estrés identificable. Puede diagnos- excluyentes, y con frecuencia se presentan juntos. Por ejem-
ticarse tras la muerte de un ser querido si la intensidad, cali- plo, un individuo podría estar gravemente discapacitado por
dad o persistencia de las reacciones de duelo superan lo que los síntomas de un trastorno de síntomas somáticos después
normalmente cabría esperar. de un infarto no complicado de miocardio incluso aunque el
propio infarto de miocardio no haya producido ninguna dis-
PREVALENCIA capacidad. Si existe otra afección médica o un riesgo elevado
Son trastornos comunes. La prevalencia varía en función de de desarrollarla (p. ej., una fuerte historia familiar asociada)
la población estudiada. 5-20% de los individuos en trata- los pensamientos, sentimientos y comportamientos asocia-
miento ambulatorio de salud mental. Alcanza el 50% en en- dos con esta afección son excesivos (Criterio B).
torno de consulta psiquiátrica hospitalaria (diagnóstico más
común). CARACTERÍSTICAS QUE APOYAN EL DIAGNÓSTICO
Síntomas específicos o inespecíficos. Atención centrada en
DESARROLLO Y CURSO síntomas, revisión corporal, búsqueda de atención médica
Por definición, comienza dentro de los 3 meses de inicio de (sobre todo en casos graves y persistentes). Suelen rechazar
un factor de estrés y no dura más de 6 meses después de derivación a salud mental. Riesgo de suicidio, si depresión
haber cesado el agente estresante o sus consecuencias. comórbida. Puede haber concurrencia con enfermedades
médicas (importancia del criterio B).
FACTORES DE RIESGO Y PRONÓSTICO
Medioambientales: Entornos de vida desfavorecido, factores PREVALENCIA
estresantes. Desconocida. Se estima mayor que la del trastorno de soma-
tización (<1%) pero menor que la del trastorno somatomor-
ASPECTOS RELACIONADOS CON LA CULTURA fo indiferenciado (19%). En población general podría estar
La naturaleza, el significado y la experiencia de los factores entre 5-7%.
estresantes, así como la evaluación de la respuesta a dichos
factores puede variar de una cultura a otra. DESARROLLO Y CURSO
Puede estar infradiagnosticado en adultos mayores. El tras-
DIAGNÓSTICO DIFERENCIAL torno depresivo concomitante es común en las personas ma-
Trastorno Depresivo Mayor (perfil de los síntomas); Trastorno yores que se presentan con numerosos síntomas somáticos.
de Estrés Postraumático y Trastorno de Estrés Agudo (factor En niños, los síntomas más comunes son dolor abdominal
estresante de cualquier gravedad); Trastornos de la Perso- recurrente, el dolor de cabeza, la fatiga y las náuseas. Es más
nalidad (funcionamiento de la personalidad a lo largo de la frecuente que predomine un solo síntoma en los niños que
vida); factores psicológicos que influyen en otras condicio- en los adultos. Pese a que los niños pequeños pueden tener
nes médicas (entidades psicológicas específicas que pueden quejas somáticas, es raro que se preocupen por la “enferme-
agravar una afección médica); reacciones normales de estrés dad” per se antes de la adolescencia.
(si magnitud de la angustia es superior a lo que cabría espe-
rar o hay deterioro). FACTORES DE RIESGO Y PRONÓSTICO
Temperamentales: el rasgo de personalidad de afectividad
REPERCUSIONES FUNCIONALES negativa (neuroticismo) es un factor de riesgo. La comorbili-
Disminución del rendimiento en el trabajo o escuela, cam- dad con ansiedad o depresión es frecuente y puede agravar
bios temporales en las relaciones sociales. El curso de las en- los síntomas y el deterioro.
fermedades médicas puede verse complicado con la presen- Ambientales: el trastorno es más frecuente en los individuos
cia de un trastorno de adaptación (por ejemplo, disminución con pocos años de educación y bajo nivel socioeconómico, y
del cumplimiento terapéutico, aumento de la duración de en aquellos que han experimentado recientemente eventos
un ingreso). estresantes en sus vidas.

59
Modificación del curso: los síntomas somáticos persistentes Parece ser generalmente una afección crónica y recurrente
se asocian con determinadas características demográficas con una edad de inicio en la edad adulta media temprana.
(sexo femenino, más edad, menos años de educación, ni- En las muestras poblacionales, la ansiedad relacionada con
vel socioeconómico bajo, desempleo), con los antecedentes la salud aumenta con la edad, pero la edad de los indivi-
de abuso sexual u otra adversidad en la niñez, con una en- duos con ansiedad importante sobre la salud en los entornos
fermedad física crónica concomitante o trastornos psiquiá- médicos no parece diferir de la de otros pacientes en esos
tricos, con el estrés social y con factores de reforzamiento ámbitos. En las personas mayores, la ansiedad relacionada
social, tales como los beneficios de la enfermedad. con la salud a menudo se centra en la pérdida de memoria;
ASPECTOS RELACIONADOS CON LA CULTURA el trastorno parece poco frecuente en los niños.
El “agotamiento”, la sensación de pesadez o las quejas de
“gases”, un exceso de calor en el cuerpo o el ardor en la ca- FACTORES DE RIESGO Y PRONÓSTICO
beza son ejemplos de síntomas que son comunes en algunas Ambientales: puede estar precipitado por un estrés vital im-
culturas o grupos étnicos, pero infrecuentes en otras. Los portante o una amenaza seria, pero en última instancia, be-
modelos explicativos también varían y los síntomas somáti- nigna para la salud del individuo. Los antecedentes de abuso
cos se pueden atribuir a diversas causas, como el estrés per- en la infancia o de una enfermedad grave en la niñez pueden
sonal, familiar, laboral o ambiental, una enfermedad médica predisponer a desarrollar la enfermedad en la edad adulta.
general, la represión de los sentimientos de ira y resentimien-
to, o ciertos fenómenos específicos de la cultura, como la ASPECTOS RELACIONADOS CON LA CULTURA
pérdida de semen. El diagnóstico se debería hacer con precaución en los indivi-
duos cuyas ideas sobre la enfermedad son congruentes con
ASPECTOS RELACIONADOS CON EL GÉNERO unas creencias culturalmente aceptadas y muy extendidas.
Las mujeres informan en mayor medida de síntomas físicos No se sabe mucho sobre la fenomenología de la enfermedad
en general. en todas las culturas, aunque la prevalencia parece ser simi-
lar entre los diversos países con culturas diferentes.
CONSECUENCIAS FUNCIONALES
El trastorno se asocia con un deterioro importante del estado CONSECUENCIAS FUNCIONALES
de salud. Es probable que muchos individuos con trastorno El trastorno de ansiedad por enfermedad causa un deterioro
de síntomas somáticos graves tengan un deterioro de más importante en la función física y en la calidad de vida relacio-
de 2 desviaciones estándar por debajo de la población nor- nada con la salud. Las preocupaciones sobre la salud a me-
mal en la clasificación del estado de salud. nudo interfieren en las relaciones interpersonales, perturban
la vida familiar y alteran el desempeño ocupacional.
COMORBILIDAD
Trastornos médicos, trastornos de ansiedad y depresión. COMORBILIDAD
Desconocida (por ser trastorno nuevo). La hipocondría con-
curre con los trastornos de ansiedad y depresivos. Es proba-
ble que, aproximadamente, dos tercios de los individuos con
trastorno de la ansiedad por enfermedad tengan al menos
TRASTORNO DE ANSIEDAD POR ENFERME- otro trastorno mental mayor comórbido. Las personas con
DAD trastorno de ansiedad por enfermedad pueden tener un ries-
go elevado para el trastorno de síntomas somáticos y para
CARACTERÍSTICAS DIAGNÓSTICAS los trastornos de la personalidad.
La mayoría de las personas con hipocondría ahora son clasifi-
cadas como un trastorno de síntomas somáticos; sin embar-
go, en una minoría de los casos será más apropiado utilizar
en su lugar el diagnóstico de trastorno de ansiedad por en-
fermedad. El trastorno de ansiedad por enfermedad implica TRASTORNO DE CONVERSIÓN (TRASTORNO
una preocupación por tener o adquirir una enfermedad mé- DE SÍNTOMAS NEUROLÓGICOS FUNCIONA-
dica grave no diagnosticada (Criterio A). LES)
No aparecen síntomas somáticos o, si lo hacen, sólo son de
intensidad leve (Criterio B). CARACTERÍSTICAS DIAGNÓSTICAS
Nombres alternativos (“funcional”, “psicógeno”). Inicio en
CARACTERÍSTICAS QUE APOYAN EL DIAGNÓSTICO posible relación con factores de estrés. Presencia de sínto-
25% de los pacientes antes dx de hipocondría. Reciben mas disociativos sobre todo al inicio de los ataques conver-
amplia (e insatisfactoria) atención médica. Mayor tasa de sivos. Belle indifférence asociada (no es criterio diagnóstico).
asistencia médica que la población general. Algunos muy
ansiosos evitan la asistencia. Paradójica exacerbación de la CARACTERÍSTICAS ASOCIADAS QUE APOYAN EL DIAG-
ansiedad tras las distintas atenciones. NÓSTICO
El inicio puede estar asociado con el estrés o el trauma, ya
PREVALENCIA sea de naturaleza psicológica o física. La posible relevancia
Es similar en hombre y mujeres. etiológica de este estrés o trauma se puede sugerir por la
Prevalencia a 12-24 meses en PG: 1,3-10%. En población estrecha relación temporal. Sin embargo, aunque es impor-
ambulatoria, prevalencia a 6-12 meses: 3-8%. tante realizar la evaluación para el estrés y el trauma, su au-
sencia no debe excluir el diagnóstico.
DESARROLLO Y CURSO

60
DESARROLLO Y CURSO
Inicio en cualquier momento de la vida. Convulsiones psicó- TRASTORNOS FACTICIOS
genas: máximo pico en tercera década. Síntomas conversivos
motores: máximo pico en cuarta década. Los síntomas pue-
den ser transitorios o persistentes.
TRASTORNO FACTICIO
Mejor pronóstico en niños más pequeños que en adolescen-
CARACTERÍSTICAS DIAGNÓSTICAS
tes y adultos.
La característica esencial del trastorno facticio es la simula-
FACTORES DE RIESGO Y PRONÓSTICO
ción de signos y síntomas médicos o psicológicos, en uno
Rasgos de personalidad desadaptativos, antecedentes de
mismo o en otros, asociada a un engaño identificado. Los
abuso/negligencia en infancia, eventos vitales estresantes,
individuos con trastorno facticio también pueden buscar
enfermedad neurológica que cause síntomas similares (con-
tratamiento para sí mismos o para otros después de pro-
vulsiones conversivas más frecuentes en pacientes que tam-
vocar la lesión o la enfermedad. El diagnóstico requiere la
bién tienen epilepsia).
demostración de que el individuo está cometiendo acciones
La corta duración de los síntomas y la aceptación del diag-
subrepticias para tergiversar, simular o causar signos o sínto-
nóstico son factores pronósticos positivos.
mas de enfermedad o de lesión en ausencia de recompensas
Los rasgos de personalidad desadaptativos, la presencia de
externas obvias.
comorbilidad física y la recepción de beneficios por la incapa-
CARACTERÍSTICAS QUE APOYAN EL DIAGNÓSTICO
cidad pueden ser factores pronósticos negativos.
Aplicado a otro: el diagnóstico se aplica a autor, no a la víc-
tima. Para hacer el diagnóstico hay que demostrar la causa-
ASPECTOS RELACIONADOS CON LA CULTURA
ción de los signos/síntomas. Gran afectación del paciente y
Las transformaciones que se asemejan a síntomas conver-
de su entorno. Persistencia del comportamiento facticio y del
sivos (y disociativos) son comunes en determinados rituales
engaño para ocultarlo.
aceptados culturalmente. Cuando los síntomas se pueden
explicar completamente en un contexto cultural particular y
PREVALENCIA
no dan lugar a angustia o a una discapacidad clínicamente
Desconocida. Se estima en 1%.
significativa, entonces no se realiza el diagnóstico de trastor-
no de conversión.
DESARROLLO Y CURSO
El curso del trastorno suele ser en forma de episodios inter-
ASPECTOS RELACIONADOS CON EL GÉNERO
mitentes. Los episodios únicos y los que se caracterizan por
Es de dos a tres veces más frecuente en las mujeres.
ser persistentes y sin remisiones son menos frecuentes. El
inicio se produce generalmente en la edad adulta temprana
CONSECUENCIAS FUNCIONALES
(a menudo, después de un ingreso por un problema médico
Los individuos con síntomas de trastorno de conversión pre-
sentan un deterioro significativo. La gravedad de este dete- o trastorno mental).
rioro es similar a la que sufren las personas con enfermeda-
des médicas similares.

COMORBILIDAD
Trastornos de ansiedad y depresivos, habitualmente apare-
cen de forma conjunta con el trastorno de conversión. El
trastorno de síntomas somáticos también puede concurrir.
Los trastornos psicóticos, por uso de sustancias y el abuso de
alcohol son infrecuentes. Los trastornos de personalidad son
más frecuentes en los individuos con trastorno de conversión
que en la población general. Los trastornos neurológicos u
otras afecciones médicas también coexisten habitualmente.

61
TRASTORNOS DISOCIATIVOS DESARROLLO Y CURSO
Se asocia a experiencias abrumadoras en la niñez (maltrato,
abuso, etc.). Puede aparecer a cualquier edad.
TRASTORNO DE IDENTIDAD DISOCIATIVO Los niños no suelen presentar alternancia de personalidades,
sino solapamientos e interferencias entre los estados menta-
CARACTERÍSTICAS DIAGNÓSTICAS
les. Desencadenantes del inicio de los cambios entre perso-
La manifestación o no de los distintos estados de personali-
nalidades: 1) se suprime la situación traumática (pe. cambio
dad que constituyen el trastorno varía en función de la mo-
de casa), 2) la persona tiene niños que alcanzan la edad a
tivación psicológica, el nivel de estrés, los conflictos internos
la que sufrió el trauma; 3) existen experiencias traumáticas
y dinámicos, y la tolerancia emocional.
posteriores (aunque puedan parecer irrelevantes, por ejem-
Durante largos periodos de tiempo, las personas con TID sin
plo, un golpe con el coche), 4) se produce la muerte o una
forma de posesión no muestran abiertamente la discontinui-
enfermedad potencialmente mortal del maltratador.
dad de su personalidad y sólo una pequeña minoría presenta
alternancia entre sus personalidades.
FACTORES DE RIESGO Y PRONÓSTICO
Cuando no se observan claramente los estados de persona-
Ambientales: abuso físico y sexual, y otras formas de expe-
lidad alternantes, el cuadro puede ser identificado por dos
riencias traumáticas, incluyendo procedimientos médicos y
tipos de síntomas: 1) alteraciones repentinas o discontinui-
quirúrgicos en la infancia, guerra, prostitución infantil y te-
dad de la personalidad, y 2) amnesias disociativas recurren-
rrorismo. Abuso infantil y negligencia en el 90% de los casos
tes muchas veces de hechos cotidianos (lagunas de memoria
en Europa, EE. UU. y Canadá.
remotas, lapsos de memoria reciente o el descubrimiento de
Modificadores de curso: peor pronóstico si curso continuo,
una evidencia de sus acciones diarias y de las tareas que no
retraumatización posterior, comorbilidad con otros TM o en-
recuerdan haber realizado).
fermedad grave y retraso del tratamiento.
Son frecuentes las amnesias disociativas en estos casos, que
se manifiestan de tres formas: 1) lagunas de memoria remo-
ASPECTOS RELACIONADOS CON LA CULTURA
tas de acontecimientos vitales personales; 2) lapsos de me-
Muchas de las características del TID pueden estar influencia-
moria recientes; y 3) descubrimiento de alguna de sus accio-
das por el trasfondo cultural de la persona.
nes diarias y de las tareas que no recuerda haber realizado.
En lugares donde la “posesión” es común, las identidades
Las fugas disociativas también son frecuentes.
fragmentadas pueden tomar la forma de espíritus posesivos,
La actitud y conciencia de los sujetos con TID hacia estas
divinidades, demonios, animales o personales mitológicos.
amnesias son variables. Es frecuente que minimicen sus sín-
A diferencia de los estados de posesión culturalmente acep-
tomas.
tados, los TID de posesión son involuntarios, angustiantes e
Las identidades en forma de posesión normalmente se ma-
incontrolables, con frecuencia recurrentes y violan las nor-
nifiestan como acciones en las que parece que un espíritu
mas culturales/religiosas.
o fuerza sobrenatural toma el control, de tal manera que
la persona afectada se comporta de forma muy diferente
ASPECTOS RELACIONADOS CON EL GÉNERO
a la habitual. La mayoría de estos cuadros son “normales”
Las mujeres con TID predominan en la práctica clínica de
dentro del contexto cultural de la persona, no cumpliendo
adultos, pero no en la de niños. Falsos negativos en varo-
criterios de TID.
nes alumnos porque nieguen trauma y disimulen síntomas.
A diferencia de los estados de posesión culturalmente acep-
Las mujeres con TID presentan en mayor medida síntomas
tados, los TID de posesión son involuntarios, angustiantes e
disociativos agudos (flashbacks, amnesias, fugas, automuti-
incontrolables, con frecuencia recurrentes y violan las nor-
laciones…).
mas culturales/religiosas.
Los varones muestran más comportamientos criminales. Los
desencadenantes más comunes para ellos son: situaciones
CARACTERÍSTICAS QUE APOYAN EL DIAGNÓSTICO
de guerra, condiciones carcelarias y agresiones físicas o se-
Comorbilidad con depresión, ansiedad, abuso de sustancias,
xuales.
autolesiones y convulsiones no epilépticas.
A menudo informan de flashbacks disociativos durante los
RIESGO DE SUICIDIO
cuales reviven sensorialmente un evento anterior como si
Más del 70% han intentado suicidarse, incluso con múlti-
estuviera ocurriendo en el presente, a menudo con cambio
ples intentos. La evaluación del riesgo de suicidio se com-
de identidad concurrente. Pueden mostrarse desorientados
plica cuando hay amnesia posterior a la conducta autolesi-
durante el episodio y olvidar posteriormente la vivencia de
va o cuando la identidad que se presenta en la consulta no
ese flashback.
presenta esa ideación autolítica y desconoce que las otras
Historia frecuente de abuso o maltrato en la infancia o pro-
personalidades sí.
cesos médicos dolorosos y largos a edades tempranas.
Automutilación y comportamiento suicida frecuentes. Ma-
DIAGNÓSTICO DIFERENCIAL
yor capacidad para la sugestión hipnótica que la población
- Otro trastorno disociativo especificado.
general. Posible implicación de la corteza orbitofrontal, el
- Trastorno depresivo mayor: elevada comorbilidad.
hipocampo, la circunvolución del hipocampo y la amígdala.
En michas ocasiones no llegan a cumplirse criterios de De-
presión Mayor por la fluctuación del estado de ánimo y de
PREVALENCIA
las cogniciones asociadas.
Prevalencia a 12 meses en un pequeño grupo de EE. UU.:
- Trastornos bipolares: con frecuencia a los pacientes
1.5%. En ese estudio, 1.6 H y 1.4 M.
con TID se les diagnostica al principio de TB II debido a los

62
cambios rápidos de estado de ánimo. En ese estudio, 1.0 H y 2.6 M.
- TEPT: valorar si la sintomatología disociativa forma DESARROLLO Y CURSO
parte de un cuadro de TEPT o si es excesiva y requiere diag- El comienzo de la amnesia generalizada suele ser repentino.
nóstico de TID. Las amnesias localizadas y selectivas raramente son eviden-
- Trastornos psicóticos: el cambio caótico de persona- tes, incluso para el propio individuo. Un episodio de amnesia
lidad y las intrusiones agudas que irrumpen los procesos de disociativa puede ser factor de riesgo para episodios futuros.
pensamiento pueden ayudar al diagnóstico diferencial. La duración varía de minutos a décadas. Las capacidades di-
- Trastornos inducidos por sustancias o medicamen- sociativas pueden disminuir con la edad, aunque no siempre.
tos. A medida que la amnesia remite, puede aumentar la angus-
- Trastornos de la personalidad: especialmente difícil tia, la conducta autolítica y los síntomas de TEPT.
el diagnóstico diferencial con respecto al TLP. Valorar la dis- Para el diagnóstico en niños, es conveniente contar con la in-
función persistente y generalizada de la gestión emocional y formación procedentes de distintas fuentes. Puede ser difícil
de las relaciones interpersonales propia de los TTPP. de diferenciar de la falta de atención, el ensimismamiento,
- Trastornos conversivos (trastorno de síntomas neu- los problemas de ansiedad, las dificultades de aprendizaje y
rológicos funcionales): afectación más limitada y circunscrita las conductas oposicionistas.
en lo conversivo.
- Trastornos convulsivos: EEG alterado en los cuadros FACTORES DE RIESGO Y PRONÓSTICO
orgánicos. Ambientales: experiencias traumáticas simples o reiteradas
- Trastorno facticio y simulación: en estos casos los son antecedentes frecuentes. Se asocia a: 1) mayor número
síntomas se fingen (simulación) o de producen intencional- de experiencias adversas en la infancia, 2) violencia interper-
mente (facticio). sonal y 3) aumento de la gravedad, frecuencia y violencia del
- trauma.
REPERCUSIONES FUNCIONALES Genéticos y fisiológicos: no hay estudios genéticos en am-
Muy variable, desde uno aparentemente mínimo a un pro- nesia disociativa. En disociación en sentido amplio, distintos
fundo. Con el tratamiento adecuado, muchos pacientes estudios revelan factores genéticos y ambientales tanto en
muestran mejoría funcional laboral y personal. Algunos pue- población clínica como no clínica.
den responder al tratamiento muy lentamente, por lo que Modificadores del curso: la eliminación del agente traumá-
debe mantenerse a largo plazo. tico puede dar lugar a una recuperación rápida de la me-
moria. Los que presenta fuga disociativa son especialmente
COMORBILIDAD resistentes al tratamiento. El inicio de síntomas TEPT puede
- TEPT. hacer disminuir la amnesia localizada, selectiva o sistemática
- TDM. y recuperarse la memoria en forma de flashbacks, aunque
- Otros trastornos relacionados con el trauma y los luego pueda generarse amnesia sobre el contenido de esos
factores de estrés. flashbacks.
- TTPP (límite y evitativa).
- Trastornos de conversión. ASPECTOS RELACIONADOS CON LA CULTURA
- Trastorno de síntomas somáticos. En Asia, Medio Oriente y América, las convulsiones no epi-
- TCA. lépticas y otros síntomas neurológicos funcionales pueden
- Trastornos relacionados con sustancias. acompañar a la amnesia disociativa. En las culturas con tradi-
- TOC. ciones muy restrictivas, el desencadénate de la amnesia pue-
- Trastornos del sueño. de no llegar a ser algo tan traumático (tensiones o conflictos
psicológicos, de pareja, familiares, etc.).

ASPECTOS RELACIONADOS CON EL GÉNERO


AMNESIA DISOCIATIVA No se recogen en DSM 5.

CARACTERÍSTICAS DIAGNÓSTICAS RIESGO DE SUICIDIO


Dentro de este apartado, el DSM 5 describe los 5 tipos de Conductas autolesivas frecuentes. El comportamiento sui-
amnesia disociativa constituían un especificador en el DSM- cidad supone especial riesgo cuando la amnesia remite de
IV-TR: localizada, selectiva, generalizada, sistematizada y repente y la memoria oprime al individuo con recuerdos in-
continua. tolerables.

CARACTERÍSTICAS QUE APOYAN EL DIAGNÓSTICO DIAGNÓSTICO DIFERENCIAL


Muchos pacientes presentan dificultad crónica para formar y - TID: desregulación más amplia en el TID (sentido
mantener relaciones satisfactorias. del yo e identidad, no sólo memoria). Además, las amnesias
Historia de trauma, abuso y victimización frecuente. Algu- localizadas, selectivas y generalizadas son más estables en
nos experimentan flashbacks disociativos. el tiempo que las amnesias que presenta el TID. Las amne-
Muchos tienen antecedentes de automutilación, intentos de sias en el TID suelen ser de hechos cotidianos, fluctuaciones
suicidio y comportamientos de alto riesgo. bruscas en habilidades o conocimientos, lagunas mnésicas
Elevada capacidad para la hipnosis. Disfunciones sexuales breves de encuentros interpersonales…y no sólo de material
frecuentes. potencialmente traumático.
- TEPT: pueden recordar parte del trauma o su tota-
PREVALENCIA lidad. Si no recuerdan nada, puede hacerse diagnóstico co-
Prevalencia anual en un pequeño estudio de EE. UU.: 1.8%. mórbido.

63
- TNC: base orgánica. Afectación de memoria y otras hipotálamo-hipófiso-suprarrenal, lóbulo parietal inferior y
áreas cognitivas. circuito prefrontal-límbico.
- Trastornos relacionados con sustancias: blackouts PREVALENCIA
sólo en contexto de intoxicación por alcohol. El diagnóstico Síntomas transitorios son comunes en la población general.
diferencial puede ser difícil si la persona con amnesia diso- La mitad de los adultos han tenido en alguna ocasión algu-
ciativa hace uso del alcohol. El uso prolongado de alcohol na experiencia de despersonalización o desrealización. No se
u otras sustancias puede facilitar el desarrollo de trastornos dispone de datos de prevalencia muy claros. Se estima 2%.
neurocognitivos con afectación de la memoria, pero en este H = M.
caso, acompañada de afectación en otras áreas intelectua-
les. DESARROLLO Y CURSO
- Amnesia postraumática debida a lesión cerebral: Edad media de inicio: 16 años. Menos del 20% aparece des-
base orgánica. El problema mnésico en un TNC debido a pués de los 20 años. Menos del 5% aparece después de los
traumatismo debe aparecer inmediatamente después de la 25 años. La aparición en la cuarta década y más tarde es
lesión, su presentación es variable e implica cambios en dis- muy inusual. Inicio repentino o gradual. Curso a menudo
tintos dominios cognitivos (no exclusivamente en memoria). crónico. 1/3 episódico. 1/3 continuo. 1/3 episódico que evo-
- Trastornos convulsivos: fugas disociativas vs fugas luciona a continuo. La intensidad de los síntomas varía. El
epilépticas previas a la crisis convulsiva. En las fugas, el com- estrés puede empeorar la sintomatología.
portamiento de la persona es más errático, de vagabundeo,
sin propósito, y se acompaña de alteraciones electroencefa- FACTORES DE RIESGO Y PRONÓSTICO
lográficas. Temperamentales: evitación del daño, defensas inmaduras
- Estupor catatónico: suele acompañarse de otros (negación) y esquemas tanto de desconexión (deficiencia e
síntomas catatónicos. Aunque el mutismo de las catatonías inhibición emocionales, frecuente en casos de abuso, negli-
puede sugerir problemas mnésicos, el recuerdo no está afec- gencias y privación) como de sobreconexión (autonomía de-
tado. teriorada, se corresponden con dependencia, vulnerabilidad
- Trastorno facticio y simulación: en estos casos los e incompetencia).
síntomas se fingen (simulación) o de producen intencional- Ambientales: traumas interpersonales en la infancia como
mente (facticio). abuso o negligencia emocional (pero en menor medida que
en TID). Abuso físico y sexual, ser testigo de violencia en ám-
REPERCUSIONES FUNCIONALES bito familiar, crecer con un progenitor facetado por trastor-
En casos de amnesia localizada, selectiva y sistematizada, la no mental grave, la muerte inesperada o por suicidio de un
limitación funcional es entre limitada y grave. familiar o alguien cercano.
Si amnesia disociativa sistematizada. crónica, mayor repercu- Factores precipitantes: estrés grave, depresión, ansiedad y
sión en el funcionamiento. consumo de drogas (perturbadores, sobre todo).

COMORBILIDAD ASPECTOS RELACIONADOS CON LA CULTURA


- Alteraciones afectivas cuando la amnesia remite: Las experiencias voluntarias de despersonalización y desrea-
disforia, pena, rabia, culpa, vergüenza… lización en prácticas de meditación no se deben considerar
- Trastorno depresivo persistente. como trastorno. Algunos casos empiezan siendo voluntarias
- Trastorno depresivo mayor. y pierden el control sobre ellos.
- Trastorno de adaptación con síntomas depresivos o
mixtos. ASPECTOS RELACIONADOS CON EL GÉNERO
- TEPT. No se recogen en DSM 5.
- Trastornos de síntomas somáticos y relacionados.
- TTPP (límite, evitativa y dependiente). RIESGO DE SUICIDIO
No se recoge este apartado para este diagnóstico en DSM 5.

DIAGNÓSTICO DIFERENCIAL
- Trastorno de ansiedad por enfermedad: las mani-
TRASTORNO DE DESPERSONALIZACIÓN/DES- festaciones de preocupación por la presencia de síntomas
REALIZACIÓN somáticos y su repercusión sobre la salud es más extensa en
los pacientes con Trastorno de ansiedad por enfermedad.
CARACTERÍSTICAS DIAGNÓSTICAS - Trastorno depresivo mayor: sensaciones de fatiga,
De describen en este apartado los síntomas de despersonali- cansancio, entumecimiento, etc. son frecuentes de depre-
zación y desrealización. sión y remiten con la mejoría afectiva. Además, estos sínto-
mas se asocian a otros síntomas afectivos que no aparecen
CARACTERÍSTICAS QUE APOYAN EL DIAGNÓSTICO en el Trastorno de despersonalización/desrealización. Puede
Muchos individuos tienen dificultades para describir sus sín- hacerse diagnóstico adicional si se cumplen ambas condicio-
tomas. Pueden sentir que están “volviéndose locos” o temer nes clínicas de manera concurrente.
un daño cerebral irremediable. Alteración subjetiva de la per- - TOC: los pacientes con Trastorno de despersonaliza-
cepción del paso del tiempo. ción/desrealización pueden manifestar preocupación excesi-
Síntomas somáticos tenues como la saturación de la cabeza, va por su sintomatología y desarrollar conductas de chequeo
hormigueo, sensación de desmayo… Preocupaciones obse- de su estado, pero otros síntomas del TOC están ausentes.
sivas y rumiaciones marcadas. Hiporreactividad fisiológica - Otros trastornos disociativos: no debe diagnosticar-
ante estímulos emocionales. Sustratos neuronales en el eje se Trastorno de despersonalización/desrealización si estos

64
síntomas solo aparecen en el contexto de otro trastorno di-
sociativo.
- Trastornos de ansiedad: no debe diagnosticarse
Trastorno de despersonalización/desrealización si estos sín- TRASTORNOS DE LA CONDUCTA
tomas solo aparecen en el contexto de un ataque de páni-
co. Se puede hacer un diagnóstico adicional de Trastorno de ALIMENTARIA Y DE LA INGESTA DE
despersonalización/desrealización en cuadros de ansiedad si: ALIMENTOS
1) el componente de despersonalización/desrealización des-
taca mucho desde el inicio, o 2) estos síntomas continúan
cuando el pánico ya ha remitido. PICA
- Trastornos psicóticos: el juicio de realidad se man-
tiene intacto durante los episodios de despersonalización/ CARACTERÍSTICAS DIAGNÓSTICAS
desrealización y la presencia de síntomas positivos propios Las sustancias que típicamente se suelen ingerir varían con la
de la psicosis son poco frecuentes (salvo ideas de carácter edad y disponibilidad (podrían ser papel, jabón, ropa, pelo,
nihilista). cuerdas, lana, tierra, tiza, polvos de talco, pintura, chicles,
- Trastornos inducidos por sustancias o medicamen- metales, guijarros, carbón vegetal o mineral, cenizas, barro,
tos: el consumo de sustancias perturbadoras puede generar almidón o hielo. No existe por lo general aversión a la comi-
sintomatología de despersonalización/desrealización. Si los da.
síntomas se extienden más allá del consumo, se debe hacer
el diagnóstico de Trastorno de despersonalización/desreali- CARACTERÍSTICAS QUE APOYAN EL DIAGNÓSTICO
zación. En un 15% de los casos de trastorno, éste se precipi- A pesar de que se han descrito deficiencias de vitaminas o
ta por la ingesta de estas sustancias. minerales (p.ej. zinc, hierro), no se suelen hallar anormalida-
- Trastornos debidos a afectación orgánica: estos sín- des biológicas específicas. Puede haber complicaciones mé-
tomas puedes aparecer en cuadros de epilepsia del lóbulo dicas (p.ej. problemas de la mecánica intestinal, obstrucción,
temporal y, en menor medida, en las epilepsias frontales y perforación, infecciones).
parietales. La aparición más tarde de los 40 años y los sínto-
mas atípicos pueden sugerir organicidad. Es recomendable PREVALENCIA
realizar una evaluación médica y neurológica completa. No está clara. Parece aumentar con la gravedad de la Disca-
pacidad Intelectual.
REPERCUSIONES FUNCIONALES
Repercusiones derivadas de la afectividad aplanada y robóti- DESARROLLO Y CURSO
ca que experimentan estos pacientes cuando están con sín- Lo más frecuente es inicio en la infancia, aunque puede ser a
tomas de despersonalización o desrealización. Baja conexión cualquier edad. Si inicia en la adultez, lo hace más frecuente-
emocional con los demás. Sensación de desconexión de la mente en el contexto de una Discapacidad Intelectual u otro
vida. trastorno mental y durante el embarazo (puede haber anto-
jos específicos como de tiza o hielo). El curso puede ser pro-
COMORBILIDAD longado y dar lugar a urgencias médicas; puede ser poten-
- Alta comorbilidad con: Trastorno depresivo mayor y cialmente mortal dependiendo de las sustancias ingeridas.
Trastornos de ansiedad.
- Baja comorbilidad con TEPT. FACTORES DE RIESGO Y PRONÓSTICO
- TTPP: evitativa, límite y obsesivo-compulsiva. El abandono, la falta de supervisión y el retraso del desarrollo
pueden aumentar el riesgo.

ASPECTOS RELACIONADOS CON LA CULTURA
En algunas culturas comer tierra u otras sustancias nutritivas
es una creencia con valor espiritual, medicinal o social; en
este caso no está justificado el diagnóstico.

ASPECTOS RELACIONADOS CON EL GÉNERO


Se produce en los dos sexos. Puede darse en mujeres emba-
razadas, pero se sabe poco del curso en el postparto.

DIAGNÓSTICO DIFERENCIAL
La pica puede darse junto con otro trastorno mental (Dis-
capacidad Intelectual, Trastorno del Espectro del Autismo,
Esquizofrenia…); solo se efectúa el diagnóstico adicional si
requiere atención clínica adicional o es grave y persistente.
Anorexia nerviosa: si la ingestión de sustancias nutritivas se
hace como método para controlar el peso, la anorexia es
el diagnóstico primario (p.ej. ingerir pañuelos de papel para
controlar el apetito). Trastorno Facticio: algunos individuos
pueden ingerir intencionadamente como parte del patrón
de falsificación de síntomas físicos. Autolesión no suicida y
conductas autolesivas no suicidas en los Trastornos de Perso-

65
nalidad: ingestión de objetos potencialmente dañinos (p.ej. REPERCUSIONES FUNCIONALES
alfileres, agujas, cuchillos). Malnutrición secundaria que puede estar asociada a retraso
del crecimiento y afectar negativamente al potencial de de-
REPERCUSIONES FUNCIONALES sarrollo y aprendizaje.
Complicaciones gastrointestinales, envenenamiento, infec-
ción y deficiencias nutricionales. COMORBILIDAD
Puede presentarse regurgitación con rumiación asociada en
COMORBILIDAD contexto de afección médica concurrente u otro trastorno
Los más frecuentes como diagnósticos comórbidos son el mental (por ejemplo, Trastorno de Ansiedad Generalizada).
Trastorno del Espectro Autista y la Discapacidad Intelectual,
y en menor grado la Esquizofrenia y el Trastorno Obsesi-
vo-Compulsivo. Puede asociarse también a la Tricotilomanía,
Excoriación, Evitación/Restricción de la ingesta de alimentos. TRASTORNO DE EVITACIÓN/RESTRICCIÓN DE
LA INGESTA DE ALIMENTOS
TRASTORNO DE RUMIACIÓN CARACTERÍSTICAS DIAGNÓSTICAS
Este trastorno sustituye y amplía el diagnóstico de trastorno
CARACTERÍSTICAS DIAGNÓSTICAS de alimentación en la infancia y primera niñez. La determi-
La comida tragada, que puede estar parcialmente digerida, nación del grado de deficiencia nutricional depende también
se devuelve a la boca aparentemente sin náuseas, arcadas ni de la valoración clínica y el efecto sobre la salud física puede
desagrado. La comida se puede volver a masticar para des- ser de una gravedad similar al que se produce en la anore-
pués escupirse de la boca o tragarse. Los individuos pueden xia nerviosa (por ejemplo, hipotermia, bradicardia, anemia).
describir el comportamiento como habitual o fuera de su Puede haber dependencia de una sonda de alimentación por
control. gastrostomía o de suplementos nutricionales orales comple-
tos. En algunos individuos, la evitación o restricción puede
CARACTERÍSTICAS QUE APOYAN EL DIAGNÓSTICO estar basada en las características organolépticas de la co-
Los niños pequeños adoptan una postura característica, de mida y tratarse de una sensibilidad extrema a su apariencia,
esfuerzo y de arqueo de la espalda con la cabeza mantenida color, olor, textura, temperatura o gusto. Se ha descrito tam-
hacia atrás, mientras realizan movimientos de succión con la bién como “alimentación restrictiva”, “alimentación selecti-
lengua. Pueden dar la impresión de obtener satisfacción con va”, “alimentación exigente”, “alimentación perseverante”,
dicha actividad, y estar irritables y hambrientos entre episo- “rechazo crónico de los alimentos” y “neofobia alimenta-
dios. La pérdida de peso / no consecución del esperado, son ria”.
frecuentes. Los adolescentes ya adultos pueden intentar disi-
mular tapándose la boca con la mano o tosiendo, o evitando CARACTERÍSTICAS QUE APOYAN EL DIAGNÓSTICO
comer en situaciones sociales. Los bebés y niños pequeños pueden no interactuar con su
cuidador principal durante la alimentación o no mostrar
PREVALENCIA que tienen hambre; niños pequeños y adolescentes puede
Datos no concluyentes, si bien suele darse con más frecuen- estar asociada a problemas emocionales más generalizados,
cia en grupos como los individuos con Discapacidad Intelec- denominándose “trastorno emocional de evitación de ali-
tual. mentos”. Marcadores diagnósticos: malnutrición, bajo peso,
retraso de crecimiento y necesidad de nutrición artificial en
DESARROLLO Y CURSO ausencia de otra afección médica.
Puede iniciar a cualquier edad. En bebés suele ser entre los
3 y 12 meses, frecuentemente remitiendo espontáneamen- DESARROLLO Y CURSO
te, aunque su curso puede ser prolongado y acabar en ur- Resultados a largo plazo sugieren que la evitación o restric-
gencias. Potencialmente mortal en periodo de lactancia. En ción basada en aspectos sensoriales es relativamente esta-
individuos con Trastorno del Espectro Autista o Discapaci- ble y duradera, pero si persiste hasta la edad adulta puede
dad Intelectual puede tener función autotranquilizadora o asociarse a un funcionamiento relativamente normal. No
de autoestimulación similar a la de otros comportamientos hay suficientes indicios que relacionen este trastorno con un
motores repetitivos. posterior inicio de un Trastorno de la Conducta Alimentaria.
Si al cambiar de cuidadores se nota una mejoría de la ali-
FACTORES DE RIESGO Y PRONÓSTICO mentación y el peso, este hecho es indicativo de la posible
Pueden ser factores predisponentes la falta de estimulación, existencia de psicopatología parental, malos tratos o desa-
la negligencia, situaciones vitales estresantes y problemas de tención del niño.
relación entre padres e hijos.
FACTORES DE RIESGO Y PRONÓSTICO
DIAGNÓSTICO DIFERENCIAL Temperamentales: Trastornos de Ansiedad, Trastorno del Es-
Afecciones gastrointestinales: descartar reflujo esofágico o pectro Autista, Trastorno Obsesivo-Compulsivo y Trastorno
vómitos, gastroparesia, estenosis pilórica, hernia de hiato, por Déficit de Atención/Hiperactividad pueden aumentar el
síndrome de Sandifer en bebés; Anorexia Nerviosa y Bulimia riesgo.
Nerviosa: personas con estos diagnósticos pueden presentar Ambientales: Ansiedad familiar, hijos de madres con Trastor-
regurgitación como sistema para perder las calorías ingeri- no de la Conducta Alimentaria.
das. Genéticos y fisiológicos: Antecedentes de afecciones gas-
trointestinales, reflujo gastroesofágico, vómitos, etc.

66
ASPECTOS RELACIONADOS CON LA CULTURA de crecimiento esperado. Los clínicos deberían considerar las
No diagnosticar cuando la evitación/restricción esté relacio- guías numéricas disponibles además de la constitución física
nada únicamente con determinadas prácticas religiosas o del individuo, los antecedentes referentes al peso y cualquier
culturales. alteración fisiológica.
Si los individuos buscan ayuda por su cuenta, normalmente
ASPECTOS RELACIONADOS CON EL GÉNERO es por malestar asociado a las secuelas somáticas y psicológi-
Frecuente en niños y niñas, tanto en lactancia como en pri- cas de hambre; normalmente son llevados por los miembros
mera infancia; cuando es comórbido con Trastorno del Es- de su familia. Frecuentemente no tienen un concepto claro
pectro Autista, predomina en varones. Puede producirse en del problema o directamente lo niegan.
relación con la sensibilidad sensorial alterada del embarazo.
CARACTERÍSTICAS QUE APOYAN EL DIAGNÓSTICO
DIAGNÓSTICO DIFERENCIAL El compromiso nutricional asociado afecta a la mayor parte
Otras afecciones médicas (por ejemplo, enfermedad gas- de los sistemas orgánicos principales y puede producir va-
trointestinal, alergia e intolerancia alimentaria, cáncer ocul- rias alteraciones, como la amenorrea y las anomalías de las
to): los síntomas de este trastorno pueden presentarse en constantes vitales. La mayoría de las alteraciones son rever-
otras afecciones médicas, especialmente en aquellas que sibles, pero algunas como la pérdida de densidad mineral
cursan con vómitos, pérdida de apetito, náuseas, dolor ab- ósea no lo son del todo. Muchas personas tienen síntomas
dominal o diarrea; trastornos congénitos, estructurales, neu- depresivos (aislamiento, irritabilidad, insomnio, disminución
rológicos/neuromusculares específicos y afecciones asocia- deseo sexual) cuando tienen bajo peso. Los síntomas obse-
das con problemas de alimentación: problemas sobre todo sivo-compulsivos son a menudo destacados (preocupación y
en la función y estructura oral/esofágica/faríngea, como la pensamientos sobre la comida, compulsiones relacionadas
hipotonía de la musculatura, la protusión de la lengua y la con la comida, etc.). Puede haber preocupación por comer
deglución insegura; Trastorno de Apego Reactivo; Trastorno en público, sensación de ineficacia, deseos de controlar lo
de Espectro Autista; Fobia Específica, Trastorno de Ansiedad que les rodea, pensamiento inflexible, espontaneidad social
Social y otros Trastornos de Ansiedad: fobia a situaciones limitada y contención excesiva de la expresión emocional. En
específicas que pueden derivar en ahogo o vómitos, o miedo comparación en el subtipo restrictivo, el subtipo con atra-
a ser observado por los demás mientras come; Anorexia Ner- cones/purgas presentan niveles superiores de impulsividad y
viosa: en algunos casos, el trastorno de evitación/restricción son más propensos a abusar del alcohol y de otras drogas.
e la ingesta de alimentos puede preceder a la anorexia - me- Pueden presentar niveles excesivos de actividad física o usar
jor el diagnóstico diferencial en clínica duradera-; Trastorno incorrectamente los medicamentos (por ejemplo, individuos
Obsesivo-Compulsivo, Trastorno Depresivo Mayor, Trastor- con diabetes que omitan o reduzcan las dosis de insulina
nos del Espectro de la Esquizofrenia y Trastorno Facticio. para minimizar el metabolismo de los carbohidratos).
Marcadores diagnósticos: leucopenia, anemia, trombocito-
REPERCUSIONES FUNCIONALES penia, problemas hemorrágicos, niveles elevados de nitróge-
Deterioro del desarrollo físico y las dificultades sociales, que no ureico en sangre, hipercolesterolemia, niveles elevados de
pueden tener una importante repercusión negativa en el enzimas hepáticas, hipomagnesemia, hipozinquemia, hipo-
funcionamiento familiar. fosfatemia e hiperamilasemia, alcalosis metabólica, hipoclo-
remia, hipopotasemia, acidosis metabólica leve, niveles bajo
COMORBILIDAD de tirosina y de triyodotironina, mujeres niveles bajos de es-
Trastornos de Ansiedad, Trastorno Obsesivo-Compulsivo y trógenos y hombres de testosterona, bradicardia sinusal, rara
trastornos del Neurodesarrollo (especialmente Trastorno del vez se detectan arritmias, baja densidad mineral ósea, riesgo
Espectro Autista, Trastorno por Déficit de Atención/Hiperac- de fracturas, encefalopatía metabólica, reducción significati-
tividad y Discapacidad Intelectual). va del gasto energético en reposo, amenorrea, estreñimien-
to, dolor abdominal, intolerancia al frio, letargia, exceso de
energía, emaciación, hipotensión, hipotermia y bradicardia,
ANOREXIA NERVIOSA lanugo, edema periférico (durante la recuperación de peso),
hipercarotenemia (color amarillento de la piel), hipertrofia de
CARACTERÍSTICAS DIAGNÓSTICAS las glándulas salivales (parótidas aumentadas), erosión del
Subtipos: la mayoría del subtipo con atracones/purgas, que esmalte dental, cicatrices o callos en la superficie dorsal de la
se atracan de comida, también se purgan a través del vómito mano (signo de Russell).
autoprovocado o la utilización incorrecta de laxantes, diuré-
ticos o enemas. Algunas personas no se atracan de comida, PREVALENCIA
pero se purgan regularmente después de consumir peque- En mujeres, a los 12 meses la prevalencia es del 0,4%. Se
ñas cantidades de alimentos. El intercambio entre subtipos sabe menos de la prevalencia en hombres, pero es mucho
durante el curso del trastorno NO es infrecuente (el subtipo menos común, aproximadamente 1:10.
sirve más para describir los síntomas actuales que para des-
cribir el curso longitudinal). DESARROLLO Y CURSO
Para calcular el IMC es útil basarse en las consideraciones de Suele empezar en la adolescencia o en la edad adulta tem-
los CDC (Centers for Disease Control and Prevention) y la prana. Rara vez empieza antes de la pubertad o después de
OMS. En niños y adolescentes no es tan fácil determinar los los 40 años, aunque se han descrito casos de inicio tanto
estándares de peso por las variaciones del curso y desarro- temprano como tardío. El inicio suele asociarse a un aconte-
llo; los CDC han utilizado un IMC según la edad por debajo cimiento vital estresante, como dejar el hogar familiar para
del percentil 5 como indicativo de peso insuficiente, o por ir a la universidad. El curso y el desenlace son muy variables.
encima de esta referencia si no logran mantener el curso Las más jóvenes pueden manifestar características atípicas

67
como el negar el “miedo a estar gordo”. En los individuos de te en el subtipo restrictivo. Abuso de alcohol y de otras sus-
más edad es más probable que tenga una mayor duración y tancias pueden ser comórbidos especialmente en el subtipo
la presentación clínica puede contar con más signos y sínto- con atracones/purgas.
mas propios de un trastorno de larga duración. Los clínicos
no deberían excluir el diagnóstico basándose únicamente en BULIMIA NERVIOSA
la mayor edad del individuo.
Algunos individuos se recuperan completamente después de CARACTERÍSTICAS DIAGNÓSTICAS
un solo episodio; algunos muestran un patrón fluctuante de Algunos individuos describen características disociativas du-
aumento de peso seguido de una recaída y otros presentan rante o después de los episodios de atracones. El tipo de
un curso crónico durante muchos años. Puede ser necesa- alimento consumido durante los atracones varía entre los
ria la hospitalización para recuperar el peso y solucionar las distintos individuos y en un mismo individuo. Durante los
complicaciones médicas. La mayoría presentan una remisión atracones, los individuos tienden a comer alimentos que en
a los 5 años del comienzo. Entre los ingresados, las tasas de otras circunstancias evitarían. Se sienten típicamente aver-
remisión total pueden ser menores. La tasa bruta de mor- gonzados de sus problemas e intentan ocultar sus síntomas,
talidad (TBM) es de aproximadamente un 5% por década. cometiendo los atracones a escondidas o lo más discreta-
La muerte se suele producir por las complicaciones médicas mente posibles. El antecedente más frecuente de los atra-
asociadas a la propia enfermedad o por suicidio. cones es el sentimiento negativo, los factores estresantes
Riesgo de suicidio: es elevado (tasas de 12/100.000 por año). interpersonales, la restricción de la dieta, los sentimientos
negativos acerca del peso corporal, la constitución corporal
FACTORES DE RIESGO Y PRONÓSTICO y los alimentos, y el aburrimiento. En algunos casos, vomitar
Temperamentales: Trastornos de Ansiedad o rasgos obsesi- es una finalidad en sí misma.
vos en la infancia.
Ambientales: Variabilidad histórica e intercultural; mayor CARACTERÍSTICAS QUE APOYAN EL DIAGNÓSTICO
prevalencia en entornos en los que se valora la delgadez. Los individuos con Bulimia Nerviosa suelen estar dentro del
Profesiones como modelo o deportista de élite. rango del peso normal o tener sobrepeso, siendo infrecuen-
Genéticos y fisiológicos: Mayor riesgo de Anorexia Nerviosa te en personas obesas. Pueden producirse irregularidades
y Bulimia Nerviosa en familiares de primer grado. También menstruales o amenorrea (no está claro si relacionada con las
mayor riesgo de Trastorno Depresivo Mayor y Trastorno Bi- fluctuaciones del peso, por deficiencias nutricionales o por el
polar en familiares de primer grado, sobre todo en el subtipo malestar emocional). Puede haber alteraciones hidroelectro-
atracones/purgas. Los porcentajes en gemelos monocigóti- líticas graves consecuencia de las purgas, llegando raramen-
cos es mayor que en dicigóticos. Se han descrito hallazgos y te a desgarros esofágicos, perforación gástrica y arritmias
anomalías cerebrales gracias a técnicas de imagen funcional. cardíacas. El uso crónico de laxantes puede hacer desarrollar
No está claro hasta qué punto estos hallazgos reflejan alte- una dependencia a estos para estimular los movimientos in-
raciones asociadas a la desnutrición o anomalías primarias al testinales. También se ha descrito el prolapso rectal.
trastorno.
PREVALENCIA
ASPECTOS RELACIONADOS CON LA CULTURA La prevalencia a 12 meses en mujeres jóvenes es del 1-1,5%.
Hay variaciones interculturales; más prevalente en países in- La prevalencia puntual es mayor en adultos jóvenes (pico en
dustrializados con rentas altas, aunque se desconoce su inci- adolescencia/edad adulta temprana). Poco se sabe sobre la
dencia en la mayoría de los países con rentas medias y bajas. prevalencia en hombres, aunque se sabe que es mucho me-
nos común con proporción de 1:10.
DIAGNÓSTICO DIFERENCIAL
Afecciones médicas (p.je. enfermedad gastrointestinal, hi- DESARROLLO Y CURSO
pertiroidismo, cáncer oculto y SIDA): puede haber importan- Suele comenzar en la adolescencia o en la edad adulta tem-
tes pérdidas de peso; pueden desencadenar ocasionalmente prana. El inicio antes de la pubertad o después de los 40
el inicio o recurrencia de la Anorexia Nerviosa. Rara vez se años es infrecuente. Los atracones frecuentemente empie-
desarrolla tras cirugía bariátrica. zan durante o después de una dieta para perder peso. La
Trastorno Depresivo mayor, Esquizofrenia, Trastorno por experiencia de acontecimiento vitales estresantes puede pre-
Consumo de Sustancias (estimulantes y anorexígenos), Tras- cipitar el inicio de la Bulimia Nerviosa. El curso puede ser cró-
torno de Ansiedad social, Trastorno Obsesivo-Compulsivo y nico o intermitente, con periodos de remisión alternándose
Trastorno Dismórfico Corporal, Bulimia Nerviosa (mantienen con recurrencias de atracones. En el seguimiento a largo pla-
el peso corporal igual o mayor que el límite normal), Tras- zo, los síntomas parecen disminuir con o sin tratamiento. Las
torno de Evitación/Restricción de la infesta de alimentos (no remisiones mayores de un año se asocian a mejores resulta-
tienen miedo a ganar peso o engordar ni ninguna alteración dos a largo plazo. Riesgo alto de mortalidad (por suicidio y
en la percepción de su peso o forma corporal). por todas las causas).
El cambio de diagnóstico de Bulimia Nerviosa inicial a Ano-
REPERCUSIONES FUNCIONALES rexia Nerviosa se produce en una minoría de los casos (10-
Algunas personas siguen activas profesional y socialmente, 15%). Hay individuos que continúan con los atracones, pero
otros muestran aislamiento significativo e incapacidad para dejan de realizar comportamientos compensatorios inapro-
mantener el rendimiento. piados (cumplen criterios de trastorno de atracón u otro
Trastorno de la Conducta Alimentaria especificado).
COMORBILIDAD
Trastornos de Ansiedad, Depresivos y Bipolares. Se han des- FACTORES DE RIESGO Y PRONÓSTICO
crito casos de Trastorno Obsesivo-Compulsivo, especialmen- Temperamentales: Preocupaciones sobre el peso, baja auto-

68
estima, síntomas depresivos, Trastorno de Ansiedad Social y y los alimentos, y el aburrimiento. En algunos casos, vomitar
Trastorno de Ansiedad Generalizada en la infancia. es una finalidad en sí misma.
Ambientales: Interiorización de un ideal de cuerpo delgado, CARACTERÍSTICAS QUE APOYAN EL DIAGNÓSTICO
abuso físico o sexual en la infancia. Se produce normalmente en personas de peso normal/so-
Genéticos y fisiológicos: Obesidad en la infancia, desarro- brepeso y en personas obesas. Sin embargo, este trastorno
llo puberal temprano. Puede existir transmisión familiar, así es distinto de la obesidad. La mayoría de las personas obesas
como vulnerabilidad genética para el trastorno. no tiene atracones recurrentes.
Modificadores del curso: La gravedad de la comorbilidad psi-
quiátrica predice peores resultados a largo plazo. PREVALENCIA
La prevalencia a 12 meses en los adultos es del 1,6% muje-
ASPECTOS RELACIONADOS CON LA CULTURA res y 0,8% hombres. La proporción por género está mucho
Prevalencia similar en la mayoría de los países industrializa- menos desequilibrada en el Trastorno por Atracón que en la
dos. Bulimia Nerviosa. Es igual de prevalente interracialmente. Es
más prevalente entre los individuos que buscan tratamiento
ASPECTOS RELACIONADOS CON EL GÉNERO para perder peso que en la población general.
Mucho más común en mujeres que en hombres. Los hom-
bres están especialmente infrarrepresentados en las mues- DESARROLLO Y CURSO
tras de personas que buscan tratamiento por razones que Se conoce poco sobre el desarrollo. Se puede presentar en ni-
aún no se han examinado. ños y se asocia a un aumento de la grasa corporal, de peso y
aumento de los síntomas psicológicos. La pérdida de control
DIAGNÓSTICO DIFERENCIAL puede representar la fase prodrómica de un Trastorno de la
Anorexia Nerviosa, del tipo con atracones/purgas (peso por Conducta Alimentaria. Muchos individuos con Trastorno por
debajo de los límites normales); Trastorno de Atracones (no Atracón hacen régimen después de desarrollar los atracones
comportamientos compensatorios); Síndrome de Kleine-Le- (en contraste con la Bulimia Nerviosa, en la que el cumpli-
vine (puede aparecer alteración del comportamiento alimen- miento de un régimen disfuncional generalmente precede el
tario, pero no preocupaciones por el peso y la constitución inicio del atracón). Típicamente empieza en la adolescencia o
corporal); Trastorno Depresivo Mayor con características atí- edad adulta temprana, aunque también puede empezar más
picas (es frecuente la sobrealimentación, pero no hay com- tardíamente. Las tasas más altas de remisión están en los
portamientos compensatorios ni preocupación por el peso/ individuos que buscan tratamiento. Parece ser un trastorno
figura; Trastorno de Personalidad Límite (pueden hacerse relativamente persistente, con curso comparable a la Bulimia
ambos diagnósticos). Nerviosa en términos de gravedad y duración. La oscilación
de este trastorno con otro Trastorno de la Conducta Alimen-
REPERCUSIONES FUNCIONALES taria es poco frecuente.
Una minoría refiere deterioros funcionales graves y el ámbito
de la vida social es el que más probablemente se puede ver FACTORES DE RIESGO Y PRONÓSTICO
afectado. El riesgo de suicidio es elevado. Genéticos y fisiológicos: El Trastorno por Atracón se da den-
tro de algunas familias, lo que puede reflejar influencias ge-
COMORBILIDAD néticas aditivas.
La mayoría presenta, al menos, otro trastorno mental. Hay
alteraciones del estado de ánimo que puede preceder a la ASPECTOS RELACIONADOS CON LA CULTURA
Bulimia Nerviosa, también síntomas de ansiedad. Normal- La prevalencia es bastante similar entre las diferentes cultu-
mente remiten tras un tratamiento eficaz. La prevalencia del ras.
consumo de sustancias (particularmente alcohol y estimulan-
tes) es de al menos el 30%, a menudo iniciándose el consu- DIAGNÓSTICO DIFERENCIAL
mo como un intento de controlar el peso y el apetito. Puede Bulimia Nerviosa (hay comportamientos compensatorios in-
haber comorbilidad con Trastornos de Personalidad, sobre adecuados). Las tasas de mejoría son mejores en pacientes
todo con el Límite. con Trastorno por Atracón que en Bulimia Nerviosa.
Obesidad: el Trastorno por Atracón se asocia a sobrepeso
y obesidad, pero tiene características fundamentalmente
TRASTORNO DE ATRACONES distintivas; sobreevaluación del peso y constitución corporal
mayor en obesos con Trastorno por Atracón, tasas de co-
CARACTERÍSTICAS DIAGNÓSTICAS morbilidad psiquiátrica más altas en obesos con Trastorno
Algunos individuos describen características disociativas du- por Atracón; resultados satisfactorios a largo plazo de los
rante o después de los episodios de atracones. El tipo de tratamientos psicológicos para Trastorno por Atracón con-
alimento consumido durante los atracones varía entre los trastan con la ausencia de tratamientos eficaces a largo pla-
distintos individuos y en un mismo individuo. Durante los zo para obesidad.
atracones, los individuos tienden a comer alimentos que en Trastorno Depresivo Mayor, Trastorno Bipolar y Trastorno de
otras circunstancias evitarían. Se sienten típicamente aver- Personalidad Límite (comorbilidad del Trastorno de Persona-
gonzados de sus problemas e intentan ocultar sus síntomas, lidad Límite comparable a la Anorexia Nerviosa y a la Bulimia
cometiendo los atracones a escondidas o lo más discreta- Nerviosa).
mente posibles. El antecedente más frecuente de los atra-
cones es el sentimiento negativo, los factores estresantes REPERCUSIONES FUNCIONALES
interpersonales, la restricción de la dieta, los sentimientos Problemas de adaptación al rol social, deterioro de la calidad
negativos acerca del peso corporal, la constitución corporal de vida relacionada con la salud y con la satisfacción en la

69
vida, aumento de morbilidad médica y mortalidad u mayor DISFUNCIONES SEXUALES
utilización de los servicios de salud. Aumento del riesgo a
ganar peso y desarrollar obesidad.
EYACULACIÓN RETARDADA
COMORBILIDAD
Los trastornos comórbidos más frecuentes son Trastorno Bi- CARACTERÍSTICAS DIAGNÓSTICAS
polar, Trastorno Depresivo Mayor, Trastornos de Ansiedad Retraso marcado o incapacidad para la eyaculación. La defi-
y, en menor grado, Trastornos por Consumo de Sustancias. nición de “retardado” no tiene límites precisos, no hay con-
La comorbilidad psiquiátrica está relacionada con la grave- senso sobre un tiempo razonable para alcanzar el orgasmo.
dad de los atracones y no con el grado de obesidad. El diagnostico se realiza en base a lo referido por el propio
paciente.

CARACTERÍSTICAS QUE APOYAN EL DIAGNÓSTICO


Intentos prolongados para alcanzar el orgasmo hasta el pun-
to de producir agotamiento o molestias genitales. Evitación
de la actividad debido a la incapacidad para eyacular. La pa-
reja sexual puede referir sentirse menos atractiva sexualmen-
te.

Deben considerarse:
1) factores de pareja.
2) factores de la relación.
3) factores de vulnerabilidad individual, comorbilidad
psiquiátrica y factores de estrés.
4) factores culturales o religiosos.
5) factores médicos relevantes.

PREVALENCIA
No está clara por falta de definición precisa. Queja sexual
masculina menos frecuente: menos del 1% refiere proble-
mas para alcanzar la eyaculación de más de 6 meses de du-
ración.

DESARROLLO Y CURSO
La prevalencia se mantiene hasta los 50 años de edad, cuan-
do la incidencia comienza a aumentar de manera significati-
va. En la década de los 80 refieren el doble de dificultad para
eyacular que los menores de 59 años.

FACTORES DE RIESGO Y PRONÓSTICO


Genéticos y fisiológicos: Pérdida de nervios sensoriales peri-
féricos de conducción rápida y disminución de la secreción
de esteroides sexuales.

ASPECTOS RELACIONADOS CON LA CULTURA


Más común en población asiática que en Europa, Australia
o EEUU.

DIAGNÓSTICO DIFERENCIAL
El diagnóstico diferencial principal se realiza con la eyacula-
ción retardada explicada por otra afección o lesión médica.
La presencia de un aspecto situacional sugiere problema psi-
cológico.

REPERCUSIONES FUNCIONALES
Dificultades para concebir. Sufrimiento psicológico de uno o
ambos miembros de la pareja.

COMORBILIDAD
La eyaculación retardada puede ser más frecuente en los ca-
sos graves de depresión mayor.

70
TRASTORNO ERECTIL ferencial debe incluir las etiologías médicas, en especial en-
fermedad vascular. La presencia de enfermedad orgánica no
CARACTERÍSTICAS DIAGNÓSTICAS es suficiente para confirmar una relación causal. Cuando se
Fracaso repetido para conseguir o mantener las erecciones debe a factores orgánicos es generalizada de inicio gradual.
durante la actividad sexual en pareja. Debe estar presente Excepción: problemas de erección tras lesión traumática de
durante un período de tiempo significativo (unos 6 meses) la inervación de los órganos genitales.
y suceder la mayoría de las ocasiones de actividad sexual (al - Cuando son situacionales e inconstantes y tienen
menos 75%). Puede aparecer sólo en situaciones concretas un inicio agudo tras un acontecimiento estresante se deben
que implican determinados tipos de estimulación o de pare- con mayor frecuencia a sucesos psicológicos. Menores de 40
ja, o ser generalizada. años sugieren etiología psicológica.
- Otras disfunciones sexuales: puede coexistir con
CARACTERÍSTICAS QUE APOYAN EL DIAGNÓSTICO eyaculación prematura y con trastorno del deseo sexual hi-
Baja autoestima, poca confianza en sí mismos y sentimien- poactivo en el varón.
to de masculinidad disminuido. Afecto deprimido. Temor o
evitación de los encuentros sexuales. Disminución de la satis- REPERCUSIONES FUNCIONALES
facción y el deseo en la pareja. Puede interferir con la fertilidad y producir sufrimiento per-
sonal e interpersonal.
Deben considerarse:
1) factores de pareja. COMORBILIDAD
2) factores de la relación. Con otros diagnósticos de tipo sexual (eyaculación prema-
3) factores de vulnerabilidad individual, comorbilidad tura, deseo sexual hipoactivo), trastornos de ansiedad y de-
psiquiátrica y factores de estrés. presivos. Más frecuente en los hombres con síntomas de vías
4) factores culturales o religiosos. urinarias inferiores relacionados con la hipertrofia de prós-
5) factores médicos relevantes. tata.

PREVALENCIA
- Importante incremento asociado a la edad a partir
de los 50 años. TRASTORNO ORGÁSMICO FEMENINO
- 13-21% de varones con edades entre 40 y 80 años.
- 40-50% de los varones de más de 60-70 años. CARACTERÍSTICAS DIAGNÓSTICAS
- 8% presentaron problemas en su primera experien Dificultad para experimentar el orgasmo o marcada dismi-
cia. nución de la intensidad de las sensaciones. Deben estar pre-
sentes en todas o casi todas las ocasiones (75-100%) de la
DESARROLLO Y CURSO actividad sexual (en situaciones concretas o generalizadas),
Escasos indicios de persistencia de problemas después del y tiene una duración mínima de “unos” 6 meses (queda a
primer intento. La mayoría remite espontáneamente sin in- criterio clínico).
tervención profesional, algunos siguen teniendo problemas
episódicos. No se diagnostica cuando:
De por vida: asociado a factores psicológicos autolimitados o - Factores interpersonales o contextuales significati-
que responden a intervenciones psicológicas. vos (alteración grave de la relación, violencia u otras estre-
Adquirido: relacionado con factores biológicos y persistente. santes).
Aumenta la incidencia con la edad. Mayor sufrimiento en los - Si experimenta orgasmo a través de estimulación del
más jóvenes. clítoris, pero no durante el coito
- Si se debe a una estimulación sexual inadecuada.
FACTORES DE RIESGO Y PRONÓSTICO
Temperamento: Rasgos neuróticos en estudiantes universi- Características que apoyan el diagnóstico:
tarios; rasgos sumisos en hombres de 40 o más. Alexitimia - Mayores dificultades de comunicación en cuestio-
común en disfunción eréctil “psicógena”. Comunes en de- nes sexuales.
presión y TEPT. - La satisfacción sexual global no se relaciona con la
Trastorno eréctil adquirido: edad, tabaquismo, falta de ejer- experimentación del orgasmo.
cicio físico, diabetes y deseo disminuido. - Aparecen a menudo junto a problemas relacionados
con el interés y la excitación sexuales.
ASPECTOS RELACIONADOS CON LA CULTURA Deben considerarse:
Varían en los diferentes países. Se desconoce si representan 1) factores de pareja.
diferencias en las experiencias culturales o diferencias en la 2) factores de la relación.
frecuencia. 3) factores de vulnerabilidad individual, comorbilidad
psiquiátrica y factores de estrés.
DIAGNÓSTICO DIFERENCIAL 4) factores culturales o religiosos.
- Trastornos mentales no sexuales: Depresión mayor y 5) factores médicos relevantes.
trastorno eréctil muy relacionados.
- Uso de sustancias/medicamentos: comienza con el PREVALENCIA
uso y desaparece con el cese el medicamento/sustancia. 10-42%, no tiene en cuenta el malestar. Solo una parte pre-
- Otra afección médica: si tiene más de 40-50 años o senta malestar asociado. 10% nunca experimenta orgasmo.
tiene problemas médicos concomitantes, el diagnóstico di-

71
DESARROLLO Y CURSO pueden variar de forma marcada en relación a edad, entorno
Primer orgasmo muy variable. Con la edad aumenta el nú- cultural, duración y presencia de malestar asociado.
mero de mujeres que refiere haber experimentado un or-
gasmo. DESARROLLO Y CURSO
“de por vida” o “adquirido”. Puede haber cambios adapta-
FACTORES DE RIESGO Y PRONÓSTICO tivos normales que pueden ser transitorios.
Temperamento: ansiedad o preocupación por el embarazo
interfieren. FACTORES DE RIESGO Y PRONÓSTICO
Ambientales: factores socioculturales, problemas de rela- Temperamento: cogniciones y actitudes negativas acerca de
ción, salud física y salud mental asociados a dificultades or- la sexualidad y antecedentes de trastornos mentales. Dife-
gásmicas. rencias en la propensión a excitarse sexualmente y la inhibi-
Genéticos y fisiológicos: múltiples afecciones médicas y me- ción sexual pueden predecir la probabilidad.
dicamentos. Menopausia asociada consistentemente. Ambientales: Dificultades en la relación, funcionamiento se-
xual de la pareja, historia de desarrollo y relacione tempranas
ASPECTOS RELACIONADOS CON LA CULTURA con cuidadores o factores estresantes en la infancia.
Marcadas diferencias socioculturales y generacionales. Genéticos y fisiológicos: Múltiples afecciones médicas y me-
dicamentos. Menopausia asociada consistentemente.
DIAGNÓSTICO DIFERENCIAL
- Trastornos mentales no sexuales: depresión mayor ASPECTOS RELACIONADOS CON LA CULTURA
asociada a disminución del interés o del placer por todas o Marcada variabilidad en la prevalencia en diferentes culturas.
casi todas las actividades, que pueden explicar. Si lo explican,
no se diagnostica. ASPECTOS RELACIONADOS CON EL GÉNERO
- Otra afección médica: si lo explica no se diagnostica. Solo se realiza en mujeres. En varón: trastorno del deseo se-
- Factores interpersonales: si lo explican no se diag- xual hipoactivo en el varón.
nostica.
- Otra disfunción: puede aparecer asociado a otras. DIAGNÓSTICO DIFERENCIAL
La presencia de otra disfunción sexual no descarta el diag- - Trastorno de depresión mayor: La disminución im-
nóstico de trastorno orgásmico femenino. portante del interés o del placer por todas o casi todas las
actividades la mayor parte del día, casi todos los días, podría
COMORBILIDAD explicar la falta de interés/excitación
Pueden tener dificultades concurrentes en el interés/excita- - Otra afección médica: Si lo explica no se diagnosti-
ción sexual. ca.
- Factores interpersonales: si lo explican no se diag-
nostica.
- Otra disfunción: La presencia de otra disfunción se-
TRASTORNO DEL INTERÉS/EXCITACIÓN SE- xual no descarta el diagnóstico. Es común la presencia de
XUAL FEMENINO más de una disfunción sexual. Ej.: la presencia de dolor pue-
de llevar a la falta de deseo.
CARACTERÍSTICAS DIAGNÓSTICAS - Estímulos sexuales inadecuados o ausentes no se
3/6 indicadores durante mínimo 6 meses. Se debe tener en haría el diagnóstico.
cuenta el contexto interpersonal, la discrepancia en el deseo
con su parea no es suficiente para el diagnóstico. Puede ha- REPERCUSIONES FUNCIONALES
ber diferentes perfiles sintomáticos, así como variabilidad en Menor satisfacción con las relaciones.
la manera de expresar el interés y la excitación sexuales.
COMORBILIDAD
CARACTERÍSTICAS QUE APOYAN EL DIAGNÓSTICO Muy común con otras dificultades sexuales. El malestar se-
Dificultades para experimentar el orgasmo, dolor durante la xual y la insatisfacción con la vida sexual también están muy
actividad sexual, actividad sexual infrecuente y discrepancias relacionada con el deseo sexual disminuido de las mujeres.
de deseo en la pareja. Las dificultades en la relación y los El deseo sexual disminuido causante de malestar se asocia
trastornos del estado de ánimo suelen asociarse. Expectati- a la depresión, problemas tiroideos, ansiedad, incontinencia
vas y estándar poco realistas, técnicas sexuales pobres y falta urinaria y otros factores médicos. También se asocia a artri-
de información pueden ser evidentes. tis, enfermedad inflamatoria intestinal y colon irritable.
Parece presentar comorbilidad con depresión, abusos sexua-
Deben considerarse: les y físicos en la edad adulta, el funcionamiento mental glo-
1) factores de pareja. bal y el consumo de alcohol.
2) factores de la relación.
3) factores de vulnerabilidad individual, comorbilidad
psiquiátrica y factores de estrés.
4) factores culturales o religiosos. TRASTORNO DEL DOLOR GÉNITO-PÉLVICO/
5) factores médicos relevantes. PENETRACIÓN
PREVALENCIA CARACTERÍSTICAS DIAGNÓSTICAS
Se desconoce. La prevalencia del deseo sexual disminución Hace referencia a 4 diferentes dimensiones de síntomas que
y de los problemas de excitación sexual (DSM-IV o CIE-10) a menudo presentan comorbilidad entre sí: 1) dificultades

72
para mantener relaciones sexuales; 2) dolor génito-pélvico, ASPECTOS RELACIONADOS CON EL GÉNERO
3) temor al dolor o a la penetración vaginal y 4) tensión en Solo puede diagnosticarse en mujeres por definición. Hay
los músculos del suelo pélvico. investigaciones que sugieren que los varones pueden sufrir
El diagnóstico se podría hacer en base a las dificultades en algunos problemas similares. Se pueden utilizar los diagnós-
una sola de las dimensiones sintomáticas, pero deben eva- ticos de otra disfunción sexual especificada o de disfunción
luarse las cuatro dimensiones. sexual no especificada para los hombres que parezcan cum-
plir este patrón.
CARACTERÍSTICAS QUE APOYAN EL DIAGNÓSTICO
Frecuentemente asociado con otras disfunciones sexuales, DIAGNÓSTICO DIFERENCIAL
en especial deseo e interés sexual disminuidos. A veces el - Otra afección médica: En algunos casos tratar la
deseo y el interés están conservados en situaciones no do- afección médica presente puede aliviar el trastorno de dolor,
lorosas, o que no requieren penetración. Suele haber con- pero en gran parte de las veces no es así. Las afecciones mé-
ductas de evitación de las situaciones y de las oportunidades dicas asociadas son difíciles de diagnosticar y de tratar.
sexuales. Evitación de las exploraciones ginecológicas a pesar - Trastorno de síntomas somáticos y trastornos rela-
de las recomendaciones médicas. Patrón de evitación similar cionados: no está claro si ambos pueden ser diferenciados de
a los trastornos fóbicos. Es frecuente que acudan a buscar manera fiable. Algunas mujeres con diagnóstico de trastorno
tratamiento únicamente cuando desean concebir. Muchas de dolor génito-pélvico / penetración también serán diag-
sufren problemas de relación/maritales asociados; también nosticadas de una fobia específica.
síntomas refieren sentimientos de feminidad disminuidos. - Estímulos sexuales inadecuados: se deben evaluar,
en algunas situaciones puede no ser apropiado el diagnósti-
Deben considerarse: co
1) factores de pareja.
2) factores de la relación. REPERCUSIONES FUNCIONALES
3) factores de vulnerabilidad individual, comorbilidad Insatisfacción de las relaciones y dificultades para concebir.
psiquiátrica y factores de estrés.
4) factores culturales o religiosos. COMORBILIDAD
5) factores médicos relevantes. La comorbilidad con otras dificultades sexuales parece ser
común. También con los conflictos en la relación. Es proba-
PREVALENCIA ble que haya una mayor prevalencia de otros trastornos re-
Se desconoce. Aproximadamente el 15% de mujeres de lacionados con el suelo pélvico o con los órganos reproduc-
América del Norte refiere dolor recurrente durante las re- tores. Parece presentar comorbilidad con depresión, abusos
laciones sexuales. Las dificultades para mantener relaciones sexuales y físicos en la edad adulta, el funcionamiento men-
sexuales parecen ser un motivo frecuente de derivación a las tal global y el consumo de alcohol.
consultas de disfunción sexual y a los especialistas clínicos.

DESARROLLO Y CURSO TRASTORNO DEL DESEO SEXUAL HIPOACTI-


El desarrollo y curso no está claro. Puesto que no suelen VO EN EL VARÓN
buscar tratamiento hasta que experimentan problemas en
el funcionamiento sexual, puede ser difícil caracterizar como CARACTERÍSTICAS DIAGNÓSTICAS
de por vida (primario) o adquirido (secundario). Debe tenerse en cuenta el contexto interpersonal, una “dis-
Aunque acuden a recibir atención clínica después de haber crepancia en el deseo” no es suficiente para el diagnóstico.
iniciado la actividad sexual, a menudo hay signos clínicos
más tempranos: dificultades para el uso de tampones. CARACTERÍSTICAS QUE APOYAN EL DIAGNÓSTICO
Las quejas relativas al dolor génito-pélvico muestran picos Problemas de erección o eyaculación. Ej.: las dificultades
en la edad adulta temprana y en el período peri y posme- para conseguir una erección pueden llevar a que el hombre
nopáusico. También puede haber un aumento en el período pierda el interés por la actividad sexual. Escasa iniciativa y
del posparto. receptividad. En ocasiones puede haber actividades sexuales
incluso habiendo un deseo sexual reducido.
FACTORES DE RIESGO Y PRONÓSTICO
Ambientales: Abuso sexual y maltrato físico como factores Deben considerarse:
predictivos de los trastornos sexuales por dolor (dispareunia 1) factores de pareja.
y vaginismo) en DSM-IV. Tema controvertido en la bibliogra- 2) factores de la relación.
fía actual. 3) factores de vulnerabilidad individual, comorbilidad
Genéticos y fisiológicos: Historia previa de infecciones vagi- psiquiátrica y factores de estrés.
nales. El dolor durante la inserción de tampones o la incapa- 4) factores culturales o religiosos.
cidad para insertaros antes de haber tenido contacto sexual 5) factores médicos relevantes.
es un factor de riesgo.
PREVALENCIA
ASPECTOS RELACIONADOS CON LA CULTURA 6% de los varones jóvenes (18-24 años) y el 41% en los ma-
Educación sexual inadecuada y ortodoxia religiosa se han yores (66-74%) tienen problemas en el deseo sexual. La falta
considerado en el pasado como factores culturales predis- de interés persistente (6 meses o más) por el sexo afecta sólo
ponentes en el DSM-IV para vaginismo. Las investigaciones, a una pequeña porción 1,8% (16-44 años).
aunque limitadas, no apoyan esta idea.
DESARROLLO Y CURSO

73
Con la edad es normal la disminución en el deseo sexual. 1) factores de pareja.
2) factores de la relación.
FACTORES DE RIESGO Y PRONÓSTICO 3) factores de vulnerabilidad individual, comorbilidad
Temperamentales: síntomas del estado de ánimo y de an- psiquiátrica y factores de estrés.
siedad son importantes factores predictivos. Hasta ½ de los 4) factores culturales o religiosos.
varones con antecedentes de síntomas psiquiátricos pueden 5) factores médicos relevantes.
tener una pérdida del deseo moderada o grave frente al PREVALENCIA
15% de los que no tienen antecedentes. Factores contex- Las estimaciones varían ampliamente dependiendo de la de-
tuales afectan positiva y negativamente. finición que se utilice. Más del 20-30% de los varones de
Ambientales: uso de alcohol, homofobia hacia sí mismos (en entre 18 y 70 años refiere preocupación acerca de la rapidez
homosexuales), problemas interpersonales, actitudes, falta con la que eyaculan. Con la definición de eyaculación pre-
de educación sexual y traumas debidos a experiencias vitales matura como aquella que tiene lugar antes de aproximada-
tempranas afectan. mente 1 minuto desde la penetración vaginal, sólo el 1-3%
Genéticos y fisiológicos. Trastornos endocrinos, edad, niveles de los hombres sería diagnosticado de este trastorno.
de testosterona o hipogonadismo. La prevalencia puede aumentar con la edad.

ASPECTOS RELACIONADOS CON LA CULTURA DESARROLLO Y CURSO


Marcada variabilidad del 12,8 al 28%´ Algunos hombres pueden experimentar eyaculación prema-
tura durante los primeros encuentros sexuales, pero adquie-
ASPECTOS RELACIONADOS CON EL GÉNERO ren el control eyaculatorio con el tiempo. La persistencia del
En las mujeres los trastornos del deseo y excitación se han problema (más de 6 meses) es lo que determina el diagnós-
unido. En varones se mantiene como constructos diferentes. tico de eyaculación prematura.
Los hombres refieren una intensidad y frecuencia del deseo La eyaculación precoz adquirida probablemente tenga un
significativamente mayor que las mujeres. inicio más tardío, apareciendo habitualmente después de la
cuarta década de vida.
DIAGNÓSTICO DIFERENCIAL La forma de por vida es relativamente estable a lo largo de
- Trastorno de depresión mayor: La disminución im- la vida. Comienza con las experiencias sexuales tempranas y
portante del interés o del placer por todas o casi todas las persiste a lo largo de la vida del individuo.
actividades la mayor parte del día, casi todos los días, podría Se conoce poco acerca de su curso. En un 20% las latencias
explicar la falta de interés/excitación. eyaculatorias se reducen todavía más con la edad. La edad y
- Otra afección médica / uso de sustancias medica- la duración de la pareja se han asociado negativamente a la
mentos: Si lo explica no se diagnostica. prevalencia de la eyaculación prematura.
- Factores interpersonales: si lo explican no se diag-
nostica. FACTORES DE RIESGO Y PRONÓSTICO
- Otra disfunción: La presencia de otra disfunción se- Temperamentales: más común en los hombres con trastor-
xual no descarta el diagnóstico. Hasta la mitad de los hom- nos de ansiedad, especialmente trastorno de ansiedad social.
bres con deseo sexual reducido también tienen problemas Genéticos y fisiológicos: Contribución genética moderada a
de erección, y algunos menos pueden tener, además, dificul- la eyaculación prematura de por vida. La eyaculación pre-
tades de eyaculación precoz. matura puede estar asociada con polimorfismos en el gen
- Si el deseo reducido en un varón se explica por la del transportador de la dopamina o con polimorfismos en el
identificación como “asexual” sobre sí mismo, no se diag- gen del trasportados de la serotonina. La patología tiroidea,
nostica. la prostatitis y la abstinencia de drogas se han asociado con
la eyaculación prematura adquirida. La tomografía por emi-
COMORBILIDAD sión de positrones obtenida durante la eyaculación muestra
Depresión y otros trastornos mentales, así como los factores activación primaria en la zona de transición mesencefálica
endocrinos, frecuentemente presentan comorbilidad. (incluye área tegmental ventral).

ASPECTOS RELACIONADOS CON LA CULTURA


Lo que se considera latencia eyaculatoria norma es diferen-
EYACULACIÓN PREMATURA (PRECOZ) te en muchas culturas. Las latencias eyaculatorias medias
pueden diferir en algunos países. Estas diferencias pueden
CARACTERÍSTICAS DIAGNÓSTICAS explicarse por factores culturales o religiosos, así como por
Eyaculación que tiene lugar antes o poco tiempo después de diferencias genéticas entre las poblaciones.
la penetración vaginal y se concreta con el tiempo de latencia
eyaculatoria (tiempo que transcurre antes de la eyaculación) ASPECTOS RELACIONADOS CON EL GÉNERO
tras la penetración vaginal estimado para el individuo. Las Los hombres y sus parejas sexuales pueden diferir en su per-
estimaciones referidas por el propio individuo son suficientes cepción de lo que constituye una latencia aceptable. Podría
para realizar el diagnóstico. haber una preocupación creciente entre las mujeres acerca
de la eyaculación precoz de sus parejas sexuales, lo que po-
CARACTERÍSTICAS QUE APOYAN EL DIAGNÓSTICO dría ser reflejo de los cambios de actitud de la sociedad hacia
Falta de control sobre la eyaculación. Aprensión por la antici- la actividad sexual de las mujeres.
pación de su incapacidad para retrasar la eyaculación en los
encuentros sexuales futuros. MARCADORES DIAGNÓSTICOS
Deben considerarse: En la investigación, la latencia eyaculatoria habitualmente se

74
monitoriza mediante el uso de un dispositivo para medir el TRASTORNOS PARAFÍLICOS
tiempo por parte de la pareja sexual, aunque esto no es lo
ideal en la vida real. Para el coito vaginal se mide el tiempo
entre penetración vaginal y eyaculación. TRASTORNO DE VOYEURISMO
DIAGNÓSTICO DIFERENCIAL CARACTERÍSTICAS DIAGNÓSTICAS
- Disfunción sexual inducida por sustancias/medica- Si refieren que no sufren malestar, demostrado por ausencia
mentos: Cuando la eyaculación prematura se debe exclusi- de ansiedad, obsesiones, culpa o vergüenza por los impulsos
vamente al uso, intoxicación o abstinencia de sustancias, se parafílicos, no hay deterioro en otras áreas importantes del
debe diagnostica una disfunción sexual inducida por sustan- funcionamiento y sus antecedentes psiquiátricos o legales
cias/medicamentos. indican que no actúa de esa manera, puede afirmarse que
- Problemas eyaculatorios que no cumplen criterios tiene interés sexual de voyeurismo, pero NO deben ser diag-
diagnósticos: Hay que identificar a aquellos con latencias nosticados de trastorno.
normales que desean latencias mayores, y a aquellos que tie- No reconocer el interés o los actos no impide el diagnóstico.
nen eyaculación prematura episódica. Ninguna de estas re- “Recurrentemente” puede ser interpretado, como norma
cibe el diagnóstico de eyaculación prematura, aunque estas general, como la presencia de 3 o más víctimas en diferentes
situaciones pueden producir malestar en algunos hombres. ocasiones. Un número menor puede satisfacer criterios siem-
pre que se espíe a la misma víctima en varias ocasiones o si
COMORBILIDAD hay pruebas que confirman un interés.
La eyaculación prematura puede estar asociada con proble- La existencia de varias víctimas es condición suficiente pero
mas de erección. Puede ser difícil determinar qué dificultad no necesaria (el interés intenso es suficiente)
precedió a cuál. La eyaculación prematura de por vida puede Normal en la adolescencia puberal, por ello, diagnóstico con
asociarse a determinados trastornos de ansiedad. La adquiri- mínimo 18 años.
da puede asociarse con la prostatitis, la patología tiroidea o
PREVALENCIA
la abstinencia de drogas (ej.: opiáceos).
Los más frecuentes para un potencial comportamiento se-
  xual delictivo. Se desconoce la prevalencia, pero se estima
12% hombres y 4% mujeres.

  DESARROLLO Y CURSO
Los hombres se dan cuenta por primera vez del interés en la
adolescencia. La persistencia no está clara.

FACTORES DE RIESGO Y PRONÓSTICO


Temperamentales: El voyeurismo es una condición previa ne-
cesaria para el trastorno. Los factores de riesgo del voyeuris-
mo incrementan las tasas del trastorno.
Ambientales: Abuso sexual en la infancia, abuso de sustan-
cias y preocupación sexual/ hipersexualidad como factores
de riesgo, aunque no se conoce la relación causal.

ASPECTOS RELACIONADOS CON EL GÉNERO


Muy raro en mujeres en contextos clínicos. La proporción
hombres/mujeres que realizan actos aislados podría ser 3:1.

DIAGNÓSTICO DIFERENCIAL
- Trastorno de conducta y trastorno de la personali-
dad antisocial: en estos habría comportamientos añadidos
antisociales y transgresión de normas, que estaría ausente en
el trastorno de voyeurismo.
- Trastorno por consumo de sustancias: pueden en-
trañar episodios voyeurísticos aislados, pero no incluyen el
típico interés sexual por espiar.

COMORBILIDAD
Hipersexualidad y otros trastornos parafílicos, especialmen-
te exhibicionismo. Trastorno depresivo, bipolar, ansiedad y
consumo de sustancias. El TDAH, el trastorno de conducta y
el TP antisocial también son frecuentes.

75
TRASTORNO DE EXHIBICIONISMO funcionamiento y sus antecedentes psiquiátricos o legales
indican que no actúa de esa manera, puede afirmarse que
CARACTERÍSTICAS DIAGNÓSTICAS tiene interés sexual de frotteurismo, pero NO deben ser diag-
Si refieren que no sufren malestar, demostrado por ausencia nosticados de trastorno.
de ansiedad, obsesiones, culpa o vergüenza por los impulsos No reconocer el interés o los actos no impide el diagnóstico.
parafílicos, no hay deterioro en otras áreas importantes del “Recurrente” puede ser interpretado, como norma general,
funcionamiento y sus antecedentes psiquiátricos o legales como la presencia de 3 o más víctimas en diferentes ocasio-
indican que no actúa de esa manera, puede afirmarse que nes. Un número menor puede satisfacer criterios siempre sea
tiene interés sexual de exhibicionismo, pero NO deben ser la misma víctima en varias ocasiones o si hay pruebas que
diagnosticados de trastorno. confirman un interés.
No reconocer el interés o los actos no impide el diagnóstico. La existencia de varias víctimas es condición suficiente pero
“Recurrente” puede ser interpretado, como norma general, no necesaria (el interés intenso es suficiente).
como la presencia de 3 o más víctimas en diferentes ocasio-
nes. Un número menor puede satisfacer criterios siempre sea PREVALENCIA
la misma víctima en varias ocasiones o si hay pruebas que 30% de comportamientos froteuristas de los hombres adul-
confirman un interés. tos en población general. 10-14% de los varones adultos
La existencia de varias víctimas es condición suficiente pero atendidos por problemas parafílicos e hipersexuales. En po-
no necesaria (el interés intenso es suficiente). blación general es desconocida, no es probable que exceda
la prevalencia clínica.
PREVALENCIA
Se desconoce. Se estima que el 2-4% de los varones. Se es- DESARROLLO Y CURSO
tima que en mujeres es muy inferior. Aparecen en la adolescencia o primeras etapas de la edad
adulta. Se conoce poco sobre su persistencia. El aumento
DESARROLLO Y CURSO de la edad puede asociarse a una disminución de comporta-
Aparecen en la adolescencia o primeras etapas de la edad miento y preferencias frotteurista.
adulta. Se conoce poco sobre su persistencia. El aumento
de la edad puede asociarse a una disminución de comporta- FACTORES DE RIESGO Y PRONÓSTICO
miento y preferencias exhibicionistas. Temperamentales: el comportamiento antisocial no sexual
y la hipersexualidad / preocupación sexual pueden ser facto-
FACTORES DE RIESGO Y PRONÓSTICO res de riesgo inespecíficos. El froteurismo es condición pre-
Temperamentales: el exhibicionismo es una condición previa via necesaria del trastorno de froteurismo, de modo que los
necesaria para el trastorno. Los factores de riesgo del exhibi- factores de riesgo del froteurismo incrementan las tasas de
cionismo incrementan las tasas del trastorno. trastorno.
Ambientales: abuso emocional y sexual en la infancia e hi-
persexualidad o preocupaciones sexuales. ASPECTOS RELACIONADOS CON EL GÉNERO
Parece haber sustancialmente menos mujeres con preferen-
ASPECTOS RELACIONADOS CON EL GÉNERO cias sexuales froteuristas que hombres.
Muy inusual en mujeres, aunque pueden producirse episo-
dios aislados, como mucho la mitad en comparación con los DIAGNÓSTICO DIFERENCIAL
hombres. - Trastorno de conducta y trastorno de la personali-
dad antisocial: en estos habría comportamientos añadidos
DIAGNÓSTICO DIFERENCIAL antisociales y transgresión de normas, estaría ausente el in-
- Trastorno de conducta y trastorno de la personali- terés sexual específico.
dad antisocial: en estos habría comportamientos añadidos - Trastorno por consumo de sustancias: pueden en-
antisociales y transgresión de normas, estaría ausente el in- trañar episodios aislados, pero no incluyen el típico interés
terés sexual específico sexual.
- Trastorno por consumo de sustancias: pueden en-
trañar episodios exhibicionistas aislados, pero no incluyen el COMORBILIDAD
típico interés sexual por exponer los genitales. Hipersexualidad y otros trastornos parafílicos, en especial el
trastorno de exhibicionismo y el trastorno de voyeurismo.
COMORBILIDAD También el trastorno de conducta, el trastorno de la per-
Trastornos depresivos, bipolar, de ansiedad y por consumo sonalidad antisocial y los trastornos depresivo, bipolar, de
de sustancias, la hipersexualidad, el trastorno por déficit de ansiedad y por consumo de sustancias.
atención/hiperactividad, otros trastornos parafílicos y el tras-
torno de la personalidad antisocial.

TRASTORNO DE MASOQUISMO SEXUAL


TRASTORNO DE FROTTEURISMO
CARACTERÍSTICAS DIAGNÓSTICAS
CARACTERÍSTICAS DIAGNÓSTICAS Reconocen abiertamente su excitación sexual intensa al ser
Si refieren que no sufren malestar, demostrado por ausencia humillados, golpeados, atados o sometidos a sufrimiento
de ansiedad, obsesiones, culpa o vergüenza por los impulsos de cualquier otra forma, y el trastorno se manifiesta por sus
parafílicos, no hay deterioro en otras áreas importantes del fantasías, deseos sexuales irrefrenables o comportamientos.

76
Si no hay sufrimiento o interferencia, podría haber inclinacio- COMORBILIDAD
nes sexuales masoquistas, pero NO trastorno. Otros trastornos parafílicos.
Apoya el diagnóstico: uso intensivo de pornografía del tipo.
TRASTORNO DE PEDOFILIA
PREVALENCIA
En población general se desconoce. En Australia 2,2% hom- CARACTERÍSTICAS DIAGNÓSTICAS
bres y 1,3% mujeres han practicado actos de servidumbre en Si refieren que no sufren malestar, demostrado por ausencia
los últimos 12 meses. de ansiedad, obsesiones, culpa o vergüenza por los impulsos
parafílicos, no hay deterioro en otras áreas importantes del
DESARROLLO Y CURSO funcionamiento y sus antecedentes psiquiátricos o legales
Edad media de inicio 19,3 años. Fantasías pueden comen- indican que no actúa de esa manera, puede afirmarse que
zar en infancia y adolescencia. Aumento de la edad efecto tiene interés sexual, pero NO deben ser diagnosticados de
reductor. trastorno.
No reconocer el interés o los actos no impide el diagnóstico.
DIAGNÓSTICO DIFERENCIAL La existencia de varias víctimas es condición suficiente pero
Otros trastornos parafílicos, que a la vez son comorbilidades. no necesaria (el interés es suficiente).
Apoya el diagnóstico: uso frecuente de pornografía en la
REPERCUSIONES FUNCIONALES que aparezcan prepúberes.
Riesgo de muerte accidental mientras practican la asfixiofilia
u otras prácticas autoeróticas. PREVALENCIA
En población se desconoce. La máxima prevalencia en pobla-
COMORBILIDAD ción masculina es aproximadamente un 3-5%. En mujeres
Otros parafílicos como fetichismo travéstico. probablemente es una pequeña fracción de la prevalencia
en hombres.

TRASTORNO DE SADISMO SEXUAL DESARROLLO Y CURSO


Pueden referir darse cuenta de su interés por niños en la
CARACTERÍSTICAS DIAGNÓSTICAS pubertad. Se requiere mínimo 16 años y al menos una edad
Si refieren que no sufren malestar, demostrado por ausencia de 5 años más que el niño.
de ansiedad, obsesiones, culpa o vergüenza por los impulsos La pedofilia parece ser un trastorno de por vida. El trastor-
parafílicos, no hay deterioro en otras áreas importantes del no de pedofilia incluye necesariamente otros elementos que
funcionamiento y sus antecedentes psiquiátricos o legales pueden fluctuar, aumentar o disminuir con la edad. Con la
indican que no actúa de esa manera, puede afirmarse que edad parecen disminuir los comportamientos.
tiene interés sexual, pero NO deben ser diagnosticados de
trastorno. FACTORES DE RIESGO Y PRONÓSTICO
No reconocer el interés o los actos no impide el diagnóstico. Temperamentales: interacción entre pedofilia y comporta-
“recurrente” puede ser interpretado, como norma general, miento antisocial, hombres con ambos rasgos es más pro-
como la presencia de 3 o más víctimas en diferentes ocasio- bable que mantengan relaciones sexuales con niños. TP an-
nes. Un número menor puede satisfacer criterios siempre sea tisocial como factor de riesgo para trastorno de pedofilia en
la misma víctima en varias ocasiones o si hay pruebas que hombres con pedofilia.
confirman un interés. Ambientales: objeto de abuso sexual en la infancia. No está
La existencia de varias víctimas es condición suficiente pero clara la relación causal.
no necesaria (el interés intenso es suficiente) Genéticos y fisiológicos: alteración en el neurodesarrollo en
Apoya el diagnóstico: uso intensivo de pornografía del tipo. el útero incrementaría probabilidad.

PREVALENCIA MARCADORES DIAGNÓSTICOS


En población se desconoce. 2-30% en población forense. Medidas psicofisiológicas del interés sexual: pletismografía
Entre los agresores sexuales condenados en EEUU, menos peneana o tiempo de observación.
del 10% presenta sadismo sexual. Entre individuos que han
cometido homicidios con motivación sexual, la tasa de sadis- ASPECTOS RELACIONADOS CON EL GÉNERO
mo sexual varía del 37-75%. Las pruebas psicofisiológicas de laboratorio que miden inte-
En población forense, casi exclusivamente hombres. Austra- rés sexual y que son útiles en hombres, no parecen serlo en
lia indicó 2,2% hombres y el 1,3% mujeres afirmaba haber mujeres.
practicado servidumbre y castigo.
DIAGNÓSTICO DIFERENCIAL
FACTORES DE RIESGO Y PRONÓSTICO - TP Antisocial: Incrementa la probabilidad de que
Mujeres se dan cuenta de su orientación sadomasoquista en una persona en principio atraída por el físico maduro se acer-
una edad adulta temprana, en hombres edad media 19,4 que a los niños aprovechándola disponibilidad. Muestra a
años. Edad efecto reductor. menudo otros signos del trastorno de personalidad.
- Trastornos por consumo de alcohol y de sustancias:
DIAGNÓSTICO DIFERENCIAL el efecto desinhibitorio de la intoxicación puede incrementar
Otros trastornos parafílicos, que a la vez son comorbilidades. la probabilidad de que personas a la que les atraiga el físico
maduro se acerquen sexualmente a niños.
- Trastorno obsesivo compulsivo: quejas de pen-

77
samientos egodistónicos y preocupación por una posible torno de travestismo. Cuando la fantasía es “travestirse de
atracción hacia los niños. La entrevista revela ausencia de forma forzada” y se excita sobre todo por la dominación o
pensamientos sexuales relacionados con niños en momentos humillación asociada el diagnóstico es de masoquismo.
de excitación sexual y a veces se añaden ideas sexuales ego- - Comportamiento fetichista sin trastorno de fetichis-
distónicas intrusivas. mo: presencia de objeto fetiche, pero sin malestar ni reper-
cusión.
COMORBILIDAD -
La comorbilidad psiquiátrica incluye trastorno por consumo COMORBILIDAD
de sustancias, trastorno depresivo, bipolar y de ansiedad, los Otros trastornos parafílicos.
TP antisocial y otros trastornos parafílicos.

TRASTORNO DE TRAVESTISMO
TRASTORNO DE FETICHISMO
CARACTERÍSTICAS DIAGNÓSTICAS
CARACTERÍSTICAS DIAGNÓSTICAS No se aplica a todos los individuos que visten del sexo opues-
Uso repetitivo y persistente o dependencia de objetos in- to. Se aplica a los individuos cuyo travestismo o pensamiento
animados, o gran interés específico por ciertas partes del de travestirse se acompaña a menudo o siempre de excita-
cuerpo (típicamente no genitales) como elementos primarios ción sexual.
asociados a la excitación sexual. Pueden utilizar sólo 1 o 2 prendas de vestir (ej.: ropa interior
Objetos fetiches frecuentes: ropa interior femenina, zapatos femenina), o vestirse completamente con atuendos del sexo
de hombre o de mujer, artículos de goma, ropa de curo y opuesto.
otros tipos de prendas de vestir. Es habitual el patrón de conducta de “adquirir y descartar”,
Partes del cuerpo: pies, dedos del pie y cabello. comprar ropa femenina, se deshacen de ella debido al ma-
No es inusual que se incluyan ambas clases, objetos inanima- lestar y el intento de superar su deseo de travestirse, pero
dos y parte del cuerpo (ej.: calcetines sucios y pies). enseguida vuelve a adquirir vestimenta de mujer.
Reincorpora el PARCIALISMO (interés específico por una par-
te concreta del cuerpo). CARACTERÍSTICAS QUE APOYAN EL DIAGNÓSTICO
En hombres se acompaña a menudo de AUTOGINOFILIA
CARACTERÍSTICAS QUE APOYAN EL DIAGNÓSTICO (tendencia parafílica a sentirse sexualmente excitados por
Puede ser una experiencia multisensorial que incluya aga- pensamientos o imágenes de uno mismo como mujer). Pue-
rrar, degustar, frotar, introducir u oler el objeto mientras se den centrarse en la idea de tener funciones fisiológicas fe-
lleva a cabo la masturbación, o preferir que el compañero meninas (lactancia, menstruación), de realizar acciones de
sexual vista o utilice un objeto fetiche durante los encuentros mujer estereotipadas (hacer punto) o de poseer anatomía
sexuales. Pueden llegar a reunir gran colección de objetos femenina (pechos).
fetiche.
PREVALENCIA
DESARROLLO Y CURSO Desconocida. Raro en hombres, extremadamente raro en
Normalmente las parafilias se inician durante la pubertad, mujeres. Menos del 3% de hombres refiere haberse excitado
pero los fetiches pueden desarrollarse antes de la adoles- al haber vestido atuendos de mujer.
cencia. Curso continuado que fluctúa en frecuencia e inten- La mayoría se identifica como heterosexuales, aunque algu-
sidad. nos de ellos mantienen relaciones sexuales esporádicas con
otros hombres, especialmente cuando están travestidos.
ASPECTOS RELACIONADOS CON LA CULTURA
Distinguir de los aspectos normativos. DESARROLLO Y CURSO
Los primeros síntomas pueden aparecer en la infancia (fuerte
ASPECTOS RELACIONADOS CON EL GÉNERO fascinación por algún atuendo femenino en particular). An-
No se ha comunicado de forma sistemática su presencia en tes de la pubertad, travestirse produce un sentimiento gene-
mujeres. En muestras clínicas se ha descrito casi exclusiva- ralizado de excitación placentera. En la pubertad, el vestirse
mente en hombres. de mujer empieza a provocar erecciones del pene y en algu-
nos casos lleva a la primera eyaculación.
DIAGNÓSTICO DIFERENCIAL En muchos casos, el acto de travestirse conlleva menos ex-
- Trastorno de travestismo: El trastorno de fetichismo citación sexual con la edad, dejando de producir erección.
no se diagnostica cuando los objetos fetiche se limita a artí- Pero, al mismo tiempo, el deseo de travestirse a menudo
culos de ropa que se llevan exclusivamente durante el acto permanece o se hace más intenso. La excitación se sustituye
de travestirse o cuando el objeto estimula genitalmente por- por sensación de confort y bienestar.
que ha sido diseñado con ese propósito (ej.: vibrador). En algunos casos el trastorno es continúo y en otros episó-
- Disfunción sexual en una relación de pareja román- dico. Suele disminuir, de forma temporal, al enamorarse por
tica: cuando el objeto fetiche (o partes del cuerpo) son in- primera vez de una mujer y empezar la relación.
accesibles durante la estimulación previa al coito o durante En algunos casos progresa a disforia de género. El desarrollo
éste. de la disforia de género se acompaña de una reducción o
- Trastorno de masoquismo sexual u otros trastornos eliminación de la excitación sexual asociada a travestirse.
parafílicos: los fetiches pueden concurrir con otros trastor- La gravedad del trastorno es mayor en la edad adulta, pu-
nos parafílicos, especialmente el sadomasoquismo y el tras- diendo causar conflictos de rendimiento en las relaciones

78
sexuales heterosexuales y en los deseos de casarse y fundar DISFORIA DE GÉNERO
una familia.
Los hombres mayores y de mediana edad con antecedentes
de travestismo presentan con menor probabilidad el trastor-
DISFORIA DE GÉNERO
no de travestismo que la disforia de género.
CARACTERÍSTICAS DIAGNÓSTICAS
Incongruencia marcada entre el sexo que les ha sido asigna-
DIAGNÓSTICO DIFERENCIAL
do y el sexo que sienten o expresan.
- Trastorno de fetichismo: la distinción depende de
El sexo que se experimenta puede incluir identidades alter-
los pensamientos específicos del individuo durante la mas-
nativas de sexo más allá de los estereotipos binarios. Se ma-
turbación llevando prendas femeninas y la presencia de otros
nifiesta de modo diferente en los diferentes grupos de edad:
fetiches.
- Disforia de género: los individuos con trastorno de
- Chicas prepuberales: pueden expresar el deseo de ser un
travestismo no refieren contradicciones entre el género que
chico, afirmar que son un chico o afirmar que cuando crez-
experimentan y el asignado, ni deseos de pertenecer al otro
can serán un hombre. Prefieren las ropas y el estilo de pelo
género, y no suelen tener antecedentes de comportamiento
de los chicos, son percibidas por los desconocidos como chi-
de travestismo en la infancia. Si se cumplen todos los crite-
cos y puede que pidan ser llamadas por el nombre de un
rios de ambos, se pueden recibir ambos diagnósticos.
chico. Manifiestan intensas reacciones negativas hacia los
intentos parentales de hacerles llevar vestidos o atuendos
REPERCUSIONES FUNCIONALES
femeninos. Algunas pueden rehusar ir a la escuela o a actos
Pueden interferir o desvirtuar las relaciones heterosexuales.
sociales donde se requieran tales vestimentas. Pueden mos-
trar una marcada identificación con el otro sexo en juegos
COMORBILIDAD
de rol, sueños y fantasías. Prefieren a menudo los depor-
Otras parafilias. Las más frecuentes con el fetichismo y el ma-
tes de contacto, los juegos de pelea, los juegos masculinos
soquismo. La asfixia autoerótica (forma de masoquismo), se
tradicionales y jugar con compañeros masculinos. Muestran
asocia al travestismo en una proporción sustancial de casos
poco interés en juguetes o actividades que se basan en los
mortales. estereotipos femeninos. Ocasionalmente se niegan a orinar
  en posición sedente. Algunas pueden manifestar el deseo
de tener pene, o asegurar que tienen pene, o que cuando
crezcan lo tendrán. También pueden afirmar que no quieren
desarrollar el pecho ni menstruar.

- Chicos prepuberales: pueden expresar el deseo de ser chi-


ca, afirmar que son una chica o que cuando crezcan serán
una mujer. Su preferencia es vestir con ropas de chicas o
de mujeres y pueden improvisar vestidos con lo que tengan
a mano. Juegan a hacer el papel de figuras femeninas y se
interesan por las fantasías femeninas. Prefieren actividades
que son estereotipos tradicionales femeninos y hacen dibu-
jos femeninos y ven los programas de televisión o videos de
personajes que prefieren las chicas. Las muñecas que son
estereotipos femeninos (Barbie) son a menudo los juegos
favoritos, y las chicas sus compañeras de juego. Evitan los
juegos de peleas y deportes competitivos y tienen poco inte-
rés en los juguetes que son estereotipos masculinos. Pueden
fingir que no tienen pene e insisten en sentarse para orinar.
Con menos frecuencia, encuentran su pene o sus testículos
desagradables, quieren quitárselos o que tienen o quieren
tener una vagina.

- Jóvenes adolescentes: las características clínicas se aseme-


jan a las de los niños o a las de los adultos con este trastorno,
dependiendo del nivel de desarrollo. Si no se han desarro-
llado aún las características sexuales secundarias pueden no
manifestar disgusto sobre ellos mismos, aunque están preo-
cupados por los próximos cambios físicos.

- Adultos con disforia de género. Se acompaña a menudo,


no siempre, de deseo de liberarse de sus características se-
xuales primarias o secundarias y de adquirir algunas de las
características sexuales del otro sexo. Pueden adoptar la
conducta, el vestido y los manierismos del sexo que se ex-
perimenta. Se sienten incómodos al ser considerados por los
otros, o al funcionar en la sociedad, como miembros del sexo

79
que tienen asignado. Pueden tener un fuerte deseo de ser Casi todos en los que persiste la Disforia de género se sienten
de un sexo diferente y ser tratados como tal, y pueden tener atraídos sexualmente por sujetos de su sexo de nacimiento.
la suficiente certeza interior como para sentir y responder En varones donde no persiste la Disforia de género presen-
de acuerdo con el sexo que experimentan sin buscar un tra- tan androfilia (63-100%). En mujeres donde no persiste la
tamiento médico para alterar sus características corporales. Disforia de género el porcentaje de ginefilia es del 32-50%.
Pueden encontrar otras formas de resolver la incongruencia:
vivir parcialmente en el rol deseado o adoptar un rol sexual Dos trayectorias para el desarrollo de la Disforia de género:
que no es ni convencionalmente femenino ni masculino.
1) Comienzo temprano. Se inicia en la infancia y con-
CARACTERÍSTICAS QUE APOYAN EL DIAGNÓSTICO tinúa hasta la adolescencia y la edad adulta, o hay un pe-
Los chicos puberales se afeitan las piernas al primer signo de ríodo intermedio en el que la Disforia de género cesa y los
crecimiento del vello, esconden los genitales para que sean sujetos se identifican como homosexuales, seguido de una
menos visibles las erecciones. recurrencia de la Disforia de género. Suelen sentirse atraídos
Las chicas puberales esconden el pecho, caminan encorva- sexualmente casi siempre por hombres. En nacidos varones
das o usan jerséis sueltos. buscan atención clínica para tratamiento hormonal y cirugía
Los adolescentes pueden solicitar u obtener sin prescripción de reasignación a edades más tempranas que los del grupo
ni supervisión médica, supresores hormonales (bloqueantes) de comienzo tardío. En nacidos mujeres, el curso más común
de los esteroides gonadales. Quien a menudo tratamiento es la forma de comienzo temprano. Las nacidas mujeres pre-
hormonal y muchos desean cirugía de reasignación de sexo. sentan casi siempre ginefilia
Los adolescentes que viven en un entorno que les acepta 2) Comienzo tardío. Ocurre en la pubertad o en la
pueden expresar abiertamente el deseo de ser y de ser tra- edad adulta. Algunos refieren que han tenido deseos de ha-
tados como el sexo que experimentan, y pueden vestirse ber sido del otro sexo en la infancia y que no los expresaron
conforme al sexo experimentado, llevar peinados típicos de verbalmente a otras personas. Otros no recuerdan signos de
dicho sexo, buscar amigos preferentemente entre los com- Disforia de género en la infancia. Con frecuencia se invo-
pañeros del otro sexo y adoptar un nombre. lucran en conductas travestis tas con excitación sexual. La
Los adolescentes mayores, cuando son sexualmente activos, mayoría tiene ginefilia, o se sienten atraídos sexualmente por
no muestran ni permiten que las parejas toquen sus órganos otros nacidos varones y con Disforia de género de comienzo
sexuales. tardío tras la transición. Un porcentaje considerable cohabi-
En los adultos con aversión hacia sus genitales, la actividad ta o está casado con sujetos nacidos mujeres. Después de
sexual está marcada por la preferencia de que sus parejas no la transición muchos se identifican como lesbianas. Puede
vean ni toquen sus genitales. Algunos adultos pueden bus- haber más fluctuaciones en el grado de disforia y pueden
car tratamiento hormonal y cirugía de reasignación de sexo. ser más ambivalentes respecto a la cirugía, siendo menos
Otros se encuentran satisfechos con uno u otro. probable que queden satisfechos tras la misma. Esta formó
Adolescentes y adultos con Disforia de género antes de la re- es mucho menos común en las nacidas mujeres que en los
asignación tienen un riesgo aumentado de ideación suicida, nacidos varones.
intentos de suicidio y suicidio. Tras la reasignación, la adap-
tación puede variar y el riesgo de suicidio puede persistir. Disforia de género con trastorno del desarrollo sexual: La ma-
yoría que desarrolla Disforia de género ha acudido en busca
PREVALENCIA de atención médica a edades tempranas. Los trastornos de
Adultos nacidos varones: 0,005-0.014%. Adultas nacidas desarrollo sexual por lo general se asocian con una conducta
mujeres: 0,002-0.003%. Cifras probablemente subestima- sexual atípica que comienza en la infancia temprana, pero en
das porque no acuden a consulta. la mayoría de los casos no conduce a la Disforia de género.
Las diferencias de sexo en población clínica varían por grupo
de edad: 1) niños v:m entre 2:1 y 4.5:1. 2) adolescentes: FACTORES DE RIESGO Y PRONÓSTICO
igual hombres y mujeres. 3) adultos: más varones, en pro- Temperamentales: es posible que un alto grado de atipici-
porción 1:1 – 6,1:1. dad en la conducta sexual en la edad preescolar haga que el
desarrollo de la disforia de género y su persistencia sea más
DESARROLLO Y CURSO probable.
La expresión es diferente con la edad, por lo que los criterios Ambientales: en hombres con Disforia de género es más
son diferentes. frecuente que tengan hermanos mayores varones. En sujetos
Niños: criterios más concretos y conductuales. Se apoyan en de comienzo tardío, factores predisponentes son el traves-
las diferencias de comportamiento de sexo. En niños meno- tismo fetiche habitual con autoginofilia y otras formas de
res es menos probable la expresión de disforia anatómica problemas sociales, psicológicos o del desarrollo.
persistente. Genéticos y fisiológicos:
Adolescentes y adultos: la incongruencia entre el sexo que 1) En Disforia de género sin un trastorno de desarrollo
se experimenta y el somático es un rasgo central del diag- sexual:
nóstico. - Contribución genética (datos débiles): Mayor con-
Los factores relacionados con el malestar y el deterioro va- cordancia Gemelos Monocigóticos vs. Dicigóticos.
rían también con la edad. - Hallazgos endocrinológicos: no se han encontrado
alteraciones sistémicas endógenas en los niveles de hormo-
Las tasas de persistencia de la disforia de género desde la nas sexuales en los sujetos XY con 46 cromosomas, mientras
infancia a la adolescencia o la edad adulta varían: que parece que existen niveles aumentados de andrógenos
- Nacidos varones: 2,2-30%. en los sujetos XX con 46 cromosomas. Evidencia insuficiente.
- Nacidas mujeres: 12-50%. 2) En Disforia de género asociada a trastornos del de-

80
sarrollo sexual: La probabilidad de Disforia de género incre- TRASTORNOS DEL SUEÑO-VIGILIA
menta si la producción prenatal y la utilización de andró-
genos son extremadamente atípicos en comparación con lo
que normalmente se ve en los sujetos del mismo sexo asig- TRASTORNO DE INSOMNIO
nado.
CARACTERÍSTICASDIAGNÓSTICAS
ASPECTOS RELACIONADOS CON LA CULTURA Insatisfacción con la cantidad o calidad del sueño con quejas
Se ha descrito un equivalente a la disforia de género en suje- sobre la dificultad para conciliar (insomnio inicial) o mantener
tos que viven en culturas con otras categorías sexuales insti- el sueño (insomnio medio), o despertar temprano con inca-
tucionalizadas distintas del hombre y la mujer. pacidad para volver a dormir (insomnio tardío). La dificultad
del sueño se produce al menos 3 noches/semana, durante 3
DIAGNÓSTICO DIFERENCIAL meses, genera malestar, y no se explica mejor por otro tras-
- Inconformidad con los roles sexuales: no es sufi- torno del sueño, otro trastorno mental u organicidad.
ciente el rechazo a los estereotipos. Conlleva tanto alteraciones diurnas como nocturnas (fatiga,
- Trastorno de travestismo: si hay este más Disforia somnolencia diurna).
de género se diagnostican ambos. La dificultad para mantener el sueño es el síntoma indivi-
- Trastorno dismórfico corporal: en estos se percibe dual más frecuente, seguido por la dificultad para quedarse
una parte de modo anormal, sin relación con la represen- dormido, aunque la combinación de estos suele ser lo más
tación del sexo asignado. Si se dan ambos, se diagnostican habitual. El tipo de insomnio específico suele variar con el
ambos. tiempo.
- Esquizofrenia y psicóticos: son raros los delirios de El sueño no reparador constituye un síntoma frecuente que
pertenencia a otro sexo distinto. En ausencia de síntomas suele acompañar a la dificultad para iniciar o mantener el
psicóticos, la insistencia de un sujeto de que es de otro sexo sueño o aparecer en solitario con menor frecuencia.
distinto no se considera delirio. Pueden concurrir. Los síntomas se pueden cuantificar a través del autoinforme
retrospectivo del sujeto, de los diarios de sueño de la actigra-
REPERCUSIONES FUNCIONALES fía o de la polisomnografía, aunque el diagnóstico se basa
Las preocupaciones con deseos de cambio de sexo pueden en la percepción subjetiva de la persona sobre el sueño o la
aparecer a cualquier edad tras los primeros 2-3 años e inter- información de un cuidador.
fieren en las actividades diarias. El insomnio persistente puede ser un factor de riesgo para la
Puede haber aislamiento social y malestar. Pueden negarse a depresión y es un síntoma residual frecuente después de su
ir al colegio por burlas, acoso o presión para vestir del sexo tratamiento.
asignado.
En adultos y adolescentes son frecuentes las dificultades en Criterios de gravedad:
las relaciones, incluidas las sexuales, puede deteriorarse el - Insomnio de conciliación: latencia subjetiva del sue-
funcionamiento escolar o laboral. ño > 20-30 minutos.
La disforia y la expresión atípica del sexo se asocian a latos - Insomnio de mantenimiento: después de haberse
niveles de estigmatización, discriminación y victimización, lo conciliado el sueño, período de tiempo subjetivo despierto
que genera autoconcepto negativo, altas tasas de trastornos de > 20-30 minutos.
mentales, abandono escolar y marginación económica (des- - Despertar precoz: despertarse ≥ 30 minutos antes
empleo, riesgos sociales y mentales). Puede haber problemas de la hora programada y antes de que el tiempo total de
para el acceso a servicios de salud y salud mental por barre- sueño alcance las 6 horas y media.
ras estructurales (desagrado institucional, inexperiencia).
CARACTERÍSTICASQUE APOYAN EL DIAGNÓSTICO
COMORBILIDAD Activación fisiológica y cognitiva. Factores de condiciona-
En niños: Problemas emocionales y conductuales (ansiedad, miento que interfieren con el sueño.
trastornos disruptivos del control de los impulsos y depresi- La preocupación por el sueño y por el malestar por la difi-
vos). cultad para dormir puede conducir a un círculo vicioso: más
Niños prepuberales: el aumento de la edad se asocia con esfuerzo por dormir, más frustración, más dificultad para
más problemas emocionales y conductuales. dormir.
El trastorno del espectro autista es más prevalente en los Hábitos de sueño desadaptativos, cogniciones desadaptati-
niños con disforia de género que se derivan en la clínica que vas.
entre la población general. Quejas y síntomas diurnos: fatiga, disminución de energía,
Los adolescentes en muestra clínica parecen tener trastornos alteraciones del ánimo (irritabilidad, labilidad afectiva y sín-
mentales comórbidos, los más frecuentes ansiedad y depre- tomas de ansiedad y/o depresión).
sión. También es más prevalente el trastorno del espectro
autista en la muestra clínica que en población general. PREVALENCIA
En adultos de muestra clínica más problemas de ansiedad y Síntomas de insomnio: 1/3 adultos.
Alteraciones durante el día: 10-15%.
depresión.
Trastorno de insomnio: 6-10%. Trastorno del sueño más
  prevalente.
En atención primaria: 10-20% tiene síntomas significativos.
Mayor prevalencia en mujeres que en hombres (1,44:1).
Frecuentemente asociado a otra afección médica u otro tras-
torno mental (el 40-50% de sujetos con insomnio también

81
presenta otro trastorno mental). bilidad diurna y mala concentración). Mayor riesgo de tras-
torno depresivo mayor, hipertensión e infarto de miocardio.
DESARROLLO Y CURSO Mayor absentismo y menor productividad en el trabajo, me-
Inicio en la edad adulta temprana. nor calidad de vida y más problemas económicos.
En las mujeres, puede aparecer durante la menopausia y per-
sistir tras la desaparición de otros síntomas. Puede iniciarse DIAGNÓSTICO DIFERENCIAL
en etapas tardías de la vida, junto a otros problemas de sa- - Variaciones normales del sueño: ausencia de dificul-
lud. tad para iniciar o mantener el sueño. No síntomas diurnos.
El curso puede ser agudo/situacional, persistente o recurren- - Insomnio agudo/situacional: dura pocos días o po-
te. El agudo suele durar pocos días y estar asociado a aconte- cas semanas, a menudo asociada con acontecimientos vita-
cimientos vitales o cambios rápidos en los horarios de sueño les o cambios en los horarios de sueño.
o del entorno. También puede ser episódico, con episodios - Trastornos del ritmo circadiano de sueño-vigilia del
recurrentes relacionados con la presencia de acontecimien- tipo de fases de sueño retrasadas y del tipo asociado a tur-
tos estresantes. Las tasas de cronicidad oscilan entre el 45 y nos laborales: insomnio sólo cuando intenta dormir en los
el 75 % para un seguimiento de 1-7 años. horarios socialmente aceptados; antecedente de trabajo por
Los factores precipitantes del insomnio pueden ser diferen- turnos reciente.
tes de los de mantenimiento. - Síndrome de las piernas inquietas: necesidad de mo-
Los problemas de insomnio son más prevalentes entre los ver las piernas y sensaciones desagradables.
adultos de edad mediana o avanzada: - Trastornos del sueño relacionados con la respira-
- Insomnio de conciliación: más en adultos jóvenes. ción: enfermedad respiratoria durante el sueño.
- Insomnio de mantenimiento: más en adultos de me- - Narcolepsia: tétrada de la narcolepsia.
diana o avanzada edad. - Parasomnias: presencia de comportamientos o fe-
Las dificultades de sueño en niños pueden relacionarse con nómenos fisiológicos anormales durante el sueño o transi-
condicionamiento o ausencia de horarios constantes o ruti- ción sueño-vigilia.
nas de sueño. En adolescentes, se inicia o exacerba por ho- - Trastorno del sueño inducido por sustancias/medi-
rarios de sueño irregulares. camentos, tipo con insomnio: causa orgánica (p. ej., consu-
mo importante de cafeína).
FACTORES DE RIESGO Y PRONÓSTICO
Más probabilidad cuando los sujetos predispuestos se expo- COMORBILIDAD
nen a acontecimientos precipitantes (sucesos vitales mayo- Relación de riesgo bidireccional con afecciones médicas (dia-
res, o estrés menor crónico). betes, enfermedad coronaria, EPOC, artritis, fibromialgia).
Temperamentales: personalidades y estilos cognitivos ansio- Normalmente, trastorno mental comórbido (trastorno bipo-
sos y tendentes a la preocupación, mayor predisposición a la lar, depresivo y de ansiedad). El insomnio persistente repre-
activación y represión emocional. senta un factor de riesgo o un síntoma precoz de un poste-
Ambientales: ruido, luz, temperaturas molestas y altitud ele- rior trastorno bipolar, depresivo, de ansiedad o de consumo
vada. de sustancias.
Genéticos y fisiológicos: género femenino; edad avanzada;
predisposición familiar.
Modificadores del curso: mala higiene de sueño. TRASTORNO DE HIPERSOMNIA
ASPECTOS RELACIONADOS CON EL GÉNERO CARACTERÍSTICAS DIAGNÓSTICAS
Más prevalente en mujeres. Inicio con el nacimiento de un Somnolencia excesiva con excesiva cantidad de sueño, al-
nuevo hijo o la menopausia. teración de la calidad de la vigilia y/o inercia de sueño. El
Mejor preservación de la continuidad del sueño y el sueño problema de sueño se produce al menos 3 veces/semana,
profundo en mujeres mayores (vs. hombres mayores). durante 3 meses, genera malestar, y no se explica mejor por
otro trastorno del sueño, otro trastorno mental u organici-
MARCADORES DIAGNÓSTICOS dad.
Polisomnografía: alteraciones de la continuidad del sueño Se duermen rápidamente y tienen una buena eficiencia del
(más latencia de sueño, más tiempo de vigilia tras el inicio sueño (> 90 %). Pueden tener dificultad para despertarse
del sueño, menos tiempo dormido) y prolongación de fase 1 por la mañana o tras una siesta diurna y a veces parecen
y disminución de fases 3 y 4. confundidos, combativos o atáxicos, con disminución del
Sin embargo, los sujetos con insomnio a menudo subesti- rendimiento y la vigilancia (inercia del sueño o borrachera
man la duración del sueño o sobrestiman los despertares en de sueño).
comparación con la polisomnografía. La necesidad persistente de dormir puede llevar a una con-
Más electroencefalogramas de alta frecuencia que las perso- ducta automática que se realiza con poco o ningún recuerdo
nas sin insomnio en el período de inicio del sueño y durante posterior.
el sueño no REM (mayor activación cortical). Hay sujetos cuyo episodio principal de sueño dura más de 9
Indicios de aumento del arousal y activación del eje hipotála- horas, pero que no es reparador y se sigue de dificultad para
mo-hipofisario-adrenal. despertar por la mañana. Hay otros sujetos con un sueño de
Síntomas relacionados con el estrés. duración normal donde la somnolencia excesiva se caracte-
riza por siestas diurnas involuntarias (en situaciones de baja
CONSECUENCIAS FUNCIONALES estimulación o actividad).
Problemas interpersonales, sociales y laborales (por insom-
nio, excesiva preocupación por el sueño, aumento de irrita- CARACTERÍSTICAS QUE APOYAN EL DIAGNÓSTICO

82
Sueño no reparador (80%), conducta automática, dificulta- NARCOLEPSIA
des para despertarse por la mañana (80%), inercia del sue-
ño (36-50%). Antecedentes familiares de hipersomnia, sín- CARACTERÍSTICAS DIAGNÓSTICAS
tomas de disfunción del sistema nervioso autónomo (como Somnolencia excesiva 3 veces/semana, durante 3 meses. Ca-
dolores de cabeza vasculares o síndrome de Raynaud). taplejía frecuentemente desencadenada por emociones (risa,
bromas). Déficit de hipocretina en el líquido cefalorraquídeo
PREVALENCIA (LCR). Polisomnografía con corta latencia de sueño REM (≤
Prevalencia similar en hombres y mujeres. 15 minutos) o test de latencias múltiples de sueño (TLMS)
5-10% de los sujetos que acuden a clínicas de sueño por con latencia media de sueño ≤ 8 min o ≥ 2 períodos de REM
problemas de sueño diurno. al inicio del sueño en 4-5 siestas.
Episodios de inercia del sueño en población general: 1%.

DESARROLLO Y CURSO CARACTERÍSTICAS QUE APOYAN EL DIAGNÓSTICO


Comienzo progresivo, con síntomas que comienzan a los 15- Conductas automáticas si la somnolencia es grave.
25 años. Se manifiesta por completo en la mayoría de los ca- 20-60% presentan alucinaciones hipnagógicas o hipnopóm-
sos en la adolescencia tardía o en la edad adulta temprana, picas o parálisis del sueño.
con una edad media de inicio de 17-24 años. Son diagnos- Pesadillas, trastorno del comportamiento del sueño REM.
ticados 10-15 años después de la aparición de los primeros Ingesta nocturna, obesidad.
síntomas. Raro en niños. Reflejos abolidos durante la cataplejía (vs. trastorno de con-
Curso persistente y estable (si no se inicia tratamiento). versión).

FACTORES DE RIESGO Y PRONÓSTICO PREVALENCIA


Ambientales: estrés psicológico, consumo de alcohol, infec- 0,02-0,04% en población general.
ciones virales (10%), traumatismo craneal. Prevalencia similar en hombres y mujeres, ligeramente supe-
Genéticos y fisiológicos: herencia autosómica dominante. rior en hombres.

MARCADORES DIAGNÓSTICOS DESARROLLO Y CURSO


Duración del sueño desde normal a prolongada, latencia del Inicio en infancia-adolescencia/adultos jóvenes. 2 picos: 15-
sueño acortada y continuidad del sueño de normal a aumen- 25 años y 30-35 años.
tada. Distribución normal del sueño REM. La eficiencia del Comienzo abrupto o progresivo. Mayor gravedad si el ini-
sueño suele ser > 90 %. Algunos presentan gran cantidad cio es abrupto en niños (el inicio abrupto puede asociarse
de sueño de ondas lentas. El promedio de la latencia del sue- con obesidad y pubertad prematura), luego disminuye con
ño es característicamente < 10 minutos y frecuentemente ≤ la edad o tratamiento. Una vez manifestado, el curso es per-
8 minutos. Pueden aparecer períodos REM al comienzo del sistente y crónico.
sueño, pero < 2 veces en cada 4-5 siestas. En el 90% de los casos, el primer síntoma que se manifies-
ta es la somnolencia o el aumento de sueño, seguido de la
CONSECUENCIAS FUNCIONALES cataplejía (un 50% al año, un 85% a los 3 años). La som-
Disminución de eficiencia, concentración y memoria. Dificul- nolencia, las alucinaciones hipnagógicas, los sueños vívidos
tad para cumplir con responsabilidades matutinas. Peligrosi- y el trastorno comportamental del sueño REM son síntomas
dad asociada a los episodios de sueño involuntarios diurnos. precoces. En niños, la parálisis del sueño se suele desarrollar
en la pubertad.
DIAGNÓSTICO DIFERENCIAL Niños y adolescentes suelen presentar agresividad o proble-
- Variaciones normales del sueño: no hay somnolen- mas conductuales secundarios a la somnolencia y/o a la inte-
cia excesiva, inercia del sueño ni conductas automáticas. rrupción del sueño nocturno.
Sueño reparador.
- Mala calidad del sueño y fatiga. FACTORES DE RIESGO Y PRONÓSTICO
- Trastornos del sueño relacionados con la respira- Temperamentales: parasomnias más frecuentes. Necesitan
ción: enfermedad respiratoria durante el sueño. Episodios de más tiempo para dormir que el resto de la familia.
apnea. Ambientales: infecciones de garganta por el estreptococo del
- Trastornos del ritmo circadiano de sueño-vigilia: an- grupo A, virus de la influenza u otras infecciones invernales
tecedente de un horario anormal de sueño-vigilia. son probables desencadenantes de un proceso autoinmu-
- Parasomnias: no suelen producir sueño prolongado ne que produce narcolepsia posteriormente. Traumatismos
nocturno ni somnolencia diurna. craneales y cambios bruscos en los patrones de sueño-vigilia
- Otros trastornos mentales: p. ej., episodio depresivo pueden ser desencadenantes.
mayor con rasgos atípicos, fase depresiva del trastorno bipo- Genéticos y fisiológicos: concordancia gemelos monocigóti-
lar. cos: 25-32%. Prevalencia de 1-2% en los familiares de pri-
mer grado (aumento del riesgo entre 10 y 40 veces).
COMORBILIDAD
Trastornos depresivos, bipolares (en episodio depresivo) y ASPECTOS RELACIONADOS CON LA CULTURA
trastorno depresivo mayor de patrón estacional. Trastorno En americanos de raza negra hay más casos sin cataplejía o
por consumo de sustancias. Trastornos neurodegenerativos. con cataplejía atípica.

MARCADORES DIAGNÓSTICOS
Neuroimagen funcional: alteración de respuestas hipotalá-

83
micas a estímulos hormonales. diurna.
Polisomnografía y TLMS: período REM al comienzo del sueño Se diagnostica en base a la polisomnografía y a los síntomas:
en la polisomnografía (latencia de sueño REM ≤ 15 minutos); ≥ 5 apneas/hipopneas obstructivas por hora junto a altera-
latencia de sueño promedio de ≤ 8 minutos y períodos REM ciones de la respiración nocturna y/o somnolencia diurna o
al comienzo del sueño en ≥ 2 siestas del test de 4-5 siestas; sueño no reparador; o ≥ 15 apneas/hipopneas obstructivas
despertares frecuentes; disminución de la eficacia del sueño; por hora.
aumento de fase 1 del sueño. Movimientos periódicos de las Es el trastorno relacionado con la respiración más frecuente.
piernas (en un 40%) y apnea del sueño frecuentes.
Deficiencia de hipocretina, demostrada midiendo la inmuno- CARACTERÍSTICAS QUE APOYAN EL DIAGNÓSTICO
rreactividad en el LCR a hipocretina-1. Insomnio. Ardor de estómago, nicturia, dolores de cabeza
matinales, boca seca, disfunción eréctil y disminución de la
CONSECUENCIAS FUNCIONALES libido. Hipertensión en más del 60%.
Deterioro en la conducción y el trabajo. Riesgo de aislamien-
to social y de lesiones accidentales a ellos mismos u otros. PREVALENCIA
Deterioro en las relaciones sociales al evitar la cataplejía con- Niños: 1-2%. No hay diferencias sexuales.
trolando sus emociones. Adultos: 2-15%. Mayor prevalencia en hombres que en mu-
jeres (2-4:1).
DIAGNÓSTICO DIFERENCIAL Ancianos: >20%. Las diferencias por sexo disminuyen a eda-
- Otras hipersomnias: sueño nocturno más largo y des avanzadas, posiblemente por un aumento de la preva-
menos interrumpido, mayor dificultad para despertarse, más lencia en las mujeres tras la menopausia.
somnolencia diurna persistente, episodios de sueño diurno
más largos y menos reparadores, y poco o ninguna ensoña- DESARROLLO Y CURSO
ción durante las siestas diurnas. Disposición por edades en forma de J.
- Privación de sueño e insuficiente sueño nocturno. Pico en niños de 3-8 años, se reduce la prevalencia con el
- Síndromes de apnea del sueño. crecimiento de la vía aérea, y aumenta según aumenta la
- Trastorno depresivo mayor: no hay cataplejía, di- prevalencia de la obesidad en la edad adulta y las mujeres
sociación entre la somnolencia subjetiva y la objetiva (en entran en la menopausia.
TLMS). Comienzo insidioso, progresión gradual y curso persistente.
- Trastorno de síntomas neurológicos funcionales: Se presenta con más frecuencia entre los 40-60 años. Mayor
rasgos atípicos. TLMS sin REM al inicio del sueño. incidencia de apneas/hipopneas en sujetos mayores, varo-
- Trastorno por déficit de atención/hiperactividad y nes, con mayor índice de masa corporal.
otros problemas conductuales: en niños y adolescentes la Los niños pueden presentar un retraso del desarrollo, sien-
somnolencia puede causar problemas conductuales (agresi- do la obesidad un factor de riesgo menos frecuente. Se ha
vidad e inatención). observado una variación estacional, así como mejoría con el
- Crisis epilépticas: no se desencadenan por emocio- crecimiento general. Puede haber enuresis nocturna, respi-
nes, tienen más probabilidad de herirse cuando se caen, no ración por la boca por el día, dificultad para tragar y mala
hay atonía aislada. articulación del lenguaje. En menores de 5 años: más sínto-
- Corea y trastornos del movimiento. mas nocturnos. En mayores de 5 años: más importancia de
- Esquizofrenia: no hay cataplejía. síntomas diurnos.

COMORBILIDAD FACTORES DE RIESGO Y PRONÓSTICO


Trastorno bipolar, depresivo o de ansiedad. Raramente co- Genéticos y fisiológicos: obesidad, sexo masculino. Retrog-
morbilidad con esquizofrenia. natia máxilo-mandibular y micrognatia, antecedentes fami-
Índice de masa corporal aumentado u obesidad (especial- liares, síndromes genéticos que reducen la permeabilidad
mente en narcolepsia sin tratamiento). de las vías aéreas superiores (p. ej., el síndrome de Down),
hipertrofia adenoamigdalar, menopausia y varios síndromes
endocrinos (p. ej., la acromegalia).
Fuerte base genética: prevalencia 2 veces mayor en familia-
APNEA E HIPOPNEA OBSTRUCTIVA DEL SUE- res de primer grado. 1/3 de la varianza del índice apnea-hi-
ÑO popnea se explica por factores familiares comunes.

ESPECIFICADORES ASPECTOS RELACIONADOS CON LA CULTURA


Independientemente del índice de apnea-hipopnea, una ma- Los sujetos asiáticos pueden tener mayor riesgo de apnea-hi-
yor gravedad está relacionada con desaturación significativa popnea obstructiva del sueño a pesar de tener un índice de
de oxígeno de la hemoglobina o sueño muy fragmentado. masa corporal bajo.

CARACTERÍSTICAS DIAGNÓSTICAS ASPECTOS RELACIONADOS CON EL GÉNERO


Episodios repetidos de obstrucción de las vías aéreas altas Las mujeres pueden referir con más frecuencia fatiga que
durante el sueño. somnolencia. Pueden no presentar ronquidos.
Apnea/hipopnea: reducción en la respiración de ≥ 10 segun-
dos de duración en adultos o 2 respiraciones perdidas en MARCADORES DIAGNÓSTICOS
niños, y se asocia con descensos de saturación de oxígeno ≥ Las gasometrías sanguíneas, mientras el sujeto está despier-
3% y/o despertar electroencefalográfico. to, son habitualmente normales, pero algunos sujetos pue-
Los principales síntomas son el ronquido y la somnolencia den tener hipoxemia o hipercapnia al despertarse.

84
Niños: dificultades respiratorias, hipoventilación obstructiva La idiopática y la respiración Cheyne-Stokes se caracterizan
parcial con desaturaciones cíclicas, hipercapnia y movimien- por una inestabilidad en la ventilación y los niveles de CO2
tos paradójicos. Se utilizan para el índice de apnea-hipopnea alveolar, llamada respiración periódica (hiperventilación-hi-
valores de 2 para definir el umbral de la anormalidad. poventilación).
Si la apnea central del sueño ocurre en asociación con el sín-
CONSECUENCIAS FUNCIONALES drome de apnea e hipopnea obstructiva del sueño, se llama
Somnolencia diurna (> 50%). Ronquido y somnolencia mul- apnea del sueño compleja.
tiplican por 2 el riesgo de accidentes laborales. Incremento
de accidentes de tráfico por 7. CARACTERÍSTICAS QUE APOYAN EL DIAGNÓSTICO
Somnolencia o insomnio. Quejas de sueño fragmentado, in-
DIAGNÓSTICO DIFERENCIAL cluidos despertares con disnea.
- Ronquido primario y otros trastornos del sueño: no
tienen alteraciones en la polisomnografía nocturna. PREVALENCIA
- Narcolepsia: cataplejía, alucinaciones relacionadas Prevalencia superior en hombres que en mujeres. Aumenta
con el sueño, parálisis del sueño, ausencia de ronquidos so- con la edad (> 60 años). Idiopática: prevalencia desconocida.
noros y jadeos o apneas. Episodios de sueño diurno cortos, Respiración de Cheyne-Stokes: prevalencia ≥ 20% en sujetos
aportan más descanso y se asocian a ensoñaciones. Múlti- con una fracción de eyección ventricular < 45%. En 20% de
ples períodos de REM al comienzo del sueño. sujetos con infarto cerebral.
- Trastorno de insomnio: ausencia de ronquido y de Consumo concurrente de opiáceos: en 30% de sujetos que
antecedentes; signos y síntomas característicos. toma opiáceos para el dolor no oncológico y en tratamiento
- Ataques de pánico: menor frecuencia de episodios, de mantenimiento con metadona.
intensa activación autonómica y falta de excesiva somnolen-
cia. Polisomnografía sin patrón típico. DESARROLLO Y CURSO
- Trastorno por déficit de atención/hiperactividad. Respiración de Cheyne-Stokes: inicio ligado a insuficiencia
- Insomnio o hipersomnia inducidos por sustancias/ cardíaca.
medicamentos. Consumo concurrente de opiáceos: inicio tras uso crónico de
opiáceos (varios meses).
COMORBILIDAD
Hipertensión sistémica, enfermedad coronaria, fallo cardía- FACTORES DE RIESGO Y PRONÓSTICO
co, infarto cerebral, diabetes y mayor mortalidad. Afecciones Genéticos y fisiológicos: Cheyne-Stokes: insuficiencia car-
médicas o neurológicas. díaca, fibrilación auricular, edad avanzada, sexo masculino,
1/3 refiere síntomas de depresión y 10% depresión modera- infarto cerebral agudo, insuficiencia renal. Consumo concu-
da-grave. A mayor gravedad de la apnea e hipoapnea obs- rrente de opiáceos: opiáceos de duración prolongada.
tructiva del sueño, mayor gravedad de los síntomas depresi-
vos (mayor correlación en hombres). MARCADORES DIAGNÓSTICOS
Los hallazgos físicos en el subtipo Cheyne-Stokes se relacio-
nan con sus factores de riesgo.
La duración del ciclo de la respiración de Cheyne-Stokes
APNEA CENTRAL DEL SUEÑO (tiempo desde el final de una apnea central hasta el final de
la siguiente apnea) es de unos 60 segundos.
ESPECIFICADORES
Continuidad y calidad del sueño alteradas con las reduc- CONSECUENCIAS FUNCIONALES
ciones del sueño de ondas lentas. En sujetos con subtipo Somnolencia excesiva e insomnio. La coexistencia de respira-
Cheyne-Stokes, el patrón también se puede observar en los ción Cheyne-Stokes e insuficiencia cardíaca puede conllevar
descansos de la vigilia (marcador de mortalidad). un aumento de arritmias, mortalidad y trasplantes cardíacos.

CARACTERÍSTICAS DIAGNÓSTICAS DIAGNÓSTICO DIFERENCIAL


Episodios repetidos de apneas e hipopneas durante el sueño, Otros trastornos del sueño relacionados con la respiración y
causados por la variabilidad del esfuerzo respiratorio (≥ 5 ap- trastornos del sueño.
neas centrales por hora de sueño).
- Idiopática: somnolencia, insomnio y despertares de- COMORBILIDAD
bidos a la disnea relacionada con ≥ 5 apneas centrales por Consumidores de opiáceos de duración prolongada, como
hora. la metadona. Apnea e hipopnea obstructiva del sueño. La
- Respiración de Cheyne-Stokes: variaciones perió- respiración de Cheyne-Stokes se observa con frecuencia aso-
dicas de aumento-disminución del volumen corriente que ciada a insuficiencia cardíaca, infarto cerebral y fibrilación
origina apneas centrales e hipopneas que ocurren con una auricular. Los sujetos con este subtipo de apnea suelen ser
frecuencia de ≥ 5 apneas centrales por hora y que se acom- hombres mayores, de menor peso que los que presentan ap-
pañan de despertares. Tanto este subtipo como el anterior nea e hipopnea obstructiva del sueño.
se caracterizan por niveles de dióxido de carbono (CO2) en
sangre normales o ligeramente bajos mientras los sujetos es-
tán despiertos.
- Consumo concurrente de opiáceos: se caracteriza
por niveles de CO2 en sangre elevados mientras los sujetos
están despiertos.

85
HIPOVENTILACIÓN RELACIONADA CON EL gre y el corazón, lo que da lugar a disfunción cognitiva, po-
SUEÑO licitemia y arritmias.
Hipercapnia: depresión del impulso ventilatorio e insuficien-
CARACTERÍSTICAS DIAGNÓSTICAS cia respiratoria progresiva.
Episodios de disminución de la respiración asociados a una
elevación de la concentración de CO2. Puede producirse de DIAGNÓSTICO DIFERENCIAL
forma independiente o como comorbilidad ligada a proble- - Otras afecciones médicas que afectan a la ventila-
mas médicos o neurológicos o a trastornos por consumo de ción.
medicamentos /sustancias (más frecuente). - Otros trastornos del sueño relacionados con la respi-
ración: características clínicas y polisomnografía.
CARACTERÍSTICAS QUE APOYAN EL DIAGNÓSTICO
Insomnio, somnolencia. Ortopnea en los sujetos con debili- COMORBILIDAD
dad diafragmática. Dolores de cabeza matinales. Signos de - Enfermedad pulmonar, enfermedad neuromuscular
disfunción neurocognitiva o de depresión. Durante el sueño, o de la pared torácica, uso de medicamentos (p. ej., benzo-
episodios de respiración superficial, pudiendo coexistir con diazepinas, opiáceos).
apnea e hipopnea obstructiva o central del sueño. Los epi- - Hipoventilación alveolar central congénita: disfun-
sodios de hipoventilación pueden asociarse con despertares ción autonómica, enfermedad de Hirschsprung.
frecuentes o bradi-taquicardia. Pueden aparecer las conse- - Una apnea/ hipopnea obstructiva del sueño comór-
cuencias de la insuficiencia ventilatoria (p. ej., hipertensión bida exacerbaría la hipoxemia y la hipercapnia durante el
pulmonar). sueño y la vigilia.

PREVALENCIA
Muy poco frecuente.
Hipoventilación concurrente relacionada con el sueño es el TRASTORNOS DEL RITMO CIRCADIANO DE
subtipo más frecuente. SUEÑO-VIGILIA
DESARROLLO Y CURSO TIPO DE FASE DE SUEÑO RETRASADA
- Idiopática: trastorno de deterioro respiratorio lenta-
mente progresivo. CARACTERÍSTICAS DIAGNÓSTICAS
- Hipoventilación alveolar central congénita: suele Retraso en el horario del período principal de sueño (normal-
manifestarse al nacer por una respiración superficial, errática mente > 2 horas) en relación con el deseado para dormirse y
o ausente. También se puede manifestar durante la niñez, despertarse. Insomnio y somnolencia.
la infancia y adultez a causa de la penetrancia variable de Cuando pueden establecer su propio horario, su calidad y
la mutación PHOX2B. Tienen más probabilidades de tener duración del sueño es normal para su edad.
trastornos del sistema nervioso autónomo, enfermedad de
Hirschsprung, tumores de la cresta neural y cara cuadrada. CARACTERÍSTICAS QUE APOYAN EL DIAGNÓSTICO
Antecedentes de trastornos mentales y presencia de uno
FACTORES DE RIESGO Y PRONÓSTICO concurrente. Dificultad extrema y prolongada para desper-
Ambientales: consumo de depresores del sistema nervioso tar, con confusión matutina.
central (benzodiacepinas, opiáceos, alcohol).
Genéticos y fisiológicos: Idiopática: disminución del impulso PREVALENCIA
ventilatorio debido a una respuesta química aplanada al CO2 Población general: 0,17%. Adolescentes: > 7%. Suelen te-
que refleja déficits neurológicos subyacentes en los centros ner antecedentes familiares.
que controlan la ventilación. Lo más frecuente es que sea co-
mórbida con otra afección médica. Hipoventilación alveolar DESARROLLO Y CURSO
central congénita: trastorno genético atribuible a las muta- Curso persistente. Los síntomas comienzan típicamente en
ciones de PHOX2B. la adolescencia y en la edad adulta temprana. La gravedad
puede disminuir con la edad.
ASPECTOS RELACIONADOS CON EL GÉNERO
La distribución por sexos de la hipoventilación relacionada FACTORES DE RIESGO Y PRONÓSTICO
con el sueño que se presenta asociada a enfermedades co- Genéticos y fisiológicos: período circadiano más largo, cam-
mórbidas sigue el mismo patrón de distribución por sexos bios en la sensibilidad a la luz (hipersensibilidad a la luz de la
que dichas enfermedades comórbidas. tarde; hiposensibilidad a la luz de la mañana) y alteraciones
del impulso homeostático al dormir. Influencia de factores
MARCADORES DIAGNÓSTICOS genéticos (mutaciones de los genes circadianos).
Hipoxemia e hipercapnia relacionadas con el sueño (polisom-
nografía). Presentación de niveles arteriales de pCO2 > 55 MARCADORES DIAGNÓSTICOS
mm Hg durante el sueño o un aumento de ≥ 10 mm Hg en Historia completa y uso de un diario de sueño o un actígrafo.
los niveles de pCO2 (hasta un nivel > 50 mm Hg) durante el
sueño en comparación con los valores de la vigilia en posi- CONSECUENCIAS FUNCIONALES
ción supina, durante ≥ 10 minutos. Somnolencia excesiva al inicio del día. Dificultad extrema
para despertarse, con confusión matinal (inercia del sueño).
REPERCUSIONES FUNCIONALES
Hipoxemia: disfunción de órganos como el cerebro, la san- DIAGNÓSTICO DIFERENCIAL

86
- Variaciones normales del sueño: ausencia de males- despertar precoz.
tar y deterioro del funcionamiento.
- Otros trastornos del sueño.
TIPO DE SUEÑO-VIGILIA IRREGULAR
COMORBILIDAD
Depresión, trastorno de personalidad, de síntomas somáti- CARACTERÍSTICAS DIAGNÓSTICAS
cos, de ansiedad por la enfermedad. Ausencia de un ritmo circadiano sueño-vigilia reconocible.
Los sujetos videntes con trastorno del tipo del sueño-vigilia No hay un período principal de sueño y el sueño está frag-
no ajustado a las 24 horas tienen frecuentemente antece- mentado en ≥ 3 períodos a lo largo de las 24 horas.
dentes de retraso de la fase circadiana del sueño.
CARACTERÍSTICAS QUE APOYAN EL DIAGNÓSTICO
Insomnio o somnolencia excesiva. El período de sueño más
TIPO DE FASE DE SUEÑO AVANZADAS largo suele ser < 4 horas, (2 am-6 am).
Asociación frecuente con trastornos neurodegenerativos, y
ESPECIFICADORES con trastornos del neurodesarrollo en niños.
Suele haber historia familiar de fases del sueño avanzas. En
la forma familiar: inicio en la infancia/adultez temprana, cur- PREVALENCIA
so persistente, gravedad que aumenta con la edad. Desconocida.

CARACTERÍSTICAS DIAGNÓSTICAS DESARROLLO Y CURSO


Avance del horario del período principal de sueño (normal- Curso persistente. Más frecuente en adultos mayores.
mente > 2 horas) en relación con el tiempo deseado para
dormir y levantarse, con despertar precoz y somnolencia FACTORES DE RIESGO Y PRONÓSTICO
diurna excesiva. Temperamentales: trastornos neurodegenerativos, trastor-
Cuando pueden establecer su propio horario, su calidad y nos del neurodesarrollo en los niños.
duración del sueño es normal para su edad. Ambientales: disminución de la exposición a luz ambiental y
de actividad estructurada durante el día (sujetos hospitaliza-
CARACTERÍSTICAS QUE APOYAN EL DIAGNÓSTICO dos, personas con trastorno neurocognitivo mayor).
Horarios de los biomarcadores circadianos (melatonina y rit-
mos de temperatura corporal) adelantados 2-4 horas. MARCADORES DIAGNÓSTICOS
Historia de sueño, diario de sueño, actigrafía.
PREVALENCIA
Adultos: 1 %. Ancianos: mayor prevalencia. CONSECUENCIAS FUNCIONALES
Insomnio o somnolencia excesiva. Interrupción del sueño del
DESARROLLO Y CURSO cuidador.
Inicio en la edad adulta tardía. Curso persistente. Es el tipo
más frecuente en adultos mayores. DIAGNÓSTICO DIFERENCIAL
- Variaciones normales del sueño.
FACTORES DE RIESGO Y PRONÓSTICO - Otras afecciones médicas y trastornos mentales.
Ambientales: disminución de la exposición a la luz al final del
mediodía/comienzo de la tarde y exposición a la luz de pri- COMORBILIDAD
mera hora de la mañana debido a un despertar prematuro. Trastornos neurodegenerativos y del neurodesarrollo. Daño
Genéticos y fisiológicos: herencia autosómica dominante, in- cerebral traumático. Afecciones médicas y otros trastornos
cluidas una mutación del gen PER2 y una mutación sin sen- mentales en los que haya aislamiento social o falta de luz y
tido en CKI. actividades estructuradas.

ASPECTOS RELACIONADOS CON LA CULTURA


Los norteamericanos de raza negra pueden tener mayor ries- TIPO DE SUEÑO-VIGILIA NO AJUSTADO A LAS 24 HORAS
go (período circadiano más corto y avances de fase a la luz
mayores). CARACTERÍSTICAS DIAGNÓSTICAS
Insomnio o somnolencia excesiva relacionados con la sincro-
MARCADORES DIAGNÓSTICOS nización anormal entre el ciclo luz-oscuridad de 24 horas y el
Diarios del sueño y actigrafía. ritmo circadiano endógeno.

REPERCUSIONES FUNCIONALES CARACTERÍSTICAS QUE APOYAN EL DIAGNÓSTICO


Somnolencia excesiva: alteración del rendimiento cognitivo, Invidentes y personas con alteraciones de la visión (menor
interacción social y seguridad. percepción lumínica). En videntes hay un aumento en la du-
ración del sueño.
DIAGNÓSTICO DIFERENCIAL
- Otros trastornos del sueño. PREVALENCIA
- Trastornos bipolar y depresivo. 50% de los sujetos invidentes.

COMORBILIDAD DESARROLLO Y CURSO


Enfermedades médicas y trastornos mentales que cursan con Curso persistente. Edad de inicio variable, dependiendo del

87
comienzo de la alteración visual. En videntes, por solapa- TRASTORNOS DEL DESPERTAR DEL SUEÑO
miento con el tipo de fases de sueño retrasadas, puede de- NO REM
sarrollarse en la adolescencia o en la edad adulta temprana.
CARACTERÍSTICAS DIAGNÓSTICAS
FACTORES DE RIESGO Y PRONÓSTICO Despertares incompletos (de típicamente 1-10 minutos) ge-
Ambientales: disminución de la exposición o sensibilidad a neralmente en el primer tercio del sueño, acompañados de
la luz y la actividad social y física. Pacientes hospitalizados. sonambulismo o terrores nocturnos (“terrores de la noche”
Genéticos y fisiológicos: ceguera, lesión cerebral traumática. o “pavor nocturno”). No se recuerdan los sueños, hay amne-
sia de los episodios, generan malestar, y no se explican mejor
MARCADORES DIAGNÓSTICOS por otro trastorno mental u organicidad.
Historia de sueño, diario de sueño, actígrafo.

REPERCUSIONES FUNCIONALES CARACTERÍSTICAS QUE APOYAN EL DIAGNÓSTICO


Insomnio, somnolencia excesiva. Dificultad para asistir al co- La mayoría de las conductas durante los episodios de sonam-
legio o mantener un trabajo estable. Riesgo de aislamiento bulismo es rutinaria y de baja complejidad. Pueden aparecer
social. conductas inapropiadas (la más frecuente es orinar en un
armario/papelera). Los episodios suelen durar desde varios
DIAGNÓSTICO DIFERENCIAL minutos hasta media hora. En terrores nocturnos hay sen-
- Trastornos del ritmo circadiano de sueño-vigilia. timiento de terror abrumador con compulsión por escapar.
- Trastornos depresivos. Normalmente sólo 1 episodio/noche.
COMORBILIDAD PREVALENCIA
Ceguera. Trastornos depresivos y bipolares con aislamiento Episodios de sonambulismo: 10-30% niños (frecuente en
social. 2-3%); 1-7% adultos. Trastorno de sonambulismo: 1-5%.
Episodios de terrores nocturnos: 36,9% a los 18 meses,
19,7% a los 30 meses, 2,2% en adultos. Se desconoce la
TIPO ASOCIADO A TURNOS LABORALES prevalencia del trastorno en población general.
CARACTERÍSTICAS DIAGNÓSTICAS DESARROLLO Y CURSO
Presencia de antecedentes laborales fuera del horario diario Más frecuentes en la infancia. La frecuencia disminuye con
normal de 8:00 am a 6:00 pm (especialmente por las noches) la edad.
de forma habitual. Las personas que viajan frecuentemente a
distintas zonas horarias pueden presentar efectos similares. FACTORES DE RIESGO Y PRONÓSTICO
Ambientales: sedantes, privación de sueño, fiebre, alteracio-
PREVALENCIA nes del horario de sueño-vigilia, fatiga, estrés físico/emocio-
5-10% de la población que trabaja de noche (16-20% de los nal.
trabajadores). Mayor prevalencia con la edad. Genéticos y fisiológicos: historia familiar en hasta 80% de
sujetos con sonambulismo (el riesgo aumenta cuando ambos
DESARROLLO Y CURSO padres tienen antecedentes del trastorno; 60% de los hijos).
Mayor prevalencia en > 50 años. Empeora con el paso del 10 veces más de prevalencia de terrores nocturnos en fami-
tiempo. liares de primer grado de pacientes con terrores.
FACTORES DE RIESGO Y PRONÓSTICO ASPECTOS RELACIONADOS CON EL GÉNERO
Temperamentales: predisposición a ser “matinal”, necesidad Sexomnia: más en hombres. Ingesta alimentaria: más en mu-
de mayor duración del sueño (> 8 horas) para sentirse bien jeres.
descansado, necesidades sociales y domésticas importantes. Sonambulismo: más prevalente en niñas en infancia, más en
Genéticos y fisiológicos: apnea obstructiva del sueño (por hombres en edad adulta.
obesidad). Terrores: más prevalente en niños en infancia, siendo la pre-
valencia similar en edad adulta.
MARCADORES DIAGNÓSTICOS
Historia de sueño, diario de sueño, actigrafía. MARCADORES DIAGNÓSTICOS
Se presentan en cualquier estadio del sueño no REM, pero es
CONSECUENCIAS FUNCIONALES más frecuente que aparezcan en el primer tercio de la noche.
Peor rendimiento laboral, riesgo de accidentes laborales/de No suelen suceder durante las siestas diurnas.
tráfico. Mala salud mental y física. Problemas interpersona- Electroencefalograma: actividad teta o alfa (despertar in-
les. completo).
Despertares del sueño no REM de los terrores nocturnos: se
DIAGNÓSTICO DIFERENCIAL inician de forma brusca, sin cambios autonómicos previos
Variaciones normales en el sueño con turnos laborales. (vs. despertares del sueño REM en pesadillas).
COMORBILIDAD CONSECUENCIAS FUNCIONALES
Aumento del trastorno por consumo de alcohol y otras sus- Deterioro de relaciones sociales por vergüenza. Aislamiento
tancias, depresión. Trastornos físicos. social, dificultades laborales. Es poco frecuente que los suje-
tos se autolesionen o lesionen a otros que traten de conso-

88
larles. El sonambulismo no se asocia con trastornos menta- DESARROLLO Y CURSO
les significativos. Inicio: 3-6 años. Pico de prevalencia y gravedad: adolescen-
cia tardía-adultez temprana.
DIAGNÓSTICO DIFERENCIAL
- Trastorno de pesadillas: despertar completo, sueños FACTORES DE RIESGO Y PRONÓSTICO
vívidos, con historia, al final de la noche (sueño REM). Temperamentales: mayor frecuencia de acontecimientos ad-
- Trastornos del sueño relacionados con la respira- versos, alteraciones de la personalidad o diagnósticos psi-
ción: síntomas específicos. quiátricos.
- Trastorno del comportamiento del sueño REM: en Ambientales: privación o fragmentación del sueño y horarios
sueño REM, despertar fácil, contenido más detallado del sue- irregulares de sueño-vigilia (que alteran la intensidad o can-
ño. tidad de sueño REM).
- Síndrome con solapamiento de parasomnia: rasgos Genéticos y fisiológicos: efectos genéticos sobre la predispo-
clínicos y polisomnográficos de sonambulismo y de trastorno sición a pesadillas y concomitancia con otras parasomnias.
del comportamiento del sueño REM. Modificadores del curso: conductas de los padres en la cama
- Crisis epilépticas relacionadas con el sueño: más es- (consolar al niño) pueden proteger de la cronificación.
tereotipadas, varias veces/noche, pueden ocurrir en siestas
diurnas. MARCADORES DIAGNÓSTICOS
- Desmayos inducidos por el alcohol. Despertares bruscos del sueño REM, generalmente durante
- Amnesia disociativa, con fuga disociativa: a partir de la segunda mitad de la noche, antes de referir una pesadilla.
un período de vigilia durante el sueño. Las pesadillas postraumáticas también pueden aparecer du-
- Simulación u otra conducta voluntaria que puede rante el sueño no REM, en particular en el estadio 2.
ocurrir durante la vigilia.
- Trastorno de pánico: despertar rápido y completo, ASPECTOS RELACIONADOS CON LA CULTURA
sin confusión, amnesia o actividad motora típica. Diferencias interculturales en el significado de las pesadillas.
- Conductas complejas inducidas por la medicación.
- Síndrome de la ingesta nocturna: retraso en el ritmo ASPECTOS RELACIONADOS CON EL GÉNERO
circadiano de la ingesta de comida. Insomnio y depresión. Adultos: mayor prevalencia en mujeres que en hombres.
Diferencias de contenido en función del sexo:
COMORBILIDAD - Mujeres: acoso sexual, desaparición/muerte de seres
Adultos con sonambulismo: depresión mayor o trastorno ob- queridos.
sesivo-compulsivo. Niños o adultos con terrores: depresión y - Hombres: agresión física, guerra, terror.
ansiedad.
CONSECUENCIAS FUNCIONALES
Somnolencia diurna, mala concentración, depresión, ansie-
TRASTORNO DE PESADILLAS dad o irritabilidad.

CARACTERÍSTICAS DIAGNÓSTICAS DIAGNÓSTICO DIFERENCIAL


Historias que parecen reales y provocan ansiedad, miedo u - Trastorno de terrores nocturnos: primer tercio no-
otras emociones disfóricas. Las que suceden tras experiencias che, falta de recuerdo.
traumáticas pueden replicar la situación amenazante (“pesa- - Trastorno de comportamiento del sueño REM: acti-
dillas de réplica”), pero la mayoría no lo hace. Al despertarse vidad motora compleja durante los sueños.
se recuerdan bien. Casi exclusivamente en REM (más pro- - Duelo: pérdida y tristeza.
bables en la 2ª mitad del período de sueño). Al despertar, - Narcolepsia: tétrada.
el sujeto se orienta rápido y está alerta. Genera malestar, y - Crisis epilépticas nocturnas: actividad motora este-
no se explica mejor por otro trastorno mental u organicidad. reotipada.
Las llamadas “malos sueños”, pueden no hacer que la perso- - Trastornos del sueño relacionados con la respira-
na se despierte y que sólo se recuerden más tarde. ción.
Si aparecen al inicio del sueño REM (hipnagógicas), la dis- - Trastorno de pánico.
foria se suele acompañar de sentimiento de estar despierto - Trastornos disociativos relacionados con el sueño:
e incapacidad para moverse voluntariamente (parálisis del trauma.
sueño aislada). - Consumo de medicamentos o de sustancias: p. ej.,
medicamentos dopaminérgicos, estimulantes, antidepresi-
CARACTERÍSTICAS QUE APOYAN EL DIAGNÓSTICO vos, ayudas para dejar de fumar, melatonina. Abstinencia al
Activación autonómica ligera (sudoración, taquicardia, ta- alcohol y medicamentos que suprimen el sueño REM.
quipnea). Los sujetos con pesadillas frecuentes tienen un
mayor riesgo de ideación e intentos suicidas. COMORBILIDAD
Afecciones médicas (enfermedad coronaria, cáncer, parkin-
PREVALENCIA sonismo, dolor). En tratamientos médicos (hemodiálisis), o
Pesadillas “a menudo” o “siempre”: 1,3-3,9% niños prees- en la abstinencia de medicamentos/sustancias.
colares. La prevalencia aumenta desde los 10 a los 13 años Trastorno de estrés postraumático, trastorno de insomnio,
en ambos sexos, pero sigue aumentando entre los 20-29 esquizofrenia, psicosis, trastornos del ánimo, de ansiedad,
años en mujeres (en hombres disminuye), llegando a ser dos de adaptación y de personalidad, y el duelo.
veces mayor en mujeres que en hombres (2:1). Adultos: 6%
pesadillas mensuales, 1-2% frecuentes. TRASTORNO DEL COMPORTAMIENTO DEL

89
SUEÑO REM retrasar el comienzo del sueño y despertar al sujeto, y se aso-
cian con una importante fragmentación del sueño.
CARACTERÍSTICAS DIAGNÓSTICAS
Episodios repetidos de despertar, asociados con vocaliza- CARACTERÍSTICAS QUE APOYAN EL DIAGNÓSTICO
ciones (ruidosas, llenas de emoción y soeces) y/o compor- Movimientos periódicos de las piernas durante el sueño (y
tamientos motores complejos (“representación del sueño”), durante la vigilia). Historia familiar del síndrome en los fami-
que se producen en el sueño REM. Pueden producir lesio- liares de primer grado. Reducción de los síntomas, al menos
nes. Al despertar, el sujeto está totalmente despierto, alerta inicialmente, con tratamiento dopaminérgico. Insomnio y
y orientado. Sueño REM sin atonía en polisomnografía o an- somnolencia diurna excesiva.
tecedentes de trastorno del comportamiento del sueño REM
y diagnóstico de sinucleinopatía. Genera malestar, y no se PREVALENCIA
explica mejor por otro trastorno mental u organicidad. 1,6%. Mayor prevalencia en mujeres que en hombres (1,5-
2:1).
CARACTERÍSTICAS QUE APOYAN EL DIAGNÓSTICO
Gravedad según naturaleza, consecuencias del comporta- DESARROLLO Y CURSO
miento y frecuencia. Inicio: 20-30 años. 40% experimenta síntomas con menos
de 10 años. Aumentan con la edad, hasta los 60 años, des-
PREVALENCIA pués se estabilizan o disminuyen.
Población general: 0,38-0,5%. Mayor prevalencia en tras- Inicio temprano (< 45 años): progresión lenta. Inicio tardío:
tornos psiquiátricos. Más prevalente en hombres, > 50 años. progresión rápida.
Síndrome de piernas inquietas familiar: menor edad de ini-
DESARROLLO Y CURSO cio, progresión lenta.
Inicio gradual o rápido. Curso progresivo.
Gran relación con la aparición de trastorno degenerativo (si- FACTORES DE RIESGO Y PRONÓSTICO
nucleinopatía). Genéticos y fisiológicos: sexo femenino, edad avanzada, va-
riantes de riesgo genético (MEIS1, BTBD9, MAP2K5), antece-
FACTORES DE RIESGO Y PRONÓSTICO dentes familiares. Alteraciones en el sistema dopaminérgico
Genéticos y fisiológicos: consumo de algunos medicamentos central, alteraciones en el metabolismo del hierro.
(antidepresivos tricíclicos, ISRS, etc.).
ASPECTOS RELACIONADOS CON EL GÉNERO
MARCADORES DIAGNÓSTICOS Mayor prevalencia en mujeres. Prevalencia multiplicada por
Aumento de la actividad electromiográfica tónica y/o fásica 2-3 durante el embarazo.
durante el sueño REM (atonía muscular). Actividad electro-
miográfica periódica o aperiódica muy frecuente durante el MARCADORES DIAGNÓSTICOS
sueño no REM. Aumento de la latencia del sueño, mayor índice de desper-
tares.
CONSECUENCIAS FUNCIONALES
Deterioro en las relaciones sociales por vergüenza. CONSECUENCIAS FUNCIONALES
En 2-3% de la población: síntomas muy graves, con deterio-
DIAGNÓSTICO DIFERENCIAL ro del funcionamiento o depresión y ansiedad.
- Otras parasomnias: atonía REM normal. Alteraciones del sueño (reducción del tiempo, fragmenta-
- Crisis epilépticas nocturnas: comportamientos más ción, alteración global), depresión, trastorno de ansiedad
estereotipados, atonía en REM. generalizada, trastorno de pánico, trastorno de estrés pos-
- Apnea obstructiva del sueño: sueño REM con ato- traumático, alteraciones de la calidad de vida.
nía.
- Otros trastornos disociativos especificados (trastor- DIAGNÓSTICO DIFERENCIAL
no disociativo psicógeno asociado al sueño): en vigilia duran- Calambres de las piernas, incomodidad postural, artralgias/
te el período de sueño. artritis, mialgias, isquemia postural (entumecimiento), edema
- Simulación. de las piernas, neuropatía periférica, radiculopatía, golpeteo
con los pies habitual. Acatisia neuroléptica, mielopatía, insu-
COMORBILIDAD ficiencia venosa sintomática, enfermedad arterial periférica,
Narcolepsia en 30% (frecuencia similar en hombres y muje- eccema, otros problemas ortopédicos, inquietud provocada
res). > 50% desarrollará después una enfermedad neurode- por la ansiedad.
generativa.
COMORBILIDAD
SÍNDROME DE LAS PIERNAS INQUIETAS - Trastornos depresivos, de ansiedad y atencionales.
- Enfermedad cardiovascular (principal trastorno mé-
CARACTERÍSTICAS DIAGNÓSTICAS dico comórbido).
Trastorno del sueño sensoriomotor y neurológico caracteri- - Hipertensión, narcolepsia, migraña, Parkinson, ap-
zado por deseo de mover las piernas o los brazos normal- nea obstructiva del sueño, fibromialgia, obesidad.
mente asociado a sensaciones incómodas (calambres, cos- - Deficiencia de hierro, embarazo, insuficiencia renal
quilleos, hormigueo, quemazón o picor). 3 veces/semana, crónica.
durante 3 meses. Genera malestar, y no se explica mejor por TRASTORNOS DISRUPTIVOS, DEL
otro trastorno mental u organicidad. Los síntomas pueden

90
CONTROL DE LOS IMPULSOS Y DE de respuesta a las órdenes no se deba a problemas de com-
prensión del lenguaje.
LA CONDUCTA - Trastorno de ansiedad social.

REPERCUSIONES FUNCIONALES
TRASTORNO NEGATIVISTA DESAFIANTE (TND) Conflictos frecuentes con padres, profesores, supervisores,
compañeros y parejas. Deterioro de la adaptación emocio-
CARACTERISTICAS DIAGNÓSTICAS
nal, social, académica y laboral.
No es inusual que estas personas muestren sólo síntomas
conductuales sin problemas de estado de ánimo negativo. Si
COMORBILIDAD
hay enfado/irritabilidad suelen mostrar también problemas
TDAH (factores temperamentales comunes). Trastorno de
conductuales. Es frecuente que los problemas aparezcan
conducta. Trastorno de ansiedad. Trastorno depresivo ma-
únicamente en casa (lo que dificulta la evaluación). NO sue-
yor. Trastornos por consumo de sustancias.
len considerarse a sí mismas enfadados, negativistas o desa-
fiantes. Justifican su comportamiento por las circunstancias.

CARACTERÍSTICAS QUE APOYAN EL DIAGNÓSTICO


Mayor prevalencia en familias que tienen sucesión de cui- TRASTORNO DE CONDUCTA
dadores para los niños y en las familias que dispensan un
trato severo o negligente. Estas personas ven aumentado su CARACTERISTICAS DIAGNÓSTICAS
riesgo de suicidio, incluso después de controlar los trastornos Los individuos con trastorno de conducta son proclives a mi-
comórbidos. nimizar sus problemas de conducta, por lo que los clínicos a
menudo deben confiar en otros informantes.
PREVALENCIA
1-11% Media estimada del 3,3%. Variaciones que depen- CARACTERÍSTICAS QUE APOYAN EL DIAGNÓSTICO
den de edad y género. Más prevalente en niños que en niñas En situaciones ambiguas, tienden a malinterpretar las inten-
(1,4:1). No se han hallado estas diferencias de forma siste- ciones de los demás como hostiles y amenazante s, por lo
mática en adolescentes y adultos. que responden con agresividad que consideran razonable o
justificada. Emocionalidad negativa, bajo autocontrol, baja
DESARROLLO Y CURSO tolerancia a la frustración, irritabilidad, arrebatos, suspicacia,
Los primeros síntomas suelen aparecer durante la etapa pre- insensibilidad al castigo, búsqueda de emociones.
escolar y raramente más tarde de la adolescencia temprana. Mayor riesgo de intentos de suicidio y suicidios consumados.
Los síntomas de desafío, discusión y venganza aumentan Consumo de sustancias en chicas.
riesgo de trastorno de conducta. Los síntomas de enfado –
irritabilidad aumentan el riesgo de trastornos emocionales. PREVALENCIA
Manifestaciones uniformes a lo largo del desarrollo. 2-10%. Media estimada del 4%. H > M. Las tasas de preva-
lencia aumentan de la infancia a la adolescencia.
FACTORES DE RIESGO Y PRONÓSTICO
Temperamentales: falta de regulación emocional (alta reacti- DESARROLLO Y CURSO
vidad emocional, baja tolerancia a la frustración) Inicio del trastorno incluso en edad preescolar, aunque los
Ambientales: trato severo o negligente en la infancia. síntomas significativos aparezcan entre la infancia media y la
Genéticos y fisiológicos: marcadores neurobiológico s como adolescencia. Después de los 16 años, inicio raro. Los sínto-
baja tasa cardiaca, baja conductancia de la piel, reactividad mas varían con la edad: síntomas en infancia más leve y se
reducida de cortisol basal, anormalidades en córtex prefron- van haciendo más graves.
tal y amígdala. Curso variable. La mayoría remite en edad adulta, otros evo-
lucionan hacia TP antisocial.
ASPECTOS RELACIONADOS CON LA CULTURA
Prevalencia relativamente constante entre países que difie- FACTORES DE RIESGO Y PRONÓSTICO
ren en etnia y raza. Temperamentales: temperamento infantil de difícil control e
inteligencia por debajo de la media (sobre todo a nivel ver-
DIAGNÓSTICO DIFERENCIAL bal).
- Trastorno de conducta: comportamientos más gra- Ambientales: a nivel FAMILIAR rechazo y negligencia en el
ves en el Trastorno de conducta. cuidado, crianza incoherente, disciplina severa, abuso físico
- TDAH: asegurarnos de que la falta de respuesta a las o sexual, falta de supervisión, institucionalización temprana,
órdenes no se deba a problemas de desatención o hiperacti- cambios frecuentes de cuidadores, familia muy numerosa,
vidad. delincuencia o psicopatología parental; y a nivel COMUNITA-
- TDDEA: son excluyentes. RIO, rechazo de los compañeros, compañeros delincuentes,
- Trastorno explosivo intermitente: conlleva mayor barrio expuesto a violencia.
grado de ira y agresividad hacia los demás. Conlleva un alto Genéticos y fisiológicos: frecuencia cardiaca en reposo más
grado de ira intermitente. lenta, reducida respuesta autonómica de condicionamiento
- Discapacidad intelectual: considerar desarrollo men- al miedo, baja conductancia de la piel, alteraciones funcio-
tal para valorar lo normativo del comportamiento negativis- nales en conexiones límbico-frontotemporal es (amígdala y
ta. córtex prefrontal implicados).
- Trastorno del lenguaje: asegurarnos de que la falta Modificadores de curso: si inicio infantil, mayor gravedad,
más riesgo de cronificación y mayor vinculación con “emo-

91
ciones prosociales limitadas”. Genéticos y fisiológicos: mayor riesgo en familiares de pri-
mer grado. Anormalidades serotoninérgicas, sobre todo en
ASPECTOS RELACIONADOS CON LA CULTURA sistema límbico (cingulado anterior) y córtex orbitofrontal.
Debe tenerse en cuenta el contexto cultural en el que se Respuesta de la amígdala aumentada.
producen las alteraciones conductuales.
ASPECTOS RELACIONADOS CON LA CULTURA
ASPECTOS RELACIONADOS CON EL GÉNERO Variaciones culturales. Menor prevalencia en algunas regio-
Chicos: problemas de disciplina escolar, vandalismo, robo y nes (Asia, Oriente Medio, Rumanía, Nigeria) que en EEUU.
peleas. Más agresiones físicas. Chicas: mentiras, absentismo
escolar, escaparse de casa, consumo de sustancias, prostitu- ASPECTOS RELACIONADOS CON EL GÉNERO
ción. Más agresiones relacionales. Algunos estudios indican H > M y otros no encuentran di-
ferencias.
DIAGNÓSTICO DIFERENCIAL DIAGNÓSTICO DIFERENCIAL
- TND: pueden darse juntos en DSM 5. - NO diagnosticamos TEI si: se explica por otro TM o
- TDAH: alta comorbilidad. por organicidad, ni en niños de 6-18 años cuando los arre-
- Trastorno depresivo y bipolar: diferencias de curso. batos agresivos se producen en el contexto de un trastorno
Pueden darse juntos. de adaptación.
- TEI: si se cumplen criterios para ambos, sólo diag- - TDDEA: son excluyentes.
nosticamos TEI adicional si los arrebatos agresivos requieren - TP Antisocial o Límite: nivel de agresión impulsiva es
atención clínica independiente. menor que en el TEI.
- Trastornos de adaptación (con alteración mixta - TNCM o cambio de personalidad debido a otra afec-
emocional y conductual): estresor identificable. ción médica, tipo agresivo: organicidad de base. Intoxicación
o abstinencia de sustancia: organicidad de base.
REPERCUSIONES FUNCIONALES - TDAH, TND y TEA: en estos trastornos el nivel de
Problemas escolares, sociales, legales, laborales, ETS, emba- agresión impulsiva es menor que en el TEI (alta comorbili-
razos no deseados, lesiones físicas por accidentes o peleas… dad).

COMORBILIDAD REPERCUSIONES FUNCIONALES


TDAH, Trastorno negativista desafiante, Trastornos de Problemas sociales, laborales, económico s y legales.
aprendizaje, Trastorno de ansiedad, Trastornos depresivos y
bipolares y Trastorno por consumo de sustancias. COMORBILIDAD
Trastornos depresivos. Trastornos de ansiedad. Trastornos
por consumo de sustancias. Personas con TP antisocial. TLP.
TDAH. TND. Trastorno de conducta.
TRASTORNO EXPLOSIVO INTERMITENTE (TEI)

CARACTERISTICAS DIAGNÓSTICAS
Inicio rápido de los arrebatos agresivos impulsivos. Escaso o PIROMANÍA
nulo periodo prodrómico. Los arrebatos suelen durar menos
de 30 minutos y suelen darse en respuesta a una pequeña CARACTERISTICAS DIAGNÓSTICAS
provocación por parte de alguien cercano. A menudo son “mirones” de incendios y colaboran en las
La característica principal es la falta de control del compor- brigadas de extinción de incendios o son bomberos.
tamiento agresivo impulsivo en respuesta a una provocación
experimentada subjetivamente (factor estresante psicosocial) CARACTERÍSTICAS QUE APOYAN EL DIAGNÓSTICO
que no produciría normalmente un arrebato agresivo. Pueden preparar con antelación el inicio del fuego. Pueden
mostrarse indiferentes a las consecuencias del fuego o sentir
CARACTERÍSTICAS QUE APOYAN EL DIAGNÓSTICO satisfacción por la destrucción generada.
Suelen acompañarse de trastornos del estado de ánimo uni-
polares, trastornos de ansiedad y trastornos de consumo de PREVALENCIA
sustancia que suelen tener un inicio más tardío. Desconocida. Alta comorbilidad con otros trastornos (TP an-
tisocial, Trastorno por consumo de sustancias, trastorno bi-
PREVALENCIA polar y ludopatía). Es raro como diagnóstico primario.
2,7% en EEUU. Más prevalente en personas jóvenes (meno-
res de 35-40 años) que en más mayores. DESARROLLO Y CURSO
No hay datos para establecer edad típica de inicio. La rela-
DESARROLLO Y CURSO ción entre provocar incendios en la infancia y la piromanía
Inicio de síntomas más frecuente en la infancia tardía o la en la edad adulta no está bien documentada. Se desconoce
adolescencia. Rara vez por encima de los 40 años. Suele ex- curso longitudinal. La piromanía en infancia suele ser rara.
tenderse durante años. Curso variable: episódico vs crónico
y persistente. ASPECTOS RELACIONADOS CON EL GÉNERO
H > M, especialmente en varones con peores HHSS y proble-
FACTORES DE RIESGO Y PRONÓSTICO mas de aprendizaje.
Ambientales: trauma emocional y físico en las dos primeras
décadas de vida. DIAGNÓSTICO DIFERENCIAL

92
- Otras causas de provocación de incendios intencio- TRASTORNO DE LA PERSONALIDAD ANTISO-
nados: motivaciones económicas o de venganza subyacen- CIAL
tes.
- Otros Trastornos mentales
(Ver apartado de los Trastorno de la Personalidad).
COMORBILIDAD  
Trastornos de consumo de sustancias, Ludopatía, Trastorno
depresivo y bipolar y Otros trastornos de este capítulo.

CLEPTOMANÍA

CARACTERISTICAS DIAGNÓSTICAS
En ocasiones, el individuo puede guardar los objetos roba-
dos o devolverlos subrepticiamente. Generalmente evitarán
robar si pueden ser arrestados (cleptómanos sí, tontos). Nor-
malmente no planean los robos ni son plenamente cons-
cientes de que pueden ser detenidos. Roban sin asistencia o
colaboración de otros.

CARACTERÍSTICAS QUE APOYAN EL DIAGNÓSTICO


Suelen intentar suprimir su impulso de robar y son conscien-
tes de que el acto está mal y carece de sentido. Culpa tras el
robo es frecuente. Implicadas vías de neurotransmisión aso-
ciadas a las adicciones (5HT, Da y sistema opioide).

PREVALENCIA
PG 0,30,6%. 4-24% entre la población detenida por robar
en tiendas 3M > 1 H.

DESARROLLO Y CURSO
Inicio variable. Suele empezar en la adolescencia. Raro es
que aparezca en edad adulta tardía. Curso variable: esporá-
dico vs EPISÓDICO VS. CRÓNICO.

FACTORES DE RIESGO Y PRONÓSTICO


Genéticos y fisiológicos: familiares de primer gado presen-
tan tasas más altas de TOC, y de trastorno por consumo de
sustancias.

DIAGNÓSTICO DIFERENCIAL
- Robo ordinario: robo deliberado y motivado por la
utilidad del objeto robado. Es frecuente. –
- Hacerse el enfermo: simular síntomas de cleptoma-
nía para evitar un juicio penal.
- TP antisocial y trastorno de conducta: patrón más
general de comportamiento antisocial.
- Episodios maniacos, psicóticos y TNCM.

REPERCUSIONES FUNCIONALES
Problemas personales, profesionales, familiares y legales.

COMORBILIDAD
Compras compulsivas, Trastorno de estado de ánimo (sobre
todo, depresivo), Trastornos de ansiedad TCA (sobre todo
BN), TP Trastornos por consumo de sustancias (sobre todo,
alcohol) y Otros trastornos de este capítulo.

93
TRASTORNOS DE PERSONALIDAD problemas en sus relaciones cercanas.
Las personas con este trastorno pueden experimentar epi-
sodios psicóticos transitorios en respuesta al estrés (con una
TRASTORNO GENERAL DE PERSONALIDAD duración de minutos a horas).

CARACTERÍSTICAS DIAGNÓSTICAS PREVALENCIA


La evaluación puede ser complicada porque los rasgos de 2,3 % - 4,4 %
estos trastornos son a menudo egosintónicos, con lo que
puede ser útil obtener información complementaria de otros DESARROLLO Y CURSO
informantes. El trastorno de la personalidad paranoide puede ser evidente
El diagnóstico de los Trastornos de la personalidad requie- desde la infancia y la adolescencia, con soledad, escasas rela-
re una evaluación del funcionamiento del individuo a largo ciones con los compañeros, ansiedad social, bajo rendimien-
plazo, distinguiéndose de las características que surgen en to en la escuela, hipersensibilidad, pensamientos y lenguaje
respuesta a los factores de estrés situacionales específicos o peculiares, y fantasías idiosincrásicas. Estos niños suelen pa-
a los estados mentales transitorios. recer “extraños” o “excéntricos”, y son objeto de burlas.

DESARROLLO Y CURSO FACTORES DE RIESGO Y PRONÓSTICO


Algunos tipos de trastornos de la personalidad (en particular Genético s y fisiológicos. Existe cierta evidencia de un au-
los trastornos de la personalidad antisocial y límite) tienden mento de la prevalencia de este trastorno en los familiares
a ser menos evidentes o remiten con la edad, mientras que de los sujetos con esquizofrenia, sobre todo en aquellos con
esto parece ser menos cierto para algunos otros tipos (p. relación familiar con el trastorno delirante de tipo persecu-
ej., los trastornos de la personalidad obsesivo-compulsiva torio.
y esquizotípica). El diagnóstico se puede aplicar a niños o
adolescentes en casos excepcionales en los que los rasgos ASPECTOS RELACIONADOS CON LA CULTURA
desadaptativos de la personalidad son especialmente do- Algunos comportamientos, que se ven influidos por los con-
minantes, persistentes, y es improbable que se limiten a un textos socioculturales o por las circunstancias específicas de
momento particular del desarrollo o a la presencia de otro la vida, pueden ser erróneamente etiquetados como para-
trastorno mental. Para diagnosticar en un individuo menor noides e incluso confirmados por el propio proceso de eva-
de 18 años un trastorno de la personalidad, deberían apare- luación clínica. Los miembros de los grupos minoritarios, los
cer las características durante al menos un año. inmigrantes, los refugiados políticos y económicos, o las per-
sonas de diferentes orígenes étnicos pueden mostrar com-
ASPECTOS RELACIONADOS CON LA CULTURA portamientos cautelosos o defensivos.
Los trastornos de la personalidad no se deben confundir con
los problemas relacionados con la aculturación tras la inmi- ASPECTOS RELACIONADOS CON EL GÉNERO
gración o con la expresión de los hábitos, las costumbres o Hombres.
los valores religiosos y políticos que se profesen en la cultura
originaria del individuo. DIAGNÓSTICO DIFERENCIAL
-Otros trastornos mentales con síntomas psicóticos.
ASPECTOS RELACIONADOS CON EL GÉNERO -Cambio de la personalidad debido a otra afección médica:
Los clínicos deben ser cautelosos y no sobrediagnosticar o los rasgos que aparecen son atribuibles a los efectos directos
infradiagnosticar ciertos trastornos de la personalidad en las de la afección médica sobre el sistema nervioso central.
mujeres o en los hombres debido a los estereotipos sociales -Trastornos por consumo de sustancias: síntomas asociados
que condicionan los roles típicos de género y los comporta- a consumos persistentes de sustancias.
mientos. -Rasgos paranoides asociados con minusvalías físicas (p. ej.
discapacidad auditiva).
DIAGNÓSTICO DIFERENCIAL
Trastornos psicóticos, trastornos ansiosos y depresivos, TEPT, COMORBILIDAD
consumo de sustancias. En algunos casos, el trastorno de la personalidad paranoide
puede aparecer como el antecedente del trastorno delirante
o de la esquizofrenia.
Los individuos con trastorno de la personalidad paranoide
TRASTORNO DE PERSONALIDAD PARANOIDE pueden desarrollar un trastorno depresivo mayor y tienen un
riesgo aumentado de padecer agorafobia y trastorno obse-
CARACTERÍSTICAS QUE APOYAN EL DIAGNÓSTICO sivo compulsivo.
Su suspicacia y hostilidad excesivas se pueden expresar dis- Son frecuentes los trastornos por alcohol y otras sustancias.
cutiéndolo todo abiertamente, mediante quejas recurrentes Los trastornos de la personalidad concomitantes más comu-
o con una actitud distante y aparentemente hostil. Debido a nes son el esquizotípico, el esquizoide, el narcisista, el evita-
que están hipervigilantes en busca de amenazas potenciales, tivo y el límite.
pueden actuar de una manera cautelosa, secreta o maliciosa
y parece que son “fríos” y faltos de sentimientos de ternura.
Se les puede percibir como “fanáticos” y pueden adherirse TRASTORNO DE PERSONALIDAD ESQUIZOIDE
firmemente a “cultos” o a grupos de otras personas que
comparten su sistema de creencias paranoide. CARACTERISTICAS DIAGNÓSTICAS
Son generalmente difíciles de llevar y con frecuencia tienen A menudo presentan una constricción afectiva, y parecen

94
fríos y distantes. Sin embargo, en circunstancias muy excep- puede aparecer como el antecedente previo del trastorno
cionales en las que estas personas se sienten cómodas y se delirante o de la esquizofrenia. Las personas con este trastor-
sinceran, pueden reconocer que tienen sentimientos doloro- no a veces pueden desarrollar un trastorno depresivo mayor.
sos, sobre todo relacionados con las interacciones sociales. Este trastorno coincide con mayor frecuencia con los trastor-
nos de la personalidad esquizotípica, paranoide y evitativa.
CARACTERÍSTICAS QUE APOYAN EL DIAGNÓSTICO
Pueden tener especial dificultad para expresar la ira, incluso
en respuesta a la provocación directa, lo que contribuye a la
impresión de que les falta la emoción. TRASTORNO DE PERSONALIDAD ESQUIZOTÍ-
A veces parece que sus vidas carecen de dirección y puede PICO
parecer que van a la deriva en cuanto a sus objetivos y me-
tas. Suelen reaccionar de forma pasiva a las circunstancias CARACTERISTICAS DIAGNÓSTICAS
adversas. Los individuos con trastorno de la personalidad esquizotípica
Las personas con este trastorno pueden experimentar epi- experimentan la intimidad interpersonal como problemática
sodios psicóticos transitorios en respuesta al estrés (con una y se sienten incómodos en relación con otras personas. A pe-
duración de minutos a horas). sar de que pueden expresar descontento acerca de su falta
de relaciones, su comportamiento o sugiere una disminución
PREVALENCIA del deseo de los contactos íntimos.
3,1 – 4,9%
CARACTERÍSTICAS QUE APOYAN EL DIAGNÓSTICO
DESARROLLO Y CURSO Los individuos con trastorno de la personalidad esquizotípica
El trastorno de la personalidad esquizoide puede ser eviden- suelen buscar tratamiento para los síntomas asociados de
te desde la infancia y la adolescencia, con soledad, relaciones ansiedad o depresión y no por las características s del tras-
escasas con los compañeros y bajo rendimiento en la escue- torno de la personalidad en sí mismo.
la, que convierten a estos niños o adolescentes en “diferen- Específicamente, las personas con este trastorno pueden ex-
tes” y proclives a ser objeto de burlas. perimentar episodios psicóticos transitorios en respuesta al
estrés (con una duración de minutos a horas).
FACTORES DE RIESGO Y PRONÓSTICO
Prevalencia más elevada en los familiares de las personas con PREVALENCIA
esquizofrenia o con trastorno de la personalidad esquizotí- 3,9 – 4,6%. La prevalencia del trastorno en las poblaciones
pica. clínicas parece ser poco frecuente con una prevalencia esti-
mada más elevada en población general.
ASPECTOS RELACIONADOS CON LA CULTURA
Los individuos de diversos orígenes culturales a veces exhi- DESARROLLO Y CURSO
ben comportamientos defensivos y estilos comportamenta- El trastorno de la personalidad esquizotípica tiene un curso
les que pueden ser etiquetados erróneamente como “esqui- relativamente estable, con sólo una pequeña proporción de
zoides”. las personas que desarrolla una esquizofrenia u otro tras-
Los que se han trasladado desde las zonas rurales a entor- torno psicótico. El trastorno de la personalidad esquizotípi-
nos metropolitanos pueden reaccionar con la “congelación ca puede hacerse evidente, en la infancia y la adolescencia,
emocional”, que puede durar varios meses y manifestarse con soledad, pobres relaciones con los compañeros, ansie-
mediante la realización de actividades solitarias, un afecto dad social, bajo rendimiento en la escuela, hipersensibilidad,
constreñido y otros déficits en la comunicación. pensamientos y lenguaje peculiares y fantasías estrafalarias.
Pueden ser tachados de “extraños” o “excéntricos” y ser
ASPECTOS RELACIONADOS CON EL GÉNERO objeto de burlas.
Hombres
FACTORES DE RIESGO Y PRONÓSTICO
DIAGNÓSTICO DIFERENCIAL Tiene agregación familiar y es más frecuente entre los fami-
- Otros trastornos mentales con síntomas psicóticos: liares biológicos de primer grado de los individuos con esqui-
debería distinguirse del trastorno delirante, la esquizofrenia, zofrenia que en la población general.
y del trastorno bipolar o del trastorno depresivo con carac-
terísticas psicóticas, porque estos trastornos se caracterizan ASPECTOS RELACIONADOS CON LA CULTURA
por un período de síntomas psicóticos persistentes (delirios Las características generalizadas y determinadas cultural-
y alucinaciones). Para realizar un diagnóstico adicional de un mente, en particular las relativas a las creencias y los rituales
TP, éste debería haber aparecido antes de la presencia de los religiosos, pueden parecer esquizotípicas para el forastero
síntomas psicóticos y persistir. no informado.
- Trastorno del espectro autista: presentan un dete-
rioro más grave de la interacción social y comportamientos e ASPETOS RELACIONADOS CON EL GÉNERO
intereses estereotipados. Hombres
- Cambio de la personalidad debido a otra afección
médica. DIAGNÓSTICO DIFERENCIAL
- Trastornos por consumo de sustancias. -Otros trastornos mentales con síntomas psicóticos.
-Trastornos del neurodesarrollo: mayor falta de sensibilidad
COMORBILIDAD social y de reciprocidad emocional, y conductas e intereses
En algunos casos, el trastorno esquizoide de la personalidad estereotipados, compartiendo peculiaridades en el lenguaje

95
y aislamiento. ASPECTOS RELACIONADOS CON EL GÉNERO
-Cambio de la personalidad debido a otra afección médica. Hombres. Posible infradiagnóstico en mujeres.
-Trastornos por consumo de sustancias.
DIAGNÓSTICO DIFERENCIAL
COMORBILIDAD - Trastornos por consumo de sustancias.
Del 30 al 50 % de las personas diagnosticadas con este tras- - Esquizofrenia y trastornos bipolares: pueden tener
torno tiene un diagnóstico simultáneo de trastorno depresi- comportamientos antisociales en fases agudas, pero edad de
vo mayor durante su internamiento o en un entorno clínico. inicio y personalidad basal diferentes.
Existe una considerable coincidencia con los trastornos de la - Otros trastornos de la personalidad.
personalidad esquizoide, paranoide, evitativa y límite. - Comportamiento criminal no asociado con un tras-
torno de personalidad.

COMORBILIDAD
TRASTORNO DE PERSONALIDAD ANTISOCIAL También pueden experimentar disforia, con quejas de estrés,
incapacidad para tolerar el aburrimiento y depresión. Pueden
CARACTERISTICAS DIAGNÓSTICAS haber desarrollado trastornos de ansiedad, trastornos depre-
Pueden ser indiferentes o justificar de manera superficial los sivos, trastornos por consumo de sustancias, juego patológi-
daños, el maltrato o los robos a las personas (p. ej., “la vida co, trastorno de somatización y trastornos del control de los
es dura”, “los perdedores merecen perder”). Estas personas impulsos. También suelen tener características de personali-
pueden culpar a las víctimas de ser ingenuas, de estar desva- dad que cumplen con los criterios de otros trastornos de la
lidas o de merecer su destino (p. ej., “se lo merecía de todos personalidad, sobre todo de la personalidad límite, histrióni-
modos” o “de todas formas le hubiese ocurrido”). ca y narcisista.
En general, no compensan o reparan las consecuencias de
su comportamiento.

TRASTORNO DE PERSONALIDAD LÍMITE


CARACTERÍSTICAS QUE APOYAN EL DIAGNÓSTICO
Los individuos con trastorno de la personalidad antisocial CARACTERISTICAS DIAGNÓSTICAS
con frecuencia carecen de empatía y tienden a ser crueles, Estos individuos son muy sensibles a las circunstancias am-
cínicos y despectivos con los sentimientos, derechos y sufri- bientales, experimentan un miedo intenso al abandono e ira
mientos de los demás. La falta de empatía, la concepción de inapropiada incluso cuando se enfrentan con una separación
sí mismo elevada y el encanto superficial son característica s real de tiempo limitado o cuando se producen cambios inevi-
que han sido frecuentemente incluidas en la definición tra- tables en los planes
dicional de la psicopatía, y tienen mayor tendencia a morir Hay cambios repentinos y dramáticos de la autoimagen, ca-
prematuramente por medios violentos que las personas de racterizados por metas, valores y aspiraciones profesionales
la población general. cambiantes. Puede haber modificaciones repentinas de las
opiniones y proyectos acerca de la profesión, la identidad
PREVALENCIA sexual, los valores y los tipos de amigos.
0,2 – 3,3% El suicidio consumado se produce en el 8-10 % de estos
individuos, y son habituales los actos de autolesión.
DESARROLLO Y CURSO
Se puede tornar menos evidente o remitir con la edad, sobre CARACTERÍSTICAS QUE APOYAN EL DIAGNÓSTICO
todo hacia la cuarta década de la vida, particularmente evi- Pueden mostrar un patrón en el que se boicotean a sí mis-
dente en lo que respecta a la implicación en actos delictivos, mas cuando están a punto de alcanzar una meta (p. ej.,
no tanto en otros rasgos del espectro de los comportamien- abandonan la facultad justo antes de graduarse, empeoran
tos antisociales y del consumo de sustancias. gravemente después de haber tratado en la consulta los pro-
gresos en la terapia, rompen una buena relación cuando es
FACTORES DE RIESGO Y PRONÓSTICO evidente que la relación puede durar).
Genético s y fisiológicos. El trastorno de la personalidad anti- El abuso físico y sexual, el abandono, los conflictos hostiles y
social es más común entre los familiares biológicos de primer la pérdida prematura de los padres son frecuentes entre los
grado de las personas con el trastorno que en la población antecedentes infantiles de las personas con trastorno de la
general. El riesgo para los familiares biológicos de las muje- personalidad límite.
res con el trastorno tiende a ser mayor que el riesgo de los
familiares biológicos de los hombres con el trastorno. PREVALENCIA
PG:1,9 – 5,8%.
ASPECTOS RELACIONADOS CON LA CULTURA PC ambulatoria: 10% / PC hospitalaria: 20%.
Parece estar asociado con un nivel socioeconómico bajo y La prevalencia suele disminuir en los grupos de mayor edad.
con los entornos urbanos. Preocupa que ocasionalmente el
diagnóstico se pueda aplicar incorrectamente a personas de DESARROLLO Y CURSO
ámbitos en los que la conducta aparentemente antisocial Existe una considerable variabilidad en el curso del trastorno
podría ser parte de una estrategia de supervivencia y de pro- de la personalidad límite. El deterioro de la afección y el ries-
tección. go de suicidio son mayores en los adultos jóvenes y disminu-
yen gradualmente con la edad. La tendencia a experimentar
emociones intensas, a actuar de forma impulsiva y a vivir las

96
relaciones con gran intensidad suele perdurar toda la vida. en las relaciones románticas o sexuales. Sin ser conscientes
Durante la década de los 30 y 40 años, la mayoría de las de ello, a menudo desempeñan un papel (p. ej., “víctima” o
personas con este trastorno logra una mayor estabilidad en “princesa”) en sus relaciones con los demás. Los individuos
sus relaciones y funcionamiento profesional. con este trastorno a menudo tienen relaciones deficientes
con los amigos del mismo sexo porque su estilo interperso-
FACTORES DE RIESGO Y PRONÓSTICO nal de provocación sexual puede ser una amenaza para las
Genético s y fisiológico s. El trastorno de la personalidad lími- parejas de sus amigos.
te es aproximadamente cinco veces más frecuente entre los Estos individuos son a menudo intolerantes, o están frustra-
familiares biológicos de primer grado de las personas con el dos por las situaciones que implican una gratificación diferi-
trastorno que en la población general. También hay un ma- da, y sus acciones se dirigen a menudo a la obtención de una
yor riesgo familiar de trastornos por consumo de sustancia s, satisfacción inmediata.
trastorno antisocial de la personalidad y trastornos depresivo El riesgo real de suicidio no se conoce, pero la experiencia clí-
s o bipolares. nica sugiere que los individuos con este trastorno tienen un
mayor riesgo de gestos suicidas y de amenazas para llamar la
ASPECTOS RELACIONADOS CON LA CULTURA atención y para forzar unos cuidados mejores.
El patrón de comportamiento se ha identificado en múltiples
contextos de todo el mundo. Los adolescentes y los adultos PREVALENCIA
jóvenes con problemas de identidad pueden manifestar ras- 1,84%.
gos límites como inestabilidad emocional, dilemas “existen-
ciales”, incertidumbre, elecciones que provocan ansiedad, ASPECTOS RELACIONADOS CON LA CULTURA
los conflictos sobre la orientación sexual y las presiones so- Las normas de comportamiento interpersonal, la apariencia
ciales contrapuestas para decidir sobre el futuro vocacional. personal y la expresividad emocional varían ampliamente en-
tre las culturas, géneros y grupos de edad.
ASPECTOS RELACIONADOS CON EL GÉNERO
Mujeres (3:1) ASPECTOS RELACIONADOS CON EL GÉNERO
Mujeres. Algunos estudios que utilizan evaluaciones estruc-
DIAGNÓSTICO DIFERENCIAL turadas han obtenido similares tasas de prevalencia entre
-Trastornos depresivos y bipolares: mayor inestabilidad afec- hombres y mujeres.
tiva del TLP. Puede darse comorbilidad.
-Otros trastornos de la personalidad. DIAGNÓSTICO DIFERENCIAL
-Cambio de la personalidad debido a otra afección médica. - Otros trastornos de la personalidad.
-Trastornos por consumo de sustancias. - Cambio de la personalidad debido a otra afección
-Problemas de identidad. médica.
- Trastornos por consumo de sustancias.
COMORBILIDAD -
Los trastornos coexistentes más frecuentes son los trastornos
depresivos y bipolares, los trastornos de consumo de sustan- COMORBILIDAD
cias, los trastornos de la conducta alimentaria (especialmen- El trastorno de la personalidad histriónica se ha asociado
te la bulimia nerviosa), el trastorno de estrés postraumático con tasas más altas de trastorno de síntomas somáticos, de
y el trastorno por déficit de atención. También aparece de trastorno de conversión (trastorno de síntomas neurológicos
manera comórbida con otros trastornos de la personalidad. funcional) y de trastorno depresivo mayor. Los trastornos de
la personalidad dependiente, límite, narcisista y antisocial a
menudo coinciden con el trastorno de la personalidad his-
triónica.
TRASTORNO DE PERSONALIDAD HISTRIÓNI-
CO

CARACTERISTICAS DIAGNÓSTICAS TRASTORNO DE PERSONALIDAD NARCISISTA


Tienen un estilo de discurso que es excesivamente impre-
sionista y carente de detalle. Las opiniones se expresan con CARACTERISTICAS DIAGNÓSTICAS
un toque dramático, pero las razones y argumentos subya- Es probable que insistan en tratarse sólo con la persona “lí-
centes suelen ser vagos y difusos, sin apoyar los hechos y der o mejor valorada” (doctor, abogado, peluquero, instruc-
detalles. tor personal) o miembro de las “mejores” instituciones, pero
A menudo se muestran alegres y teatrales y tienden a llamar a la vez devalúan las credenciales y méritos de aquellas per-
la atención sobre sí mismas, e inicialmente pueden seducir sonas que les defraudan.
o maravillar a las nuevas amistades por su entusiasmo, su Buscan constantemente cumplidos y piropos, a menudo con
aparente apertura o su coquetería. Estas cualidades se des- un gran encanto. Tienen falta de empatía y dificultades para
vanecen pronto, puesto que continuamente demandan ser reconocer los deseos, experiencias subjetivas y sentimientos
el centro de atención. de los demás. Los sentimientos prolongados de vergüenza o
humillación y la autocrítica asociada pueden estar relaciona-
CARACTERÍSTICAS QUE APOYAN EL DIAGNÓSTICO dos con el aislamiento social, un estado de ánimo depresivo,
Las personas con trastorno de la personalidad histriónica y un trastorno depresivo persistente (distimia) o un trastorno
pueden tener dificultades para lograr intimidad emocional depresivo mayor. En contraste, los períodos mantenidos de
grandiosidad pueden estar asociados a un estado de ánimo

97
hipomaníaco. PREVALENCIA
Cuando reconocen las necesidades, los deseos o los sen- 2,4 %.
timientos de otras personas, los suelen ver con desprecio,
como signos de debilidad o vulnerabilidad. DESARROLLO Y CURSO
La conducta de evitación a menudo comienza en la infancia
CARACTERÍSTICAS QUE APOYAN EL DIAGNÓSTICO o la niñez con timidez, aislamiento y miedo a los extraños
Esta autoestima tan vulnerable hace que las personas con y a las situaciones nuevas. Aunque la timidez en la infancia
trastorno de la personalidad narcisista sean muy sensibles al es un precursor común del trastorno, en la mayoría de las
“daño” de la crítica o la derrota. personas tiende a disiparse gradualmente a medida que en-
vejecen. En contraste, las personas que llegan a desarrollar
PREVALENCIA el trastorno de la personalidad evitativa pueden llegar a ser
4,2% cada vez más tímidas y evasivas durante la adolescencia y la
adultez temprana.
DESARROLLO Y CURSO
Los rasgos narcisistas pueden ser particularmente frecuentes ASPECTOS RELACIONADOS CON LA CULTURA
en los adolescentes. La conducta de evitación puede ser el resultado de los pro-
Pueden tener dificultades especiales para adaptarse a la apa- blemas de aculturación consecuentes a la inmigración.
rición de limitaciones físicas y ocupacionales que son inhe-
rentes al proceso de envejecimiento. ASPECTOS RELACIONADOS CON EL GÉNERO
Hombres = Mujeres.
ASPETOS RELACIONADOS CON EL GÉNERO
50-75% Hombres. DIAGNÓSTICO DIFERENCIAL
- Trastornos de ansiedad.
DIAGNÓSTICO DIFERENCIAL - Otros trastornos de la personalidad.
- Otros trastornos de la personalidad. - Cambio de la personalidad debido a otra afección
- Manía o hipomanía: comparten la grandiosidad. No médica.
comparten: inestabilidad anímica de los episodios afectivos - Trastornos por consumo de sustancias.
ni la afectación funcional durante los mismos.
- Trastornos por consumo de sustancias. COMORBILIDAD
Otros trastornos que se diagnostican frecuentemente con el
COMORBILIDAD trastorno de la personalidad evitativa son los trastornos de-
El trastorno de la personalidad narcisista también se relacio- presivos, bipolares y de ansiedad, especialmente el trastorno
na con la anorexia nerviosa y los trastornos por consumo de de ansiedad social (fobia social).
sustancias (sobre todo los relacionados con la cocaína). El trastorno de la personalidad evitativa se diagnostica a me-
Los trastornos de la personalidad histriónica, límite, antiso- nudo junto con el trastorno de la personalidad dependiente.
cial y paranoide pueden ser comórbidos con el trastorno de
la personalidad narcisista.

TRASTORNO DE PERSONALIDAD DEPENDIEN-


TE
TRASTORNO DE PERSONALIDAD EVITATIVA
CARACTERISTICAS DIAGNÓSTICAS
CARACTERISTICAS DIAGNÓSTICAS Los adultos con este trastorno, por lo general dependen de
Si alguien muestra incluso una ligera desaprobación o crítica, un padre o cónyuge que decide dónde deben vivir, qué tipo
pueden sentirse muy lastimados. Ellos tienden a ser tímidos, de trabajo deberían tener y con quién deberían tener amis-
callados, inhibidos e “invisibles” a causa del temor de que tad. Los adolescentes con este trastorno pueden permitir que
cualquier atención pueda ocasionar una situación degradan- sus padres decidan cómo deben vestirse, con quién deben
te o de rechazo. Creen que a nadie le importa lo que ellos relacionarse, cómo deben pasar su tiempo libre y qué ca-
dicen, o que les “sentará mal”, por lo que pueden no hablar rrera estudiar. Su necesidad de mantener un vínculo impor-
nada en absoluto. Reaccionan enérgicamente a las señales tante ocasiona relaciones desequilibrada s o distorsionadas.
sutiles que son sugestivas de burla o mofa. Pueden sacrificarse a sí mismos de manera extraordinaria o
tolerar el abuso verbal, físico o sexual. Las personas con este
trastorno se sienten incómodas o indefensas cuando están
CARACTERÍSTICAS QUE APOYAN EL DIAGNÓSTICO solas, debido a sus temores exagerados de ser incapaces de
Su comportamiento temeroso y tenso puede provocar la ri- cuidar de sí mismos.
diculización y la burla de los demás, lo que a su vez confirma
sus propios temores. La baja autoestima y la hipersensibili- CARACTERÍSTICAS QUE APOYAN EL DIAGNÓSTICO
dad al rechazo están asociadas con los contactos interper- Las personas con trastorno de la personalidad dependiente a
sonal es restringidos. Estos individuos pueden llegar a estar menudo se caracterizan por el pesimismo y la duda, tienden
relativamente aislados y, por lo general, no tienen una red de a menospreciar sus capacidades y recursos, y pueden referir-
apoyo social amplia que puede ayudarles en los tiempos de se constantemente a sí mismos como “inútiles”.
crisis. Desean el afecto y la aceptación, y pueden fantasear Suelen evitar los puestos de responsabilidad y sienten an-
sobre relaciones idealizadas con otros. siedad cuando se enfrentan a una toma de decisiones. Las
relaciones sociales tienden a limitarse a las pocas personas

98
con las que el individuo tiene dependencia. PREVALENCIA
PREVALENCIA 2,1 – 7,9%.
0,49 – 0,6%
ASPECTOS RELACIONADOS CON LA CULTURA
DESARROLLO Y CURSO Algunas culturas ponen un énfasis sustancial en el trabajo y
En los niños y en los adolescentes, el comportamiento de- la productividad.
pendiente podría ser apropiado para el desarrollo.
ASPECTOS RELACIONADOS CON EL GÉNERO
ASPECTOS RELACIONADOS CON LA CULTURA El doble en Hombres.
El énfasis en la pasividad, la cortesía y el trato deferente es
característico de algunas sociedades y pueden ser valorados DIAGNÓSTICO DIFERENCIAL
incorrectamente como rasgos del trastorno de la personali- - Trastorno obsesivo compulsivo: presencia de verda-
dad dependiente. deras obsesiones y compulsiones. Pueden aparecer ambos
ASPECTOS RELACIONADOS CON EL GÉNERO diagnósticos.
Mujeres. Otros estudios igualan la prevalencia. - Trastorno de acumulación.
- Otros trastornos de la personalidad.
DIAGNÓSTICO DIFERENCIAL - Cambio de la personalidad debido a otra afección
- Otros trastornos mentales y afecciones médicas. médica.
- Otros trastornos de la personalidad. - Trastornos por consumo de sustancias.
- Cambio de la personalidad debido a otra afección
médica. COMORBILIDAD
- Trastornos por consumo de sustancias. Trastorno de ansiedad generalizad a, trastorno de ansiedad
social (fobia social) y las fobias específicas y trastorno obsesi-
COMORBILIDAD vo compulsivo (TOC). La mayoría de los individuos con TOC
Existe un mayor riesgo de trastornos depresivos, trastornos no tiene un patrón de comportamiento que cumpla con los
de ansiedad y trastornos de adaptación. criterios de este trastorno de la personalidad. de la perso-
El trastorno de la personalidad dependiente a menudo se nalidad. Existe una asociación entre el trastorno de la per-
desarrolla junto con otros trastornos de la personalidad, es- sonalidad obsesivo- compulsiva y los trastornos depresivos,
pecialmente el límite, el evitativo y el histriónico. bipolares y de la conducta alimentaria.
La afección física crónica o el trastorno de ansiedad por se-
paración en la infancia o en la adolescencia predisponen al AMPLIACIONES MANUALES 3º EDICIÓN
individuo a desarrollar este trastorno. TEMA 3. TRASTORNOS ADICTIVOS Y RELA-
CIONADOS CON SUSTANCIAS DSM 5
PÁG COLUMNA PÁRRAFO
TRASTORNO DE PERSONALIDAD OBSESI-
VA-COMPULSIVA 59 TOMO I 2 2. Añadir frase a
continuación del
CARACTERISTICAS DIAGNÓSTICAS párrafo 2, antes
El énfasis está en el rendimiento perfecto. de (Ver tabla 26).
Estos individuos convierten el juego en una tarea estructu-
rada. Pueden forzarse a sí mismos y a los demás a seguir AMPLIACIÓN
principios morales rígidos y normas de funcionamiento muy
estrictas. También pueden ser despiadadamente autocríticos 3.9. Trastornos relacionados con opiáceos
sobre sus propios errores. Los individuos con este trastorno
muestran una deferencia rígida con la autoridad y las normas El primer consumo produce náuseas, vómitos y disforia, pos-
e insisten en un cumplimiento literal, sin excepción por cir- teriormente el consumo produce placer, euforia, reducción
cunstancias especiales. de la ansiedad... lo que lleva a un consumo habitual para
encontrarse bien y evitar así el síndrome de abstinencia.
CARACTERÍSTICAS QUE APOYAN EL DIAGNÓSTICO Los sujetos que realizan un consumo regular de heroína y
Los individuos con trastorno de la personalidad obsesi- morfina desarrollan rápidamente tolerancia y dependencia
vo-compulsiva pueden tener tantas dificultades para decidir (PIR 19, 121).
qué tareas tienen prioridad o cuál es la mejor manera de Así mismo, el trastorno por consumo de opiáceos puede co-
hacer una tarea en particular, que no comiencen ninguna. menzar a cualquier edad, pero los problemas asociados con
Tienen tendencia a molestarse o a enfadarse en situaciones el uso de opiáceos se observan con mayor frecuencia en la
en las que no son capaces de mantener el control de su en- adolescencia tardía o en los primeros años de la veintena. El
torno físico o interpersonal, aunque la ira normalmente no curso es abrupto con remisiones y recaídas. Las tasas de mor-
se expresa directamente. talidad a largo plazo pueden ser de hasta un 2 % por año
Los individuos con este trastorno pueden estar especialmen- y alrededor del 20-30 % de las personas con este trastorno
te atentos a su estatus relativo en las relaciones de dominio y logra la abstinencia a largo plazo.
sumisión, y mostrar excesiva deferencia a una autoridad que
respetan, y resistencia exagerada a las figuras de autoridad
que no respetan.

99
TEMA 4. TRASTORNOS DEL ESPECTRO DE LA TEMA 12. TRASTORNOS ALIMENTARIOS Y DE
ESQUIZOFRENIA Y OTROS TRASTORNOS PSI- LA INGESTIÓN DSM 5
CÓTICOS DSM 5
PÁG COLUMNA PÁRRAFO
PÁG COLUMNA PÁRRAFO
57 TOMO II 1 2
77 TOMO I Tabla 17. Alteracio- Añadir en el apar- (Debajo de la ta-
nes neurológicas en tado ESTRUCTU- bla 11)
la esquizofrenia. RALES de la Tabla
17. Alteraciones
neurológicas en la
AMPLIACIÓN
esquizofrenia (PIR
19, 152).
12.5 Bulimia nerviosa
AMPLIACIÓN
Epidemiología y curso
4.2. Esquizofrenia
Es un trastorno que no se detecta con facilidad porque los
ESTRUCTURALES (PIR 99, 98) pacientes intentan ocultarlo y porque además el peso puede
• Aumento en surcos corticales. encontrarse dentro de unos valores normales (PIR 99, 124).
• Disminución del volumen del tálamo. La incidencia es mayor en el sexo femenino (90% de los ca-
• Disminución cerebral, especialmente en lóbulos sos). La prevalencia en mujeres está en torno a 1-3%.
frontales y temporales (hipocampo, giro parahipocampal, Aparece entre los 18 y 25 años y está más distribuido social-
amígdala y segmento palidal interno). mente que la anorexia nerviosa, siendo más prevalente que
• Disminución craneal. la anorexia en mujeres jóvenes (PIR 19, 127).

TEMA 7. TRASTORNO OBSESIVO-COMPULSI-


VO Y TRASTORNOS RELACIONADOS DSM 5

PÁG COLUMNA PÁRRAFO


148 TOMO I 1 6.
Modificar el
2º párrafo del
apartado “Cur-
so”, para añadir
información:
cambiarlo por el
texto aportado en
esta tabla.

AMPLIACIÓN
7.2. Trastorno obsesivo-compulsivo

La OMS sitúa el TOC dentro de una de las diez condiciones


más incapacitantes. El British National Psychiatric Morbidity
Study del 2000 apuntó que las personas con TOC presenta-
ban gran interferencia a nivel social y laboral. Los síntomas
específicos pueden originar obstáculos específicos, pudiendo
generar problemas en las relaciones interpersonales, fracaso
escolar o pérdida de empleo. Es por ello por lo que podría
decirse que, en comparación con trastornos de ansiedad y
depresión, los pacientes con TOC tienen mayor probabilidad
de ser solteros y estar en paro (PIR 19, 155).
Son poco frecuentes las remisiones completas y después de
una remisión puede existir recaídas producidas por ansiedad,
depresión (gran comorbilidad (PIR 15, 226)), fatiga u otros
factores. El deterioro puede fluctuar de moderado a muy
grave y en algunos casos incapacita totalmente al sujeto.

100
RESEÑAS

101
CLÍNICA INFANTIL
RESEÑAS

- Insomnio primario.
- Hipersomnia primaria.
- Narcolepsia.
PÁG COLUMNA PÁRRAFO - Trastorno del sueño relacionado con la respiración.
- Trastorno del ritmo circadiano (antes trastorno del ritmo
sueño-vigilia).
- Disomnia no especificada.
• Parasomnias (PIR19,84)
TEXTO DEL MANUAL
FUERA DE TEMARIO
PÁG COLUMNA PÁRRAFO
88 2 Tabla 1
TEXTO AMPLIADO

Tabla de Miedos citada por Bragado y Morris y Kratochwill


(PIR19,69) TEXTO DEL MANUAL

DSM 5 TRASTORNOS DISRUPTIVOS, DEL CONTROL DE LOS


IMPULSOS Y DE LA CONDUCTA
PÁG COLUMNA PÁRRAFO
Trastorno explosivo intermitente
93 1 2
(solo se menciona el trastorno en la siguiente tabla)

TEXTO DEL MANUAL Tabla 1. Comparativa DSM-IV-TR vs. DSM 5. (PIR 19, 85).

8.3. TRASTORNO DISOCIAL (DSM-IV-TR) – TRASTORNO DE


CONDUCTA (DSM 5)
TEXTO AMPLIADO
Teoría de la coacción (Patterson)
Los criterios diagnósticos del trastorno explosivo intermiten-
- La Trampa del reforzamiento negativo (PIR 08, 70; PIR 13, te aparecen en el MANUAL DE PSICOLOGÍA CLÍNICA TOMO
28). En los procesos de coacción presentes en las relaciones II (página 119):
entre padres e hijos destaca el papel explicativo del refor-
zamiento negativo (PIR 16, 210). Los padres ante las con- Tabla 5. Trastorno explosivo intermitente (DSM 5) (PIR19,
ductas del niño no actúan correctamente y ceden ante sus 85).
deseos. A corto plazo resulta agradable para ambas partes,
pero a largo plazo se convierte en un grave problema. Los
padres al ceder ante sus hijos caen en la “trampa del refor-
zamiento”, fortaleciendo este tipo de conductas negativas y PÁG COLUMNA PÁRRAFO
coercitivas. (PIR19,78) 127 2 5

PÁG COLUMNA PÁRRAFO TEXTO DEL MANUAL

115 1 Tabla 2 13.6 TRASTORNOS DEL ESTADO DE ÁNIMO EN LA INFANCIA

El DSM 5 ha incluido dentro de los Trastornos Depresivos el


TEXTO DEL MANUAL Trastorno de la Disregulación Disruptiva del Estado de Áni-
mo, que según el DSM es un intento de clarificar los tras-
TABLA 2.TRASTORNOS DEL SUEÑO EN DSM. tornos bipolares, evitando así el sobrediagnóstico y excesivo
DSM-IV-TR TRASTORNOS DEL SUEÑO tratamiento (PIR19, 86).

TRASTORNOS PRIMARIOS DEL SUEÑO


• Disomnias (PIR 97, 151):

102
TEXTO AMPLIADO EN TRASTORNO DE ANSIEDAD POR SEPARACIÓN

Los criterios diagnósticos del trastorno de disregulación des- “Las niñas manifiestan una mayor reticencia o evitación a
tructiva del estado de ánimo aparecen en el MANUAL DE asistir a la escuela que los niños. La expresión indirecta del
PSICOLOGÍA CLÍNICA TOMO I (página 94): miedo a la separación puede ser más común en los varones
que en las mujeres, por ejemplo, a través de una limitación
Tabla 10: Trastorno de disregulación destructiva del estado en el funcionamiento independiente, rechazo a estar fuera
de casa solo, angustia cuando el cónyuge o los hijos hacen
de ánimo (DSM 5) (PIR 19, 86).
cosas de forma independiente, o cuando el contacto con el
cónyuge o los hijos no es posible” (PIR19, 139)

PÁG COLUMNA PÁRRAFO


116 1 2 PÁG COLUMNA PÁRRAFO
54 2 6
TEXTO DEL MANUAL

12.3 PARASOMNIAS
TEXTO DEL MANUAL
Terrores nocturnos
4.5. TRASTORNO FONOLÓGICO O DISLALIA
Los padecen en torno al 1-6% de los niños, siendo más fre-
cuente en varones. Típicos entre los 2-3 años, van desapa- CLASIFICACIÓN ETIOLÓGICA
reciendo de manera espontánea al llegar a la adolescencia.
Se producen en el primer tercio de la noche, no en las fases - Disartrias
REM, sino en las fases 3-4 del sueño lento (sueño profundo) Alteración o defecto de la articulación debido a lesiones en
(PIR19, 112). el control muscular. Puede deberse a una lesión en el sistema
nervioso central, periférico o en ambos. (PIR19, 144)

PÁG COLUMNA PÁRRAFO


PÁG COLUMNA PÁRRAFO
11 1 Tabla 8 Párrafo 3 (Cri-
terio C) 28 Tabla 2

TEXTO DEL MANUAL


TEXTO DEL MANUAL
11.3. MUTISMO SELECTIVO
2.2. CRITERIOS DIAGNÓSTICOS.
MUTISMO SELECTIVO (DSM-IV-TR Y DSM 5)
DISCAPACIDAD INTELECTUAL (TRASTORNO DEL DESARRO-
C. La duración de la alteración es de por lo menos 1 mes (no LLO INTELECTUAL) (DSM 5) (PIR19, 177)
limitada al primer mes de escuela) (PIR19, 138)
*La respuesta no figura tal cual en un párrafo, sino que hace
referencia a la tabla completa.

PÁG COLUMNA PÁRRAFO

TEXTO DEL MANUAL

FUERA DE TEMARIO

TEXTO AMPLIADO

TEXTO EXTRAÍDO DE DSM 5. PÁGINA 193, DENTRO DEL EPÍ-


GRAFE “ASPECTOS DIAGNÓSTICOS RELACIONADOS CON
EL GÉNERO”

103
CLÍNICA INFANTIL
AMPLIACIONES

edad adulta hasta el punto que el diagnóstico de discapaci


AMPLIACIONES dad intelectual podría dejar de ser aplicable.

FACTORES DE RIESGO Y PRONÓSTICO


TEMA 2: DISCAPACIDAD INTELECTUAL Genéticos y fisiológicos.
-
DISCAPACIDAD INTELECTUAL - Causas prenatales: síndromes genéticos, errores
innatos del metabolismo, malformaciones cerebrales, en-
CARACTERÍSTICAS DIAGNÓSTICAS. fermedad materna (incluida la patología de la placenta) e
Se recogen los criterios diagnósticos del cuadro. influencias ambientales (p. ej., alcohol, otras drogas, tóxicos,
teratógenos).
CARACTERÍSTICAS ASOCIADAS QUE APOYAN EL DIAGNÓSTICO - Causas perinatales: acontecimientos relacionados
La discapacidad intelectual es una afección heterogénea con con el preparto y el parto que conducen a la encefalopatía
múltiples causas. Se pueden asociar dificultades para el juicio neonatal.
social, la evaluación de riesgo, la autogestión del compor- - Causas posnatales: daño hipóxico isquémico, daño
tamiento, las emociones o las relaciones interpersonales, o cerebral traumático, infecciones, trastornos desmielinizan-
la motivación en los entornos de la escuela o el trabajo. La tes, trastornos convulsivos, privación social grave y crónica y
falta de habilidades para la comunicación puede predispo- síndromes tóxico-metabólicos e intoxicaciones (p. ej., plomo,
ner a comportamientos disruptivos y agresivos. La credulidad mercurio).
muchas veces es un rasgo que incluye la ingenuidad en las
situaciones sociales y la tendencia a ser fácilmente arrastrado ASPECTOS DIAGNÓSTICOS RELACIONADOS CON LA CULTURA
por otros. Los individuos diagnosticados de discapacidad in- La discapacidad intelectual ocurre en todas las razas y cul-
telectual con trastornos mentales concurrentes tienen riesgo turas.
de suicidio. La valoración de los pensamientos suicidas es
esencial en el proceso de evaluación. Debido a la falta de ASPECTOS DIAGNÓSTICOS RELACIONADOS CON EL GÉNERO
conciencia del riesgo y el peligro, el porcentaje de lesiones En general, los varones tienen más posibilidades que las mu-
accidentales puede ser elevado. jeres de ser diagnosticados de formas de discapacidad inte-
lectual leves (1,6:1) y graves (1,2:1). Sin embargo, la propor-
PREVALENCIA 1% población general, las tasas varían según ción entre los sexos varía mucho en los estudios publicados.
la edad. La prevalencia de discapacidad intelectual grave es
aproximadamente del 6 por 1000. MARCADORES DIAGNÓSTICOS
Su evaluación exhaustiva incluiría una evaluación de la capa-
DESARROLLO Y CURSO cidad intelectual y del funcionamiento adaptativo, la iden-
El inicio de la discapacidad intelectual se produce durante tificación de las etiologías genéticas y no genéticas, la eva-
el período de desarrollo. La edad y los rasgos característi- luación de las afecciones médicas asociadas (p. ej., parálisis
cos al inicio dependen de la etiología y de la gravedad de cerebral, trastorno convulsivo), y la evaluación de los tras-
la disfunción cerebral. Cuando la discapacidad intelectual se tornos mentales, emocionales y conductuales concurrentes.
asocia a un síndrome genético puede haber un aspecto físico
característico (como, p. ej., en el síndrome de Down). Algu- DIAGNÓSTICO DIFERENCIAL
nos síndromes tienen un fenotipo conductual, lo que hace Trastornos neurocognitivos mayores y leves. Los trastornos
referencia a comportamientos específicos que son caracterís- neurocognitivos se caracterizan por una pérdida del fun-
ticos de un trastorno genético particular (p. ei., el síndrome cionamiento cognitivo. El trastorno neurocognitivo mayor
de Lesch-Nyhan). En las formas adquiridas, el inicio puede puede ser concurrente con la discapacidad intelectual (p.
ser abrupto después de una enfermedad, como una menin- ej., un individuo con síndrome de Down que desarrolla una
gitis o una encefalitis, o de un traumatismo craneal cuando enfermedad de Alzheimer, o un individuo con discapacidad
éste se produce durante el período de desarrollo. Aunque, intelectual que pierde más capacidad cognitiva después de
en general, la discapacidad intelectual no es progresiva, en una lesión craneal). En tales casos, se pueden realizar los
algunos trastornos genéticos (p. ej., el síndrome de Rett) hay diagnósticos de discapacidad intelectual y de trastorno neu-
períodos de empeoramiento seguidos de estabilización y en rocognitivo.
otros (p. ej., el síndrome de San Filippo) se produce un em-
peoramiento progresivo de la función intelectual. Después Trastornos de la comunicación y trastorno específico del
de la primera infancia, el trastorno generalmente es para aprendizaje. Estos trastornos del neurodesarrollo son espe-
toda la vida, aunque los niveles de gravedad pueden cambiar cíficos de los dominios de la comunicación y del aprendizaje
con el tiempo. El curso puede verse influido por las afeccio- y no muestran deficiencias del comportamiento intelectual y
nes médicas o genéticas y por la presencia de cuadros con- adaptativo. Pueden concurrir con la discapacidad intelectual.
currentes (p. ej., deficiencias auditivas o visuales, epilepsia). En tal caso, se realizan los dos diagnósticos.
Las intervenciones tempranas y continuadas pueden mejorar Trastorno del espectro autista. La discapacidad intelectual es
el funcionamiento adaptativo durante toda la infancia y la frecuente entre los individuos con trastorno del espectro au-

104
tista. Es esencial la evaluación apropiada del funcionamien- señalar con las manos o para mostrar o llevar objetos cuyo
to intelectual en el trastorno del espectro autista, así como interés pueda compartirse con otros, o por incapacidad para
la reevaluación durante el período de desarrollo, ya que las seguir cómo otro señala con las manos o dirige la mirada.
puntuaciones del CI en el trastorno del espectro autista pue- Los individuos pueden aprender algunos gestos funcionales,
den ser inestables, particularmente en la primera infancia. pero su repertorio es más pequeño que el de otras personas
Comorbilidad y a menudo no utilizan gestos expresivos espontáneamente
en la comunicación. Entre los adultos con un lenguaje fluido,
En la discapacidad intelectual son frecuentes las afecciones las difcultades para coordinar la comunicación no verbal con
mentales, del neurodesarrollo, médicas y físicas, producién- el habla pueden dar la impresión de un “lenguaje corporal”
dose algunas afecciones (p. ej., trastornos mentales, parálisis extraño, rígido o exagerado durante las interacciones. Las
cerebral y epilepsia) con frecuencias tres a cuatro veces ma- alteraciones pueden ser relativamente sutiles dentro de los
yores que las de la población general. Los trastornos menta- dominios individuales (por ejemplo, alguien puede mante-
les y del neurodesarrollo concurrentes con mayor frecuencia ner contacto con los ojos relativamente bien mientras habla),
son el trastorno por déficit de atención/hiperactividad, los pero van a ser más pronunciadas al integrarse escasamente,
trastornos depresivo y bipolar, los trastornos de ansiedad, el en la comunicación social, el contacto ocular,los gestos, la
trastorno del espectro autista, el trastorno de movimientos postura corporal, la prosodia y la expresión facial.
estereotipados (con o sin comportamientos auto-lesivos), los
trastornos del control de los impulsos, y el trastorno neuro- Las deficiencias para desarrollar, mantener y entender las
cognitivo mayor. El trastorno de depresión mayor puede pro- relaciones se deberían valorar según la normalidad para la
ducirse en todo el espectro de gravedad de la discapacidad edad, el género y la cultura. Puede haber un interés social
intelectual. El comportamiento autolesivo requiere atención ausente, reducido o atípico, manifestado por el rechazo a
diagnóstica rápida y puede justificar un diagnóstico inde- las otras personas, la pasividad o un trato inapropiado que
pendiente de trastorno de movimientos estereotipados. Los parece agresivo o destructivo. Estas dificultades son parti-
individuos con discapacidad intelectual, particularmente los cularmente evidentes en los niños pequeños, que muchas
que tienen una discapacidad intelectual más grave, pueden veces muestran falta de juegos sociales y de imaginación
también mostrar hostilidad y comportamientos disruptivos, compartidos y después insisten en jugar según unas reglas
con daños a terceros o destrucción de propiedades. muy inflexibles. Los individuos más mayores pueden tener
  problemas para entender qué comportamientos se conside-
TEMA 3: TRASTORNOS DEL ESPECTRO ran apropiados en una situación, pero no en otras, o las dife-
rentes maneras en que el lenguaje puede utilizarse para co-
AUTISTA DSM 5 Y OTROS TRASTOR- municar (ironía, mentiras piadosas). Puede haber una obvia
NOS GLOBALES DE LA INFANCIA – preferencia por las actividades solitarias o las interacciones
con gente mucho más pequeña o mayor. Frecuentemente
existe el deseo de establecer amistades sin una idea comple-
TRASTORNO DEL ESPECTRO AUTISTA ta o realista de lo que supone la amistad (por ejemplo, amis-
tades unilaterales o basadas únicamente en intereses espe-
CARACTERÍSTICAS DIAGNÓSTICAS ciales compartidos). También es importante tener en cuenta
Las deficiencias verbales y no verbales de la comunicación las relaciones con los hermanos, los compañeros de trabajo
social tienen diferentes manifestaciones según la edad, el y los cuidadores (en términos de reciprocidad).
nivel intelectual y las capacidades lingüísticas del individuo,
además de otros factores, como la historia del tratamiento En cuanto a los co0mportamientos, intereses o actividades
y las ayudas actuales. Muchos individuos tienen deficiencias restringidos, algunas fascinaciones y rutinas pueden tener
lingüísticas que varían entre la ausencia completa del habla relación con una aparente hiper o hiporreactividad a los es-
por retrasos del lenguaje, hasta la escasa comprensión del tímulos sensoriales. Son frecuentes las reacciones extremas
habla de los demás, la ecolalia o el lenguaje poco natural y y los rituales relacionados con el sabor, el olor, la textura o
demasiado literal. Aun cuando estén intactas las capacidades la apariencia de los alimentos, y las restricciones alimentarias
formales del lenguaje (por ejemplo, vocabulario, gramática), excesivas, que pueden ser una característica de la presenta-
el uso de este en el TEA estaría deteriorado para la comuni- ción del trastorno del espectro autista.
cación social recíproca.
Muchos adultos con TEA sin discapacidad intelectual o del
En los adultos sin discapacidades intelectuales o retrasos del lenguaje aprenden a suprimir el comportamiento repetitivo
lenguaje, las deficiencias de la reciprocidad socioemocional en público. Los intereses especiales pueden ser una fuente
pueden ser más evidentes en la dificultad para procesar y de placer y motivación, y pueden ofrecer vías para la educa-
responder a las señales sociales complejas (por ejemplo, ción y el empleo en fases posteriores de la vida. Los criterios
cuándo y cómo unirse a una conversación, cosas que no se diagnósticos se pueden cumplir si los patrones de compor-
deberían decir). Los adultos que han desarrollado estrategias tamiento, intereses o actividades restringidos y repetitivos ya
de compensación para algunas dificultades sociales siguen estaban claramente presentes durante la infancia o en algún
teniendo problemas en las situaciones nuevas o sin ayuda, momento del pasado, aunque no continúen.
y sufren por el esfuerzo y la ansiedad de tener que calcular
conscientemente lo que resulta socialmente intuitivo para la CARACTERÍSTICAS ASOCIADAS QUE APOYAN EL DIAG-
mayoría de las personas. NÓSTICO
Muchos pacientes con TEA también tienen un deterioro in-
Una característica temprana del TEA es una alteración de la telectual y/o del lenguaje (por ejemplo, lentitud al hablar,
atención compartida que se manifiesta por incapacidad para comprensión del lenguaje peor que la producción). Incluso

105
los que tienen una inteligencia normal o alta tienen un perfil le), patrones de juego extraños (por ejemplo, llevar juguetes
de capacidades desnivelado. La brecha entre las habilidades de un lado a otro sin jugar con ellos) y patrones de comuni-
intelectuales y funcionales adaptativas es, a menudo, gra- cación inusuales (por ejemplo, saber el alfabeto pero no res-
de. Muchas veces existen deficiencias motoras, como una ponder a su propio nombre). Puede sospecharse una sorde-
manera de caminar extraña, torpeza y otros signos motores ra, pero normalmente se descarta. Durante el segundo año,
anormales (por ejemplo, caminar de puntillas). Pueden pro- los comportamientos extraños y repetitivos y la ausencia de
ducirse autolesiones y son más comunes los comportamien- juego característica se hacen más evidentes. Como muchos
tos disruptivos/difíciles en los niños y en los adolescentes con niños pequeños de desarrollo normal tienen fuertes preve-
TEA, que en aquellos con otros trastornos, incluida la disca- rencias y les gusta la repetición (como comer los mismos ali-
pacidad intelectual. Los adolescentes y adultos con TEA tie- mentos, mirar el mismo vídeo varias veces), puede ser difícil
nen tendencia a la ansiedad y depresión. Algunos individuos distinguir los comportamientos restringidos y repetitivos tí-
presentan un comportamiento motor similar a la catatonía picos del TEA en los niños de edad preescolar. La distinción
(ir más despacio y “congelándose” en medio de una acción), clínica se basa en el tipo, la frecuencia y la intensidad del
pero estos no alcanzan normalmente la magnitud que se comportamiento (por ejemplo, un niño que alinea objetos
observa en un episodio catatónico. Sin embargo, es posible durante horas cada día y presenta mucho malestar si se le
que los individuos con TEA presenten un deterioro marcado mueve cualquiera de esos objetos).
de los síntomas motores y muestren un episodio catatónico
completo con síntomas como el mutismo, las posturas, las El TEA no es un trastorno degenerativo y es normal que el
muecas y la flexibilidad cérea. El periodo de riesgo para la ca- aprendizaje y la compensación continúen durante toda la
tatonía comórbida parece ser mayor durante la adolescencia. vida. Los síntomas a menudo son muy marcados durante
la infancia temprana y en los primeros años escolares, con
PREVALENCIA mejoría del desarrollo típico en la infancia más tardía, al me-
En los últimos años, las frecuencias descritas para el TEA en nos en algunas áreas (como puede ser el aumento de interés
EEUU y otros países han llegado a cerca del 1% de la pobla- por la interacción social). Una pequeña proporción de indivi-
ción, con estimaciones parecidas en las muestras infantiles duos se deteriora conductualmente durante la adolescencia,
y de adultos. No está claro si las tasas más altas reflejan la mientras que la mayoría mejora. Solamente una minoría de
expansión de los criterios diagnósticos del DSM-IV para in- individuos con TEA vive y trabaja independientemente en la
cluir los casos subumbrales, un aumento de la conciencia edad adulta. Los que lo hacen tienden a tener mayores ca-
del trastorno, las diferentes metodologías de estudio o un pacidades lingüísticas e intelectuales y logran encontrar un
aumento real de la frecuencia del TEA. área que compagina sus especiales intereses y capacidades.
En general, los individuos co0n un nivel menor de deterioro
DESARROLLO Y CURSO pueden seguir siendo socialmente ingenuos y vulnerables,
También se deberían describir la edad y el patrón de inicio pueden tener dificultades para organizar las exigencias prác-
en el TEA. Los síntomas se reconocen normalmente duran- ticas sin ayuda y pueden tener tendencia a la ansiedad y la
te el segundo año de vida (12-24 meses de edad), pero se depresión. Muchos adultos describen que utilizan estrategias
pueden observar antes de los 12 meses si los retrasos del de compensación y afrontamiento para esconder sus dificul-
desarrollo son graves, o notar después de los 24 meses si lo tades en público, pero sufren el estrés y esfuerzo de mante-
síntomas son más sutiles. La descripción del patrón de inicio ner una fachada social aceptable. Se sabe muy poco sobre la
puede incluir información acerca de los retrasos tempranos edad avanzada en el TEA.
del desarrollo o de cualquier pérdida de capacidades sociales
o de lenguaje. Normalmente, esto ocurriría entre los 12 y Algunos individuos consultan para un primer diagnóstico en
24 meses de edad, y se debe plantear la distinción de los la edad adulta, quizás impulsados por el diagnóstico de au-
casos raros de regresión del desarrollo que ocurren después tismo en un niño de la familia o un empeoramiento de las
de, por lo menos, 2 años de desarrollo normal (previamente relaciones en el trabajo o en casa. En tales casos puede ser
descrito como trastorno desintegrativo de la infancia). difícil obtener una historia detallada del desarrollo y sería im-
Las características conductuales del TEA empiezan a ser evi- portante tener en cuenta las dificultades descritas por el pro-
dentes en la primera infancia, presentando algunos caos fal- pio paciente. Cuando la observación clínica sugiere que ac-
ta de interés por la interacción social durante el primer año tualmente se cumplen los criterios, se puede diagnosticar el
de vida. Algunos niños con TEA presentan una paralización TEA siempre que no haya pruebas de una buena capacidad
o una regresión del desarrollo con deterioro gradual o relati- social y de comunicación durante la infancia. Por ejemplo,
vamente rápido de los comportamientos sociales o el uso del la descripción (por los padres u otro familiar) de que el indi-
lenguaje durante los 2 primeros años de vida. Estas pérdidas viduo tenía amistades recíprocas normales y continuadas, y
son raras en otros trastornos y pueden utilizarse como “se- buenas habilidades para la comunicación no verbal durante
ñal de alerta” para el tEA. Mucho más infrecuentes son – y toda la infancia, descartaría el diagnóstico de TEA; sin em-
justifican un estudio médico más extenso – las pérdidas de bargo, la ausencia de información en sí no sería motivo para
capacidades más allá de la comunicación social (por ejemplo, descartar el diagnóstico.
las pérdidas del autocuidado, de la capacidad de ir al lavabo,
de las capacidades motoras) o las que ocurren después del Las manifestaciones en deterioro social y de la comunica-
segundo aniversario. ción, y los comportamientos restringidos/repetitivos que de-
finen el TEA son claros en el periodo de desarrollo. En etapas
Los primeros síntomas del TEA implican frecuentemente un posteriores de la vida, la intervención y la compensación, y
retraso en el desarrollo del lenguaje, acompañado a menudo también los apoyos actuales, pueden enmascarar estas difi-
por falta de interés social o interacciones sociales inusuales cultades en, al menos, algunos contextos. Sin embargo, los
(por ejemplo, llevar a alguien de la mano sin intentar mirar- síntomas son suficientes como para causar deterioro actual

106
en el funcionamiento social u ocupacional, o en otras áreas. tener dificultades para establecerse por independencia por la
continua rigidez y sus problemas para aceptar novedades.
FACTORES DE RIESGO Y PRONÓSTICO Muchos individuos con TEA, incluso sin discapacidad intelec-
Los factores pronósticos mejor establecidos para el resultado tual, tienen un funcionamiento psicosocial bajo en la edad
individual dentro del TEA son la presencia o ausencia de una adulta, según indican medidas como vivir independiente-
discapacidad intelectual asociada, el deterioro del lenguaje mente y el empleo remunerado. Las consecuencias funciona-
(por ejemplo, un lenguaje funcional al llegar a los 5 años les en la edad avanzada se desconocen, pero el aislamiento
de edad es un buen signo pronóstico) y los otros problemas social y los problemas de comunicación (como la poca bús-
añadidos de salud mental. La epilepsia, como diagnóstico queda de ayuda) probablemente tengan consecuencias para
comórbido, se asocia a una mayor discapacidad intelectual y la salud en la madurez tardía.
a una menor capacidad verbal.
DIAGNÓSTICO DIFERENCIAL
- Ambiental: existen diversos factores de riesgo ines- - Síndrome de Rett. Puede observarse una alteración
pecíficos, como la edad avanzada de los padres, el bajo peso de la interacción social durante la fase regresiva del síndrome
al nacer o la exposición fetal al valproato, que podrían con- de Rett (típicamente entre 1 y 4 años de edad); así pues, una
tribuir al riesgo de presentar TEA. proporción sustancia de las niñas pequeñas afectadas po-
- Genético y fisiológico. Las estimaciones de la here- drían debutar con una presentación que cumpla los criterios
dabilidad del TEA varían entre el 37 y más del 90% basándo- diagnósticos del TEA. Sin embargo, después de este periodo,
se en la tasa de concordancia entre gemelos. Actualmente la mayoría de los individuos con síndrome de Rett mejora sus
hasta el 15% de los casos de TEA parece asociarse a una habilidades para la comunicación social y los rasgos autistas
mutación genética conocida, siendo muchas las variantes de dejan de ser tan problemáticos. Por tanto, se debería tener
novo, del número de copias y las mutaciones de novo en ge- en cuenta el TEA solamente si se cumplen todos los criterios
nes específicos que se asocian al trastorno en las diferentes diagnósticos.
familias. Sin embargo, incluso si el TEA está asociado a una - Mutismo selectivo. En el mutismo selectivo, el de-
mutación genética conocida, esta no parece ser completa- sarrollo temprano no está habitualmente alterado. El niño
mente penetrante. El riesgo en el resto de los casos parece afectado muestra normalmente capacidades de comunica-
ser poligénico, quizás con centenares de loci genéticos que ción apropiadas en ciertos contextos y entornos. Incluso en
realizan contribuciones relativamente pequeñas. los entornos donde el niño está mudo, la reciprocidad social
no está deteriorada y no se presentan patrones de compor-
ASPECTOS DIAGNÓSTICOS RELACIONADOS CON LA CULTURA. tamiento restringidos o repetitivos.
Aunque existen diferencias culturales respecto a la norma- - Trastornos del lenguaje y de la comunicación so-
lidad de la interacción social, la comunicación no verbal y cial (pragmática). En algunas formas del trastorno del len-
las relaciones, los individuos con TEA tienen una alteración guaje puede haber problemas de comunicación y algunas
marcadamente distinta de la normalidad dentro de su con- dificultades sociales secundarias. Sin embargo, el trastorno
texto cultural. Diversos factores culturales y socioeconómicos del lenguaje específico normalmente no está asociado con
pueden afectar a la edad del reconocimiento o del diagnós- una comunicación no verbal anormal, ni con la presencia de
tico; por ejemplo, en EEUU pueden producirse diagnósticos patrones de comportamiento, intereses o actividades res-
tardíos o infradiagnósticos de TEA entre los niños afroame- tringidos y repetitivos. Si un individuo muestra deterioro de
ricanos. la comunicación social y las interacciones sociales, pero no
muestra comportamientos o intereses restringidos y repetiti-
ASPECTOS DIAGNÓSTICOS RELACIONADOS CON EL GÉNERO. vos, se pueden cumplir los criterios de trastorno de la comu-
El TEA se diagnostica cuatro veces más frecuentemente en nicación social (pragmática) en vez de los del TEA. El diag-
el sexo masculino que en el femenino. En muestras clínicas, nóstico de TEA sustituye al de trastorno de la comunicación
las niñas tienden a tener más probabilidades de presentar social (pragmática) siempre que se cumplan los criterios de
discapacidad intelectual acompañante, lo que sugiere que, TEA. Habría que asegurarse de investigar bien los comporta-
en las niñas sin deterioro intelectual acompañante o retrasos mientos restringidos o repetitivos del pasado y del momento
del lenguaje, el trastorno podría no reconocerse; quizás por presente.
ser más sutil la manifestación de las dificultades sociales y de
comunicación. - Discapacidad intelectual (trastorno del desarrollo in-
telectual) sin TEA. La discapacidad intelectual sin TEA puede
CONSECUENCIAS FUNCIONALES DEL TEA. ser difícil de diferenciar del TEA en niños muy pequeños. Los
En los niños pequeños con TEA, la falta de habilidades so- individuos con discapacidad intelectual, que no han desarro-
ciales y de comunicación puede impedir el aprendizaje, par- llado capacidades del lenguaje o simbólicas, también consti-
ticularmente el aprendizaje a través de la interacción social o tuyen un reto para el diagnóstico diferencial, ya que los com-
en las situaciones con compañeros. En casa, la insistencia en portamientos repetitivos a menudo se presentan también en
las rutinas y la aversión a los cambios, así como las sensibili- estos individuos. Un diagnóstico de TEA en un individuo con
dades sensoriales, pueden interferir con el comer y el dormir discapacidad intelectual sería correcto si la comunicación y
y hacer muy difíciles los cuidados rutinarios (por ejemplo, la interacción social están significativamente deterioradas en
cortes de pelo, intervenciones dentales). Las habilidades comparación con el nivel de desarrollo de las capacidades
adaptativas están típicamente por debajo del CI medio. Las no verbales del individuo (p.ej., habilidades motoras finas,
dificultades extremas para planificar, organizar y enfrentarse resolución de problemas no verbales). En cambio, la disca-
a los cambios impactan negativamente en los logros acadé- pacidad intelectual sería el diagnóstico apropiado si no hay
micos, incluso en los estudiantes con inteligencia por encima ninguna discrepancia aparente entre el nivel de las capaci-
de la media. Durante la edad adulta, estos individuos pueden dades sociales/comunicativas y el de las demás capacidades

107
intelectuales. TEMA 4: TRASTORNOS DE LA COMUNI-
- Trastorno de movimientos estereotipados. Las es- CACIÓN DSM 5
tereotipias se incluyen entre las características diagnósticas
de TEA y, por tanto, no se realiza un diagnóstico adicional TRASTORNO DEL LENGUAJE
de trastorno de movimientos estereotipados cuando tales
comportamientos repetitivos se pueden explicar mejor por CARACTERÍSTICAS DIAGNÓSTICAS
la presencia de TEA. Sin embargo, cuando las estereotipias Las características diagnósticas centrales del trastorno del
causan autolesiones y llegan a ser uno de los objetivos del lenguaje son las dificultades para la adquisición y uso del
tratamiento, los dos diagnósticos podrían ser apropiados. lenguaje debido a deficiencias de la comprensión o la pro-
- TDAH. Las anormalidades en la atención (excesiva- ducción del vocabulario, las estructuras gramaticales y el
mente centrado o fácilmente distraído) son frecuentes en los discurso. Las deficiencias son evidentes en la comunicación
individuos con TEA, al igual que la hiperactividad. Se debería hablada, escrita y el lenguaje de señas. El aprendizaje y uso
considerar el diagnóstico de TDAH si las dificultades aten- del lenguaje dependen de capacidades receptivas y expre-
cionales o la hiperactividad superan las que normalmente se sivas. La capacidad expresiva se refiere a la producción de
observan en los individuos de edad mental comparable. señales vocales, gestuales y verbales mientras que, la recep-
- Esquizofrenia. La esquizofrenia de inicio en la infan- tiva, comprende el proceso de recibir y comprender los men-
cia normalmente se desarrolla tras un periodo de desarrollo sajes lingüísticos. En cuanto a la evaluación hay que tener en
normal o casi normal. Se ha descrito un estado prodrómico cuenta que ambas capacidades puede tener una afectación
en el que se produce deterioro social y aparecen intereses y diferente. Por ejemplo, un individuo puede tener un lenguaje
creencias atípicas, lo que podría confundirse con las deficien- expresivo gravemente limitado sin tener casi ninguna defi-
cias sociales observadas en el TEA. Las alucinaciones y los de- ciencia en su lenguaje receptivo.
lirios, que son las características definitorias de la esquizofre-
nia, no son rasgos del TEA. Sin embargo, los clínicos deben El trastorno del lenguaje normalmente afecta al vocabula-
tener en cuenta la posibilidad de que los individuos con TEA rio y la gramática, limitando la capacidad del discurso. Las
interpretan de manera excesivamente concreta las preguntas primeras palabras y frases del niño es probable que se ini-
relacionadas con las características claves de la esquizofrenia cien de manera retrasada, con vocabulario más limitado y
(por ejemplo “¿Oyes voces cuando no hay nadie presente?” menos variado de lo esperado y frases más cortas y menos
“Sí [en la radio]”). complejas, con errores gramaticales, especialmente en tiem-
po pasado. Las deficiencias de comprensión frecuentemente
COMORBILIDAD se subestiman, ya que los niños pueden tener la habilidad
El TEA se asocia frecuentemente y a alteraciones estructu- de utilizar el contexto para inferir significados. Puede haber
rales del lenguaje (es decir, incapacidad para comprender y problemas para encontrar las palabras, las definiciones ver-
construir frases gramaticalmente correctas) que se deberían bales están empobrecidas o existe poca comprensión de los
registrar con los correspondientes especificadores en cada sinónimos, los significados múltiples o los juegos de palabras
caso. Muchos individuos con TEA tienen síntomas psiquiá- de forma apropiada para la edad y la cultura. Se manifiestan
tricos que no forman parte de los criterios diagnósticos del problemas para recordar nuevas palabras y frases, así como
trastorno (aproximadamente el 70% de los individuos con dificultades para seguir instrucciones más largas, para recor-
TEA puede tener un trastorno mental comórbido y el 40% dar series de datos verbales (por ejemplo, un número de te-
puede tener dos o más trastornos mentales comórbidos). léfono o una lista de la compra) y para recordar secuencias
Cuando se cumplen los criterios del TDAH y del TEA, se de- de sonidos nuevos (habilidad que podría ser importante para
berían realizar los dos diagnósticos. Este mismo principio se aprender nuevas palabras). Las dificultades del discurso se
aplica a los diagnósticos concurrentes de TEA y trastorno del muestran mediante una capacidad recudida para producir
desarrollo de la coordinación, trastorno de ansiedad, trastor- información adecuada sobre acontecimientos clave y narrar
no depresivo y otros diagnósticos comórbidos. Entre los indi- una historia coherente.
viduos que tienen deficiencias en el lenguaje o no verbales,
signos observables como serían las alteraciones del sueño o El diagnóstico de este trastorno se debería realizar basándo-
la alimentación y el incremento de conductas problemáti- se en la síntesis de los antecedentes de la persona, las obser-
cas deberían llevar a evaluar si existe ansiedad o depresión. vaciones clínicas directas en diferentes contextos (es decir,
Las dificultades específicas del aprendizaje (lectoescritura y casa, escuela y trabajo) y los resultados de las pruebas estan-
aritmética) son frecuentes, así como el trastorno del desarro- darizadas para medir la capacidad de lenguaje, que pueden
llo de la coordinación. Las afecciones médicas comúnmente usarse para orientar las estimaciones de la gravedad.
asociadas al TEA deberían ser anotadas bajo el especificador Características asociadas que apoyan el diagnóstico
“asociado con una conocida afección médica/genética o am- A menudo están presentes antecedentes familiares de tras-
biental/adquirida”. Tales afecciones médicas son la epilepsia, tornos del lenguaje. Los individuos, incluso los niños, pueden
los problemas del sueño y el estreñimiento. El trastorno de ser hábiles para adaptarse a su lenguaje limitado. Pueden
la conducta alimentaria de la ingesta de alimentos es una parecer tímidos o reticentes a hablar. Los individuos afecta-
forma característica de presentación del TEA bastante fre- dos pueden preferir comunicarse solamente con miembros
cuente. Las preferencias extremas y restringidas de alimentos de su familia u otros individuos conocidos. Aunque estos in-
pueden persistir en el tiempo. dicadores sociales no son diagnósticos de un trastorno del
lenguaje, si son notables y persistentes sí justifican la deriva-
ción para una evaluación completa. El trastorno del lenguaje,
particularmente las deficiencias expresivas, puede concurrir
  con un trastorno fonológico.

108
DESARROLLO Y CURSO guaje en un niño menor de 3 años puede ser un signo de
La adquisición del lenguaje está marcada por cambios que se trastorno del espectro autista (con regresión del desarrollo) o
producen desde su inicio en la niñez temprana hasta alcan- de una afectación neurológica específica, como el síndrome
zar el nivel de competencia del adulto, que aparece durante de Landau-Kleffner. Entre los niños mayores de 3 años, la
la adolescencia. Los cambios aparecen en las diferentes di- pérdida del lenguaje puede ser un síntoma de crisis epilép-
mensiones del lenguaje (sonidos, palabras, gramática, narra- tica, por lo que sería necesaria una evaluación diagnóstica
ciones/textos expositivos y habilidades conversacionales) en para excluir la presencia de una epilepsia (es decir, electroen-
aumento y sincronía según la edad. El trastorno del lengua- cefalogramas normales y de sueño).
je aparece durante el periodo de desarrollo temprano; sin
embargo, hay una variación considerable en la adquisición COMORBILIDAD
temprana del vocabulario y en las primeras combinaciones El trastorno del lenguaje se asocia de forma importante a
de palabras, y las diferencias individuales no son, como in- otros trastornos del neurodesarrollo, como el trastorno espe-
dicadores aislados, muy predictivas de los resultados poste- cífico del aprendizaje (lectoescritura y aritmética), el TDAH,
riores. A los 4 años, las diferencias individuales en cuanto a el trastorno del espectro autista y el trastorno del desarrollo
capacidad del lenguaje son más estables, se miden con más de la coordinación. También está asociado al trastorno de la
precisión y son muy predictivas de los resultados posteriores. comunicación social (pragmático). Existen a menudo ante-
El trastorno del lenguaje diagnosticado a partir de los 4 años cedentes familiares de trastornos del habla o del lenguaje.
suele ser estable en el tiempo y normalmente persiste has-
ta la edad adulta, aunque es probable que cambie el perfil TRASTORNO FONOLÓGICO
particular de puntos fuertes y débiles durante el curso del
desarrollo. CARACTERÍSTICAS DIAGNÓSTICAS
La producción fonológica describe la articulación clara de los
FACTORES DE RIESGO Y PRONÓSTICO fonemas (es decir, los sonidos individuales) que se combinan
Los niños con deficiencias receptivas del lenguaje tienen peor para crear palabras habladas. La producción fonológica re-
pronóstico que los que tienen deficiencias expresivas predo- quiere tanto el conocimiento fonológico de los sonidos del
minantes. Son más resistentes al tratamiento y se observan habla como la habilidad de coordinar los movimientos de las
frecuentemente dificultades en la comprensión de la lectura. articulaciones (es decir, mandíbula, lengua y labios) con la
Genético y fisiológico. Los trastornos del lenguaje son alta- respiración y la vocalización del habla. Los niños con dificul-
mente heredables y los miembros de la familia tienen más tades para la producción fonológica pueden presentar pro-
posibilidades de tener antecedentes de deficiencias lingüís- blemas de diferentes grados en el conocimiento fonológico
ticas. de los sonidos hablados o en la habilidad para coordinar los
movimientos del habla. El trastorno fonológico es, por tan-
DIAGNÓSTICO DIFERENCIAL to, heterogéneo en sus mecanismos subyacentes e incluye el
- Variaciones normales del lenguaje: el trastorno del trastorno fonológico y el de articulación. El trastorno fono-
lenguaje debe diferenciarse de las variaciones normales del lógico se diagnostica cuando la producción de los sonidos
desarrollo. Puede ser difícil hacer esta distinción antes de los del habla no es la que se esperaría de un niño para su edad
4 años de edad. Hay que tener en cuenta las variaciones del y etapa de desarrollo, y cuando las deficiencias no son el
lenguaje culturales/técnicas (es decir, los dialectos), sociales resultado de una alteración física, estructurar, neurológica
y regionales cuando se evalúan las deficiencias del lenguaje o auditiva. En los niños de 4 años con desarrollo normal, el
de un individuo. habla general debería ser inteligible, mientras que a los 2
años solo es comprensible el 50%.
- Audición u otra deficiencia sensorial: se tiene que
descartar la discapacidad auditiva como causa primaria de la CARACTERÍSTICAS ASOCIADAS QUE APOYAN EL DIAGNÓSTICO
dificultad con el lenguaje. Las deficiencias del lenguaje pue- El trastorno del lenguaje, en particular las deficiencias expre-
den asociarse a una discapacidad auditiva, a otra deficiencia sivas, pueden concurrir con el trastorno fonológico. A menu-
sensorial o a una deficiencia motora del habla. Cuando las do están presentes antecedentes familiares de trastornos del
deficiencias del lenguaje exceden las que normalmente se habla o del lenguaje.
asocian a estos problemas, se puede hacer el diagnóstico de
Trastorno del lenguaje. Si la capacidad de coordinar rápidamente los articuladores
está claramente mermada, puede haber antecedentes de re-
- Discapacidad intelectual (trastorno de desarrollo in- traso o incoordinación en la adquisición de las habilidades
telectual): un retraso del lenguaje es a menudo la presenta- que también utilizan los articuladores y la musculatura facial
ción característica de una discapacidad intelectual, y es po- relacionada; entre otras, estas habilidades incluyen masticar,
sible que no se pueda realizar el diagnóstico definitivo hasta mantener la boca cerrada y sonarse la nariz. Pueden estar
que el niño pueda completar las evaluaciones estandariza- alteradas otras áreas de coordinación motora, como en el
das. No se realizará un diagnóstico por separado a no ser trastorno del desarrollo de la coordinación. Dispraxia verbal
que las deficiencias del lenguaje sean claramente mayores es un término que también se utiliza para los problemas de
que las limitaciones intelectuales. la producción del habla.
- Trastornos neurológicos: el trastorno del lenguaje se El habla puede estar alterada de forma diferente en ciertas
puede producir asociado a los mismos, incluida la epilepsia afecciones genéticas (por ejemplo, el síndrome de Down, la
(por ejemplo, afasia adquirida o síndrome de Landau-Kleff- deleción del 22q, la mutación del gen FoxP2). Si se presen-
ner). tan, se deberían codificar también.

- Regresión de lenguaje: la pérdida del habla y el len-

109
DESARROLLO Y CURSO chos niños con mutismo selectivo muestran un habla normal
Aprender a producir los sonidos del habla claramente y con en los entornos “seguros”, como en casa o con los amigos
precisión, y aprender a producir un habla fluida son habili- cercanos.
dades del desarrollo. La articulación de los sonidos del habla
sigue un patrón de desarrollo que se refleja en las pruebas COMORBILIDAD
estandarizadas para cada edad. No es inusual que los niños No figura en el DSM5.
con un desarrollo normal acorten palabras y sílabas cuan-
do aprenden a hablar, pero la progresión en el dominio de TRASTORNO DE LA FLUIDEZ DE INICIO EN LA INFANCIA
la producción fonológica debería conducirles hacia un ha- (TARTAMUDEO)
bla mayoritariamente inteligible a los 3 años. Los niños con
trastorno fonológico siguen utilizando procesos inmaduros CARACTERÍSTICAS DIAGNÓSTICAS
de simplificación fonológica después de la edad en que la El alcance de la alteración de la fluidez varía en las distintas
mayoría puede emitir palabras claramente. situaciones y a menudo es más grave cuando la comunica-
Al llegar a los 7 años se debería producir la mayoría de los ción está sujeta a algún tipo de presión (como hacer una pre-
sonidos del habla con claridad, y la mayoría de palabras se sentación en el colegio o realizar una entrevista de trabajo).
debería pronunciar correctamente conforme a la normali- La disfluencia muchas veces está ausente durante la lectura
dad para la edad y la comunidad. Cuando están implicados oral y al cantar o hablar con objetos inanimados o mascotas.
múltiples sonidos en una mala articulación, lo apropiado se- Características asociadas que apoyan el diagnóstico
ría quizá centrarse en algunos como parte de un plan para Se puede desarrollar una anticipación temerosa del proble-
mejorar la inteligibilidad antes de la edad en que casi todos ma. El locutor puede intentar evitar las disfluencias mediante
los niños pueden producirlos correctamente. El ceceo es par- mecanismos lingüísticos (por ejemplo, alterando la velocidad
ticularmente frecuente y puede implicar patrones frontales del habla, evitando ciertas palabras o sonidos) o evitando
o laterales en la dirección de las corrientes de aire. Puede ciertas situaciones, como telefonear o hablar en público.
asociarse a un patrón anormal de la deglución con un em- Además de ser característicos de la afección, se ha mostrado
puje lingual. que el estrés y la ansiedad exacerban la disfluencia.
La mayoría de los niños con trastorno fonológico responde El trastorno de la fluidez de inicio en la infancia también
bien al tratamiento y las dificultades del habla mejoran con puede acompañarse de movimientos motores (por ejemplo,
el tiempo, con lo que el trastorno podría no ser para toda la parpadeos de ojos, tics, temblores en los labios o la cara,
vida. Sin embargo, cuando también está presente el trastor- sacudidas de cabeza, movimientos de la respiración, apretar
no del lenguaje, el fonológico tiene peor pronóstico y puede los puños). Los niños con trastorno de la fluidez muestran
asociarse a trastornos específicos del aprendizaje. diferentes niveles de habilidades del lenguaje, y la relación
entre el trastorno de la fluidez y las habilidades del lenguaje
FACTORES DE RIESGO Y PRONÓSTICO no está clara.
No figuran en DSM5
DESARROLLO Y CURSO
DIAGNÓSTICO DIFERENCIAL El trastorno de la fluidez se presenta antes de los 6 años en
- Variaciones normales del habla: se deberían tener el 80-90% de los individuos afectados, situándose la edad
en cuenta las variaciones del habla regionales, sociales y cul- de inicio entre los 2 y los 7 años. El inicio puede ser insidio-
turales/étnicas antes de hacer este diagnóstico. so o más repentino. Típicamente, las disfluencias empiezan
- Audición u otra deficiencia sensorial: la deficiencia gradualmente con repetición de las consonantes iniciales, las
auditiva o sordera puede conllevar anormalidades del habla. primeras palabras de una frase o las palabras largas. El niño
Las deficiencias de la producción fonológica pueden asociar- puede no ser consciente de estas alteraciones de la fluidez.
se a una deficiencia auditiva, a otras deficiencias sensoriales Cuando el trastorno progresa, las disfluencias llegan a ser
o a una deficiencia motora del habla. Cuando las deficiencias más frecuentes e interfieren más, ocurriendo en las palabras
del habla superan las que normalmente se asocian a estos o frases más significativas de lo que se expresa. Cuando el
problemas, se puede hacer el diagnóstico de trastorno fono- niño llega a ser consciente de la dificultad para hablar, puede
lógico. desarrollar mecanismos para evitar la disfluidez y las respues-
- Deficiencias estructurales: las alteraciones del habla tas emocionales asociadas, evitando, por ejemplo, hablar en
pueden estar causadas por deficiencias estructurales, como público y utilizando frases cortas y sencillas. Los estudios lon-
un paladar hendido. gitudinales muestran que un 65-85% de los niños se recupe-
- Disartria: las alteraciones del habla pueden atribuir- ra de la alteración de la fluidez, y la gravedad del trastorno a
se a un trastorno motor, como la parálisis cerebral. Los sig- los 8 años predice la recuperación o persistencia del mismo
nos neurológicos, así como las características distintivas de la durante la adolescencia y después.
voz, diferencian la disartria del trastorno fonológico, aunque
en los niños pequeños (menores de 3 años) la diferenciación FACTORES DE RIESGO Y PRONÓSTICO
puede ser difícil, particularmente cuando la afectación mo- Genético y fisiológico. El riesgo de tartamudeo entre los
tora general es inexistente o mínima (como, por ejemplo, en familiares biológicos de primer grado de los individuos con
el síndrome de Worster-Drought). trastorno de la fluidez de inicio en la infancia es más de tres
- Mutismo selectivo: el uso limitado del habla puede veces mayor que el riesgo de la población general.
ser un signo de mutismo selectivo, un trastorno de ansiedad Consecuencias funcionales del trastorno de la fluidez de ini-
que se caracteriza por ausencia del habla en uno o más con- cio en la infancia (tartamudeo).
textos o entornos. El mutismo selectivo se puede manifes-
tar en los niños que padecen un trastorno del habla porque Además de ser características de la afección, el estrés y la an-
sientan vergüenza a causa de sus deficiencias, aunque mu- siedad pueden exacerbar la disfluencia. La alteración del fun-

110
cionamiento social puede ser el resultado de esta ansiedad. ciales empiezan a ser más complejas.
Diagnóstico diferencial El resultado del trastorno de la comunicación social (prag-
mático) es variable, con algunos niños que mejoran sustan-
- Déficit sensitivo. Las disfluencias del habla pueden cialmente con el tiempo y otros cuyas dificultades persisten
asociarse a una deficiencia auditiva u otro déficit sensitivo hasta la edad adulta. Incluso entre los que tienen una mejo-
o motor del habla. Cuando las disfluencias del habla sobre- ría significativa, las deficiencias tempranas en la pragmática
pasan las que normalmente se asocian a estos problemas, pueden causar alteraciones duraderas en las relaciones y los
se puede hacer el diagnóstico del trastorno de la fluidez de comportamientos sociales, y también en la adquisición de
inicio en la infancia. otras capacidades relacionadas, como la expresión escrita.
Factores de riesgo y pronóstico
- Disfluencias normales el habla. El trastorno debe Genético y fisiológico: los antecedentes familiares de TEA,
distinguirse de las disfluencias normales que ocurren fre- los trastornos de la comunicación y el trastorno específico
cuentemente en los niños pequeños, como repeticiones de del aprendizaje parecen aumentar el riesgo de trastorno de
palabras enteras o frases (por ejemplo “yo quiero, yo quiero la comunicación social (pragmático).
helado”), frases incompletas, interjecciones, pausas vacías y
comentarios aparte. Si estas dificultades aumentan en fre- DIAGNÓSTICO DIFERENCIAL
cuencia o complejidad mientras el niño crece, el diagnóstico - Trastorno del espectro autista: El TEA es la principal
de trastorno de la fluidez de inicio en la infancia sería correc- consideración diagnóstica para los individuos que presentan
to. deficiencias de la comunicación social. Se pueden diferenciar
- Efectos secundarios de la medicación. El tartamu- los dos trastornos por la presencia en el TEA de patrones
deo puede ocurrir como efecto secundario de una medica- restringidos/repetitivos de comportamiento, intereses o ac-
ción, lo que puede detectarse por su relación temporal con tividades, y su ausencia en el trastorno de la comunicación
la exposición a dicha medicación. social (pragmático). Los individuos con TEA pueden mostrar
- Disfluencias de inicio en el adulto. Si el inicio de las solo patrones restringidos/repetitivos del comportamiento,
disfluencias es durante la adolescencia o después de ésta, intereses o actividades durante las primeras fases del pe-
se trata de una “disfluencia de inicio en el adulto” y no de riodo de desarrollo y, por lo tanto, se debería obtener una
un trastorno del neurodesarrollo. Las disfluencias de inicio historia completa. La ausencia actual de síntomas no exclu-
en el adulto se asocian a daños neurológicos específicos y a ye el diagnóstico de TEA si anteriormente existían intereses
diversas afecciones médicas y trastornos mentales, y pueden restringidos y comportamientos repetitivos. Se debería con-
ser especificadas con ellos, aunque no constituyen un diag- siderar el diagnóstico de trastorno de la comunicación so-
nóstico del DSM 5. cial (pragmático) solamente si la historia del desarrollo no
- Trastorno de la Tourette. Los tics vocales y las voca- muestra ningún tipo de patrones restringidos/repetitivos de
lizaciones repetitivas del trastorno de la Tourette deberían comportamiento, intereses o actividades.
ser distinguibles de los sonidos repetivivos del trastorno de la - TDAH: los déficits primarios del TDAH pueden cau-
fluidez de inicio en la infancia por su naturaleza y ritmo. sar un deterioro de la comunicación social y limitaciones fun-
cionales de la comunicación eficaz, la participación social o
COMORBILIDAD los logros académicos.
No aparece en DSM 5. - Trastorno de ansiedad social (fobia social): los sín-
tomas del trastorno de la comunicación social se solapan
TRASTORNO DE LA COMUNICACIÓN SOCIAL (PRAG- con los del trastorno de ansiedad social. La característica
MÁTICO) que los diferencia es el momento del inicio de los síntomas.
En el trastorno de la comunicación social (pragmático), el
CARACTERÍSTICAS DIAGNÓSTICAS. individuo nunca ha tenido una comunicación social eficaz;
Se recogen los criterios diagnósticos del cuadro. mientras que, en el trastorno de ansiedad social, dicha ca-
Características asociadas que apoyan el diagnóstico pacidad se desarrolló apropiadamente, pero no se utiliza por
La más frecuente es el deterioro del lenguaje, que se carac- ansiedad, miedo o angustia en relación con las interacciones
teriza por antecedentes de retraso para alcanzar los hitos del sociales.
lenguaje y problemas con el lenguaje estructural desde hace - Discapacidad intelectual (trastorno del desarrollo in-
tiempo o en el momento actual. Los individuos con deficien- telectual) y retraso general del desarrollo: las habilidades de
cias de la comunicación social pueden evitar las interacciones comunicación social pueden ser deficientes entre los indivi-
sociales. El TDAH, los problemas conductuales y los trastor- duos con retraso general del desarrollo o con discapacidad
nos específicos del aprendizaje también son más frecuentes intelectual, pero no se da un diagnóstico por separado a no
entre los individuos afectados. ser que las deficiencias de la comunicación social superen
claramente las limitaciones intelectuales.
DESARROLLO Y CURSO
Como la comunicación social (pragmático) depende del pro- COMORBILIDAD
greso adecuado para el desarrollo del habla y el lenguaje, No aparece en el DSM5
el diagnóstico de este trastorno es raro entre los niños de  
menos de 4 años. Al llegar a los 4 o 5 años, la mayoría de los TEMA 5: TRASTORNOS DEL APRENDI-
niños debería tener capacidades del habla y el lenguaje su-
ficientes como para permitir la identificación de deficiencias ZAJE
específicas en la comunicación social. Las formas más leves
del trastorno pueden no resultar evidentes hasta la adoles- TRASTORNO ESPECÍFICO DEL APRENDIZAJE
cencia temprana, cuando el lenguaje y las interacciones so-

111
CARACTERÍSTICAS DIAGNÓSTICAS. trastornos de ansiedad, con quejas somáticas o ataques de
Se recogen los criterios diagnósticos del cuadro. pánico, son frecuentes durante toda la vida y acompañan
tanto a la expresión circunscrita como a la expresión más
CARACTERÍSTICAS ASOCIADAS QUE APOYAN EL DIAG- amplia de las dificultades del aprendizaje.
NÓSTICO
El trastorno específico del aprendizaje frecuentemente, pero FACTORES DE RIESGO Y PRONÓSTICO
no siempre, va precedido en los años preescolares de retra- Ambiental. Nacimiento prematuro o con un peso muy bajo y
sos de la atención, el lenguaje o las habilidades motoras que también la exposición prenatal a la nicotina.
pueden persistir y concurrir con el trastorno específico del Genético y fisiológico. El riesgo relativo del trastorno espe-
aprendizaje. Es frecuente un perfil de aptitudes desiguales. cífico del aprendizaje en la lectura o las matemáticas es sus-
Los individuos con trastorno específico del aprendizaje nor- tancialmente más alto (p. ej., 4-8 veces y 5-10 veces más
malmente (pero no siempre) muestran resultados bajos en alto, respectivamente) en los familiares de primer grado de
las pruebas psicológicas del procesamiento cognitivo. Sin los individuos con estas dificultades de aprendizaje en com-
embargo, aún no está claro si estas anormalidades cognitivas paración con los individuos que no las tienen. Los antece-
son la causa, el correlato o la consecuencia de las dificultades dentes familiares de dificultades para la lectura (dislexia) y
de aprendizaje. Además, estas dificultades cognitivas apare- en las habilidades de lectoescritura de los padres predicen
cen en otros trastornos del neurodesarrollo, por lo que no los problemas de lectoescritura o el trastorno específico del
sería necesaria la evaluación de las deficiencias del procesa- aprendizaje en los hijos, lo que indica una intervención com-
miento cognitivo para la evaluación diagnóstica. Para hacer binada de factores genéticos y ambientales. Los genes rela-
el diagnóstico tampoco son útiles las técnicas de neuroima- cionados con una presentación determinada del trastorno
gen, ni las pruebas genéticas. No existen marcadores bio- se correlacionan en gran medida con los genes relacionados
lógicos conocidos del trastorno específico del aprendizaje. con otras manifestaciones.
El trastorno específico del aprendizaje se asocia a un riesgo
elevado de ideas suicidas e intentos de suicidio en los niños, Modificadores del curso. Los problemas de inatención en
los adolescentes y los adultos. los años preescolares predicen dificultades más tardías en la
lectura y las matemáticas (pero no necesariamente el trastor-
PREVALENCIA no específico del aprendizaje). El retraso o los trastornos del
5-15 % en los niños de edad escolar de diferentes lenguas y habla o del lenguaje, o el deterioro del procesamiento cogni-
culturas. 4 % aproximadamente en adultos. tivo en los años preescolares predicen el posterior trastorno
específico del aprendizaje para la lectura y la expresión escri-
DESARROLLO Y CURSO ta. La comorbilidad con el TDAH predice un peor resultado
El inicio, el reconocimiento y el diagnóstico del trastorno es- de salud mental que la asociada al trastorno específico del
pecífico del aprendizaje normalmente se producen durante aprendizaje sin TDAH. Una enseñanza sistemática, intensiva
los años de la educación primaria, cuando los niños tienen e individualizada podría mejorar o disminuir las dificultades
que aprender a leer, deletrear, escribir y calcular. Sin em- del aprendizaje en algunos individuos o promover el uso de
bargo, en la primera infancia se observan frecuentemente estrategias compensatorias en otros.
algunos precursores, como los retrasos o las deficiencias del
lenguaje, las dificultades para rimar o contar, y las dificul- ASPECTOS DIAGNÓSTICOS RELACIONADOS CON LA
tades con las habilidades motoras finas que son necesarias CULTURA
para escribir, antes de comenzar la instrucción formal. Las El trastorno específico del aprendizaje se produce en las di-
manifestaciones pueden ser conductuales (p. ej., resistencia ferentes lenguas, culturas, razas y niveles socioeconómicos,
a participar en el aprendizaje, conducta negativista). El tras- aunque su manifestación puede variar según la naturaleza
torno específico del aprendizaje dura toda la vida, pero el de los sistemas simbólicos hablados y escritos, y según las
curso y la expresión clínica son variables, dependiendo en prácticas culturales y educativas. Por ejemplo, en la lengua
parte de la interacción entre las exigencias que plantean las inglesa, el clásico síntoma clínico observable de la dificultad
tareas del entorno, la variedad y gravedad de las dificultades para aprender a leer es la lectura imprecisa y lenta de pala-
de aprendizaje del individuo, las aptitudes de aprendizaje del bras sueltas; en otras lenguas alfabéticas (p. ej., el castellano,
individuo, la comorbilidad y los sistemas de apoyo e inter- el alemán) y en las lenguas no alfabéticas (p. ej., el chino, el
vención disponibles. No obstante, los problemas de fluidez japonés), el síntoma clásico es la lectura lenta pero precisa.
y comprensión de la lectura, con la ortografía, de expresión
escrita y con las aptitudes numéricas cotidianas persisten ASPECTOS DIAGNÓSTICOS RELACIONADOS CON EL GÉNE-
normalmente en la edad adulta. La manifestación de los sín- RO
tomas puede cambiar con la edad y, por tanto, los proble- Más frecuente en el sexo masculino que en el femenino (pro-
mas de aprendizaje del individuo pueden persistir o cambiar porciones varían entre aproximadamente 2:1 a 3:1).
a lo largo de la vida. Los adolescentes y los adultos pueden Consecuencias funcionales del trastorno específico del
evitar las actividades que requieran la lectura o la aritmética aprendizaje
(leer por placer, leer instrucciones). Los adultos con trastorno
específico del aprendizaje tienen problemas duraderos con El trastorno específico del aprendizaje puede tener conse-
la ortografía, una lectura lenta y con esfuerzo, y problemas cuencias funcionales negativas durante toda la vida, como
para hacer inferencias a partir de los datos numéricos de los logros académicos bajos, tasas más altas de abandono de la
documentos escritos relacionados con el trabajo. La evita- escuela secundaria, bajos porcentajes de educación secun-
ción o la resistencia a participar en actividades que requieren daria, altos niveles de malestar psicológico y problemas de
aptitudes académicas es frecuente en los niños, los adoles- salud mental general, tasas más altas de desempleo o in-
centes y los adultos. Los episodios de ansiedad grave y de fraempleo e ingresos más bajos. El abandono escolar y los

112
síntomas depresivos concurrentes aumentan el riesgo de PREVALENCIA
padecer problemas de salud mental, incluido el riesgo de En los niños de 5 a 11 años de edad es del 5-6%. El sexo
suicidio, mientras que unos niveles altos de apoyo social o masculino está más afectado que el femenino, con una pro-
emocional predicen mejores resultados de salud mental. porción entre 2:1 y 7:1.

DIAGNÓSTICO DIFERENCIAL DESARROLLO Y CURSO


Variaciones normales en los logros académicos. En el tras- El curso del trastorno es variable pero estable hasta el primer
torno específico del aprendizaje las dificultades de aprendi- año de seguimiento. Aunque puede haber mejorías a largo
zaje persisten en presencia de las oportunidades educativas plazo, se calcula que los problemas de los movimientos coor-
adecuadas, de una exposición a la misma formación que el dinados continúan durante la adolescencia en el 50-70% de
grupo de compañeros, y de la competencia en la lengua aca- los niños.
démica, incluso si difiere de la lengua principal de uno.
Discapacidad intelectual (trastorno del desarrollo intelec- FACTORES DE RIESGO Y PRONÓSTICO
tual). En el trastorno específico del aprendizaje las dificulta- - Ambiental: el trastorno es más frecuente después
des se producen en presencia de niveles normales de funcio- de la exposición prenatal a alcohol, en los niños prematuros
namiento intelectual. Si existe una discapacidad intelectual, y en los que tienen bajo peso al nacer.
el trastorno específico del aprendizaje solamente se puede
diagnosticar cuando las dificultades de aprendizaje superan - Genético y fisiológico: Se han hallado deterioros
las que normalmente se asocian a la discapacidad intelectual. en los procesos subyacentes del neurodesarrollo, particu-
Dificultades de aprendizaje debidas a trastornos neurológi- larmente en las habilidades visomotoras. Se ha propuesto
cos o sensoriales. En caso de déficits neurológicos o senso- la existencia de una disfunción cerebelosa, pero todavía no
riales hay hallazgos anormales en la exploración neurológica. está clara la base neural del trastorno del trastorno. Debido a
Trastornos neurocognitivos. En el trastorno específico del la concurrencia entre el trastorno del desarrollo de la coordi-
aprendizaje la expresión clínica de las dificultades específi- nación y TDAH, las discapacidades del aprendizaje específico
cas se produce durante el período de desarrollo y no se ma- y el trastorno del espectro autista, se ha propuesto la existen-
nifiestan como un empeoramiento marcado de un estado cia de un efecto genético compartido.
anterior.
- Modificadores del curso: Los individuos con TDAH
TRASTORNO POR DÉFICIT DE ATENCIÓN/HIPERACTIVIDAD. y trastorno de desarrollo de la coordinación presentan un
En el TDAH las dificultades no son necesariamente para mayor deterioro que los individuos sin TDAH.
aprender aptitudes académicas específicas sino que puede
haber dificultades para utilizarlas. Es frecuente la concurren- ASPECTOS DIAGNÓSTICOS RELACIONADOS CON LA CULTURA
cia entre el TDAH y el trastorno específico de aprendizaje, en
tal caso, se pueden hacer ambos diagnósticos. El trastorno ocurre en todas las culturas, razas y niveles so-
Trastornos psicóticos. En estos trastornos hay una pérdida (a cioeconómicos.
veces rápida) de aptitudes académicas y de procesamiento
cognitivo. Consecuencias funcionales del trastorno del desarrollo de la
coordinación
COMORBILIDAD
El trastorno específico del aprendizaje concurre frecuente- Conlleva un deterioro funcional en las actividades de la vida
mente con otros trastornos del neurodesarrollo (p. ej., TDAH, cotidiana; menor participación en los juegos y deportes de
trastornos de la comunicación, trastorno del desarrollo de la equipo, niveles bajos de autoestima y autovaloración, pro-
coordinación, trastorno del espectro autista) u otros trastor- blemas emocionales o conductuales, deterioro de los logros
nos mentales (p. ej., trastornos de ansiedad, trastornos de- académicos, mala salud física, poca actividad física y obesi-
presivos y bipolar). dad.

TEMA 6: TRASTORNOS MOTORES DIAGNÓSTICO DIFERENCIAL

TRASTORNO DEL DESARROLLO DE LA COORDINACIÓN - Deterioros motores debidos a otra afección médica

CARACTERÍSTICAS DIAGNÓSTICAS - Discapacidad intelectual

El comienzo de los síntomas debe producirse en las primeras - Trastorno por déficit de atención/hiperactividad
fases del desarrollo, sin embargo, no se diagnostica normal-
mente antes de los 5 años de edad porque existe una varia- - Trastorno del espectro autista
ción considerable en la edad en la que se adquieren muchas
habilidades motoras. - Síndrome de hiperlaxitud articular

CARACTERÍSTICAS ASOCIADAS QUE APOYAN EL DIAGNÓSTICO


COMORBILIDAD
Los trastornos que frecuentemente concurren con el trastor-
Algunos niños muestran actividades motoras añadidas como no del desarrollo de la coordinación son el trastorno fono-
movimientos coreiformes o movimientos en espejo. Estos lógico y del lenguaje, el trastorno específico del aprendizaje
movimientos de “desbordamiento” se consideran inmadu- (especialmente, lectura y expresión escrita), los problemas de
reces del neurodesarrollo o signos neurológicos menores. inatenciónm incluido el TDAH (la afección concurrente más

113
frecuente, 50%), el trastorno del espectro autista, los pro- - Trastornos de tics
blemas conductuales disruptivos y emocionales y el síndrome
de hiperlaxitud articular. - Trastornos obsesivo-compulsivo y trastornos relacio
nados
TRASTORNO DE MOVIMIENTOS ESTEREOTIPADOS
- Otras afecciones neurológicas y médicas
CARACTERÍSTICAS DIAGNÓSTICAS
El repertorio de comportamientos es variable y cada indivi- COMORBILIDAD
duo presenta su comportamiento particular. Los movimien- Las estereotipias son una manifestación común de varios tras-
tos estereotipados se pueden producir muchas veces duran- tornos neurogenéticos, como el síndrome de Lesch-Nyhan,
te el día, durando de pocos segundos a varios minutos o el síndrome de Rett, el síndrome del X frágil, el síndrome de
más tiempo. Los comportamientos varían según el contexto, Cornelia de Lange y el síndrome de Smith-Magenis. Cuando
produciéndose cuando el individuo está absorto en otras ac- los movimientos estereotipados coexisten con otras afección
tividades o cuando está emocionado, estresado, fatigado o médica, deberían codificarse los dos trastornos.
aburrido. La presencia de movimientos estereotipados puede
indicar un problema del neurodesarrollo no detectado, espe- TRASTORNOS DE TICS
cialmente en los niños de 1 a 3 años de edad.
CARACTERÍSTICAS DIAGNÓSTICAS
PREVALENCIA Los tics son movimientos o vocalizaciones súbitos, rápidos,
Los movimientos estereotipados simples (mecerse) son fre- recurrentes y no rítmicos. Ciertos tics, como parpadear los
cuentes en los niños pequeños con desarrollo normal. Los ojos o carraspear, son frecuentes en todas las poblaciones de
movimientos estereotipados complejos son mucho menos pacientes. Pueden ser simples o complejos. Los simples son
comunes (3-4%). Entre el 4% y el 16% de los individuos de corta duración (milisegundos) y los complejos tienen una
con discapacidad intelectual presenta estereotipias y auto- duración más larga (segundos).
lesiones.
La gravedad de los tics fluctúa y algunos pueden pasar de
DESARROLLO Y CURSO semanas a meses sin tics. El comienzo debe producirse antes
Comienzan normalmente en los primeros 3 años de vida. Los de los 18 años de edad. Es muy raro que aparezcan los sín-
movimientos estereotipados simples son frecuentes en la in- tomas de tics en la edad adulta y frecuentemente se asocian
fancia y pueden estar implicados en la adquisición del domi- a la exposición de drogas (como uso excesivo de cocaína) o
nio motor. En la mayoría de los niños con desarrollo normal, son el resultado de daños en el SNC.
estos movimientos se resuelven con el tiempo. El comienzo
de las estereotipias motoras complejas puede producirse PREVALENCIA
en la infancia o más tarde durante el periodo de desarrollo. Los tics son frecuentes en la infancia, aunque transitorios
Entre los individuos con discapacidad intelectual, los com- en la mayoría de los casos. La prevalencia del trastorno de
portamientos estereotipados o autolesivos pueden persistir la Tourette varía entre el 3 y el 8 por 1000 en los niños de
durante años, aunque el patrón puede cambiar. edad escolar y el sexo masculino se afecta con más frecuen-
cia (4:1).
FACTORES DE RIESGO Y PRONÓSTICO
Ambiental: el aislamiento social es un factor de riesgo para la DESARROLLO Y CURSO
autoestimulación que puede progresar hasta los movimien- El comienzo de los tics se produce típicamente entre los 4 y
tos estereotipados con autolesiones repetitivas. El estrés 6 años y la mayor gravedad se encuentra entre los 10 y 12
ambiental también puede desencadenar comportamientos años, con una disminución de la gravedad durante la ado-
estereotipados. lescencia. La gravedad fluctúa y los grupos musculares afec-
tados y las vocalizaciones cambian con el tiempo. Cuando
Genético y fisiológico: el funcionamiento cognitivo bajo está los niños se hacen mayores empiezan a explicar que sus tics
vinculado a un mayor riesgo de comportamientos estereoti- van asociados a un impulso premonitorio y a una sensación
pados y a una peor respuesta a las intervenciones. Los mo- de reducción de la tensión después. Los niños prepúberes
vimientos estereotipados son más frecuentes entre los indi- con trastornos de tics tienen más posibilidades de presentar
viduos con discapacidad intelectual de moderada a grave/ TDAH, TOC y TAS.
profunda.
FACTORES DE RIESGO Y PRONÓSTICO
ASPECTOS DIAGNÓSTICOS RELACIONADOS CON LA CULTURA Temperamental: Los tics empeoran con la ansiedad, la emo-
Se produce en todas las razas y culturas. Las actitudes cultu- ción y el cansancio, y mejoran durante las actividades tran-
rales hacia los comportamientos inusuales pueden provocar quilas y con propósito.
que el diagnósticos sea tardío.
Ambiental: Cuando una persona con trastorno de tics perci-
be un gesto o sonido de otra persona, puede hacer un gesto
DIAGNÓSTICO DIFERENCIAL o sonido similar que los demás pueden creer incorrectamen-
te que es a propósito.
- Desarrollo normal
Genética y fisiológico: Se han identificado importantes alelos
- Trastorno del espectro autista de riesgo para la Tourette y las complicaciones obstétricas, la
mayor edad paternal, el bajo peso al nacer se asocian a una

114
mayor gravedad de los tics. mas cognitivos en las pruebas de atención, de función ejecu-
tiva o de memoria. Al llegar a la juventud, el TDAH se asocia
ASPECTOS DIAGNÓSTICOS RELACIONADOS CON LA CULTURA a un mayor riesgo de intentos de suicidio, principalmente
Los trastornos de tics no parecen variar en sus características cuando hay trastornos comórbidos del estado de ánimo o de
clínicas, curso o etiología dependiendo de la raza, la etnia y la conducta o exista consumo de sustancias.
la cultura.
PREVALENCIA
ASPECTOS DIAGNÓSTICOS RELACIONADOS CON EL GÉNERO El TDAH ocurre en la mayoría de las culturas en aproximada-
El sexo masaculino se afecta más frecuentemente que el fe- mente el 5% de los niños y el 2,5% de los adultos.
menino y las mujeres suelen tener más probabilidad de pre-
sentar ansiedad y depresión. DESARROLLO Y CURSO
El TDAH se identifica más frecuentemente durante la escue-
CONSECUENCIAS FUNCIONALES DEL TRASTORNO DE TICS la primaria, cuando la inatención llega a ser más destacada
Muchas personas con tics de gravedad leve a moderada no y se deteriora el rendimiento. El trastorno es relativamente
presentan malestar y los individuos con síntomas más graves estable durante la adolescencia temprana, pero algunas per-
tienen mayor deterioro en la vida cotidiana. La presencia de sonas tienen un curso que empeora con la aparición de com-
afección concurrente, como el TDAH o el TOC, puede afec- portamientos antisociales. En la mayoría de los individuos los
tar más al funcionamiento. Menos frecuentemente, los tics síntomas de hiperactividad motora llegan a ser menos obvios
pueden llevar al aislamiento, a la victimización, a la incapaci- durante la adolescencia y la vida adulta, aunque pueden per-
dad para trabajar o llegar a provocar lesiones físicas. sistir las dificultades debidas a la inquietud, la inatención, la
poca planificación y la impulsividad. Una proporción sustan-
DIAGNÓSTICO DIFERENCIAL cial de niños con TDAH tiene deterioros que persisten duran-
-Movimientos anormales que pueden acompañar otras afec- te la edad adulta.
ciones médicas y el trastorno de movimientos estereotipados
-Discinesias paroxísticas inducidas por sustancias FACTORES DE RIESGO Y PRONÓSTICO
-Mioclonias Temperamental: El TDAH se acompaña de una menor inhi-
-Trastorno obsesivo compulsivo y trastornos relacionados bición conductual, mayores esfuerzos para ponerse freno,
emotividad negativa y una elevada búsqueda de novedades.
COMORBILIDAD
El TDAH y el TOC son particularmente frecuentes. Las perso- Ambiental: El peso al nacer muy bajo (menos de 1500g) mul-
nas con tics pueden tener otros trastornos del movimiento tiplica el riesgo de TDAH de dos a tres veces, pero la mayoría
y otros trastornos mentales, como los trastornos depresivo, de niños al nacer no desarrollan ningún TDAH. Puede haber
bipolar y por consumo de sustancias. antecedentes de maltrato infantil, negligencia, múltiples ca-
sas de acogida temporal, exposición a neurotóxicos, infec-
ciones o exposición al alcohol durante la vida intrauterina.

TEMA 7: TRASTORNOS POR DÉFICIT DE Genético y fisiológico: El TDAH es más frecuente entre los
familiares biológicos de primer grado de los individuos con
ATENCIÓN TDAH, siendo la heredabilidad considerable. La frecuencia
de anomalías físicas menores puede ser relativamente ele-
TDAH vada, y pueden producirse retrasos motores sutiles y otros
signos neurológicos menores.
CARACTERÍSTICAS DIAGNÓSTICAS
El TDAH empieza en la infancia y es requisito que varios sín- ASPECTOS DIAGNÓSTICOS RELACIONADOS CON LA CULTURA
tomas estén presentes antes de los 12 años de edad. La me- Las diferencias entre las prevalencias del TDAH de diferentes
moria de los síntomas infantiles en los adultos tiende a ser regiones parecen ser atribuibles principalmente a las diferen-
poco fiable y convendría obtener información adicional. Las tes prácticas diagnósticas y metodológicas.
manifestaciones del trastorno deben estar presentes en más
de un entorno y los síntomas varían dependiendo del con- ASPECTOS DIAGNÓSTICOS RELACIONADOS CON EL GÉNERO
texto dentro de cada entorno. Los signos del trastorno pue- El TDAH es más frecuente en el sexo masculino que en el
den ser mínimos o estar ausentes cuando el individuo recibe femenino entre la población general, con una proporción de
recompensas frecuentes por comportamientos apropiados, aproximadamente 2:1 en los niños y 1,6:1 en los adultos.
está bajo estrecha supervisión, está en una situación nueva, Las mujeres tiendena presentar principalmente rasgos de
está participando en actividades especialmente interesantes, inatención.
tiene una estimulación externa constante (p. ej., por panta-
llas electrónicas), o está en situaciones donde interactúa cara CONSECUENCIAS FUNCIONALES DEL TRASTORNO POR DÉ-
con cara con otra persona (p. ej., la consulta del clínico). FICIT DE ATENCIÓN/ HIPERACTIVIDAD
El TDAH se asocia a bajo rendimiento escolar, escasos logros
CARACTERÍSTICAS ASOCIADAS QUE APOYAN EL DIAGNÓSTICO académicos y rechazo social, en los adultos, se asocia a me-
Los retrasos leves del desarrollo lingüístico motor o social no nores logros laborales, mayor absentismo y a más probabili-
son específicos del TDAH, pero concurren de manera fre- dades de desempleo, además de un mayor número de con-
cuente. También son características asociadas la baja tole- flictos interpersonales. Los accidentes y las infracciones de
rancia a la frustración, la irritabilidad y la labilidad del estado tráfico son más frecuentes entre los conductores con TDAH.
de ánimo. Las personas con TDAH pueden mostrar proble- Puede haber un elevado riesgo de obesidad entre los indivi-

115
duos con TDAH. La dedicación insuficiente o variable a las ta- TEMA 10: TRASTORNOS DE LA EXCRE-
reas que requieren esfuerzo sostenido se interpreta muchas
veces por los otros como pereza, irresponsabilidad o falta de CIÓN
cooperación. Las relaciones familiares pueden caracterizarse
por la discordia y las interacciones negativas y las relaciones ENURESIS
con los compañeros muchas veces están afectadas por el SUBTIPOS.
rechazo, la negligencia o las burlas. Normalmente, las per- Según el DSM-5, existen los siguientes subtipos, que consti-
sonas con TDAH tienen menor educación escolar, menores tuyen los especificadores del diagnóstico:
logros vocacionales y puntuaciones intelectuales más bajas • Enuresis nocturna (monosintomática) – Los episo-
que sus compañeros, aunque hay mucha variabilidad. En su dios de incontinencia tienen lugar durante el sueño noctur-
forma grave, el trastorno causa un gran deterioro, afectando no (sobre todo en el primer tercio de la noche). Es el subtipo
a la adaptación social, familiar y académica. más frecuente.
• Enuresis diurna – Los episodios de incontinencia tie-
DIAGNÓSTICO DIFERENCIAL nen lugar durante el día. Hay dos subtipos:
o Incontinencia urgente – Presentan urgencia urinaria
- Trastorno negativista desafiante e inestabilidad del detrusor.
- Trastorno explosivo intermitente o Emisión pospuesta – Se contiene la evacuación de
- Otros trastornos del neurodesarrollo orina hasta que se produce el episodio de incontinencia.
- Trastorno específico del aprendizaje • Enuresis nocturna y diurna (no monosintomática).
- Discapacidad intelectual Características diagnósticas.
- Trastorno del espectro autista Se recogen los criterios diagnósticos del cuadro.
- Trastorno de apego reactivo Características asociadas que apoyan el diagnóstico.
- Trastornos de ansiedad En la enuresis nocturna, en algunas ocasiones, el episodio
- Trastornos depresivos enurético se produce durante el periodo de sueño REM y el
- Trastorno bipolar niño, al despertar, puede recordar lo que estaba soñando.
- Trastorno desregulación disruptiva del estado de En la enuresis diurna, puede ocurrir que el niño contenga
ánimo la micción hasta que se produzca la incontinencia. En es-
- Trastorno por consumo de sustancias tos casos, el episodio enurético se produce principalmente
- Trastornos de la personalidad a primera hora de la tarde en los días de escuela, pudiendo
- Trastornos psicóticos relacionarse con conductas disruptivas.
- Síntomas de TDAH inducidos por medicación Prevalencia.
- Trastornos neurocognitivos En los niños de 5 años, la prevalencia de la enuresis se si-
túa entre el 5-10 %. Esta prevalencia disminuye conforme
COMORBILIDAD aumenta la edad, siendo del 3-5 % en los niños de 10 y de
Entre la población general, el trastorno negativista desafiante aproximadamente el 1 % en las personas de 15 o más años.
concurre con el TDAH en aproximadamente la mitad de los
niños con presentación combinada y en cerca de una cuarta DESARROLLO Y CURSO.
parte de los niños y adolescentes con presentación predomi- Hay dos tipos de curso de la enuresis:
nantemente inatenta. El trastorno de conducta concurre en • Primario: el sujeto nunca ha alcanzado el control del
aproximadamente una cuarta parte de los niños y los adoles- esfínter vesical. La enuresis primaria, por tanto, comienza a
centes con presentación combinada. La mayoría de los niños los 5 años.
y adolescentes con trastorno de desregulación disruptiva del • Secundario: el sujeto ha alcanzado la continencia
estado de ánimo tiene síntomas que también cumple crite- urinaria durante un tiempo. Aunque se puede iniciar a cual-
rios del TDAH y sin embargo un menor porcentaje de ni- quier edad, suele iniciarse entre los 5 y los 8 años.
ños con TDAH tiene síntomas que cumplen los criterios del La enuresis tiene un porcentaje de remisión espontánea del
trastorno de disregulación disruptiva del estado de ánimo. El 5-10% transcurridos 5 años desde su inicio. Un 1% de los
trastorno específico del aprendizaje frecuentemente concu- casos continúa en la edad adulta.
rre con el TDAH. Los trastornos de ansiedad y el trastorno de Antes de los 9 años, la enuresis diurna es infrecuente.
depresión mayor ocurren en una minoría de individuos con
el TDAH, aunque más frecuentemente que en la población FACTORES DE RIESGO Y PRONÓSTICO.
general. El trastorno explosivo intermitente ocurre en una El DSM-5 establece dos tipos de factores de riesgo y pronós-
minoría de los adultos con TDAH, pero a niveles por encima tico:
de los que presenta la población general. Aunque los trastor- - Ambientales – Son factores que predisponen a la enuresis,
nos por consumo de sustancias son relativamente más fre- el retraso y el aprendizaje inconsistente o inadecuado para ir
cuentes entre los adultos con TDAH que entre la población al baño, así como estar sometido a estrés psicosocial.
general, los trastornos están presentes solamente en una - Genéticos y fisiológicos – La enuresis se relaciona con re-
minoría de adultos con TDAH. En los adultos, el trastorno de trasos del desarrollo de los ritmos circadianos normales de
personalidad antisocial puede concurrir con el TDAH. Otros producción de orina y con factores genéticos hereditarios (la
trastornos que pueden concurrir con el TDAH son el TOC, el enuresis nocturna es 3,6 veces más prevalente en hijos de
trastorno de tics y el trastorno del espectro autista. madres enuréticas y 10,6 veces más prevalente en hijos de
padres enuréticos). Existe similar probabilidad de presentar
enuresis nocturna o diurna.
Aspectos diagnósticos relacionados con la cultura.
Las tasas de prevalencia de enuresis son similares en dife-

116
rentes países de Europa, África, Asia o EEUU. Sin embargo, evitar lugares en los que pudieran tener el episodio encopré-
sí que se encuentran prevalencias superiores de enuresis en tico (ej. excursiones o escuela).
niños institucionalizados en internados u orfanatos, proba- Cuando el episodio de incontinencia se produce de manera
blemente debido al aprendizaje inadecuado del hábito de ir deliberada, puede estar asociado al trastorno negativista o
al baño. desafiante o al trastorno de conducta.
Niños con encopresis retentiva pueden también presentar
ASPECTOS DIAGNÓSTICOS RELACIONADOS CON EL GÉNERO. enuresis y diversas infecciones urinarias crónicas, que remiti-
La enuresis nocturna es más frecuente en los niños y la diur- rían con el tratamiento del estreñimiento.
na es más común en las niñas.
Consecuencias funcionales de la enuresis. PREVALENCIA.
La enuresis puede causar limitaciones en las actividades so- Se estima una prevalencia de aproximadamente un 1% en
ciales del niño o tener consecuencias sobre su autoestima niños de 5 años y es un trastorno más frecuente en niños.
debido al castigo, enfado o rechazo que pueden sufrir por
parte de los cuidadores o por el rechazo social de sus com- DESARROLLO Y CURSO.
pañeros y amigos. Hay dos tipos de curso de la encopresis:
• Primario – El sujeto nunca ha tenido continencia fe-
DIAGNÓSTICO DIFERENCIAL. cal. La encopresis primaria, por tanto, comienza a los 4 años.
- Vejiga neurogénica o afección médica – No diag- • Secundario – El sujeto ha tenido un periodo de con-
nosticaremos enuresis si existe alguna afección médica tinencia fecal previo al diagnóstico de encopresis.
(como una vejiga neurogena) que cause poliuria o urgencia Este trastorno puede cursar con exacerbaciones intermiten-
en la micción, sea crónica o aguda (ej. infección aguda de tes a lo largo de los años.
la vía urinaria). Si la incontinencia urinaria era anterior a la
aparición de dichas alteraciones médicas o si persiste una FACTORES DE RIESGO Y PRONÓSTICO.
vez solucionado el cuadro médico, sí podremos realizar el El DSM-5 establece el siguiente factor:
diagnóstico de enuresis. - Genéticos/Fisiológicos – Tras episodios de defecaciones
- Efectos secundarios de la medicación - La enuresis dolorosas, el niño puede retener las heces, produciéndose
puede aparecer tras la ingesta de algunos fármacos (antipsi- estreñimiento y aumentando la probabilidad de la aparición
cóticos, diuréticos u otros). En estos casos, únicamente reali- de encopresis. Por otra parte, algunas medicaciones (ej. anti-
zaremos el diagnóstico si la enuresis apareció anteriormente tusígenos y anticonvulsivantes) también pueden incrementar
a la ingesta de dicho tratamiento. el estreñimiento y aumentar la probabilidad de que el niño
desarrolle este cuadro.
COMORBILIDAD. Marcadores diagnósticos.
La mayoría de los niños diagnosticados de enuresis no pre- Algunas pruebas como las radiografías abdominales pueden
senta ningún otro diagnóstico, aunque cabe destacar que es ser de ayuda para comprobar si existe retención de heces
más frecuente la presencia de síntomas conductuales, retra- o de gas en el colon. También se pueden administrar otras
sos en el desarrollo del habla, del lenguaje, del aprendizaje y pruebas complementarias (como el enema de bario o la ma-
de las habilidades motoras entre esta población, en compa- nografía anorrectal) para realizar el diagnóstico diferencial
ración con la población de niños sin enuresis. Puede existir con otras alteraciones médicas.
comorbilidad con la encopresis, el sonambulismo y el tras- Diagnóstico diferencial.
torno de terrores nocturnos. También son más frecuentes Para diagnosticar encopresis es necesario que no esté cau-
infecciones urinarias en los niños con enuresis (sobre todo sada por alteraciones médicas. Sin embargo, sí que podría
diurna) en comparación con los niños que no tienen este coexistir con alteraciones médicas, siempre que no se pueda
diagnóstico. explicar por ellas.

ENCOPRESIS COMORBILIDAD.
SUBTIPOS.
Según el DSM-5, existen los siguientes subtipos, que consti- La encopresis puede ser comórbida con las infecciones uri-
tuyen los especificadores del diagnóstico: narias, que además presentan mayor frecuencia en las niñas.
• Con estreñimiento e incontinencia por desborda-
miento – Las heces, en este subtipo, suelen estar poco for-
madas y la incontinencia se produce normalmente durante
el día, de manera frecuente o continua. La incontinencia úni-
camente se soluciona después de tratar el estreñimiento.
• Sin estreñimiento e incontinencia por desborda-
miento – Las heces, en este subtipo, suelen tener forma y
consistencia normales y la deposición se realiza de manera
intermitente. Este subtipo es menos frecuente que el ante-
rior y la evacuación se puede producir en lugares visibles,
asociándose normalmente con el trastorno negativista desa-
fiante, trastorno de conducta o masturbación anal.
Características diagnósticas.
Se recogen los criterios diagnósticos del cuadro.
Características asociadas que apoyan el diagnóstico.
Los niños con encopresis pueden sentir vergüenza y, por ello,

117
TEMA 11: Otros trastornos de inicio en la ASPECTOS DIAGNÓSTICOS RELACIONADOS CON EL GÉNERO
Las niñas manifiestan una mayor reticencia o evitación asistir
infancia y la adolescencia a la escuela que los niños. La expresión indirecta del miedo
a la separación puede ser más común en los varones que en
TRASTORNO DE ANSIEDAD POR SEPARA- las mujeres, por ejemplo, a través de una limitación en el
CIÓN. funcionamiento independiente, rechazo a estar fuera de
casa solo etc.

CARACTERÍSTICAS ASOCIADAS QUE APOYAN EL DIAGNÓSTICO RIESGO DE SUICIDIO


Características asociadas que apoyan el diagnóstico El TAS en los niños puede estar asociado con un riesgo ma-
Al alejarse del hogar o de las personas de gran apego, los yor de suicidio. Consecuencias funcionales del trastorno de
niños con TAS pueden manifestar de modo recurrente retrai- ansiedad por separación Los sujetos con TAS a menudo limi-
miento social, apatía, tristeza o dificultad para concentrarse tan sus actividades independientes lejos del hogar o de las
en el trabajo o en el juego. Algunos individuos se sienten in- personas por las que sienten apego.
cómodos y melancólicos, incluso tristes, cuando están fuera
de casa. El TAS en los niños puede dar lugar a una negativa Diagnóstico diferencial
a ir al colegio, lo que puede originar problemas académicos y -TAG
de evitación social. Cuando están solos, especialmente por -Trastorno de pánico
la noche, los niños de menor edad pueden experimentar -Agorafobia
percepciones inusuales. Los niños con este trastorno suelen -Trastornos de conducta
describirse como exigentes, intrusivos y con necesidad de -Trastorno de ansiedad social
atención constante. las excesivas peticiones del niño suelen -Trastorno de estrés postraumático
producir frecuentes frustraciones en sus padres. -Trastorno de ansiedad por enfermedad
-Duelo
PREVALENCIA -Trastornos depresivos y bipolares
La prevalencia anual del TAS entre los adultos de Estados -Trastorno negativista desafiante
Unidos es del 0,9 al 1,9%. -Trastornos psicóticos
La prevalencia del TAS disminuye desde la infancia hasta la -Trastornos de personalidad
adolescencia y la edad adulta, y es el trastorno de ansiedad
más prevalente en los niños menroes de 12 años. COMORBILIDAD
En los niños el TAS es muy comórbido con el TAG y la fobia
DESARROLLO Y CURSO específica. En los adultos, las dolencias comórbidas comunes
Los periodos de ansiedad elevada por la separación de las son la fobia específica, el TEPT, el trastorno de pánico, el
figuras de apego forman parte normal del desarrollo tem- TAG, el trastorno de ansiedad social, la agorafobia, el TOC
prano y pueden indicar el desarrollo de relaciones de apego y los TP. Los trastornos depresivos y bipolares son también
seguras. El inicio puede producirse precozmente, en la edad comórbidos con el trastorno de ansiedad por separación en
preescolar, pero también es posible en cualquier momento- los adultos.
La mayoría de los niños con TAS no padecerá trastornos de
ansiedad deteriorantes a lo largo de la vida. Las manifesta-
ciones del TAS varían con la edad. Los niños más pequeños
pueden ser mas reacios a ir a la escuela o pueden eludirla por MUTISMO SELECTIVO
completo. Es posible que los niños más pequeños no expre-
sen miedos específicos en relación con amenazas concretas CARACTERÍSTICAS DIAGNÓSTICAS
para sus padres, su hogar o ellos mismos, y que la ansiedad No inician el diálogo cuando se encuentran con otras per-
sólo se manifieste cuando se experimenta la separación. Los sonas o no responden cuando los demás inician la conver-
adultos cone este trastorno están demasiado preocupados sación. Esto les sucede tanto en interacciones con adultos
por sus hijos y cónyugues, al tiempo que experimentan un como con otros niños.
notable malestar al separarse de ellos. Suelen hablar ante familiares de primer grado, pero con
frecuencia presentan dificultad ante la familia extensa. Se
FACTORES DE RIESGO Y PRONÓSTICO acompaña de una elevada ansiedad social. Suelen negarse
-Ambientales: el TAS se desarrolla con frecuencia después de a ir a la escuela, lo que entorpece su desarrollo académico.
un estrés vital, en especial una pérdida. La sobreprotección Suelen usar medios no verbales (gruñidos, señales, escritura)
parental y el intrusismo pueden estar asociados con el TAS. para comunicarse y pueden incluso desear participar en en-
-Genéticos y fisiológicos: El TAS en los niños puede ser here- cuentros sociales en los que no se les sea requerido hablar.
ditario. Los niños con TAS muestran un particular aumento
de la sensibilidad a la estimulación respiratoria mediante aire CARACTERÍSTICAS QUE APOYAN EL DIAGNÓSTICO
enriquecido con CO2. Se asocian a este trastorno: timidez excesiva, temor a la hu-
millación, aislamiento y retraimiento social, rasgos compul-
ASPECTOS DIAGNÓSTICOS RELACIONADOS CON LA CULTURA
sivos, comportamiento controlador, negativismo, presencia
El grado en que sería deseable tolerar la separación varía de de pataletas (especialmente en casa), “pegarse” a otros…
unas culturas a otras, por ejemplo, la edad a la que se espera Suelen poseer habilidades lingüísticas normales, aunque
que la descendencia abandone el hogar familiar varía entre puede haber comorbilidad con trastornos de la comunica-
los distintos países y culturas. ción. Suelen recibir diagnóstico adicional de algún trastorno
de ansiedad (sobre todo de fobia social).

118
PREVALENCIA COMORBILIDAD
Relativamente infrecuente. - Trastornos de ansiedad: sobre todo de ansiedad social, an-
Prevalencia entre el 0.03-1%, dependiendo de la muestra siedad por separación y fobia especifica (por este orden).
(escolar vs clínica) y la edad. - Comportamientos oposicionistas: muchas veces limitados a
Sin diferencias por género, raza o etnia. las situaciones en las que se les exige hablar.
Más frecuente en niños, que en adolescentes y adultos. -Trastornos o retrasos de la comunicación.
DESARROLLO Y CURSO
Inicio habitual antes de los 5 años.
El diagnóstico puede demorarse hasta inicio de escolaridad.
Duración variable. Curso longitudinal desconocido. Muchos
lo “superan”. Si síntomas de ansiedad social comórbidos,
estos tienden a permanecer.

FACTORES DE RIESGO Y PRONÓSTICO


- Temperamentales: no bien identificados. Neuroticismo,
inhibición conductual, antecedentes familiares de timidez,
aislamiento y ansiedad social. Dificultades sutiles en lenguaje
receptivo (aún así en rango normal).
- Ambientales: inhibición social de los padres que sirva como
modelo, padres más controladores o protectores que padres
de niños sin mutismo o de niños con otros problemas de
ansiedad.
- Genéticos y fisiológicos: posibles mecanismos compartidos
con ansiedad social (elevada comorbilidad).

ASPECTOS RELACIONADOS CON LA CULTURA


Niños de familias que emigran a un país con distinto idioma
pueden negarse a hablar el idioma por falta de conocimiento
del mismo. Si la comprensión de la nueva lengua es adecua-
da, pero se mantienen la negativa a hablarla, pude justificar-
se el diagnóstico de mutismo selectivo.

ASPECTOS RELACIONADOS CON EL GÉNERO


No se recogen en el DSM 5.

DIAGNÓSTICO DIFERENCIAL
- Trastornos de la comunicación: el mutismo selectivo tiene
que distinguirse de los trastornos del habla que se explican
por trastornos de la comunicación (trastorno del lenguaje,
trastornos fonológicos, trastorno de la fluidez de inicio en
la infancia o el trastorno de la comunicación social pragmá-
tica). En estos cuadros, la alteración del habla no se limita a
una situación social concreta (no es selectiva).
- Trastornos del neurodesarrollo tipo TEA, esquizofrenia
y otros trastornos psicóticos: el mutismo selectivo permite
hablar con normalidad al niño en muchas circunstancias (es
selectivo), mientras que las dificultades con el habla en estos
cuadros no lo son.
- Trastorno de ansiedad social: puede darse comorbilidad en-
tre ambos cuadros.

REPERCUSIONES FUNCIONALES
Deterioro social y académico.
Burlas por parte de sus compañeros.
Puede convertirse en estrategia compensatoria para dismi-
nuir la activación ansiosa en los contextos sociales.

119
TEMA 3: Trastornos del espectro autista

PÁG COLUMNA PÁRRAFO PÁG COLUMNA PÁRRAFO


43 1 10 (APARTADO 43 2 1 (al final del
EPIDEMIOLOGÍA, mismo)
AL FINAL)

AMPLIACIÓN AMPLIACIÓN

En muestras clínicas, las niñas tienden a tener más proba- Los factores que se asocian al peor pronóstico del trastor-
bilidades de presentar discapacidad intelectual acompañan- no son: la presencia de discapacidad intelectual asociada, el
te, lo que sugiere que, en las niñas sin deterioro intelectual deterioro del lenguaje y otros problemas añadidos de salud
acompañante o retrasos del lenguaje, el trastorno podría no mental.
reconocerse; quizás por ser más sutil la manifestación de las
dificultades sociales y de comunicación.

PÁG COLUMNA PÁRRAFO


43 2 2 (al final del
PÁG COLUMNA PÁRRAFO mismo)
43 1 10 (ENTRE LOS
APARTADOS CUR- AMPLIACIÓN
SO Y EPIMIOLOGÍA)
DIAGNÓSTICO DIFERENCIAL

AMPLIACIÓN DSM-5 destaca una serie de trastornos a diferenciar del Tras-


torno del espectro autista:
- Mutismo selectivo.
ETIOLOGÍA: - Trastornos del lenguaje y de la comunicación social
Existen diversos factores de riesgo a nivel ambiental, como (pragmática): el diagnóstico de TEA sustituye al de trastorno
la edad avanzada de los padres, el bajo peso al nacer o la de la comunicación social (pragmática) siempre que se cum-
exposición fetal al valproato. plan los criterios de TEA.
En cuanto a los factores de riesgo a nivel genético, las es- - Discapacidad intelectual sin TEA: un diagnóstico de
timaciones de la heredabilidad del TEA varían entre el 37 y TEA en un individuo con discapacidad intelectual sería co-
más del 90% basándose en la tasa de concordancia entre rrecto si la comunicación y la interacción social están signi-
gemelos. ficativamente deterioradas en comparación con el nivel de
desarrollo de las capacidades no verbales del individuo (p.ej.,
habilidades motoras finas, resolución de problemas no ver-
PÁG COLUMNA PÁRRAFO bales). En cambio, la discapacidad intelectual sería el diag-
nóstico apropiado si no hay ninguna discrepancia aparente
43 1 11 CURSO entre el nivel de las capacidades sociales/comunicativas y el
de las demás capacidades intelectuales.
- Trastorno de movimientos estereotipados: las este-
reotipias se incluyen entre las características diagnósticas de
AMPLIACIÓN TEA, pero sería apropiado realizar un diagnóstico adicional
de trastorno de movimientos estereotipados si las estereoti-
Los primeros síntomas del TEA implican frecuentemente un pias revierten autolesiones y llegan a ser uno de los objetivos
retraso en el desarrollo del lenguaje, acompañado a menudo del tratamiento.
por falta de interés social o interacciones sociales inusuales. - TDAH: se debería considerar el diagnóstico de TDAH
Los síntomas a menudo son muy marcados durante la infan- si las dificultades atencionales o la hiperactividad superan
cia temprana y en los primeros años escolares, con mejoría las que normalmente se observan en individuos con TEA de
del desarrollo típico en la infancia más tardía, al menos en edad mental comparable.
algunas áreas (p. ej., aumento del interés por la interacción - Esquizofrenia: la esquizofrenia de inicio en la infan-
social). Aunque la mayoría de estos niños mejora, una pe- cia normalmente se desarrolla tras un periodo de desarrollo
queña proporción se deteriora conductualmente durante la normal o casi normal. Las alucinaciones y los delirios, que
adolescencia. son las características definitorias de la esquizofrenia, no son
rasgos del TEA.

120
abandonar la consulta del veterinario si no le pegaban un
PÁG COLUMNA PÁRRAFO
tiro.
44 1 1 (al final del Grandeza: Un niño tenía la firme creencia de que era dife-
mismo) rente y capaz de matar a otras personas. Sintió que se volvió
muy fuerte cuando Dios “zumbó” a través suyo (PIR19, 81).

CAMBIO DE ALGUNA FRASE DEL TEXTO


TEMA 4: Trastornos de la comunicación.
(…) aunque en este caso suelen ser de menor elaboración y
estructuración. Las alucinaciones auditivas son las más habi- PÁG COLUMNA PÁRRAFO
tuales, seguidas por las visuales, que también se presentarían
50 2 3 (añadir al final
con bastante frecuencia. En cuanto a las auditivas, las más
del mismo)
frecuentes eran las órdenes, seguidas de las voces que con-
versan.

AMPLIACIÓN
AMPLIACIÓN
Estos niños pueden ser hábiles para adaptarse a su lengua-
En diversos estudios realizados con niños con esquizofrenia, je limitado, pudiendo parecer tímidos o reticentes a hablar.
citados en el manual de Rita Wicks-Nelson y Allen C. Israel Pueden preferir comunicarse solamente con su familia o per-
(1997), entre la sintomatología positiva también destacaba sonas conocidas.
la presencia de ideas delirantes con relativa frecuencia y de
forma consistente.
Las ideas delirantes más frecuentes eran las persecutorias y
las somáticas (de hipocondría), seguidas por las extravagan- PÁG COLUMNA PÁRRAFO
tes y las de referencia y grandeza. Las menos frecuentes ver-
saban sobre el control del pensamiento y temática religiosa 50 2 4
(PIR19, 81).
Algunos ejemplos de las mismas son:
- Persecutoria: Un niño creía que su padre se había CAMBIO DE ALGUNA FRASE DEL TEXTO
fugado de la cárcel y que iba a matarle.
- Somática (de hipocondría): Un niño creía que el es- Los trastornos a los que suele ir asociado el trastorno del
píritu de un niño y el de una niña vivían dentro de su cabeza. lenguaje son los siguientes: Trastorno fonológico (pueden
- Extravagante: Un niño estaba convencido de que concurrir ambos particularmente cuando hay deficiencias
era un perro y que le estaba creciendo el pelo. Una vez se expresivas), Trastorno del espectro del aprendizaje (lectoes-
negó a abandonar la consulta del veterinario si no le pega- critura-aritmética), TDAH, TEA, Trastorno del desarrollo de
ban un tiro. la coordinación y Trastorno de la comunicación social prag-
- Grandeza: Un niño tenía la firme creencia de que mática.
era diferente y capaz de matar a otras personas. Sintió que
se volvió muy fuerte cuando Dios “zumbó” a través suyo.

PÁG COLUMNA PÁRRAFO


PÁG COLUMNA PÁRRAFO
54 1 1 (después del
44 2 A continuación mismo)
del último párrafo

AMPLIACIÓN AMPLIACIÓN

Según Wicks-Nelson, las ideas delirantes, creencias falsas Es importante realizar el diagnóstico diferencial entre Tras-
que se mantienen incluso ante la evidencia de una contradic- torno del Lenguaje y:
ción real, se encontraron con relativa frecuencia y de forma - Variaciones normales del lenguaje: Esta distinción es
consistente entre las muestras de Kolvin et al., 1971, Green más difícil antes de los 4 años de edad, ya que existen varia-
et al., 1996, Russell et al., 1989 y Volkmar et al., 1988. A ciones normales en el desarrollo. Hay que tener en cuenta
continuación, se presentan algunos ejemplos. factores culturales, sociales y regionales para realizar este
diagnóstico diferencial.
Persecutoria: Un niño creía que su padre se había fugado de - Deficiencias sensoriales: cuando las deficiencias del
la cárcel y que iba a matarle. Somática: Un niño creía que lenguaje exceden las que normalmente se asocian a estos
el espíritu de un niño y el de una niña vivían dentro de su problemas, se puede hacer el diagnóstico de Trastorno del
cabeza. lenguaje.
Extravagante: Un niño estaba convencido de que era un pe- - Discapacidad intelectual: a menudo aparece un re-
rro y que le estaba creciendo el pelo. Una vez se negó a traso en el lenguaje en estos casos, por lo que es posible

121
que no se pueda realizar el diagnóstico definitivo hasta que
PÁG COLUMNA PÁRRAFO
el niño pueda completar evaluaciones estandarizadas. No
se realizará un diagnóstico por separado a no ser que las 56 1 5 (al final del
deficiencias del lenguaje sean claramente mayores que las mismo)
limitaciones intelectuales.
- Trastornos neurológicos: el trastorno del lenguaje se
puede producir asociado a los mismos.
- Regresión de lenguaje: la pérdida del habla y el len- CAMBIO DE ALGUNA FRASE DEL TEXTO
guaje en un niño menor de 3 años puede ser un signo de
trastorno del espectro autista o de una afectación neuroló- En otras circunstancias, incluso en los casos más graves, el
gica específica, como el síndrome de Landau-Kleffner. Entre tartamudeo suele desaparecer.
los niños mayores de 3 años la pérdida del lenguaje puede
ser un síntoma de crisis epiléptica.
AMPLIACIÓN
PÁG COLUMNA PÁRRAFO
Por ejemplo, la disfluencia muchas veces está ausente duran-
54 2 11 (al final del te la lectura oral y al cantar o hablar con objetos inanimados
mismo. Apartado o mascotas.
EPIDEMIOLOGÍA
Y CURSO)
PÁG COLUMNA PÁRRAFO

AMPLIACIÓN 56 2 4 (al comienzo)

El trastorno del lenguaje, en particular las deficiencias expre-


sivas, pueden concurrir con el trastorno fonológico. A menu- CAMBIO DE ALGUNA FRASE DEL TEXTO
do están presentes antecedentes familiares de trastornos del
habla o del lenguaje. Aunque el inicio puede ser insidioso o más repentino, su
aparición es típicamente gradual y suele comenzar con pro-
blemas en las consonantes al inicio de las frases, especial-
mente con sonidos oclusivos.
PÁG COLUMNA PÁRRAFO
55 1 4 (apartado Diag-
nóstico diferen- PÁG COLUMNA PÁRRAFO
cial)
56 2 8: Hay que eliminar
el párrafo final con:
“los estudios lon-
gitudinales…”, ya
CAMBIO DE ALGUNA FRASE DEL TEXTO que está repetido
exactamente en el
Se debe descartar que las dificultades que aparecen en la párrafo 5.
pronunciación de los fonemas se deban a: variaciones nor-
males del habla, deficiencias sensoriales, discapacidad inte-
lectual, déficit motor del habla, deficiencias estructurales (p.
ej., un paladar hendido) o privación ambiental. PÁG COLUMNA PÁRRAFO
56 2 9 (apartado Diagnósti-
co diferencial. Al final
AMPLIACIÓN del mismo)

Es importante diferenciarlo también del Mutismo selectivo. AMPLIACIÓN


Este último se caracteriza por ser un trastorno de ansiedad
en el que se produce ausencia del habla en uno o más con- En el caso en que el trastorno comience durante la adoles-
textos o entornos. El mutismo selectivo se puede manifestar cencia o después de esta, se trata de una “disfluencia de
en niños que padecen un trastorno del habla debido a la inicio en el adulto” y no de un trastorno del neurodesarro-
vergüenza que puedan experimentar por sus deficiencias, llo. Este tipo de disfluencias se asocian a daños neurológi-
aunque muchos niños con mutismo selectivo muestran un cos específicos y a diversas afecciones médicas y trastornos
habla normal en entornos “seguros”, como en casa o con mentales y pueden ser especificadas por ellos, aunque no
los amigos cercanos. constituyen un diagnóstico del DSM-5.
Asimismo es importante distinguir las características propias
de la disfemia de los tics vocales y vocalizaciones repetitivas
que se dan en el Trastorno de la Tourette, por su naturaleza
y ritmo.

122
rrollo: no se da un diagnóstico por separado de trastorno de
PÁG COLUMNA PÁRRAFO
la comunicación social a no ser que las deficiencias en este
57 1 1 (al final del mis- campo superen claramente las limitaciones intelectuales; ya
mo) que las habilidades de comunicación social pueden ser defi-
cientes entre los individuos con discapacidad intelectual de-
bido a las características propias del cuadro.

AMPLIACIÓN
PÁG COLUMNA PÁRRAFO
El riesgo de tartamudeo entre los familiares biológicos de
57 2 6 (al final del mismo)
primer grado de los individuos con trastorno de la fluidez de
inicio en la infancia es más de tres veces mayor que el riesgo
de la población general.
AMPLIACIÓN
PÁG COLUMNA PÁRRAFO Como factores de riesgo cabe destacar los antecedentes fa-
miliares de trastorno del espectro autista, trastornos de la
57 2 5 comunicación y trastorno específico del aprendizaje.

CAMBIO DE ALGUNA FRASE DEL TEXTO


Como vimos cuando abordamos (…) pueden ser diagnosti-
cados de un Trastorno de la Comunicación Social (Pragmáti-
ca). Como la comunicación social depende del progreso ade-
cuado del desarrollo del habla y el lenguaje, este diagnóstico
es raro en niños menores de 4 años. A esta edad (entre los 4
y los 5 años), la mayoría de los niños debería tener las capa-
cidades de habla y lenguaje suficientes como para permitir la
identificación de deficiencias específicas en la comunicación
social. De hecho, las formas más leves del trastorno pueden
no resultar evidentes hasta la adolescencia temprana, cuan-
do las interacciones y el lenguaje empiezan a ser más com-
plejos. [CONTINÚA CON: EL CURSO ES VARIABLE…]

PÁG COLUMNA PÁRRAFO


57 2 5 (al final del mismo)

AMPLIACIÓN
Teniendo en cuenta los trastornos que pueden aparecer aso-
ciados, es importante realizar un diagnóstico diferencial con
los mismos.
- Trastorno del espectro autista: se debe considerar
el diagnóstico de trastorno de la comunicación social (prag-
mático) solamente si la historia del desarrollo no muestra
ningún tipo de patrones restringidos/repetitivos de compor-
tamiento, intereses o actividades.
- TDAH: los déficits primarios del TDAH pueden cau-
sar un deterioro de la comunicación social y limitaciones fun-
cionales de la comunicación eficaz.
- Trastorno de ansiedad social: se diferencian por el
momento de inicio de los síntomas. En el trastorno de la co-
municación social (pragmático) el individuo nunca ha tenido
una comunicación social eficaz mientras que, en el trastorno
de ansiedad social, dicha capacidad no presenta problemas,
pero no se utiliza por miedo o angustia en relación con las
interacciones sociales.
- Discapacidad intelectual y retraso general del desa-

123
TEMA 10 sí que se encuentran prevalencias superiores de enuresis en
niños institucionalizados en internados u orfanatos, proba-
PÁG COLUMNA PÁRRAFO blemente debido al aprendizaje inadecuado de ir al baño.
ASPECTOS DIAGNÓSTICOS RELACIONADOS CON EL GÉNE-
100 2 Antes de apartado RO.
“Clasificaciones” La enuresis nocturna es más frecuente en los niños y la diur-
na es más común en las niñas.

AMPLIACIÓN PÁG COLUMNA PÁRRAFO

(Nuevo apartado) 102 1 Tabla 5


CARACTERÍSTICAS ASOCIADAS
En la enuresis nocturna, en algunas ocasiones el episodio
enurético se produce durante el periodo de sueño REM y el
niño, al despertar, puede recordar lo que estaba soñando.
En la enuresis diurna, puede ocurrir que el niño contenga CAMBIO DE ALGUNA FRASE DEL TEXTO
la micción hasta que se produzca la incontinencia. En es-
tos casos, el episodio enurético se produce principalmente Antes de párrafo: “Puede realizarse el diagnóstico en los ca-
a primera hora de la tarde en los días de escuela, pudiendo sos...”
relacionarse con conductas disruptivas.

AMPLIACIÓN
(Añadir)
PÁG COLUMNA PÁRRAFO La enuresis puede aparecer tras la ingesta de algunos fárma-
cos. En estos casos, únicamente realizaremos el diagnóstico
102 1 Antes de párrafo 4:
“Presenta alta co- si la enuresis apareció anteriormente a la ingesta de dicho
morbilidad con...” tratamiento.

AMPLIACIÓN
PÁG COLUMNA PÁRRAFO
La enuresis diurna es infrecuente antes de los 9 años.
102 2 Antes de apartado
“Etiología”

PÁG COLUMNA PÁRRAFO


AMPLIACIÓN
102 1 Antes de apartado
“Diagnóstico Dife- (Nuevo apartado)
rencial” COMORBILIDAD
La mayoría de los niños diagnosticados de Enuresis no pre-
senta ningún otro diagnóstico, aunque cabe destacar que es
AMPLIACIÓN más frecuente entre esta población la presencia de síntomas
conductuales, retrasos del desarrollo del habla, del lenguaje,
(Nuevo apartado) del aprendizaje y de las habilidades motoras en comparación
FACTORES DE RIESGO Y PRONÓSTICO. con la población de niños sin enuresis. Puede existir comor-
El DSM-5 establece dos tipos de factores de riesgo y pronós- bilidad con la encopresis, el sonambulismo y el trastorno de
tico: terrores nocturnos. También son más frecuentes infecciones
- Ambientales – Son factores que predisponen a la enuresis urinarias en los niños con enuresis (sobre todo en la diurna)
el retraso o el aprendizaje inconsistente o inadecuado para ir en comparación con los niños que no tienen este diagnós-
al baño, así como estar sometido a estrés psicosocial. tico.
- Genéticos y fisiológicos – La enuresis se relaciona con retra-
sos del desarrollo de los ritmos circadianos normales de pro-
ducción de orina, con factores genéticos (la enuresis noctur-
na es 3,6 veces más prevalente en hijos de madres enuréticas
y 10,6 veces más prevalente en hijos de padres enuréticos). PÁG COLUMNA PÁRRAFO
ASPECTOS DIAGNÓSTICOS RELACIONADOS CON LA CUL- 103 2 Antes de apartado
TURA. “Clasificaciones”
Las tasas de prevalencia de enuresis son similares en dife-
rentes países de Europa, África, Asia o EEUU. Sin embargo,

124
AMPLIACIÓN PÁG COLUMNA PÁRRAFO
(Nuevo apartado) 111 2 Diagnóstifo
CARACTERÍSTICAS ASOCIADAS. diferencial
Los niños con encopresis pueden sentir vergüenza y que esto
les lleve a evitar estar en lugares en los que pudieran tener el
episodio encoprético (ej. excursiones o escuela). AMPLIACIÓN
Cuando el episodio de incontinencia se produce de manera
deliberada, puede estar asociado al trastorno negativista o TEXTO EXTRAÍDO DE DSM 5, PAGINA 267. TRASTORNO DE
desafiante o al trastorno de conducta. APEGO REACTIVO – DIAGNÓSTICO DIFERENCIAL
Niños con encopresis retentiva pueden también presentar
enuresis y diversas infecciones urinarias crónicas, que remiti- “Trastorno del espectro autista. Los comportamientos socia-
rían con el tratamiento del estreñimiento. les aberrantes no sólo se manifiestan en los niños pequeños
con trastorno de apego reactivo, sino que también son la
principal característica del trastorno del espectro autista. En
concreto, los niños pequeños de ambas afecciones pueden
PÁG COLUMNA PÁRRAFO manifestar una expresión inhibida de las emociones positi-
vas, retrasos cognitivos y del lenguaje, y alteraciones en la
104 1 Antes de apartado reciprocidad social. Por ello, sería necesario diferenciar el
“Diagnóstico Dife- trastorno de apego reactivo del trastorno del espectro au-
rencial”
tista. Estos dos trastornos se pueden distinguir basándose
en las diferentes historias de abandono y por la presencia
AMPLIACIÓN de intereses restringidos o comportamientos ritualizados, un
déficit específico en la comunicación social y comportamien-
(Nuevo apartado) tos de apego selectivos.
FACTORES DE RIESGO Y PRONÓSTICO. Los niños con trastorno de apego reactivo han experimen-
El DSM-5 establece el siguiente factor: tado una historia de abandono social grave, aunque no
- Genéticos/Fisiológicos – Tras episodios de defecaciones do- siempre es posible obtener historias detalladas sobre la na-
lorosas el niño puede retener las heces, produciéndose es- turaleza precisa de sus experiencias, sobre todo en las eva-
treñimiento y aumentando la probabilidad de la aparición de luaciones iniciales. No es común que los niños con trastorno
la encopresis. Por otra parte, algunas medicaciones (ej. anti- del espectro autista tengan una historia de abandono social.
tusígenos y anticonvulsivantes) también pueden incrementar Los intereses restringidos y los comportamientos repetitivos,
el estreñimiento y aumentar la probabilidad de que el niño característicos del trastorno del espectro autista, no son una
desarrolle encopresis. particularidad del trastorno de apego reactivo. Estas carac-
MARCADORES DIAGNÓSTICOS. terísticas clínicas manifiestan una fijación excesiva a rituales
Algunas pruebas como las radiografías abdominales pueden y rutinas, con intereses restringidos, obsesivos y reacciones
ser de ayuda para comprobar si existe retención de heces sensoriales inusuales. Sin embargo, es importante destacar
o de gas en el colon. También se pueden administrar otras que los niños con cualquiera de las dos afecciones pueden
pruebas complementarias (como el enema de bario o la ma- exhibir comportamientos estereotipados, tales como mecer-
nografía anorrectal) para realizar el diagnóstico diferencial se o moverse. Los niños con cualquiera de ambos trastornos
con otras alteraciones médicas. también pueden mostrar una variedad de funcionamiento
intelectual, pero sólo los niños con trastorno del espectro au-
tista muestran alteraciones selectivas en los comportamien-
tos sociales de comunicación tales como la comunicación
PÁG COLUMNA PÁRRAFO intencional (p. ej., dificultades en la comunicación delibera-
da, dirigida a una meta y con objeto de influir en el com-
104 1 Antes de apartado
portamiento del destinatario). Los niños con trastorno de
“Etiología”
apego reactivo muestran un funcionamiento de comunica-
ción social comparable a su nivel general de funcionamiento
intelectual. Por último, los niños con trastorno del espectro
autista normalmente muestran una conducta de apego que
AMPLIACIÓN es característica de su nivel de desarrollo. Por el contrario,
los niños con trastorno de apego reactivo lo hacen de una
(Nuevo apartado)
manera rara o inconsecuente” (APA, 2014). (PIR19,82)
COMORBILIDAD.
La encopresis puede ser comórbida con las infecciones uri-
narias, que además presentan mayor frecuencia en las niñas.

125
PÁG COLUMNA PÁRRAFO
108 1 Después de párra-
fo: “El Trastorno
de Ansiedad por
Separación tam-
bién cumple con
esta norma afec-
tando más a niñas
que a niños en
una propoción
de 2:1.”

AMPLIACIÓN
Según el DSM-5, las niñas manifiestan una mayor reticencia
o evitación a asistir a la escuela que los niños. La expresión
indirecta del miedo a la separación puede ser más común
en los varones que en las mujeres, por ejemplo, a través de
una limitación en el funcionamiento independiente, rechazo
a estar fuera de casa solo, angustia cuando el cónyuge o los
hijos hacen cosas de forma independiente, o cuando el con-
tacto con el cónyuge o los hijos no es posible” (PIR19, 139)

126
BIBLIOGRAFÍA INFANTIL
Bragado, C. “Terapia de conducta en la infancia: trastornos de ansiedad”. Fundación Universidad-Empresa. Madrid. 1994.

WICKS-NELSON, R. (1997): Psicopatología del niño y del adolescente. Prentice Hall

AMERICAN PSYCHIATRIC ASSOCIATION (APA) (2013): Manual de Diagnóstico y Estadístico de los Trastornos Mentales, DSM
5. Editorial Médica Panamericana. Madrid.

127
TRATAMIENTOS PSICOLÓGICOS
RESEÑAS

RESEÑAS
PÁG COLUMNA PÁRRAFO
36 1 3

TEXTO AMPLIADO: TEMA 3


TEXTO DEL MANUAL
En la terapia cognitiva para la psicosis se emplean dos enfo-
4.2. Trastornos bipolares ques para comprobar las creencias de forma empírica. Por
El litio es el fármaco de primera línea (eficacia tipo I) (PIR una parte, tenemos un conjunto de procedimientos para
09, 139). Posee mejores resultados a mayor dosis adminis- poner a prueba la creencia generalizada de «No puedo con-
trada, pero también tiene importantes efectos secundarios, trolar mis voces». En primer lugar, esto se reformula como
por lo que se hace necesario la regulación de la dosis de «No puedo hacer que las voces aparezcan y desaparezcan».
manera individualizada en cada paciente (a través de analíti- Luego el terapeuta plantea situaciones para aumentar y lue-
cas frecuentes). Recientemente se han empezado a contem- go disminuir la probabilidad de oír voces. Una evaluación
plar otras opciones como los anticonvulsivantes (valproato cognitiva inicial debería identificar las señales que sirven para
o ácido valproico (PIR19, 28), carbamazepina, gabapentina, provocar las voces y una técnica que tiene una elevada pro-
lamotrigina y topiramato), aunque el litio es el fármaco que- babilidad de eliminar las voces de forma duradera es la ver-
mejor protege contra el suicidio. balización concurrente (Birchwood, 1986) (PIR 19, 33). La
mejor protege contra el suicidio.
persona aviva y reprime las voces varias veces con el fin de
proporcionar una prueba completa.
otrigina y topiramato), que se consideran una alternativaai-
TEXTO AMPLIADO: TEMA 4

El mecanismo de acción de la fototerapia consiste, en par-


te, en disminuir la depleción de triptófano presente en los TEXTO AMPLIADO: TEMA 3
pacientes con TAE. La dosis inicial de administración debe
de ser de 15-20 min al día y gradualmente ir aumentando El proyecto HORYZONS (PIR 19, 36) nace con el propósito
hasta 30-45 min al día a 10000 lux (unidad photométrica) de facilitar el acceso a intervenciones basadas en la eviden-
(PIR 19, 29). Los estudios sobre la hora de administración de cia a los jóvenes con psicosis, haciendo uso de una página
la luminoterapia han encontrado resultados contradictorios. web interactiva. Se trata de utilizar la tecnología digital para
Parece que las sesiones por la mañana producen mejores re- extender los beneficios de las tradicionales intervenciones
sultados que las administradas por las tardes, aunque esta sí psicosociales cara a cara. Este sistema comprende una pla-
son superiores al placebo. taforma que facilita el acceso a una amplia variedad de in-
tervenciones psicosociales interactivas, potenciadas por un
entorno de red social online moderado por profesionales y
usuarios expertos que procura la interacción social y el apoyo
mutuo.
PÁG COLUMNA PÁRRAFO
33 2 2
PÁG COLUMNA PÁRRAFO
26 2 3
TEXTO DEL MANUAL

4.1. Trastornos depresivos TEXTO DEL MANUAL

3. TERAPIA INTERPERSONAL DE KLERMAN (TIP) Tratamientos Multimodales: Terapia Psicológica Integrada


Klerman y Weissman diseñaron esta terapia desde el contex- (IPT) de Roder y Brenner (2007)
to psiquiátrico (PIR19, 30) con la idea de convertirla en una
terapia de mantenimiento para la depresión tras la adminis- La IPT es un programa de tratamiento conductual para la
tración del tratamiento farmacológico (PIR 18, 18) esquizofrenia, grupal y jerarquizado, que se compone de 5
módulos (PIR 14, 167), en los que se incluye: la rehabilita-
ción cognitiva (módulos: 1. Diferenciación cognitiva (PIR19,
37)

128
tiva son la práctica de ejercicios cardiovasculares, la hiper-
ventilación y las inhalaciones de dióxido de carbono (PIR 18,
174). En la última versión del programa de Barlow, la rela-
jación se ha sustituido por re-entrenamiento en respiración
(patrón de respiración pausado) ya que era el componente
que se había mostrado menos efectivo del programa (PIR19,
PÁG COLUMNA PÁRRAFO 42).
27 2 14

PÁG COLUMNA PÁRRAFO


TEXTO DEL MANUAL 49 Tabla 8 Fila 4
Columna 4
Terapia Cognitiva o Terapia de Focalización de Bentall,
Haddock y Slade (1994). TEXTO DEL MANUAL
Tiene como objetivo la disminución de la frecuencia de las 5.3. Fobia social
voces así como la reducción del malestar asociado a ellas, a
través de la reatribución de manera gradual de las voces a = que TCCG
uno mismo (PIR 13, 147; PIR 14, 162). Por lo tanto, es un Heimberg pero
tratamiento específico para las alucinaciones auditivas (PIR añadiendo:
11, 150; PIR 12, 152). Bentall afirma que la causa de las • Psicoeducación.
alucinaciones es un fallo de la metacognición (teoría del dé- • Videofeedback (PIR19, 43).
ficit en la habilidad metacognitiva de discriminación de la • Desviación de la atención (PIR 13, 155).
realidad de Slade y Bentall), por lo que se atribuyen sucesos
internos autogenerados a una fuente externa. Consta de tres
fases (PIR 14, 165):
1. Prestar atención a la forma y aspectos físicos de las voces
PÁG COLUMNA PÁRRAFO
(PIR 18, 161; PIR19, 38) 50 Tabla 9 Fila 4
Columna 4

PÁG COLUMNA PÁRRAFO


TEXTO DEL MANUAL
18 2 5
5.4. Trastorno de Ansiedad Generalizada

TEXTO DEL MANUAL • Intervenciones para la preocupación:


- En el problema.
2.1. Alcoholismo - En la emoción (PIR 11, 134).
Prevención de recaídas de Marlatt y Gordon (PIR 14, 178) - Exposición funcional cognitiva* (PIR 08, 145; PIR 14, 204;
En relación a esto, los autores hablan del efecto de violación PIR19, 44).
de la abstinencia o EVA (PIR 15, 100). Este efecto explica
por qué el paciente pasa de la caída a la recaída. La intensi-
dad del mismo dependerá del grado de compromiso que el TEXTO AMPLIADO: TEMA 5
paciente tenga en mantener la abstinencia, la duración de la
misma (cuanta más, mayor será el efecto) y el valor subjetivo Exposición funcional cognitiva* ante preocupaciones que se
que se le dé a la conducta. Por contra, una atribución exter- refieren a acontecimientos altamente improbables. (PIR 08,
na, inestable y controlable disminuye el EVA. Se compone de 145; PIR 14, 204; PIR19, 44).
dos elementos:
• Disonancia cognitiva: entra en contradicción con el auto-
concepto e abstinente. (PIR19, 40).
PÁG COLUMNA PÁRRAFO
48 1 2
PÁG COLUMNA PÁRRAFO
41 2 1

TEXTO DEL MANUAL


TEXTO DEL MANUAL Tratamiento psicológico

5.1. Trastorno de pánico y agorafobia Terapia cognitivo-conductual (TCC) (PIR 19, 45)

Algunas de las técnicas para facilitar la exposición interocep-

129
Las últimas investigaciones muestran que las TCC son supe-
PÁG COLUMNA PÁRRAFO
riores a la farmacoterapia con benzodiacepinas (PIR 16, 137)
mientras que éstas no parecen aportar beneficios considera-
51 1 6
bles cuando se las usa junto a la TCC. Barlow específica que
las más eficaces son aquellas que combinan las TCC con rela-
jación y cuyo objetivo es colocar el proceso de preocupación TEXTO DEL MANUAL
bajo control del cliente.
Tratamientos psicológicos

TEXTO AMPLIADO: TEMA 5 Exposición con prevención de respuesta (EPR)

Barlow específica que las más eficaces son aquellas que com- Tradicionalmente la intervención psicológica en el TOC se
binan las TCC con relajación y cuyo objetivo es colocar el ha dividido en dos grandes grupos: el de los procedimientos
proceso de preocupación bajo control del cliente, enseñan- de exposición (que se enfocan a disminuir la ansiedad y el
do al paciente que la preocupación patológica improductiva malestar que producen las obsesiones) y el de las técnicas de
es una cadena de pensamientos sobre problemas distales o supresión o bloqueo (orientadas a bloquear las compulsio-
abstractos, que se centra en la emoción negativa asociada nes,
con la situación objeto de preocupación (PIR 19, 45). esperando con ello una reducción también de las obsesio-
nes).
Así, la exposición con prevención de respuesta (EPR, Meyer)
supone una combinación de ambos enfoques (PIR 15, 69) y
PÁG COLUMNA PÁRRAFO marcó un punto de inflexión en el tratamiento del TOC.
La EPR se considera eficaz (E1) para el tratamiento del TOC y
58 2 3 constituye el tratamiento de elección, manteniendo resulta-
dos
incluso en seguimientos a largo plazo (hasta 3 años) y espe-
TEXTO DEL MANUAL cialmente
para los síntomas compulsivos del trastorno (PIR 07, 233;
Programas cognitivo-conductuales estructurados PIR 14, 203; PIR 15, 113; PIR 19, 48).
(PIR 07, 244)

• Terapia cognitivo-conductual de Warwick y Salkovskis


(PIR 09, 132, PIR 19, 46). PÁG COLUMNA PÁRRAFO
Basada en las interpretaciones catastróficas y los supuestos
disfuncionales. La terapia consiste en ayudar al paciente a 54 2 4
detectar y cambiar los pensamientos automáticos negativos
sobre sus síntomas físicos, las creencias desapatativas sobre
la salud y enfermedad y las conductas problemáticas. TEXTO DEL MANUAL

6.3. Tricotilomanía
El procedimiento más recomendado es la terapia de conduc-
PÁG COLUMNA PÁRRAFO ta
e inversión del hábito (PIR 15, 116; PIR 19, 49), a lo que se
51 2 5 pueden sumar
medidas farmacológicas (ISRS, antipsicóticos o litio) y medi-
TEXTO DEL MANUAL das
dermatológicas (esteroides, antihistamínicos).
Terapia cognitiva (TC) (PIR 19, 47)
La terapia cognitiva en el contexto del tratamiento del TOC
aparece bien como un suplemento que potencia-facilita las
técnicas conductuales como la EPR, bien como tratamiento
principal que incluye como experimento conductual la EPR TEXTO AMPLIADO: TEMA 15
para modificar cogniciones erróneas (PIR 18, 165).
Los objetivos de la psicoterapia ante un TP deben enmar-
TEXTO AMPLIADO: TEMA 6 carse dentro de un plan de confirmación de la experiencia
disfuncional, respetando el modo de ser del paciente. Des-
La terapia cognitiva en el contexto del tratamiento del TOC de esta perspectiva, el principio de gradualidad (PIR 19, 50)
aparece bien como un suplemento que potencia-facilita las hace referencia a que los cambios deben ser graduales, pro-
técnicas conductuales como la EPR, bien como tratamiento gresando desde las manifestaciones periféricas hacia los pa-
principal mediante el trabajo con las valoraciones y creencias trones básicos de la organización personal.
disfuncionales asociadas a las obsesiones (PIR 19, 47), que
incluye como experimento conductual la EPR para modificar
cogniciones erróneas (PIR 18, 165).

130
PÁG COLUMNA PÁRRAFO PÁG COLUMNA PÁRRAFO

69 2 1 145 2 5

TEXTO DEL MANUAL TEXTO DEL MANUAL

Tratamiento psicológico. Psicoterapia


La psicoterapia individual persigue el objetivo de mejorar el
Es la disfunción sexual masculina que mejor responde al tra- funcionamiento del paciente. Por tanto, puede ir dirigida a la
tamiento psicológico (PIR 10, 199). Desde los años 70 se mejora de la dinámica familiar, a la intervención sobre pro-
han venido utilizando dos técnicas, sobre todo: blemas psicológicos (ansiedad, depresión, etc.), al temor a la
- Técnica de parada y arranque o “Stop-start” (Semans, muerte o a la recaída, a problemas laborales desencadena-
1956) (PIR 04, 123; PIR 06, 244; PIR 09, 162; PIR 12, 228; dos por la enfermedad, etc.
PIR 19, 52). Es importante que las sesiones con los pacientes se adapten
al ritmo y a las necesidades de éstos.
Un ejemplo de psicoterapia que ha sido diseñada específica-
mente para pacientes con cáncer es la terapia psicológica ad-
TEXTO AMPLIADO: TEMA 4 yuvante (TPA) de Greer (PIR 14, 223; PIR 16, 72; PIR 19,58)

En los trastornos depresivos, la aparición de cambios físicos


es habitual y suele ser uno de los motivos principales por PÁG COLUMNA PÁRRAFO
los que se solicita la ayuda de un profesional. Un síntoma
típico, que afecta a un 70-80% de los pacientes depresivos, 17 1 2
es los problemas de sueño. Otros síntomas físicos comunes
son la fatiga, la pérdida de apetito y una disminución de la
actividad y el deseo sexuales. Es decir, como parte del estado TEXTO AMPLIADO: TEMA 2
general de anhedonia, anergia, etc., las conductas y deseos
sexuales se ven casi siempre afectados negativamente (PIR Actualmente Miller y Rollnick (2013) proponen que el cómo
19, 53). y el por qué las personas cambian, sucede en un proceso
recursivo en el que se superponen y confluyen cuatro tipos
de procesos (PIR 19, 62):
1) Comprometiéndose: El primer proceso de la Entre-
PÁG COLUMNA PÁRRAFO vista Motivacional es enganchar al paciente en una relación
de trabajo colaborativo. Es un proceso por el cual ambas par-
17 1 1 tes establecen una conexión útil y una relación de trabajo.
2) Enfocándose: Es el proceso mediante el cual el tera-
peuta desarrolla y mantiene un curso específico en la conver-
TEXTO DEL MANUAL sación acerca del cambio. Se trata de ayudar a los pacientes
a clarificar sus objetivos.
EL MANUAL DE TRATAMIENTOS TE REDIRIGE AL DE CLÍNICA 3) Evocando: Este proceso por el cual el terapeuta in-
PARA ESTA PARTE DEL TEMARIO duce la motivación del cliente hacia el cambio.
4) Planeando: Es un proceso mediante el cual el tera-
peuta ayuda al paciente a formular planes de acción especí-
ficos para alcanzar el cambio deseado.
TEXTO AMPLIADO: TEMA 36

Un programa de automanejo para pacientes con asma sería TEXTO AMPLIADO: TEMA 0
el medidor de Peak-flow (PIR 19, 56), una técnica de manejo
utilizada en atención primaria. El flujo espiratorio máximo La Guía de Práctica Clínica sobre el manejo de la Depresión
(FEM) o peak expiratory flow (PEF) en terminología anglosa- en el adulto del Sistema Nacional de Salud establece la pro-
jona, es el mayor flujo que se alcanza durante una maniobra puesta de SIGN (Scottish Intercollegiate Guidelines Network)
de espiración forzada. Se consigue al haber espirado el 75- como modelo de clasificación de los niveles de evidencia clí-
80% de la capacidad pulmonar total. Entre sus ventajas, ob- nica y de grados de recomendación.
vian la necesidad de registro manual de los valores por parte
del paciente, aumentan la exactitud del registro y graban el El grado D (PIR 19, 64) se corresponde con evidencia cien-
momento del día en que se realiza la maniobra. Su principal tífica de nivel 3 (estudios no analíticos, como informes de
inconveniente es su alto precio. casos y series de casos) o 4 (opinión de expertos); o evidencia
científica extrapolada desde estudios clasificados como 2+
(estudios de cohortes o de casos y controles bien realizados
con bajo riesgo de sesgo y con una moderada probabilidad
de establecer una relación causal).

131
PÁG COLUMNA PÁRRAFO PÁG COLUMNA PÁRRAFO

55 2 3 102 1 1

TEXTO DEL MANUAL TEXTO DEL MANUAL

Desensibilización y reprocesamiento por movimientos ocula- Respecto al tratamiento de los tics, Comeche señala la im-
res de Shapiro, 1955 (EMDR o DRMO en español) (PIR 04, portancia de tener en cuenta que rara vez se erradican por
79; PIR 07, 84) completo a pesar del tratamiento; por tanto, el objetivo de
Esta técnica consta de 8 fases (PIR 19, 70) en las que el intervención será la disminución de los mismos, especial-
sujeto se expone al recuerdo traumático mientras recibe es- mente en el síndrome de Tourette. Las técnicas que se con-
timulación bilateral. sideran de elección en el tratamiento de los tics ya han sido
mencionadas a lo largo de este manual en referencia a otros
trastornos: principalmente, la técnica de la inversión o rever-
TEXTO AMPLIADO: TEMA 7 sión del hábito y la exposición con prevención de respuesta
o EPR. También se han utilizado la práctica masiva o negativa
Esta técnica consta de 8 fases tanto para niños como para (PIR 00, 148), el self-monitoring, el manejo de contingencias,
adultos (PIR 19, 70) en las que el sujeto se expone al recuer- el entrenamiento en relajación e hipnosis, la TCC y el bioneu-
do traumático mientras recibe estimulación bilateral. rofeedback. El único tratamiento empíricamente validado es
la inversión o reversión del hábito, y es el que se ha demos-
trado más eficaz (PIR 15, 93; PIR 05, 155; PIR 19, 73).

PÁG COLUMNA PÁRRAFO TEXTO AMPLIADO: TEMA 20

El único tratamiento empíricamente validado es la inversión


118 1 1
o reversión del hábito, y es el que se ha demostrado más
eficaz tanto a corto como a largo plazo (PIR 15, 93; PIR 05,
155; PIR 19, 73).
TEXTO DEL MANUAL

Entrenamiento en el uso del baño (hábitos defecatorios) (PIR PÁG COLUMNA PÁRRAFO
13, 132; PIR 16, 200; PIR 18, 234).
104 1 3
Procedimiento multicomponente en el que intervienen el
reforzamiento positivo, el control de estímulos, instigado-
res, desvanecimiento y manejo del ambiente. El objetivo es TEXTO DEL MANUAL
enseñar al niño a defecar de forma regular en el inodoro,
haciendo coincidir el momento de sentarse (después de cada Entrenamiento en comunicación y habilidades sociales
comida principal) con la activación de los reflejos gastroileal
y gastrocólico, lo que suele acontecer a los 15-20 minutos La técnica de la tortuga de López Soler y Martínez Núñez
de haber ingerido alimentos, con el propósito final de que la (1999), que consiste en enseñar a los niños a relajarse a
conducta de sentarse quede bajo el control de los estímulos través de la analogía de “meterse en su caparazón” como
fisiológicos (Ed) que preceden a la defecación (movimientos hacen las tortugas cada vez que sientan frustración, ira o
intestinales y sensación de distensión rectal). Así pues, se re- enfado (PIR 19, 74), y una vez aprendan esto se realiza un
comienda que el niño se siente en el inodoro durante 10-20 entrenamiento en resolución de problemas interpersonales.
minutos. En algunos casos se añade el uso de laxantes o Esta técnica se utiliza con niños más pequeños (Educación
enemas (PIR 17, 228). Infantil y primeros cursos de Educación Primaria) (PIR 16,
201).
Bragado recomienda no sólo reforzar al niño cuando se pro-
duzca la defecación, sino también reforzar la conducta de
sentarse en el inodoro (PIR 19, 72)
PÁG COLUMNA PÁRRAFO

TEXTO AMPLIADO: TEMA 24 127 1 2

Bragado, basándose fundamentalmente en los principios del


condicionamiento operante, recomienda no sólo reforzar al TEXTO DEL MANUAL
niño cuando se produzca la defecación, sino también refor-
zar la conducta de sentarse en el inodoro (PIR 19, 72). 27.5. TAG Y TAS

El tratamiento de elección es la terapia cognitivo-conductual

132
o TCC, incluyendo los siguientes componentes: •Brazos y hombros: «imagina que eres un gato que se des-
pereza, estirando sus patas y arqueando el lomo» (PIR 19,
- Entrenamiento en relajación progresiva (técnica de 79).
elección dentro de la TCC), adaptada a los niños u otras va- •Hombros y cuello: «imagina que eres una tortuga, toman-
riantes como masaje relajante, juegos, o en imaginación. do el sol en una roca. De repente ves acercarse una gran ga-
Algunas pautas que podemos seguir para aplicar la relajación viota dispuesta a picarte. Escóndete dentro de tu caparazón,
infantil son: hacer sesiones más cortas y frecuentes, eliminar encogiendo el cuello y los hombros para protegerte».
estímulos distractores, presentar los ejercicios como un jue- •Mandíbula: «imagina que estás masticando un gran chicle,
go, adaptar el lenguaje al del niño, modelar los ejerci- cios o duro y elástico. Tensa tus dientes como si te costara mor-
utilizar muñecos antes de aplicarlos, instigación y guía física, derlo».
trabajo con grandes zonas musculares y en total en menor •Cara y nariz: «imagina que una mosca se ha detenido en
número de zonas a trabajar. tu nariz, te molesta y quieres que se vaya. Intenta arrugar tu
nariz para conseguirlo».
•Estómago: «estás tranquilamente tumbado al sol, y ves un
TEXTO AMPLIADO: TEMA 27 pesado elefante caminar hacia ti. Imagina que va a poner su
pata sobre tu barriga. Aprieta el estómago para aguantar su
Para poder aplicar la técnica de relajación progresiva, Cau- peso».
tela y Groden (1989) señalan unas habilidades mínimas que •Piernas y pies: «imagina que eres un explorador y estás ca-
deberá adquirir el niño antes de comenzar la enseñanza de minando por la selva. De repente pisas en un barrizal, pa-
la relajación (PIR 19, 76): recen arenas movedizas. Intenta mantenerte sin hundirte,
apretando tus piernas y encogiendo los dedos de los pies».
-Permanecer quieto durante cinco segundos.
-Mantener la mirada durante cinco segundos.
-Ser capaz de imitar acciones sencillas como levantar su
mano por encima de la cabeza, tocar la mesa y tocarse el PÁG COLUMNA PÁRRAFO
pecho.
-Obedecer órdenes sencillas como “levántate”, “siéntate” y 111 1 3
“ven aquí”.

TEXTO DEL MANUAL


PÁG COLUMNA PÁRRAFO Programa de intervención de Herbert (PIR 19, 80)

128 2 3 Está diseñado para padres que tienen dificultades para ma-
nejar eficazmente el comportamiento de sus hijos, siendo
el principal objetivo dotarles de información y habilidades
TEXTO DEL MANUAL para conseguir de ellos un estilo educativo democrático. A
partir de ese objetivo principal, surgen varios objetivos espe-
ACTION STARK Y KENDALL cíficos que son: 1. Establecer límites firmes y equitativos; 2.
Técnicas (PIR 19, 77): Comunicar normas razonables y apropiadas; 3. Proporcionar
•Actividades agradables. instrucciones y órdenes claras, correctas y asertivas; 4. Elo-
•Autocontrol. giar y fomentar la cooperación; y 5. Aplicar consecuencias
•Relajación. consistentes y coherentes a los malos comportamientos.
•Reestructuración cognitiva.
•Entrenamiento en asertividad.
•Solución de problemas.
•Solución de problemas interpersonales. PÁG COLUMNA PÁRRAFO
•Educación emocional.
•Autoinstrucciones. 22 Tabla 6

TEXTO DEL MANUAL


TEXTO AMPLIADO: TEMA 27
ECHEBURÚA Y BÁEZ (1994) (PIR 08, 152) (PIR 19, 83)
Una variante de la relajación progresiva, que incluye el em- •Control de estímulos (PIR 09, 152; PIR 12, 184; PIR 16,
pleo de imágenes que ayudan al niño a tensar cada grupo 176).
muscular y a mantenerlo motivado hacia el ejercicio es el en- •Exposición con prevención de respuesta o EPR (PIR 00, 117;
trenamiento en imaginación de Koeppen (1974). Se narran PIR 17, 112; PIR 18, 171) (en relación al mecanismo de eje-
varias historias que el niño va escuchando, y en un momento cución conductual, (PIR 15, 102).
de ellas debe tensar una zona de su cuerpo: •Terapia de grupo cognitivo-conductual (simultánea al
•Manos y brazos: «imagina que estás exprimiendo un limón, tratamiento individual, 5-6 pacientes).
apriétalo fuertemente en tu mano hasta que no le quede
nada de jugo».

133
carles una convulsión. La técnica a día de hoy consiste en la
PÁG COLUMNA PÁRRAFO
administración controlada de una breve descarga eléctrica,
aplicada en la cabeza y que provoca una convulsión eléctrica
75 2 4
(entre 6 y 10 sesiones). La TEC está indicada como trata-
miento para la depresión, especialmente para la endógena.
TEXTO DEL MANUAL En estos casos, los efectos de la TEC pueden ser más rápidos
y en ocasiones más efectivos, sacando a una persona de una
Tratamiento farmacológico depresión profunda en pocos días. El principal efecto secun-
dario de la TEC es la alteración de la memoria tanto retrógra-
Presión nasal positiva (CPAP) da como anterógrada (transitoria y reversible: sólo en el 1%
de los pacientes persiste un déficit de memoria retrógrada
Es un tratamiento ideado por Sullivan, Berthon-Jones, Issa para nombres y fechas).
y Eves. Se trata de una mascarilla unida a un generador de
aire que administra un flujo continuo de aire a más presión
sobre las vías respiratorias superiores, para prevenir la apari- TEXTO AMPLIADO: TEMA 4
ción de apneas. Se considera el tratamiento de elección (PIR
19, 115) ya que produce mejoría tanto en la sintomatología Dada la eficacia de los antidepresivos actuales para el tra-
diurna como nocturna. tamiento de la Depresión, se recomienda la TEC sólo en los
siguientes casos (PIR 19, 182):
-Riesgos serios de suicidio
-Depresiones con importante inhibición o agitación psicomo-
tora, e ideas delirantes.
PÁG COLUMNA PÁRRAFO -Grave compromiso del estado general (desnutrición, deshi-
dratación) que requieren de una intervención rápida y eficaz.
64 2 1 -Fracaso de los antidepresivos, al menos dos, de distinta fa-
milia, por tiempo y dosis suficientes.
TEXTO DEL MANUAL -Contradicciones para el uso de antidepresivos por enferme-
dades (cardiovasculares, hepáticas, etc.), o porque durante
Tratamiento farmacológico el tratamiento aparezcan efectos colaterales muy intensos.
-Historia personal de buena respuesta a TEC y/o escasa res-
Al contrario que en anorexia, los fármacos pueden jugar un puesta a los antidepresivos en episodios previos.
papel importante en el caso de la bulimia, siendo los más
destacados los antidepresivos. Diversos estudios confirman
que se reducen los atracones y mejora el estado de ánimo,
aunque parece que los efectos antibulímicos serían indepen-
dientes de los efectos antidepresivos, ya que para que se den PÁG COLUMNA PÁRRAFO
dichos efectos antibulímicos se requieren dosis más altas que
para el tratamiento de la depresión (60 mg/día frente a 20 55 1 2
mg/día en el caso de la depresión) (PIR 17, 55). Si compara-
mos el tratamiento farmacológico con el resto de tratamien-
tos para la bulimia, la TCC sigue siendo más eficaz aunque TEXTO DEL MANUAL
se puede ver potenciada por el uso de antidepresivos (PIR
19, 181). Tratamientos psicológicos

Exposición

PÁG COLUMNA PÁRRAFO La exposición prolongada (PIR 19, 184) en imaginación de


los recuerdos temidos es el tratamiento de elección y E1 para
31 1 2 el TEPT; para las situaciones que son evitadas, se utiliza la
exposición en vivo. El grupo de Foa y cols., desde su modelo
teórico, habla de que lo que funciona de la exposición en
TEXTO DEL MANUAL imaginación es que posibilita el reprocesamiento emocional
del trauma, más que la habituación que se produce a la an-
Tratamientos farmacológicos siedad.

Cabe hacer mención en este apartado a la Terapia Electro-


convulsiva (TEC), que, si bien no es un tratamiento farmaco-
lógico (se encuadra dentro de los tratamientos bio- lógicos
para la depresión), se utiliza como alternativa a los psico-
fármacos en determinados casos. La historia de la TEC se
remonta a principios del siglo XX con los tratamientos de
choque. Su descubridor es el psiquiatra Meduna (1934)
quien observó la mejoría de enfermos psiquiátricos al provo-

134
135
TRATAMIENTOS PSICOLÓGICOS
AMPLIACIONES
INTRODUCCIÓN

PÁG COLUMNA PÁRRAFO

13 2

AMPLIACIÓN
0.1. Niveles de evidencia y grados de recomendación
de las guías clínicas del Sistema Nacional de Salud

La Guía de Práctica Clínica sobre el manejo de la Depresión


en el adulto del Sistema Nacional de Salud establece la pro-
puesta de SIGN (Scottish Intercollegiate Guidelines Network)
como modelo de clasificación de los niveles de evidencia clí-
nica y de grados de recomendación (PIR 19, 64).
(Ver tablas 3 y 4 en la página siguiente)

136
TEMA 2 : TRATAMIENTO ADICCIONES
ponente con mayor efectividad y se ha comprobado que la
PÁG COLUMNA PÁRRAFO
“prueba de realidad” no resulta eficaz si no va precedida del
“desafío verbal”.
17 1 6
A continuación, veremos de forma más detallada el procedi-
CAMBIO DE ALGUNA FRASE DEL TEXTO miento seguido por esta modalidad de terapia en la interven-
ción sobre las voces e ideas delirantes.
Añadir a la tabla, como un cuarto apartado (debajo de ele-
mentos básicos para producir la motivación para el cambio). Terapia cognitiva para las voces

AMPLIACIÓN El propósito de emplear la terapia cognitiva para las voces es


suavizar el malestar y la conducta problemática reduciendo
PROCESOS QUE COMPONEN LA ENTREVISTA MOTIVA- las creencias sobre la omnipotencia, la malevolencia o be-
CIONAL. nevolencia, y la obediencia. Esta terapia cognitiva se apoya
notablemente en el trabajo de Beck, aunque se ha adaptado
Actualmente Miller y Rollnick (2013) proponen que el y desarrollado modificaciones de la terapia cognitiva tradi-
cómo y el por qué las personas cambian, sucede en un pro- cional.
ceso recursivo en el que se superponen y confluyen cuatro
tipos de procesos: En un principio se definen las creencias centrales junto con
las pruebas empleadas para mantenerlas, y discutimos de
1.Comprometiéndose: El primer proceso de la Entrevista qué manera cualquier malestar y perturbación atribuidos a
Motivacional es enganchar al paciente en una relación de las voces es realmente una consecuencia de las creencias que
trabajo colaborativo. Es un proceso por el cual ambas partes sostiene el individuo. La atmósfera es de «empirismo colabo-
establecen una conexión útil y una relación de trabajo. rador», considerándose a las creencias como posibilidades
2.Enfocándose: Es el proceso mediante el cual el terapeuta que pueden ser o no razonables.
desarrolla y mantiene un curso específico en la conversación
acerca del cambio. Se trata de ayudar a los pacientes a clari- El cuestionamiento de la veracidad de unas creencias impli-
ficar sus objetivos. ca el empleo de técnicas cognitivas estándar. En un primer
3.Evocando: Este proceso por el cual el terapeuta in duce la momento, se cuestionan las pruebas de cada creencia; este
motivación del cliente hacia el cambio. proceso empieza con las evidencias que el individuo consi-
4.Planeando: Es un proceso mediante el cual el terapeuta dera menos importantes y continúa hasta las más importan-
ayuda al paciente a formular planes de acción específicos tes. Luego, el terapeuta cuestiona la creencia directamente.
para alcanzar el cambio deseado. Esto implica señalar, primero, ejemplos de inconsistencia e
irracionalidad y, segundo, ofrecer una explicación alternati-
va de los acontecimientos. La alternativa es siempre que las
creencias constituyen una reacción comprensible ante, y un
intento de entender el sentido de, las voces. Esto hace que la
TEMA 3 : TRATAMIENTO DE LA ESQUIZOFRENIA persona intente comprender el significado de las alucinacio-
nes. Se plantea el concepto de que las voces son generadas
por uno mismo y se intenta explorar la posible conexión, o
PÁG COLUMNA PÁRRAFO
significado personal, entre el contenido de las voces y la his-
toria del individuo.
28 1 4
Se emplean dos enfoques para comprobar las creencias de
AMPLIACIÓN forma empírica. Por una parte, tenemos un conjunto de pro-
cedimientos para poner a prueba la creencia generalizada
• Terapia Cognitiva para Delirios, Voces y Paranoia de de «No puedo controlar mis voces». En primer lugar, esto se
Chadwick, Birchwood y Trower (1996). reformula como «No puedo hacer que las voces aparezcan y
También se denomina “Terapia de modificación de creen- desaparezcan». Luego el terapeuta plantea situaciones para
cias”, ya que se fundamenta en la reestructuración cognitiva aumentar y luego disminuir la probabilidad de oír voces. Una
(PIR 00, 112). El terapeuta ha de mantener una actitud no evaluación cognitiva inicial debería identificar las señales que
confrontadora con el paciente. Consta fundamentalmente sirven para provocar las voces y una técnica que tiene una
de dos intervenciones que van dirigidas al ámbito cognitivo elevada probabilidad de eliminar las voces de forma dura-
(PIR 12, 148; PIR 18, 159): dera es la verbalización concurrente (Birchwood, 1986) (PIR
- Desafío verbal (se ayuda al paciente a modificar el con- 19, 33). La persona aviva y reprime las voces varias veces con
tenido de la creencia distorsionada desde una posición no el fin de proporcionar una prueba completa.
confrontadora, graduando las creencias delirantes en orden En el caso de las demás creencias, se negocia la comproba-
de importancia para el paciente). ción empírica entre el paciente y el terapeuta. Es esencial
- Prueba de realidad (se realiza una comprobación de la examinar previamente que las implicaciones de la prueba no
creencia delirante del paciente mediante una prueba empí- confirmen la creencia si ésta debe ser modificada o adapta-
rica, con el fin de confirmar o refutar dicha creencia en fun- da, o si el paciente tiene lista una explicación de los resulta-
ción de los resultados que se obtengan). dos que deja intacta la creencia.
Ambas técnicas van destinadas a la reducción de las creen-
cias delirantes (PIR 10, 201). El “desafío verbal” es el com-

137
Terapia cognitiva para las ideas delirantes La posición de ACT para el tratamiento de la psicosis se cen-
tra en los siguientes aspectos:
Algunos autores están en contra del empleo de la confron-
tación, señalando que es más probable que la confronta- a) Centrarse en el impacto de los síntomas: explorar los efec-
ción produzca un mayor aumento de la perturbación que tos de la fusión cognitiva y la evitación experiencial con los
un enfoque de no confrontación. Así pues, al igual que en delirios y las alucinaciones.
la terapia cognitiva para las voces, en vez de decirles a los b) La experiencia (en las voces) o los resultados temidos de
sujetos que están equivocados, se les anima a que vean sus ella (delirios) son objetivos para evitar y controlar, lo que au-
ideas delirantes como sólo una posible interpretación de los menta el impacto de los síntomas.
acontecimientos y se les piden que consideren y evalúen un c) Los síntomas negativos pueden considerarse un posible
punto de vista alternativo. resultado de la evitación crónica (refuerzo social limitado).
d) Enfatizar la aceptación en lugar de la disputa.
Se empiezan la fase de reto verbal cuestionando sólo la evi- e) Un criterio de verdad pragmático: enfocado a hacer avan-
dencia de la creencia y en un orden que está inversamente zar las cosas en lugar de encontrar la causa de los síntomas
relacionado con su importancia para la idea delirante. Una psicóticos.
parte integral de esta discusión implica que el terapeuta f) Enfoca los síntomas indirectamente al alterar el contexto
aclare a los individuos cómo las creencias que se mantienen dentro del cual se experimentan en lugar de la frecuencia y
con firmeza pueden ejercer una profunda influencia sobre la credibilidad per se.
su conducta y su afecto. El terapeuta sigue luego cuestio-
nando la creencia misma en tres etapas: en primer lugar, se Las características específicas de las personas con psicosis
señala y se discute cualquier inconsistencia e irracionalidad; hacen necesaria una serie de adaptaciones de los protocolos
en segundo lugar, se ofrece una explicación alternativa, es estándar de la ACT, con el fin de facilitar el recuerdo y la
decir, que la idea delirante se formó en respuesta a, y como comprensión:
una forma de tratar de explicar, determinadas experiencias,
esto incluía a menudo la implicación de un síntoma principal, a) Emplear la repetición y una estructura clara y predecible
pero en algunos pacientes se postula que la idea delirante en las sesiones.
era, en parte, una respuesta a acontecimientos importantes b) Simplificar las metáforas y utilizar metáforas físicas, usan-
de la vida. do objetos.
c) Recurrir a videos y viñetas de casos para facilitar a los pa-
Finalmente, a la luz de esta nueva información, se reevalúa cientes compartir sus experiencias.
la interpretación del individuo y la alternativa del terapeuta. d) Utilizar una metáfora central para fomentar el aprendi-
Después del cuestionamiento verbal, el terapeuta y el pa- zaje.
ciente colaboran para diseñar y llevar a la práctica una com- e) Practicar ejercicios de mindfulness más breves y más ha-
probación empírica de la creencia mediante experimentos blados.
conductuales. f) Introducir pronto los valores en el tratamiento para pa-
cientes involuntarios.
g) Ligar la adherencia al tratamiento a los valores.
PÁG COLUMNA PÁRRAFO h) Educar a los familiares y al equipo de tratamiento cuando
sea posible sobre los objetivos de la ACT (aceptación y de-
29 1 2 fusión).
i) Advertir de posibles aumentos de síntomas de psicosis tras
las intervenciones.
j) En casos de déficits cognitivos usar ejercicios más cortos
AMPLIACIÓN y menos abstractos, aumentar las repeticiones y reducir la
duración de las sesiones, y grabar las sesiones de terapia y
simplificar la presentación de los contenidos.
Terapia de Aceptación y Compromiso para la Psicosis
(ACTp) Por otro lado, los objetivos terapéuticos para aplicar adecua-
Se presenta como una intervención alternativa a las terapias damente la ACT a la esquizofrenia serían los siguientes:
cognitivos-conductuales para el tratamiento de la psicosis.
Este modelo hace uso de una amplia serie de estrategias 1.Crear un estado de desesperanza creativa.
para fomentar la flexibilidad psicológica dependiendo de las 2.Plantear la posibilidad de que el problema sea el control.
fortalezas del paciente, de su historia de aprendizaje y de 3.Crear una distancia con respecto al lenguaje.
sus circunstancias. Los problemas de la psicosis se formula 4.Ayudar a crear un sentido trascendental del “yo”.
en términos de inflexibilidad psicológica, lo que hace que los 5.Aclarar y fortalecer los valores del paciente.
individuos experimenten las circunstancias de la vida de ma- 6.Desarrollar la voluntad.
nera limitada debido a los excesos de evitación experiencial.
Para conseguir el cambio se utilizan ejercicios experienciales, En cuanto a la evidencia empírica de la ACTp, en una recien-
metáforas y tareas conductuales, y se da menos importancia te revisión sistemática realizada con 10 intervenciones, los
a los procedimientos de análisis lógico. Asimismo, las técni resultados señalan la utilidad para el tratamiento comunita-
cas de defusión, el contacto con el momento presente y el rio y en unidades de agudos de personas con psicosis, tanto
yo, como observador de la experiencia, proporcionarían el las que viven su primer episodio psicótico como aquellas que
camino para la reducción de los niveles de preocupación y presentan un curso crónico. Además, han mostrado su efica-
de convicción de los síntomas psicóticos. cia para producir cambios en el afecto, en el funcionamien-

138
to, en las tasas de hospitalización, en la perturbación genera- La terapia AVATAR
da por los síntomas, en la credibilidad y en la flexibilidad. No
obstante, es necesario realizar estudios aleatorizados con- Es una intervención psicológica desarrollada y testada re-
trolados más amplios para proporcionar una evidencia más cientemente que tiene como objetivo la disminución de la
sólida a esta forma de intervención. frecuencia y malestar de las alucinaciones verbales auditivas
resistentes a la medicación en sujetos con trastornos psicóti-
cos. Originariamente desarrollada por el profesor Julian Leff,
consiste en la interacción a tres bandas (o triálogo) entre la
persona que oye la voz, el terapeuta y la representación di-
PÁG COLUMNA PÁRRAFO gital de la imagen y voz del ente que la persona cree que se
está comunicando con ella.
30 1 2
Mediante la ayuda de un software la persona crea la repre-
AMPLIACIÓN sentación digital de este ente (humano o no humano) y un
software adicional permite transformar la voz del terapeuta
Innovaciones en el Tratamiento de la Psicosis con el fin de conseguir el tono y timbre de la voz que la per-
sona oye. Finalmente se combinan los dos procesos para dar
●PROYECTO HORYZONS como resultado una única simulación computarizada (agen-
HORYZONS nace con el propósito de facilitar el acceso a in- te virtual o avatar) que el terapeuta utilizará para establecer
tervenciones basadas en la evidencia a los jóvenes con psi- un diálogo con la persona.
cosis, haciendo uso de una página web interactiva. Se trata
de utilizar la tecnología digital para extender los beneficios El terapeuta tiene dos funciones claves:
de las tradicionales intervenciones psicosociales cara a cara. a) Ser capaz de representar de manera veraz el carácter y
Este sistema comprende una plataforma que facilita el ac- papel de la voz en la relación con la persona.
ceso a una amplia variedad de intervenciones psicosociales b) Ayudar a la persona a interactuar con la voz y enfrentarse
interactivas, potenciadas por un entorno de red social online a los comentarios negativos de esta con el fin de modificar
moderado por profesionales y usuarios expertos que procura dicha relación.
la interacción social y el apoyo mutuo (PIR 19, 36)
Así pues, está integrado por: Esta novedosa intervención psicológica es parte de la nueva
generación de terapias que adopta explícitamente un enfo-
a) Intervenciones psicosociales interactivas que relacional y dialéctico para trabajar con alucinaciones
Al ingresar en el sistema, el paciente responde a una serie de auditivas de carácter negativo. No busca necesariamente la
preguntas estandarizadas que guían la entrega de módulos erradicación de la experiencia de oír voces, sino que se cen-
de psicoeducación interactivos personalizados. Los módulos tra en aspectos específicos de la relación entre la persona y
están basados en intervenciones psicosociales a partir de las la voz con el fin de mejorar las atribuciones de control por
investigaciones desarrolladas en los últimos años, y se diri- parte del individuo sobre la voz.
gen a factores de riesgo clave para la recaída psicótica y as-
pectos fundamentales en el proceso de recuperación social La terapia AVATAR se encuentra aún en sus inicios. Los dos
y vocacional. estudios clínicos realizados han mostrado la eficacia de esta
intervención a corto plazo, pero no aportan evidencia sufi-
b) Red social entre iguales online (The Cafe) ciente sobre los resultados a largo plazo.
The Cafe incluye un portal de noticias al estilo Facebook don-
de los usuarios y moderadores pueden publicar comentarios Diálogo Abierto
e información, subir imágenes y videos, y dar “me gusta” a Este abordaje se ha convertido en el de mayor éxito en el
diferentes contenidos. mundo occidental para el tratamiento de la psicosis. Tal
como explican sus creadores (Seikkula y Trimble, 2005), su
c) Moderación por expertos éxito se debe a la integración de dos aspectos fundamen-
La moderación por parte del personal (psicólogos clínicos y tales: el cambio estructural en el sistema de atención y el
trabajadores sociales expertos en recuperación vocacional) proceso dialógico que se desarrolla en las reuniones que
es esencial para aumentar la adhesión al sistema. El modera- mantienen con las personas a las que atienden.
dor puede recomendar determinados módulos que el usua- Para entender la propuesta de este enfoque, es imprescindi-
rio a su vez puede seguir o descartar. ble partir de su núcleo fundamental, que es el cambio en la
conceptualización de la psicosis. Diálogo Abierto considera
Los resultados del estudio piloto sugieren que HORYZONS que experiencias como las alucinaciones incluyen informa-
es una prometedora intervención online para jóvenes con ción sobre eventos, muchas veces traumáticos, de la vida
un primer episodio de psicosis, ya que consiguió una alta de las personas. Lo que llamamos psicosis son vivencias que
aceptación y uso, un bajo número de abandonos, una alta están intentando comunicarse pero que aún no han encon-
satisfacción y una mayor conexión social y empoderamiento. trado las palabras para hacerlo. La psicosis se entiende, por
Estamos a la espera de que se publiquen los datos del ensayo tanto, como “una alienación temporal radical de las prácti-
clínico controlado de cinco años, que pretende probar su cas comunicativas compartidas”, una “tierra de nadie” en la
eficacia. que las experiencias emocionales intensas no tienen palabras
para poder ser comunicadas. Así pues, los síntomas psicóti-
cos que aparecen en las crisis no son síntomas de una enfer-
medad subyacentes, sino maneras de sobrevivir experiencias

139
extremadamente confusas, dolorosas o extrañas, y estrate- traba recibiendo una prestación por discapacidad, y sólo el
gias que la mente pone en funcionamiento como respuesta 29% había usado neurolépticos en este periodo de segui-
a determinadas situaciones o contextos. miento (Seikkula et al., 2006). Se han realizado otros estu-
En cuanto a la perspectiva del dialogismo, el diálogo es más dios en los que los hallazgos van en esta misma dirección.
que una conversación que busca llegar a un acuerdo entre
las partes implicadas. Dialogar es más bien “el arte de pensar
juntos”. Para establecer verdaderos diálogos, es imprescin-
dible considerar al interlocutor como un “sujeto-otro”, esto TEMA 4: TRATAMIENTO DE LOS TRASTORNOS DEL ES-
es, como alguien capaz de explicarse el mundo y de ver la TADO DE ÁNIMO
realidad con sus propios ojos, aunque esa realidad sea dife-
rente a la propia.
PÁG COLUMNA PÁRRAFO
Por otro lado, la modificación en la organización del trata-
31 1 4
miento es uno de los factores que el equipo de profesionales
que ha desarrollado este enfoque señala como determinan-
tes de los buenos resultados que obtienen. El cambio es- AMPLIACIÓN
tructural al que aluden se basa en la disponibilidad y flexibi- TRATAMIENTO FARMACOLÓGICO
lidad de un equipo profesional no jerarquizado, que cuenta
con un teléfono abierto 24 horas al día, al que puede llamar La técnica a día de hoy consiste en la administración contro-
cualquier persona. Cuando alguien llama, se acuerda con él lada de una breve descarga eléctrica, aplicada en la cabe-
dónde le gustaría ser atendido y qué otras personas sería za y que provoca una convulsión eléctrica (Entre 6 y 10-12
interesante o importante que asistiesen a ese encuentro. La sesiones). Habitualmente se realizan entre 2-3 sesiones se-
primera reunión significa la constitución de lo que llaman sis- manales, y puede hacerse una aplicación de la estimulación
tema de tratamiento, que está formado por los profesionales unilateral (en el hemisferio no dominante) o bilateral (nor-
que han acudido, la persona en crisis y su red social. A partir malmente bitemporal o temporofrontal, aunque en algunos
de ese momento todas las decisiones se toman en las reunio- estudios se ha aplicado bifrontal en un intento de reducir los
nes que mantiene el sistema. De esta manera, atienden a las efectos adversos). Se ha demostrado que la aplicación bilate-
personas desde el inicio en su propio contexto, y el ingreso ral es más efectiva que la unilateral.
en una planta de psiquiatría deja de ser la primera opción.
La TEC está indicada como tratamiento para la depresión, es-
pecialmente para la endógena. En estos casos, los efectos de
Los siete principios básicos de este enfoque son: la TEC pueden ser más rápidos y en ocasiones más efectivos,
sacando a una persona de una depresión profunda en pocos
1.Respuesta inmediata: la primera reunión se celebra antes días. El principal efecto secundario de la TEC es la alteración
de que transcurran 24horas. de la memoria tanto retrógrada como anterógrada (transi-
toria y reversible: sólo en el 1% de los pacientes persiste un
2.Flexibilidad y movilidad: adaptándose a las necesidades es- déficit de memoria retrógrada para nombres y fechas).
pecíficas y cambiantes de cada paciente y cada red social. Dada la eficacia de los antidepresivos actuales para el tra-
3.Perspectiva de red social: invitando a las reuniones de tra- tamiento de la Depresión, se recomienda la TEC sólo en los
tamiento a todas las personas para las que la situación de siguientes casos (PIR19, 182):
crisis que está ocurriendo sea relevante y que estén dispues-
tos a colaborar. 1) Riesgos serios de suicidio.
2) Depresiones con importante inhibición o agitación psico-
4.Responsabilidad: la cual es asumida por el profesional que motora, e ideas delirantes.
recibe la llamada, organizando al equipo. 3) Grave compromiso del estado general (desnutrición, des-
hidratación)que requieren de una intervención rápida y efi-
5.Continuidad psicológica: permaneciendo el mismo equipo caz.
durante todo el proceso, no derivando a otros equipos a lo 4) Fracaso de los antidepresivos, al menos dos, de distinta
largo del proceso. familia, por tiempo y dosis suficientes.
5) Contradicciones para el uso de antidepresivos por enfer-
6.Tolerancia a la incertidumbre: aplazando la toma de deci- medades (cardiovasculares, hepáticas, etc.), o porque duran-
siones hasta que el diálogo produzca respuestas que disuel- te el tratamiento aparezcan efectos colaterales muy intensos.
van la necesidad de actuar. 6) Historia personal de buena respuesta a TEC y/o escasa
respuesta a los antidepresivos en episodios previos.
7.Dialogicidad: ya que el diálogo facilita la conexión de ideas
previamente experimentadas como inconexas, lo que permi- La TEC no tiene contraindicaciones absolutas, aunque sí
te ir desarrollando una narrativa que va llenándose de sen- ciertas situaciones de riesgo relativo: presencia de lesiones
tido. cerebrales ocupantes de espacio (tumores, o hemorragias)
o cualquier otra situación en la que esté elevada la presión
La investigación sobre este abordaje ofrece resultados es- intracraneal, enfermedades cardiovasculares recientes, trata-
peranzadores. En un estudio de seguimiento de cinco años miento con IMAO o litio, y los riesgos propios de la anestesia
se encontró que el 82% de las personas no tenían ningún general.
síntoma psicótico, el 86% había retomado sus estudios o un
trabajo a jornada completa, únicamente el 14% se encon- Podemos decir que la TEC es un tratamiento efectivo en la

140
depresión mayor grave, que es más efectiva que el trata-
PÁG COLUMNA PÁRRAFO
miento a corto plazo con algunos antidepresivos, y que la
combinación con farmacoterapia no ha demostrado un ma-
33 2 2
yor efecto a corto plazo que la TEC sola. También se ha visto
que en pacientes que han sido tratados con TEC, la conti-
nuación con antidepresivos tricíclicos y/o litio ha reducido las
recaídas en comparación con el placebo. CAMBIO DE ALGUNA FRASE DEL TEXTO
A pesar de que existen varias teorías explicativas de los me-
canismos de acción de la TEC ninguna ha podido esclarecer- (...) y continuar si fuera necesario con tratamiento farmaco-
los por el momento. lógico como mantenimiento).
Existe un método alternativo para modificar la actividad eléc- Dada la procedencia inicial de esta terapia, podemos decir
trica cerebral que aún muestra resultados contradictorios: la que se inscribe dentro del modelo médico (PIR19, 30).
estimulación magnética transcraneal.

Por último, cabe señalar el uso de la fototerapia o terapia lu- TEMA 7: TRATAMIENTO DEL TRASTORNO ESTRÉS POS-
mínica dentro de los tratamientos biológicos, concretamente TTRAUMÁTICO
para el trastorno afectivo estacional (TAE), que es un trastor-
no caracterizado por alteraciones del humor comunes en la PÁG COLUMNA PÁRRAFO
depresión, que suceden habitualmente en invierno y se re-
suelve en primavera. Para el tratamiento del TAE existen hoy 55 2 3
en día diferentes herramientas terapéuticas: la fototerapia,
la farmacoterapia y la psicoterapia. Las indicaciones para la
fototerapia son: depresión estacional no psicótica bien docu- AMPLIACIÓN
mentada, episodios depresivos en invierno, sin existencia un
trastorno depresivo mayor recurrente, episodios depresivos
en invierno, sin existencia de un TAB II, y episodios depresi- EMDR: Esta técnica consta de 8 fases tanto para niños
vos estacionales leves. La fototerapia será en estos casos la como para adultos (PIR 19, 70) en las que el sujeto se
primera línea de tratamiento cuando no existencia de ries- expone al recuerdo traumático mientras recibe estimulación
go suicida, cuando haya una contraindicación médica para bilateral.
toma de antidepresivo, antecedentes personales de buena
respuesta, el paciente lo requiera y cuando esté indicado por
un terapeuta experto. TEMA 15: TRATAMIENTO DE LOS TRASTORNOS DE LA
PERSONALIDAD
El mecanismo de acción de la fototerapia consiste, en parte,
en disminuir la depleción de triptófano presente en los pa- PÁG COLUMNA PÁRRAFO
cientes con TAE. La dosis inicial de administración debe de
ser de 15-20 min al día y gradualmente ir aumentando hasta 89 1 Sustituir todo el
apartado “Terapia
30-45 min al día a 10000 lux (unidad photométrica). Esta
cognitiva centrada
dosis es suficiente para revertir los síntomas en la mayoría de en los esquemas de
los pacientes. La duración media del tratamiento debe de ser Young”
de unas 2-4 semanas, ya que el efecto antidepresivo pleno
de la fototerapia tarda unas 3 semanas en aparecer. Los AMPLIACIÓN
estudios sobre la hora de administración de la luminoterapia
han encontrado resultados contradictorios. Parece que las Terapia cognitiva centrada en los esquemas de Young
sesiones por la mañana producen mejores resultados que las
administradas por las tardes, aunque estas sí son superiores Esta terapia es una adaptación de la terapia cognitiva de
al placebo (PIR19, 29). Beck (PIR13, 108), que integra con elementos de la teoría
del apego, la terapia de la gestalt, las relaciones objetales,
el constructivismo y el psicoanálisis. A diferencia de otras
En lo relativo a los fármacos, los más estudiados han sido terapias cognitivas de corte racionalista, enfatiza la impor-
los ISRS (fluoxetina). El estudio CAN-SAD comparó la efica- tancia de llevar a cabo un análisis causal sobre la génesis de
cia de la fototerapia frente a la fluoxetina, y la fototerapia las creencias y distorsiones cognitivas del paciente, y hace
mostró respuesta más temprana y menor número de efectos mayor hincapié́ en la relación terapéutica y en las técnicas
secundarios, pero sin diferencias estadísticamente significa- emocionales (PIR 08, 148).
tivas en cuanto a la respuesta clínica entre las dos medidas Pone el foco de atención en los esquemas disfuncionales
terapéuticas. tempranos, definidos como patrones cognitivos y afectivos
que engloban recuerdos, emociones, cogniciones y sensa-
ciones corporales relativos a uno mismo y la relación con los
otros. Estos esquemas tienen su génesis en la infancia y la
adolescencia y están directamente relacionados con la satis-
facción de las necesidades emocionales nucleares, las expe-
riencias tempranas y el temperamento emocional. Además,
estos esquemas son la antesala de determinado afronta-
miento que en la edad adulta se convierte en desadaptativo.

141
Young define 5 emociones nucleares básicas universales que • Cambio de los esquemas.
deben ser satisfechas o de lo contrario pueden ser el germen Young propone cuatro tipos de intervención para modificar
de estos esquemas disfuncionales: los esquemas:

1) Apego seguro hacia otros (seguridad, estabilidad y acep- - Técnicas emotivas. Se suelen utilizar al inicio del tratamien-
tación). to para ‘’liberar” los esquemas y hacerlos más flexibles al
2) Autonomía, competencia y sentido de identidad. cambio. Suelen ser técnicas de la terapia Gestalt (silla vacía,
3) Libertad para expresar necesidades válidas y emociones. inversión de roles…).
4) Espontaneidad y juego.
5) Límites realistas y autocontrol. - Técnicas interpersonales. Es de especial importancia para
esta terapia la relación terapéutica. El estilo del terapeuta,
Determina también 18 tipos de esquemas que los clasifica además, debe ser diferente en función de los esquemas que
en cinco dimensiones, cada uno de ellos generado por dife- se perciban en el paciente. Independientemente de este
rentes experiencias en la infancia, determinado estilo familiar estilo, debe tener dos características fundamentales: la re-
y la satisfacción de las necesidades que se ha realizado en parentalización limitida y la confrontación empática. Con
estos primeros años de vida. el primer concepto Young se refiere que el terapeuta debe
comportarse “como aquel padre/madre que nunca tuvo”,
Un concepto de gran relevancia para el proceso terapéutico es decir, debe seguir un estilo genuino que promueva un
y, en concreto, para el trabajo con pacientes con trastorno apego seguro, de cuidado, de validación y empático. Dicha
límite de la personalidad es el “modo”. Con este término parentalización va a ser transversal a toda la terapia indepen-
Young se refiere al agrupamiento específico de esquemas o dientemente de la técnica concreta o la fase en la que nos
respuestas de afrontamiento activos momento a momento encontremos. Con el concepto de confrontación empática
que experimentamos todas las personas y que han sido acti- se refiere a tener empatía hacia los esquemas del paciente
vados por situaciones vitales. Este concepto fue incluido en cuando surgen hacia el terapeuta junto a la devolución al
la terapia por la dificultad de trabajar a la vez con todos los paciente de que esas reacciones hacia el terapeuta a menu-
esquemas que tiene activados el paciente con TLP. do están distorsionadas y muestran esquemas y estilos de
afrontamiento disfuncionales.
Young ha identificado 10 modos que ha clasificado en 4,
aunque dice que no descarta que haya más. En la tabla del - Técnicas cognitivas. Repasar las pruebas en apoyo de los
apartado final se pueden ver los diferentes modos. Se pre- esquemas; examinar de forma critica las pruebas de apoyo;
tende que cada vez más prevalezca el modo adulto sano. repasar las pruebas que contradicen los esquemas; etc.

El objetivo de la terapia es buscar formas de satisfacer las - Técnicas conductuales. Para cambiar las conductas regula-
necesidades emocionales centrales más adaptativas.Para das por los esquemas.
conseguir este objetivo, la terapia se divide en dos partes
básicas: la conceptualización del caso y el cambio de los es-
quemas:

• Conceptualización del caso (identificación de esquemas).

Young (1994) propone ocho pasos para la identificación de


los esquemas:

1.Identificar problemas y síntomas en la sesión inicial. Conse-


guir una anamnesis concreta.
2.Pasar el Inventario Multimodal de la Historia de Vida, de
Lazarus (1971), y el Cuestionario de Esquemas, de Young
(1994).
3.Educar al paciente sobre los esquemas y hablar sobre el
Cuestionario de Esquemas.
4.Desencadenar esquemas en la sesión y fuera de ella.
5.Enfrentarse a la evitación y a la compensación de esque-
mas.
6.Identificar conductas reguladas por esquemas: manteni-
miento, evitación y compensación de esquemas.
7.Integrar la información anterior en un concepto coherente
sobre el paciente. Vincular los problemas actuales, las ex-
periencias infantiles (orígenes), las emociones y los patrones
conductuales de la adolescencia y de la vida adulta, y la re-
lación terapéutica con los esquemas tempranos disfuncio-
nales.
8. Colocar como objetivo un esquema central para el pro-
ceso de cambio.

142
ESQUEMAS ASOCIADOS AL DOMINIO DE DESCONEXIÓN Y RECHAZO

ESQUEMAS ASOCIADOS AL DOMINIO DE DETERIORO EN AUTONOMÍA

ESQUEMAS ASOCIADOS AL DOMINIO DE LÍMITES DEFICITARIOS

143
ESQUEMAS ASOCIADOS AL DOMINIO DIRIGIDO POR LAS NECESIDADES DE LOS DEMÁS

ESQUEMAS ASOCIADOS AL DOMINIO DE SOBREVIGILANCIA E INHIBBICIÓN

DIFERENTES DOMINIOS Y ESQUEMAS CONTEMPLADOS POR YOUNG

DIFERENTES MODOS IDENTIFICADOS POR YOUNG.

144
-Narrativas empobrecidas / estilo narrativo intelectualizado:
PÁG COLUMNA PÁRRAFO
dificultad para proporcionar memorias autobiográficas ricas
y detalladas a la hora de describir los aspectos cognitivos,
90 1 2
emocionales y somáticos de la experiencia subjetiva.

AMPLIACIÓN -Fluctuante / limitado sentido de agencia: es decir, de la con-


ciencia subjetiva de ser poseedor de deseos, intenciones y
Terapia Metacognitiva Interpersonal (MIT) para los metas, así como de la capacidad de iniciar, llevar a cabo y
Trastornos de la Personalidad de Dimaggio y Semerari controlar acciones destinadas a lograrlos.
La metacognición es la habilidad humana para compren-
der los estados mentales propios y de los otros, así como -Esquemas interpersonales patógenos: los esquemas inter-
su capacidad de reflexión y dominio en relación a ello. Los personales son estructuras cognitivo-afectivo-somáticas que
trastornos graves de la personalidad presentan un mal fun- las personas construyen para dar sentido a las relaciones so-
cionamiento metacognitivo, que es distinto en los diferentes ciales. En los TP, los esquemas son fácilmente activados y
casos, es decir, la diferencia en los distintos perfiles de mal frecuentemente generan emociones intensas y desreguladas
funcionamiento se refleja en diferencias importantes en la que son difíciles de modular. Los pacientes son incapaces de
realidad clínica. cuestionar las asunciones subyacentes al esquema en rela-
ción a si son ciertas o falsas.
La metacognición supone una combinación de habilidades
que permite a los humanos: -Ciclos interpersonales disfuncionales: sobre la base de un
esquema interpersonal individual y de cómo sus deseos se
-Identificar estados mentales (EMs) y atribuirlos a sí mismos emparejan con los deseos, perspectivas, intenciones y pro-
o a otros yectos de los otros, algunos ciclos interpersonales llegan a
-Pensar, reflexionar y razonar acerca de los propios EMs y ser activados en las relaciones actuales. El ciclo interpersonal
sobre los de los demás. es una secuencia intersubjetiva de eventos que inducen mu-
-Usar los conocimientos y reflexiones sobre los EMs para to- tuamente acciones, conductas y afectos que mantienen la
mar decisiones, resolver problemas o conflictos psicológicos relación disfuncional (proceso automáticos e inconscientes).
o interpersonales y manejar el sufrimiento subjetivo.
-Estados mentales recurrentes: formas de experiencia sub-
Las habilidades de la metacognición son: jetiva caracterizados por un grupo estable y recurrente de
elementos como pensamientos, creencias, emociones, sen-
-Supervisión o monitoreo: La comprensión de la propia men- timientos, sensaciones físicas, intenciones y deseos que se
te tiene, como punto de partida, la identificación de los pen- manifiestan conjuntamente en el fluir de la conciencia.
samientos y de las emociones que constituyen los estados
mentales. La actividad de monitoreo no se limita a la simple -Disfunciones metacognitivas.
identificación de cada uno de los componentes particulares,
sino que normalmente somos capaces de reconocer los ne- -Estrategias de afrontamiento desadaptativas y sesgos cog-
xos inmediatos que existen entre ideas y emociones, y entre nitivos.
éstas y los hechos externos.
-Dificultades para regular los afectos.
-Diferenciación: capacidad que nos permite jugar con la rea-
lidad, perdernos en fantasías sin que éstas se transformen De los procedimientos psicoterapéuticos de los que hace uso
en acciones, y nos capacita para darnos cuenta de que nues- cabe destacar que:
tra visión de las cosas puede no ser verdadera, dado que el
comportamiento está guiado por creencias que pueden ser -MIT adopta procedimientos formalizados, detallados paso
falsas. a paso.

-Integración: Capacidad para tener una visión unificada de -Lo anterior implica determinar una secuencia de interven-
uno mismo, a pesar de la alternancia de diferentes EMs, al- ciones, pero también resaltar los marcadores de progreso,
gunas veces inclusive contradictorios y de la alternancia de la que nos indicarán si es posible / aconsejable pasar a interven-
propia conducta ante diferentes contextos. ciones más complejas (previo a ello, las más simples, habrán
de haberse realizado con éxito).
-Descentramiento: Habilidad para ponerse en el lugar del
otro y formular inferencias sobre sus estados mentales sin -Según la terapia avanza, las intervenciones requerirán ac-
tener en cuenta la perspectiva propia, la manera propia de tos metacognitivos cada vez más complejos por parte de los
evaluar o de interpretar los eventos y la participación en la pacientes.
relación.
-MIT se divide en 2 secciones:
-Dominio: habilidad para usar el propio conocimiento de los
estados mentales para reducir el malestar psicológico y resol- •Formulación compartida del funcionamiento: me-
ver problemas interpersonales y conflictos. jora de habilidades autorreflexivas. Se detalla con el paciente
su funcionamiento mental.
Así pues, la MIT se enfoca en las siguientes áreas psicopato-
lógicas de los pacientes con TP: •Promoción del cambio: acciones dirigidas a redu-

145
TEMA 27: TRATAMIENTO DE LOS TRASTORNOS EMO-
cir el sufrimiento, construir narrativas alternativas, acceder a CIONALES EN LA INFANCIA
experiencias subjetivas más saludables, adoptar comporta-
mientos más adaptativos. PÁG COLUMNA PÁRRAFO
6.PRINCIPIOS GENERALES DE INTERVENCIÓN DESDE 127 1 2
UNA PERSPECTIVA INTEGRATIVA

Los objetivos de la psicoterapia ante un TP deben enmar-


carse dentro de un plan de confirmación de la experiencia AMPLIACIÓN
disfuncional, respetando el modo de ser del paciente. Desde
esta perspectiva: 27.5. TAG Y TAS

a) Los cambios deben ser graduales, progresando desde las El tratamiento de elección es la terapia cognitivo-conductual
manifestaciones periféricas hacia los patrones básicos de la o TCC, incluyendo los siguientes componentes:
organización personal (PIR 19, 50).
b) El proceso terapéutico puede desdoblarse en diferentes Entrenamiento en relajación progresiva (técnica de elección
etapas, de acuerdo con las condiciones del paciente y los dentro de la TCC), adaptada a los niños u otras variantes
recursos disponibles. como masaje relajante, juegos, o en imaginación. Algunas
c) La relación terapéutica requiere un elevado nivel de com- pautas que podemos seguir para aplicar la relajación infantil
promiso de ambas partes. Será de ayuda considerar la posi- son: hacer sesiones más cortas y frecuentes, eliminar estí-
bilidad de que el rol del terapeuta se distribuya en un equipo mulos distractores, presentar los ejercicios como un juego,
asistencial. adaptar el lenguaje al del niño, modelar los ejercicios o uti-
d) La acción psicoeducativa suele requerir distintas presen- lizar muñecos antes de aplicarlos, instigación y guía física,
taciones. trabajo con grandes zonas musculares y en total en menor
e) Es útil considerar la posibilidad de modificar el contexto número de zonas a trabajar.
como un recurso terapéutico.
f) Los procedimientos indirectos suelen facilitar habitualmen- Para poder aplicar la técnica de relajación progresiva, Cau-
te la promoción de cambios. tela y Groden (1989) señalan unas habilidades mínimas que
g) Los dispositivos grupales potencian la adquisición de nue- deberá adquirir el niño antes de comenzar la enseñanza de
vos aprendizajes y la consolidación de la experiencia. la relajación (PIR 19, 76):
h) El terapeuta debe haber recibido el entrenamiento ade-
cuado y contar con una supervisión regular de la tarea. • Permanecer quieto durante cinco segundos.
• Mantener la mirada durante cinco segundos.
Es fundamental entender que el objetivo final de esta psi- • Ser capaz de imitar acciones sencillas como levantar su
coterapia no es la remisión de una situación clínica determi- mano por encima de la cabeza, tocar la mesa y tocarse el
nada, ni la eliminación de un foco disfuncional como ocurre pecho.
con otras patologías. Esto tiene un doble significado. Por un • Obedecer órdenes sencillas como “levántate”, “siéntate”
lado, indica que los cambios llegarán hasta el límite de lo que y “ven aquí”.
admitan las opciones funcionales de la estructura básica, y
esto implica que el paciente conservará, a grandes rasgos, Una variante de la relajación progresiva, que incluye el em-
el perfil y el estilo habitual de su organización mental carac- pleo de imágenes que ayudan al niño a tensar cada grupo
terística. Al mismo tiempo, más allá de cualquier decisión muscular y a mantenerlo motivado hacia el ejercicio es el en-
consciente, el paciente movilizará acciones para conservar su trenamiento en imaginación de Koeppen (1974). Se narran
identidad frente a las justificadas necesidades de cambio que varias historias que el niño va escuchando, y en un momento
le exige su padecimiento. Una de las paradojas centrales de de ellas debe tensar una zona de su cuerpo:
la existencia humana (identidad vs. cambio) se condensa en
esta situación clínica: el malestar o el sufrimiento que lleva
al paciente a la consulta exige un cambio, mientras que la
necesidad del reconocimiento (de sí mismo y de los demás) •Manos y brazos: «imagina que estás exprimiendo un limón,
lo obliga a preservar el núcleo de la identidad que garantiza apriétalo fuertemente en tu mano hasta que no le quede
la continuidad de su vida psíquica. nada de jugo».
Este fenómeno señala la importancia que tienen las estrate- •Brazos y hombros: «imagina que eres un gato que se des-
gias de validación como un paso introductorio para la consti- pereza, estirando sus patas y arqueando el lomo» (PIR 19,
tución de una buena relación terapéutica, y se asocia con las 79).
propuestas de una filosofía de aceptación como una manera •Hombros y cuello: «imagina que eres una tortuga, toman-
de ampliar nuestra comprensión del proceso de cambio te- do el sol en una roca. De repente ves acercarse una gran ga-
rapéutico. viota dispuesta a picarte. Escóndete dentro de tu caparazón,
encogiendo el cuello y los hombros para protegerte».
•Mandíbula: «imagina que estás masticando un gran chicle,
duro y elástico. Tensa tus dientes como si te costara mor-
derlo».
•Cara y nariz: «imagina que una mosca se ha detenido en
tu nariz, te molesta y quieres que se vaya. Intenta arrugar tu
nariz para conseguirlo».

146
•Estómago: «estás tranquilamente tumbado al sol, y ves un
pesado elefante caminar hacia ti. Imagina que va a poner su
pata sobre tu barriga. Aprieta el estómago para aguantar su
peso».
•Piernas y pies: «imagina que eres un explorador y estás ca-
minando por la selva. De repente pisas en un barrizal, pa-
recen arenas movedizas. Intenta mantenerte sin hundirte,
apretando tus piernas y encogiendo los dedos de los pies».

TEMA 36: ASMA

PÁG COLUMNA PÁRRAFO

155 2 4

AMPLIACIÓN

Otro programa de automanejo para pacientes con asma se-


ría el medidor de Peak-flow, una técnica de manejo utilizada
en atención primaria.
El flujo espiratorio máximo (FEM) o peak expiratory flow
(PEF) en terminología anglosajona, es el mayor flujo que se
alcanza durante una maniobra de espiración forzada. Se
consigue al haber espirado el 75-80% de la capacidad pul-
monar total. Refleja el estado de las vías aéreas. La utilidad
del FEM se centra en la posibilidad de medición en distintas
circunstancias mediante medidores portátiles. La ventaja de
éstos es que pueden ser transportados y manejados por el
paciente de manera sencilla. Se trata de aparatos, general-
mente tubos, que en su interior presentan un mecanismo
de pistón-muelle o de aspa que se mueve al aplicar un flujo
de aire durante una maniobra de espiración forzada. Existen
medidores portátiles electrónicos. Entre sus ventajas, obvian
la necesidad de registro manual de los valores por parte del
paciente, aumentan la exactitud del registro y graban el mo-
mento del día en que se realiza la maniobra. Su principal
inconveniente es su alto precio.
Para un correcto registro del FEM es necesario entrenar al
paciente en el uso del medidor especificando cada uno de
los pasos a seguir.
Para el seguimiento por parte del paciente se utiliza el au-
torregistro del FEM donde se apuntan los valores obtenidos
por el propio paciente en una hoja determinada a tal efecto
(diario del asmático) en la que conste además día y hora de
la medición, síntomatología, pauta terapéutica y ocasional-
mente algún comentario sobre posibles desencadenantes de
crisis (incumplimiento terapéutico, circunstancias físicas-at-
moféricas, etc.).

147
REFERENCIAS BIBLIOGRÁFICAS (AÑADIR AL FINAL DEL MANUAL):
Dimaggio, G., Semerari, A. (2003). Los Trastornos de la Personalidad. Modelos y tratamientos. Bilbao: Desclée de Brower.

Diéguez, M., González, F., Fernández, A. (2012). Terapia interpersonal. Editorial Síntesis. pp 31.

Fonseca, E. (2019). Tratamientos psicológicos para la psicosis. Madrid: Pirámide.

Caballo, V. (2007). Manual para el tratamiento cognitivo-conductual de los trastornos psicológicos. Madrid: Siglo XXI. Vol
1. p 670.

Belloch, A. (2010). Tratado de los trastornos de la personalidad. Editorial Síntesis. pp 297.

Gatón, M.A., González, M.A., Gaviria, M. Trastornos afectivos estacionales, “winter blues”. Rev. Asoc. Esp. Neuropsiq.,
2015; 35 (126), 367-380.

Guía de Práctica Clínica sobre el manejo de la depresión en el adulto Ministerio de Sanidad, 2009).

Koeppen, A. S. (1974). Relaxation training for children. Elementary School Guidance & Counseling, 9(1), 14-21.

148
149
10.3. El Modelo de McCrae y Costa.
PSICOLOGÍA DIFERENCIAL DE LA PERSONALIDAD
RESEÑAS

Es importante reseñar la generalización que se puede hacer


en el análisis y aplicación de los cinco factores en otros ám-
bitos culturales y geográficos (PIR 09, 05; PIR 11, 10). Se hi-
PÁG COLUMNA PÁRRAFO cieron estudios similares, réplicas del análisis factorial, tanto
en inglés como en italiano, holandés, húngaro, castellano o
60 1 1 alemán, apoyándose el planteamiento del modelo. Otro ele-
mento de estudio fue analizar la estabilidad de estos factores
a lo largo del tiempo, y también se pudo comprobar que se
trataba de elementos relativamente estables de la personali-
TEXTO DEL MANUAL dad que apenas se veían modificados con la edad. Dentro de
los cambios que sí se pudieron captar, encontramos que el
neuroticismo, la apertura y la extraversión aparecen menos
• Rasgos. (PIR 19, 11). elevados en grupos de mayor edad si los comparamos con
Son constructos que tienen su base en las respuestas habi- los jóvenes pero no se dan cambios en todos los rasgos de
tuales anteriores. Una persona sociable es una persona que cada factor. En general estos descensos se han demostrado
habitualmente prefiere la compañía o que suele hablar con reales con la edad, pero parciales y moderados (PIR19, 16).
otras personas.
• Tipos.
Son dimensiones continuas en las que se sitúa el sujeto en PÁG COLUMNA PÁRRAFO
algún punto de forma no categorial, refiriéndose a agrupa-
ciones de rasgos. 108 2 4

PÁG COLUMNA PÁRRAFO TEXTO DEL MANUAL

60 1 3 2.3 Genética de la conducta:


- Otros conceptos relacionados con la genética de la con-
ducta
TEXTO DEL MANUAL Además se han considerado otros conceptos:
Varianza de homogamia (S2hv) (PIR 93, 250).
10.3. El Modelo de McCrae y Costa. Se refiere a la parte de la variabilidad que el sujeto recibe por
• Neuroticismo vs. estabilidad emocional. parte de las características fenotípicas que suelen compartir
Conciben la idea de Neuroticismo como un nivel determi- los progenitores, aún cuando sus genotipos sean distintos
nado de ajuste emocional. Si este nivel de Neuroticismo es entre sí. Se ha observado que las parejas acaban desarrollan-
alto los sujetos tendrán ideas poco realistas, dificultades para do gustos o rasgos similares que terminan transmitiendo de
tolerar la frustración, necesidades excesivas o tendencia al un modo u otro a la descendencia. La influencia de esto se
malestar psicológico. Los términos que están detrás de esta mide en la varianza de homogamia.
dimensión son los de inseguridad, preocupación, tendencia Índice de heredabilidad amplia (PIR 94, 130; PIR 05, 34 ;
a la ansiedad o la depresión o desconcierto. El Neuroticismo PIR19, 17). Trata de explicar la parte del fenotipo que pro-
también está muy conectado con la impulsividad (PIR 04, viene del genotipo.
68) (PIR19, 15) en comportamientos como el consumo de
alcohol o tabaco, y la dificultad para controlar estas conduc-
tas. PÁG COLUMNA PÁRRAFO

Tabla 2. Rasgos que componen cada uno de los cinco gran- 118 1 1
des> Neuroticismo > Impulsividad (PIR04, 68; PIR15, 22;
PIR19, 15)
TEXTO DEL MANUAL

PÁG COLUMNA PÁRRAFO 4.4. Modelos factoriales de la inteligencia:


- Modelo de los tres estratos de Carroll John B. Carroll expo-
67 2 1 ne un compendio integrador en su obra “Human cognitive
abilities”, desarrollando en 1997 su teoría, que pretende ser
la síntesis final que abarque todo el mapa de capacidades
TEXTO DEL MANUAL de la inteligencia humana que se conocen o se espera que
existan. Especificamente se hallaron 2272 factores de primer
orden (“ciudades”), 572 de segundo orden (“provin- cias”)

150
y 36 de tercer orden (“comunidades”). similares a las mantenidas en el pasado, pero que generan
En el primer estrato, nivel concreto, se ubica un gran núme- desagrado o malestar en la persona.
ro de aptitudes primarias con similitudes con Guilford. En El autoconcepto actual está referido al presente, y engloba
el segundo estrato o nivel de mayor grado de generalidad el conjunto de características, pensamientos y actitudes que,
dad encontramos: Inteligencia fluida, inteligencia cristalizada en relación con uno mismo, se activan y se hacen accesibles
(PIR 16, 212; PIR19, 176), memoria y aprendizaje, percep- en un momento determinado. El hecho de que estos esque-
ción visual, percepción auditiva, capacidad de recuperación, mas centrales, los cuales definen lo que cada uno cree que es
velocidad cognitiva y rapidez de procesamiento. En el tercer y cómo es, estén también presentes cuando se activan otros
estrato hallamos el factor de inteligencia general (3G) que tipos de contenidos en contextos y momentos determina-
correspondería al G de Spearman. Los factores mejor repre- dos, da apoyo a la idea del autoconcepto como un sistema
sentados por este factor general serían la inteligencia fluida de representaciones que se mantienen relativamente inva-
y la cristalizada. riables ante los cambios situacionales; pero que comprende
también un conjunto de descripciones más flexibles, abiertas
PÁG COLUMNA PÁRRAFO a ser modificadas o revisadas en función de las demandas
que planteen las situaciones más inmediatas.
118 1 4
Un tercer tipo de autoconcepto es el que hace referencia al
futuro, el cual es denominado por Markus como los posibles
yoes. Este tipo de conocimiento engloba el conjunto de es-
TEXTO DEL MANUAL peranzas, temores y deseos que resultan relevantes para el
individuo; así como las diferentes cualidades que a la perso-
Teoría Triárquica de Sternberg na le gustaría tener o podría tener, y aquellas otras ante las
- Subteoría componencial para la inteligencia analítica (PIR que se sentiría mal en el caso de que las tuviera (PIR19, 8)
15, 38) Los posibles yoes actúan como incentivos de conductas fu-
Se encarga de estudiar la relación entre la inteligencia y el turas, en la medida en que constituyen partes de uno mismo
mundo interno del sujeto (PIR 05, 32), poniendo el énfasis que simbolizan aspiraciones y metas, aunque a veces, tam-
en los procesos que intervienen en el pensamiento. Buscará bién pueden reflejar los temores y las amenazas que tiene la
por tanto cómo se genera el comportamiento inteligente. Se persona. En este sentido los posibles yoes pueden funcionar
trata de las tres teorías, de la que más contrastación empírica como un puente cognitivo entre el yo presente (lo que yo soy
se ha obtenido. aquí y ahora) y el yo futuro (lo que yo quiero o puedo ser),
La unidad de análisis de la inteligencia será el componente, favoreciendo la construcción mental de escenarios y cursos
será un proceso básico de información que actúa sobre las de acción apropiados.
representaciones internas de los datos recogidos del exterior.
(PIR19, 13) Al operar puede traducir estos datos externos en De este modo un individuo puede seleccionar conductas de
representaciones conceptuales, transformar estas represen- aproximación, para lograr que partes potenciales de sí mis-
taciones en otras o desarrollar a partir de ellas, una acción mo, que representan cualidades positivas, se conviertan en
motora. reales; o, por el contrario, puede actuar de forma que evite
que algunos de los posibles yoes que son temidos o rechaza-
dos se hagan realidad.
PÁG COLUMNA PÁRRAFO

100 2 4
PÁG COLUMNA PÁRRAFO

100 2 4
TEXTO AMPLIADO SI LA FUENTE ES OTRA

14.4 La multiplicidad de yoes.


- La propuesta de Markus sobre el “Self” TEXTO AMPLIADO SI LA FUENTE ES OTRA
Teniendo en cuenta el marco de referencia temporal de los
contenidos del autoconcepto, Markus propone una posible 14.7. INFLUENCIAS CULTURALES EN LA PERSONALIDAD
diferenciación entre los diversos tipos de «yoes» o autocon- La cultura es un sistema de significados (creencias, valores,
ceptos. Así, en la persona, puede coexistir un autoconcep- normas, actitudes, conductas, conocimientos, habilidades,
to ligado al pasado («cómo era yo cuando mis hijos eran etc.) compartido por un grupo determinado, que habla una
pequeños»), otro referido al momento presente («cómo lengua común, en un período histórico específico y en una
soy cuando estoy con mi familia»), y otro relativo al futuro región geográfica concreta (Triandis y Suh, 2002). El sistema
(«cómo sería mi parte más optimista»). de significados cultural debe ser aprendido por cada gene-
ración mediante el proceso de socialización, a través de las
El autoconcepto relativo al pasado comprenderá el conoci- prácticas de crianza de los padres, las escuelas y otras insti-
miento que se tiene acerca de cómo era la persona en un tuciones sociales. La dimensión principal de diferenciación
tiempo pasado. Este tipo de autoconcepto se hace más so- cultural estudiada ha sido la de individualismo-colectivismo
bresaliente cuando la persona ha cambiado algunos de los (refiriéndose al grado en que la persona está integrada en
rasgos o atributos que le caracterizaban en otros momentos el grupo), que define a las personas como idiocéntricas o
de su vida, pero que ya no lo hacen. O cuando se desea cam- alocéntricas (PIR 19, 5).
biar algunas de las partes del autoconcepto actual que, son

151
Triandis (2001) diferencia entre culturas colectivistas e indi- éxito y las colectivistas a la evitación del fracaso. Esto pro-
vidualistas. Las personas dentro de las culturas colectivistas voca que en los individualistas la motivación se incremente
enfatizan la conexión con su grupo, considerándose inter- después del éxito mientras que en las colectivistas lo haga
dependientes del mismo, se describen más como miembros después del fracaso. Los individualistas presentan sesgos de
del grupo que como individuos, creen que la conducta social autoensalzamiento y los colectivistas presentan mayores ta-
está más determinada por los elementos externos (normas sas de autocriticismo, ambas características asociadas con el
del grupo) que por los internos (rasgos o características de buen rendimiento.
personalidad) y enfatizan las metas colectivas. Sin embargo, - Emociones y bienestar:
las personas de culturas individualistas enfatizan el carácter Las personas de sociedades colectivistas presentan más emo-
único e independiente del individuo, se describen a sí mis- ciones implicativas (simpatía, respeto, cercanía, amistad, cul-
mas mediante atributos personales, creen que la conducta pa, vergüenza…) que no implicativas (orgullo, autoconfian-
social está más determinada por estos atributos personales za, sentimiento de superioridad, ira, frustración…), mientras
que por las características externas y dan prioridad a sus me- que en las sociedades individualistas se presenta el patrón
tas personales sobre las del grupo. inverso. En las sociedades colectivistas también destaca el
control de la expresión emocional (sobre todo de la negati-
Tradicionalmente, enmarcamos dentro de las culturas indivi- va), ya que se considera que la libre expresión de las mismas
dualistas a las de Norte América, Australia y Europa y dentro podría interferir en las relaciones y alterar la consecución de
de las colectivistas al este de Asia (China, Japón, Corea), Áfri- relaciones armónicas con los demás.
ca, Latino-América y las islas del Pacífico. - Cognición:
Con respecto al error fundamental de atribución (sobreesti-
Hay dos modelos teóricos en el estudio de la personalidad y mar las causas internas e infravalorar las situacionales en la
la cultura: explicación de la conducta), se ha comprobado que no es un
1. La psicología cultural enfatiza que la personalidad sesgo universal como se creía y que es mucho más débil en
no se puede separar del contexto social en el que se desarro- las culturas colectivistas. Los individuos colectivistas utilizan
lla y se expresa y, por tanto, se considera que la personalidad en mayor medida causas de tipo situacional para explicar la
se construye socialmente a través de las interacciones entre conducta. En los colectivistas la atención es más holística,
el individuo y su ambiente cultural. mientras que en los individualistas está más focalizada. En las
2. La psicología transcultural que refiere que persona- culturas individualistas predomina el pensamiento analítico.
lidad y cultura son entidades distintas, entendiendo que la - Rasgos:
cultura es la variable independiente que tiene influencia en La psicología cultural no pone en duda la existencia de los
la personalidad, entendida como variable dependiente. rasgos en las diversas culturas, sino su relevancia. Se han
hecho estudios transculturales sobre los cinco grandes fac-
Podemos estudiar algunos constructos siguiendo a ambos tores de personalidad, basándose en el uso del NEO-PI-R y
modelos teóricos, comparando las culturas individualistas y también replicando el procedimiento original, es decir, acu-
colectivistas: diendo al diccionario para recoger todos los términos relati-
vos a la personalidad en una lengua determinada. Se pue-
- El self: de concluir que los cinco factores se replican en países de
Se distingue el self independiente, propio de las culturas in- América del Norte y el norte de Europa pero en otros países
dividualistas, del self interdependiente, más frecuente en las emergen dimensiones indígenas o específicas, mientras que
sociedades colectivistas. Las personas individualistas se per- no se obtienen algunos de los cinco grandes y, además, las
ciben como independientes, autónomas y completas; acen- dimensiones indígenas parecen mucho más predictivas que
túan los límites entre el yo y los otros, percibiéndose como cualquiera de las cinco dimensiones básicas.
agentes separados que actúan para conseguir sus propias
metas; su propio self actúa como fuente de acción y motiva-
ción; se describen con atributos internos y se experimentan PÁG COLUMNA PÁRRAFO
como inmutables a través de las situaciones (el mundo es más
mutable que el self). Por otra parte, las culturas colectivistas 108 2 1
se perciben como interdependientes de los demás, sienten
conexión con los miembros del grupo a los que pertenecen
y actúan en sintonía con las metas y los deseos de los otros TEXTO DEL MANUAL
cercanos; son los otros la fuente de acción y motivación; se
describen a sí mismas mucho más mediante relaciones y ro- La correlación entre el ambiente y el genotipo del sujeto se
les que con atributos internos; conciben estos roles como da de tres maneras:
inmutables y al mundo social como duradero y permanente. • Pasiva. (PIR19, 9)
- Autoestima: Un rasgo se ve facilitado tanto por la predisposición genéti-
Se ha encontrado que las personas individualistas puntúan ca como por la influencia del ambiente.
más alto en las escalas de autoestimas que las colectivistas. • Reactiva.
La autoestima es uno de los predictores del bienestar más Los sujetos, como reacción al genotipo diferente proporcio-
potentes en las culturas individualistas pero no en las colec- nan un ambiente adecuado al mismo, capaz de reflejar y
tivistas, en las que la armonía en las relaciones y la adheren- correlacionar con estas diferencias (PIR07, 45).
cia exitosa a las normas son mucho más determinantes del • Activa.
mismo. El sujeto busca de forma activa, el entorno adecuado a su
- Motivación: contenido genético.
Las personas individualistas se orientan más a conseguir el

152
PÁG COLUMNA PÁRRAFO

116 1 Tabla 6

TEXTO DEL MANUAL


N: producción convergente. En este caso será el desarro-
llo de conclusiones lógicas a partir de la información dada
(PIR06, 60; PIR 19, 10).

PÁG COLUMNA PÁRRAFO

135 1 6

TEXTO DEL MANUAL

Como patrón general, la estabilidad de la inteligencia está


más demostrada que la estabilidad de la personalidad
(PIR08, 160; PIR 19, 12).
(Ver tabla 1 en la página siguiente)

153
el conjunto de características, pensamientos y actitudes que,
PSICOLOGÍA DIFERENCIALenDE LAcon
relación PERSONALIDAD
uno mismo, se activan y se hacen accesibles
AMPLIACIONES

AMPLIACIONES
en un momento determinado. El hecho de que estos esque-
LA IDENTIDAD PERSONAL
mas centrales, los cuales definen lo que cada uno cree que es
AUTOCONCEPTO Y AUTOESTIMA(AÑADIR DESPUÉS DEL
y cómo es, estén también presentes cuando se activan otros
APARTADO 14.1 COMO OTRO APARTADO, SERÍA EL 14.2)
tipos de contenidos en contextos y momentos determina-
Para entender la forma en que actuamos, nos relacionamos
dos, da apoyo a la idea del autoconcepto como un sistema
e interpretamos la realidad tenemos que entender el con-
de representaciones que se mantienen relativamente inva-
junto de creencias y percepciones que tenemos acerca de
riables ante los cambios situacionales; pero que comprende
quiénes somos y cómo somos. Nuestro autoconcepto puede
también un conjunto de descripciones más flexibles, abiertas
ser considerado como una estructura cognitiva compleja, re-
a ser modificadas o revisadas en función de las demandas
lativamente estable a lo largo de nuestra vida, pero también
que planteen las situaciones más inmediatas.
comprende un conjunto de contenidos mentales flexibles
Un tercer tipo de autoconcepto es el que hace referencia al
que varían en función de nuestras experiencias, metas, nece-
futuro, el cual es denominado por Markus como los posibles
sidades e interacciones. De esta visión más dinámica el auto-
yoes. Este tipo de conocimiento engloba el conjunto de es-
concepto se compondría de características pasadas que tuvo
peranzas, temores y deseos que resultan relevantes para el
una persona, las del momento presente y las que le gustaría
individuo; así como las diferentes cualidades que a la perso-
desarrollar en el futuro. Es preciso tener en cuenta también
na le gustaría tener o podría tener, y aquellas otras ante las
que las personas somos capaces de construir diferentes au-
que se sentiría mal en el caso de que las tuviera (PIR19, 8)
toconceptos, variando en función de los distintos contextos
Los posibles yoes actúan como incentivos de conductas fu-
en los que nos desenvolvamos, de los acontecimientos vita-
turas, en la medida en que constituyen partes de uno mismo
les que experimentemos, de las relaciones sociales que es-
que simbolizan aspiraciones y metas, aunque a veces, tam-
tablezcamos o de los roles sociales que desempeñemos. El
bién pueden reflejar los temores y las amenazas que tiene la
autoconcepto posee de este modo una naturaleza social y
persona. En este sentido los posibles yoes pueden funcionar
simbólica, y, por lo tanto, es capaz de influir en la selección y
como un puente cognitivo entre el yo presente (lo que yo soy
en el modo de interpretar la información que, referida a uno
aquí y ahora) y el yo futuro (lo que yo quiero o puedo ser),
mismo, se obtiene a partir de las relaciones sociales.
favoreciendo la construcción mental de escenarios y cursos
Ligada a la evaluación y el contenido del autoconcepto se
de acción apropiados.
encuentra la autoestima, que viene dada por la valoración
De este modo un individuo puede seleccionar conductas de
positiva o negativa y por el sentimiento de afecto y reconoci-
aproximación, para lograr que partes potenciales de sí mis-
miento hacia uno mismo, con importantes consecuencias en
mo, que representan cualidades positivas, se conviertan en
nuestra regulación, estado emocional y ajuste psicológico.
reales; o, por el contrario, puede actuar de forma que evite
que algunos de los posibles yoes que son temidos o rechaza-
14.4 LA MULTIPLICIDAD DE YOES (CON LA NUEVA NU-
dos se hagan realidad.
MERACIÓN SERÍA 14.5)

LA PROPUESTA DE MARKUS SOBRE EL “SELF” (AÑADIR


LA COMPLEJIDAD DEL AUTOCONCEPTO: EL MODELO DE
TEORÍA NUEVA, ANTES DEL MODELO DE LA AUTOCOMPLE-
LINVILLE (AÑADIR AL FINAL DEL PÁRRAFO DE ESTA TEORÍA)
JIDAD DE LINVILLE)
En el polo opuesto, Linville desarrolla una segunda hipóte-
Teniendo en cuenta el marco de referencia temporal de los
sis, denominada “hipótesis amortiguadora”, según la cual
contenidos del autoconcepto, Markus propone una posible
se espera que la complejidad del autoconcepto actúe como
diferenciación entre los diversos tipos de «yoes» o autocon-
un moderador en la relación del estrés con la salud física y
ceptos. Así, en la persona, puede coexistir un autoconcep-
psicológica de la persona. Es decir, que ante una situación
to ligado al pasado («cómo era yo cuando mis hijos eran
estresante, las personas con alta autocomplejidad sufrirán
pequeños»), otro referido al momento presente («cómo
menos las consecuencias negativas sobre su salud, ya que el
soy cuando estoy con mi familia»), y otro relativo al futuro
estrés actuará solo sobre unos pocos componentes de su au-
(«cómo sería mi parte más optimista»).
toconcepto, así el efecto sobre los pensamientos y las emo-
El autoconcepto relativo al pasado comprenderá el conoci-
ciones negativas será menor.
miento que se tiene acerca de cómo era la persona en un
tiempo pasado. Este tipo de autoconcepto se hace más so-
bresaliente cuando la persona ha cambiado algunos de los
LA ORGANIZACIÓN EVALUATIVA DEL AUTOCONCEPTO: EL
rasgos o atributos que le caracterizaban en otros momentos
MODELO DE SHOWERS(AÑADIR ESTA NUEVA TEORÍA DES-
de su vida, pero que ya no lo hacen. O cuando se desea cam-
PUÉS DEL MODELO DE LINVILLE)
biar algunas de las partes del autoconcepto actual que, son
similares a las mantenidas en el pasado, pero que generan
Este autor da importancia a la organización de las diferentes
desagrado o malestar en la persona.
categorías que componen el autoconcepto (aspectos refe-
El autoconcepto actual está referido al presente, y engloba
ridos a uno mismo y conectados a diversos contextos). Este

154
modelo comparte con Linville la visión de que el tipo de or- está más determinada por los elementos externos (normas
ganización del autoconcepto puede moderar el impacto del del grupo) que por los internos (rasgos o características de
estrés y su posible efecto negativo. Sin embargo, para él, a la personalidad) y enfatizan las metas colectivas. Sin embargo,
hora de configurar el autoconcepto, es necesario considerar las personas de culturas individualistas enfatizan el carácter
la valoración que hace la persona sobre sus descripciones único e independiente del individuo, se describen a sí mis-
(positiva o negativa). Así el autoconcepto puede organizarse mas mediante atributos personales, creen que la conducta
de forma evaluativa compartimentalizada (categorías sepa- social está más determinada por estos atributos personales
radas donde cada una engloba descripciones bien positivas que por las características externas y dan prioridad a sus me-
bien negativas) o de forma evaluativa integrada (cada des- tas personales sobre las del grupo.
cripción se podría valorar simultáneamente de forma positiva
y negativa, serían conectadas por la persona de forma que Tradicionalmente, enmarcamos dentro de las culturas indivi-
tengan sentido para ella (por ejemplo, “soy muy bromista, dualistas a las de Norte América, Australia y Europa y dentro
pero en los contextos requeridos soy capaz de mantenerme de las colectivistas al este de Asia (China, Japón, Corea), Áfri-
serio”). De este modo es posible considerar un continuo en ca, Latino-América y las islas del Pacífico.
la organización del autoconcepto, que comprendería desde
la compartimentalización hasta la integración evaluativa. Hay dos modelos teóricos en el estudio de la personalidad y
la cultura:
A partir de los modos de organización y del diferente nivel 1. La psicología cultural enfatiza que la personalidad
de accesibilidad que tengan las características descriptivas, no se puede separar del contexto social en el que se desarro-
el modelo de Showers predice una serie de relaciones con la lla y se expresa y, por tanto, se considera que la personalidad
autoestima y el bienestar psicológico e interpersonal. Cuan- se construye socialmente a través de las interacciones entre
do el autoconcepto es de tipo compartimentalizado, si los el individuo y su ambiente cultural.
compartimentos que se activan con mayor frecuencia son 2. La psicología transcultural que refiere que persona-
los positivos, el estado de ánimo del individuo será más po- lidad y cultura son entidades distintas, entendiendo que la
sitivo y su autoestima más alta, y viceversa ocurrirá con los cultura es la variable independiente que tiene influencia en
compartimentos negativos. En el caso de un autoconcep- la personalidad, entendida como variable dependiente.
to integrado, Showers prevé que a presencia conjunta de
creencias positivas y negativas tenderá a moderar el estado Podemos estudiar algunos constructos siguiendo a ambos
de ánimo, evitando que éste sea excesivamente positivo o modelos teóricos, comparando las culturas individualistas y
negativo y contribuyendo a mantener el nivel de autoestima. colectivistas:
En consecuencia, los mayores niveles de bienestar emocional
y autoestima se van a asociar con un autoconcepto con com- - El self:
partimentalización postiva; siendo estos niveles más mode- Se distingue el self independiente, propio de las culturas in-
rados en el caso de la organización integrada. Y mucho más dividualistas, del self interdependiente, más frecuente en las
bajos cuando el autoconcepto presente una compartimenta- sociedades colectivistas. Las personas individualistas se per-
lización negativa. ciben como independientes, autónomas y completas; acen-
túan los límites entre el yo y los otros, percibiéndose como
agentes separados que actúan para conseguir sus propias
CONTINUAR CON LA TEORÍA DE LA AUTODISCREPANCIA metas; su propio self actúa como fuente de acción y motiva-
DE HIGGINS ción; se describen con atributos internos y se experimentan
como inmutables a través de las situaciones (el mundo es más
14.8. INFLUENCIAS CULTURALES EN LA PERSONALIDAD mutable que el self). Por otra parte, las culturas colectivistas
(AÑADIR DESPUÉS DEL APARTADO EL SELF Y LA ME- se perciben como interdependientes de los demás, sienten
MORIA COMO UN APARTADO NUEVO) conexión con los miembros del grupo a los que pertenecen
y actúan en sintonía con las metas y los deseos de los otros
La cultura es un sistema de significados (creencias, valores, cercanos; son los otros la fuente de acción y motivación; se
normas, actitudes, conductas, conocimientos, habilidades, describen a sí mismas mucho más mediante relaciones y ro-
etc.) compartido por un grupo determinado, que habla una les que con atributos internos; conciben estos roles como
lengua común, en un período histórico específico y en una inmutables y al mundo social como duradero y permanente.
región geográfica concreta (Triandis y Suh, 2002). El sistema - Autoestima:
de significados cultural debe ser aprendido por cada gene- Se ha encontrado que las personas individualistas puntúan
ración mediante el proceso de socialización, a través de las más alto en las escalas de autoestimas que las colectivistas.
prácticas de crianza de los padres, las escuelas y otras insti- La autoestima es uno de los predictores del bienestar más
tuciones sociales. La dimensión principal de diferenciación potentes en las culturas individualistas pero no en las colec-
cultural estudiada ha sido la de individualismo-colectivismo tivistas, en las que la armonía en las relaciones y la adheren-
(refiriéndose al grado en que la persona está integrada en cia exitosa a las normas son mucho más determinantes del
el grupo), que define a las personas como idiocéntricas o mismo.
alocéntricas (PIR 19, 5). - Motivación:
Las personas individualistas se orientan más a conseguir el
Triandis (2001) diferencia entre culturas colectivistas e indi- éxito y las colectivistas a la evitación del fracaso. Esto pro-
vidualistas. Las personas dentro de las culturas colectivistas voca que en los individualistas la motivación se incremente
enfatizan la conexión con su grupo, considerándose inter- después del éxito mientras que en las colectivistas lo haga
dependientes del mismo, se describen más como miembros después del fracaso. Los individualistas presentan sesgos de
del grupo que como individuos, creen que la conducta social autoensalzamiento y los colectivistas presentan mayores ta-

155
sas de autocriticismo, ambas características asociadas con el
buen rendimiento.
- Emociones y bienestar:
Las personas de sociedades colectivistas presentan más emo-
ciones implicativas (simpatía, respeto, cercanía, amistad, cul-
pa, vergüenza…) que no implicativas (orgullo, autoconfian-
za, sentimiento de superioridad, ira, frustración…), mientras
que en las sociedades individualistas se presenta el patrón
inverso. En las sociedades colectivistas también destaca el
control de la expresión emocional (sobre todo de la negati-
va), ya que se considera que la libre expresión de las mismas
podría interferir en las relaciones y alterar la consecución de
relaciones armónicas con los demás.
- Cognición:
Con respecto al error fundamental de atribución (sobreesti-
mar las causas internas e infravalorar las situacionales en la
explicación de la conducta), se ha comprobado que no es un
sesgo universal como se creía y que es mucho más débil en
las culturas colectivistas. Los individuos colectivistas utilizan
en mayor medida causas de tipo situacional para explicar la
conducta. En los colectivistas la atención es más holística,
mientras que en los individualistas está más focalizada. En las
culturas individualistas predomina el pensamiento analítico.
- Rasgos:
La psicología cultural no pone en duda la existencia de los
rasgos en las diversas culturas, sino su relevancia. Se han he-
cho estudios transculturales sobre los cinco grandes factores
de personalidad, basándose en el uso del NEOPIR y también
replicando el procedimiento original, es decir, acudiendo
al diccionario para recoger todos los términos relativos a la
personalidad en una lengua determinada. Se puede concluir
que los cinco factores se replican en países de América del
Norte y el norte de Europa pero en otros países emergen di-
mensiones indígenas o específicas, mientras que no se obtie-
nen algunos de los cinco grandes y, además, las dimensiones
indígenas parecen mucho más predictivas que cualquiera de
las cinco dimensiones básicas.

156
157
PSICOTERAPIAS
RESEÑAS

conducta con la que ya contaba en su repertorio tras cons-


tatar que el modelo las realiza sin consecuencias negativas.
RESEÑAS Aplicaciones: fobias. También se le conoce como extinción
PÁG COLUMNA PÁRRAFO vicaria (PIR 19, 67).

110 2 11
PÁG COLUMNA PÁRRAFO
TEXTO DEL MANUAL
85 2 5
Desesperanza creativa: Intentar que la persona tome con-
ciencia de la inutilidad de sus intentos para solucionar sus
problemas y busque otras alternativas (PIR19, 57) TEXTO DEL MANUAL

Según la competencia del modelo:


- Modelado mastery/dominio
PÁG COLUMNA PÁRRAFO - Modelado coping/de afrontamiento: el modelo em-
pieza con un nivel similar al del observador y, poco a poco,
118 2 6 va mostrando las habilidades necesarias para resolver la si-
tuación. Se muestra ansioso al principio y relajado al final. Se
utiliza en problemas de ansiedad (miedos, fobias…) (PIR 19,
TEXTO DEL MANUAL 65).

Uno de los factores comunes a todas las psicoterapias, se-


gún Frank, es que exista un procedimiento que requiera la PÁG COLUMNA PÁRRAFO
participación activa tanto del terapeuta como del cliente (PIR
19, 61). 65 2 6

TEXTO DEL MANUAL


PÁG COLUMNA PÁRRAFO
Consiste en aprender a asociar determinadas frases con re-
113 2 3 presentaciones de sensaciones corporales. Así, se podrá in-
ducir mediante sugestión dichas sensaciones (PIR 19, 66).
TEXTO AMPLIADO SI LA FUENTE ES OTRA

El modo objeto conduce habitualmente al síndrome cogni- PÁG COLUMNA PÁRRAFO


tivo atencional caracterizado por un estilo de pensamiento
perseverativo (obsesiones o rumiaciones) e hipervigilancia 86 1 3
ante las amenazas. Esto mantiene los trastornos mentales.
Por ello, teniendo en cuenta dichas características, algunas
TEXTO AMPLIADO SI LA FUENTE ES OTRA
de las técnicas que incluye la TMC son el mindfulness des-
apegado, entrenamiento en atención y la reorientación de
Efecto desinhibitorio. El observador puede desinhibir una
la atención situacional (PIR 19, 63). Tanto la TMC (terapia
conducta con la que ya contaba en su repertorio tras cons-
metacognitiva) como la TC (terapia cognitiva) comparten la
tatar que el modelo las realiza sin consecuencias negativas.
importancia que le dan al procesamiento mental en los tras-
Aplicaciones: fobias. También se le conoce como extinción
tornos psicológicos. Sin embargo, la TMC se dirige más a los
vicaria (PIR 19, 67).
procesos que a los contenidos cognitivos. La TMC conserva
el uso de estrategias típicas de la TC (cuestionamiento so-
crático, experimentos cognitivos…) dirigidas a modificar me-
tacreencias con el trastorno mientras que la TC estándar lo PÁG COLUMNA PÁRRAFO
hace con las creencias ordinarias. La TMC focaliza en el estilo
del pensamiento y formula el síndrome cognitivo atencional. 72 1 2
Incluye estrategias de entrenamiento atencional y conciencia
plena “desapegada”.

Efecto desinhibitorio. El observador puede desinhibir una

158
a la inercia de los sistemas familiares que les hace resistirse al
PÁG COLUMNA PÁRRAFO
cambio (PIR 19, 41).
72 1 2
PÁG COLUMNA PÁRRAFO
TEXTO DEL MANUAL
115 2 11
Desensibilización por contacto, modelado participativo o
participante (PIR17, 157). Diseñado por Ritter. Se utiliza
principalmente con niños. Combina la DS con el modelado TEXTO AMPLIADO SI LA FUENTE ES OTRA
(PIR 98, 200; PIR 10, 133; PIR 18,195; PIR 19,68). Primero
se expone el terapeuta delante del sujeto, quien lo hace a 2. Contemplación.
continuación. El terapeuta le ayuda guía y refuerza. Gradual- Ya consciente del problema (problemas que le causa) y se
mente se retiran los apoyos. plantea cambiar en 6 meses, busca información, pero no lo
ha intentado ni se compromete en tratamiento pues se en-
cuentra aún ambivalente (PIR 13, 181; PIR 19, 55).
PÁG COLUMNA PÁRRAFO

72 2 5
PÁG COLUMNA PÁRRAFO

TEXTO DEL MANUAL 108 1 En terapia de


esquemas de
Escenificaciones emotivas. Es una variante de la anterior, Young. En objeti-
pero con presentación en vivo a través del juego, que genera vos de la terapia.
emociones positivas que inhiben la ansiedad. Se utiliza para Entre el tercer
las fobias infantiles e incluye como componentes la exposi- y cuarto punto
ción gradual en vivo, modelado participante y reforzamiento añadimos un
positivo (PIR 10, 137; PIR 11,103; PIR 17, 194; PIR 19, 71). punto más.

TEXTO AMPLIADO SI LA FUENTE ES OTRA


PÁG COLUMNA PÁRRAFO
Objetivos de la terapia
110 2 Penúltimo • Reconocer los esquemas, estilos de afrontamiento y modos
inadaptados.
• Establecer una apropiada y curativa relación paciente-te-
TEXTO AMPLIADO SI LA FUENTE ES OTRA rapeuta.
• Confrontación empática de los esquemas inadaptados y el
• Desesperanza creativa (PIR 19, 31): aprendizaje de nuevos estilos de afrontamiento.
Intentar que la persona tome conciencia de la inutilidad de • Promover cambios en los esquemas disfuncionales tempra-
sus intentos para solucionar sus problemas y busque otras nos (PIR 19, 51).
alternativas. • Desarrollar un modo adulto saludable del paciente logran-
do la remisión o el control de los modos disfuncionales.

PÁG COLUMNA PÁRRAFO PÁG COLUMNA PÁRRAFO

51 TABLA En el apartado de 109 2 1


la tabla: “Segun-
da Fase”. En el
guion: “Intensifi- TEXTO DEL MANUAL
cación”
Procedimiento

TEXTO AMPLIADO SI LA FUENTE ES OTRA


TEXTO AMPLIADO SI LA FUENTE ES OTRA
Una vez dentro del sistema familiar:
• Técnica del desafío. Se compone de tres fases: Procedimiento (PIR 19, 183)
- Escenificación de una pauta disfuncional.
- Focalización en la información importante.
• - Intensificación del mensaje del terapeuta. Normal-
mente es necesario repetir muchas veces el mensaje debido

159
PÁG COLUMNA PÁRRAFO

115 1 11
En terapia inter-
personal. En los
puntos de las
áreas problemá-
ticas.

TEXTO AMPLIADO SI LA FUENTE ES OTRA

Los síntomas se relacionan con problemas en una o más


áreas problemáticas (PIR 13, 120; PIR 19, 32):
• Duelo.
Pérdidas de personas significativas.
• Disputas interpersonales de rol.
• Transiciones de roles.
Renunciar al rol actual, expresar sentimientos de culpa, en-
fado o pérdida, adquirir habilidades y establecer nuevos vín-
culos (PIR 12, 142).
• Déficits interpersonales.

PÁG COLUMNA PÁRRAFO

115 1 9
En terapia inter-
personal, primer
párrafo

TEXTO AMPLIADO SI LA FUENTE ES OTRA

Terapia breve focalizada en el terreno de lo interpersonal,


basada en el modelo médico de enfermedad depresiva (PIR
19, 30). Ha demostrado ser eficaz para la depresión (PIR
00, 179) y “probablemente eficaz” en depresión adolescen-
te (PIR 11, 176) y, junto a la TCC, en Bulimia Nerviosa (PIR
14, 173).

160
AMPLIACIONES

161
tema 3
PSICOTERAPIAS
AMPLIACIONES

RECOMENDACIONES DE ESTUDIO
PÁG COLUMNA PÁRRAFO
PÁG COLUMNA PÁRRAFO
21 Tabla 4
En características
7 prefundacional

AMPLIACIÓN
MODIFICACIÓN
Se trata de una asignatura importante, con una contribución
al examen PIR de un 10% de las preguntas en las últimas • Distinción entre neurosis y bús-queda de métodos
convocatorias. para su tra-tamiento.
Los temas más importantes en cuanto a número de pregun- • Teoría del trauma.
tas en el examen PIR son, con diferencia, terapias con-duc-
tuales, terapias cognitivas y terapias conductuales de tercera
generación. Este último tema ha ido consolidando su impor- TEMA 6
tancia a lo largo de las últimas convocatorias incrementán-
PÁG COLUMNA PÁRRAFO
dose el número de preguntas hasta igualar a terapias cog-
nitivas y haciéndose las preguntas mucho más concretas y
específicas. De estos tres temas es funda-mental conocer los 44 2 Añadir un punto
distintos conceptos técnicos en profundidad y saber abstraer entre el primero y
estos contenidos para poder apli-carlos a ejemplos prácticos segundo punto
sobre las distintas técnicas de intervención. También habrá
que conocer los autores de los distintos modelos y las técni-
cas que desarrollan. AMPLIACIÓN
De los cinco primeros temas es suficiente saber asociar a
cada autor con su teoría o modelo psicoterapéutico, definir • Ni las personas ni sus proble-mas existen en un va-
conceptos y relacionar las técnicas con modelos y autores. cío, ambos están ligados a sistemas recíprocos más amplios,
Del tema de terapia familiar hay que saber aso-ciar autores siendo la fa-milia el principal. El origen de la sintomatología
y conceptos principales con cada una de las escuelas de te- suele situarse en las dificultades y crisis a las que se ven so-
rapia familiar sistémica y profundizar, más que en los cinco metidos los siste-mas familiares en su devenir vi-tal. Hay que
primeros temas, en los elementos técnicos de cada de las tener en cuenta el funcionamiento familiar en su conjunto y
intervenciones propias de este modelo. no sólo el del paciente identificado.
Es importante que haya una adecuada proporción del tiem-
po dedicado al estudio de cada uno de los modelos de psico-
terapia, debiendo dedicar un setenta por ciento del tiempo
de estudio a los temas de terapia de conducta, terapia cog- PÁG COLUMNA PÁRRAFO
nitiva y terapias conductuales de tercera generación.
45 2 En conceptos bá-
sicos (párrafo 5).
TEMA 2 Añadir lo siguien-
PÁG COLUMNA PÁRRAFO te como primer
punto
17 Tabla 3
AMPLIACIÓN
MODIFICACIÓN
• Sistema: Conjunto de elemen-tos dinámicamente
Integra conceptos de distintas teorías. Villegas (1990) lo lla- estructurados cuya totalidad genera unas pro-piedades que,
ma integracionismo en parte, son in-dependientes de las que poseen sus elemen-
y distingue entre: tos por separado.
• Integración asimilativa: reformula conceptos de una teoría
en términos de otra.
• Integración acomodativa: Articula elementos teóricos com-
patibles en una teoría nueva.

162
PÁG COLUMNA PÁRRAFO PÁG COLUMNA PÁRRAFO

45 2 Antepenúltimo 46 TABLA Derecha de limi-


tación/proceso es-
tocástico
AMPLIACIÓN
• Fronteras. AMPLIACIÓN
Límites que separan a la familia nu-clear del exterior (familia
extensa, comunidad...). Marcan las interaccio-nes de la fami- Cuando el sistema adopta una determinada se-cuencia de
lia o de sus miembros con agentes externos a ella. interacción, disminuye la probabilidad de que emita otra res-
puesta diferente. La secuencia se reitera en el tiempo.

PÁG COLUMNA PÁRRAFO


PÁG COLUMNA PÁRRAFO
45 2 Penúltimo
46 1 5

AMPLIACIÓN
AMPLIACIÓN
• Sistema abierto.
Aquél que intercambia información, materia o energía con - Nivel de relación o analógi-co.
el exterior y tiene capacidad para adaptarse al me-dio, por Todo mensaje contiene aspectos implí-citos que definen la
lo que su estado final puede ser distinto de las condiciones relación. Este nivel califica o rotula al de contenido. La co-
iniciales. municación es eficaz si podemos comunicar sobre la relación
(“meta-comunicar”).

PÁG COLUMNA PÁRRAFO PÁG COLUMNA PÁRRAFO

46 TABLA Derecha de cau- 46 2 2


salidad circular

AMPLIACIÓN AMPLIACIÓN

4. La definición de una interacción está condicionada por la


Las conductas de los miembros se codeterminan recíproca- puntuación de la secuencia de hechos que hacen los parti-
mente, de forma redundante (secuencia de conductas). Esto cipantes.
no es patológico en si mismo, no obstante, cuando habla- La puntuación es la forma de organizar y narrar la secuencia
mos de se-cuencia sintomática, hacemos referencia a con- de hechos en una interacción (PIR 17, 166). En realidad,
ductas articuladas alrededor de un síntoma y reguladas por se trata de una secuencia infinita, por lo que tanto el inicio
una causalidad circular. como el final es artificial, o bien, pactado en la relación. Se
trata de un planteamiento basado en la circularidad y, por
tanto, imposible de ajustar a modelos de explicación lineal
causa-efecto. La discrepancia en la puntuación es frecuente,
PÁG COLUMNA PÁRRAFO
de hecho, los miembros de una relación pueden puntuar la
secuencia de maneras distintas, lo cual genera numerosos
46 TABLA Derecha de equi- conflictos (p. ej., una pareja que discute porque la mujer
causalidad considera justo su enfado como reacción a una acción de
él, pero el marido plantea que esa acción fue a su vez una
reacción de una acción de ella previa).
AMPLIACIÓN Clínicamente, los conflictos de puntuación suelen partir de
dos supuestos erróneos:
Una misma condición inicial puede dar lugar a estados fina- - Suponer que el otro cuenta con la misma información que
les distintos, razón por la que no se puede conocer el origen uno y que debe sacar las mimas conclusiones.
único de un problema. - Suponer que solo hay una realidad posible (“tal y como yo
lo veo”) (PIR 17, 166).

163
2.El doble vínculo es recurrente, convirtiéndose en la expec-
PÁG COLUMNA PÁRRAFO
tativa habitual.
47 ELIMINAR TODO
3.Un mandato negativo primario, la alusión a un castigo
APARTADO DE
como la retirada de afecto o la expresión de odio.
VARIABLES DEL
FUNCIONAMIEN-
4.Un mandato negativo secundario comunicado a un nivel
TO FAMILIAR
más abstracto, generalmente no verbal, que entra en con-
flicto con el mandato primario.

5.Un mandato negativo terciario, el individuo no puede es-


capar de la situación, pues no puede metacomunicar acerca
de la incongruencia de los mensajes.
PÁG COLUMNA PÁRRAFO

47 AÑADIR APARTA-
DO DONDE ESTA-
PÁG COLUMNA PÁRRAFO
BA EL ANTERIOR
ELIMINADO
48 1 Todo el último
AMPLIACIÓN
AMPLIACIÓN
Patrones de comunicación disfun-cional
• Doble vínculo. El curso vital de las familias evoluciona a través de una se-
Contradicción entre la información comunicada a nivel ana- cuencia de etapas bastante universal, por lo que se denomina
lógica y a nivel digital, en una relación significativa. Relacio- «normativo», a pesar de las diferencias culturales (Cárter y
nada con la etiología de la es-quizofrenia (Bateson). McGoldrick, 1989). A pesar de variaciones idiosincrásicas en
• Escalada simétrica. cuanto al momento en que tienen lugar los cambios de una
Relación simétrica en la que se hace un esfuerzo excesivo por etapa a otra y a las estrategias empleadas para afrontarlos,
lograr igual-dad y reducir las diferencias en tal grado que se el desarrollo familiar sigue una misma progresión de comple-
considera patológico. Existen luchas constantes intentando jidad creciente. Existen períodos de equilibrio y adaptación y
ser “más igual que el otro”. períodos de desequilibrio y cambio. Los primeros se caracte-
• Complementariedad rígida. Cuando en una relación rizan por el dominio de las tareas y aptitudes pertinentes a la
complementa-ria, los participantes se adhieren a sus posicio- etapa del ciclo que atraviesa el grupo familiar, mientras los
nes de forma rígida. Es decir, hay un miembro que rígida- segundos implican el paso a un estadio nuevo y más com-
mente ocu-pa una posición jerárquica superior en la relación, plejo, y requieren que se elaboren tareas y aptitudes nuevas.
y otro la inferior. Algunas de las etapas marcadas son (McGoldrick y Cáeter,
1980): adulto joven independiente, formación de la pareja,
familia con hijos pequeños, familia con hijos adolescentes,
el despegue de los hijos y la familia en la vejez. Cada familia
responde de manera diferente a estas crisis o momentos/
PÁG COLUMNA PÁRRAFO etapas, y numerosas investigaciones muestran cómo los sín-
tomas psiquiátricos frecuentemente se agrupan en torno a
48 1 4 estos momentos de crisis familiar.

AMPLIACIÓN
PÁG COLUMNA PÁRRAFO
Teoría propuesta por el grupo de Palo Alto, liderada por Ba-
teson, para estudiar y explicar la comunicación de familias en 48 2 3
las que existía un miembro diagnosticado de esquizofrenia.
Se trata de un contexto de habituales callejones sin salida en AMPLIACIÓN
la comunicación impuestos unos a otros por personas que se
encuentran dentro de un sistema de relación. Comunicación Objetivo: modificar los parámetros relacionales que provo-
a muchos niveles en que una demanda manifiesta en un ni- can la conducta disfuncional o el estancamiento relacional
vel era a la vez anulada o contradicha en otro nivel. Se trata del sistema familiar (PIR 93, 169). El estancamiento es la
de una especie de paradoja a dos niveles: uno explícito y otro imposibilidad de generar nuevas modalidades de relación,
implícita. El doble vínculo comprende un componente cogni- por diferentes motivos, internos y externos a la psicopatolo-
tivo que impide una discriminación adecuada de la situación, gía: dificultades de estructuración familiar, de desarrollo in-
y un componente afectivo referido a la hostilidad hacia las dividual de sus miembros o de incapacidad del sistema para
personas que ejecutan el doble vínculo. Los requisitos bási- integrar los cambios propios del ciclo vital (PIR 93, 170; PIR
cos para que se produzca un doble vínculo son: 95, 116).
1.El sujeto está involucrado en una relación intensa y per-
durable.

164
patoria. El objetivo de la intervención paradójica sería reducir
PÁG COLUMNA PÁRRAFO
alguna conducta no deseada por medio de prescripciones
paradójicas, como prescribir esa conducta. En el caso de las
48 2 5
ideas o conductas autolesivas no sería moral, ético o profe-
sional prescribir esas conductas a un paciente, por eso no
son adecuadas para trabajar con esta técnica. (PIR 17, 116).
AMPLIACIÓN
• Redefinición. PÁG COLUMNA PÁRRAFO
Desafiar la definición del síntoma que hace la familia. Se
transforma el lenguaje para definirlo como conductas con- 49 1 4
trolables y concretas. Cuestiona visiones demasiado rígidas
del problema y las sustituye por otras más benévolas. Por
ejemplo, transformando conductas demasiado controlado- AMPLIACIÓN
ras en preocupación por los demás, sobreimplicación en ex-
ceso de amor… • Ordalía.
Consiste en imponer una orden al paciente que sea más se-
vera que el problema que le aqueja. Muchos pacientes aban-
donan el síntoma antes de realiza la ordalía. Se pretende
PÁG COLUMNA PÁRRAFO
extinguir un comportamiento inadaptado, introduciendo a
continuación de la conducta problema (de manera contin-
48 2 6 gente) una actividad que resulte más molesta que el compor-
tamiento problemático. Por ejemplo, en un caso de bulimia
AMPLIACIÓN con atracones, se acuerda con el paciente que, cada vez que
se produzca un atracón, deberá́ levantarse esa misma noche
• Connotación positiva. a las 3 de la madrugada para hacer ejercicios abdominales
Atribuir un significado positivo al síntoma y al contexto don- durante treinta minutos. Para utilizar esta técnica debe exis-
de cumple una función (PIR 93, 52). Incide en la funcionali- tir una estrecha alianza terapéutica y estar el paciente muy
dad del síntoma en ese sistema, subrayando su intencionali- motivado al cambio.
dad positiva. Por ejemplo, un niño de padres separados tiene
rabietas desde entonces lo cual hace que su padre vuelva a
casa y los dos progenitores se encarguen de esta situación, PÁG COLUMNA PÁRRAFO
podemos connotar positivamente su conducta como una
forma de mostrar amor y deseo de unión de su familia. 49 1 5

PÁG COLUMNA PÁRRAFO AMPLIACIÓN

48 2 Penúltimo • Uso de analogías (proviene de Erickson).


Se utiliza una historia similar metafóricamente al problema
del paciente para abordar la resistencia.
AMPLIACIÓN

• Intervención paradójica.
Consiste en la prescripción controlada y específica de aque-
llo mismo que constituye el problema (PIR 14, 144). Los PÁG COLUMNA PÁRRAFO
orígenes de esta técnica se remontan a Adler y Frankl. Se
usa la paradoja terapéutica para responder a la paradoja del 49 2 1
paciente (“ayúdeme a cambiar, pero no cambie nada”). Se añadir al párrafo
emplean cuando la situación es de orden y secuencialidad lo siguiente:
rígidas, como forma de introducir confusión y flexibilidad
en el sistema de creencias. El procedimiento clásico, ahora
prácticamente abandonado, consiste en: connotación positi-
va + redefinición + prescripción de la secuencia sintomática AMPLIACIÓN
+ un límite temporal arbitrario durante el cual la secuencia
sintomática debe seguir vigente. La prescripción de tareas La máxima de esta escuela es, por lo tanto: “La solución es
indirectas, y dentro de estas de la intención paradójica, está el problema”.
indicada para pacientes que no cumplen con una prescrip-
ción terapéutica directa, para pacientes resistentes al cambio
u oposicionistas. La eficacia de técnica se basa en el principio
fundamental de que los pacientes intenten llevar a cabo la
conducta que están evitando. De esta manera, el proceso cir-
cular mantenedor se rompería reduciendo la ansiedad antici-

165
to, sin dejarle acabar con éxito. Por ejemplo, en un paciente
PÁG COLUMNA PÁRRAFO
con dificultad para relacionarse con el sexo opuesto, se le
pide que intente ser rechazado en su próximo encuentro.
49 2 1 • Llegar a un acuerdo mediante coacción (sabotaje
Añadir nuevo benévolo). Comúnmente utilizado en padres de adolescen-
párrafo tes conflictivos que insisten en el deber de ser obedecidos
respetuosamente. La técnica consiste en que los padres
adopten una actitud sumisa, para que desaparezca la acti-
tud rebelde de los hijos. Al pedir algo los padres utilizarán la
AMPLIACIÓN fórmula “me gustaría que…” y si el adolescente desobedece
utilizarán consecuencias negativas reales e imprevisibles, por
Se interesan por la forma en la que la familia mantiene los ejemplo, cerrar la puerta por dentro cuando el adolescente
problemas a través de sus intentos de solución, que, si bien llegue más tarde de lo acordado, mostrándose desconcerta-
son bienintencionados, resultan desafortunados. Ante el fra- dos por lo ocurrido y disculpándose.
caso de la solución, la familia aplica una mayor dosis de esta, • Conseguir sumisión a través de la libre aceptación.
agravando y manteniendo el problema. Habitual en relaciones padre hijo y en relaciones de pareja.
Se utiliza en sujetos que piensan que si piden algo directa-
mente su petición quedará invalidada por no haber conse-
PÁG COLUMNA PÁRRAFO guido el cambio en el otro de forma espontánea. La inter-
vención se fundamenta en la necesidad de hacer peticiones
directas, valorándolas como algo beneficioso, aunque difícil
49 2 2
de poner en práctica.
• Confirmar las sospechas del acusador mediante la
AMPLIACIÓN autodefensa. Se utiliza cuando hay un conflicto entre dos
personas en el cual “A” sospecha que “B” realiza un acto
Apuestan por una terapia breve, de no más de 10 sesio- que ambos consideran inadecuado. “A” vigila y acusa a “B”,
nes semanales. Se pretende buscar la función de problema y quién lo niega y se justifica, confirmando las sospechas de
neutralizar la “solución”, para cortar el feedback y modificar “A”. A través de la técnica de la interferencia o confusión
la estructura del sistema. Suponen que, alterando la solución de señales, se rompe la cadena de acusaciones y justificacio-
intentada, romperán la secuencia sintomática produciendo nes. Por ejemplo, se le indica a un marido que actúe como
el cambio. Distinguen dos tipos de cambio: si estuviera borracho cuando no lo esté y viceversa, y se le
• “Cambios-1”. pide a la esposa que intente averiguar cuándo ha bebido y
Soluciones aplicadas por la familia, se rigen por el sentido cuándo no, pero sin decir nada. Ambos realizan un registro
común y suelen ser hacer lo contrario al síntoma. No modifi- y los llevan a la siguiente sesión.
can la estructura del sistema y contribuyen al mantenimiento
del síntoma.
• “Cambios-2”. PÁG COLUMNA PÁRRAFO
Modifican los parámetros del sistema haciendo que la sinto-
matología se reduca o desaparezca. Utilizan redefiniciones, 49 2 10
técnicas paradójicas y directas, metáforas, ordalías (pacto
con el diablo) e incluso técnicas hipnóticas.
AMPLIACIÓN
PÁG COLUMNA PÁRRAFO Intervenciones generales:
Son intervenciones de tipo estratégico a utilizar cuando las
49 2 5 específicas no han dado resultado o cuando se quieren com-
plementar.
AMPLIACIÓN • No apresurarse o petición de cambio lento.
En pacientes con hiperintención en la solución de sus sínto-
Intervenciones específicas: mas (lo cual constituye el problema), pacientes pasivos que
Estas intervenciones están basadas en una tipología de solu- urgen al profesional a llevar a cabo acciones curativas y siem-
ciones intentadas por las familias, estas soluciones intenta- pre que haya mejorías, recordando que los cambios lentos
das dan nombre a las técnicas. son más sólidos. Con esto prevenimos recaídas y ayudamos
• Forzar algo que sólo puede ocurrir espontáneamen- a valorar los pequeños cambios.
te. Para insomnio, tics, dolores de cabeza, rendimiento se- • Peligros de una mejoría.
xual, etc. El paciente define como problemáticas fluctuacio- Útil cuando el individuo no ha realizado una prescripción y
nes en el funcionamiento normal. Consiste en provocar el en casos de ansiedad. En el primer caso se puede acelerar el
síntoma intencionadamente, para observarlo sin la angustia cambio, y en el segundo, se reduce la obligación de exigirse
de luchar contra él. rendir en exceso, modificando ese intento de solución.
• Dominar un acontecimiento temido aplazándolo. • Cambio de dirección.
Para pacientes con impulso de dominar completamente una Cambiar de estrategia cuando la actual no está dando resul-
tarea antes de enfrentarse a ella, para no fracasar, lo cual tados, argumentando el haberse dado cuenta de que se ha
hace que nunca se enfrenten a ella. seguido el camino inadecuado o que se ha consultado a un
Se expone al paciente a tarea temida, con dominio incomple- experto.

166
• Cómo empeorar el problema. positiva del síntoma, aprendizaje de complementariedad y
Cuando hacia el final de la terapia sigue sin haber cambios. fijación/reestructuración.
Se manifiesta no saber como ayudarles, pero si saber como
empeorar el problema. Sin culpabilizar y educadamente, se
enumera todo lo que paciente y familia han hecho para in-
tentar solucionar el problema. Esto subraya su papel activo PÁG COLUMNA PÁRRAFO
en persistencia del síntoma.
50 1 1

PÁG COLUMNA PÁRRAFO AMPLIACIÓN


49 2 Penúltimo Los síntomas aparecen cuando hay un retraso o detención
en el ciclo vital familiar. Debido a que la familia, por su es-
tructura, no puede manejar-resolver bien la tensión gene-
rada en los cambios de etapa (p. ej., la marcha de un hijo).
AMPLIACIÓN

Minuchin defiende que los procesos del sistema familiar se PÁG COLUMNA PÁRRAFO
reflejan en sus estructuras. La estructura se compone de
una jerarquía, los límites entre subsistemas y fronteras con
50 1 2
el exterior, así como las reglas que rigen el poder y la co-
municación. Además, existen alianzas (relaciones de especial
cercanía entre miembros que no implican conflicto con un
tercero) y coaliciones (unión de dos personas contra otra per- AMPLIACIÓN
sona de la familia).
Los límites entre subsistemas pueden ser difusos, rígidos o
claros.
PÁG COLUMNA PÁRRAFO • Límites rígidos: Poca comunicación e intercambio
entre subsistemas. Permiten mucha a autonomía a costa de
una baja conexión emocional. Permiten el crecimiento y de-
50 Tabla en técnicas
sarrollo de competencias personales, pero se reduce el afec-
principales MRI
to, apoyo y calidez.
palo alto (primera
• Límites difusos: Excesivo intercambio comunicacio-
fila)
nal entre subsistemas. Gran pertenencia y apoyo, con baja
autonomía individual. Pueden tener problemas para relacio-
AMPLIACIÓN narse fuera de la familia.
• Límites claros: Existe comunicación e intercambio
• Redefiniciones. entre subsistemas, manteniendo la diferenciación y la auto-
• Técnicas paradójicas y directas. nomía.
• Metáforas. Las disfunciones estructurales ocurren cuando los límites son
• Ordalías y pacto con el diablo. Esta última al final de difusos o rígidos.
las sesiones si hay buena alianza y pocos cambios. Decimos
que sabemos como solucionar el problema pero que solo lo
revelamos si van a cumplir la prescripción, de lo contrario PÁG COLUMNA PÁRRAFO
están fuera de la terapia.
• Técnicas hipnóticas Eriksonianas. 50 2 3
• Intervenciones específicas y generales.
AMPLIACIÓN

Las relaciones familiares se consideran patológicas cuando


PÁG COLUMNA PÁRRAFO existe un patrón de desviación del conflicto (dos padres ma-
nifiestan ausencia de conflicto entre ellos y se unen contra
50 Tabla en técnicas el hijo en forma de “chivo expiatorio”) y cuando se crean
principales de es- coaliciones intergeneracionales permanentes (por ejemplo,
cuela estructural dos progenitores en conflicto que intentan ganar el apoyo
(segunda fila) de un hijo en contra del otro progenitor).

El síntoma se mantiene por la tendencia homeostática de la


AMPLIACIÓN familia, no dejando espacio al crecimiento y cambio.

• Técnica del desafío.


• Técnicas de reestructuración: desequilibrio, redefinición

167
para desequilibrar el sistema y provocar crisis (PIR 02, 217).
PÁG COLUMNA PÁRRAFO

51 1 1
PÁG COLUMNA PÁRRAFO
AMPLIACIÓN
51 1 TABLA: AÑADIR
UNA TERCERA FILA
El objetivo terapéutico es cambiar la organización familiar, CON EL TÍTULO DE:
los límites entre subsistemas y las jerarquías y adecuar las OTRAS TÉCNICAS
pautas transaccionales a las necesidades (PIR 93, 53). Busca
activar un cambio en la estructura familiar.

AMPLIACIÓN
PÁG COLUMNA PÁRRAFO • Técnicas de cambio de visión. Afectan a la percep-
ción de los miembros con respecto al síntoma:
51 1 TABLA - Modificación de constructos cognitivos.
DERECHA DE - Intervenciones paradójicas: Constan de tres fases:
SEGUNDA FASE Redefinición, prescripción de la secuencia sintomática y res-
(PARTE 1). tricción (se pide el no cambio).
- Facetas fuertes: Enfatizar las fortalezas familiares
para que las apliquen en la resolución del problema.

AMPLIACIÓN
PÁG COLUMNA PÁRRAFO
Una vez dentro del sistema familiar:
• Técnica del desafío. Se compone de tres fases:
51 1 5
1. Escenificación de una pauta disfuncional. Aquí se
logra alianza terapéutica, se define al sistema como disfun-
cional y nos permite distanciarnos y observar. Tras observar
las interacciones espontáneas, decidimos sobre cual interve- AMPLIACIÓN
nir pidiendo que escenifiquen una secuencia. Se interviene
modificando la intensidad y/o duración y luego se proponen Madanes sostiene que los síntomas son actos comunicativos
interacciones alternativas. analógicos, con una función dentro del sistema: un intento
2. Focalización en la información importante. de solución, insatisfactoria, a los problemas familiares. El sín-
3. Intensificación del mensaje del terapeuta. Normal- toma muestra confusión en la jerarquía del sistema y luchas
mente es necesario repetir muchas veces el mensaje debido de poder. Haley (1987) manifiesta que la conducta sintomá-
a la inercia de los sistemas familiares que les hace resistirse al tica es señal de que el ordenamiento jerárquico es confuso y
cambio (PIR 19, 41). de que existe una lucha de poder para aclarar las posiciones
en la jerarquía. Haley plantea que la organización familiar es
patológica si contiene triángulos perversos, coalición que se
PÁG COLUMNA PÁRRAFO establece entre personas de distintos niveles jerárquicos. Se
le llama también coalición intergeneracional. Las característi-
51 1 TABLA cas de este triángulo según Haley son (PIR 17, 162):
DERECHA DE
SEGUNDA FASE
(PARTE 2).
PÁG COLUMNA PÁRRAFO

AMPLIACIÓN 51 1 Añadir un párrafo


nuevo entre los
• Técnicas de reestructuración: dos últimos
- Redefinición positiva del síntoma: relectura relacional alter-
nativa para cuestionar las definiciones de la familia. AMPLIACIÓN
- Aprendizaje de complementariedad: Cuestiona la visión fa-
miliar del problema y la causalidad lineal del síntoma, subra- Otro concepto importante desde la terapia estratégica es
yando la contribución global de la familia. el de jerarquía incongruente: Definición simultánea de dos
- Fijación/Reestructuración de límites. Se utiliza para regular posiciones incompatibles de poder dentro del sistema. Por
la permeabilidad entre subsistemas. Prescripción de tareas ejemplo, un hijo que mediante su coalición con uno de los
conjuntas por parte de varios miembros para formar nuevas padres traspasa los límites intergeneracionales, ocupando a
alianzas. la vez una posición inferior a sus progenitores por pertenecer
- Desequilibrio: El objetivo es modificar la jerarquía familiar. al subsistema filial y una posición superior, fruto de su coa-
El terapeuta se alia o coaliga con unos de forma temporal lición con el padre «aliado», minando la autoridad del otro.

168
del sistema.
PÁG COLUMNA PÁRRAFO
2.Romper coalición madre – hijo con una tarea. Tres alterna-
tivas: Ser más firme y exigente, disminuir niveles de involu-
51 2 Añadir nuevo
cración o intervenciones paradójicas.
párrafo tras (PIR
3.Unir a los progenitores para que ayuden al hijo a eliminar
94, 240).
el síntoma.

AMPLIACIÓN •Cuando el paciente identificado es un niño (Madanes,


1984):
El objetivo es cambiar las secuencias de conducta inadecua- 1.El padre pide al hijo que tenga el síntoma. Cuando el sín-
das que encierran una jerarquía incongruente y ampliar las toma parece proteger a ese progenitor. Así asume una posi-
alternativas conductuales y epistemológicas del sistema. ción de poder bloqueando la ayuda del niño.
2.El padre pide al hijo que simule o imagine tener el síntoma.
3.El padre pide al hijo que simule ayudarle. Así se hace explí-
cita la ayuda al progenitor que era implícita con el síntoma.
PÁG COLUMNA PÁRRAFO De este modo se resistirán a esta incongruencia jerárquica.

51 2 5

PÁG COLUMNA PÁRRAFO


AMPLIACIÓN
52 1 5
• Redefinición.
Presentar el síntoma en un marco conceptual distinto al de
la familia, considerándolo resoluble y parte del sistema (por AMPLIACIÓN
ejemplo, hablar de “irresponsabilidad” en lugar de “depre-
sión”). Según este enfoque, las familias se resisten al tratamiento,
presentando un mensaje paradójico al que el terapeuta no
puede responder: “cámbienos, pero no nos cambie”. Para
anularlo desarrollan la intervención contraparadójica: doble
PÁG COLUMNA PÁRRAFO vínculo terapéutico. Se delega a la familia la responsabilidad
del cambio. La terapia se dirige a desmontar el juego rela-
51 2 7 cional.

AMPLIACIÓN PÁG COLUMNA PÁRRAFO


• Intervenciones conductuales. Tareas que la familia
52 1 Añadir este antes
debe hacer dentro y/o fuera de sesión.
del último párra-
- Directas.
fo de la columna.
Permiten valorar la capacidad de cambio del sistema.
Antes de: Existen
- Paradójicas.
dos tipos princi-
Prescribir el síntoma para que pierda su funcionalidad.
pales de juegos
Madanes utiliza la “simulación” o “imaginación” del sínto-
psicóticos:
ma o conducta problema.

AMPLIACIÓN
PÁG COLUMNA PÁRRAFO
Juego familiar: Relaciones de los miembros del sistema fami-
51 2 8 liar, las creencias que tienen unos de otros, de sí mismos y
de la familia en conjunto. Se vuelve patológico cuando deja
de ser adaptativo para el buen funcionamiento del sistema,
AMPLIACIÓN asignando a roles y funciones que se separan del momento
evolutivo de los miembros.
•Cuando el problema incluye dos generaciones (Haley,
1980). Tres estrategias:

1.Usar persona periférica. En el caso de jerarquía incon-


gruente utilizamos a la persona desplazada. Por ejemplo, en
coalición madre hijo, decimos al padre que se encargue del
hijo. Se desplaza a la madre, se trabaja para unir a los proge-
nitores y al hijo con sus iguales y luego el terapeuta “sale”

169
PÁG COLUMNA PÁRRAFO prescribirse como gesto simbólico, experimento, rito transi-
cional… (por ejemplo, prescribir un ritual funerario).
• Tarea ritualizada de los días pares/días impares.
52 2 1
Prescribir dos conductas contradictorias en días alternos. Se
utiliza en escalada simétrica entre los padres con estilos de
AMPLIACIÓN control de síntomas verbalizados como irreconciliables. Se
entrena realización de conductas alternativas y en división de
Centro de interés. El hijo sabe que forma parte de una re- funciones, controlando la competitividad.
lación ilícita, y cuando se da cuenta de que lo han estado • La prescripción invariable o universal (Palazzoli y Pra-
utilizando, se siente traicionado y condenado al silencio, por ta).
lo ilícito de esa relación. El síntoma del hijo es una forma de Parte de la idea de que todas las familias psicóticas presentan
reivindicar de forma encubierta la situación, vengándose. problema de diferenciación entre subsistema parental y fi-
lial. Se separa a los subsistemas de forma gradual para evitar
coaliciones y clarificar límites.

PÁG COLUMNA PÁRRAFO


PÁG COLUMNA PÁRRAFO
52 1 FIGURA 1.
Hueco 3 52 2 4

AMPLIACIÓN
AMPLIACIÓN
La conducta inusitada del hijo: el hijo toma partido secreta-
mente por el provocador pasivo y se esfuerza para desequi- Las directrices que guían las entrevistas son hipotetización
librar el conflicto a su favor. Se comporta de forma “rara”, (guía la recogida de información y diseño de intervención),
insultando o no respondiendo al aparente “vencedor”, así, circularidad (capacidad del terapeuta par confirmar o falsar
desafía a éste y le enseña al “perdedor” como rebelarse. hipótesis) y neutralidad.

PÁG COLUMNA PÁRRAFO PÁG COLUMNA PÁRRAFO

52 2 3 52 2 7

AMPLIACIÓN AMPLIACIÓN
El Grupo de Milán aboga por la no directividad y utiliza es- 1. Lineales.
pejo unidireccional para asegurar la neutralidad. El objetivo Preguntas directas. Como la familia considera que las causas
es cambiar la rigidez del sistema de creencias de la familia del síntoma son lineales, estas preguntas buscan conocer la
para que luego ellos encuentren el cambio, es la familia definición que da la familia. “¿Cuál es la problemática”?
quien debe cambiar. Este cambio ocurre cuando se cambia
el juego familiar por otro menos perjudicial; es más profun-
do si se produce en las creencias de la familia (PIR 13, 127). PÁG COLUMNA PÁRRAFO

52 2 8

PÁG COLUMNA PÁRRAFO


AMPLIACIÓN
53 1 11
2. Circulares.
Para investigar las definiciones de relación de los miembros
AMPLIACIÓN y poner de manifiesto la relación entre eventos (PIR 16,
152). Sirven para confirmar o falsar hipótesis sobre el juego
- Prescripciones ritualizadas. familiar. Tipos de preguntas circulares:
Introducir una secuencia estructurada de conductas que in- • - Tríadicas.
troduce reglas novedosas, sin dar explicaciones. Se usan para Se pregunta a un tercero sobre la relación entre otros dos.
introducir orden cuando la situación es confusa. Varios tipos: “¿Cómo ves la relación entre tu marido y tu madre?”
• Rituales. • - Sobre alianzas.
Secuencia de conductas en la que se establece qué ha de ha- Se pregunta sobre la cercanía entre los miembros. “¿Con
cer cada miembro de la familia, dónde y cuándo. Contribuye quién pasas más tiempo con tu hermano o con tu herma-
a clarificar contradicciones e interacciones caóticas. Puede na?”

170
• - Para clasificar. que el terapeuta considera beneficiosos (redefiniciones, ac-
Para trazar un mapa de relaciones o establecer un orden ciones alternativas, voluntad, disculpa y perdón).
sobre algo concreto. “¿Manuel empezó a comportarse de - Preguntas de comparación normativa. Finalidad de orientar
esta forma antes que te tuvieras dificultades con tu pareja a la familia hacia patrones más normativos o “sanos” com-
o después?” parándose con normas sociales, evolutivas y dificultades o
• - Explicativas. comportamientos normalizados.
Explicar por qué ocurrió algo, buscando información sobre - Preguntas que clarifican distinciones. Se utilizan para clarifi-
creencias y secretos. “¿Porqué crees que tu hermano se car atribuciones causales que hacen los familiares.
comportó de forma tan contundente?”
• - Hipotéticas.
Tantear la posible reacción de la familia ante una acción. PÁG COLUMNA PÁRRAFO
Establece un nuevo mapa de relaciones y la posibilidad de al-
ternativas de significado y acción. “¿Qué pasaría si en cinco
53 1 Penúltimo
años tu marido dejara de trabajar?”

AMPLIACIÓN
PÁG COLUMNA PÁRRAFO De Shazer considera que las personas y familias tienen los
recursos y el potencial de generar soluciones. Por eso sólo
52 2 9 busca crear las condiciones que faciliten el cambio, dedican-
do poco tiempo a conocer el problema, enfatizando las si-
tuaciones en las cuales no aparece la conducta problema o
AMPLIACIÓN aparece y es controlada.

3. Estratégicas.
Preguntas con un objetivo correctivo, para movilizar un siste-
ma atascado. “¿Por qué no hablas con él de tus problemas PÁG COLUMNA PÁRRAFO
en lugar de con tus hijos?”
53 2 1

PÁG COLUMNA PÁRRAFO


AMPLIACIÓN
52 2 10 Concreta: cómo notarán ellos o el entorno que han cambia-
do). Se presupone que quieren, saben y pueden cambiar. Se
AMPLIACIÓN orientan hacia las fortalezas. El objetivo es ampliar los aspec-
tos satisfactorios del funcionamiento de los miembros de la
4. Reflexivas. familia y las soluciones eficaces.
Para que la familia reflexione intencionadamente y genere
por sí misma las soluciones.
- Orientadas al futuro. Se pueden utilizar para: desarrollar
metas, explorar el resultado esperado, resaltar consecuen-
cias posibles si los patrones comportamentales persisten,
explorar expectativas catastróficas para exponer temas ocul-
tos, explorar posibilidades hipotéticas y suscitar esperanza y
optimismo.
- Que convierten al interrogado en observador.
Sirven para que los miembros de la familia distingan compor-
tamientos, eventos y percepciones que no habían percibido. AMPLIACIÓN
Pueden ir dirigidas a una persona sobre su relación con otro
para tomar conciencia de uno mismo (¿“Qué crees que pien- Técnicas
sa él sobre esta situación, y cómo crees que se siente?”) o • Búsqueda de excepciones.
para investigar la percepción interpersonal (¿” Qué piensa él Se pregunta por momentos en los que el síntoma no apare-
que piensas tú cuando tiene un atracón?”), y también pue- ce (PIR 93, 54).
den ser triádicas : Cuando tu madre empieza a discutir con • Preguntas presuposicionales.
tu hermana, ¿qué es lo que hace tu padre habitualmente, se Preguntas que presuponen algo positivo (que hay excepcio-
implica o se queda al margen? nes, o que ha habido mejorías).
- De cambio inesperado de contexto. • Elogios.
Ayudan a liberar disposiciones cognitivas rígidas y a valorar • Connotación positiva del síntoma.
otras perspectivas. Sirven para explorar contextos y significa- • Preguntas escalares.
dos opuestos y la necesidad de mantener el statu quo. Situar en una escala de 0 a 10 cómo se encuentra en este
- Preguntas con sugerencia implícita. Incluyen contenidos momento; formular preguntas sobre cómo podría bajar o

171
PÁG COLUMNA PÁRRAFO

53 2 2

AMPLIACIÓN
Técnicas
• Búsqueda de excepciones.
Se pregunta por momentos en los que el síntoma no apare-
ce (PIR 93, 54).
• Preguntas presuposicionales.
Preguntas que presuponen algo positivo (que hay excepcio-
nes, o que ha habido mejorías).
• Elogios.
• Connotación positiva del síntoma.
• Preguntas escalares.
Situar en una escala de 0 a 10 cómo se encuentra en este
momento; formular preguntas sobre cómo podría bajar o su-
bir en esa escala, para generar soluciones.
• Pregunta del milagro.
Preguntar por una situación imaginaria en la ha sucedido un
milagro y el problema que trae a consulta ha desaparecido.
Ayuda a clarificar objetivos.

172
TEMA 9
subir en esa escala, para generar soluciones.
PÁG COLUMNA PÁRRAFO PÁG COLUMNA PÁRRAFO
• Pregunta del milagro.
Preguntar por una situación imaginaria en la ha sucedido un
113 1 Penúltimo
milagro 108 1 a consulta
y el problema que trae Enha
terapia de
desaparecido.
Ayuda a clarificar objetivos. esquemas de
Young. En objeti-
vos de la terapia.
AMPLIACIÓN
TEMA 10 Entre el tercer
1. Pensar pros y contras de la situación experimentada.
y cuarto punto
añadimos un
punto más.

TEXTO AMPLIADO SI LA FUENTE ES OTRA PÁG COLUMNA PÁRRAFO

Objetivos de la terapia 113 1 Añadir una ta-


• Reconocer los esquemas, estilos de afrontamiento y modos bla con el título:
inadaptados. Reglas de la for-
• Establecer una apropiada y curativa relación paciente-te- mación de habili-
rapeuta. dades en Terapia
• Confrontación empática de los esquemas inadaptados y el Dialéctico Con-
aprendizaje de nuevos estilos de afrontamiento. ductual
• Promover cambios en los esquemas disfuncionales tempra-
nos (PIR 19, 51).
• Desarrollar un modo adulto saludable del paciente logran-
do la remisión o el control de los modos disfuncionales. AMPLIACIÓN

1. Los clientes que abandonan la terapia están fuera


de la terapia (perderse 4 semanas seguidas, no podrán volver
a entrar hasta que finalice).
PÁG COLUMNA PÁRRAFO
2. Todos los clientes tienen que seguir una terapia in-
dividual (no pueden estar 4 semanas seguidas sin acudir).
112 2 5
3. No acudir a sesiones bajo la influencia de drogas o
alcohol.
4. No se puede hablar de anteriores conductas para-
AMPLIACIÓN suicidas con otros pacientes fuera de sesión.
5. Quién llame a alguien para solicitar ayuda cuando
El déficit en regulación emocional se caracteriza por dificul- tiene tendencias suicidas debe estar dispuesto a aceptar esa
tades en inhibir conductas poco eficaces, concentrarse y dis- ayuda.
minuir activación fisiológica ante emociones intensas. Siendo 6. Información de las sesiones y nombres de pacientes
la conducta desadaptada y explosiva un intento de alivio del es confidencial.
malestar intenso y duradero. 7. Quién vaya a llegar tarde a una sesión o faltar debe
llamar para avisar.
8. No se pueden entablar relaciones privadas fuera de
PÁG COLUMNA PÁRRAFO las sesiones de formación.
9. Las parejas que mantienen relaciones sexuales no
pueden formar parte del mismo grupo.
112 2 6

PÁG COLUMNA PÁRRAFO


AMPLIACIÓN
113 1 En referencias bi-
Se pone el acento en los procesos dialécticos. Según Linehan bliográficas
la persona con TLP quedaría atrapada en polaridades (es de-
cir, entre pensamientos, conducta o emociones dicotómicas
y extremas), sin lograr conseguir la síntesis. La conducta y las
emociones dicotómicas y extremas características del TLP son AMPLIACIÓN
fracasos dialécticos. Polaridades:
• Necesidad de cambiar y necesidad de aceptarse MORENO FERNÁNDEZ, A. (2015): Manual de Terapia Sis-
(principio dialéctico fundamental). témica. Principios y estrategias de intervención. Desclée de
• Obtener lo que necesito y perder apoyo si me hago Brower. Bilbao.
competente.

173
PÁG COLUMNA PÁRRAFO PÁG COLUMNA PÁRRAFO

110 2 1 110 2 12

AMPLIACIÓN AMPLIACIÓN

Las relaciones que se establecen entre estímulos son: de vín- • Abordar que el control es el problema: Los intentos de con-
culo Mutuo (si A=B -> B=A), vínculo combinatorio (si A=B y trol responden a la lógica de la solución de problemas, pero
B=C -> A=C) y transformación de funciones (todas las fun- en este caso, intentar controlar un evento privado, no hace
ciones asociadas a A, se transfieren a B y C porque hay una más que facilitar su aparición. Se puede utilizar la metáfora
relación de equivalencia, en este caso). Así, si el pensamien- de “la lucha contra el monstruo” o la del “polígrafo”.
to “estoy gorda”, se equipara con “estar gorda” y eso se
equipara con recibir burlas de los compañeros o ser menos
valiosa, este pensamiento adquirirá funciones aversivas y la
persona tenderá a evitarlo (construirá entonces un conjunto PÁG COLUMNA PÁRRAFO
de razones o patrones de regulación verbal, con los que in-
tentará justificar sus conductas alimentarias). Así, los pensa- 110 2 13
mientos adquieren la función del estímulo que representan,
generando en la persona una reacción como si ese pensa-
miento fuese la realidad.
AMPLIACIÓN

PÁG COLUMNA PÁRRAFO • Aceptación de los eventos privados desagradables. La


aceptación es una actitud abierta, flexible y receptiva respec-
110 2 10 to a la experiencia. No es resignación ni algo pasivo, sino una
acción positiva que cambia la función de las experiencias. Se
puede usar la metáfora “de las 2 escalas”.

AMPLIACIÓN

Los problemas psicológicos vienen cuando el pensamiento


se torna literal y estos eventos privados PÁG COLUMNA PÁRRAFO
adquieren el control del comportamiento (fusión cognitiva)
y no por la simple presencia de estos eventos. 110 2 14

AMPLIACIÓN
PÁG COLUMNA PÁRRAFO
• Defusión cognitiva o distanciamiento de los eventos pri-
110 2 11 vados. Se pretende eliminar la fusión entre el lenguaje y la
realidad. Es decir, se anima a las personas a ver los even-
tos mentales como simples eventos mentales que van y vie-
AMPLIACIÓN nen y que no tienen que corresponderse con la realidad. Se
pueden usar ejercicios como “ver las nubes que pasan” o
No se trata de una terapia estructurada pero cuenta con los ejercicios de desliteralización de palabras, de etiquetado de
siguientes componentes: pensamientos o de contradicción del pensamiento con la ac-
• Desesperanza creativa (PIR 19, 31):: ción (decir “no puedo caminar”, mientras se camina, por
Intentar que la persona tome conciencia de la inutilidad de ejemplo).
sus intentos para solucionar sus problemas y busque otras
alternativas. Algunas metáforas que se pueden utilizar con
este fin son las de “las arenas movidizas” o “el hombre en
el hoyo”. PÁG COLUMNA PÁRRAFO

111 1 1

AMPLIACIÓN

• El Yo contexto.
El yo en perspectiva que permite eliminar la identificación de

174
los pensamientos y emociones (Yo contenido) con el yo con-
PÁG COLUMNA PÁRRAFO
texto. Es la capacidad de conectar con el sentido del Yo que
va más allá de los propios eventos privados. Se puede em-
111 1 6
plear la metáfora de “la casa y los muebles”, el ejercicio del
Yo observador, que permite verse a uno mismo en diferentes
momentos o roles, para hacer ver que uno no se reduce a sus AMPLIACIÓN
eventos privados sino que uno es algo más, que trasciende.
También encontramos la metáfora “del tablero de ajedrez”.
Los recursos se adaptarán al patrón de regulación ineficaz
que se observe en el análisis funcional de cada paciente. Se
utilizan múltiples recursos técnicos, principalmente el uso
PÁG COLUMNA PÁRRAFO de metáforas y analogías. Las mejores metáforas (Vargas
Mendoza, 2006) son aquellas que contienen elementos que
111 1 3 parten del propio cliente, poseen propiedades no arbitrarias,
son análogas al problema del cliente y el cliente tiene expe-
riencia con sus contenidos. Otros recursos utilizados son...

AMPLIACIÓN

• Identificación de valores. PÁG COLUMNA PÁRRAFO


Aspecto esencial de la ACT. Consiste en dirigir la actividad,
el esfuerzo y la determinación a lograr lo que la persona
111 1 7
quiere. Se puede recurrir a la metáfora del jardín, el ejercicio
del epitafio o el funeral. Estos ejercicios también sirven para
conocer la discrepancia entre los valores y la acción actual.
Existen asimismo, cuestionarios de valores.
AMPLIACIÓN
En cuanto a la evidencia científica de ACT, solo hay evidencia
sólida de su eficacia en dolor crónico. Se muestra superior a
PÁG COLUMNA PÁRRAFO las condiciones de control, pero aún está por establecerse su
superioridad respecto a la TCC.
111 1 5 Según un meta-análisis de Öst (2014): 1) ACT no cuenta to-
davía con respaldo empírico de calidad para alcanzar el es-
tatus de tratamiento bien establecido para ningún trastorno
mental; 2) ACT puede considerarse un tratamiento
AMPLIACIÓN probablemente eficaz para dolor crónico; 3) ACT posible-
mente pueda ser eficaz en depresión, sintomatología psicó-
Es decir, se busca desarrollar una mayor flexibilidad cognitiva tica, TOC, ansiedad, abuso de sustancias y estrés laboral.
(ver tabla del Modelo Hexaflex, para conocer los componen-
tes de la flexibilidad e inflexibilidad psicológica)

Modelo Hexaflex en ACT


Procesos de Flexibilidad psicológica (terapéuticos Procesos de Inflexibilidad Psicológica (psicopato-
y saludables) lógicos)

Consciencia del momento presente Pérdida contacto flexible con el presente

Clarificación de Valores Falta de claridad en los valores

Compromiso con la acción Inacción, impulsividad o evitación

Fortalecimiento del Yo Contexto (Yo perspectiva, Yo Apego al Yo conceptualizado o contenido (fusión con
observador o Yo trascendental) la propia historia o con los eventos privados)

Defusion o distanciamiento Fusión cognitiva

Aceptación Evitación experiencial

175
por AC de Alternative Coping (afrontamiento alternativo)
PÁG COLUMNA PÁRRAFO
para referirse a una nueva pauta de acción que rompe el
patrón de evitación, abriendo la posibilidad a cambiar la si-
111 1 (Sustituir todo el
tuación depresógena.
apartado 10.3)
- ACTION: La A de Assess (evaluar si con lo que hace
AMPLIACIÓN está manteniendo el problema), la C de Choose (decidir si
está actuando de forma resolutiva), la T de Try (probar las
Este tratamiento está destinado a la depresión y la preven- acciones decididas), la I de Integrate (integrar las nuevas ac-
ción de recaídas. tividades en las rutinas diarias), la O de Observe (evaluar si
Recupera las teorías conductuales de Fester y Lewinsohn so- tiene otros resultados diferentes) y la N de Never give up
bre la depresión. Fester defiende que la depresión es fruto de (seguir intentándolo).
una historia de aprendizaje en que las acciones de los indivi- Una vez que el paciente es instruido en el modelo, se co-
duos no generan reforzadores positivos y que las acciones de mienza con la monitorización de las actividades diarias del
escape o evitación resultan reforzadas negativamente. Esto paciente que tendrá un papel relevante durante todo el tra-
genera dos consecuencias, lo que se conoce como “encerra- tamiento. En cada una de las sesiones se repasará la cone-
miento en sí”, es decir, el sujeto se centra más en respon- xión entre este aumento de actividades y las consecuencias
der a su propia privación que en operar sobre las fuentes obtenidas a nivel afectivo, personal, social, etc. El objetivo es
de reforzamiento ambiental, y la reducción del repertorio que de forma progresiva el paciente vaya implicándose en
de conductas adaptativas. Lewinsohn, por su parte, señala mayor número y variedad de actividades relacionadas con
tres aspectos importantes: el número de eventos potencial- sus valores, abandonando el patrón de evitación conductual.
mente reforzantes para el individuo, la disponibilidad de re- Respecto a las variables cognitivas, la AC entiende que las
forzamiento en el ambiente y a conducta instrumental para personas podemos tener dos patrones de pensamiento: el
obtener reforzadores. Ambos modelos conductuales de la reflexivo y el rumiativo. Es el rumiativo es que se suele dar
depresión fueron desbancados por la influencia de las teorías más en pacientes en situación de depresión y es entendido
cognitivas, hasta que en 1996, las investigaciones de Neil Ja- como una forma de evitación cognitiva. Durante la terapia se
cobson ponen de relieve que el componente conductual de entrena al paciente a reconocer este tipo de pensamiento y
la terapia cognitiva por sí solo tiene la misma eficacia de la sustituirlo por la atención al momento presente, la toma de
terapia completa, así como mayor eficiencia. Esto da origen decisiones y la implicación en actividades.
a la Activación Conductual (AC). La AC cuenta con la posibilidad de añadir otros procedimien-
El objetivo de la terapia es conseguir mayor activación con- tos y técnicas conductuales tradicionales, como los ensayos
ductual y romper el patrón de evitación conductual (retirada de conducta, entrenamiento en asertividad, exposición, etc.
social, retira no social, evitación cognitiva…). También procedimientos de otras terapias contextuales,
Incluye un modelo teórico, por tanto, y un plan de trata- como la psicoterapia analítico funcional, la terapia de acep-
miento. El modelo teórico debe ser presentado al paciente tación y compromiso (ACT) y el mindfulness.
en las primeras sesiones de la terapia y asegurarnos un ade- La aceptación y los valores de la ACT van a tener especial
cuado entendimiento por su parte. relevancia en esta terapia. Por un lado, el estilo del terapeuta
En la presentación del modelo, es importante que el paciente de AC se caracteriza por la validación y aceptación incon-
y el terapeuta entiende que el paciente no está deprimido, si dicional, por reforzar los avances respecto a la activación y
no que está en situación de depresión, alejándose de plan- mostrarse comprensivo y empático. Ya que la AC propone
teamientos internistas, como los que proponen el modelo empezar la activación a pesar de no tener ganas, se requiere
cognitivo y el médico. Debe entender que el ánimo triste aceptación y atención no juzgadora de la experiencia pre-
desencadenado por una o más situaciones le lleva a abando- sente en contra como desánimo, tristeza, desesperación y
nar actividades o a implicarse en menor medida en acciones desmoralización. Por otro lado, se requiere que la activación
importantes para él, lo que disminuye el número de reforza- está orientada por los valores, más allá del efecto y de la
dores y favorece la tristeza y la apatía. Esta situación, le lleva recompensa inmediata.
a seguir evitando experiencias (darse de baja, rumiaciones, Marino Pérez resume en 10 los principios básicos de la AC:
etc.) lo que disminuye aún más la oportunidad de tener re- • La clave para cambiar cómo se siente la gente es
forzadores. Esto se le puede presentar al paciente como el ayudarle a cambiar lo que hace (el sentimiento sigue a la
círculo vicioso de la depresión. acción).
(Incluir “Figura 1. Círculo vicioso de la depresión” que está • Cambios en la vida pueden llevar a la depresión y las
en versión antigua) estrategias de evitación/escape pueden mantenerla.
El uso de diagramas, como el presentado en la imagen, ayu- • Las claves para saber qué será antidepresivo para un
dará al paciente a entender la conexión que hay entre las cliente se encuentran en lo que precede y sigue a las con-
situaciones, el patrón conductual que tiene y las consecuen- ductas importantes (utilización de registros para establecer la
cias que le siguen. conexión entre conductas y estados de ánimo).
(Incluir esquema que se incluye después de la presente ta- • Actuar de acuerdo a una meta en lugar de actuar de
bla). acuerdo a cómo uno se siente.
También el terapeuta se puede apoyar en algunos acrónimos • El cambio será poco a poco.
para ayudar al paciente a entender el funcionamiento actual: • Proponer actividades naturalmente reforzantes.
- TRAP: Está formado por T de Trigger (precipitante); • Actuar como un coach.
R de Response (respuestas depresivas ante el precipitante) y • Utilizar un enfoque orientado a la solución de pro-
AP de Avoidance Pattern (patrón de evitación respuestas de blemas.
tristeza, desánimo). • No se trata de rellenar el tiempo, sino de hacer ta-
- TRACT: Está formado por T y R, como en TRAP, y reas con un sentido práctico, para que algo cambie en el

176
ámbito cotidiano. en la relación con el terapeuta. Es por ello, que le va a trans-
• Solucionar barreras que puedan surgir en la activa- mitir al paciente desde el inicio lo importancia de que esto
ción. ocurra y el sentido que tiene.
Se considera que la AC es uno de los tratamientos más efica- El terapeuta va a ordenar las conductas de cliente por su
ces para la depresión incluyendo las más graves. relevancia clínica y las va a clasificar en:
A la par del desarrollo de la AC por Jacobson y sus colegas, 1) Conductas clínicamente relevantes tipo 1 (CCR1).
apareció también otra terapia muy similar formulada por Serían las conductas problema, las que ponen de manifiesto
otros autores. Es el caso de la Terapia de Activación Con- las dificultades interpersonales.
ductual para la Depresión (TADC; Lejuez, Hopko y Hopko, 2) Conductas clínicamente relevantes tipo 2 (CCR2).
2002). Añade a la base teórica de la AC la “ley de la igua- Serían las conductas positivas, las que se quiere desarrollar
lación” de Hernstein, que señala que en la depresión se da en el paciente.
a la vez el reforzamiento aumentado de las conductas de- 3) Conductas clínicamente relevantes tipo 3 (CCR3)
presivas. Es una terapia más breve, en torno a 10 sesiones, que hacen referencia a las interpretaciones que hace el pa-
que se propone como recomendable en casos de depresión ciente de su conducta.
moderada. Por tanto, las cinco reglas que van a guiar la labor del tera-
peuta son:
1. Buscar las CCR.
2. Provocar la ocurrencia del CCR1.
PÁG COLUMNA PÁRRAFO 3. Reforzar la CCR2. Es la operación terapéutica por
antonomasia de la PAF.
111 1 7 4. Observar los efectos potencialmente reforzantes de
la conducta del terapeuta respecto a las CCR.
5. Ofrecer interpretaciones alternativas funcionales (en
AMPLIACIÓN base a antecedentes y consecuentes).
La PAF, además, ofrece una teoría sobre el desarrollo del yo,
La psicoterapia analítica funcional (PAF) va destinada a tra- una clasificación de los problemas del yo (inseguro, inautén-
bajar con aquellas personas que tienen problemas interper- tico, límite, narcisista, personalidad múltiple), así como un
sonales. abordaje dirigido al fortalecimiento de este.
Tiene base en el conductismo radical de Skinner, aunque en
muchos aspectos (de ahí el nombre) va a recordar al psicoa-
nálisis.
Cinco son las características distintivas de esta terapia:
- Relación terapéutica. La PAF aplica los principios
conductuales derivados del análisis funcional de conducta a
la relación terapéutica. Entiende que esta relación es un mi-
crocosmo donde se van a dar las dificultades que el paciente
tiene en su ambiente natural. En esta relación se va a dar el
problema y también la mejoría.
- Equivalencia funcional. Este concepto hace referen-
cia a que lo que ocurre en las sesiones es muy simular a lo
que le ocurre al paciente en su entorno. El terapeuta y la
relación deben ser lo más similar posible al contexto natural
donde surgen los problemas. Por eso es importante analizar
las funciones que tiene el terapeuta y la estructura de la si-
tuación terapéutica.
- Funciones del terapeuta. Tres van a ser las funcio-
nes:
1) Suscitación de reacciones emocionales.
2) Evocación de conductas operantes.
3) Provisión de consecuencias a las conductas que emi-
te el paciente para reforzarlas o para disminuirlas o eliminar-
las.
- Situaciones terapéuticas. Contempla que muchas
situaciones de la terapia deben ser utilizadas para trabajar
situaciones de la vida real (final de la terapia, cambios de
hora, deseos de terminar la terapia antes…).
- Puente del lenguaje. Lo que dice el paciente en la
consulta puede tener una función diferente a la aparente.
Es importante, por tanto, realizar un análisis funcional del
lenguaje también en base a antecedentes y consecuentes.
Skinner describe 5 funciones del lenguaje (ver tabla …)
(Incluir tabla que aparece al final del documento)
La PAF se va a interesar desde el principio en que los proble-
mas de la vida cotidiana del cliente se presenten y emerjan

177
Figura 2. Modelo de la depresión presentado al paciente (To-
mado de Pérez (2014))

Intraverbal El control de la conducta verbal es también verbal


(verbalizaciones suscitadas por otras verbalizacio-
nes).

Autoclítica Comentarios hacia la propia conducta verbal (me-


taconducta verbal).

Tacto Controlada pon un estímulo discriminativo.

Tiene la función de referenciar y de nombrar (in-


formar sobre algo).

Mando Reforzada por una consecuencia característica.


Tiene una función instrumental (de conseguir
algo).
Se trata de verbalizaciones relacionadas con solici-
tudes, peticiones, súplica…
Existen también los mandos disfrazados de tactos,
de especial relevancia en esta terapia.

Tabla… . Algunas funciones de la conducta verbal del pa-


ciente.

178
PÁG COLUMNA PÁRRAFO

113 1 4

AMPLIACIÓN SI LA FUENTE ES OTRA

Cuatro módulos de formación de habilidades (PIR 11, 206)


(Ver tabla 2). (Añadida al final del documento)

179
PÁG COLUMNA PÁRRAFO PÁG COLUMNA PÁRRAFO

110 1 1 110 1 8

TEXTO DEL MANUAL TEXTO DEL MANUAL


Se trata de un conjunto de terapias desarrolladas en los 90 Las dos grandes principios de estas terapias son la acepta-
que busca superar, en parte, el carácter “mecanicista” de ción y la activación.
las generaciones anteriores. En relación a las terapias cogni- La aceptación se define como el abandono de la búsqueda
tivo-conductuales previas, las terapias de tercera generación permanente del bienestar, tomando los síntomas y el ma-
recuperan el papel del contexto en la vida del paciente y por lestar como experiencia vital normal. La activación hace re-
tanto, en su salud mental. Contemplan el individuo de for- ferencia a la consecución de objetivos valiosos en la vida,
ma más holística, y surgen del rechazo a las posiciones de que es posible cuando el foco deja de estar en los síntomas
las generaciones anteriores, centradas o bien en la conducta (aceptación).
(mecanicistas) o en la cognición (internalistas). Sin embargo, La eficacia en estas terapias no se mide por la disminución
no hay que olvidar que las terapias de tercera generación o eliminación de los síntomas, sino por los logros de los pa-
parte de las anteriores, y que por tanto, comparten ciertos cientes en consonancia con sus valores.
aspectos. Siguen poniendo el acento en la conducta y en los
pensamientos, si bien cambia el objetivo: no se trata de eli-
minar los síntomas, sino de relacionarse de forma diferente
con ellos.
PÁG COLUMNA PÁRRAFO

113 2 1
PÁG COLUMNA PÁRRAFO

110 1 3
AMPLIACIÓN SI LA FUENTE ES OTRA
TEXTO DEL MANUAL Adrian Wells, el autor de esta teoría, la considera como una
extensión de la TCC clásica, ya que busca tomar conciencia
Bajo el nombre de terapias de tercera generación están eng- de un producto cognitivo como paso previo a la reestructu-
lobadas gran variedad de terapias, que hoy en día siguen ración cognitiva de las creencias metacognitivas. Sin embar-
experimentado un gran auge y proliferación. Cada una de go, se incluye dentro de las terapias de tercera generación
ellas introduce aspectos diferentes en la forma de entender porque es una terapia que desplaza el foco de los contenidos
los trastornos y en las técnicas empleadas, pero todas ellas cognitivos a los procesos, del “qué” al “cómo”. Wells plan-
comparten una base teórica común. Las características co- tea que lo que determina las emociones de una persona y su
munes a estas terapias son: control sobre ellas depende de cómo piensa acerca de eso,
lo que llamamos metacognición.

PÁG COLUMNA PÁRRAFO


PÁG COLUMNA PÁRRAFO
110 1 3
113 2 2

AMPLIACIÓN SI LA FUENTE ES OTRA AMPLIACIÓN SI LA FUENTE ES OTRA


Las terapias de tercera generación ponen el énfasis en las Distingue entre las preocupaciones tipo 1, que serían las que
dimensiones clínicas comunes, más que en los hacen referencia a eventos que suponen una amenaza, y las
preocupaciones tipo 2 o metapreocupaciones, que son las
mecanismos específicos de los trastornos, dados los proble- aquellas que derivan de darse cuenta de que uno se está
mas planteados con los sistemas diagnósticos. Del mismo preocupando y el efecto que esto puede tener “preocuparse
modo, enfatizan los principios terapéuticos generales, más puede llevar a sufrir un infarto”.
que las técnicas específicas.

180
estas cogniciones le producen. La TMC conserva el uso de
PÁG COLUMNA PÁRRAFO
estrategias típicas de la TC (cuestionamiento socrático, expe-
rimentos cognitivos…) dirigidas a modificar metacreencias
113 2 3 con el trastorno mientras que la TC estándar lo hace con las
creencias ordinarias. La TMC focaliza en el estilo del pensa-
miento y formula el síndrome cognitivo atencional. Incluye
AMPLIACIÓN SI LA FUENTE ES OTRA estrategias de entrenamiento atencional y conciencia plena
“desapegada”.
La terapia metacognitiva describe el Síndrome Cognitivo
Atencional (SCA), como explicación del mantenimiento de Tema:11
los trastornos mentales. Este patrón se caracteriza por:
- Rumiación/Obsesiones PÁG COLUMNA PÁRRAFO
- Preocupación
- Atención fija 115 1 11
- Conductas de afrontamiento o estrategias de autorregula- En terapia inter-
ción poco útiles personal. En los
puntos de las
Este síndrome provocaría un sesgo en el que la atención áreas problemá-
queda fijada en estímulos que supongan una amenaza o ticas.
peligro, llamado “monitoreo de amenazas”. El tener el foco
atencional constantemente en los eventos amenazantes o
negativos, hace que sean menos accesibles o reconocibles
TEXTO AMPLIADO SI LA FUENTE ES OTRA
los eventos positivos que ocurran en sus vidas, lo que perpe-
tuaría este sesgo.
Los síntomas se relacionan con problemas en una o más
áreas problemáticas (PIR 13, 120; PIR 19, 32):
Wells diferencia dos modos de la experiencia:
• Duelo.
Pérdidas de personas significativas.
- Modo objeto: aquel en el que se funden los pensamientos
• Disputas interpersonales de rol.
con la realidad, no son
• Transiciones de roles.
considerados eventos internos, sino como fiel reflejo de lo
Renunciar al rol actual, expresar sentimientos de culpa, en-
que ocurre en el mundo externo.
fado o pérdida, adquirir habilidades y establecer nuevos vín-
- Modo metacognitivo: es aquel en el que los pensamientos
culos (PIR 12, 142).
son conscientemente observados
• Déficits interpersonales.
de “modo desapegado” como eventos separados del self
y evaluados como sucesos mentales independientes de la
realidad.
El modo objeto conduce habitualmente al síndrome cogni-
tivo atencional.

Dado que el modo metacognitivo es el objetivo de la terapia,


y que lograrlo requiere práctica y aprendizaje, dentro del pro-
tocolo se incluyen ejercicios específicos para desarrollar este
modo de procesamiento. Algunas de las técnicas que inclu-
ye la TMC son el mindfulness desapegado, entrenamiento
en atención y la reorientación de la atención situacional (PIR
19, 63). La atención plena desapegada o “detached mind-
fulness”es una versión de la atención plena clásica dirigida
a impedir la activación del SCA, ya que las personas no que-
darían enganchadas a sus pensamientos y la atención podría
dirigirse a otros aspectos, permitiendo incorporar así otras
experiencias. Tanto la TMC (terapia metacognitiva) como la
TC (terapia cognitiva) comparten la importancia que le dan
al procesamiento mental en los trastornos psicológicos. Sin
embargo, la TMC se dirige más a los procesos que a los con-
tenidos cognitivos. Mientras que la TCC aborda las creencias
acerca de sí mismo y el mundo, la terapia de Wells explora
las creencias acerca del pensar y preocuparse, es decir, se
centra en las reacciones que provoca en las personas tener
determinados pensamientos negativos. Por tanto, la diferen-
cia radica en que para la Terapia Metacognitiva, no es el
contenido de los pensamientos lo que determina la presen-
cia de un trastorno, ya que la mayoría de la población tiene
pensamientos negativos, sino la forma en la que la persona
se queda “enganchada” a las emociones desagradables que

181
REFERENCA BIBLIOGRÁFICA

Álvarez, M. P. (2014). Las terapias de tercera generación como terapias contextuales. Editorial Síntesis.

Ruiz Fernández, M. Á., Díaz García, M. I., & Villalobos Crespo, A. (2012). Manual de técnicas de inter-
vención cognitivo conductuales.Bilbao: Ed Desclée De Brouwer, SA.

WELLS, A. (2019) . Terapia metacognitiva para la ansiedad y la depresión. Desclée de Brouwer. Bilbao.

Garay, C. J., & Keegan, E. (2016). Terapia metacognitiva. El síndrome cognitivo atencional y los proce-
sos cognitivos. Revista Argentina de Clínica Psicológica, 25(2), 125-134.

WEISSMAN MYRNA, M., MARKOWITZ JOHN, C., KLERMAN GERALD, L. (2013): Manual de Psicotera-
pia Interpersonal. Editorial Grupo 5. Madrid.

JEFFREY E. YOUNG, JANET S. KLOSKO, MAJORIE E. WEISHAAR. (2015): Terapia de Esquemas: Guía
Práctica. Desclée de Brouwer. Bilbao.

MORENO FERNÁNDEZ, A. (2015): Manual de Terapia Sistémica, Principios y estrategias de interven-


ción. Desclée de Brower. Bilbao.

182
183
ESTADÍSTICA, MÉTODO EXPERIMENTAL Y PSICOMETRÍA
RESEÑAS

RESEÑAS
PÁG COLUMNA PÁRRAFO PÁG COLUMNA PÁRRAFO

60 2 Penúltimo 35 1 4

TEXTO AMPLIADO SI LA FUENTE ES OTRA TEXTO DEL MANUAL

Texto en las ampliaciones El análisis de varianza


Previamente a la realización del análisis de varianza, se tie-
nen que cumplir varios supuestos comunes a otras pruebas
paramétricas (PIR 95, 175; PIR 06, 26), y que ya hemos
PÁG COLUMNA PÁRRAFO mencionado:
• La normalidad. Se dice que la prueba ANOVA no es robus-
73 TABLA 1 ta a la violación del
supuesto de normalidad. Es decir, el supuesto de normalidad
es fundamental para la realización de tal análisis. En caso de
TEXTO DEL MANUAL que no se cumpla el supuesto, existen diferentes tipos de
transformación que pueden ayudar a “normalizar” la mues-
ECUACIÓN QUE RELACIONA LA tra (transformación logarítmica o cuadrática, p. ej.) (PIR 99,
FIABILIDAD CON LA VARIABLE/LONGITUD DEL TEST (TABLA) 52).
• La independencia. Este supuesto depende esencialmen-
Ecuación general de Spearman-Brown (PIR 18, 39; PIR19, te de la aleatoriedad del muestreo. Para confirmar este
157): supuesto, se suele utilizar la prueba de Durbin Watson (PIR
Rxx = n rxx / 1+ (n-1) rxx 16, 230; PIR 18, 35). En caso de que no se cumpla este su-
Rxx = nuevo coeficiente de fiabilidad puesto, se puede aplicar la técnica de mínimos cuadrados.
rxx = coeficiente de fiabilidad del test original. • Homocedasticidad. En caso de que no se cumpla este su-
n = número de veces que aumento la longitud del test (y no puesto, se suele transformar los datos mediante el modelo
el número de ítems que lo alargo). de logaritmo neperiano o el método de mínimos cuadrados.

Una vez verificado el cumplimiento de los supuestos, se pro-


cede al análisis propiamente dicho. Cada grupo estudiado
PÁG COLUMNA PÁRRAFO reúne un número específico de sujetos, que puede diferir.
Cuando todos los k grupos estudiados contienen el mismo
50 2 2 número de sujetos, se dice que el modelo es equilibrado o
balanceado (PIR 97, 36; PIR 02, 170). El cálculo del estadís-
tico F (PIR 93, 61) como cociente de las medias cuadráticas
TEXTO AMPLIADO SI LA FUENTE ES OTRA intergrupo e intragrupo (F empírica) y su comparación la F
teórica (o sea, la F que se encuentra en las tablas de dis-
Texto en las ampliaciones tribución de densidad de probabilidad) permite tomar una
decisión acerca del mantenimiento o rechazo de la hipótesis
nula (PIR 99, 42). Si consideramos Fk como la F empírica y
F1−α,k−1,n−k como la F teórica:
PÁG COLUMNA PÁRRAFO • Si , entonces se rechaza H0.
• Si , entonces se mantiene H0.
25 2 2
En caso de que se rechace H0, se admite que existen diferen-
cias significativas entre los grupos. Sin embargo, el estadís-
tico F no específica entre qué grupos existen las diferencias
TEXTO AMPLIADO SI LA FUENTE ES OTRA significativas. Para averiguar este aspecto, se usa la prue-
ba de Scheffé y otras comparaciones múltiples (PIR 94, 60;
Texto en las ampliaciones PIR19, 160).

TEXTO AMPLIADO SI LA FUENTE ES OTRA

Texto en las ampliaciones

184
• El bloqueo de variables relevantes.
PÁG COLUMNA PÁRRAFO
Esta técnica consiste en neutralizar el efecto de una variable
contaminadora mediante la formación de conjuntos o blo-
52 2 4
ques de sujetos que comparten una característica relevante
al fenómeno estudiado (PIR19, 162). A continuación, se
TEXTO DEL MANUAL forman grupos que tenga un número similar de sujetos en
cada uno de los bloques.
B2. Discontinuidad en la regresión (PIR 16, 47; PIR19, 161)

En este diseño, la regla de asignación de los sujetos a los


tratamientos se conoce y se mide. Los sujetos se asignan a la
condición experimental o a la condición de control en fun- PÁG COLUMNA PÁRRAFO
ción de las puntuaciones en una determinada medida pre-
tratamiento, una vez establecido un punto de corte en ésta. 32 1 1
Finalmente, se administra un postest sobre ambos grupos.
Según algunos autores, es uno de los diseños cuasiexperi-
mentales con los que puede inferirse relaciones causales con
más garantías. No obstante, se plantea la siguiente limita- TEXTO DEL MANUAL
ción: la posibilidad de que exista interacción entre el factor
tratamiento y el nivel de maduración de los sujetos que con- La regla de decisión
forman ambos grupos; de manera que aquellos que puntúen …
por debajo del punto de corte en la medida pre-tratamiento
presenten un ritmo más lento de desarrollo madurativo. De manera paralela, se ha definido un error de tipo II, que
define la probabilidad de considerar una hipótesis como
verdadera cuando es en realidad falsa (PIR 97, 35). Se
TEXTO AMPLIADO SI LA FUENTE ES OTRA representa por la letra griega β. Existe por lo tanto cuatro
tipos de decisiones posibles al realizar un contraste de de
Texto en las ampliaciones hipótesis:

• Rechazar (o sea, considerar como falsa) H0 cuando es en


realidad verdadera. Esto representa el error de tipo I y es
igual al valor de α
PÁG COLUMNA PÁRRAFO (PIR 93, 76; PIR 99, 45; PIR 08, 226; PIR 11, 242; PIR19,
163).
46 2 4 • Mantener H0 cuando es verdadera, es decir, tomar la de-
cisión correcta. La probabilidad que se asocia a este acierto
es 1-α, esto es, el nivel de confianza (PIR 96, 48; PIR 98,
TEXTO DEL MANUAL 43).
• Mantener (o sea considerar como verdadera) H0 cuando
Otras técnicas de control es en realidad falsa. Esto representa el error de tipo II y es
• El simple y el doble ciego. igual al valor de β (PIR 14, 27).
A veces, informar a los sujetos del objeto de estudio del ex- • Rechazar H0 cuando es falsa, es decir, tomar la decisión
perimento puede influir en sus respuestas. La técnica de sim- correcta. La probabilidad de este acierto es 1-β. Esto repre-
ple ciego consiste por lo tanto en esconder a los sujetos el senta la potencia de la prueba (PIR 18, 36).
objeto del experimento y su hipótesis (PIR 00, 11). Además
de los sujetos, el mismo experimentador puede influir en la
respuesta (PIR 07, 58), facilitando indicios para que los suje-
tos respondan de tal o cual manera a los experimentos.
La técnica de doble ciego (PIR 95, 178) consiste en escon-
der al experimentador (o sea, a la persona encargada de
dar las
instrucciones a los sujetos) las condiciones del experimento
que realiza.
• El tratamiento placebo.
Consiste en hacer creer a uno de los grupos de estudio que
está recibiendo un tratamiento cuando no lo está recibien-
do (PIR 08, 228). Esto puede generar un efecto consistente
en la observación de un cambio favorable en la patología
de un paciente que es atribuible a los atributos positivos
que el
mismo paciente da al tratamiento que recibe. El efecto no-
cebo consiste al contrario en un empeoramiento de una pa-
tología que se debe a los atributos negativos que el paciente
da al tratamiento que recibe.

185
ESTADÍSTICA, MÉTODO EXPERIMENTAL Y PSICOMETRÍA
AMPLIACIONES

AMPLIACIONES • Rechazar (o sea, considerar como falsa) H0 cuando es en


realidad verdadera. Esto representa el error de tipo I y es
igual al valor de α
Tema 1. Estadística. (PIR 93, 76; PIR 99, 45; PIR 08, 226; PIR 11, 242; PIR19,
163).
• Mantener H0 cuando es verdadera, es decir, tomar la de-
PÁG COLUMNA PÁRRAFO
cisión correcta. La probabilidad que se asocia a este acierto
es 1-α, esto es, el nivel de confianza (PIR 96, 48; PIR 98, 43).
25 2 2 • Mantener (o sea considerar como verdadera) H0 cuando es
en realidad falsa. Esto representa el error de tipo II y es igual
TEXTO AMPLIADO SI LA FUENTE ES OTRA al valor de β (PIR 14, 27).
• Rechazar H0 cuando es falsa, es decir, tomar la decisión co-
La regresión múltiple rrecta. La probabilidad de este acierto es 1-β. Esto representa
la potencia de la prueba (PIR 18, 36).
[…] Por ejemplo, mientras la regresión simple procuraría
predecir el tiempo necesario de un coche para acabar una
carrera a partir de su velocidad, la regresión múltiple toma- PÁG COLUMNA PÁRRAFO
ría además en cuenta las capacidades de reflejos del piloto,
o/y su capacidad de atención, o/y las condiciones climáticas, 35 1 4
etc. Los predictores en la regresión lineal múltiple pueden
ser de un nivel de medida cuantitativo, ordinal o nominal (o
categóricos). TEXTO DEL MANUAL
La ecuación de regresión sería la de un plano (para dos va- El análisis de varianza
riables predictoras): Previamente a la realización del análisis de varianza, se tie-
nen que cumplir varios supuestos comunes a otras pruebas
Y = B0 + B1X1 + B2X2 paramétricas (PIR 95, 175; PIR 06, 26), y que ya hemos
mencionado:
Con “B0” representando la altura a la que corta el plano el • La normalidad. Se dice que la prueba ANOVA no es robus-
punto Y (el eje de ordenadas). Este es el valor de la constan- ta a la violación del
te, esto es, el valor de Y cuando el predictor/es es/son igual supuesto de normalidad. Es decir, el supuesto de normalidad
a 0. es fundamental para la realización de tal análisis. En caso de
Con “B1” representando la pendiente del plano en el eje 1. que no se cumpla el supuesto, existen diferentes tipos de
Con “B2” representando la pendiente del plano en el eje 2. transformación que pueden ayudar a “normalizar” la mues-
tra (transformación logarítmica o cuadrática, p. ej.) (PIR 99,
Hay diferentes métodos para identificar el mejor modelo pre- 52).
dictivo, con el menor número de predictores posibles. Entre • La independencia. Este supuesto depende esencialmente
ellos, están los siguientes: 1) jerárquico; 2) introducir (simul- de la aleatoriedad del muestreo. Para confirmar este supues-
táneo); 3) escalonado (stepwise); 4) hacia atrás (backward); to, se suele utilizar la prueba de Durbin Watson (PIR 16, 230;
5) hacia adelante (forward); y 6) eliminar (PIR19, 159). PIR 18, 35). En caso de que no se cumpla este supuesto, se
puede aplicar la técnica de mínimos cuadrados.
• Homocedasticidad. En caso de que no se cumpla este su-
PÁG COLUMNA PÁRRAFO puesto, se suele transformar los datos mediante el modelo
de logaritmo neperiano o el método de mínimos cuadrados.
32 1 1
Una vez verificado el cumplimiento de los supuestos, se pro-
cede al análisis propiamente dicho. Cada grupo estudiado
reúne un número específico de sujetos, que puede diferir.
TEXTO DEL MANUAL Cuando todos los k grupos estudiados contienen el mismo
número de sujetos, se dice que el modelo es equilibrado o
La regla de decisión balanceado (PIR 97, 36; PIR 02, 170). El cálculo del estadís-
… tico F (PIR 93, 61) como cociente de las medias cuadráticas
intergrupo e intragrupo (F empírica) y su comparación la F
De manera paralela, se ha definido un error de tipo II, que teórica (o sea, la F que se encuentra en las tablas de dis-
define la probabilidad de considerar una hipótesis como ver- tribución de densidad de probabilidad) permite tomar una
dadera cuando es en realidad falsa (PIR 97, 35). Se repre- decisión acerca del mantenimiento o rechazo de la hipótesis
senta por la letra griega β. Existe por lo tanto cuatro tipos de nula (PIR 99, 42). Si consideramos Fk como la F empírica y
decisiones posibles al realizar un contraste de de hipótesis: F1−α,k−1,n−k como la F teórica:

186
• Si , entonces se rechaza H0. está recibiendo un tratamiento cuando no lo está recibiendo
• Si , entonces se mantiene H0. (PIR 08, 228). Esto puede generar un efecto consistente en
la observación de un cambio favorable en la patología de
En caso de que se rechace H0, se admite que existen diferen- un paciente que es atribuible a los atributos positivos que el
cias significativas entre los grupos. Sin embargo, el estadís- mismo paciente da al tratamiento que recibe. El efecto no-
tico F no específica entre qué grupos existen las diferencias cebo consiste al contrario en un empeoramiento de una pa-
significativas. Para averiguar este aspecto, se usa la prue- tología que se debe a los atributos negativos que el paciente
ba de Scheffé y otras comparaciones múltiples (PIR 94, 60; da al tratamiento que recibe.
PIR19, 160). • El bloqueo de variables relevantes.
Esta técnica consiste en neutralizar el efecto de una variable
contaminadora mediante la formación de conjuntos o blo-
TEXTO AMPLIADO SI LA FUENTE ES OTRA ques de sujetos que comparten una característica relevante
al fenómeno estudiado (PIR19, 162). A continuación, se for-
En caso de que se rechace la H0, se admite que existen dife- man grupos que tenga un número similar de sujetos en cada
rencias estadísticamente significativas entre los grupos. Sin uno de los bloques.
embargo, el estadístico F no específica entre qué pares de
grupos existen dichas diferencias significativas. Para averi-
guar este aspecto, se han de realizar comparaciones a poste-
riori (post hoc) o no planificadas. Entre ellas, están la prueba PÁG COLUMNA PÁRRAFO
HSD de Tukey, el procedimiento de Dunnett y la prueba de
Scheffé (PIR 94, 60). 50 2 2

No obstante, también se pueden realizar comparaciones


planificadas o a priori, antes de realizar el ANOVA, a fin de
someter a prueba determinadas hipótesis predefinidas. Entre TEXTO AMPLIADO SI LA FUENTE ES OTRA
estas últimas, se encuentran la prueba de Dunn-Bonferroni y
la prueba Dunn-Sidák. Procedimiento de análisis general
Un diseño factorial permite estudiar los siguientes dos gran-
No obstante, en la actualidad se emplean otras pruebas, no des
basadas en la significación estadística de decisión dicotómica tipos de efectos:
(rechazo versus mantengo la H0), como son los índices de • Efecto principal: Se trata de la influencia de cada factor ais-
tamaño de efecto. En el caso del ANOVA, se recomienda el lado sobre la variable dependiente. Existirían tantos efectos
uso de omega cuadrado, y no del índice eta-cuadrado, por principales como número de factores incluidos en un deter-
ser este último un estimador sesgado del tamaño del efecto minado diseño. Por ejemplo, el efecto principal A, el efecto
a nivel poblacional (PIR19, 160). principal B, etc.
• Efecto de interacción: Cuando la influencia de un factor
sobre una variable dependiente varía en función de los valo-
res que adopta otro factor o factores. Por ejemplo, el efecto
Tema 2. Método científico y interactivo AxB, el efecto interactivo AxBxC.

experimental. Es muy importante resaltar que hay una jerarquía en la inter-


pretación de resultados, puesto que, si el efecto interactivo
resulta estadísticamente significativo, no se interpretarán los
PÁG COLUMNA PÁRRAFO
efectos principales de los factores que conforman tal inte-
racción, sino que sólo se analizará dicho efecto interactivo y
46 2 4
los correspondientes efectos simples. Es interesante conocer
que los efectos principales pueden no resultar estadística-
TEXTO DEL MANUAL mente significativos, pero sí alcanzar la significación estadís-
tica un efecto interactivo, al ser éste último el que mejor
Otras técnicas de control explique la varianza de la variable dependiente (PIR 16, 48;
• El simple y el doble ciego. PIR 18, 32; PIR19, 158).
A veces, informar a los sujetos del objeto de estudio del ex-
perimento puede influir en sus respuestas. La técnica de sim-
ple ciego consiste por lo tanto en esconder a los sujetos el
objeto del experimento y su hipótesis (PIR 00, 11). Además
de los sujetos, el mismo experimentador puede influir en la PÁG COLUMNA PÁRRAFO
respuesta (PIR 07, 58), facilitando indicios para que los suje-
tos respondan de tal o cual manera a los experimentos. 52 2 4
La técnica de doble ciego (PIR 95, 178) consiste en esconder
al experimentador (o sea, a la persona encargada de dar las
instrucciones a los sujetos) las condiciones del experimento TEXTO DEL MANUAL
que realiza.
• El tratamiento placebo. B2. Discontinuidad en la regresión (PIR 16, 47; PIR19, 161)
Consiste en hacer creer a uno de los grupos de estudio que
En este diseño, la regla de asignación de los sujetos a los

187
tratamientos se conoce y se mide. Los sujetos se asignan a la
PÁG COLUMNA PÁRRAFO
condición experimental o a la condición de control en fun-
ción de las puntuaciones en una determinada medida pre-
tratamiento, una vez establecido un punto de corte en ésta. 73 TABLA 1
Finalmente, se administra un postest sobre ambos grupos.
Según algunos autores, es uno de los diseños cuasiexperi- TEXTO DEL MANUAL
mentales con los que puede inferirse relaciones causales con
más garantías. No obstante, se plantea la siguiente limita- ECUACIÓN QUE RELACIONA LA
ción: la posibilidad de que exista interacción entre el factor FIABILIDAD CON LA VARIABLE/LONGITUD DEL TEST (TABLA)
tratamiento y el nivel de maduración de los sujetos que con-
forman ambos grupos; de manera que aquellos que puntúen Ecuación general de Spearman-Brown (PIR 18, 39; PIR19,
por debajo del punto de corte en la medida pre-tratamiento 157):
presenten un ritmo más lento de desarrollo madurativo. Rxx = n rxx / 1+ (n-1) rxx
Rxx = nuevo coeficiente de fiabilidad
rxx = coeficiente de fiabilidad del test original.
TEXTO AMPLIADO SI LA FUENTE ES OTRA n = número de veces que aumento la longitud del test (y no
el número de ítems que lo alargo).
B2. Discontinuidad en la regresión (PIR 16, 47; PIR19, 161)

En este diseño, la regla de asignación de los sujetos a los


tratamientos se conoce y se mide; frente a los dos diseños
anteriores cuyos grupos a comparar ya estaban previamente
formados (grupos no equivalentes) y la regla de asignación
era desconocida. Los sujetos se asignan a la condición expe-
rimental o a la condición de control en función de las pun-
tuaciones en una determinada medida pretratamiento, una
vez establecido un punto de corte en ésta. Finalmente, se
administra un postest sobre ambos grupos. En este sentido,
es un diseño pretest-postest con grupo de control, pero con
un método diferente de asignación de los participantes a los
distintos grupos. Según algunos autores, es uno de los dise-
ños cuasiexperimentales con los que puede inferirse relacio-
nes causales con más garantías. No obstante, se plantea la
siguiente limitación: la posibilidad de que exista interacción
entre el factor tratamiento y el nivel de maduración de los
sujetos que conforman ambos grupos; de manera que aque-
llos que puntúen por debajo del punto de corte en la medida
pre-tratamiento presenten un ritmo más lento de desarrollo
madurativo.

Tema 3. Psicometría.
PÁG COLUMNA PÁRRAFO

60 2 Penúltimo

TEXTO AMPLIADO SI LA FUENTE ES OTRA

[...]. Es decir, en la TCT la medida de precisión y la de error


del test será la misma para todos los sujetos, independien-
temente del nivel de éstos en la variable de medida. Por lo
tanto, la TCT considera un nivel de precisión uniforme para
todo el test y la ausencia de invarianza en su capacidad pre-
dictiva. Estas dos cuestiones mencionadas son limitaciones
teóricas que la TRI ha superado (PIR19, 156).

188
189
EVALUACIÓN PSICOLÓGICA
RESEÑAS
PÁG COLUMNA PÁRRAFO

65 Imagen el esque- Imagen el esque-


PÁG COLUMNA PÁRRAFO ma de la estruc- ma de la estruc-
tura tura
33 2 Punto nuevo al
final. TEXTO DEL MANUAL

TEXTO AMPLIADO SI LA FUENTE ES OTRA ESTRUCTURA DEL WAIS IV.


Escala comprensión verbal
4.6. Autoinformes en niños. Pruebas principales: Semejanzas, Vocabulario, Información
En cuanto a sus limitaciones, una de las fundamentales que Pruebas opcionales:Comprensión
debemos tener en cuenta de cara a la evaluación infantil es Escala razonamiento perceptivo
el nivel de desarrollo cognitivo del niño o niña que estamos Pruebas principales:Cubos, Matrices, Puzles visuales
evaluando, no siendo fiable la información recogida con es- Pruebas opcionales:Balanzas (solo 16-69 años),Figuras in-
tas técnicas en edades inferiores a los 7 años. Esto podemos completas
relacionarlo con que previamente a los 7 años, el niño o la Escala memoria de trabajo
niña se encuentran en el estadio preoperacional del desarro- Pruebas principales:Dígitos, Aritmética
llo cognitivo, y no es hasta aproximadamente los 7 años que Pruebas opcionales:Letras y números (solo 16-69 años)
se alcanza el estadio de las operaciones concretas, en el cual Escala velocidad de procesamiento
comienzan a desarrollarse múltiples habilidades entre ellas Pruebas principales:Búsqueda de símbolos, Clave de núme-
las relacionadas con la identificación de las propias caracte- ros (PIR19, 102)
rísticas psicológicas y la diferenciación de los aspectos físicos Pruebas opcionales:Cancelación (solo 16-69 años)
y mentales de uno mismo. (PIR19, 87) Escala

PÁG COLUMNA PÁRRAFO PÁG COLUMNA PÁRRAFO

37 1 2 59 1 5

TEXTO AMPLIADO SI LA FUENTE ES OTRA TEXTO DEL MANUAL

Texto en las ampliaciones TÉCNICA EXPRESIVA O GRÁFICA


Test de árbol de Koch (PIR19, 103)
Creado por Koch (1952). Es una variación del anterior. Se
basa en el supuesto de que el dibujo del árbol refleja los sen-
timientos más profundos e inconscientes que el sujeto tiene
PÁG COLUMNA PÁRRAFO de sí mismo.
Se pide que dibuje tres árboles diferentes. Se descarta el di-
71 2 1 bujo si el árbol que dibuja el sujeto es un pino (favorece la ex-
presión de geometrizaciones defensivas). Las raíces informan
sobre la vida instintiva e inconsciente. El tronco simboliza
TEXTO DEL MANUAL el Yo, el carácter del evaluado. La copa representa la vida
mental superior.
EPA-2 (Evaluación del Potencial de Aprendizaje) Es aplicable a partir de los 5 años de edad.
Se aconseja utilizar la EPA-2 para estimar la capacidad de
aprendizaje de niños mayores de 10 años que en un test
tradicional tendría puntuaciones bajas debido a problemas PÁG COLUMNA PÁRRAFO
de aprendizaje, déficit intelectual, diferencias culturales, etc.
También se aconseja en adultos con un nivel educativo bajo 36 1 2
y se quiera saber su potencial de aprendizaje, y en ancianos
con posible deterioro cognitivo o que se quiera conocer su TEXTO DEL MANUAL
plasticidad cognitiva. (PIR19, 101)
CLASIFICACIÓN MORENO ROSET DE LAS ENTREVISTAS

190
CLASIFICACIÓN DE LAS ENTREVISTAS SEGÚN MORENO
ROSSET (2003).
- Entrevistas diagnósticas. Con el fin de evaluar y diagnos-
ticar.
- Entrevistas consultivas. (PIR 19, 104).Pretende dar una res-
puesta a una pregunta en relacióncon un tema específico.
Suele tener un objetivo focal, y no se va a proseguir con un
trabajo clínico posterior.
- Entrevistas de orientación vocacional (PIR 13, 71).
Según las capacidades, intereses,... del sujeto, se pretende
orientarle en qué estudios o qué ámbito profesional es el
que mejor se adecua a él.
- Entrevistas terapéuticas.Para llevar a cabo el tratamiento
mediante entrevistas conel fin de conseguir un cambio acor-
dado con el paciente.
- Entrevistas de investigación. Recoge información con el fin
de si el sujeto se puede adscribir o no a la investigación.

PÁG COLUMNA PÁRRAFO

27 2 1

TEXTO AMPLIADO SI LA FUENTE ES OTRA

Texto en las ampliaciones

PÁG COLUMNA PÁRRAFO

35 2 1

TEXTO AMPLIADO SI LA FUENTE ES OTRA

Texto en las ampliaciones (PIR19, 106)

191
PÁG COLUMNA PÁRRAFO

67 Imagen del cua- Imagen del cua-


dro dro

PÁG COLUMNA PÁRRAFO

44 Tabla 1. Fila 2
Sistema nervioso
autónomo o ve-
getativo.

TEXTO AMPLIADO SI LA FUENTE ES OTRA

Texto en las ampliaciones (PIR19, 108)

192
193
EVALUACIÓN PSICOLÓGICA
AMPLIACIONES

AMPLIACIONES 2. Muestreo de eventos. Sólo se registran conductas que for-


men parte del objeto de estudio. Por ejemplo, si únicamente
nos interesan las posturas de equilibrio en sesiones de activi-
TEMA 3 dad física para personas de tercera edad.

Moreno Rosset distingue dos niveles en el muestreo de tiem- 3. Muestreo temporal. Se elige la información que se registra
po: en función del tiempo. Este puede dividirse en: Muestreo
• Intersesional: en el que se incluiría el periodo de observa- instantáneo o de puntos de tiempo. muestreo de intervalos
ción, periodicidad de las sesiones, número mínimo de sesio- total y muestreo de intervalos parcial. Por ejemplo, si en una
nes y criterios de inicio y fin de sesión (PIR 14, 46). sesión de clase se muestrea cada 30 segundos a un alumno
para saber si se mantiene bien sentado. (PIR19, 105).
• Intrasesional: Información registrada dentro de cada se-
sión. Desglosando este tipo de registro sería: 4. Muestreo focal. Se utiliza cuando se observan varios su-
jetos, observando a cada uno de ellos mediante rondas su-
1.. Registro continuo de toda la sesión. Se registra toda la cesivas. Por ejemplo, en la evaluación de un entrenamiento
información relevante de la sesión según nuestro objetivo. deportivo, cinco minutos de focalización con cada uno de los
Por ejemplo: los desplazamientos mediante una actividad. participantes de forma sucesiva.

TEMA 9
Estas tablas se añaden a la página 67.
Tabla 9: Estructura
Estas tablas del WPPSI-IV
se añaden primera
a la página 67. etapa (2:6 a 3:11 años)
Tabla 9: Estructura del WPPSI-IV primera eta-
pa (2:6 a 3:11 años)

194
.

Tabla 10: Estructura del WPPSI-IV segunda etapa (4 a 7:7


años)

195
EPA-2 (Evaluación del Potencial de Aprendizaje)

Se aconseja utilizar la EPA-2 para estimar la capacidad de


aprendizaje de niños mayores de 10 años que en un test
tradicional tendría puntuaciones bajas debido a problemas
de aprendizaje, déficit intelectual, diferencias culturales, etc.
En general, la evaluación dinámica del potencial de apren-
dizaje es apropiada para determinar el tratamiento en casos
de: sujetos de bajo nivel cognitivo; sujetos que presentan
rendimiento académico inadecuado y que no puede ser
justificado mediante las medidas cognitivas psicométricas
clásicas, teniendo estos niños dificultades para aprendizajes
específicos; y en aquellos casos en que sea preciso clarificar
los procesos motivacionales implícitos en los aprendizajes.
(PIR19, 101).
De forma más concreta se puede aplicar en multitud de ám-
bitos y edades:
- En el ámbito clínico se utiliza para evaluar el poten-
cial de aprendizaje de niños con: afectos de retraso mental,
dificultades de aprendizaje lector, ciegos o con dificultades
de lenguaje.
- En el ámbito educativo ha sido empleado para la
identificación de superdotación entre grupos sociocultural o
lingüísticamente desfavorecidos.
- En el ámbito sociocultural se utiliza en la evaluación
cognitiva de grupos pertenecientes a minorías étnicas (en ca-
sos de inmigración, o grupos especiales).
- En el ámbito laboral se ha usado para la selección
de sujetos que pueden seguir determinado tipo de entrena-
miento laboral.
- En la tercera edad se utiliza para evaluar la plastici-
dad cognitiva de ancianos y frenar el deterioro intelectual.

196
TEMA 5

5.2 Clasificación de las entrevistas.

Página 35. En el apartado de entrevista estructurada se aña-


de como anexo la información respecto a un sistema exper-
to.
Entrevista estructurada. • Centrar el problema/revisión de síntomas. Cuando el
Un sistema experto es un sistema informático que emula entrevistado trivializa con anécdotas es necesario saber en-
el razonamiento humano actuando tal y como lo haría un carar los conflictos.
experto en un área de conocimiento. Si bien no son unas • Paráfrasis o parafraseo. Consiste en que el entrevistador
entrevistas estructuradas, se basan en criterios diagnósticos expresa con sus propias palabras el contenido de lo que ha
de forma bastante estructurada. Tratan de reproducir el al- explicado el entrevistado, para ver si ha comprendido bien lo
goritmo que el clínico haría para llegar al diagnóstico. De que le ha querido decir. (PIR19, 100)
este modo, podemos establecer que los sistemas expertos
son una de las aplicaciones de la inteligencia artificial que Página 38. Antes del apartado 5.6 se añade este texto.
pretende simular el razonamiento humano, de la misma ma- 5.6 Entrevistas clínicas diagnósticas.
nera que lo haría un experto en un área de especialización
(PIR19, 106). Las entrevistas diagnosticas son los instrumentos de evalua-
ción más recomendados para el diagnóstico de trastornos
Página 37. En otras estrategias de intervención verbal se aña- clínicos, el cual se establece en base a criterios categoriales
de este texto: (a diferencia de las escalas clínicas, cuya finalidad es la eva-
• Técnicas de confrontación directa. Hacer tomar con- luación dimensional de la gravedad del trastorno). El carácter
ciencia al entrevistado de sus contradicciones entre lo que estructurado o semiestructurado de las entrevistas diagnosti-
dice y hace o lo que dice en distintos momentos. cas garantiza su fiabilidad. En la tabla 6 se recoge la informa-
• Recordar límites. Presión del tiempo. Cuando el entrevis- ción fundamental sobre las entrevistas más recomendadas
tado no se acomoda en sesiones sucesivas al límite de tiem- para el diagnóstico de los distintos trastornos en adultos y
po es necesario recordárselo. en la tabla 7 para niños (en negrita las más recomendadas).

Tabla 7. Principales características de las entrevistas más recomendadas para el diag-


nóstico clínico en niños.

Tabla 6. Principales características de las entrevistas más recomendadas para el diagnóstico clínico en adultos.

Tabla 7. Principales características de las entrevistas más recomendadas para el diagnóstico clínico en niños.

197
TEMA 6 determinadas categorías
- Conductas ante situaciones de interacción social (e.g., con-
ductas asertivas, nivel de aproximación)
Página 44. Se añade este párrafo sobre la actividad electro-
- Conductas ante interacciones padre/madre-hijo (castigos,
dérmica (tabla 1) Actividad electrodérmica: Los parámetros
refuerzos, aprobación, modelados).
más frecuentemente estudiados en las respuestas de con-
ductancia son: la amplitud de la respuesta (deflexión máxima
Otras técnicas útiles en la conceptualización del caso para los
en comparación con el nivel de conductancia inmediatamen-
trastornos de ansiedad son los autorregistros, los test con-
te anterior) y el tiempo de latencia (periodo transcurrido des-
ductuales o la evaluación conductual análoga. Han mostrado
de el inicio de la presentación del estímulo hasta el comienzo
fuerte validez convergente y discriminativa para la evalua-
de la respuesta). (PIR19, 108)
ción del funcionamiento social en los trastornos de ansiedad.
Los autorregistros presentan como ventajas:
- Proporcionan información única para entender los proble-
TEMA 12 mas de ansiedad. Por ejemplo, la información sobre las situa-
ciones, los objetos o las circunstancias que disparan o indu-
cen la ansiedad así como las señales interoceptivas o alguna
Página 96. En el apartado 12.1. se añade el siguiente texto. de las cogniciones (creencias, expectativas, ideas) asociadas
a la ansiedad pueden también ser evaluadas mediante dia-
Victoria del Barrio distingue tres fases en la evaluación de rios o registros conductuales elaborados ad hoc y que pre-
trastornos psicopatológicos: viamente son diseñados por el evaluador dependiendo de
cada caso.
1) Diagnóstico y delimitación del problema. Como he- - Son buenas herramientas para la conceptualización de los
rramientas diagnósticas, las entrevistas estructuradas son los casos y la planificación del tratamiento, por la versatilidad y
instrumentos de elección. No obstante, algunos cuestiona- adecuación al caso particular.
rios también son utilizados para la obtención del diagnós-
tico desde una perspectiva dimensional, en especial para la Página 100. Justo después del título poblacion adulta y antes
estimación de la intensidad y gravedad de los síntomas (por de la tabla 11, se añade este texto:
ejemplo en el caso de la depresión o el TOC). En la tabla 1 se Los estudios muestran que la depresión es un trastorno que
presentan los instrumentos más recomendados para el diag- tiene dificultades diagnósticas. Se diagnostican muchos ca-
nóstico de los trastornos en adultos. sos, pero también se cometen dos tipos de errores: se infra-
diagnostica (falsos negativos), pero también se sobrediag-
2) Conceptualización del caso (FC en tabla 1 y 2). Se nostica (falsos positivos).
utilizan escalas clínicas para la evaluación dimensional de los Como la depresión es un trastorno interiorizado y la mayor
síntomas y de los factores asociados al trastorno, autorregis- parte de la sintomatología no es conductual, sino sentimen-
tros y técnicas de observación. tal, el acceso mejor que tenemos a los sentimientos proviene
de la información verbal que los sujetos proporcionan sobre
3) Evaluación del progreso y eficacia del tratamiento a sus estados de ánimo. Esto no quiere decir que no se lleven a
través de las mismas escalas empleadas en la conceptualiza- cabo registros de conducta en la evaluación de la depresión
ción (medida post), medidas conductuales o cuestionarios de sino que la mayor parte de los instrumentos que se usan son
funcionamiento. los de lápiz y papel.
Página 99. En la primera columna (después del apartado
ISRA y antes del epígrafe de población infantil) se añade el Los cuestionarios expuestos a continuación tienen unas ca-
siguiente texto: racterísticas psicométricas adecuadas y aceptables. No obs-
Además de las escalas clínicas, la observación conductual es tante, los instrumentos de elección para evaluar depresión
una estrategia muy útil para la conceptualización del caso en adultos son la entrevista SCID (diagnóstico cualitativo) y
en los trastornos de ansiedad. La observación tiene como cuestionario el BDI-II (diagnóstico cuantitiativo).
objetivo la identificación y cuantificación de las siguientes
conductas: Página 101. El texto del apartado 12.5 que está antes de la
- Conductas de evitación ante determinadas tareas (e.g., tabla 13 se sustituye por este (más completo):
porcentaje de pasos de aproximación a un objetivo evitado) La mayoría de las pruebas que existen para la evaluación de
- Juicios subjetivos sobre los niveles percibidos de miedo o este trastorno intentan encontrar el tipo de obsesiones y/o
ansiedad ante determinadas situaciones o tareas. Para ello, compulsiones que manifiesta el paciente. Para el diagnósti-
bien los observadores o los propios sujetos pueden valerse co, se recomienda utilizar las entrevistas estandarizadas ADIS
de escalas de puntuación o registros en los que aparecen o SCID, ya que son los instrumentos que mayor validez ex-
perimental aportan a la exploración inicial de las obsesiones
y compulsiones (la ADIS posee mejores propiedades psico-
métricas). Además, se recomienda evaluar con la Escala de
Yale-Brown (Y-BOCS), que es la más usada para la evalua-
ción de los síntomas del TOC y su severidad, y la Escala de
Pensamientos de Brown (BABS).

198
No obstante, debido a que el TOC es un trastorno con una ser:
incidencia baja y con un gran abanico de síntomas, muchos • Complicaciones médicas y estado físico
terapeutas prefieren realizar la evaluación mediante registros • Análisis específico de síntomas, conductas proble-
creados en el momento junto con un análisis funcional del ma y relaciones funcionales.
caso. • Hábitos alimentarios.
La evaluación de los trastornos obsesivo-compulsivos en • Imagen corporal y satisfacción con el cuerpo.
niños puede hacerse a través de entrevistas clínicas, y me- • Ejercicio físico excesivo o compulsivo.
diante registros de conductas motoras, análisis funcional de • Motivación/resistencia al cambio y conciencia de en
la conducta, cuestionarios y registros fisiológicos. En el caso fermedad.
de los niños es indicado combinar el hetero-registro con el • Cogniciones y actitudes disfuncionales.
auto-registro. • Apoyo y dinámica familiar
• Funcionamiento psicosocial.
• Emocionalidad negativa y regulación emocional
Página 102. Después del título 12.7 y antes de la tabla 15, • Calidad de vida
se añade este texto: Los registros y entrevistas no estructuradas son buenos ins-
Para el diagnóstico categorial del trastorno bipolar se han trumentos para la evaluación inicial del problema y las rela-
recomendado las entrevistas SCID y SADS (para adultos) y ciones funcionales entre el contexto y las variables del sujeto.
K-SADS (para niños). Un método recomendado es la llamada Evaluación Ecológica
Momentánea o Método EMA. Mediante este procedimiento
Página 102. Después del título 12.8 y antes de la tabla 16 se se evalúan las experiencias cotidianas y los estados psicológi-
añade este texto: cos de los individuos en su ambiente natural. Dentro de este
El diagnóstico de la esquizofrenia debe hacerse utilizando método se incluye el muestreo de experiencias y el método
entrevistas clínicas como la SADS, la DIS, la SCID (la más re- cotidiano Diario. Entre las características de este procedi-
comendada) o la CIDI. Desde una perspectiva dimensional, miento, cabe destacar:
las escalas de puntuación y los cuestionarios se han dise- • la evaluación tiene lugar en el contexto natural en el
ñado para la detección y valoración de la sintomatología y que los pacientes desarrollan su vida cotidiana;
su gravedad. Algunas de ellas permiten el diagnóstico de • los pacientes cumplimentan los instrumentos de
las alteraciones mediante el establecimiento de puntos de evaluación en relación con su estado actual (e.g., estado de
corte, aunque su principal objetivo está en el análisis de la ánimo, conductas problemas) más que en relación con se-
sintomatología, su fenomenología e intensidad más que en manas o días anteriores;
el diagnóstico. • Se recogen medidas repetidas a lo largo de un pe-
riodo de tiempo. Mediante este procedimiento, los sesgos
Página 103. Después del título 12.9 y antes de la tabla 17, se del recuerdo quedan más controlados.
añade este texto (sustituye al párrafo después de la tabla 17):
Página 104. Se añade otro apartado en el que se incluiría el
Se han desarrollado bastantes pruebas, en su mayoría au- apartado actual de “Evaluación del Alcoholismo”:
toinformes, para evaluar los trastornos de la alimentación Evaluación de los Trastornos por Consumo de Sustancias
indagando sobre todo en los patrones de ingesta, conductas La primera entrevista debe satisfacer tres objetivos funda-
compensatorias, alteraciones en la imagen corporal, explo- mentales:
ración física, etc. La evaluación de pacientes con TCA suele • Creación de una atmósfera de diálogo
acompañarse de algunas dificultades que pueden interferir • Establecer un marco de referencia para contextua-
la delimitación del problema y su posterior conceptualización lizar la relación entre terapeuta y paciente que implique la
y evaluación del tratamiento. Entre estas dificultades, se han definición de las normas (confidencialidad), tiempos y condi-
señalado: ciones durante todo el proceso de intervención
a) Negar o minimizar la sintomatología (PIR 16, 28; • Analizar la demanda, entendida como la valoración,
PIR 17, 90), bien por falta de conciencia del problema, bien compartida con el paciente, de lo que pide, de cómo lo pide
por vergüenza o miedo a la evaluación. Por esta razón se re- y de la respuesta que se le va a ofrecer, remarcando las posi-
comienda mantener un clima cooperativo, evitar las críticas bilidades y los límites de la intervención.
y la confrontación y hacer preguntas con formato abierto Siempre que sea posible, se debe contrastar la información
(«¿hábleme un poco más sobre el tipo de dieta que sigue?») obtenida por el paciente con la que aportan familiares y
que abran preguntas más específicas («¿en qué situaciones otras personas de su entorno social, ya que completan el
ocurren los vómitos?»); análisis de la demanda muchas veces sesgado, que el indi-
b) Sesgos en el recuerdo, derivados de los problemas viduo posee de su trastorno, pues frecuentemente los dro-
de memoria y atención causados por el estado de desnutri- godependientes tienden a minimizar su problema y la dosis
ción o por los estados emocionales negativos. Para amorti- consumida, restar importancia a otras drogas que consumen
guar estas dificultades se recomienda evaluar retrospectiva- e infravalorar los problemas familiares, laborales, somáticos
mente períodos de tiempo específicos (e.g., semanas) hasta y psicopatológicos asociados.
abarcar, al menos un periodo de 12 semanas. La realización de los autorregistros ayudará al paciente a to-
c) Evaluación de niños y adolescentes que requieren mar conciencia de su comportamiento mecánico y, además,
el uso de estrategias de evaluación adaptadas a su nivel de puede registrar y cuantificar objetivamente las variables que
comprensión: uso de metáforas o analogías, vocabulario controlan su conducta adictiva. Es necesario que fijemos
sencillo, uso de dibujos o material pictográfico, entre otras previamente los parámetros concretos que el paciente debe
observar y registrar (frecuencia, situación, grado de deseo,
Las variables a explorar en la evaluación de los TCA deben etc.).

199
Esto es, las condiciones en las que ocurre y las consecuen-
cias que le siguen, siendo posible que la conducta empiece
a modificarse por el hecho mismo de estarla observando,
fenómeno que se conoce por el nombre de «efectos reac-
tivos»; no obstante, la reactividad de la autoobservación es
un fenómeno frecuente, aunque sus efectos son poco du-
raderos. Los autorregistros son especialmente utilizados en
tabaco y juego.

Página 105. Al final de la columna de la derecha se añade


este texto:
Otro instrumento para la evaluación del autismo infantil es el
Cuestionario de Comunicación Social SCQ, (Berument, Rut-
ter et al., 1999; ed. española, Rutter, Bailey y Lord, 2005).
Es un cuestionario aplicable a padres tiene la ventaja de una
corta duración (10-12 minutos), pero, sin embargo, tiene
una fiabilidad y validez limitada y es necesario usarlo sólo
como medio de despistaje para pasar posteriormente una
evaluación más específica.
En la figura 1 se presenta un resumen respecto a los instru-
mentos más relevantes en la evaluación de trastornos en la
infancia.

200
RESEÑAS

201
PSICOBIOLOGÍA
RESEÑAS

PÁG COLUMNA PÁRRAFO PÁG COLUMNA PÁRRAFO

131 Tabla 4 Columna 2 “CIR- 128 1 12


CUNVOLUCIÓN
ANGULAR Y SU-
PRAMARGINAL” TEXTO AMPLIADO SI LA FUENTE ES OTRA

TEXTO DEL MANUAL Texto en las ampliaciones

14.4. Lenguaje
Comprensión lectora
Procesamiento del significado de la palabra escrita
Integración de estímulos visuales y auditivos: conversión gra-
fema-fonema Centro el lenguaje
PÁG COLUMNA PÁRRAFO

144 2 6
TEXTO AMPLIADO SI LA FUENTE ES OTRA

Texto en las ampliaciones TEXTO DEL MANUAL

Patrones de respuesta
Corteza prefrontal.
La corteza prefrontal ventromedial inhibe la adquisición
PÁG COLUMNA PÁRRAFO del miedo (PIR 16, 07). Es clave en la memoria de extinción
en las respuestas de miedo a largo plazo, inhibe la expresión
49 1 4 del miedo (aunque determinadas regiones de ésta también
la favorecen), participa en el condicionamiento instrumental,
planificación y atención + (PIR19, 22)
TEXTO DEL MANUAL

6.2. Encéfalo
El hipocampo se encuentra alrededor de la parte interna del
prosencéfalo.
Está conectado al resto de estructuras cerebrales, mediante
un conjunto de axones que recibe el nombre de fórnix (PIR
PÁG COLUMNA PÁRRAFO
19, 19).
76 2 3

PÁG COLUMNA PÁRRAFO TEXTO DEL MANUAL

Alteraciones en la diferenciación sexual


146 1 1
Síndrome del conducto müleriano persistente
Tiene lugar en varones genéticos (XY) por la incapacidad de
producir hormona antimülleriana o por la ausencia de recep-
TEXTO AMPLIADO SI LA FUENTE ES OTRA tores para ésta.

Texto en las ampliaciones Tiene lugar el efecto masculinizante de los andrógenos, pero
no ocurre el efecto desfeminizante, por lo que la perso-
na presenta órganos sexuales internos de ambos sexos +
(PIR19, 23)

202
PÁG COLUMNA PÁRRAFO PÁG COLUMNA PÁRRAFO

18 2 3 141 2 1

TEXTO DEL MANUAL TEXTO DEL MANUAL

1.3. Transmisión del impulso nervioso Base neural del Sueño Rem.
Comunicación intraneuronal El Área Peribraquial se prolonga en un núcleo, localizado
Estos mecanismos de regulación son pasivos, puesto que no más centralmente, que se llama Formación Reticular Pontina
requieren Medial (FRPM), que participa también en el desencadena-
gasto de energía y son complementados por otro de carácter miento
activo denominado bomba de sodio-potasio, que expulsa del sueño REM. (Ver figura 5).
iones La FRPN envía proyecciones a las neuronas colinérgicas del
de Na+ al exterior a la vez que introduce iones de K+ al in- prosencéfalo basal para activarlas y generar así la activación
terior + (PIR19, 24) de la corteza (PIR 19, 26). Luego, el área peribraquial excita
la vía PGO (protuberancia geniculada occipital) para ocasio-
nar ondas PGO y movimientos oculares. Por último, la FRPM
provoca la atonía del sueño REM a través de una vía que
envía un input al Núcleo subcoeruleus, localizado justo de-
PÁG COLUMNA PÁRRAFO trás de ella. El núcleo subcoeruleus excita al Núcleo Magno-
celular del Bulbo Raquídeo, que manda proyecciones a las
neuronas motoras espinales
20 1 1 para inhibirlas, de modo que se consiga la parálisis durante
el periodo de sueño REM. Si se inhibe el locus coeruleus y el
rafe se inducirá el sueño REM (PIR 03, 97).
TEXTO DEL MANUAL

(a continuación de) La sustancia neurotransmisora, que es


liberada al espacio sináptico mediante exocitosis.
PÁG COLUMNA PÁRRAFO

141 1 2
TEXTO AMPLIADO SI LA FUENTE ES OTRA

Texto en las ampliaciones TEXTO DEL MANUAL

Base neural del Sueño No Rem.


Tiene un papel importante el área preóptica ventrolateral
PÁG COLUMNA PÁRRAFO (APVL), situada en el hipotálamo anterior. (PIR 19, 27).
En ella existen neuronas gabaérgicas que proyectan hacia
19 2 18 el núcleo tuberomamilar, la protuberancia dorsal, los nú-
cleos del rafe y el locus coeruleus, inhibiéndolos. Además,
recibe aferencias inhibitorias de las mismas regiones que
TEXTO DEL MANUAL inhibe, conformando así un circuito de inhibición recíproca
llamado “oscilador flip-flop” que permite alternar sueño y
Comunicación interneuronal vigilia. Este oscilador se estabiliza mediante un conjunto de
neuronas hipocretinérgicas del hipotálamo lateral. Cuando
este mecanismo oscilador falla, se produce la narcolepsia.
También se ha constatado que la adenosina (sustancia que
se produce tras la actividad cortical) (PIR 17, 212) actuando
en el Prosencéfalo Basal como neurotransmisor inhibitorio,
induce sueño.

PÁG COLUMNA PÁRRAFO

33 2 3

TEXTO DEL MANUAL


*Donde pone Canales de Ca+ dependientes de voltaje + (PIR19, 25)
Estabilizadores de ánimo (eutimizantes).

203
Los estabilizadores están especialmente indicados para el trastornos de conducta y en algunos casos en el trastorno
tratamiento de los trastornos afectivos, en especial del tras- bipolar, entre otros.
torno bipolar. Los más importantes son:
• Litio (Plenur).
Se ha demostrado eficaz para el tratamiento agudo de epi-
sodios
maniacos, en prevención de recaídas en el trastorno bipolar, PÁG COLUMNA PÁRRAFO
disminuyendo el riesgo de suicidio (PIR 99, 109;
PIR 09, 139). Presenta un estrecho margen terapéutico, 33 2 2
por lo que es necesario realizar controles periódicos de los
niveles de litio (litemias) por su riesgo de toxicidad. Presenta
efectos secundarios renales, neurológicos y digestivos. Aún TEXTO DEL MANUAL
así, sigue siendo el fármaco más utilizado para el trastorno
bipolar (PIR 16, 129). Antipsicóticos atípicos.
• Ácido valproico (Depakine). Estos fármacos son antagonistas tanto de la DA como de
Es utilizado principalmente cono antiepiléptico, para evitar 5-HT. Se presentan como una alternativa a los antipsicóticos
las convulsiones. Presenta reacciones adversas sobre todo clásicos, ya que producen menos efectos secundarios que
neurológicas, como sedación. (PIR 19, 28). estos, y siguen siendo eficaces contra los síntomas positivos.
• Otros. No provocan síntomas extrapiramidales ni alteraciones rela-
Carbamacepina (Tegretol) (PIR 02, 54), que fue el primer cionadas con la prolactina (PIR 15, 220). Además, provoca
anticonvulsionante que mostró su posible utilidad para el cierta mejoría al mejorar la activación de la vía mesocortical.
tratamiento del trastorno bipolar; Topiramato (Topamax), Los efectos adversos que pueden aparecer por el uso de
Oxcarbazepina (Trileptal). fármacos antipsicóticos atípicos son: aumento de peso, ta-
quicardia, hipotensión, xialorrea (o sialorrea, que consiste en
exceso
de salivación) o sedación.
• Altas dosis de Risperidona puede provocar hiperprolacti-
PÁG COLUMNA PÁRRAFO nemia.
• La Clozapina es considerado el más eficaz en esquizofre-
33 2 2 nias resistentes.
El problema de este fármaco es una posible alteración en
la síntesis de los glóbulos blancos de la sangre denominada
TEXTO DEL MANUAL agranulocitosis, que puede ser mortal si no se detecta a
tiempo. Aparece hasta en un 2% de pacientes.
Antipsicóticos atípicos. Fármacos antipsicóticos atípicos:
Estos fármacos son antagonistas tanto de la DA como de Clozapina (Leponex) (PIR 97, 177; PIR 01, 169) y Olanzapi-
5-HT. Se presentan como una alternativa a los antipsicóticos na (Zyprexa) (PIR 17, 154), derivados de dibenzodiacepinas;
clásicos, ya que producen menos efectos secundarios que Risperidona
estos, y siguen siendo eficaces contra los síntomas positivos. (Risperdal) y Ziprasidona (Zeldox), derivado de los ben-
No provocan síntomas extrapiramidales ni alteraciones rela- zoxioxazoles;
cionadas con la prolactina (PIR 15, 220). Además, provoca Quetiapina (Seroquel) (PIR 18, 150; PIR 19, 39) derivado
cierta mejoría al mejorar la activación de la vía mesocortical. de dibenzotiacipina, y Sulpirida (Dogmatil), derivado de las
Los efectos adversos que pueden aparecer por el uso de benzamidas.
fármacos antipsicóticos atípicos son: aumento de peso, ta- Son indicados para tratamiento de trastornos psicóticos,
quicardia, hipotensión, xialorrea (o sialorrea, que consiste en trastornos de conducta y en algunos casos en el trastorno
exceso bipolar, entre otros.
de salivación) o sedación.
• Altas dosis de Risperidona puede provocar hiperprolacti-
nemia.
• La Clozapina es considerado el más eficaz en esquizofre-
PÁG COLUMNA PÁRRAFO
nias resistentes.
El problema de este fármaco es una posible alteración en
la síntesis de los glóbulos blancos de la sangre denominada 33 2 2
agranulocitosis, que puede ser mortal si no se detecta a
tiempo. Aparece hasta en un 2% de pacientes. (PIR 19, 34).
Fármacos antipsicóticos atípicos:
Clozapina (Leponex) (PIR 97, 177; PIR 01, 169) y Olanzapi-
na (Zyprexa) (PIR 17, 154), derivados de dibenzodiacepinas;
Risperidona
(Risperdal) y Ziprasidona (Zeldox), derivado de los ben-
zoxioxazoles;
Quetiapina (Seroquel) (PIR 18, 150), derivado de dibenzo-
tiacipina, y Sulpirida (Dogmatil), derivado de las benzamidas.
Son indicados para tratamiento de trastornos psicóticos,

204
PSICOBIOLOGÍA
AMPLIACIONES
PÁG COLUMNA PÁRRAFO PÁG COLUMNA PÁRRAFO
AMPLIACIONES
20 1 1 131 Tabla 4 Columna 2 “CIR-
CUNVOLUCIÓN
ANGULAR Y SU-
TEXTO AMPLIADO SI LA FUENTE ES OTRA PRAMARGINAL”

La zona de liberación de la membrana presináptica contiene TEXTO AMPLIADO SI LA FUENTE ES OTRA


canales de calcio dependientes del voltaje. Cuando la mem-
brana de la terminal nerviosa se despolariza por la llegada Habría que añadir el contenido de la pregunta aquí:
de un potencial de acción, los canales de calcio se abren. La
entrada de calcio en el botón terminal permite la liberación Memoria verbal (p.ej “hechos aritméticos”, operaciones au-
de los neurotransmisores. (PIR19, 25) tomatizadas
(PIR 19, 18))
(GABRIEL RÓDENAS PEREA. Hospital Virgen del Rocío, SevI-
lla) TEMA 1 BASES NEURALES DEL PROCESAMIENTO NUMÉRICO Y
lla) TEMA 1 DEL CÁLCULO.

Numerosos estudios señalan que el lóbulo parietal es la re-


PÁG COLUMNA PÁRRAFO gión de mayor relevancia en el procesamiento numérico y
del cálculo. No obstante, el correcto desempeño de esta
compleja función involucra un circuito fronto-temporal. Di-
128 1 12
ferentes subtareas serán procesadas en distintas regiones.

TEXTO AMPLIADO SI LA FUENTE ES OTRA Lóbulo parietal


Surco intraparietal
El surco intraparietal proporciona la representación interna
Habría que añadir el contenido de la pregunta:
de las cantidades y la relación existente entre estas. Esto
La PKM-zeta es una enzima que interviene en la PLP-LD
incluye el procesamiento abstracto de magnitudes, ya sean
(larga duración). El ARN mensajero del gen que codifica la
estas o no numéricas, contribuyendo a la cuantificación y a
enzima PKM-zeta se produce continuamente en el núcleo y
la estimación de magnitudes. Esta región está involucrada
se transporta a las espinas dendríticas, donde su traducción
en tareas de cálculo aproximado y en tareas de cálculo no
está bloqueada por la acción de otra enzima, la Pin 1.
mediadas por el lenguaje.
La entrada de calcio en la espina dendrítica que se produ-
Giro angular
ce en la PLP, va a activar otras enzimas, que se unen a la
Esta región, especialmente del hemisferio izquierdo, está in-
Pin 1 inactivándola, permitiendo así la síntesis de PKM-zeta
volucrada en aquellas tareas de procesamiento numérico y
libremente. La PKM-zeta tiene a su vez la capacidad de su-
cálculo que requieren un procesamiento verbal. Al contrario
primir la Pin1 asegurándose también su propia producción.
que el surco intraparietal, su activación es mayor en tareas
Además, activa la enzima NSF (proteína reguladora del tráfi-
de cálculo exacto y concretamente en los denominados “he-
co) que causa el traslado de receptores AMPA lateralmente
chos aritméticos”. Los “hechos aritméticos” son operacio-
desde el tallo dendrítico a la membrana postsináptica de la
nes matemáticas simples y automatizadas que se almacenan
espina dendrítica. Los receptores AMPA controlan los cana-
en la memoria verbal. Un ejemplo podrían ser las tablas de
les de sodio; así, cuando estos son activados por el glutama-
multiplicar (“2x2=4”) o adiciones de pequeñas cantidades
to, producen PEPS en la membrana de la espina dendrítica.
(“5+10=15”).
Con mayor cantidad de receptores AMPA, la liberación de
Sistema parietal posterior superior
glutamato por las terminales nerviosas causa un potencial
Está implicado en procesos atencionales de tipo espacial de
excitador postsináptico mayor. Así, las sinapsis se hacen más
apoyo al cálculo. Se trata de una región multimodal, que
fuertes.
participa en una gran variedad de tareas visuoespaciales y de
(PIR 19, 21)
memoria de trabajo espacial.
(BERTA PINILLA SANTOS. Psiquiatría ámbito privado, Madrid)
Lóbulo frontal
TEMA 14
La corteza prefrontal lateral y ventral participa en el procesa-
miento del cálculo proporcionando el mantenimiento provi-
sional de los resultados intermedios (memoria de trabajo), la
planificación y ordenación temporal de los componentes de
las tareas, la comprobación de resultados y la corrección de
errores. Se puede decir, que la corteza prefrontal es predo-
minante en aquellas tareas aritméticas complejas que requie-

205
ren mayores recursos atencionales, de planificación y memo-
ria de trabajo. Respecto al neurodesarrollo, se ha observado
una progresiva especialización parietal, de modo que inicial-
mente hay un mayor uso de áreas prefrontales para la reso-
lución de cálculos aritméticos en consonancia con una mayor
dependencia de recursos atencionales y de memoria para la
realización de dichas tareas en edades tempranas.

(IRENE ALONSO FERNANDEZ. Hospital Universitario Central

de Asturias, Oviedo) TEMA 14.

PÁG COLUMNA PÁRRAFO

146 1 1

TEXTO AMPLIADO SI LA FUENTE ES OTRA

Habría que añadir el contenido de la pregunta aquí:

Reconocimiento emocional
El hemisferio derecho juega un papel más importante que
el izquierdo en este reconocimiento. En este reconocimiento
también participan:

• El sistema auditivo (tono voz).


• El sistema visual (expresión facial).
• La amígdala, que recibe información directamen-
te desde estructuras subcorticales y no desde la corteza de
asociación visual para el reconocimiento de las expresiones
faciales negativas, especialmente expresiones de miedo.
• El surco temporal superior, que interviene en el re-
conocimiento de la dirección de la mirada.
• Los ganglios basales e ínsula, importantes para reco-
nocer la emoción de asco, desagrado o repugnancia (pérdida
de esta capacidad en Corea Huntington y TOC) y experimen-
tar el propio disgusto (PIR 19, 20).

(IÑIGO ALBERDI PÁRAMO. Hospital Clínico San Carlos, Ma-


drid) TEMA 16

206
207
DESARROLLO PSICOLÓGICO
RESEÑAS

lugar de decirnos que sirven para escribir o dibujar.


PÁG COLUMNA PÁRRAFO Centración: Atender de manera selectiva a un aspecto de la
realidad, siendo incapaz de coordinar varias perspectivas o
105 1 1 compensar varias dimensiones de un determinado objeto.
Esto se observa en las famosas tareas de Piaget de conser-
RESEÑAS vación. Por ejemplo, si en una tarea de conservación del nú-
TEXTO DEL MANUAL mero se presentan dos filas con 5 fichas cada una y en una
de las filas las
NIVEL I. MORALIDAD PRECONVENCIONAL (4-10): fichas están más separadas, la mayoría de los niños de 3 o 4
1. Orientación hacia el castigo y la obediencia. Por ejemplo, años que la fila más larga tiene más fichas. El niño se centra
“¿Qué me va a pasar?” Los niños obedecen reglas para evi- únicamente en la longitud de la fila para dar su respuesta.
tar el castigo. Ignoran los motivos de un acto y se concentran (PIR 19, 2).
en su forma física (como el tamaño de una mentira) o en sus
consecuencias, por ejemplo, la cantidad de daño físico. (PIR
19, 1).
2. Propósito instrumental e intercambio. “Hoy por ti mañana
por mí”. Los niños se conforman a las reglas en aras de su
PÁG COLUMNA PÁRRAFO
interés y en consideración a lo que los demás pueden hacer
con ellos. Examinan una acción en términos de las necesida-
des humanas que satisface y distinguen este valor de la 124 1 2
forma física y las consecuencias del acto.

TEXTO DEL MANUAL

Enfoque de los estadios.


PÁG COLUMNA PÁRRAFO Existen diferentes concepciones de “estadio” de unas teorías
a otras. Así Vaillant, por ejemplo, entiende los estadios como
tareas básicas para adaptarse de forma adecuada, según las
63 1 1
edades de establecimiento, consolidación y transición. Por
su parte Keagan propone varias teorías evolutivas y señala
seis fases del desarrollo (incorporativa, impulsiva, imperial,
TEXTO DEL MANUAL interpersonal, institucional e interindividual) que dependen
del
Período de preparación y organización de las operaciones significado que establece el entorno y no tanto de la edad.
concretas (de 2 a 11 años). Así mismo este autor diferencia entre aspectos femeninos y
masculinos.
Yuxtaposición o incapacidad sintética. Por otro lado, pueden diferenciarse dos posturas extremas
El niño yuxtapone en sus representaciones las cosas siendo en relación al significado de estadio:
incapaz de hacer una síntesis coherente y utilizando afirma- • Aquella que considera el estadio como una manera de cla-
ciones fragmentarias e incoherentes que carecen de cone- sificar o definir a los individuos, haciendo hincapié en los
xiones intervalos de vida de un individuo.
causales, temporales y lógicas. Así mismo el niño es incapaz • Las que consideran los estadios como organizaciones lógi-
de integrar objetos en una clase general y por tanto cas de la personalidad, es decir que conciben la estructura de
no puede clasificar. la personalidad como un conjunto de estadios.
Sincretismo. Las diferentes teorías también difieren en la formulación que
Inclinación del niño a percibir los objetos según una visión hacen del concepto de crisis o de acontecimientos vitales y
global y por esquemas subjetivos, buscando analogías entre señalan distintos acontecimientos relevantes. Así podemos
objetos y sucesos sin antes realizar un análisis previo. Una distinguir las siguientes perspectivas:
respuesta sincrética es lo opuesto a una respuesta concreta. • Centradas en el estrés que producen los acontecimientos y
Así si, por ejemplo, le preguntamos a un niño del periodo en las estrategias de afrontamiento de ese estrés.
preoperatorio, en que se parecen un lápiz, un bolígrafo o un • Basadas en el significado que tienen los acontecimientos y
rotulador podría responder, que “en que son objetos”. En en la manera en que influyen las crisis en la construcción del

208
autoconcepto.
PÁG COLUMNA PÁRRAFO
• Centradas en las crisis como el principio de transformación,
transición y desarrollo. La crisis indican momentos relevantes
de cambio en el ciclo 64 1 2
vital.
A continuación nos centraremos en describir los modelos TEXTO AMPLIADO SI LA FUENTE ES OTRA
más
Importantes. Texto ampliado (PIR19, 75)
La teoría psicosocial de Erikson. Como vimos en el tema
de introducción a la psicología del desarrollo, Erik Erikson
(1902-1994), formuló una teoría de 8 estadios del desarrollo
del yo que acontecían desde la infancia hasta la vejez (ver
tabla 11).
A partir de la crisis de identidad del yo que transcurría du-
rante
la adolescencia, los siguientes estadios definidos por Erikson,
aludían a la edad adulta y la vejez:
• Intimidad vs. aislamiento.
• Generatividad vs. estancamiento. (PIR 19, 3).
• Integridad vs. desesperación.

En relación a la generatividad Erikson sostenía que las per-


sonas
funcionan dentro de 3 dominios (ver tabla 12).
En la etapa de integridad vs. desesperación el deseo de en-
vejecer
de forma satisfactoria se contrapone a la ansiedad que pro-
ducen
los pensamientos de pérdida de autonomía y muerte.

PÁG COLUMNA PÁRRAFO

143 2 2

TEXTO DEL MANUAL

Modelos Constructivistas.
• El aprendizaje tiene que ser inductivo, por tanto, surgir a
partir de datos, ejemplos y situaciones particulares. El
alumno se vale de la experimentación y la comprobación de
hipótesis, en lugar de fundamentarse en lecturas o en las
explicaciones del maestro.
Basándose en esta idea, Bruner plantea un método para
favorecer el aprendizaje por descubrimiento en el aula:
- Método del ejemplo-regla.
Consiste en presentar ejemplos particulares para que el
alumno, basándose en ellos, pueda inducir el principio gene-
ral en el que se incluyen (PIR 04, 206).
• El profesor debe promover que sea el propio alumno el que
descubra la estructura de una asignatura mediante el descu-
brimiento guiado. Para ello, debe proponer situaciones
y problemas que inciten al alumno a intentar hallar las solu-
ciones y las respuestas.
Otro aspecto muy destacado de la teoría de Bruner es el con-
cepto de andamiaje. Con este concepto hace referencia al
apoyo eficaz que el adulto le facilita al niño, aquel que se
adecua a sus posibilidades en cada momento y que va cam-
biando en la medida en que el niño adquiere más responsa-
bilidad en la actividad. (PIR 19, 4).

209
DESARROLLO PSICOLÓGICO
AMPLIACIONES

AMPLIACIONES
Para Piaget, el niño preoperatorio es incapaz de comprender
las transformaciones perceptivas de los objetos, construir se-
ries ordenadas de elementos, o clasificar. Estas limitaciones
del pensamiento preoperatorio también condicionan el tipo
de psicoterapia aplicable con niños, teniendo que ajustar las
intervenciones a la edad del mismo. Algunos autores con-
sideran que debido a estas limitaciones ciertos modelos de
terapia como el A-B-C de terapia cognitiva de Ellis no son
aplicables durante el periodo preoperatorio. Estas interven-
ciones requieren utilizar la lógica, clasificar los pensamientos
y atender simultáneamente a varios aspectos de la realidad
(como antecedentes, creencias y consecuencias) por lo que
no serían aplicables antes del periodo de las operaciones
concretas. (PIR19, 75)
Esta visión de las limitaciones de estadio preoperatorio no es
unánime, ya que los trabajos de otros autores desde la pers-
pectiva del procesamiento de la información y desde la teoría
de Vigotsky han concluido que en este periodo el niño posee
una competencia mayor de la que suponía Piaget.

210
211
PSICOLOGÍA BÁSICA
RESEÑAS

RESEÑAS
adictivas. La bajada de intensidad en la reacción provocada
por el estímulo tras muchas presentaciones del mismo, en
PÁG COLUMNA PÁRRAFO este caso cuando el sujeto se encuentra bajo la influencia de
la sustancia adictiva, se entiende como la tolerancia a dicha
208 1 2 sustancia + (PIR19, 164)

TEXTO DEL MANUAL


PÁG COLUMNA PÁRRAFO
8.3. Razonamiento inductivo

Heurístico de accesibilidad o disponibilidad: Tversky y 34 1 4


Kahneman
Este heurístico hace referencia a que la valoración subjetiva
de la probabilidad de ocurrencia de un hecho determinado, TEXTO DEL MANUAL
se incrementa en función de la saliencia cognitiva o acce-
sibilidad de un ejemplo o asociación que se produce en el Los modelos de filtro
sujeto. Modelos de filtro precategorial
O lo que es lo mismo, asignar la probabilidad de ocurren- En los modelos precategoriales, se realiza una selección aten-
cia de un suceso en función de la facilidad con que puede cional temprana, que opera en los momentos iniciales del
recordarse dicho acontecimiento (PIR 97, 23). El cambio se procesamiento de la información atendida.
produce en la probabilidad subjetiva, no así en la estadística • El modelo de filtro rígido de Broadbent.
u objetiva. Por ejemplo, si a un vecino o amigo le toca la lote- Como veíamos en el punto anterior, los experimentos basa-
ría, se produce un incremento en la probabilidad que estimo dos en mensajes dicóticos, le permitió a Broadbent (1926-
de que el mismo hecho me ocurra a mí + (PIR19, 07) 1993) desarrollar la que sería la primera teoría de la atención
atendiendo a su carácter selectivo.
Broadbent utilizó especialmente dos modelos de trabajo en
escucha dicótica:

PÁG COLUMNA PÁRRAFO - El paradigma de seguimiento de Cherry, anteriormente ex-


plicado, en el que se presentan dos estímulos distintos y se le
103 2 1 pide al sujeto que atienda a uno solo.
Con este procedimiento se observaba que la atención es
selectiva, ya que se recordaba perfectamente el estímulo
atendido, pero también que el estímulo no atendido podía
TEXTO DEL MANUAL mantener alguna de sus características sensoriales más grue-
sas en el recuerdo del sujeto, aunque no su significado +
Teorías sobre el aprendizaje no asociativo (PIR19, 165)
Teorías E-R
• Teoría de los procesos oponentes de Solomon y Corbit.
Solomon y Corbit sostienen que todo estímulo produce
un patrón estándar de la dinámica afectiva. Este patrón
supone que la presentación del estímulo genera en el or- PÁG COLUMNA PÁRRAFO
ganismo una reacción afectiva primaria (proceso A), ante la
cual, el organismo de forma automática desencadena un
55 2 7
proceso de signo emocional contrario para compensar (pro-
ceso B) (PIR 93, 239).
El objetivo de este proceso opuesto al que provoca el estímu- TEXTO DEL MANUAL
lo parece ser el de la estabilidad emocional, considerándose
como hemos dicho un mecanismo de tipo homeostático: el Procesos de recuperación
organismo busca este equilibrio generando un proceso que Hipótesis de la especificidad de la codificación
compense cualquier tipo de respuesta emocional de forma Tulving evidencia dos fenómenos incompatibles con la teoría
automática. de la generación-reconocimiento:
La teoría del proceso oponente se ha relacionado profunda- • Pueden darse procesos de generación sin reconocimiento.
mente con los procesos que se dan durante las conductas • En ocasiones, el recuerdo es superior al reconocimiento

212
(PIR 94, 18).
PÁG COLUMNA PÁRRAFO
En contra de la idea de que las palabras tienen su represen-
tación
física fija, propone que existe especificidad de la codificación 56 2 4
de un término en cada situación particular. Cada vez que se
presenta una palabra se codifica de forma específica según
la situación. Sobre esta base, sostiene que le rend miento en TEXTO DEL MANUAL
el recuerdo será mejor cuando se usa el mismo indicio que
en la codificación (PIR 93, 207; PIR 96, 16; PIR 98, 19). + Justo antes de comenzar el punto 3.4.
(PIR19, 167)

TEXTO AMPLIADO SI LA FUENTE ES OTRA

- Fenómeno del olvido inducido por la recuperación (PIR19,


PÁG COLUMNA PÁRRAFO 185):

42 1 2 Practicar la recuperación de cierta información a partir de


una clave de recuperación puede hacer que, posteriormente,
otros contenidos asociados (semánticamente) a esa misma
TEXTO DEL MANUAL clave de recuperación sean más difíciles de recuperar.

Después de:
• Red de control ejecutivo: Controla procesos ejecutivos
como la inhibición de información distractora o la supervi-
sión de la propia conducta.

TEXTO AMPLIADO SI LA FUENTE ES OTRA

MÁS TEXTO EN LAS AMPLIACIONES.

Para evaluar el funcionamiento de estas tres redes atencio-


nales podemos utilizar el Attention Network Test (ANT), apli-
cable en sujetos animales y humanos, tanto adultos como
niños. El ANT se puede utilizar además como estímulo en
pruebas de neuroimágen. En este test, aplicable en unos 30
minutos, el sujeto debe indicar si una flecha que aparece
en pantalla apunta a la izquierda o la derecha, midiéndose
el tiempo de reacción (TR). Esta flecha (estímulo objetivo)
puede aparecer sola, acompañada de flechas que apuntan
en la misma dirección (flanqueo congruente), o de flechas
que apuntan en dirección contraria (flanqueo incongruente).
Adicionalmente, la presentación de la flecha puede ir prece-
dida por un estímulo pista que fija la atención del sujeto en
un punto central, el punto donde aparecerá la flecha (pista
espacial), o dos puntos simultáneamente (pista doble).
La combinación de pistas y estímulos objetivos permite eva-
luar las redes atencionales del siguiente modo:
- Red de alerta: se resta el TR medio en la condición
de pista doble del TR medio en la condición de no pista.
- Red de orientación: se resta el TR medio en la condi-
ción de pista espacial del TR medio en la condición de pista
central. + (PIR19, 166)
- Red ejecutiva: se resta el TR medio de todas las si-
tuaciones de flanqueo congruente del TR medio en todas las
situaciones de flanqueo incongruente.
(Idealmente habría que incluir la imagen que adjunto, pero
ocupa bastante y es de un artículo original, así que no sé si
es posible.)

213
PSICOLOGÍA BÁSICA
AMPLIACIONES

AMPLIACIONES

PÁG COLUMNA PÁRRAFO PÁG COLUMNA PÁRRAFO

42 1 2 103 TEMA 5 2 2 Justo antes de


la Teoría de la de-
presión sinóptica
CAMBIO DE ALGUNA FRASE DEL TEXTO de Kandel.

Ampliación después de:


• Red de control ejecutivo: Controla procesos ejecutivos
como la inhibición de información distractora o la supervi- AMPLIACIÓN
sión de la propia conducta.
· Teoría de la tolerancia a la droga basada en el condiciona-
miento de Siegel
Este modelo atribuye la tolerancia a las respuestas compen-
AMPLIACIÓN satorias condicionadas a los estímulos ambientales empare-
jados con la administración de la droga (PIR19, 164).
Para evaluar el funcionamiento de estas tres redes atencio- Las respuestas condicionadas a la droga podrían ser también
nales podemos utilizar el Attention Network Test (ANT), apli- responsables de la aparición de los síntomas de abstinencia.
cable en sujetos animales y humanos, tanto adultos como Por lo tanto, la respuesta condicionada de abstinencia se de-
niños. El ANT se puede utilizar además como estímulo en bería en parte al emparejamiento de la abstinencia orgánica
pruebas de neuroimágen. En este test, aplicable en unos 30 con ciertos estímulos ambientales.
minutos, el sujeto debe indicar si una flecha que aparece
en pantalla apunta a la izquierda o la derecha, midiéndose
el tiempo de reacción (TR). Esta flecha (estímulo objetivo)
puede aparecer sola, acompañada de flechas que apuntan PÁG COLUMNA PÁRRAFO
en la misma dirección (flanqueo congruente), o de flechas
que apuntan en dirección contraria (flanqueo incongruente). 56 TEMA 3 2 4
Adicionalmente, la presentación de la flecha puede ir prece- Justo antes de co-
dida por un estímulo pista que fija la atención del sujeto en menzar el punto
un punto central, el punto donde aparecerá la flecha (pista 3.4.
espacial), o dos puntos simultáneamente (pista doble).
La combinación de pistas y estímulos objetivos permite eva-
luar las redes atencionales del siguiente modo:
- Red de alerta: se resta el TR medio en la condición AMPLIACIÓN
de pista doble del TR medio en la condición de no pista.
- Red de orientación: se resta el TR medio en la condi- - Fenómeno del olvido inducido por la recuperación (PIR19,
ción de pista espacial del TR medio en la condición de pista 185):
central. (PIR 2020, 166)
- Red ejecutiva: se resta el TR medio de todas las si- Practicar la recuperación de cierta información a partir de
tuaciones de flanqueo congruente del TR medio en todas las una clave de recuperación puede hacer que, posteriormente,
situaciones de flanqueo incongruente. otros contenidos asociados (semánticamente) a esa misma
(Idealmente habría que incluir la imagen que adjunto, pero clave de recuperación sean más difíciles de recuperar.
ocupa bastante y es de un artículo original, así que no sé si
es posible.)

214
PÁG COLUMNA PÁRRAFO PÁG COLUMNA PÁRRAFO

178 TEMA 7 2 2 (Entre el aparta- 55 2 7


do de Schreder y
el de Lang)

AMPLIACIÓN CAMBIO DE ALGUNA FRASE DEL TEXTO


Para poder estudiar la estructura de la experiencia emocional Procesos de recuperación
Nico Hipótesis de la especificidad de la codificación Tulving evi-
H. Frijda propuso 11 leyes que rigen los factores que deter- dencia dos fenómenos incompatibles con la teoría de la ge-
minan la emoción, o sus consecuencias. neración-reconocimiento:
• Pueden darse procesos de generación sin reconocimiento.
1) Ley del significado situacional: las emociones de- • En ocasiones, el recuerdo es superior al reconocimiento
penden del significado que se le dé a la situación. (PIR 94, 18).
2) Ley del interés: las emociones surgen en respuesta a En contra de la idea de que las palabras tienen su represen-
acontecimientos importantes para la persona. (PIR19, 6). tación física fija, propone que existe especificidad de la codi-
3) Ley de la realidad aparente: las emociones se elici- ficación de un término en cada situación articular. Cada vez
tan pro acontecimientos evaluados como reales, con mayor que se presenta una palabra se codifica de forma específica
intensidad cuanto mayor sea el grado de realidad. según la situación, es decir, con claves tanto para el conte-
4) Ley del sentimiento comparado: la intensidad de la nido como para el contexto que se almacenan juntas. Sobre
emoción depende de cómo se evalúe según un marco de esta base, sostiene que el rendimiento en el recuerdo será
referencia. mejor cuando se usa el mismo indicio que en la codificación.
5) Ley del cambio: los cambios positivos o negativos (PIR 93, 207; PIR 96, 16; PIR 98, 19). + (PIR19, 167)
generan emociones del mismo tipo, tanto si son cambios
reales como esperados.
6) Ley de la habituación: los fenómenos de habitua-
ción afectan a la emoción, disminuyendo su intensidad con
el tiempo y la exposición repetida.
7) Ley de la asimetría hedónica: el placer es contingen-
te con el cambio, y desaparece con la satisfacción continua-
da. El dolor en cambio puede persistir si lo hacen las condi-
ciones adversas.
8) Ley de conservación del momento
Emocional: los fenómenos emocionales (en forma de recuer-
dos) mantienen la capacidad de elicitar la emoción a lo largo
del tiempo, a menos que la exposición repetida ocasione ha-
bituación o extinción.
9) Ley de cierre (o gestalt): las emociones tienden a for-
mar una idea absolutista de la realidad (“en blanco y negro”)
10) Ley de control o cuidado de las consecuencias: la
emoción tiende a modularse en función de las consecuencias
que puede tener.
11) Ley de menor carga y mayor beneficio: cuando una
situación puede verse desde distintas perspectivas, se tiende
a percibir de forma que maximice la emoción positiva y mini-
mice la negativa.

215
PSICOLOGÍA DE LA SALUD
RESEÑAS

RESEÑAS

PÁG COLUMNA PÁRRAFO

14 2 AÑADIR APARTA-
DO AL MISMO NI-
VEL QUE “SALUD
Y ENFERMEDAD”

TEXTO AMPLIADO SI LA FUENTE ES OTRA

Texto en las ampliaciones

PÁG COLUMNA PÁRRAFO

15 TABLA 1 Segunda columna


Segunda Fila

TEXTO DEL MANUAL

2.1 Prevención y salud

PREVENCIÓN SECUNDARIA
(PIR 17, 30; PIR 19, 54)

PÁG COLUMNA PÁRRAFO

16 1 5

TEXTO DEL MANUAL

2.2 Conductas de salud y aproximaciones teóricas

Teoría de la acción razonada (Fishbein y Ajzen, 1975).


La intención conductual es el determinante inmediato de la
adhesión (PIR 18, 224; PIR19, 60). La intención conductual,
a su vez, está determinada por nuestras actitudes hacia la
conducta (PIR19, 169) y nuestras normas subjetivas (PIR 17,
33).

216
PSICOLOGÍA DE LA SALUD
AMPLIACIONES

AMPLIACIONES

PÁG COLUMNA PÁRRAFO

14 2 AÑADIR APARTA-
DO AL MISMO NI-
VEL QUE “SALUD
Y ENFERMEDAD”

AMPLIACIÓN

1.3 Perspectivas actuales de los trastornos psicosomáticos

Rol de enfermo
El estar enfermo, dice Parsons, no es sólo experimentar la
condición física de enfermedad. Estar enfermo más bien
constituye un rol social porque implica conductas basadas en
expectativas institucionales y, con frecuencia, ese rol social
se ve reforzado por las normas de la sociedad que correspon-
den a esas expectativas.
El concepto de “rol de enfermo” de Parsons (1951) se basa
en el supuesto de que estar enfermo no es una elección
consciente y deliberada, aunque una enfermedad pueda
producirse como consecuencia de una exposición intencio-
nada a una infección o una lesión.
Los requisitos que el enfoque de Parsons señala que deben
cumplirse para asumir el rol de enfermo son:
- A la persona que enferma se le eximen de sus roles
sociales normales: esto puede hacer que algunas personas se
sientan atraídas por el rol de enfermo.
- La persona enferma no es responsable de su condi-
ción.
- La persona enferma debe recuperar su estado de
salud (PIR19, 59).
- La persona que enferma debe buscar ayuda de ex-
pertos competentes y colaborar en la recuperación.

217
(o viceversa) en una situación concreta, así como el
PSICOLOGÍA SOCIAL
RESEÑAS

RESEÑAS

PÁG COLUMNA PÁRRAFO nivel social que será más utilizado, esto es, qué categorías
sociales específicas, entre las diferentes categorías posibles,
29 TABLA 8 2 serán psicológicamente significativas en una situación deter-
minada.

“Accesibilidad y ajuste como determinantes de la saliencia”


TEXTO DEL MANUAL
Accesibilidad: hace referencia a la disposición de las personas
2.1. Cognición social a utilizar categorías que son centrales, relevantes o útiles, en
HEURÍSTICO DE DISPONIBILIDAD (PIR 15, 146; PIR 19, 7) función de la experiencia pasada, las expectativas presentes,
y los motivos, valores, metas y necesidades actuales.
Ajuste: se refiere a la relación entre las categorías y la reali-
dad externa, y se divide en dos aspectos complementarios:
PÁG COLUMNA PÁRRAFO Ajuste comparativo, que se basa en el principio según el cual
las categorías que van a ser más salientes son aquellas que
31 1 1 maximizan la “razón de meta-contraste”, esto es, que hacen
que las diferencias entre los grupos sean máximas y que las
diferencias dentro del grupo sean mínimas en las dimensio-
nes relevantes de comparación. Ajuste normativo, se refiere
TEXTO DEL MANUAL a que esas diferencias intergrupo y semejanzas intragrupo
deben coincidir con las expectativas normativas o estereo-
2.2. Procesos de atribución tipos de dichas categorías para que éstas sean salientes.
Kelley planteó un modelo de covariación en el que una per- (PIR19, 170)
sona atribuye la causa de una conducta o acción a la per-
sona que realiza la acción, a la entidad u objeto a la cual va
dirigida la acción o a las circunstancias que la rodean. Esta
atribución va a depender fundamentalmente de tres factores
(PIR 97, 59; PIR 19, 168) (ver tabla 13 en la página anterior). PÁG COLUMNA PÁRRAFO

86 2 2

PÁG COLUMNA PÁRRAFO

86 1 3
TEXTO DEL MANUAL

4.1. La conducta intergrupal


La identidad social resulta inadecuada cuando el grupo al
TEXTO AMPLIADO SI LA FUENTE ES OTRA que el individuo pertenece no le proporcione una identidad
social positiva, por lo que la comparación con otros grupos
Más texto en las ampliaciones. resultará negativa. Existen tres estrategias para enfrentarse a
Teoría de la categorización del yo: tal identidad social negativa (PIR 19, 171):
La forma en que nos categorizamos a nosotros mismos y en
que categorizamos a los otros en un contexto social deter-
minará quién va a ser considerado similar y quien diferen-
te, quién va a ser considerado un aliado y quién rechazado PÁG COLUMNA PÁRRAFO
como un extraño en función de la saliencia que tenga una
información u otra. Por tanto, las categorías sociales definen 86 2 2
a quién consideramos como uno de los nuestros (endogru-
po) y a quién como uno de ellos (exogrupo).
Según la Teoría de la categorización del yo (Turner, 1982; TEXTO DEL MANUAL
Turner y cols, 1987) la saliencia de las categorías hace refe-
rencia a las condiciones que conducen a las personas a cate- 3.5. Estrategias de cambio de actitud
gorizarse en un nivel social, con preferencia a uno individual A partir de los planteamientos de McGuire sobre la relevan-

218
cia de la aceptación del mensaje, Petty y Cacioppo (1986)
desarrollaron el “Modelo de la Probabilidad de la Elabora- TEXTO AMPLIADO SI LA FUENTE ES OTRA
ción”. Sus estudios sugieren que la evaluación de un men-
saje persuasivo se lleva a cabo a través de dos formas dife- Se podría añadir como condición la última opción de la pre-
rentes de procesamiento de la información: la ruta central gunta:
y la ruta periférica hacia la persuasión (PIR 96, 79; PIR 97,
53; PIR 99, 32; PIR 19, 172). A través de la ruta central se • La existencia de un contexto de alto estrés.
ha observado que los receptores gastan bastante tiempo y
esfuerzo en la evaluación crítica del contenido del mensaje.
De hecho, se ha constatado que a veces los receptores son
incapaces de cumplir este proceso evaluativo y acaban ba-
sando su decisión de aceptar o no el mensaje en aspectos PÁG COLUMNA PÁRRAFO
periféricos como la racionalidad del mensaje.
18 1 9

PÁG COLUMNA PÁRRAFO


TEXTO DEL MANUAL
75 2 TABLA 9
1.5. Orientación cognitiva
Festinger (1957) abre tres áreas principales de investigación
con el fin de comprobar su teoría de la disonancia cognitiva
TEXTO DEL MANUAL (PIR 04, 133; PIR 19, 175):

3.3. Influencia social y toma de decisiones en pequeños gru-


pos
Cuando una persona se conforma a través de un proceso
de influencia normativa, es previsible que sea su conducta
manifiesta lo único que cambie, y no su convic ción previa,
que continuará manteniéndose en privado; dando lugar a
un proceso de conformidad pública o sumisión (PIR19, 173).

PÁG COLUMNA PÁRRAFO

77 1 5

TEXTO DEL MANUAL

3.3. Influencia social y toma de decisiones en pequeños gru-


pos
Janis (1972, 1977) estudió casos de tomas de decisión po-
líticas y militares catastróficas tomadas en grupos supues-
tamente “muy” inteligentes. Los individuos resultaron víc-
timas de manera extrema de polarización grupal, conocida
como pensamiento grupal. Se caracteriza por una percep-
ción exagerada de la “rectitud moral” de los planteamientos
del grupo y una “visión homogénea y estereotipada” de los
miembros del exogrupo. Ocurre cuando, es un grupo muy
cohesivo o de mentalidad semejante, está tan condicionado
por la búsqueda de consenso que se deteriora su percepción
de la realidad (PIR 18, 27). El proceso se refuerza bajo las
siguientes condiciones (PIR 19, 174):
• Que el grupo sea altamente cohesivo.
• Que esté privado de otras fuentes de información
alternativas.
• Que el líder apoye claramente una determinada op-
ción.

219
PSICOLOGÍA SOCIAL
AMPLIACIONES

tres niveles de abstracción de la categorización del yo (Turner


y cols., 1987):
AMPLIACIONES - Nivel interpersonal: nivel subordinado de abstracción
que define nuestra identidad personal (nos definimos a no-
Añadir después del cuadro de estrategias para enfrentarse sotros mismos en función de lo que nos hace únicos como
a la identidad grupal negativa y antes del apartado 4.2 el individuos en comparación con otros individuos).
rumor - Nivel intergrupal: nivel intermedio de abstracción que
define nuestra identidad social (nos definimos a nosotros
Teoría de la categorización del yo: mismos como miembros de determinados grupos en com-
J. C Turner desarrolló la Teoría de la categorización del yo paración con los miembros de otros grupos).
(Turner, 1982; Turner y cols., 1987). Se trata de una conti- - Nivel interespecies: nivel supraordenado e interespecífi-
nuación y ampliación de la Teoría de la identidad social (Ta- co de abstracción, el yo como ser humano (nos definimos a
jfel, 1982). nosotros mismos como seres humanos en comparación con
Desde la perspectiva de la Cognición Social se señala que otras especies).
la categorización es un proceso cuya finalidad es hacer más Es importante tener en cuenta que dichos niveles de abstrac-
sencillo el procesamiento de la información. Según Hamil- ción no vienen definidos por unos atributos concretos, sino
ton y Troiler (1986), categorizamos a las personas en grupos por el ámbito en el que las personas están siendo categori-
como un medio de reducir la cantidad de información a la zadas y comparadas.
que tenemos que enfrentarnos. De esta manera, se asume Por otro lado, la forma en que nos categorizamos a nosotros
que la simplificación de la realidad, propia del proceso de mismos y en que categorizamos a los otros en un contexto
categorización, conduce necesariamente a una deformación social determinará quién va a ser considerado similar y quien
de dicha realidad. Los humanos, según Fiske y Taylor (1984) diferente, quién va a ser considerado un aliado y quién re-
somos “ávaros cognitivos”, es decir, categorizamos porque chazado como un extraño. Por ejemplo, para un hipotético
tenemos una capacidad limitada de procesar información, francés conservador, un francés socialista puede ser consi-
y esta capacidad cognitiva limitada nos lleva a modificar los derado como “uno de nosotros” si las categorías nacionales
estímulos que nos llegan, lo que puede llevar a distorsiones son salientes, o como “uno de ellos” si lo que destacan son
y errores en los juicios. las categorías políticas. Por tanto, las categorías sociales de-
En los últimos años ha habido un amplio debate entre los finen a quién consideramos como uno de los nuestros (endo-
teóricos de la cognición social y los de la categorización so- grupo) y a quién como uno de ellos (exogrupo).
cial respecto a la relación entre las categorías y la realidad Según la Teoría de la categorización del yo, la saliencia de las
social. Mientras que la cognición social asume que la percep- categorías hace referencia a las condiciones que conducen a
ción categorial es una deformación de la realidad social de- las personas a categorizarse en un nivel social, con preferen-
bido a la complejidad de ésta, la categorización del yo (Tur- cia a uno individual (o viceversa) en una situación concreta,
ner, Hogg, Oakes, Reicher y Wetherell, 1987) propone que así como el nivel social que será más utilizado, esto es, qué
la manera en la que nos categorizamos a nosotros mismos categorías sociales específicas, entre las diferentes categorías
y categorizamos a los demás (para lo cual utiliza el término posibles, serán psicológicamente significativas en una situa-
de categorías sociales) está determinada por las relaciones ción determinada.
sociales que se producen en el contexto social. En lugar de
distorsionar la realidad, las categorías la reflejan siendo el “Accesibilidad y ajuste como determinantes de la sa-
resultado de la interacción entre el perceptor y la realidad liencia”
social (Turner, Oakes, Haslam y McGarty, 1994).
Accesibilidad: hace referencia a la disposición de las personas
En la Teoría de la categorización del yo de Turner, el proceso a utilizar categorías que son centrales, relevantes o útiles, en
básico es que la concepción de uno mismo refleja la cate- función de la experiencia pasada, las expectativas presentes,
gorización del yo, esto es, la agrupación cognitiva de uno y los motivos, valores, metas y necesidades actuales.
mismo como idéntico a alguna clase de estímulos en compa- Ajuste: se refiere a la relación entre las categorías y la rea-
ración con otra clase de estímulos. Como sucede con todos lidad externa, y se divide en dos aspectos complementarios:
los sistemas de categorías naturales, la categorización del Ajuste comparativo, que se basa en el principio según el
yo puede existir en diferentes niveles de abstracción, siendo cual las categorías que van a ser más salientes son aquellas
más inclusivos los niveles superiores (véase Rosch, 1978). Así, que maximizan la “razón de meta-contraste”, esto es, que
una categoría del yo, por ejemplo, “científico”, se conside- hacen que las diferencias entre los grupos sean máximas y
ra más abstracta que otra, por ejemplo, “físico”, en tanto que las diferencias dentro del grupo sean mínimas en las
aquella puede contener a ésta, pero ésta no puede conte- dimensiones relevantes de comparación.
ner a aquella (todos los físicos son científicos, pero no todos Ajuste normativo, se refiere a que esas diferencias inter-
los científicos son físicos). Por tanto, “el yo” es considerado grupo y semejanzas intragrupo deben coincidir con las ex-
como un sistema complejo que puede ser definido en varios pectativas normativas o estereotipos de dichas categorías
niveles de abstracción. para que éstas sean salientes. (PIR19, 170)
En general, la Teoría de la categorización del yo comprende

220
221
CURSO 2020- SIMULACRO 1
1. Un niño, ante la pregunta de por qué no se debe a un aspecto siendo incapaz de coordinar varias perspecti-
pegar a un compañero, contesta: “porque si la vas o compensar varias dimensiones de un objeto. Esto se
profesora te ve, te castiga”. De acuerdo con observa en las tareas de conservación como la descrita en
Kohlberg, ¿en qué nivel de desarrollo moral se la pregunta, en la que ante una tarea de conservación del
encuentra?: número, el niño se centra únicamente en una dimensión,
que es la longitud de la fila, para dar su respuesta (opción 3
1. Nivel preconvencional. correcta). El subperiodo operacional está caracterizado por
2. Nivel instrumental. la capacidad de operar mentalmente con el conocimiento,
3. Nivel formal. poniéndose de manifiesto en la resolución de problemas de
4. Nivel intuitivo. conservación, seriación y clasificación) (opción 2 incorrec-
ta). El período de operaciones formales se caracteriza por
alcanzar un pensamiento abstracto y lógico mediante la for-
Respuesta correcta:1
mulación de hipótesis (opción 1 incorrecta). APIR Manual
de Desarrollo Psicológico, Tema 4.
Kohlberg elaboró una teoría cognitiva sobre el desarrollo
moral sustentada en el modelo de Piaget, basada también
3. De acuerdo con la propuesta de Erik Erikson,
en el uso de dilemas morales que el niño ha de resolver.
en la mediana edad la crisis o desafío que las
La respuesta a estos dilemas pone en evidencia el nivel de
personas han de abordar se denomina:
razonamiento del niño. Este modelo propone tres niveles
de desarrollo moral (preconvencional, convencional y post-
convencional) y divide cada uno de ellos en dos estadios. 1. Intimidad.
En el nivel preconvencional el énfasis está en el control ex- 2. Productividad.
terno, las normas se cumplen para evitar castigos o para ob- 3. Integridad.
tener recompensas (opción 1 correcta). Uno de los estadios 4. Generatividad.
pertenecientes al nivel preconvencional es la orientación
hedonista instrumental e intercambio en la que los niños se Respuesta correcta:4
conforman con las reglas por su propio interés, sin embargo
no es uno de los tres niveles de desarrollo moral propuestos Erik Erikson propone en su teoría psicosocial, desde un
por el autor (opción 2 incorrecta). El nivel formal e intuiti- enfoque de estadios, ocho estadios del desarrollo del yo,
vo no son niveles de desarrollo moral propuestos por este que se desarrollan desde la infancia hasta la vejez. Cada
modelo (opciones 3 y 4 incorrectas). APIR Manual de De- etapa incluye una crisis (aspecto primordial del desarrollo)
sarrollo Psicológico, Tema 6. que debe ser resuelta por la persona para un desarrollo sano
del yo. Además plantea que es necesario que cada etapa
se resuelva con éxito para poder avanzar hacia la siguien-
2. Ante dos filas con el mismo número de mone- te de forma adecuada. Las diferentes crisis o desafíos que
das cada una, un niño cree que la fila más larga propone Erikson se asocian a unos rangos de edad aproxi-
tiene más monedas que la fila más corta. ¿ E n mados en los que resolverlos. El desafío de intimidad vs.
qué etapa de Piaget se encuentra este niño?: aislamiento se sitúa en la edad comprendida entre los 20-40
años (opción 1 incorrecta). La crisis de integridad del yo
1. Operaciones formales. vs. desesperación se produce en la última etapa de la vida,
2. Operaciones concretas. a partir de los 65 años. La crisis de generatividad vs. estan-
3. Preoperacional o intuitiva. camiento se asocia a la mediana edad, entre los 40-65 años
4. Sensoriomotora. (opción 3 correcta). La crisis o desafío de productividad
no es una de las crisis propuestas por Erikson en su teoría.
Respuesta correcta:3
APIR Manual de Desarrollo Psicológico, Tema 2.
Dentro de las diferentes perspectivas teóricas en psicolo-
gía del desarrollo encontramos modelos cognitivos, siendo
Piaget uno de sus autores principales. Piaget describe el 4. Una técnica de instrucción mediante la que
desarrollo humano en torno a los conceptos de funciones alguien más experto proporciona un apoyo
y estructuras cognitivas. Habla de tres componentes en su temporal al niño en aquello que está cercan
teoría (componente estructural, funcional y secuencial). a ser comprendido o en habilidades próximas a
Dentro del componente estructural se conceptualizan los ser alcanzadas por el niño, se denominan:
tres estadios generales de la teoría de Piaget: período sen-
soriomotor, período de preparación y organización de las 1. Aprendizaje vicario.
operaciones concretas y período de operaciones formales. 2. Aprendizaje proximal.
El primer estadio se caracteriza por el logro de la intencio- 3. Zona de desarrollo próximo.
nalidad y la permanencia del objeto (opción 4 incorrecta). 4. Andamiaje.
El estadio de preparación y organización de operaciones
concretas se divide en el subperiodo preoperatorio (mar- Respuesta correcta:4
cado por la aparición de la función simbólica que se ma-
nifiesta en el juego, imitación, dibujo y lenguaje). Una de El andamiaje es un concepto destacado dentro de la teo-
las características del pensamiento preoperatorio es la cen- ría de la instrucción de Bruner y hace referencia al apoyo
tración, que se caracteriza por atender de manera selectiva eficaz que el adulto facilita al niño y que se va adecuando

1
y cambiando en la medida en que el niño adquiere más res- seos de los otros cercanos; son los otros la fuente de acción
ponsabilidad en la actividad y que tiene, por tanto, carácter y motivación; se describen a sí mismas mucho más median-
transitorio, traspasando progresivamente el control al niño te relaciones y roles que con atributos internos (Opción 1
facilitando así su responsabilidad a medida que se progresa correcta). No está demostrado que los sujetos alocéntricos
en la tarea (opción 4 correcta). Este concepto es similar al sean más propensos a experimentar estrés (Opción 2 inco-
de Zona de Desarrollo próximo de Vygotsky, quien también rrecta), aunque sí pueden ser las situaciones ansiógenas de
habla de que los adultos deben ayudar en el aprendizaje diferente índole frente a los sujetos idiocéntricos. Los suje-
del niño para que luego él pueda internalizarlo. El término tos idiocéntricos son más competitivos u orientados al lo-
que recoge esta idea es el de Zona de desarrollo próximo gro, mientras que los alocéntricos se rigen más por valores
o proximal (ZDP) que es la distancia entre el nivel real de comunes en su grupo social, relacionado con la afabilidad
desarrollo del niño (capacidad para resolver un problema y compromiso común (Opción 3 incorrecta). La variable
por sí mismo) y el nivel de desarrollo potencial (capacidad Optimismo no presenta valores diferenciales por pertenecer
para resolver un problema bajo la guía de un adulto), es a culturas individualistas frente a colectivistas (Opción 4
decir, la diferencia entre lo que el niño puede hacer solo y incorrecta). Bermúdez, J. (2011). Psicología de la Persona-
lo que puede hacer con ayuda (opción 2 y 3 incorrectas). El lidad. Madrid:
aprendizaje vicario es un concepto similar al modelado. La UNED.
diferencia entre ambos tiene que ver con el momento del
proceso de aprendizaje en que se pone mayor énfasis, así en 6. Nos enfadamos cuando nuestro equipo de
el modelado se pone mayor atención en el aprendizaje de fútbol pierde una liga, nos alegramos cuando
una conducta y cómo dicho aprendizaje se ve afectado por nuestro mejor amigo consigue el trabajo q u e
las características del modelo y el observador, sin embargo anhelaba y sufrimos cuando alguien querido
en el aprendizaje vicario se pone más énfasis en aspectos nos deja. Para Frijda, la razón que explica
motivacionales relacionados con la percepción de refuerzos estas emociones es la
para la conducta que se está aprendiendo (opción 1 inco-
rrecta). APIR Manual de Desarrollo Psicológico, Tema 9. 1. Ley del Interés.
2. Ley de la realidad aparente.
3. Ley de la conservación del momento
5. Triandis (1995) propone los términos idiocen- emocional.
trismo y alocentrismo para designar 4. Ley del cierre
a las tendencias personales que constituyen
reflejos de la cultura. Algunas diferencias
Respuesta correcta:1
entre los sujetos idiocéntricos y alocéntricos
son que:
Frijda propuso 11 leyes para poder estudiar la estructura de
la experiencia emocional. Cuatro de ellas aparecen en las
1. Los sujetos idiocéntricos tienden hacia la opciones de respuesta: Ley del interés, Ley de la realidad
do minancia mientras que los alocéntri- aparente, Ley de la conservación del momento emocional y
cos muestran niveles más altos de con- Ley del cierre. La ley del interés se refiere al hecho de que
formidad. las emociones surgen en respuesta a acontecimientos que
2. Los sujetos idiocéntricos están más mo- son importantes para la persona (opción 1 correcta); la ley
tivados por el logro mientras que los su- de la realidad aparente sostiene que las emociones aparecen
jetos alocéntricos son más propensos a ante acontecimientos evaluados como reales, con mayor in-
experimentar estrés. tensidad cuanto mayor sea el grado de realidad (opción 2
3. Los sujetos idiocéntricos son más afables incorrecta); la ley de la conservación del momento emo-
mientras que los sujetos alocéntricos son cional hace referencia a que los fenómenos emocionales en
más competitivos. forma de recuerdo permiten elicitar la emoción a lo largo
4. Los sujetos idiocéntricos son más afables del tiempo, a menos que una exposición repetida ocasione
y los alocéntricos más optimistas. habituación o extinción (opción 3 incorrecta); por último,
la ley del cierre afirma que las emociones tienden a formar
Respuesta correcta:1 una idea absolutista de la realidad (opción 4 incorrecta).
APIR Manual de Psicología Básica, Tema 7.
Las personas individualistas se orientan más a conseguir
el éxito, mientras que las colectivistas a la evitación del
fracaso. Por otro lado, las variables más frecuentes en la 7. La alerta alimentaria producida tras un bro-
construcción del self en las culturas individualistas son la te de listeriosis asociada a una marca de carne
percepción de sí mismos como personas independientes, mechada produjo una drástica disminución del
autónomas y completas; acentúan los límites entre el yo y consumo no solo de este tipo de productos sino
los otros, percibiéndose como agentes separados que actúan de otros similares. El juicio del consumidor
para conseguir sus propias metas; su propio self actúa como estuvo afectado por uno de los heurísticos des-
fuente de acción y motivación. Por otra parte, en las cultu- critos por Kahneman y Tversky, el heurístico
ras colectivistas se perciben como interdependientes de los de:
demás, sienten conexión con los miembros del grupo a los
que pertenecen y actúan en sintonía con las metas y los de- 1. Conservadurismo cognitivo.

2
2. Conservadurismo cognitivo. conceptos. Así, en la persona, puede coexistir un autocon-
3. Actitud. cepto ligado al pasado («cómo era yo cuando mis hijos eran
4. Control. pequeños»), otro referido al momento presente («cómo soy
cuando estoy con mi familia»), y otro relativo al futuro
Respuesta correcta:2 («cómo sería mi parte más optimista»). El tercer tipo de
autoconcepto, el que hace referencia al futuro, es denomi-
El conservadurismo cognitivo hace referencia a la tenden- nado por Markus como los posibles yoes. Este tipo de co-
cia de la cognición humana a ser conservadora, ya que in- nocimiento engloba el conjunto de esperanzas, temores y
tentamos preservar aquello que ya está establecido, a man- deseos que resultan relevantes para el individuo; así como
tener nuestros conocimientos, actitudes e hipótesis previas las diferentes cualidades que a la persona le gustaría tener o
(Opción 1 incorrecta). El heurístico de Disponibilidad se podría tener, y aquellas otras ante las que se sentiría mal en
basa en la accesibilidad de cierta información a la memoria el caso de que las tuviera (Opción 4 correcta). Las opcio-
y supone la priorización de características más fácilmente nes 1, 2 y 3 se ofrecen como distractores. APIR Manual de
accesibles ante la evaluación de una situación de tipo so- Personalidad y Diferencial, Tema 14.
cial. En este caso, la alarma creada por el brote de listeriosis
llevó a la disminución del consumo de ciertos tipos de pro-
ductos por la facilidad con la que aparecía la información 9. ¿A qué tipo de covariación entre genes y am-
sobre la posibilidad de enfermar (opción 2 correcta). La ac- biente nos referimos si son los propios padres
titud se define como una tendencia psicológica que implica los que trasmiten tanto el genotipo como el am-
la evaluación de favorabilidad o desfavorabilidad hacia un biente propicio para el desarrollo de un rasgo?:
objeto (opción 3 incorrecta). En cuanto a la opción 4 (in-
correcta), el concepto de control puede ser muy amplio; el 1. Activa.
control cognitivo, que sería el concepto más relacionado 2. Específica.
con las otras opciones de respuesta, es una entidad de natu- 3. Pasiva.
raleza psicodinámica que regula la expresión de las necesi- 4. Reactiva.
dades en modos socialmente adaptativos. Es decir, los con-
troles cognitivos actúan en función de los requerimientos Respuesta correcta:3
de la situación (variables intermedias entre las necesidades
y la percepción). Surgirían a lo largo del desarrollo, fruto Hablamos de correlación pasiva cuando un rasgo se ve fa-
de la interacción entre los genes y el medio ambiente. APIR cilitado tanto por la predisposición genética como por la
Manual de Psicología Social, Tema 2. influencia del ambiente (Opción 3 correcta). Un ejemplo
de este tipo sería cuando los progenitores, que son quienes
aportan los genes que predisponen a un niño a un rasgo,
8. Markus (1977) propone el concepto también suministran el entorno que estimula el desarrollo
de “esquemas del self” que indica de ese rasgo. Las correlaciones activas implican cuando
que, del mismo modo que formamos estructu- el sujeto busca de forma activa el entorno adecuado a su
ras cognitivas sobre otros fenómenos, también contenido genético (opción 1 incorrecta). Las correlacio-
formamos estas estructuras en la relación con nes reactivas implican cuando los sujetos, como reacción a
el self. Respecto a estos esquemas del su genotipo, proporcionan un ambiente adecuado al mismo
self podemos afirmar que: (opción 4 incorrecta). La correlación específica es un dis-
tractor (opción 2 incorrecta). APIR Manual de Personali-
1. Afectan a la rapidez con que procesamos dad y Diferencial, Tema 2.
información relativa a nosotros mismos
pero no influyen en la memoria o recuer- 10. Atendiendo al modelo cúbico de la estructura
do de esta información. del intelecto de Guilford, señale qué facetas de
2. Son generalizaciones cognitivas sobre las dimensiones “operación mental” y “conte-
nosotros mismos por lo que no están nido” intervienen si se le pide a un niño
afectados por las influencias culturales. que diga todos los animales mamíferos que co-
3. Como estructuras cognitivas no están re- noce:
lacionadas con variables afectivas o mo-
tivacionales. 1. Convergente semántico.
4. Representan no solo la consideración ac- 2. Divergente semántico.
tual de nuestro self sino nuestros “selves 3. Convergente simbólico.
posibles” (como lo que creemos que lle- 4. Divergente simbólico.
garemos a ser o lo que nos gustaría llegar
a ser).
Respuesta correcta:1
Respuesta correcta:4 En cuanto a las operaciones mentales, la producción con-
vergente implica la elaboración de respuestas o conclu-
Teniendo en cuenta el marco de referencia temporal de los siones lógicas para alcanzar la mejor respuesta para la
contenidos del autoconcepto, Markus propone una posible información de la que disponemos (la lista de mamíferos
diferenciación entre los diversos tipos de «yoes» o auto- existentes es finita); mientras que la producción divergente

3
implica ofrecer soluciones nuevas y alternativas, relacio- ningún cambio en la personalidad, mientras que con un ho-
nado con la creatividad. Por otra parte, los contenidos se- rizonte temporal de años o décadas (opción 1 incorrecta).
mánticos hacen referencia a constructos mentales a los que Por supuesto, siempre hay que tener en cuenta que la esta-
se aplican palabras (los nombres de animales mamíferos bilidad/cambio en parámetros de personalidad está sujeto a
son contenidos semánticos); mientras que los contenidos la variabilidad individual, (opción 4 incorrecta). APIR Ma-
simbólicos utilizan información en forma de signos que no nual de Personalidad y Diferencial, Tema 7.
tienen significado por sí mismos, como serían letras, mone-
das, números... Teniendo en cuenta lo anterior la única op-
ción correcta es la número 1. APIR Manual de Personalidad 13. Mientras estudias con la ventana abierta oyes
y Diferencial, Tema 4. una canción y tu mente se traslada al verano
y la última fiesta en la que conociste a alguien
muy interesante, de quien no has vuelto a sa-
11. Según el modelo jerárquico de H.J.Eysenck, ber. Tu mente va de una imagen a otra y
señale en qué nivel se sitúa el hecho de que cuando te das cuenta han pasado 10
una persona se enfade habitualmente y minutos. Este proceso refleja, según el
tenga sentimientos de culpabilidad y ansiedad modelo de R. J. Sternberg:
tanto en casa como en el trabajo:
1. Cómo operan los “componentes” o pro-
1. Respuesta estereotipada. cesos elementales de información.
2. Patrón de respuesta. 2. El funcionamiento de los correlatos de la
3. Hábito. inte ligencia.
4. Rasgo. 3. La gran asociación entre capacidad au-
ditiva y memoria, base de las
Respuesta correcta:4 diferencias individuales e n
inteligencia.
En el modelo jerárquico de Eysenck, los rasgos hacen refe- 4. La asociación entre capacidad auditiva y
rencia a constructos que tienen su base en respuestas habi- aten ción focal.
tuales y que tienden a darse en diferentes situaciones o si-
tuaciones que no son percibidas como similares (Opción 4 Respuesta correcta:1
correcta). Por otro lado, los hábitos o respuestas habituales
implican respuestas específicas que sólo se repiten ante cir- Dentro de la teoría Triárquica de Sternberg encontramos
cunstancias similares (opción 3 incorrecta). Las opciones la Subteoría componencial para la inteligencia analítica, la
de respuesta 1 y 2 son distractores, ya que son conceptos cual estudia la relación entre la inteligencia y el mundo in-
que no pertenecen a la teoría de Eysenck. APIR Manual de terno del sujeto, poniendo el énfasis sobre los procesos que
Personalidad y Diferencial, Tema 9. intervienen en el pensamiento. La unidad de análisis es el
“componente”, entendido como proceso básico que actúa
12. Señale qué se puede afirmar, en general, sobre las representaciones internas de los datos recogidos
sobre la estabilidad de la personalidad a lo lar- del exterior. Se encarga de traducir los datos externos en
go del tiempo: representaciones conceptuales (opción 1 correcta). La teo-
ría de Sternberg está claramente centrada en el paradigma
1. La personalidad es menos estable en pe- del procesamiento de la información y pone por ello mucho
riodos cortos de tiempo que consideran- énfasis en el medio externo, el medio interno y la relación
do periodos más largos. entre ambos, pero no indaga en los correlatos anatómicos
2. Es más clara la estabilidad de la inteli- de los procesos cognitivos superiores (Opción 2 incorrec-
gencia que la de la personalidad. ta). Las opciones de respuesta 3 y 4 son distractores. APIR
Manual de Personalidad y Diferencial, Tema 4.
3. Las puntuaciones en los test de perso-
nalidad son muy estables a lo largo del
ciclo vital. 14. Respecto de la relación entre la inteligencia
4. No hay diferencias individuales en la es- (cristalizada y fluida) y la edad, señale la op-
tabilidad durante el desarrollo. ción correcta:

Respuesta correcta:2 1. A partir de la edad en que alcanzan el


máximo nivel, la fluida experimenta un
Como patrón general, la estabilidad de la inteligencia está declive más pronunciado que la cristali-
más demostrada que la estabilidad de la personalidad (op- zada.
ción 2 correcta). Los estudios de rasgos muestran ligeros 2. A partir de la edad en que alcanzan el
cambios en dominancia, mayores en jóvenes, y en cuanto a máximo nivel, ambos tipos de inteligen-
los cinco grandes, los consabidos aumentos de amabilidad cia se mantienen en el tiempo sin varia-
y responsabilidad, y descensos en extroversión, neuroticis- ción.
mo y apertura a la experiencia (opción 3 incorrecta). En 3. El curso de ambos tipos de inteligencias
pequeñas medidas temporales acontecerán menores o casi es de tipo fluctuante, sin que exista un

4
patrón definido de incremento o decre- Modelo de los Siete Grandes de Tellegen y Waller. Si bien
mento asociado a la edad. se correspondería teóricamente con el factor Neuroticismo
4. El curso de ambos tipos de inteligencia del Big Five, sigue siendo una opción incorrecta puesto que
es estable a partir de la adolescencia. el Neuroticismo hemos visto que decrece en los grupos de
edades avanzadas. APIR Manual de Personalidad y Dife-
Respuesta correcta:1 rencial, Tema 10.

La inteligencia fluida alcanza el punto máximo a los 14-15 17. Con relación a las causas de las diferencias
años y sufre un mayor declive con la edad, mientras que la individuales, ¿qué es la heredabilidad amplia?:
inteligencia cristalizada alcanza su punto máximo en torno
a los 20 años y se conserva mejor con la edad (Opción 1 1. La proporción de varianza fenotípica ex-
correcta, opciones 2,3,4 incorrectas). APIR Manual de Per- plica da por la varianza genotípica.
sonalidad y Diferencial, Tema 4. 2. La proporción de varianza genotípica ex-
plica da por la varianza fenotípica.
3. La proporción de varianza fenotípica adi-
15. En el Modelo de los Cinco Grandes, de Mc- tiva explicada por la varianza geno-
Crae y Costa, ¿a qué dimensión pertenece la típica.
faceta de impulsividad?: 4. La proporción de varianza fenotípica ex-
plica da por la varianza genotípica
1. Neuroticismo. aditiva.
2. Extraversión
3. Búsqueda de sensaciones. Respuesta correcta:1
4. Responsabilidad.
Heredabilidad amplia: explica la parte del fenotipo que
Respuesta correcta:1 proviene del genotipo. El resultado será entre 0 y 1. El valor
máximo implicaría que la variabilidad total del comporta-
Según el modelo de los Big Five, los rasgos que componen miento viene determinado por los genes. (opción 1 correc-
el factor Neuroticismo son: ansiedad, depresión, hostilidad, ta). La opción de respuesta 2 es un distractor y no tendría
impulsividad, timidez y vulnerabilidad (opción 1 correcta); sentido puesto que el fenotipo es el resultado de la inte-
y los rasgos que componen el factor Extraversión: asertivi- racción genes-ambiente. La opción de respuesta 3 también
dad, afecto, búsqueda de emociones, actividad, emociones es un distractor, ya que el concepto de varianza fenotípica
positivas y gregarismo (Opciones 2, 3 incorrectas). Los aditiva no existe. El concepto de Varianza genotípica aditi-
rasgos que componen el factores Responsabilidad son: De- va significa: parte de la variabilidad debida a un gen deter-
liberación, autodisciplina, competencia, necesidad de lo- minado en un cromosoma concreto (opción 4 incorrecta).
gro, sentido del deber y orden (Opción 4 incorrecta). APIR APIR Manual de Personalidad y Diferencial, Tema 2.
Manual de Personalidad y Diferencial, Tema 10.

16. En el modelo de los Cinco Grandes, hay una 18. Los “hechos aritméticos” (operaciones
dimensión de personalidad cuyos niveles me- automatizadas que se almacenan en la memo-
dios aumentan consistentemente a lo largo ria verbal) son recuperados de la memoria por:
del ciclo vital. ¿De cuál se trata?:
1. La corteza prefrontal dorsolateral.
1. La responsabilidad. 2. La circunvolución fusiforme.
2. La extraversión. 3. La circunvolución angular.
3. La emocionalidad negativa. 4. El surco sintraparietal.
4. La apertura a la experiencia.
Respuesta co-
rrecta:3 La corteza prefrontal lateral y ventral participa en
Respuesta correcta:1 el procesamiento del cálculo proporcionando el manteni-
miento provisional de los resultados intermedios (memoria
Los estudios sobre estabilidad de los factores que compo- de trabajo), la planificación y ordenación temporal de los
nen el modelo Big Five concluyen que se trata de elemen- componentes de las tareas, la comprobación de resultados y
tos relativamente estables de la personalidad que apenas la corrección de errores (opción 1 incorrecta). El giro fusi-
se ve modificados con la edad. Dentro de los cambios que forme no toma parte en el procesamiento numérico, su fun-
sí se pudieron captar, se aprecia que el Neuroticismo, la ción más destacable está relacionada con el reconocimiento
Apertura a la experiencia y la Extraversión aparecen menos de rostros conocidos, su lesión da lugar a prosopagnosia o
elevados en grupos de mayor edad; mientras que Respon- déficit en el reconocimiento de rostros familiares (opción 2
sabilidad y Cordialidad tienden a aumentar en las etapas incorrecta). La circunvolución angular, especialmente del
de edad más avanzada (opción 1 correcta, opciones 2 y 4 hemisferio izquierdo, está involucrada en aquellas tareas
incorrectas). La opción 3 hace referencia al término “emo- de procesamiento numérico y cálculo que requieren un pro-
cionalidad negativa” que es un concepto perteneciente al cesamiento verbal. Al contrario que el surco intraparietal,
su activación es mayor en tareas de cálculo exacto y con-

5
cretamente en los denominados “hechos aritméticos”. Los 2. La ínsula anterior.
“hechos aritméticos” son operaciones matemáticas simples 3. La unión temporoparietal.
y automatizadas que se almacenan en la memoria verbal. 4. El surco temporal superior.
Un ejemplo podrían ser las tablas de multiplicar (“2x2=4”)
o adiciones de pequeñas cantidades (“5+10=15”) (opción
3 correcta). El surco intraparietal, que no sintraparietal lo Respuesta correcta:2
cual es una errata, proporciona la representación interna
de las cantidades y la relación existente entre estas. Esto El giro fusiforme o circunvolución fusiforme interviene en
incluye el procesamiento abstracto de magnitudes, ya sean el reconocimiento de rostros conocidos, su lesión da lugar
estas o no numéricas, contribuyendo a la cuantificación y a a prosopagnosia o déficit en el reconocimiento de rostros
la estimación de magnitudes. Esta región está involucrada familiares, pero no tiene función específica sobre el reco-
en tareas de cálculo aproximado y en tareas de cálculo no nocimiento de las emociones (opción 1 incorrecta). Tanto
mediadas por el lenguaje (opción 4 incorrecta). APIR Ma- la ínsula como los ganglios basales son estructuras impor-
nual de Psicobiología, Tema 14. tantes para reconocer la emoción de asco, desagrado o re-
pugnancia y experimentar el propio disgusto (opción 2 co-
rrecta). La unión temporoparietal es una zona de asociación
19. La principal vía eferente de la formación hipo- encargada de la integración de diferentes tipos de informa-
campal es: ción tanto externa como interna. Interviene en funciones
relacionadas con la autoconciencia, la Teoría de la Mente
1. El núcleo supraquiasmático. y experiencias extracorporales (opción 3 incorrecta). El
2. La circunvolución dentada. surco temporal superior interviene en el procesamiento de
3. El fórnix. estímulos sociales y está implicada en la percepción social.
4. El campo CA3. El área de Wernicke se encuentra en esta zona y se encarga
en el procesamiento del habla (opción 4 incorrecta). APIR
Respuesta correcta:3 Manual de Psicobiología, Tema 16.

El núcleo supraquiasmático es el principal regulador de los


ritmos circadianos. Está formado por neuronas de peque- 21. ¿Qué proteína es necesaria para el establecimiento
ño tamaño, axones cortos y campos dendríticos pequeños de la potenciación a largo plazo de larga dura-
con una frecuencia espontánea de descarga muy baja, la ción (PLPLD)?:
cual aumenta proporcionalmente según la intensidad de la
luz que llega a la retina. La información de la retina pro- 1. El óxido nítrico liberado por la neurona
yectada a este núcleo procede de las células ganglionares presináptica.
con melanopsina y es transportada a través de una vía di- 2. La entrada de iones calcio y activación
recta o retino-hipotalámica, mediada por la liberación de subsiguiente de enzimas como la CaM-
glutamato, y una vía indirecta o genículo-hipotalámica KII.
mediada por el neuropéptido Y (opción1 incorrecta). La 3. La activación de los receptores de NMDA
circunvolución dentada y el campo CA3 forman parte de activados por ligando y por voltaje.
la formación hipocampal y tienen un papel importante en 4. La enzima PKM-zeta, al facilitar el des-
el aprendizaje a largo plazo. Las principales aferencias y plazamiento de los receptores AMPA a
eferencias neocorticales de la formación hipocampal pasan la membrana terminal.
a través de la corteza entorrinal, cuyas neuronas proyectan
a las células granulosas de la circunvolución dentada, que, Respuesta correcta:4
a su vez, conectan con el campo CA3. Las células de CA3
tienen receptores NMDA y No NMDA. El NMDA es un
El proceso de Potenciación a Largo Plazo (PLP) se divi-
receptor de glutamato de especial importancia en el fenó-
de en dos fases. La PLP inicial (PLP-I) consiste en el si-
meno de la Potenciación a Largo Plazo. La circunvolución
guiente proceso: primero se produce una despolarización
dentada y el campo CA3 son estructuras que componen la
de la membrana que si es suficientemente fuerte activará
propia formación hipocampal por lo que no son vías efe-
los receptores NMDA permitiendo la entrada de iones de
rentes (opción 2 y 4 incorrectas). El fórnix es una comisura
calcio (opción 3 incorrecta). Esto produce la activación de
que en su máximo grosor lleva un millón de fibras y es la
enzimas como la CaM-KII (calcio calmodulina cinasa tipo
principal vía eferente del hipocampo. Es continuación de
II) que influye en el desplazamiento de los receptores de
las eferencias de CA3, CA1 y subículo y por medio de un
AMPA a la membrana postsináptica (opción 2 incorrecta).
viaje circular, subirá sobre el tálamo y llegará a los cuerpos
El óxido nítrico actúa como un mensajero retrógrado a la
mamilares del hipotálamo. (opción 3 correcta). APIR Ma-
membrana presináptica que ayudar a la PLP-I (opción 1
nual de Psicobiología, Tema 6.
incorrecta). En la segunda fase, o PLP de larga duración
(PLP-LD) es necesario la activación de la enzima PKM-ze-
ta. La PKM-zeta activa la enzima NSF (proteína reguladora
20. Resulta esencial para detectar el disgusto en los del tráfico) que causa el traslado de receptores AMPA late-
otros y experimentar el propio disgusto: ralmente desde el tallo dendrítico a la membrana postsináp-
tica de la espina dendrítica. Los receptores AMPA contro-
1. La circunvolución fusiforme. lan los canales de sodio; así, cuando estos son activados por

6
el glutamato, producen PEPS en la membrana de la espina desarrollan los órganos internos femeninos (opción 1 in-
dendrítica. Con mayor cantidad de receptores AMPA, la li- correcta). El gen SRY produce el factor determinante de
beración de glutamato por las terminales nerviosas causa los testículos (una enzima), que provoca la maduración de
un potencial excitador postsináptico mayor. Así, las sinap- las gónadas indiferenciadas hacia testículos. Si el gen SRY
sis se hacen más fuertes (opción 4 correcta). APIR Manual no está presente, se convertirán en ovarios (opción 2 inco-
de Psicobiología, Tema 14. rrecta). El déficit en dihidrotestosterona es el causante del
desarrollo del pene retrasado, una condición que impide a
los varones desarrollarse hasta la pubertad, permaneciendo
22. La extinción de las respuestas emocionales durante la infancia el pene hipodesarrollado y los testículos
condicionadas se relaciona, fundamentalmen- en el interior, dando apariencia de genitales femeninos (op-
te, con la actividad de: ción 3 incorrecta). Por último, sí es cierto que la ausencia
de hormona inhibitoria del sistema de Müller implicaría la
1. La corteza prefrontal ventromedial. no desfeminización de los varones XY que presentan esta
2. La ínsula. característica, disponiendo estos de órganos sexuales inter-
nos de ambos sexos (opción 4 correcta). APIR Manual de
3. La corteza cingulada posterior.
Biopsicología, Tema 11.
4. El núcleo accumbens.
24. Las proteínas responsables de producir y man-
Respuesta correcta:1 tener estables las concentraciones de sodio y
potasio a ambos lados de la membrana y por
La corteza prefrontal ventromedial es clave en la memoria tanto de asegurar la estabilidad del potencial
de extinción en las respuestas de miedo a largo plazo, inhi- de reposo y permitir el futuro potencial de ac-
be la expresión del miedo (aunque determinadas regiones ción son:
de ésta también la favorecen), participa en el condiciona-
miento instrumental, planificación y atención (opción 1 co- 1. Los canales de sodio y potasio pasivos.
rrecta). La ínsula contiene representaciones interoceptivas
2. Los canales de sodio y potasio controla-
a modo de representación del estado del cuerpo, para lo
dos por voltaje.
que se encuentra bien relacionada con estructuras como la
amígdala y el estriado. La ínsula ha sido relacionada con 3. Las bombas de sodio-potasio.
la conciencia interoceptiva y emocional, la empatía y la 4. Las proteínas G.
conducta social cooperativa, así como también ejerce un
papel en la percepción auditiva, gustativa y somatosenso- Respuesta correcta:3
rial (opción 2 incorrecta). La corteza cingulada es el lugar
donde se procesa en última instancia la información sobre El potencial de membrana es la diferencia de carga eléc-
el dolor (opción 3 incorrecta). Por último, el núcleo accum- trica que existe entre el interior y el exterior de la célula.
bens forma parte de los ganglios basales y es responsable Cuando la neurona está en reposo, dicho potencial es de,
de la función reforzante de determinados estímulos sobre aproximadamente, -70 mV (potencial de reposo), es de-
la conducta y también de la función estimulante de las dro- cir que el potencial interior de la neurona es menor que el
gas (opción 4 incorrecta). APIR Manual de Biopsicología, que hay en el exterior de la membrana. Esta diferencia se
Tema 16. debe a la diferente concentración de iones (sales con car-
ga positiva o negativa) en ambos ambientes (intracelular
y extracelular). Los iones que contribuyen al potencial de
23. Durante el desarrollo embrionario, la ausencia reposo son: sodio (Na+), potasio (K+), cloro (Cl-) e io-
de hormona inhibidora del sistema de Müller nes proteínicos de carga negativa. Los iones de Na+ y de
se relaciona con: Cl- presentan una mayor concentración en el exterior de
la célula, mientras que los de K+ se concentran más en el
interior. La mayoría de los iones proteínicos permanecen
1. El síndrome de insensibilidad a los an-
en el interior, donde son formados. Esta distribución des-
drógenos.
igual de iones se debe a tres fenómenos: (a) el gradientes
2. La alteración del gen SRY (sex-determi- de concentración, o desplazamiento de iones desde zonas
ning region Y). de alta concentración hacia zonas de menor concentración;
3. La inhibición en la síntesis de dihidrotes- (b) la presión electrostática, que es la atracción de cargas
tosterona opuestas y repulsión de cargas iguales y (c) la permeabili-
4. No se produce desfeminización. dad selectiva de la membrana celular, que permite el paso
de determinados iones con más facilidad que otros. Estos
mecanismos son pasivos, por lo que no requieren gasto de
Respuesta correcta:4 energía (opción 1 incorrecta). Las bombas de sodio-potasio
son mecanismos activos que tienen como función expulsar
El síndrome de insensibilidad a los andrógenos o síndro- iones de Na+ al exterior al mismo tiempo que introducen
me de Morris consiste en la inexistencia de receptores para K+ y son las encargadas de mantener estables las concen-
los andrógenos, que impide el desarrollo de los órganos se- traciones de ambos iones (opción 3 correcta). Los canales
xuales internos masculinos, la hormona antimülleriana sí de sodio y potasio controlados por voltaje no intervienen en
provoca su efecto desfeminizante, por lo que tampoco se el mantenimiento de las concentraciones de iones sino en

7
los procesos de despolarización e hiperpolarización (opción 5. Respuesta correcta:1
2 incorrecta).El potencial de acción se producirá cuando la
suma de los potenciales excitadores postsinápticos (PEPs) El sueño REM/MOR o sueño paradójico surge tras 90 mi-
y los potenciales inhibidores postsinápticos (PIPs) alcanza nutos de sueño y se produce un cambio brusco en varias
el umbral de excitación. Las proteínas G generan una se- medidas fisiológicas que se registran. Algunas de las carac-
rie de reacciones químicas hasta conseguir que un segundo terísticas de esta fase del sueño son: EEG desincronizado,
mensajero pueda provocar la apertura del canal iónico, ade- ondas Theta acompañadas de movimientos oculares verti-
más de otros posibles efectos (opción 4 incorrecta). APIR cales, ondas PGO, atonía muscular con EMG aplanado por
Manual de Biopsicología, Tema 1. la pérdida de tono muscular y aumento de actividad Beta.
El sueño no REM se caracteriza por la predominancia de
ondas cerebrales lentas, que indican una actividad eléctrica
disminuida en el SNC (opción 4 incorrecta). En cuanto a la
25. En la transmisión sináptica, para que se libere base neural del sueño REM es importante la función de las
con éxito el neurotransmisor en el espacio si-
neuronas colinérgicas del Área Peribraquial del tronco en-
náptico es necesario:
cefálico. Esta área se prolonga en un núcleo, que es la For-
mación Reticular Pontina (FRPM) que participa también
1. Que se transmita un potencial excitatorio en el desencadenamiento del sueño REM. La FRPM envía
post sináptico a lo largo del axón y proyecciones a las neuronas colinérgicas del prosencéfalo
llegue al botón terminal. 2. Que basal para activarlas y generar así la activación de la corte-
se abran los canales de calcio controla za. Después el Área Peribraquial excita la vía PGO (protu-
dos por voltaje, y entre calcio en el botón berancia geniculada occipital) para ocasionar ondas PGO y
terminal. movimientos oculares y la FRPM provoca la atonía propia
3. Que las vesículas que contienen el neu- de esta fase (opción 1 correcta). Por otra parte, es la inhibi-
rotrans misor se fusionen con las zonas activas ción del Locus Coeruleus y el Rafe la que induce al sueño
de la membra- na postsináptica. REM (opción 2 incorrecta). En el sueño también se produ-
4. Que se activen los autorreceptores presi- cen algunos cambios fisiológicos. Con respecto al SNA, en
nápti cos. SNREM la temperatura corporal desciende, mientras que
en el SREM la temperatura corporal se va ajustando a la
temperatura ambiente. Las temperaturas ambientales extre-
Respuesta correcta:2 mas provocan interrupción del sueño, con frecuentes des-
pertares, reduciéndose la cantidad de sueño REM. (opción
La transmisión de un potencial excitatorio postsináptico 3 incorrecta). La respiración en la transición vigilia-sueño
(PEP) permite la despolarización de la neurona, pero úni- suele ser irregular, estabilizándose a medida que el SN-
camente se produce el potencial de acción cuando la suma REM se hace más profundo, mientras que durante el SREM
de los PEPs y los PIPs (potenciales inhibidores postsináp- la frecuencia respiratoria aumenta y la respiración vuelve a
ticos) supera el umbral de excitación (opción 1 incorrecta). ser irregular. La frecuencia cardíaca en SNREM disminuye
La zona de liberación de la membrana presináptica contie- y en el SREM se hace más irregular. APIR Manual de Psi-
ne canales de calcio dependientes de voltaje, que se abren cobiología, Tema 15.
cuando la membrana de la terminal nerviosa se despolariza
con la ocurrencia de un potencial de acción, permitiendo la
entrada de calcio y la consecuente liberación de los neuro- 27. El área preóptica ventrolateral (APOvl):
transmisores (opción 2 correcta). Las vesículas liberan por
medio de exocitosis el neurotransmisor al espacio sináptico 1. Es una región promotora del sueño de
(opción 3 incorrecta). Los autorreceptores presinápticos se ondas lentas
suelen encargar de controlar la cantidad de neurotransmi- 2. Facilita el aprendizaje relacional durante
sores presentes en la sinapsis (opción 4 incorrecta). APIR el sueño REM.
Manual de Biopsicología, Tema 1. 3. Se estimula por las neuronas orexinérgi-
cas del hipotálamo lateral.
26. El sueño REM: 4. Inhibe la secreción de GABA en el en-
céfalo.
1. Está producido por la estimulación coli-
nérgica de la formación reticular pontina. Respuesta correcta:1
2. Está producido por la estimulación de las
neuronas serotoninérgicas del núcleo del El Área Preóptica Ventrolateral (APVL), situada en el hipo-
rafe. tálamo anterior, tiene un papel importante en la base neural
3. Está asociado a una disminución de la del sueño No REM o sueño de ondas lentas (opción 1 co-
rrecta). En ella existen neuronas gabaérgicas que proyectan
temperatura corporal y del consumo de hacia el núcleo tuberomamilar, la protuberancia dorsal, los
energía. núcleos del rafe y el locus coeruleus, inhibiéndolos (opción
4 incorrecta). Además, recibe aferencias inhibitorias de las
4. Presenta un EEG lento y de alto voltaje. mismas regiones que inhibe, conformándose así un circui-
to de inhibición recíproca llamado “flip-flop” que permite
alternar sueño y vigilia y que se estabiliza mediante un con-

8
junto de neuronas hipocretinérgicas del hipotálamo lateral. tratamiento con antipsicóticos típicos como el Haloperidol
Las hipocretinas/orexinas son neuropéptidos sintetizados (opción 1 incorrecta). Además, el manejo inicial de las al-
por un pequeño grupo neuronal localizado en el hipotála- teraciones de conducta y la agitación puede requerir añadir
mo posterolateral. El sistema hipocretinérgico/orexinérgi- benzodiacepinas a corto plazo (opción 3 incorrecta). APIR
co muestra una gran actividad durante la vigilia, además Manual de Psicobiología, Tema 4.
la deficiencia parcial o total de estos péptidos o sus recep-
tores se asocia a la narcolepsia. Estas neuronas activan di- 29. En relación al tratamiento del trastorno
rectamente la corteza cerebral, activan grupos neuronales afectivo estacional (TAE), señale la opción
que forman parte del Sistema Reticular Ascendente de ac- FALSA:
tivación e inhiben la generación de SREM en el tegmento
Pontino Ventral (opción 3 incorrecta). La adenosina actúa 1. La fototerapia es más eficaz si es admi-
en el Prosencéfalo Basal como neurotransmisor inhibitorio nistrada por la mañana.
y también conduce al sueño. Mientras que el SNREM in- 2. La hipersomnia y la avidez por hidratos
terviene en el adecuado descanso, el SREM interviene en el de carbono son predictores de buena res-
desarrollo cerebral y consolidación del aprendizaje (opción puesta a la fototerapia.
2 incorrecta). APIR Manual de Psicobiología, Tema 15. 3. La intensidad de la luz en la fototerapia
deberá estar siempre por debajo de los
28. En relación al tratamiento de la fase maníaca 8500 lux.
en el trastorno bipolar, ¿cuál de las siguientes 4. Es importante que la luz contenga poca energía
afirmaciones es correcta?: del espectro ultravioleta para minimizar los efec
tos secundarios de la fototerapia.
1. En caso de ser necesario prescribir un
antipsicótico, se preferirá el uso de an- Respuesta correcta:4
tipsicóticos atípicos ya que estos ayu-
darán a producir un viraje hacia la fase El mecanismo de acción de la fototerapia consiste, en par-
depresiva. te, en disminuir la depleción de triptófano presente en los
2. Se deberá esperar un máximo de 48 pacientes con TAE. La dosis inicial de administración debe
horas para determinar los cambios en las de ser de 1520 min al día y gradualmente ir aumentando
con centraciones séricas de litio tras una hasta 30-45 min al día a 10000 lux (unidad photométrica)
modificación de dosis. (opción 3 correcta). Los estudios sobre la hora de adminis-
tración de la luminoterapia han encontrado resultados con-
3. La utilización de cualquier benzodiace- tradictorios. Parece que las sesiones por la mañana produ-
pina está contraindicada en esta fase del cen mejores resultados que las administradas por las tardes,
trastorno. aunque estas sí son superiores al placebo. Gatón Moreno,
4. El tratamiento con ácido valproico se M. A., González Torres, M. Á., & Gaviria, M. (2015). Tras-
considera una alternativa eficaz en pa- tornos afectivos estacionales,” winter blues”. Revista de la
cientes que no respondan al tratamiento Asociación Española de Neuropsiquiatría, 35(126), 367-
con litio. 380.

Respuesta correcta:4 30. La Terapia interpersonal ha sido ampliamente


aplicada en el tratamiento de la depresión.
Los estabilizadores del ánimo o eutimizantes son fármacos ¿Qué caracteriza a la Terapia Interpersonal?:
especialmente indicados para el tratamiento de los Tras-
tornos Afectivos, especialmente para el Trastorno Bipolar. 1. La terapia interpersonal hace hincapié en
Los eutimizantes más relevantes son el Litio (Plenur), el detectar pensamientos automáticos inter-
Ácido Valproico (Depakine) y otros como Topiramato, Ox- personales.
carbazepina y Carbamacepina (que fue el primer anticon- 2. Enfatiza el papel de las experiencias in-
vulsivante que mostró su posible utilidad para el Trastorno terpe sonales de la niñez.
Bipolar). El Litio tiene un estrecho margen terapéutico por 3. Focalizarse en cambiar la personalidad
lo que es necesario realizar litemias periódicamente por su que está generando los conflictos inter-
riesgo de toxicidad. Los valores de dosis en sangre ade- personales.
cuados oscilan entre 0,6-0,8 mili/equivalentes por litro. La 4. Se basa en el modelo médico de la enfer-
litemia se realiza aproximadamente 12 horas después de medad depresiva.
la última toma del medicamento (opción 2 incorrecta). El
Litio es eficaz para el tratamiento agudo de episodios ma- Respuesta correcta:4
níacos del Trastorno Bipolar, sin embargo, una alternativa
eficaz de tratamiento cuando se observa que la persona no de la Asociación Española de Neuropsiquiatría, 35(126),
responde al Litio es la administración de ácido Valpróico 367380.
(Depakine) (opción 4 correcta). El tratamiento con antipsi-
cóticos atípicos en fase maníaca se asocia con una reduc- La terapia interpersonal de Klerman y Weissman (1984)
ción del riesgo de viraje a depresión en comparación con el se planteó inicialmente desde una postura eminentemente

9
psiquiátrica, con la intención de prevenir las recaídas en nes de gran contenido emocional en un momento distinto
trastornos afectivos depresivos tras el tratamiento con anti- al actual (opción 4 incorrecta). APIR Manual de Psicotera-
depresivos (opción 4 correcta). Sin embargo, dada la efica- pias, Tema 10.
cia del tratamiento (demostrado por estudios tales como el
conocido estudio NIMH) se planteó la posibilidad de usarlo
como tratamiento psicológico para los trastornos afectivos, 32. El tratamiento con la Terapia interpersonal
y numerosos profesionales lo utilizan en su práctica clínica. (Klerman, Weissman y cols. 1979) se enfoca
El impacto del tratamiento ha sido tal ha sido extrapola- en resolver las siguientes cuatro áreas de pro-
do a otras aplicaciones, como su uso en adolescentes o en blema interpersonal relacionadas con el inicio
Trastorno Bipolar. El principal objetivo de esta terapia es o mantenimiento del episodio depresivo del pa-
la mejora de las relaciones interpersonales (opción 3 inco- ciente:
rrecta) centrándose en 4 áreas problemáticas (opción 1 in-
correcta): Duelo, disputas interpersonales, transición de rol 1. Duelo, disputa de roles, transiciones de
y déficits interpersonales. Los problemas que aborda esta roles, déficits interpersona-
terapia profundizan en la infancia del paciente, sino que se les.
aborda con técnicas centradas en problemas tangibles del 2. Duelo, culpa, pérdida, déficits interper-
momento actual (opción 2 incorrecta). El contenido de este sonales. 3. Pérdida, conflicto de roles,
ítem fue preguntado en el examen PIR 2018, por lo que transición de ro les, déficits interper-
lleva siendo preguntado dos años seguidos, es un dato muy sonales.
relevante a tener en cuenta. APIR Manual de Tratamientos 4. Duelo, culpa, transición de roles, déficits
Psicológicos, Tema 4. interpersonales.

Respuesta correcta:1
31. En un tratamiento psicológico el terapeuta pide
al cliente que analice cómo han sido las solu- La TIP es una terapia breve, focalizada en el terreno de lo
ciones que ha venido aplicando a su trastorno interpersonal, cuyo principal objetivo es la mejora de las
hasta el momento. Le pide valorar si le relaciones interpersonales centrándose en 4 áreas proble-
han resultado exitosas, ineficientes, o incluso máticas: 1)Duelo: pérdidas de personas significativas. 2)
perjudiciales. Está aplicando:
Disputas interpersonales. 3)Transición de rol: renunciar al
rol actual, expresar sentimientos de culpa, enfado o pér-
1. La reconstrucción del estilo afectivo de dida, adquirir habilidades y establecer nuevos vínculos. 4)
la te rapia constructivista. Déficits interpersonales. En el manual original de la Terapia
2. La fase de desesperanza creativa de la Interpersonal de Klerman y Weissman los conceptos utili-
terapia de aceptación y com- zados para nombrar las cuatro áreas problema son: Grief,
promiso. Role dispute, Role transition, Interpersonal deficits; y en la
3. La teoría del guión del análisis transac- versión traducida del manual original los conceptos utiliza-
cional. dos son: Duelo, disputas interpersonales de rol, transicio-
nes de rol y déficit interpersonales. El desarrollo de la TIP
4. La proyección temporal de la terapia consta de 3 momentos: 1. Fases iniciales: durante estas se-
cogniti va. siones se realiza una revisión minuciosa de los síntomas, se
le da nombre al síndrome y se instruye sobre la depresión,
Respuesta correcta:2 poniéndola en relación con su contexto interpersonal. En
este momento se puede otorgar al paciente el rol de enfer-
La fase de desesperanza creativa es un concepto nuclear en mo y valorar la necesidad de medicación. 2. Fases interme-
la terapia de aceptación y compromiso y refleja la inutilidad dias: se trabajan las 4 áreas problemáticas en función de lo
de los intentos que ha realizado el paciente para solucio- evaluado en las fases iniciales. 3. Fases de terminación: la
nar sus problemas de forma que éste busque otras alter- finalización de la terapia debe ser abordada explícitamente,
nativas para ello (opción 2 correcta). Teniendo en cuenta reconociéndola como un momento de posible duelo. APIR
que el enunciado de la pregunta se refiere al componente Manual de Psicoterapias, Tema 10
de desesperanza creativa, de la ACT, terapia perteneciente
a las terapias de tercera generación, descartamos la opción
1 referente a la reconstrucción del estilo afectivo. Esto es 33. En la terapia cognitiva para las psicosis se pro-
propio de las terapias constructivistas y consiste en que pone una evaluación inicial donde se deberían
a través de la relación terapéutica se lleve a cabo una re- identificar las señales que sirven para disparar
construcción del sistema cognitivo del individuo (opción 1 las alucinaciones auditivas. Si el terapeuta pide
incorrecta). La teoría del guión hace referencia a las pau- a su cliente que intente avivar y reprimir l a s
tas de comportamiento arraigadas y juegos que se repiten, voces varias veces al día ¿qué técnica está utili-
relacionados con una especie de programación paterna en zando?:
la infancia. Pertenece al Análisis Transaccional de Berne,
terapia de corte humanista (opción 3 incorrecta). La pro- 1. Autoinstrucciones.
yección temporal es una técnica cognitiva destinada a que 2. Evaluación analítica funcional.
el paciente consiga cierta objetividad, situando las imáge- 3. Verbalización concurrente.

10
4. Condicionamiento encubierto. 1. La defusión.
2. La aceptación.
3. El contacto con el momento presente.
Respuesta correcta:3 4. El yo como contexto.

En la terapia cognitiva para la psicosis se emplean dos en-


foques para comprobar las creencias de forma empírica. Respuesta correcta:0
Por una parte, tenemos un conjunto de procedimientos para
poner a prueba la creencia generalizada de «No puedo con- ANULADA. Esta pregunta aparece anulada en la plantilla
trolar mis voces». En primer lugar, esto se reformula como provisional, manteniéndose así en la definitiva también. La
«No puedo hacer que las voces aparezcan y desaparezcan». opción que podría ser correcta es la 1, la defusión cognitiva.
Luego el terapeuta plantea situaciones para aumentar y lue- Teniendo en cuenta la bibliografía existente referente a la
go disminuir la probabilidad de oír voces. Una evaluación Terapia de Aceptación y Compromiso (ACT), en el manual
cognitiva inicial debería identificar las señales que sirven de Tratamientos psicológicos para la psicosis de Fonseca
para provocar las voces y una técnica que tiene una elevada (2019) se hace referencia a lo siguiente: “ Ejemplos de ejer-
probabilidad de eliminar las voces de forma duradera es la cicios de defusión serían la desliteralización de palabras,
verbalización concurrente (Birchwood, 1986) (opción 3 co- el etiquetado de pensamientos (“estoy teniendo el pensa-
rrecta). La persona aviva y reprime las voces varias veces miento de que nada me sale bien” o “estoy oyendo voces”)
con el fin de proporcionar una prueba completa. Caballo, y la contradicción del pensamiento con la acción (decir “no
V. (2007). Manual para el tratamiento cognitivo-conductual puedo caminar cuando de hecho lo haces)”. Hay que te-
de los trastornos psicológicos. Madrid: Siglo XXI. Vol. 1. ner en cuenta que en el enunciado de la pregunta figura el
p 670. término “contracción” en lugar de “contradicción” como
aparece en dicho manual, razón por la cual se anula la pre-
gunta. Siguiendo también a Marino Pérez en su manual de
Terapias de Tercera generación encontramos lo siguiente:
34. Entre los siguientes posibles efectos secunda- “La defusión trata de crear contextos de desliteralización
rios de la clozapina, ¿cuál es el más grave?: que disminuyan las funciones regulatorias innecesarias del
lenguaje y de facilitar el contacto con los eventos en curso”.
1. Agranulocitosis. La idea es disminuir la credibilidad de los propios eventos
2. Sedación. privados y la “fusión” de uno con ellos”. FONSECA, E.
3. Aumento de peso. (2019): Tratamientos psicológicos para la psicosis. Madrid:
4. Taquicardia. Pirámide, pág.15. Pérez Álvarez, M. (2014): Las terapias
de tercera generación como terapias contextuales. Síntesis.
Madrid.
Respuesta correcta:1
36. Dentro de las intervenciones psicológicas basa-
das en internet para personas con psicosis, ¿en
La Clozapina es un tipo de antipsicótico atípico. La agranu- qué modelo de terapia se fundamenta el siste-
locitosis es una posible alteración en la síntesis de glóbulos ma online HORYZONS?:
blancos de la sangre que aparece en un 2% de pacientes
que hacen uso de Clozapina y que puede ser mortal si no es
detectada a tiempo, por lo que se considera el efecto secun- 1. Terapia web de aceptación y compromi-
dario más grave que puede provocar este fármaco (opción so.
1 correcta). Los antipsicóticos atípicos se presentan como 2. Terapia online “afrontando voces”.
alternativa a los clásicos y tienen la ventaja de producir me- 3. Terapia mindfulness basada en la web
nos efectos secundarios. Además de la agranulocitosis que para alucinaciones auditi-
puede aparecer con el uso de Clozapina, hay otros posibles vas.
efectos adversos por el uso de antipsicóticos atípicos, como 4. Terapia social online moderada.
aumento de peso, taquicardia, hipotensión, xialorrea o ex-
ceso de salivación o sedación, sin embargo, se consideran Respuesta correcta:4
efectos secundarios de menor gravedad en comparación
con los riesgos que conlleva la agranulocitosis (opciones 2, El proyecto HORYZONS nace con el propósito de faci-
3 y 4 incorrectas). APIR Manual de Psicobiología, Tema 3. litar el acceso a intervenciones basadas en la evidencia a
los jóvenes con psicosis, haciendo uso de una página web
interactiva. Se trata de utilizar la tecnología digital para
extender los beneficios de las tradicionales intervencio-
35. Dentro de la Terapia de aceptación y com- nes psicosociales cara a cara. Este sistema comprende una
promiso para la psicosis, ¿qué proceso psico- plataforma que facilita el acceso a una amplia variedad de
lógico y terapéutico se trabaja cuando se rea- intervenciones psicosociales interactivas, potenciadas por
lizan ejercicios como la desliteralización un entorno de red social online moderado por profesionales
de palabras, el etiquetado de pensamientos y usuarios expertos que procura la interacción social y el
y la contracción del pensamiento con la ac- apoyo mutuo (opción 4 correcta). Fonseca, E. (2019). Tra-
ción?: tamientos psicológicos para la psicosis. Madrid: Pirámide.

11
37. ¿Cuál es el primer módulo que se administra a esas voces. También se añade frecuentemente una cuar-
cuando se realiza una intervención según la Te- ta fase que se caracteriza por la búsqueda del significado
rapia psicológica integrada para personas con que tienen las voces para el paciente, teniendo en cuenta
esquizofrenia?: su historia pasada y los sucesos que están ocurriendo en
el presente. Se trata de otra pregunta también calló el año
1. Solución de problemas. pasado en el examen PIR 2018. Además, esta teoría lleva
2. Diferenciación cognitiva. siendo objeto de preguntas PIR desde el año 2012 de for-
3. Memoria diferida. ma recurrente, por lo que su contenido debe ser temario a
4. Entrenamiento en habilidades sociales. machacar durante el estudio PIR. Además, es bastante útil
manejar los diferentes sinónimos de los nombres de las te-
rapias para identificarlas durante el examen. APIR Manual
Respuesta correcta:2
de Tratamientos Psicológicos, Tema 3.
La IPT de Roder y Brenner (2007) es un programa de tra-
tamiento conductual para la esquizofrenia, grupal y jerar-
quizado, que se compone de 5 módulos en los que se inclu- 39. ¿Qué clase de fármaco es la quetiapina?:
ye: la rehabilitación cognitiva (módulos: 1. Diferenciación
cognitiva (opción 1 correcta), 2. Percepción social y 3. 1. Una benzodiacepina.
Comunicación verbal) y el entrenamiento en competencia 2. Un ansiolítico.
social (módulos: 4. Entrenamiento en habilidades sociales 3. Un antidepresivo.
(opción 4 incorrecta) y 5. Solución de problemas interper- 4. Un antipsicótico.
sonales): Diferenciación cognitiva, interviniendo en las
habilidades de atención y formación de conceptos (opción
2 incorrecta). Percepción social, analizando los estímulos Respuesta correcta:4
sociales. Comunicación verbal, mediante el entrenamien-
to en habilidades de comunicación. Habilidades sociales. La Quetiapina es un tipo de antipsicótico atípico (opción 4
Solución de problemas interpersonales. La opción 3 es un correcta) (opciones 1, 2 y 3 incorrectas). Los antipsicóticos
mero distractor. Esta pregunta es un clásico de los últimos atípicos son fármacos antagonistas tanto de la Dopamina
años dentro de las preguntas sobre los tratamientos de la (DA) como de la Serotonina (5-HT). Se presentan como
esquizofrenia. Concretamente, es esencial memorizar sus alternativa a los antipsicóticos clásicos y tienen la ventaja
etapas, el orden y los nombres. Así como no mezclar los de producir menos efectos secundarios que estos, además
contenidos que se abordan en la terapia con el título de sus siguen siendo eficaces para los síntomas positivos. No pro-
diferentes módulos. APIR Manual de Tratamientos Psico- vocan síntomas extrapiramidales ni alteraciones relaciona-
lógicos, Tema 3. das con la prolactina y además provocan cierta mejoría al
actuar sobre la activación de la vía mesocortical. Entre los
38. Cuando se utilizan técnicas de focalización posibles efectos adversos por el uso de antipsicóticos atí-
para el tratamiento cognitivo-conductual de las picos tenemos aumento de peso, taquicardia, hipotensión,
alucinaciones, la primera fase de intervención xialorrea o exceso de salivación y sedación. Algunos fár-
consiste en que el paciente: macos antipsicóticos atípicos además de la Quetiapina son
la Clozapina (cuyo posible efecto secundario más grave es
1. Analice las creencias y pensamientos la agranulocitosis), Olanzapina, Risperidona, Ziprasidona,
respect o a las voces. Sulpirida, etc. Algunas de las indicaciones de estos fárma-
2. Preste atención al contenido de las voces. cos son el tratamiento de Trastornos Psicóticos, Trastornos
3. Dirija la atención a la forma y caracterís- de conducta y en algunos casos en el Trastorno Bipolar.
ticas físicas de las voces. APIR Manual de Psicobiología, Tema 3.
4. Registre los sucesos que acontecen antes
de 40. Según Marlatt y Gordon (1985), en el efecto
las voces, así como los pensamientos y de violación de la abstinencia (EVA) se produce
voces que le siguen. un fenómeno de:

Respuesta correcta:3 1. Disonancia cognitiva


2. Atribución de responsabilidad a facto
La terapia cognitiva o Terapia de Focalización de Bentall, res externos incontrolables
Haddock y Slade (1994) Bentall afirma que la causa de las 3. Restablecimiento del consumo adicti
alucinaciones es un fallo de la metacognición (teoría del vo hasta sus niveles iniciales.
déficit en la habilidad metacognitiva de discriminación de 4. Generación de expectativas de resulta
la realidad de Slade y Bentall), por lo que se atribuyen su- do positivas.
cesos internos autogenerados a una fuente externa. Consta
de tres fases: 1) Prestar atención a la forma y aspectos fí- Respuesta correcta:1
sicos de las voces, 2) Prestar atención al contenido de las
voces (que suele reflejar preocupaciones que el paciente Desde el postulado de la prevención de recaídas de Marlatt
pueda tener) y 3) Identificar las creencias y pensamientos y Gordon, se considera una clara diferenciación entre los
tanto antecedentes como consecuentes, que están asociados

12
conceptos caída y recaída. El efecto de violación de la abs- es utilizada como experimento conductual para la descon-
tinencia o EVA, estaría ligado a la recaída, mientras la caída firmación de creencias erróneas posterior. Algunas de las
sería un consumo aislado en un momento determinado. El técnicas para facilitar la exposición interoceptiva son la
EVA explica ese paso de la caída a la recaída. La intensi- práctica de ejercicios cardiovasculares, la hiperventilación
dad del EVA dependerá del grado de compromiso que el y las inhalaciones de dióxido de carbono. En la última ver-
paciente tenga en mantener la abstinencia, la duración de la sión del programa de Barlow, la relajación se ha sustituido
misma (cuanta más, mayor será el efecto) y el valor subje- por re-entrenamiento en respiración (patrón de respiración
tivo que se le dé a la conducta. Por contra, una atribución pausado) ya que era el componente que se había mostrado
externa, inestable y controlable disminuye el EVA (opción menos efectivo del programa (Opción 1 correcta). El entre-
2 incorrecta). Se compone de dos elementos: 1) Disonancia namiento en Mindfulness y el cuestionamiento de las me-
cognitiva, que entra en contradicción con el autoconcepto tapreocupaciones son técnicas que corresponden a terapias
de abstinente (opción 1 correcta); 2) Atribución personal de tercera generación, que son usadas últimamente para
interna, estable y global de la conducta de beber (opción 3 múltiples trastornos, pero no ha sido incorporada dentro de
incorrecta). La opción 4 es un mero distractor. APIR Ma- la terapia del control de pánico de Barlow (opción 3 y 4
nual de Tratamientos psicológicos, Tema 3. incorrectas). Por su parte, el trabajo con exposición en ima-
ginación no es característico del tratamiento del trastorno
de pánico, que suele trabajarse con exposición interocep-
41. En la técnica del desafío de la Terapia estruc- tiva (opción 2 incorrecta). APIR Manual de Tratamientos
tural (Minuchin): Psicológicos, Tema 5.

1. Se necesita una fuerte alianza terapéutica, por


lo que el terapeuta mantiene una posición de
cercanía. 43. En el tratamiento para el trastorno de a n -
2. Se busca establecer unos límites rígidos entre siedad social, ¿qué incluye la Terapia cognitivo-conduc-
los subsistemas familiares para revertir una tual en grupo de McEvoy?:
triangulación de desviación.
3. El terapeuta da a escoger entre dos alternati 1. Entrenamiento en habilidades sociales.
vas, una de ellas es una tarea directiva y la otra 2. Entrenamiento en relajación.
una tarea mucho más onerosa que aquella. 3. Retroalimentación en vídeo.
4. Es necesario intensificar y repetir el mensaje 4. Biofeedback.
cuando la inercia de los sistemas familiares les Respuesta correcta:3
hace resistirse al cambio.
La TCCG de McEvoy es un tratamiento cognitivo conduc-
Respuesta correcta:4 tual eminentemente grupal con una duración de 7 sesiones
que amplía la propuesta de Heimberg en cuanto a compo-
importante y la intensificación del mensaje del terapeuta. nentes de la terapia. De forma que mantiene los tres pilares
Normalmente es necesario repetir muchas veces el mensaje característicos del Gold Standard de Heimberg: Reestructu-
debido a la inercia de los sistemas familiares que les hace ración Cognitiva, Exposición en vivo y Tareas para casa, a
resistirse al cambio (opción 4 correcta). La opción 3 se re- lo que le añade tres nuevos componentes: Psicoeducación,
fiere a la ordalía (opción 3 incorrecta). La opción 1 se re- Videofeedback y Desviación de la atención. En este caso, la
fiere al procedimiento de mimetismo en el cual el terapeuta traducción al castellano de Videofeedback es retroalimen-
pretende potenciar elementos de similitud con la familia tación en vídeo (opción 3 correcta). El entrenamiento en
para favorecer la alianza terapéutica (opción 1 incorrecta). habilidades sociales es un componente que añaden otros
La opción 2 hace referencia a la técnica de reestructuración tratamientos cognitivo-conductuales para la fobia social,
mediante la cual se fijan límites entre los subsistemas fami- como la TCCC de Davidson, el Set de Turner o la IJA-
liares para alterar la estructura familiar (opción 2 incorrec- FS de Olivaries, pero no es un componente del tratamien-
ta). APIR Manual de Psicoterapias, Tema 6.. to de McEvoy (opción 1 incorrecta). El entrenamiento en
relajación no es un componente que añada ninguno de los
42. ¿Qué componente se introdujo en las últimas tratamientos para la fobia social, siendo más típico de los
versiones del programa de tratamiento para el control tratamientos del trastorno de pánico (no exentos de crítica)
del pánico de Barlow y Craske (2007)?: (opción 2 incorrecta). El biofeedback podría ser un dis-
tractor si leemos la pregunta con prisa, ya que claramente
1. Re-entrenamiento en respiración. intentan confundirnos, pretendiendo hacer pasar la opción
2. Exposición en imaginación. por “videofeedback”, pero no lo es (opción 4 incorrecta). El
3. Entrenamiento en mindfulness. biofeedback es una técnica conductual que no se usa para
Fobias Sociales en las TCC más usadas. APIR Manual de
4. Cuestionamiento de las metapreocupa-
Tratamientos psicológicos, Tema 5.
ciones Respuesta correcta:1
44. En el programa d e t r a t a m i e n t o d e l
El programa de Barlow se centra más en la parte conduc- trastorno de ansiedad generalizada del grupo de Dugas,
tual (exposición a las sensaciones interoceptivas temidas), ¿cuándo se emplea la exposición funcional cognitiva?:
mientras que el programa de Clark tiene una orientación
más cognitiva, y la inducción de sensaciones interoceptivas

13
1. Ante preocupaciones que se refieren a aconteci- centra en la emoción negativa asociada con la situación ob-
mientos basados en la realidad o altamente probables. jeto de preocupación (opción 4 correcta). APIR Manual de
2. Ante preocupaciones que se refieren a aconteci- Tratamientos Psicológicos, Tema 5.
mientos altamente improbables.
3. Ante preocupaciones que se refieren a aconteci- 46. Andrea, de 35 años de edad, presenta una eleva-
mientos modificables. da ansiedad cada vez que nota presión en la cabeza y
4. Ante preocupaciones sobre la generación de solu- una leve sensación de mareo, lo que la ha llevado
ciones alternativas. en varias ocasiones al servicio de urgencias por miedo
de sufrir un “ictus o un infarto”, donde le han descar-
Respuesta correcta:2 tado organicidad, diagnosticándola de ansiedad y “so-
matizaciones” y le han pautado ansiolíticos. Desde la
primera ocasión, vigila cada vez más su cuerpo, ha
El tratamiento cognitivo conductual de Dugas y Ladoceur
evitado hacer ejercicio para no marearse y, cada vez
(1997) tiene una evidencia E3 de acuerdo con Pérez (2003). que lee o escucha hablar de enfermedades, apaga la TV
Su tratamiento propone una primera fase de análisis con- o sale de la conversación porque refiere que “noto todos
ductual del problema que se trata (real y soluble, real e irre- los síntomas que escucho, como si los tuviera”, de he-
soluble o irreal) y el trabajo en darse cuenta de ello. En cho, en la última semana ha ido al médico por notarse
una segunda fase se plantean las diferentes intervenciones los ojos enrojecidos y un dolor fluctuante pero molesto
sobre los diferentes tipos de preocupaciones: En el caso de en el costado. A pesar de consultar sus síntomas des-
que sean problemas reales y solubles, nos centraremos en el confía de los médicos porque su madre falleció cuando
problema (opción 3 incorrecta), en caso de que sean reales ella tenía 14 años, “porque los médicos no supieron lo
e irresolubles en la emoción y, finalmente, si son problemas que le pasaba”. Según la sintomatología de Andrea,
irreales nos centraremos en el uso de la exposición fun- ¿qué programa de tratamiento sería el más indicado?:
cional cognitiva (opción 1 incorrecta, opción 2 correcta),
que consiste en aplicar una flecha descendente y exposición 1. Tratamiento del trastorno de pánico de D. Barlow
con prevención de respuesta encubierta. En su última ver- (2001).
sión (2007), este tratamiento añade el reconocimiento de 2. Tratamiento cognitivo conductual de la somatiza-
la incertidumbre, la exposición y la prevención de recaídas ción de Woolfolk y Allen (2006).
como componentes de su terapia. La opción 4 es un distrac-
3. Tratamiento de la ansiedad por la salud de
tor. APIR Manual de Tratamientos Psicológicos, Tema 5.
Warwick y Salkovskis (1990, 2001).
4. Tratamiento cognitivo de las ideas delirantes (de
45. En el tratamiento cognitivo-conductual del
trastorno de ansiedad generalizada, se “enseña” a los tipo somático) de Chadwick (2009).
pacientes que la preocupación patológica o improducti-
va consiste en: Respuesta correcta:3

1. Pensamientos, imágenes, impulsos o sensaciones


físicas desagradables, de carácter intruso, involuntario e El tratamiento de la ansiedad por la salud de Warwick y
indeseado. Salkovskis (1990, 2001) (Opción 3 correcta) es un progra-
ma cognitivo-conductual estructurado y creado para los
2. Una cadena de pensamientos sobre problemas in- trastornos de ansiedad por la enfermedad (hipocondría). Di-
mediatos y realistas, centrada en la resolución de estos o de cho programa se basa en las interpretaciones catastróficas y
la situación. los supuestos disfuncionales, consiste en ayudar al paciente
a detectar y cambiar los pensamientos automáticos negati-
3. Pensamientos negativos, autorreferenciales, vin- vos sobre sus síntomas físicos, las creencias desadaptativas
culados habitualmente con eventos del pasado y que apare- sobre la salud y enfermedad y las conductas problemáticas.
cen de forma automática. El resto de opciones están creadas para otro tipo de trastor-
nos. El tratamiento del trastorno de pánico de D. Barlow
4. Una cadena de pensamientos sobre problemas (2001) (opción 1) está diseñada para el trastorno de pánico.
distales o abstractos, que se centra en la emoción negativa El tratamiento cognitivo conductual de la somatización de
asociada con la situación objeto de preocupación. Woolfolk y Allen (2006) (Opción 2) para el trastorno de
síntomas somáticos. El tratamiento cognitivo de las ideas
delirantes (de tipo somático) de Chadwick (2009) (opción
Respuesta correcta:4
4) está destinado a la reducción de las creencias delirantes.
No obstante, esta pregunta puede ser controvertida ya que
Las últimas investigaciones muestran que las TCC son no queda claro el diagnóstico de hipocondría del caso ex-
superiores a la farmacoterapia con benzodiacepinas mien- puesto en el enunciado, el criterio B del DSM descarta el
tras que éstas no parecen aportar beneficios considerables diagnóstico si existen síntomas como los descritos, por lo
cuando se las usa junto a la TCC. Barlow específica que que podría ser acertado el diagnóstico de T. de síntomas so-
las más eficaces son aquellas que combinan las TCC con máticos (siendo la opción correcta posiblemente correcta).
relajación y cuyo objetivo es colocar el proceso de preocu- APIR Manual de Tratamientos Psicológicos, Tema 8.
pación bajo control del cliente, enseñando al paciente que
la preocupación patológica improductiva es una cadena de
pensamientos sobre problemas distales o abstractos, que se
47. ¿Cuál es el componente fundamental de la

14
Terapia cognitiva específica para el tratamiento del 49. Señale para cuál de los siguientes
trastorno obsesivo-compulsivo (como, por ejemplo, trastornos está más indicado el entrenamiento en r e -
en la propuesta de Belloch et al., 2011)?: versión del hábito:

1. La exposición y prevención de respuesta a las si- 1. Trastorno obsesivo-compulsivo.


tuaciones temidas por el paciente. 2. Tricoticolomanía.
2. La exposición y prevención de respuesta a los 3. Trastorno de la personalidad obsesivo-com
contenidos de los pensamientos u obsesiones principales. pulsivo.
3. El entrenamiento en técnicas de control de la acti- 4. Trastorno dismórfico corporal.
vación (por ejemplo, relajación).
4. El trabajo con las valoraciones y creencias disfun- Respuesta correcta:2
cionales asociadas a las obsesiones.
El procedimiento más recomendado para la tricotilomanía
Respuesta correcta:4 (Opción 2) es la terapia de conducta e inversión del hábito,
a lo que se pueden sumar medidas farmacológicas (ISRS,
“El componente fundamental de la Terapia cognitiva espe- antipsicóticos o litio) y medidas dermatológicas (esteroi-
cífica para el tratamiento del trastorno obsesivo-compulsi- des, antihistamínicos). Con respecto al TOC (Opción 1), la
vo es el trabajo con las valoraciones y creencias disfuncio- exposición con prevención de respuesta, se considera efi-
nales asociadas a las obsesiones (Opción 4 Correcta). Tanto caz (E1) y de elección. Para el tratamiento para el trastorno
la opción de respuesta 1 como la 2 nos mencionan la expo- de personalidad obsesivo-compulsivo una de las técnicas
sición con prevención de respuesta, aunque la EPR se con- que más se usan al principio es la solución de problemas
sidera eficaz (E1) y constituye el tratamiento de elección, para empezar a trabajar y modificar algunos de los compor-
la terapia cognitiva en el contexto del tratamiento del TOC tamientos rígidos del pacientes, así como su pensamiento
aparece bien como un suplemento que potencia-facilita las dicotómico. Por último, no existen hoy en día tratamientos
técnicas conductuales como la EPR, bien como tratamiento empíricamente establecidos para el trastorno dismórfico
principal que incluye como experimento conductual la EPR corporal (opción 4), la TCC de Rosen es el único tratamien-
con el objetivo de trabajar con las valoraciones y creencias to recogido para este trastorno y cuenta con una eficacia E3.
disfuncionales asociadas a las obsesiones. Por otro lado, el APIR Manual de Tratamientos Psicológicos, Tema 14.
entrenamiento en técnicas de control de la activación como
la relajación (Opción 3) no sería el objetivo principal en
este tiempo de tratamiento. APIR Manual de Tratamientos 50. Entre los principios en los que se sustenta la
Psicológicos, Tema 6. psicoterapia de los trastornos de la personalidad se en-
cuentra el de “gradualidad”, que hace referencia a:

1. Promover cambios en los aspectos o manifes-


48. En pacientes con diagnóstico de trastorno o b - taciones que conllevan un mayor nivel de perturbación e
sesivo compulsivo ¿en qué caso estaría más indicado interferencia en la vida del paciente, para ir progresando
aplicar un componente de exposición con prevención paulatinamente en la mejora de su calidad de vida.
de respuesta?: 2. Establecer una jerarquía de principios y necesida-
des de cambio y acordar con el paciente la dificultad de
1. Paciente con obsesiones de tipo sexual (por ejem- cada uno de ellos para ir avanzando en función de ese gra-
plo, prácticas sexuales no deseadas o inmorales). diente.
2. Paciente con obsesiones agresivas hacia sí mismo 3. Exponerse a las situaciones que provocan males-
o los otros. tar, comenzando por las que generan un malestar interme-
3. Paciente con obsesiones sobre la contaminación o dio.
suciedad y/o compulsiones de lavado y/o limpieza. 4. Promover cambios en los aspectos o manifestacio-
4. Paciente con obsesiones blasfemas (por ejemplo, nes más periféricas del trastorno para ir progresando paula-
insultar a Dios). tinamente hacia la consecución de cambios en los patrones
Respuesta correcta:3 más básicos de la organización personal.

Podemos encontrarnos obsesiones con o sin conductas Respuesta correcta:4


compulsivas manifiestas. Con respecto a las obsesiones sin
compulsiones (Opción 1, 2 y 4), Salkovskis y Westbrook Los objetivos de la psicoterapia ante un TP deben enmar-
proponen la técnica de habituación ante los pensamientos carse dentro de un plan de confirmación de la experiencia
obsesivos, siendo el elemento clave del tratamiento la pre- disfuncional, respetando el modo de ser del paciente. Des-
dictibilidad de los estímulos a los que se expone el sujeto. de esta perspectiva, el principio de gradualidad hace refe-
Con respecto a las obsesiones con conductas compulsivas rencia a que los cambios deben ser graduales, progresando
(opción 3) si estaría más indicado aplicar un componente desde las manifestaciones periféricas hacia los patrones
de exposición con prevención de respuesta que consiste en básicos de la organización personal (opción 4 correcta).
la exposición a los estímulos externos o internos que provo- Belloch, A. (2010). Tratado de los trastornos de la persona-
can las obsesiones bloqueando la realización de los rituales. lidad. Editorial Síntesis. pp 297.
APIR Manual de Tratamientos Psicológicos, Tema 6.

15
51. El tratamiento de los trastornos de la persona- trastornos sexuales por dolor (Dispareunia y Vaginismo).
lidad basado en el modelo de J.E.Young, se basa en: La opción dada por correcta pero que finalmente fue anula-
da es la 3, la eyaculación precoz no está clasificado dentro
1. Promover cambios en los esquemas tempranos de los trastornos de la excitación sexual sino dentro de los
disfun cionales o desadaptativos (ETD). trastornos del orgasmo. APIR Manual de Tratamientos Psi-
2. Promover cambios en las conductas que se deri- cológicos, Tema 12.
van de los ETD.
3. El uso de técnicas experienciales para evocar ex-
perien cias infantiles traumáticas. 53. El tratamiento psicológico de trastornos
como la depresión, en casos en los que puede ir acom-
4. Re-elaborar las experiencias de transferencia ne- pañada de una disfunción sexual, es especialmente im-
portante tratar los problemas que tienen que ver con:
gativa. Respuesta correcta:1
La terapia cognitiva de Young se deriva de la TC de Beck y 1. Bajo deseo sexual.
se aplica en personas con trastornos de personalidad. El ob- 2. Problemas de excitación.
jetivo de ésta es lograr una relación terapéutica colaborati- 3. Eyaculación precoz.
va, modificar creencias rígidas y lograr reconocer las emo- 4. Anorgasmia.
ciones y cogniciones que se derivan de estas creencias para
desarrollar un modo adulto saludable del paciente logrando Respuesta correcta:1
la remisión o el control de los modos disfuncionales. Esto
implica que el trabajo se centre más en la reestructuración En los trastornos depresivos, la aparición de cambios físi-
cognitiva de los ETD con la posterior implicación que esto cos es habitual y suele ser uno de los motivos principales
tendría a nivel conductual y emocional (opción 1 correcta; por los que se solicita la ayuda de un profesional. Un sín-
opción 2 incorrecta). Young, en su terapia, describe dife- toma típico, que afecta a un 70-80% de los pacientes de-
rentes esquemas disfuncionales tempranos (ETD) que se presivos, es los problemas de sueño. Otros síntomas físicos
agrupan en 5 dimensiones relacionadas con la no satisfac- comunes son la fatiga, la pérdida de apetito y una dismi-
ción de las necesidades emocionales centrales. En función nución de la actividad y el deseo sexuales (Opción 1). Es
del esquema disfuncional que se active en cada momento, decir, como parte del estado general de anhedonia, anergia,
estaremos hablando de un modo de esquema que va a deter- etc., las conductas y deseos sexuales se ven casi siempre
minar el funcionamiento del paciente (a nivel de emocio- afectados negativamente. BELLOCH, A., SANDÍN, B.,
nes, pensamiento y conductas). Para llevar a cabo el pro- RAMOS, F. (2009): Manual de psicopatología (2.ª ed., Vol.
ceso de cambio de los esquemas tempranos disfuncionales 2, Pág. 254). Madrid, España: McGraw-Hill. APIR Manual
utiliza técnicas cognitivas, experienciales, conductuales y de Psicología Clínica, Tema
técnicas que trabajan con la relación paciente-terapeuta. No
se centra sólo en el uso de técnicas experienciales (opción 3 54. De las siguientes estrategias ¿cuál se dirige a
incorrecta). APIR Manual de Psicoterapias, Tema 9. detectar precozmente la existencia de una enfermedad?

1. Promoción de salud
52. La técnica de parada-arranque en la que el paciente 2. Prevención primaria
estimula su pene hasta conseguir la erección, después
se detiene hasta que vuelva al estado de flaccidez y de 3. Prevención secundaria
nuevo vuelve a estimularlo, está indicada ante proble- 4. Prevención terciaria
mas de :
Respuesta correcta:3
1. Bajo deseo sexual.
2. Aversión al sexo. La prevención secundaria se dirige a la detección tempra-
na de los problemas de salud y la facilitación de cambios
3. Excitación (en el hombre).
de conducta que permitan el tratamiento de las enfermeda-
4. Anorgasmia. des (opción 3 correcta). La prevención primaria se dirige a
prevenir la aparición de la enfermedad en pacientes sanos,
reduciendo el riesgo de nuevos casos al intervenir sobre
Respuesta correcta:3 factores de riesgo y protección (opción 2 incorrecta). La
prevención terciaria se dirige a la recuperación de la salud
La técnica de parada-arranque ha sido ampliamente utiliza- y a la prevención de recaídas (opción 4 incorrecta). Por úl-
da para el tratamiento de la eyaculación precoz. El DSM-IV timo, la promoción de la salud se dirige a la adquisición y
clasifica las disfunciones sexuales basándose (con algunas mantenimiento de hábitos saludables, es decir, a promover
modificaciones) en la diferenciación de fases en la respues- inmunógenos conductuales (opción 1 incorrecta). APIR
ta sexual establecida por Masters y Johnson (1966), dife- Manual de Psicología de la Salud, Tema 2.
renciando cuatro grupos fundamentales de trastornos: del
deseo sexual (TDS) (TDS hipoactivo y Trast. aversión al
sexo), de la excitación sexual (TES) (TES mujer y Trast. 55. Según el Modelo transteórico del cambio de
erección (hombre)), del orgasmo (Disf. orgásmica feme- Prochaska y Di Clemente (1984) ¿en qué fase estaría
nina, Disf. orgásmica masculina y Eyaculación precoz) y una persona que evalúa la posibilidad de cambiar?:

16
1. Precontemplación. máximo (FEM) o peak expiratory flow (PEF) en termino-
2. Contemplación. logía anglosajona, es el mayor flujo que se alcanza durante
3. Preparación. una maniobra de espiración forzada. Se consigue al haber
4. Acción. espirado el 75-80% de la capacidad pulmonar total. Entre
sus ventajas, obvian la necesidad de registro manual de los
Respuesta correcta:2 valores por parte del paciente, aumentan la exactitud del
registro y graban el momento del día en que se realiza la
El Modelo transteórico del cambio de Prochaska y Di Cle- maniobra. Su principal inconveniente es su alto precio. Me-
mente (1984) permite entender la conducta adictiva como didor de Peak-flow: técnica de manejo y utilidad en Aten-
un continuo en la intención de abandonar una adicción. Es ción Primaria. Medifam vol.12 no.3 mar. 2002.
un modelo tridimensional integrado por los estadios (“¿qué
se cambia?”), los procesos (“¿cómo se cambia?”) y los 57. En la Terapia de aceptación y compromiso
niveles (“¿cuándo se cambia?”) de cambio. En concreto, aplicada al dolor crónico ¿a qué se refiere la fase de
el enunciado nos pregunta por el estadio en el que se en- “desesperanza creativa”?:
cuentra una persona que evalúa la posibilidad de cambiar.
Esta persona, que empieza a darse cuenta de que tiene un 1. A la toma de conciencia de la falta de eficacia que
problema, y comienza a buscar información y se plantea han tenido las estrategias que el paciente ha aplicado hasta
un cambio (que suele ser en los próximos 6 meses), se en- el momento para el control del dolor.
cuentra en el estadio de contemplación (opción 2 correcta). 2. A ayudar al paciente a ver sus pensamientos y sen-
En este estadio, los sujetos son conscientes pero no han timientos desde la perspectiva de un observador.
realizado aún ningún intento de abandono de la conducta 3. A enseñar al paciente a identificar los valores im-
y tampoco hay compromiso con ningún tratamiento. El res- portantes de su vida.
to de opciones de respuesta serían incorrectas. El estadio 4. A ayudar al paciente a desarrollar planes de con-
de precontemplación se caracteriza por el hecho de que la ducta para cambiar su vida.
conducta no se ve como un problema, por lo que la persona
manifiesta escasos deseos de cambiarla en los próximos 6
meses. En este estadio los pacientes no son conscientes del Respuesta correcta:1
problema y acuden de forma obligada a consulta. La perso-
na descrita en el enunciado no se encontraría en este punto, La fase de desesperanza creativa es uno de los componen-
pues ya evalúa la posibilidad de cambiar (porque empieza tes de la terapia de aceptación y compromiso y refleja la
a considerar que su conducta puede ser un problema) (op- inutilidad de los intentos que ha realizado el paciente para
ción 1 incorrecta). El estadio de preparación para la acción solucionar sus problemas con el objetivo de buscar otras
se caracteriza por la decisión y el compromiso. El objeti- alternativas para solucionarlo (opción 1 correcta). Algunos
vo de la persona es modificar su conducta en los próximos de los objetivos en los que se centra la ACT son la búsque-
30 días. Además, ha intentado dejarlo durante al menos 24 da e identificación de los valores importantes de la vida
horas en el último año (opción 3 incorrecta). La persona del paciente (opción 3 incorrecta) y a ayudar al paciente a
descrita en el anunciado se plantea la posibilidad de cam- desarrollar planes de conducta en compromiso con sus va-
biar, pero todavía no se ha decidido a hacer un cambio a lores personales (opción 4 incorrecta). Otro eje fundamen-
corto plazo. En el estadio de acción, la persona inicia acti- tal de la ACT es la defusión cognitiva y consiste en ayudar
vamente la modificación de la conducta con éxito (opción 4 al paciente a ver sus pensamientos y sentimientos desde la
incorrecta). Para poder pasar a una etapa de mantenimiento perspectiva de un observador (opción 2 incorrecta). APIR
han de transcurrir al menos 6 meses. En este estadio podría Manual de Psicoterapias, Tema 10.
encontrarse un sujeto que acude por primera vez a las sesio-
nes con el psicólogo clínico para dejar de beber y cumple
con las recomendaciones del terapeuta (PIR 17, 113). APIR 58. ¿Para qué tipo de enfermedad se diseñó la Te-
Manual Psicología Clínica, Tomo I, Tema 3. rapia psicológica adyuvante de Moorey y Greer (1989)?:

1. Cáncer.
56. El entrenamiento para el uso del “peak flow 2. VIH.
meter” forma parte de los programas de automanejo
de: 3. Diabetes.
4. Fibromialgia.
1. El colon irritable.
Respuesta correcta:1
2. El asma.
3. La diabetes. “La terapia psicológica adyuvante (TPA) de Greer ha sido
4. Los problemas cardiovasculares. diseñada específicamente para pacientes con cáncer. Se tra-
ta de una adaptación de la terapia cognitiva de Beck y parte
Respuesta correcta:2 de la idea que el afrontamiento de la enfermedad depende
del significado que la persona le dé a ésta y de su estilo de
Un programa de automanejo para pacientes con asma sería afrontamiento. Se trata de un programa flexible de alrede-
el medidor de Peak-flow (opción 2 correcta), una técnica de dor de 6 sesiones que puede dividirse en tres fases, cada
manejo utilizada en atención primaria. El flujo espiratorio una de ellas con objetivos diferenciados. APIR Manual de

17
Tratamientos Psicológicos, Tema 32. principal que defiende el modelo es que las conductas de
una persona están determinadas por la amenaza percibida a
59. Para que se identifique a una persona en el rol su salud. Esta vulnerabilidad que se experimenta, a su vez,
de enfermo propuesto por Parsons (1951), ¿cuál de está condicionada por la percepción acerca de la suscep-
los siguientes requisitos es necesario cumplir? tibilidad personal a la enfermedad y la severidad percibi-
da de las consecuencias de adquirirlas. Un incremento en
1. Mostrar baja adherencia al tratamiento la percepción de vulnerabilidad ante una amenaza para la
2. Ser responsable de los síntomas salud produce una motivación y un fortalecimiento de las
3. Implicarse en la recuperación de la salud conductas de protección hacia esa amenaza. Si hay una dis-
posición alta para emitir las conductas de protección y los
4. Continuar asumiendo las mismas responsabilida-
costes de las mismas son bajos, es probable que se realice
des
la conducta preventiva o de salud, al contrario, se omitirá la
acción (opción 1 incorrecta). La teoría de la autoeficacia de
Respuesta correcta:3 Bandura plantea que el factor principal de adhesión al tra-
tamiento son las expectativas de autoeficacia. La teoría del
El estar enfermo, dice Parsons, no es sólo experimentar la aprendizaje social de Bandura se apoya en los conceptos de
condición física de enfermedad. Estar enfermo más bien expectativa de refuerzo, creencia de que con una determi-
constituye un rol social porque implica conductas basadas nada conducta en un momento concreto puede obtener el
en expectativas institucionales y, con frecuencia, ese rol refuerzo, y lugar de control, expectativa que tiene el sujeto
social se ve reforzado por las normas de la sociedad que sobre el control que tiene sobre su conducta o sobre que
corresponden a esas expectativas. El concepto de “rol de ésta dependa de factores externos a él (opción 2 incorrec-
enfermo” de Parsons (1951) se basa en el supuesto de que ta). El modelo transteórico del cambio entiende la conducta
estar enfermo no es una elección consciente y deliberada, adictiva como un continuo en la intención de abandonar
aunque una enfermedad pueda producirse como conse- una adicción. Se trata de un modelo tridimensional inte-
cuencia de una exposición intencionada a una infección o grado por estadios, procesos y niveles de cambio. Enfatiza
una lesión. Los requisitos que el enfoque de Parsons señala factores de tratamiento como la motivación al cambio (op-
que deben cumplirse para asumir el rol de enfermo son: a ción 4 incorrecta). APIR Manual de Psicología de la Salud,
la persona que enferma se le eximen de sus roles sociales Tema 2.
normales (opción 4 incorrecta), la persona enferma no es
responsable de su condición (opción 2 incorrecta), la per-
sona enferma debe recuperar su estado de salud (opción 1
incorrecta) y la persona que enferma debe buscar ayuda de
expertos competentes y colaborar en la recuperación (op- 61. J. Frank es uno de los autores más importantes
ción 3 correcta). Marín, J. R., López, M. D. C. N. (2008). en el estudio de los “factores comunes” en psicoterapia.
Manual de psicología social de la salud. Síntesis. Señale cuál de los siguientes es uno de los cuatro “facto-
res comunes” que propone como características de to-
das (o muchas) las formas de psicoterapia (Frank y
60. ¿En cuál de las siguientes propuestas teóricas
Frank, 1991):
se incluye la intención de conducta como un determi-
nante proximal de la conducta?
1. “Un ritual” o procedimiento que requiere la parti-
cipación activa del terapeuta y del cliente.
1. Modelo creencias de salud (Rosenstock, 1974)
2. Un “Condicionamiento operante” a través del cual
2. Teoría del aprendizaje social (Bandura, 1986) el terapeuta expresa aprobación o desaprobación de la con-
3. Teoría de la acción razonada (Ajzen y Fishbein, ducta del paciente.
1970) 3. Una “regulación conductual” o “experiencia co-
rrectiva” basada en la capacidad de sugestión y persuasión
4. Modelo transteórico (Prochaska y Di Clete,1984) del terapeuta.
Respuesta correcta:3 4. “Factores del terapeuta” como directividad, empa-
tía, autoridad o aceptación positiva incondicional.
En la teoría de la acción razonada, la intención conductual
es el determinante inmediato de la conducta. La intención Respuesta correcta:1
conductual, a su vez, está determinada por nuestras actitu-
des hacia la conducta y nuestras normas subjetivas (opción Desde los años 90 está ganando terreno una visión meta-te-
3 correcta). El Modelo de Creencias en Salud en su ver- rapéutica y surgió la pregunta: “Si funcionan las terapias,
sión original (Rosenstook, 1960), identifica básicamente pero no tiene que ver con sus idiosincrasias ¿cuáles son los
cuatro variables que parecen influir en el sujeto a la hora factores terapéuticos comunes?”. La obra de Frank sentó
de llevar a cabo una determinada acción preventiva. Estas la base de un análisis continuado de los factores comunes
variables se agrupan en torno a dos dimensiones generales en toda la familia de proyectos orientados a producir cu-
de creencias: La primera dimensión se refiere al grado de ración (“healing”) de trastornos o problemas psicológicos.
preparación o disposición psicológica del sujeto para lle- Comparó psicoterapia tradicional, terapias de grupo y fa-
var a cabo una determinada acción preventiva que viene miliar, tratamiento intraclínico, drogoterapia, medicina,
determinada por la vulnerabilidad percibida y la gravedad curación religiomágica en sociedades no-industrializadas,
percibida. La segunda dimensión hace referencia a los be- cultos y renacimientos. Su conclusión fue que los aspectos
neficios/ costes percibidos de la acción preventiva. La idea compartidos por todas las terapias eficaces son cuatro: 1)

18
Una relación de confianza y con contenido emocional con cognitiva) comparten la importancia que le dan al procesa-
la persona que quiere ayudar. 2)Un entorno “sanador”. 3) miento mental en los trastornos psicológicos. Sin embargo,
Un esquema conceptual, razón o mito que ofrece una ex- la TMC se dirige más a los procesos que a los contenidos
plicación plausible de los síntomas del paciente y prescribe cognitivos. La TMC conserva el uso de estrategias típicas
un ritual o procedimiento para su solución. 4) Un ritual o de la TC (cuestionamiento socrático, experimentos cogniti-
procedimiento que requiere la participación activa tanto vos…) dirigidas a modificar metacreencias con el trastorno
del paciente como del terapeuta y que ambos consideran mientras que la TC estándar lo hace con las creencias ordi-
el medio para restaurar la salud (opción 1 correcta). APIR narias. La TMC se focaliza en el estilo del pensamiento y
Manual de Psicoterapias, Tema 11. formula el síndrome cognitivo atencional. Incluye estrate-
gias de entrenamiento atencional y conciencia plena “desa-
62. La entrevista motivacional (EM), terapia d e - pegada”. Dado que para el modelo metacognitivo, la pato-
sarrollada en sus orígenes para el tratamiento de las logía psicológica está relacionada con la activación de un
adicciones, ha sido reformulada recientemente por estilo de pensar tóxico, el síndrome cognitivo atencional,
sus autores (Miller y Rollnick, 2013), quienes pro- que consiste en un estilo perseverativo de pensamiento que
ponen que son cuatro procesos los que componen la toma la forma de preocupación o rumiación, foco atencio-
EM; estos son: nal en la amenaza y estrategias de afrontamiento contra-
producentes, se propone la atención plena como un modo
1. Vincular, enfocar, evocar y planificar. de impedir la activación del síndrome cognitivo atencional
2. Precontemplación, contemplación, acción y man- (opción 1 correcta). APIR Manual de Psicoterapias, Tema
teni miento. 10.
3. Empatía, discrepancia, discusión y autoeficacia.
4. Evaluar, dialogar, escuchar, convencer.

Respuesta correcta:1 64. Las guías clínicas del Sistema Nacional de Salud
(guíasalud.es) establecen 8 niveles de evidencia (de 1++
Actualmente Miller y Rollnick (2013) proponen que el a 4) y 4 grados de recomendación (de “A” a
cómo y el por qué las personas cambian, sucede en un pro- “D”). ¿Qué grado de recomendación suponen las
ceso recursivo en el que se superponen y confluyen cuatro “opiniones de los expertos” y las “series de casos clíni-
tipos de procesos: 1) Comprometiéndose: El primer proce- cos”?:
so de la Entrevista Motivacional es enganchar al paciente
en una relación de trabajo colaborativo. Es un proceso por 1. Nivel “B”.
el cual ambas partes establecen una conexión útil y una re- 2. Nivel “C”.
lación de trabajo; 2) Enfocándose: Es el proceso mediante 3. Nivel “D”.
el cual el terapeuta desarrolla y mantiene un curso especí- 4. No se consideran como evidencia científica, por lo
fico en la conversación acerca del cambio. Se trata de ayu- que esos aspectos están excluidos como criterios
dar a los pacientes a clarificar sus objetivos; 3) Evocando: de cara a la asignación de niveles de recomenda-
Este proceso por el cual el terapeuta induce la motivación ción.
del cliente hacia el cambio; y 4) Planeando: Es un proceso
mediante el cual el terapeuta ayuda al paciente a formular Respuesta correcta:3
planes de acción específicos para alcanzar el cambio desea-
do (opción 1 correcta). Miller, W. y Rollnick, S. (2015). La “La Guía de Práctica Clínica sobre el manejo de la Depre-
entrevista motivacional (3ª ed.). Paidós Ibérica. sión (Segal sión en el adulto del Sistema Nacional de Salud establece la
y cols.) propuesta de SIGN (Scottish Intercollegiate Guidelines Ne-
twork) como modelo de clasificación de los niveles de evi-
dencia clínica y de grados de recomendación. El grado D
63. Si un psicólogo/a clínico emplea las técnicas de “min- se corresponde con evidencia científica de nivel 3 (estudios
dfulness desapegado”, “entrenamiento en atención” y no analíticos, como informes de casos y series de casos)
“reorientación” (o “refocalización”) de la atención o 4 (opinión de expertos) (opción 3 correcta); o evidencia
situacional”, en qué modelo de psicoterapia se estaría científica extrapolada desde estudios clasificados como 2+
basando: (estudios de cohortes o de casos y controles bien realizados
con bajo riesgo de sesgo y con una moderada probabilidad
1. Terapia metacognitiva (Wells). de establecer una relación causal). Biblioteca de Guías de
2. Terapia de aceptación y compromiso (Hayes y Práctica del Sistema Nacional de Salud. (2019). Niveles de
cols.). evidencia y grados de recomendación del SIGN. Recupera-
3. Psicoterapia analítico funcional (Kholenberg y do de https://portal.guiasalud.es/egpc/depresion-adulto-ni-
Tsai). veles/ “
4. Terapia cognitiva basada en mindfulness para la
depre 65. En un tratamiento psicológico orientado a la
sión superación de miedos, fobias y ansiedad en general,
la técnica de modelado más recomendable es:
Respuesta correcta:1
1. De afrontamiento.
La terapia metacognitiva (TMC) de Wells y la TC (terapia

19
2. Asertivo. como la capacidad para manejarlas (opción 1 incorrecta).
3. Con reproducción forzada. APIR Manual de Psicoterapias, Tema 8.
4. Competente.
Respuesta correcta:1 67. Cuando en la técnica de modelado, el
modelo muestra conductas evocadoras de ansiedad
Las técnicas de modelado tienen como objetivo enseñar o que no conllevan consecuencias negativas, el terapeuta
cambiar una conducta a partir de la observación de la con- intenta conseguir:
ducta de los demás y las consecuencias que le siguen. Exis-
ten variantes del modelado, en función de la competencia 1. Extinción vicaria.
del modelado encontramos: 1)Modelado mastery/dominio: 2. Moldeamiento vicario.
el modelo posee las habilidades precisas para desempeñar- 3. Castigo negativo vicario.
se correctamente desde el principio. Realiza la conducta de 4. Modelado negativo.
una forma relajada. Se utiliza para aprender conductas mo-
toras (opción 4 incorrecta). 2) Modelado coping/de afron- Respuesta correcta:1
tamiento: el modelo empieza con un nivel similar al del
observador y, poco a poco, va mostrando las habilidades Las técnicas de modelado tienen como objetivo enseñar
necesarias para resolver la situación. Se muestra ansioso o cambiar una conducta a partir de la observación de la
al principio y relajado al final. Se utiliza en problemas de conducta de un modelo y las consecuencias que le siguen.
ansiedad (opción 1 correcta). Otras variantes del modelado, Existen variantes en función del objetivo que se tenga: Si el
según el grado de dificultad de la conducta a modelar: 1) objetivo del procedimiento del modelado es que el observa-
Modelado gradual: el modelo va realizando aproximacio- dor aprenda que tras realizar una determinada conducta no
nes a la conducta meta. 2)Modelado con reproducción re- le siguen consecuencias negativas, es decir, que existe un
forzada: se van reforzando las conductas progresivamente efecto desinhibitorio, estaremos hablando de extinción vi-
más difíciles (opción 3 incorrecta). Por otro lado, el mo- caria (opción 1 correcta). Hablaremos de modelado negati-
delado asertivo es aquel que se utiliza en el contexto de vo cuando el modelo realiza una conducta inadecuada (op-
adquisición de habilidades sociales (opción 2 incorrecta). ción 4 incorrecta). El moldeamiento vicario es un proceso
APIR Manual de Psicoterapias, Tema 8. mediante el cual, el sujeto a través de la observación de un
modelo, aprende a realizar una conducta nueva a partir del
66. Señale cómo se denomina la intervención que con- refuerzo sucesivo de aproximaciones a esa conducta y la
siste en la repetición mental de fórmulas verbales extinción de tales aproximaciones a medida que se va con-
(frases elaboradas) sobre sensaciones psicofisiológicas figurando la conducta final deseada (opción 2 incorrecta).
durante breves periodos de tiempo, además de una El castigo negativo vicario es un procedimiento mediante
concentración pasiva en dichas sensaciones: el cual se observa la retirada de un estimulo reforzante tras
la emisión de una conducta que se desea reducir (opción 4
1. Autorregulación emocional. incorrecta). APIR Manual de Psicoterapias, Tema 8.
2. Entrenamiento autógeno.
3. Entrenamiento en autoinstrucciones y respiración. 68. La desensibilización por contacto es una va-
4. Entrenamiento en consciencia plena (mindful- riante de la desensibilización sistemática, e n l a
ness). cual:

Respuesta correcta:2 1. El terapeuta utiliza instrucciones verbales y guía


física para que el cliente realice la exposición con mayor
facilidad.
Existen varios procedimientos mediante los cuales la perso- 2. El terapeuta utiliza, en el proceso de desensibiliza-
na aprende a controlar su propio nivel de activación, entre ción, elementos adicionales para favorecer la imaginación
ellos encontramos técnicas como: Entrenamiento autógeno, de escenas, como fotografías, diapositivas o grabaciones
mediante el cual la persona aprende a asociar determinadas en audio.
frases con representaciones de sensaciones corporales. Se 3. El terapeuta actúa como modelo usando, incluso,
realizan dos ciclos de ejercicios, el primero se dirige a la el contacto físico con el cliente (sosteniendo la mano del
desactivación fisiológica y el segundo se dirige a contro- cliente o colocando su mano en su espalda) mientras éste
lar los estados mentales (opción 2 correcta). Respiración, realiza la exposición.
mediante el cual se pretende enseñar un adecuado control 4. El terapeuta facilita que el cliente realice contacto
voluntario de la respiración, para después automatizar este físico con los estímulos fóbicos (tocándolos con sus pro-
control y generalizarlo a situaciones estresantes (opción 3 pias manos) para así aumentar la eficacia de la exposición.
incorrecta). El mindfulness o entrenamiento en consciencia
puede concebirse como una terapia en sí misma o dentro de Respuesta correcta:3
un programa multicomponente. Se deriva de las prácticas
budistas y busca modificar la forma en la que el individuo La desensibilización sistemática es una técnica que se
se relaciona con su experiencia y reacciona ante los pen- utiliza para reducir respuestas de ansiedad y eliminar las
samientos o sentimientos desagradables (opción 4 inco- conductas de evitación asociadas a dicha ansiedad. Se lleva
rrecta). La autorregulación emocional es la capacidad para a cabo a través del entrenamiento en el uso de respuestas
experimentar emociones de forma moderada y flexible, así incompatibles con la ansiedad de forma que se impide que

20
ésta se desarrolle ante estímulos fóbicos y que se pongan incorrectas por contener en los enunciados miedos propios
en marcha conductas de evitación. Existen variaciones téc- de la edad indicada, y la opción 4 correcta. Entre los 9-12
nicas que comparten los mismos principios que la desen- años, señalan que disminuye el miedo a la oscuridad y a es-
sibilización pero difieren en el modo de aplicación: 1) La tar solo, se mantienen el miedo a las lesiones corporales, los
desensibilización por contacto combina la desensibiliza- fenómenos naturales y aparecen escolares como exámenes
ción sistemática con el modelado. El terapeuta actúa como o rendimiento académico y la muerte. Según Echeburua, en
modelo usando, incluso, el contacto físico con el cliente cambio, entre los 12 y los 18 años, son miedos evolutivos
mientras éste realiza la exposición. Se le llama también normales los miedos a las relaciones interpersonales y a la
modelado participativo o participante y se utiliza en niños. pérdida de autoestima y según la clasificación de Sandín
La característica fundamental de la DS por contacto es la en la adolescencia son miedos comunes los sexuales, los
combinación de DS con el Modelado. Las definiciones de relacionados con la autoidentidad, el rendimiento personal,
los manuales de referencia hacen hincapié en esta caracte- los sociales, los académicos, los políticos y los económi-
rística, hasta el hecho de subrayar que esta técnica estaría a cos. Por lo tanto, la pregunta ha sido anulada, ya que no
medio camino entre la DS y el modelado. Según Olivares, mencionan los autores en ella, y se solapan los miedos en
el terapeuta se expondría (actuando como modelo) a los es- función de las distintas clasificaciones. Bragado, C. “Te-
tímulos que generan ansiedad y luego lo haría el paciente. rapia de conducta en la infancia: trastornos de ansiedad”.
Según Labrador el terapeuta funciona como modelo expo- Fundación Universidad-Empresa. Madrid. 1994
niéndose a los estímulos antes que el sujeto, siendo el tera-
peuta muy activo, pudiendo incluso ayudar al paciente con
guía física cuando éste realiza la exposición. En todo caso, 70. La Desensibilización y reprocesamiento por
la característica esencial de esta técnica es la combinación movimientos oculares (EMDR) en su aplicación a
de la DS con el modelado, tal y como aparece en la opción niños:
de respuesta 3 (opción 3 correcta). 2) La desensibilización
enriquecida es aquella en la que el terapeuta utiliza, en el 1. Consta de 8 fases, igual que en adultos.
proceso de desensibilización, elementos adicionales para 2. Sólo utiliza la fase de desensibilización o repro-
favorecer la imaginación de escenas, como fotografías, cesa miento.
diapositivas o grabaciones de audio (opción 2 incorrecta). 3. Reduce las 8 fases a cuatro.
3)La práctica reforzada consiste en ir reforzando al clien-
te en las conductas de aproximación gradual a los objetos 4. Prescinde de la intervención sobre aspectos cog-
temidos hasta que pueda permanecer en contacto con ellos nitivos.
sin escapar y sin evitarlos, no actuando el terapeuta como
modelo. Se apoya en una serie de incitadores como son las Respuesta correcta:1
instrucciones verbales, guía física, extinción de las conduc-
tas fóbicas y feedback de ejecución (opciones 1 y 4 inco- La EMDR es una técnica que consta de 8 fases, tanto para
rrectas). APIR Manual de Psicoterapias, Tema 8. niños como para adultos, (opción 1 correcta) en las que el
sujeto se expone al recuerdo traumático mientras recibe es-
timulación bilateral. APIR Manual de Tratamientos Psico-
lógicos, Tema 7.
69. Dentro de los miedos normales en el desarrollo
evolutivo, señale cuál de las siguientes opciones NO
describe los miedos propios de la edad que se indica:
71. Las escenificaciones emotivas, como procedi-
miento de tratamiento de la ansiedad en niños, constan
1. A los 2 años: ruidos fuertes, oscuridad, separación de:
de los padres, objetos o máquinas grandes, cambios en el
1. Tres componentes: exposición en vivo, modelado
ambiente personal.
participante y práctica reforzada.
2. De los 3-4 años: miedo a las máscaras, oscuridad,
2. Tres componentes: terapia de juego, exposición en
animales, separación de los padres, ruidos inesperados (in-
vivo y reforzamiento diferencial.
cluyendo los nocturnos).
3. Un componente fundamental basado en la exposi-
3. De los 7-8 años: seres sobrenaturales, oscuridad,
ción con prevención de respuesta.
permanecer solos, miedos basados en sucesos informados
por medios de comunicación, daño físico, ridículo. 4. Tres componentes: desensibilización, reforza-
miento diferencial y moldeamiento.
4. De los 9-12 años: relaciones entre iguales, aspecto
físico y pérdida de autoestima.
Respuesta correcta:1
Respuesta correcta:0
La técnica de escenificaciones emotivas fue creada por
Méndez (1986). Consiste en la exposición gradual en vivo
ANULADA. El miedo es una reacción normal, con gran
con el estímulo fóbico en el transcurso de un juego de rol o
valor funcional y adaptativo. Forma parte del desarrollo
representación de papeles. La relación con el estímulo fóbi-
normal del niño y sus contenidos suelen ser cambiantes y
co se facilita mediante instrucciones, guía física, modelado
evolucionar a la vez que el niño se va desarrollando. Esta
y autoinstrucciones, y se consolida mediante reforzamiento
pregunta está planteada para contestarla con la clasificación
social y material suministrado en el marco de un sistema de
que realizan Bragado y Morris y Kratochwill (1994). Según
economía de fichas. Se describen como componentes prin-
la clasificación de estos autores, las opciones 1, 2 y 3 son
cipales la exposición en vivo, el modelado participante y

21
la práctica reforzada (opción 1 correcta; opciones 2, 3 y 4 (1999), que consiste en enseñar a los niños a relajarse a
incorrectas). APIR Manual de Psicoterapias, Tema 8. través de la analogía de “meterse en su caparazón” como
hacen las tortugas cada vez que sientan frustración, ira
o enfado (opción 4 correcta), y una vez aprendan esto se
72. El tratamiento conductual de la encopresis realiza un entrenamiento en resolución de problemas inter-
funcional (por ejemplo, Bragado, 2001): personales. Esta técnica se utiliza con niños más pequeños
(Educación Infantil y primeros cursos de Educación Prima-
1. Se basa fundamentalmente en los principios del ria). APIR Manual de Tratamientos Psicológicos, Tema 21.
comportamiento clásico.
2. Se basa fundamentalmente en los principios del
condicionamiento operante.
3. Se basa, fundamentalmente, en los principios del 75. Según el modelo A-B-C en terapia cognitiva
biofeedback. aplicada a niños y adolescentes y teniendo en cuenta los
4. No existe un tratamiento exclusivamente conduc- estadios evolutivos postulados por Piaget, señala la op-
tual de la encopresis funcional. ción correcta:

Respuesta correcta:2 1. La terapia cognitiva no es aplicable a niños en el


esta-dio de operaciones concretas.

El entrenamiento en el uso del baño es un procedimiento 2. La terapia cognitiva no es aplicable a niños en el


multicomponente en el que intervienen el reforzamiento estadio de operaciones formales.
positivo, el control de estímulos, instigadores, desvaneci- 3. La terapia cognitiva no es aplicable a niños en el
miento y manejo del ambiente. El objetivo es enseñar al estadio preoperativo.
niño a defecar de forma regular en el inodoro. Bragado, 4. La terapia cognitiva no es aplicable en los estadios
basándose fundamentalmente en los principios del condi- preoperativos y de operaciones concretas, pero sí en el es-
cionamiento operante (opción 2 correcta), recomienda no tadio de operaciones formales.
sólo reforzar al niño cuando se produzca la defecación,
sino también reforzar la conducta de sentarse en el inodoro. Respuesta correcta:3
APIR Manual de Tratamientos Psicológicos, Tema 24.

73. La técnica de Inversión del hábito (Azrin y “Para Piaget, el niño preoperatorio es incapaz de com-
Nunn): prender las transformaciones perceptivas de los objetos,
construir series ordenadas de elementos, o clasificar. Estas
1. Ha demostrado su eficacia en los trastornos por limitaciones del pensamiento preoperatorio también condi-
tics en niños, tanto a corto como a largo plazo. cionan el tipo de psicoterapia aplicable con niños, teniendo
2. No es una técnica psicológica eficaz, ya que tien que ajustar las intervenciones a la edad del mismo. Algunos
de a incrementar los tics a largo plazo. autores consideran que debido a estas limitaciones, cier-
3. Está indicada en el tratamiento del trastorno obse tos modelos de terapia como el A-B-C de terapia cognitiva
sivo-compulsivo infantil. de Ellis no son aplicables durante el periodo preoperatorio
4. Se basa fundamentalmente en la relajación de Ja (opción 3 correcta). Estas intervenciones requieren utilizar
cobson. la lógica, clasificar los pensamientos y atender simultánea-
mente a varios aspectos de la realidad (como antecedentes,
Respuesta correcta:1
creencias y consecuencias) por lo que no serían aplicables
antes del periodo de las operaciones concretas (opción 1, 2
El único tratamiento empíricamente validado para el tras- y 4 incorrectas). Esta visión de las limitaciones de estadio
torno por tics es la inversión o reversión del hábito (opción preoperatorio no es unánime, ya que los trabajos de otros
1 correcta), y es el que se ha demostrado más eficaz. APIR autores desde la perspectiva del procesamiento de la infor-
Manual de Tratamientos Psicológicos, Tema 20. mación y desde la teoría de Vigotsky han concluido que en
este periodo el niño posee una competencia mayor de la
que suponía Piaget. APIR Manual de Desarrollo Psicoló-
gico, Tema 4.
74. ¿Cuál de las siguientes técnicas está dirigida
a trabajar las conductas impulsivas en niños?:
76. En el tratamiento de la ansiedad por separa-
1. Técnica del árbol. ción y ansiedad generalizada en la infancia, para poder
2. Técnica de la alfombra mágica. aplicar la técnica de relajación progresiva, Cautela y
3. Técnica del águila. Groden (1989) señalan unas habilidades mínimas
4. Técnica de la tortuga. para realizar los ejercicios de relajación. Entre estas se
encuentran:
Respuesta correcta:4
1. Permanecer quieto durante cinco segundos.
La técnica de la tortuga de López Soler y Martínez Núñez 2. Mantener el contacto ocular durante diez segun-

22
dos. aceptada y estudiada sobre las posibles contribuciones am-
3. Cumplir instrucciones complejas. bientales a las conductas agresivas, la “Teoría de la coac-
4. Imitación de habilidades motoras complejas. ción”, a través del estudio observacional en vídeo del niño
en la escuela y en interacción con sus padres en el hogar. La
Respuesta correcta:1 teoría de la coacción explica cómo se dan patrones disocia-
les en las dinámicas familiares. Los niños utilizan conduc-
tas agresivas (rabietas, lloros…) para controlar la conducta
de sus padres. Estas conductas se refuerzan positivamente
Para poder aplicar la técnica de relajación progresiva, Cau-
(el niño consigue lo que quiere) o negativamente (el niño
tela y Groden (1989) señalan unas habilidades mínimas que
evita situaciones que no quiere). El proceso por el cual el
deberá adquirir el niño antes de comenzar la enseñanza de
niño utiliza conductas aversivas en su beneficio se denomi-
la relajación: 1) Permanecer quieto durante cinco segundos
na coacción. En los procesos de coacción presentes en las
(opción 1 correcta); 2) Mantener la mirada durante cinco
relaciones entre padres e hijos destaca el papel explicativo
segundos (opción 2 incorrecta); 3) Ser capaz de imitar ac-
del reforzamiento. Los padres ante las conductas del niño
ciones sencillas como levantar su mano por encima de la
no actúan correctamente y ceden ante sus deseos. A corto
cabeza, tocar la mesa y tocarse el pecho (opción 4 inco-
plazo resulta agradable para ambas partes, pero a largo pla-
rrecta); y 4) Obedecer órdenes sencillas como “levántate”,
zo se convierte en un grave problema. Los padres al ceder
“siéntate” y “ven aquí” (opción 3 incorrecta). Cautela, J. R.
ante sus hijos caen en la “trampa del reforzamiento”, for-
& Groden, J. (1989). Técnicas de relajación (Manual prác-
taleciendo este tipo de conductas negativas y coercitivas,
tico para adultos, niños y educación especial). Barcelona:
es por esto último por lo que se nos pregunta. En térmi-
Ediciones Martínez Roca, S. A., p.68.
nos de psicología del aprendizaje, refuerzos serían aquellas
consecuencias o estímulos que aumentarían la probabilidad
77. Señale la respuesta correcta con respecto al progra- de que repitiéramos la conducta, siendo positivo cuando
ma ACCIÓN (Stark y cols., 2010) para el tratamiento se presenta un estímulo apetitivo contingente a la conduc-
grupal de la depresión mayor:
ta (opción 1 incorrecta) y negativo cuando es la desapari-
ción de un estímulo aversivo, en este caso los gritos del
1. La población diana son adolescentes de entre 13 niño, lo que aumentan la probabilidad de que repitamos la
y 18 años. conducta, retirar la orden, ceder, lo que nos provoca alivio
2. Utiliza planificación de actividades placenteras y (opción 2 correcta). Por su parte, castigos serían aquellas
de dominio y el entrenamiento en habilidades de autocon- contingencias que disminuyen la probabilidad de conducta,
trol de la ira. siendo positivo, si es por la aparición de una consecuencia
3. No tiene en cuenta el uso de técnicas de reestruc- aversiva (opción 3 incorrecta) o negativo si es por la elimi-
turación cognitiva adaptadas a niños. nación de una consecuencia o estímulo apetitivo (opción
4. Las habilidades de solución de problemas no for- 4 incorrecta). APIR Manual de Psicología Clínica Infantil,
man parte del programa. Tema 8.

Respuesta correcta:2 79. “Imagina que eres un gato que se despereza,


estirando sus patas y arqueando el lomo”. ¿A qué co-
El programa ACTION de STARK Y KENDALL fue creado rresponde esta instrucción?:
para niños de 8 a 13 años (opción 1 incorrecta) e incluye las
siguientes técnicas: 1) Actividades agradables; 2) Autocon- 1. Ésta es una instrucción que forma parte del en-
trol (opción 2 correcta); 3) Relajación; 4) Reestructuración trenamiento en respiración profunda, para el control de la
cognitiva (opción 3 incorrecta); 5) Entrenamiento en aser- activación fisiológica excesiva en niños.
tividad; 6) Solución de problemas (opción 4 incorrecta); 2. Forma parte del entrenamiento en autoinstruccio-
7) Solución de problemas interpersonales; 8) Educación nes (Meichembaum y Goodman, 1971).
emocional; y 9) Autoinstrucciones. APIR Manual de Trata- 3. Forma parte de las instrucciones del entrenamien-
mientos Psicológicos, Tema 27. to en relajación de Koeppen (1974), una variante de la re-
lajación progresiva.
78. En la Hipótesis de la coerción (Patterson, 4. Forma parte de las autoinstrucciones utilizadas en
1982), cuando el padre retira la orden por no oír llo- el programa Coping Cat de Kendall (1990).
rar o gritar al niño porque no obedece, el mantenimien- Respuesta correcta:3
to de la conducta coercitiva se explica por:

1. Refuerzo positivo. Una variante de la relajación progresiva que incluye el em-


2. Refuerzo negativo. pleo de imágenes que ayudan al niño a tensar cada grupo
3. Castigo positivo. muscular y a mantenerlo motivado hacia el ejercicio, es el
entrenamiento en imaginación de Koeppen (1974). Se na-
4. Castigo negativo. rran varias historias que el niño va escuchando, y en un
momento de ellas debe tensar una zona de su cuerpo. Por
Respuesta correcta:2 ejemplo, para relajar brazos y hombros, la instrucción sería:
«imagina que eres un gato que se despereza, estirando sus
Patterson elabora la teoría explicativa más ampliamente patas y arqueando el lomo» (opción 3 correcta). Koeppen,

23
A. S. (1974). Relaxation training for children. Elementary trastorno de apego reactivo, sino que también son la princi-
School Guidance & Counseling, 9(1), 14-21. pal característica del trastorno del espectro autista. En con-
creto, los niños pequeños de ambas afecciones pueden ma-
nifestar una expresión inhibida de las emociones positivas
80. ¿Cuál de los siguientes NO es un programa (opción 3 incorrecta), retrasos cognitivos y del lenguaje, y
de tratamiento para el trastorno de ansiedad generali- alteraciones en la reciprocidad social (opción 4 incorrecta).
zada en la infancia?: Por ello, sería necesario diferenciar el trastorno de apego
reactivo del trastorno del espectro autista. Estos dos trastor-
1. El programa de tratamiento Coping Cat. nos se pueden distinguir basándose en las diferentes histo-
2. El programa FRIENDS. rias de abandono y por la presencia de intereses restringidos
o comportamientos ritualizados, un déficit específico en la
3. El programa de tratamiento Coping Bear.
comunicación social y comportamientos de apego selecti-
4. El programa de intervención de Herbert. vos (opción 2 correcta).Los niños con trastorno de apego
reactivo han experimentado una historia de abandono so-
Respuesta correcta:4 cial grave, aunque no siempre es posible obtener historias
detalladas sobre la naturaleza precisa de sus experiencias,
El programa de intervención de Herbert está diseñado para sobre todo en las evaluaciones iniciales. No es común que
padres (opción 4 correcta) que tienen dificultades para ma- los niños con trastorno del espectro autista tengan una his-
nejar eficazmente el comportamiento de sus hijos, siendo toria de abandono social. Los intereses restringidos y los
el principal objetivo dotarles de información y habilidades comportamientos repetitivos, característicos del trastorno
para conseguir de ellos un estilo educativo democrático. del espectro autista, no son una particularidad del trastorno
Coping Cat, Coping Bear y FRIENDS sí son programas de apego reactivo. Estas características clínicas manifies-
para el tratamiento de la ansiedad generalizada en la infan- tan una fijación excesiva a rituales y rutinas, con intereses
cia (opciones 1, 2 y 3 incorrectas). APIR Manual de Trata- restringidos, obsesivos y reacciones sensoriales inusuales.
mientos Psicológicos, Tema 27. Sin embargo, es importante destacar que los niños con
cualquiera de las dos afecciones pueden exhibir comporta-
81. Los delirios en los niños: mientos estereotipados, tales como mecerse o moverse. Los
niños con cualquiera de ambos trastornos también pueden
1. Son más frecuentes que en los adultos. mostrar una variedad de funcionamiento intelectual, pero
2. Tienden a ser muy sistematizados. sólo los niños con trastorno del espectro autista muestran
3. Son, habitualmente, de tipo persecutorio, grande- alteraciones selectivas en los comportamientos sociales de
za o hipocondría. comunicación tales como la comunicación intencional (p.
4. No se dan delirios en los niños sólo ideas sobreva- ej., dificultades en la comunicación deliberada, dirigida a
lora das. una meta y con objeto de influir en el comportamiento del
destinatario). Los niños con trastorno de apego reactivo
Respuesta correcta:3 muestran un funcionamiento de comunicación social com-
parable a su nivel general de funcionamiento intelectual.
Por último, los niños con trastorno del espectro autista nor-
malmente muestran una conducta de apego que es caracte-
rística de su nivel de desarrollo. Por el contrario, los niños
82. Según el DSM 5, señale a partir de qué carac- con trastorno de apego reactivo lo hacen de una manera
terísticas pueden diferenciarse, en niños pequeños, el rara o inconsecuente”. American Psychiatric Association,(
Trastorno de apego reactivo del Trastorno del espectro 2014), DSM - 5: Manual diagnóstico y estadístico de los
autista: trastornos mentales, 5a edición, Madrid: Editorial Médica
Panamericana.
1. Analizar los síntomas nucleares que presenta ya
que no hay semejanzas entre un trastorno y otro.
2. La historia de cuidados previos al desarrollo del
trastorno, así como observar la presencia de comporta-
83. Señale cuál de los siguientes elementos no está
mientos repetitivos e intereses restringidos.
presente en el programa para el tratamiento de las adic-
3. La manifestación de expresiones inhibidas de las ciones comportamentales de Echeburúa (Echeburúa
emociones positivas. y cols., 2005 y Echeburúa, 2009):
4. La presencia de retraso cognitivo del lenguaje y de
alteraciones de la reciprocidad social.
1. Análisis de los estímulos asociados a la adicción.
2. Control de estímulos asociados a la adicción.
Respuesta correcta:2
3. Atención plena a los estímulos asociados a la adic-
ción.
Es importante el diagnóstico diferencial entre el Trastorno
de apego reactivo y el Trastorno del espectro autista ya que 4. Exposición a estímulos asociados a la adicción.
ambos comparten algunas de sus características y manifes-
taciones (opción 1 incorrecta). Con respecto a esto, el DSM Respuesta correcta:3
5 recoge lo siguiente; “Los comportamientos sociales abe-
rrantes no sólo se manifiestan en los niños pequeños con

24
El programa de Echeburúa y Báez (1994) para el para el El caso descrito en el enunciado se podría corresponder con
tratamiento de las adicciones consta de: 1) Control de es- el trastorno explosivo intermitente (opción 2 correcta). Se-
tímulos (opciones 1 y 2 incorrectas); 2) Exposición con ñalemos los criterios de dicho trastorno que se cumplen en
prevención de respuesta o EPR (opción 4 incorrecta); y 3) este enunciado: Una niña de 9 años [CRITERIO E: El in-
Terapia de grupo cognitivo-conductual (simultánea al trata- dividuo tiene una edad cronológica de seis años por lo me-
miento individual, 5-6 pacientes). APIR Manual de Trata- nos (o un grado de desarrollo equivalente)] se caracteriza
mientos Psicológicos, Tema 2. por presentar fuertes y muy frecuentes agresiones verbales
y arrebatos agresivos físicos esporádicos (pero más de 2
84. ¿En la infancia, las parasomnias se conside- al mes), que producen destrucción de propiedades, daño a
ran?: personas y animales [CRITERIO A. Arrebatos recurrentes
en el comportamiento que reflejan una falta de control de
1. Trastornos secundarios del sueño, e incluyen nar- los impulsos de agresividad, manifestada por una de las si-
colepsia, terrores nocturnos y sonambulismo. guientes: 1. Agresión verbal o física sin daños en los últimos
2. Trastornos primarios del sueño, e incluyen pesadi- 3 meses; o 2. Tres arrebatos que provoquen daños o destruc-
llas, terrores nocturnos y sonambulismo. ción en los últimos doce meses], que no son premeditados
[CRITERIO C: Los arrebatos agresivos recurrentes no son
3. Trastornos del sueño y de la motricidad, e inclu-
premeditados (es decir, son impulsivos o provocados por
yen síndrome de piernas inquietas y sonambulismo.
la ira) ni persiguen ningún objetivo tangible (p. ej., dinero,
4. Un tipo de insomnio que incluye la apnea obstruc- poder, intimidación)], y suelen durar unos 20 o 30 minutos,
tiva y las pesadillas. echándole la culpa siempre a lo que ocurre a su alrededor,
no existiendo un periodo previo prodrómico. La opción 1
Respuesta correcta:2 es incorrecta, ya que el caso presentado en el enunciado no
presenta ni problemas de impulsividad (ej.: con frecuencia
Al preguntarnos por parasomnias lo primero que debe- juguetea con o golpea las manos o los pies o se retuerce en
mos de hacer es conocer su definición y qué trastornos lo el asiento, con frecuencia corretea o trepa en situaciones en
conforman. Se trata de trastornos que se caracterizan por las que no resulta apropiado,…) ni atencionales (ej.: con
comportamientos o fenómenos fisiológicos anormales que frecuencia falla en prestar la debida atención a detalles o
tienen lugar durante el sueño o en las fases de transición por descuido se cometen errores en las tareas, con frecuen-
del sueño-vigilia (trastorno por pesadillas, del despertar del cia parece no escuchar cuando se le habla directamente,…
sueño no REM (que aúna sonambulismo y terrores), com- ) propios del TDAH, ni un comportamiento en el que no se
portamental del sueño REM y síndrome de piernas inquie- respetan los derechos básicos de otros, las normas o reglas
tas) (opción 2 correcta). En contraposición, las disomnias sociales, característico del trastorno de conducta (opción 1
son alteraciones de la cantidad, calidad u horario del sueño incorrecta). La opción 3 es incorrecta porque para el tras-
(insomnio, hipersomnia, narcolepsia (opción 1 incorrecta), torno de disregulación disruptiva del estado de ánimo no se
trastornos del sueño relacionados con la respiración como contempla la condición de que coexistan con los síntomas
las apneas (opción 4 incorrecta) y trastornos del ritmo cir- períodos de bienestar o euforia (opción 3 incorrecta). La
cadiano). La opción 3 no puede ser correcta ya que no se opción 4 es incorrecta porque no se cumplen criterios para
pueden considerar trastornos puramente motrices o moto- un episodio maníaco (período bien definido de estado de
res, que haría referencia a aquellos trastornos con proble- ánimo anormalmente y persistentemente elevado, expansi-
mas en las habilidades motoras (opción 3 incorrecta). APIR vo o irritable, y un aumento anormal y persistente de la
Manual de Psicología Clínica, Tomo II, Tema 17. actividad o la energía junto a, por ejemplo, disminución de
la necesidad de dormir, sentimientos de grandeza, verbo-
rrea,…) (opción 4 incorrecta). APIR Manual de Psicología
85. Una niña de 9 años se caracteriza por pre- Clínica, Tomo II, Tema 18.
sentar fuertes y muy frecuentes agresiones verbales y
arrebatos agresivos físicos esporádicos (pero más de
2 al mes), que producen destrucción de propiedades, 86. Un niño de 8 años presenta accesos de cólera fre-
daño a personas y animales, que no son premeditados, cuentes, de tres a cuatro por semana, rompe cosas y se
y suelen durar unos 20 o 30 minutos , echándole la culpa enfada mucho, está irritado en general, y estas caracte-
siempre a lo que ocurre a su alrededor, no e x i s t i e n d o rísticas son más frecuentes en casa, aunque típicas
un periodo previo prodrómico. ¿Cuál de los siguien- también en el colegio. ¿Qué trastorno presenta?:
tes trastornos podría presentar?:
1. Trastorno depresivo mayor.
1. Trastorno por déficit de atención e hiperactividad,
y se podría diagnosticar también trastorno de con- 2. Trastorno bipolar.
ducta. 3. Trastorno explosivo intermitente.
2. Trastorno explosivo intermitente. 4. Trastorno de desregulación disruptiva del estado
de ánimo.
3. Trastorno de desregulación disruptiva del estado
de ánimo, siempre que se acompañe de periodos
Respuesta correcta:4
estables de bienestar y euforia.
4. Trastorno bipolar de inicio en la infancia.
El caso descrito en el enunciado se podría corresponder con
el trastorno de desregulación disruptiva del estado de áni-
Respuesta correcta:2

25
mo (opción 4 correcta). Señalemos los criterios de dicho fiable la información recogida con estas técnicas en edades
trastorno que se cumplen en este enunciado: Un niño de inferiores a los 7 años (Opción 2 correcta; Opciones 1, 3 y
8 años [CRITERIO G. El primer diagnóstico no se debe 4 incorrectas). Esto podemos relacionarlo con que previa-
hacer antes de los 6 años o después de los 18 años; CRITE- mente a los 7 años, el niño o la niña se encuentran en el es-
RIO H. Por la historia o la observación, los Criterios A-E tadio preoperacional del desarrollo cognitivo, y no es hasta
comienzan antes de los 10 años] presenta accesos de cólera aproximadamente los 7 años que se alcanza el estadio de
frecuentes [CRITERIO A. Accesos de cólera graves y re- las operaciones concretas, en el cual comienzan a desarro-
currentes que se manifiestan verbalmente (p. ej., rabietas llarse múltiples habilidades entre ellas las relacionadas con
verbales) y/o con el comportamiento (p. ej., agresión física la identificación de las propias características psicológicas
a personas o propiedades) cuya intensidad o duración son y la diferenciación de los aspectos físicos y mentales de
desproporcionadas a la situación o provocación], de tres a uno mismo. APIR Manual Evaluación Psicológica, Tema 4.
cuatro por semana [CRITERIO C. Los accesos de cólera se
producen, en término medio, tres o más veces por semana],
rompe cosas y se enfada mucho, está irritado en general 88. Respecto a las técnicas de registro de la conducta de
[CRITERIO D. El estado de ánimo entre los accesos de un sujeto al que estamos observando, ¿qué característi-
cólera es persistentemente irritable o irascible la mayor cas tiene el registro narrativo?:
parte del día, casi todos los días, y es observable por parte
de otras personas (p. ej., padres, maestros, compañeros)], 1. Es la técnica de registro observacional que permi-
y estas características son más frecuentes en casa, aunque te un mayor rigor científico respecto a las demás técnicas.
típicas también en el colegio [CRITERIO F. Los Criterios 2. Está dirigido a evaluar las interacciones que se
A y D están presentes al menos en dos de tres contextos producen entre el ambiente y la conducta, o entre un divi-
(es decir, en casa, en la escuela, con los compañeros) y son duo y un grupo.
graves al menos en uno de ellos]. La opción 1 es incorrecta 3. Consisten en realizar descripciones de lo observa-
porque no aparece irritabilidad (propia en niños) dentro del do sin existir una estructuración previa que dirija la reco-
resto de síntomas depresivos (ej.: disminución o aumento gida de datos.
de apetito, insomnio o hipersomnia, baja autoestima,…) 4. Se basan en técnicas escalares y sirven para cla-
característicos del trastorno depresivo mayor (opción 1 in- sificar las actividades de un sujeto según dimensiones pre-
correcta). La opción 2 es incorrecta porque no se cumplen viamente establecidas.
criterios para un episodio maníaco (período bien definido
de estado de ánimo anormalmente y persistentemente ele-
Respuesta correcta:3
vado, expansivo o irritable, y un aumento anormal y per-
sistente de la actividad o la energía junto a, por ejemplo,
disminución de la necesidad de dormir, sentimientos de Las técnicas de observación consisten en una serie de ins-
grandeza, verborrea,…) (opción 2 incorrecta). La opción 3 trumentos que empleamos, como su propio nombre indica,
es incorrecta porque el trastorno explosivo intermitente se para la observación. En relación a la clasificación de es-
caracteriza más por conductas agresivas como problemas tas técnicas, solemos organizarlas en un continuo en base
de control de impulsos que por irritabilidad constante más al nivel de estructuración . Ordenadas de menor a mayor
propia del trastorno de disregulación disruptiva de estado estructuración encontramos: 1) los registros narrativos, 2)
de ánimo (opción 3 incorrecta). APIR Manual de Psicolo- las escalas de apreciación, 3) los protocolos observaciona-
gía Clínica, Tomo I, Tema 5. les de conducta, 4) los códigos o sistemas de categorías,
5) los registros de productos de conducta y 6) dispositivos
electrónicos. En concreto en los registros narrativos, el ob-
87. A partir de qué edad mínima (aproximada)
servador va tomando nota de lo que está sucediendo en la
es fiable la recogida de información mediante autoinfor-
mes en los niños: situación de observación, por lo tanto, no hay una planifica-
ción previa de la recogida de datos (Opción 3 correcta). En
el empleo de esta técnica destacan dos fuentes de error que
1. 5 años. afectan a su fiabilidad. Por un lado, los observadores pue-
2. 7 años. den utilizar distintas descripciones verbales para una mis-
3. 10 años. ma conducta o patrón de conductas, y por otro, se puede lle-
4. 12 años. gar a categorizar o a dar distinta significación a los eventos.
Esto último puede controlarse previo entrenamiento de los
Respuesta correcta:2 observadores en el uso de un mismo lenguaje descriptivo.
Por estos motivos estas técnicas se consideran útiles como
trabajo previo a una recogida de datos más precisa o cuan-
Los autoinformes son manifestaciones verbales que emite do la conducta a observar es de baja frecuencia, pero indu-
un sujeto sobre cualquier manifestación propia, bien sea dablemente no son la técnica de registro observacional que
cognitiva, fisiológica o motora. Por lo tanto, el observador permite un mayor rigor científico (Opción 1 incorrecta),
es el propio sujeto. Es la técnica de evaluación más utiliza- siendo estas aquellas que presentan mayor estructuración.
da y es empleada por todos los modelos teóricos, y abarca Respecto al resto de alternativas de respuesta, la número
una amplia variedad temática. En cuanto a sus limitaciones, 2 hace referencia a los mapas de conducta o formatos de
una de las fundamentales que debemos tener en cuenta de campo, que son un tipo de protocolos observacionales de
cara a la evaluación infantil es el nivel de desarrollo cog- conducta (Opción 2 incorrecta) y la número 4 hace referen-
nitivo del niño o niña que estamos evaluando, no siendo cia a las escalas de apreciación (Opción 4 incorrecta). APIR

26
Manual Evaluación Psicológica, Tema 3. pende de un análisis global. Son técnicas muy influenciadas
por la teoría Psicoanalítica y se basan en que las respues-
tas que da el sujeto ante los estímulos que se le presentan
89. Cuando estamos evaluando la conducta de son proyecciones de los sentimientos, emociones, conflic-
un sujeto a través de la observación, ¿qué se recomien- tos, etc., más inconscientes del sujeto. Las alternativas de
da para reducir el sesgo de reactividad?: respuesta que nos proponen, la única que hace referencia
a una técnica proyectiva es el Test de apercepción temá-
1. Maximizar la interacción entre sujeto y observa- tica (TAT) de Murray (Opción 4 correcta). Esta técnica se
dor para que el sujeto se sienta más cómodo cuando es ob- engloba dentro de las técnicas temáticas o interpretativas
servado. para adultos y consiste en 31 láminas, de las cuales 30 son
2. Utilizar un amplio periodo de observación para ilustraciones estructuradas donde se representa una escena
que el sujeto se habitúe a la situación de observación. determinada (como temas laborales, de peligro y miedo,
3. Utilizar un único observador para que la recogida paterno-filiales) y 1 lámina es en blanco (para que el su-
de datos sea más homogénea. jeto imagine una escena). La tarea del sujeto consiste en
4. No utilizar dispositivos ocultos o a distancia que ir narrando una historia para cada lámina presentada que
podrían disminuir la validez ecológica de la observación. contenga un principio, un desarrollo y un final, y donde se
muestren los pensamientos y los sentimientos de los perso-
najes. Respecto al resto de alternativas de respuesta, el Di-
ferencial semántico de Osgood y la Técnica de la rejilla de
Respuesta correcta:2 Kelly son técnicas subjetivas (Opciones 1 y 3 incorrectas)
mientras que las pruebas de aptitudes perceptivas se cen-
La observación es la técnica de investigación básica, que tran en evaluar rapidez y precisión perceptiva mediante test
supone la conducta voluntaria de recogida de datos con el en los que el sujeto ha de localizar rápidamente una figura
fin de formular y posteriormente comprobar hipótesis. Esta en base a los detalles visuales, lo cual no forma parte de las
técnica, del mismo modo que cualquier otra, no está 100% técnicas proyectivas (Opción 2 incorrecta). APIR Manual
libre de fallos, y podemos encontrar ciertos tipos de sesgo Evaluación Psicológica, Tema 8.
propios de las técnicas de observación. Estos sesgos se pue-
den clasificar en tres grupos según la fuente de error, en- 91. En las Escalas de inteligencia de Wechsler
contrándonos con sesgos procedentes del sujeto observado, para adultos (WAIS-III, WAIS-IV), ¿cuáles de estas
sesgos procedentes del observador y sesgos procedentes de escalas forman parte del Índice de memoria de traba-
la modalidad de observación. En concreto la reactividad jo?:
es un sesgo propio del sujeto observado que consiste en
la modificación de la propia conducta como consecuencia
1. Clave de números y Búsqueda de Símbolos.
de saberse observado. Es directamente proporcional al gra-
do de modificación ambiental de la observación y puede 2. Vocabulario y Semejanzas.
reducirse, entre otras técnicas, minimizando la interacción 3. Aritmética y Dígitos.
entre sujeto y observador para que el sujeto se sienta más 4. Cubos y Matrices.
cómodo cuando es observado (Opción 1 incorrecta), esto
se puede conseguir con dispositivos ocultos o a distancia Respuesta correcta:3
(Opción 4 incorrecta). Otra forma de reducir la reactividad
es utilizando un amplio periodo de observación para que el
sujeto se habitúe a la situación de observación (Opción 2 Las Escalas de inteligencia de Wechsler son escalas ba-
correcta). Por otra parte, utilizar un único observador para sadas en metodología racional y arrojan puntuaciones de
que la recogida de datos sea más homogénea sería una for- CI. En concreto el WAIS se corresponde con la versión
ma de aumentar la fiabilidad, pero no de disminuir la reac- para adultos y tiene como finalidad el evaluar la inteligen-
tividad (Opción 3 incorrecta). APIR Manual de Evaluación cia general. Uno de los cambios que introduce el WAIS-
Psicológica, Tema 3. III es la inclusión de los índices de Comprensión verbal,
Memoria de trabajo, Organización perceptiva y Velocidad
de procesamiento. Las pruebas que se engloban en cada
90. ¿Cuál de las siguientes es una técnica de eva- uno de estos índices son los siguientes: 1) Comprensión
luación proyectiva?: verbal: Semejanzas, Vocabulario e Información (Opción
2 incorrecta); 2) Memoria de trabajo: Aritmética, Dígitos
1. El Diferencial semántico de Osgood. y Letras y Números (Opción 3 correcta); 3) Organización
2. El Test de aptitudes perceptivas. perceptiva: Cubos, Matrices, Figuras Incompletas (Opción
4 incorrecta); y 4) Velocidad de procesamiento: Clave de
3. La Técnica de la rejilla.
Números y Búsqueda de Símbolos (Opción 1 incorrecta).
4. El Test de apercepción temática de Murray. APIR Manual Evaluación Psicológica, Tema 9.
Respuesta correcta:4 92. Los test psicométricos se pueden clasificar
en test de rendimiento máximo y test de rendimiento
Las técnicas proyectivas se definen como técnicas especial- típico según la demanda que se le hace al sujeto. ¿En
mente sensibles para revelar aspectos inconscientes de la qué consiste un test de rendimiento típico?:
conducta, y en las que generalmente el material estimular
que se presenta es ambiguo, por lo que la interpretación de-

27
1. El sujeto tiene que escoger la única respuesta ver- Kaufman (Opción 1 incorrecta) y la número 3 al enfoque
dadera entre una serie de alternativas. de Feuerstein de evaluación del potencial de aprendizaje
2. Se trata de valorar el rendimiento más alto de una (Opción 3 incorrecta). APIR Manual Evaluación Psicoló-
persona en un constructo determinado como, por ejemplo, gica, Tema 9.
la inteligencia.
3. El objetivo principal es valorar la cantidad de 94. La evaluación de la personalidad se ha hecho desde
ítems que resuelve la persona en un tiempo limitado. diversos modelos y teorías. Desde las teorías factoria-
4. El objetivo es valorar la manera habitual de com- les, el test más representativo que ha inspirado el desa-
portarse de un individuo. rrollo de otros, es:

Respuesta correcta:4 1. Minnesota multiphasic personality inventory,


(MMPI).
Existen múltiples formas de clasificar las distintas técnicas 2. Sixteen personality factor questionnaire, (16PF).
psicométricas. A este respecto, Cronbach distingue entre: 3. Gordon personal profile (GPP).
1) Técnicas de rendimiento máximo, objetivas o de ejecu- 4. Tennessee Self Concept Scale (TSCS).
ción, en las cuales el sujeto evaluado debe rendir al máximo
de sus posibilidades, para esto se suele trabajar con pruebas Respuesta correcta:2
de tiempo limitado (Opción 3 incorrecta). Dentro de esta
clasificación se encuentran los test de inteligencia (Opción El modelo psicométrico de evaluación de la personalidad
2 incorrecta), los test de aptitud general, los test de apti- se enmarca dentro de los modelos de rasgos, enfatizan los
tudes específicas y los test de rendimiento académico. Y aspectos nomotéticos y cuantitativos. Sin embargo, los cri-
2) Técnicas de rendimiento típico, subjetivas o de respues- terios desde los que se han desarrollado las distintas prue-
ta, las cuales pretenden conocer el comportamiento coti- bas psicométricas difieren, pudiendo clasificarse en criterio
diano del sujeto en una determinada situación o en varias racionales, criterios empíricos, criterios factoriales y crite-
circunstancias diferentes (Opción 4 correcta). Implican la rios combinados. La evaluación desde criterios racionales
estabilidad de la conducta a lo largo del tiempo. Entre estas fue la primera en aparecer y se basa en la idea de que existe
se incluyen las técnicas de evaluación de la personalidad una correspondencia entre la respuesta que el sujeto da al
o de variables emocionales, Inventarios autodescriptivos, ítem presentado y determinados estados internos hipoté-
Test situacionales y Técnicas Proyectivas. APIR Manual ticos postulados desde una teoría. La primera prueba en
de Evaluación Psicológica, Tema 2. aparecer que se basa en estos criterios es la Hoja de Datos
Personales (PDS) de Woodworth (1914). Las pruebas de
93. Las Escalas McCarthy de aptitudes y psicomo- evaluación desde criterios empíricos son pruebas que están
tricidad par niños (MSCA): orientadas por un criterio externo, dando nula importancia
a la unión entre la respuesta emitida por un individuo y el
1. Evalúan las aptitudes del niño a través de dos es- comportamiento que predice. Los cuestionarios más repre-
calas principales: la de procesamiento secuencial y la de sentativos de esta categoría son el MMPI (Opción 1 inco-
procesamiento simultáneo. rrecta) y el CPI. La evaluación desde criterios factoriales
2. Son adecuadas para evaluar a niños muy pequeños consiste en que con el análisis factorial de los ítems se con-
y con dificultades de aprendizaje. sigue agruparlos en factores, siendo estos factores conside-
3. Se utilizan para valorar la inteligencia desde la rados desde esta perspectiva como rasgos dimensionales.
perspectiva de la evaluación dinámica, es decir, evalúan el Esta estrategia es la más común en la construcción de los
potencial de aprendizaje del niño. cuestionarios para la evaluación de rasgos y dimensiones
4. Son útiles para valorar la motricidad y las aptitu- de personalidad, siendo el test más representativo y que
des de niños con inteligencia dentro de la media, ya que son sirvió de base para el resto el 16PF de Cattel (Opción 2 co-
poco sensibles para evaluar a niños con retraso intelectual. rrecta). Finalmente, respecto a la evaluación desde criterios
combinados, estas pruebas personalidad han sido creadas a
Respuesta correcta:2 partir de la combinación de criterios racionales, empíricos
y factoriales, siendo las más conocidas en este ámbito las
realizadas por Millon y Cloninger. Respecto al resto de op-
ciones de respuesta, el Perfil personal de Gordon (GPP) es
una prueba factorial basada en el 16PF que evalúa Ascen-
Las Escalas McCarthy de aptitudes y psicomotricidad para dencia, Responsabilidad, Estabilidad y Sociabilidad (Op-
niños tienen como objetivo la evaluación de las capacida- ción 3 incorrecta). La Tennessee self concept scale (TSCS)
des cognoscitivas de niños pequeños (2 años y 6 meses - 8 también es una prueba factorial, pero de menor importancia
años y 6 meses) o con dificultades de aprendizaje (Opción 2 que el 16PF (Opción 4 incorrecta). APIR Manual de Eva-
correcta; Opción 4 incorrecta). Estas escalas proporcionan luación Psicológica, Tema 11.
una medida general del funcionamiento intelectual a través
del Índice Cognoscitivo General (ICG) y muestran un perfil
de capacidades. Esto lo hace empleando 6 escalas, que son:
Escala verbal, Escala perceptivo manipulativa, Escala nu- 95. ¿Qué instrumento para la medida de la inteligencia
mérica, Escala de memoria, Escala de motricidad y Escala se diseñó para evaluar el factor “g” tal y como fue defi-
general cognoscitiva. Respecto al resto de alternativas de nido por Spearman?:
respuesta, la número 1 hace referencia la escala K-ABC de

28
1. Escalas de inteligencia de Weschler. el entrevistado habla según sus necesidades, se formulan
2. Escala de inteligencia de Stanford-Binet. preguntas abiertas y el entrevistador tiene que evitar ser di-
3. Escalas de capacidad diferencial. rectivo. APIR Manual de Evaluación Psicológica, Tema 5.
4. Test de matrices progresivas de Raven.
97. ¿Qué tipo de técnica proyectiva es el test “Di-
bujo de la figura humana”?:
Respuesta correcta:4
1. Estructural.
Desde un enfoque clásico las pruebas de inteligencia se 2. Temática.
dividen en test basados en el método racional y test ba- 3. Constructiva.
sados en el método factorial. Test basados en la metodo- 4. Expresiva.
logía racional o enfoques clínicos son pruebas que están
elaboradas según la teoría a la que pertenezca el autor. El Respuesta correcta:4
máximo exponente de esta orientación es Wechsler, con sus
escalas (Opción 1 incorrecta). También podemos englobar Las técnicas proyectivas se definen como técnicas muy
en este grupo las pruebas desarrolladas siguiendo los traba- influenciadas por la teoría Psicoanalítica y especialmente
jos de Binet, donde encontramos la escala de inteligencia sensibles para revelar aspectos inconscientes de la conduc-
Stanford-Binet (Opción 2 incorrecta). Por otra parte, los ta, en las que generalmente el material estimular que se
test basados en la metodología factorial son pruebas ela- presenta es ambiguo, por lo que la interpretación depende
boradas a partir del análisis factorial, siendo Spearman el de un análisis global. Una de las principales clasificaciones
pionero de esta forma de entender la inteligencia. Dentro de las técnicas proyectivas es la propuesta por Fernández
de estas pruebas podemos encontrar dos tipos: 1) los test Ballesteros, que divide las técnicas proyectivas en: 1) Es-
de factor G, en los cuales se realiza una única medida re- tructurales: Su principal característica es la presentación
lacionada con el factor G para conocer el nivel intelectual de estímulos poco estructurados los cuales el sujeto ha de
del sujeto y siendo el ejemplo más característico el test de estructurar. Entre estas técnicas destacan el Test de Rors-
Matrices Progresivas de Raven (Opción 4 correcta); y 2) chach, el HIT de Holtzman y la Z de Zulliger. 2) Temáticas:
los test de aptitudes, que evalúan una serie de capacidades Estas pruebas se caracterizan por presentar láminas sobre
o habilidades específicas que posee el sujeto. El ejemplo las que el sujeto debe narrar una historia. Las pruebas prin-
más característico es el PMA de Thurstone. APIR Manual cipales de este grupo son el TAT, el TRO, el PFT y el CAT.
de Evaluación Psicológica, Tema 9. 3) Expresivas: En estas pruebas se le dice al sujeto que ha
de hacer un dibujo. Las técnicas principales de este subgru-
96. La Entrevista diagnóstica DIS (Diagnostic I n - po son el Test de la familia de Corman, el Test de la figura
terview Schedule), que proporciona información sobre humana de Machover (Opción 4 correcta; Opciones 1, 2 y
el inicio, duración y actualidad de los síntomas, es un 3 incorrectas) y el Test del árbol de Koch. 4) Constructivas:
ejemplo de entrevista: En estas pruebas se dan unas instrucciones y el sujeto ha de
organizar el material que se le da, siendo las pruebas prin-
1. Estructurada. cipales el Test del pueblo, el Test de la aldea imaginaria y
2. Semiestructurada. el Juego diagnóstico. Y finalmente, 5) Asociativas: En estas
3. Motivacional. técnicas el sujeto ha de responder a los estímulos que se le
4. Abierta. presentan y sus respuestas se asocian a ellos. Entre estas
pruebas encontramos las Fábulas de Düss, la Asociación de
Respuesta correcta:1 palabras y las Frases incompletas. APIR Manual Evalua-
ción Psicológica, Tema 8.
Existen múltiples formas de clasificación de las entrevistas,
una de las fundamentales es según su nivel de estructura-
ción. En esta clasificación nos encontramos en primer lugar 98. Cuando el comportamiento a observar es no
con las entrevistas estructuradas, que son entrevistas con discreto y por tanto resulta difícil determinar cuándo
preguntas prefijadas y sistema para codificar las respuestas empieza y cuándo acaba, el método de medición más
y analizarlas posteriormente. los principales ejemplos son útil es el método de intervalos, por ejemplo, el muestreo
la DIS (Diagnostic Interview Schedule) (Opción 1 correc- de intervalo completo o total. En este muestreo:
ta; Opciones 2, 3 y 4 incorrectas), DISC-R (Diagnostic in-
terview Structure For Children) PSE, PRIME-MD, DICA, 1. Se requiere que la conducta aparezca al menos una
DICA-R y CHIPS. Por otra parte, las entrevistas semies- vez a lo largo del intervalo.
tructuradas consisten en un guión previo que le permita 2. Se registra la conducta que ocurre en el momento
al entrevistador cierta flexibilidad y capacidad de adapta- en el que termina el intervalo.
ción al cliente y a las circunstancias de la entrevista. Son 3. La conducta no se registra si no se produce du-
especialmente útiles cuando se desea tener un diagnósti- rante la totalidad del intervalo de observación establecido.
co rápido, fiable y válido. Algunos ejemplos son la SCID
I (Structured Clinical Interview for DSM-IV Axis I) y la 4. Se registra la frecuencia de las conductas que apa-
SCID II (Structured Clinical Interview for DSM-IV Axis recen durante el intervalo.
II), la SCAN, la CIDI de la OMS y la IPDE. Finalmente las
entrevistas no estructuradas o libres son aquellas en las que Respuesta correcta:3

29
El muestreo consiste en las distintas técnicas utilizadas para dificultades de aprendizaje. Estas escalas proporcionan una
obtener muestras representativas de aquello que es objeto medida general del funcionamiento intelectual a través del
de observación. Dentro de los distintos tipos de muestreos Índice Cognoscitivo General (ICG) y muestran un perfil de
nos encontramos con el muestreo de tiempo y dentro de capacidades: capacidad verbal, de razonamiento no verbal,
este subtipo nos encontramos con el muestreo de intervalos aptitud numérica, memoria a corto plazo y coordinación
de observación, que consiste en dividir el tiempo total de (Opción 2 incorrecta). Finalmente, el TONI-2 permite la
observación en periodos temporales iguales. En cada inter- estimación del funcionamiento intelectual general median-
valo, el observador señala si la conducta ha aparecido o no, te la evaluación de la capacidad para resolver problemas
con independencia del número de veces que haya ocurrido abstractos de tipo gráfico y no verbal (Opción 3 incorrecta).
(Opción 4 incorrecta). Este tipo de muestreo resulta útil en APIR Manual Evaluación Psicológica, Tema 9
conductas no discretas, situaciones donde es difícil deter-
minar cuándo empieza y cuándo acaba la conducta y en 100. Entre las técnicas de entrevista verbal no direc-
conductas con una tasa de frecuencia muy elevada. Dentro tivas se encuentra:
del muestreo de intervalo nos encontramos con los siguien-
tes tipos: 1) Muestreo de intervalo total o completo: En este 1. La proxemia.
tipo de muestreo para registrar la conducta ésta debe estar 2. La paráfrasis.
presente durante todo el intervalo y resulta útil en conduc- 3. La Kinesia.
tas que queremos que se den durante largo tiempo, (p. ej., la 4. La sincronía.
atención de un niño en clase) (Opción 3 correcta). 2) Mues-
treo de intervalo parcial: En este caso, para registrar la con- Respuesta correcta:2
ducta ésta debe aparecer en algún momento del intervalo y
resulta útil para conductas breves de alta frecuencia (p. ej., En esta pregunta se busca una técnica de entrevista verbal
tics o decir tacos) (Opción 1 incorrecta). Y 3) Muestras de no directiva y se nos presentan una serie de alternativas
intervalo momentáneas: En este subgrupo, para registrar la de respuesta, de las cuales la Paráfrasis es el única que se
conducta ésta debe aparecer en un momento predetermi- considera como una técnica de entrevista y la única con un
nado del intervalo de observación, al inicio o al final del componente verbal (Opción 2 correcta). En cualquier caso
mismo (Opción 2 incorrecta). APIR Manual Evaluación procedemos a analizar una por una la definición de las cua-
Psicológica, Tema 3. tro alternativas de respuesta. En primer lugar, la proxémica
hace referencia a la conducta espacial. Su observación se
99. Respecto a la evaluación de la inteligencia infantil, centra en el análisis de la conducta en relación a aspectos
¿qué prueba permite medir la inteligencia general estáticos como el lugar en el espacio y dinámicos como el
según dos subescalas: verbal y no verbal?: conjunto de desplazamientos de un sujeto, la realización de
trayectorias… (Opción 1 incorrecta). Por otra parte, la Ki-
nesia hace referencia al lenguaje corporal y se centra en el
1. El K-Bit de Kaufman. significado expresivo, apelativo o comunicativo de los mo-
2. Las Escalas McCarthy (MSCA). vimientos corporales y de los gestos no verbales (Opción 3
3. El TONI-2. incorrecta). Respecto a la cuarta alternativa de respuesta,
4. El K-ABC de Kaufman. la sincronía puede hacer referencia a múltiples conceptos
relacionados con el análisis temporal de los sucesos pero
Respuesta correcta:1 en ningún caso se refiere a una conducta verbal (Opción
4 incorrecta). De este modo concluimos que la paráfrasis,
Dentro de las escalas de inteligencia infantil nos encontra- que consiste en que el entrevistador expresa con sus propias
mos con las escalas de inteligencia de Kaufman, entre las palabras el contenido de lo que ha explicado el entrevista-
que ubicamos el K-ABC y el K-BIT. La primera de estas do, para ver si ha comprendido bien lo que le ha querido
pruebas es una batería elaborada con el objetivo de evaluar decir, es la única alternativa de respuesta correcta (Opción
de forma diferenciada inteligencia y rendimiento académi- 2 correcta). APIR Manual Evaluación Psicológica, Tema 5.
co, centrándose en la forma de procesamiento, más que en
el resultado final. Resulta útil para la evaluación de niños
con trastornos del aprendizaje, retraso del lenguaje o sor- 101. En general, la evaluación dinámica del poten-
dos, y cuenta con escasa pruebas verbales. Resulta poco cial de aprendizaje es apropiada para determinar el
adecuada para sujetos brillantes o superdotados (Opción 4 tratamiento en casos de:
incorrecta). En relación al K-BIT, es un test de screening
(no diagnóstico), que se aplica en 15-30 minutos y es de 1. Sujetos de bajo nivel cognitivo.
fácil corrección. Esta prueba evalúa la inteligencia verbal 2. Sujetos que presentan rendimiento académico
y no verbal (Opción 1 correcta) y está considerado como adecua do.
una gran medida de la Inteligencia General. Está compues-
3. Sujetos con dificultades para aprendizajes gene-
to por dos subpruebas: 1) Vocabulario: Mide la inteligencia
rales.
verbal y consta de Vocabulario expresivo y Definiciones;
y 2) Matrices: Mide la inteligencia no verbal y está forma- 4. En aquellos casos en que sea preciso clarificar los
do por Dibujos y Figuras abstractas con el fin de eliminar pro cesos automáticos implícitos en los
la influencia cultural. Respecto al resto de alternativas de aprendizajes.
respuesta, el MSCA tienen como objetivo la evaluación
de las capacidades cognoscitivas de niños pequeños o con
Respuesta correcta:1

30
El modelo de evaluación dinámica se aplica fundamen- 3. Expresiva.
talmente en el ámbito clínico, educativo y social. En 4. Estructural.
concreto en el ámbito clínico se ha empleado para evaluar
el potencial de aprendizaje de sujetos con retraso mental, Respuesta correcta:3
dificultades de aprendizaje lector, ciegos o con dificultades
de lenguaje. Por otra parte, en el ámbito educativo ha sido Las técnicas proyectivas se definen como técnicas muy
empleado para la identificación de superdotación entre influenciadas por la teoría Psicoanalítica y especialmente
grupos sociocultural o lingüísticamente desfavorecidos. sensibles para revelar aspectos inconscientes de la conduc-
En base a esto no es de extrañar que sea en el dominio ta, en las que generalmente el material estimular que se
sociocultural donde la evaluación dinámica alcanza sus presenta es ambiguo, por lo que la interpretación depende
máximas aplicaciones, especialmente en la evaluación de un análisis global. Una de las principales clasificaciones
cognitiva de grupos pertenecientes a minorías étnicas (en de las técnicas proyectivas es la propuesta por Fernández
casos de inmigración, o grupos especiales). También se ha Ballesteros, que divide las técnicas proyectivas en: 1) Es-
aplicado en el ámbito laboral y en el de la tercera edad. A tructurales: Su principal característica es la presentación
de estímulos poco estructurados los cuales el sujeto ha de
modo general se establece que la evaluación dinámica del
estructurar. Entre estas técnicas destacan el Test de Ror-
potencial de aprendizaje es apropiada para determinar el schach, el HIT de Holtzman y la Z de Zulliger (Opción 4
tratamiento en casos de: 1) sujetos de bajo nivel cognitivo incorrecta). 2) Temáticas: Estas pruebas se caracterizan por
(Opción 1 correcta); 2) sujetos que presentan rendimiento presentar láminas sobre las que el sujeto debe narrar una
académico inadecuado que no puede ser justificado me- historia. Las pruebas principales de este grupo son el TAT,
diante las medidas cognitivas psicométricas clásicas (Op- el TRO, el PFT y el CAT. 3) Expresivas: En estas pruebas
ción 2 incorrecta); 3) sujetos con dificultades para apren- se le dice al sujeto que ha de hacer un dibujo. Las técnicas
dizajes específicos (Opción 3 incorrecta); y 4) en aquellos principales de este subgrupo son el Test de la familia de
casos en que sea preciso clarificar los procesos motivacio- Corman, el Test de la figura humana de Machover y el Test
nales implícitos en los aprendizajes (Opción 4 incorrecta). del árbol de Koch (Opción 3 correcta). 4) Constructivas:
APIR Manual Evaluación Psicológica, Tema 9. En estas pruebas se dan unas instrucciones y el sujeto ha
de organizar el material que se le da, siendo las pruebas
principales el Test del pueblo, el Test de la aldea imaginaria
y el Juego diagnóstico. 5) Asociativas: En estas técnicas el
102. En las Escalas de Inteligencia de Wechsler sujeto ha de responder a los estímulos que se le presentan
para adultos (WAIS-III y WAIS-IV), ¿de qué índice y sus respuestas se asocian a ellos. Entre estas pruebas en-
forma parte la prueba “Clave de números”?: contramos las Fábulas de Düss, la Asociación de palabras y
las Frases incompletas (Opción 1 incorrecta). Finalmente,
1. Memoria de trabajo. la opción de respuesta número 2 (Incorrecta) se correspon-
2. Comprensión verbal. de con la clasificación de Frank de las técnicas proyectivas
y se relaciona con las técnicas constructivas de la clasifica-
3. Velocidad de procesamiento.
ción de Fernández Ballesteros. APIR Manual Evaluación
4. Organización perceptiva. Psicológica, Tema 8.
Respuesta correcta:3 104. Atendiendo a la “finalidad” de la entrevista,
y en relación a la entrevista consultiva:
Las Escalas de inteligencia de Wechsler son escalas ba-
sadas en metodología racional y arrojan puntuaciones de 1. Suele tener un objetivo focal, con una finalidad
CI. En concreto el WAIS se corresponde con la versión muy concreta, en la que el entrevistado no va a
para adultos y tiene como finalidad el evaluar la inteligen- proseguir con un trabajo clínico posterior.
cia general. Uno de los cambios que introduce el WAIS- 2. Tiene un objetivo no terapéutico y de consejo.
III es la inclusión de los índices de Comprensión verbal,
3. Está orientada a establecer un diagnóstico en un
Memoria de trabajo, Organización perceptiva y Velocidad
paciente.
de procesamiento. Las pruebas que se engloban en cada
uno de estos índices son los siguientes: 1) Comprensión 4. Tiene por finalidad operar un cambio en una di-
verbal: Semejanzas, Vocabulario e Información (Opción 2 rección acordada por ambos, entrevistador y en-
incorrecta); 2) Memoria de trabajo: Aritmética, Dígitos y trevistado.
Letras y Números (Opción 1 incorrecta); 3) Organización
perceptiva: Cubos, Matrices, Figuras Incompletas (Opción Respuesta correcta:1
4 incorrecta); y 4) Velocidad de procesamiento: Clave de
Números (Opción 3 correcta) y Búsqueda de Símbolos . Existen múltiples formas de clasificación de las entrevistas
APIR Manual Evaluación Psicológica, Tema 9. y una de las fundamentales es según los objetivos plantea-
dos. A este respecto, Moreno Rosset plantea la siguiente
103. ¿Qué tipo de técnica proyectiva es el Test del clasificación: 1) Entrevistas diagnósticas: Se llevan a cabo
Árbol de Koch?: con el fin de evaluar y diagnosticar (Opción 3 incorrecta). 2)
Entrevistas consultivas: Pretenden dar una respuesta a una
pregunta en relación con un tema específico. Suele tener un
1. Asociativa.
objetivo focal, y no se va a proseguir con un trabajo clínico
2. Catártica.

31
posterior (Opción 1 correcta). 3) Entrevistas de orientación 1. Desarrollar baterías neuropsicológicas.
vocacional: Según las capacidades o intereses del sujeto, se 2. La construcción de test adaptativos informatiza-
le pretende orientar hacia qué estudios o qué ámbito profe- dos.
sional es el que mejor se adecua a él. 4) Entrevistas terapéu- 3. La utilización de la “realidad virtual”.
ticas: Su objetivo es llevar a cabo el tratamiento mediante 4. Emular el proceso que sigue el clínico en la eva-
entrevistas con el fin de conseguir un cambio acordado con lua ción para producir conclusiones
el paciente (Opción 4 incorrecta). 5) Entrevistas de inves- diagnósticas.
tigación: En estas se recoge información con el fin de si el
sujeto se puede adscribir o no a una investigación. APIR Respuesta correcta:4
Manual de Evaluación Psicológica, Tema 5.
Un sistema experto, es un sistema informático que emula
el razonamiento humano actuando tal y como lo haría un
experto en un área de conocimiento. De este modo, pode-
105. ¿Qué nivel de muestreo intrasesional estamos utili- mos establecer que los sistemas expertos son una de las
zando si en una sesión de clase se muestrea cada 30 aplicaciones de la inteligencia artificial que pretende si-
segundos a un alumno para ver si se mantiene en la po- mular el razonamiento humano, de la misma manera que
sición correcta?: lo haría un experto en un área de especialización. En base
a esto no nos queda duda de que la única alternativa de
1. Continúo de toda la sesión. respuesta apropiada es la número 4, ya que consideramos
2. De eventos. al clínico como el experto cuya función debe emular este
3. Temporal. sistema, más concretamente en el proceso de evaluación y
4. Focal. diagnostico (Opción 4 correcta). Por otra parte, el resto de
alternativas de respuesta quedan claramente descartadas ya
que en ninguna de ellas encontramos esta característica de
“emular el razonamiento humano actuando tal y como lo
Respuesta correcta:3
haría un experto” sino que todas ellas hablan del desarrollo
de herramientas de evaluación o intervención (Opciones 1,
Podemos distinguir dos tipos de muestreo de tiempo, en-
2 y 3 incorrectas). APIR Manual de Evaluación Psicológi-
contrándonos por un lado con el muestreo intersesional,
ca, Tema 1.
que consiste en analizar el periodo de observación, perio-
dicidad de las sesiones, número mínimo de sesiones y cri-
terios de inicio y fin de sesión. Y por otro lado, el muestreo
intrasesional, en el que se analizaría la información regis-
trada dentro de cada sesión. Esta última forma podemos 107. Cuando se aplica la escala de inteligencia We -
dividirla en cuatro subtipos de muestreo, que son: 1) Re- chsler (WPPSI-IV) en niños menores de cuatro años se
gistro continuo de toda la sesión: En este modo se registra incluye:
toda la información relevante de la sesión según nuestro
objetivo. Por ejemplo: los desplazamientos mediante una 1. Test de dibujos.
actividad (Opción 1 incorrecta). 2) Muestreo de eventos: 2. Aritmética.
Sólo se registran determinados tipos de conductas. Por 3. Búsqueda de animales.
ejemplo, si únicamente nos interesan las posturas de equili- 4. Test de semejanzas.
brio en sesiones de actividad física para personas de tercera
edad (Opción 2 incorrecta). 3) Muestreo temporal: Se elige Respuesta correcta:1
la información que se registra en función del tiempo real.
Este a su vez puede dividirse en muestreo instantáneo o de
puntos de tiempo, muestreo de intervalo total y muestreo Las Escalas de inteligencia de Wechsler son escalas ba-
de intervalo parcial. Un ejemplo lo encontramos si en una sadas en metodología racional y arrojan puntuaciones de
sesión de clase se muestrea cada 30 segundos a un alumno CI. En concreto el WPPSI-IV es la versión del WPPSI más
para saber si se mantiene bien sentado (Opción 3 correcta). actual y evalúa niños de preescolar y primaria con edades
Y por último, 4) Muestreo focal: Se utiliza cuando se ob- comprendidas entre 2,6 y 7,7 años. Es más flexible y pre-
servan varios sujetos, observando a cada uno de ellos me- senta una estructura ampliada y conformada por una escala
diante rondas sucesivas. Por ejemplo, en la evaluación de total, escalas primarias y secundarias para los dos posibles
un entrenamiento deportivo, cinco minutos de focalización rangos de edad de aplicación (2,63,11 y 4-7,7 años) de la
con cada uno de los participantes de forma sucesiva (Op- prueba. En concreto el enunciado nos está preguntando por
ción 4 incorrecta). APIR Manual de Evaluación Psicológi- las pruebas diseñadas para el primer rango de edad, en el
ca, Tema 3. que podemos encontrar en la escala de Lenguaje General la
prueba nombres, en la escala CI verbal las pruebas Dibujos
(Opción 1 correcta) e información y en la escala CI Ma-
nipulativo las pruebas cubos y rompecabezas. Respecto al
106. Desde los “sistemas de expertos”, propiciados resto de alternativas de respuesta, Semejanzas y Búsqueda
por el desarrollo de la Inteligencia artificial, se han de animales constituyen pruebas complementarias para el
creado programas destinados a: segundo rango de edad de WPPSI-IV (Opciones 3 y 4 in-
correctas) y Aritmética es una prueba propia del WISC, no

32
del WPPSI. APIR Manual Evaluación Psicológica, Tema 9. de primer grado de los adultos con TOC es aproximada-
mente 2 veces mayor que entre los familiares de primer
grado de los que no tienen el trastorno; sin embargo, en-
108. En la evaluación psicofisiológica de la activi- tre los familiares de primer grado de los individuos con un
dad electrodérmica, los parámetros más frecuentes inicio del TOC en la niñez o en la adolescencia, la tasa se
para medir la conductancia de la piel son la amplitud incrementa 10 veces. Esta transmisión familiar se debe en
de respuesta y: parte a factores genéticos (p. ej., una tasa de concordan-
cia de 0,57 para los gemelos monocigóticos frente a 0,22
1. El periodo de recuperación para los gemelos dicigóticos). La disfunción de la corteza
2. El tiempo de latencia orbitofrontal, de la corteza cingulada anterior y del cuerpo
estriado está fuertemente implicada. La extraversión no se
3. La resistencia
considera un factor de riesgo temperamental relacionado
4. La variabilidad de la respuesta a lo largo de todo con el TOC (opción 1 incorrecta). APIR Manual de Psico-
el proceso de medida. logía Clínica, Tomo I, Tema 7.

110. Respecto de los criterios diagnósticos para el tras-


Respuesta correcta:2 torno de ansiedad generalizada, el DSM-5 establece
como duración mínima de los síntomas nucleares (se-
Definimos la actividad electrodérmica de un organismo ñale la respuesta correcta):
como la facilitación u oposición diferencial al paso de la
corriente eléctrica. Es decir, podemos entenderla como la 1. Dos semanas.
variación en características como la conductancia de la piel,
2. Un mes.
mediadas por la sudoración. La actividad electrodérmica
es un índice de activación general, que posee un carácter 3. Tres meses.
más bien inespecífico, indicando el grado de movilización 4. Seis meses.
del organismo ante un estímulo o ante una situación deter-
minada, siendo el resultado de la actividad simpática del Respuesta correcta:4
sistema nervioso autónomo. Los parámetros más frecuente-
mente estudiados en las respuestas de conductancia son: 1) Los criterios del trastorno de ansiedad generalizada en el
La amplitud de la respuesta, que consiste en la conductan- DSM-5 son los siguientes: A. Ansiedad y preocupación
cia máxima en comparación con el nivel de conductancia excesivas (anticipación aprensiva) sobre una amplia gama
anterior; y 2) El tiempo de latencia, que se define como de acontecimientos o actividades (como el rendimiento la-
el periodo transcurrido desde el inicio de la presentación boral o escolar), que se prolonga más de 6 meses (opción
del estímulo hasta el comienzo de la respuesta (Opción 2 4 correcta; opciones 1, 2 y 3 incorrectas). B. Al individuo
correcta). El resto de alternativas de respuesta, no se co- le resulta difícil controlar este estado de constante preocu-
rresponden con parámetros evaluados de forma especial- pación. C. La ansiedad y preocupación se asocian a tres (o
mente frecuente en relación con la actividad electrodérmica más) de los seis síntomas siguientes (algunos de los cuales
(Opciones 1, 3 y 4 incorrectas). APIR Manual Evaluación han persistido más de 6 meses): 1. Inquietud o impacien-
Psicológica, Tema 6. cia; 2. Fatigabilidad fácil; 3. Dificultad para concentrarse o
tener la mente en blanco; 4. Irritabilidad; 5. Tensión mus-
cular;
109. ¿Qué factor de riesgo temperamental se relaciona 6. Alteraciones del sueño (dificultad para conciliar o man-
con el trastorno obsesivo compulsivo (DSM-5)?: tener el sueño, o sensación al despertarse de sueño no re-
parador). D. El centro de la ansiedad y de la preocupación
no se limita a los síntomas de un trastorno del Eje I; por
1. Extraversión.
ejemplo, la ansiedad o preocupación no hacen referencia
2. Reducida emotividad positiva. a la posibilidad de presentar una crisis de angustia (como
3. Comportamiento perturbador. en el trastorno de angustia), pasarlo mal en público (como
4. Síntomas de interiorización. en la fobia social), contraer una enfermedad (como en el
Respuesta correcta:4 trastorno obsesivo-compulsivo), estar lejos de casa o de los
seres queridos (como en el trastorno de ansiedad por se-
Según el DSM-5, hay diferentes factores de riesgo y pro- paración), engordar (como en la anorexia nerviosa), tener
nóstico para padecer trastorno obsesivo compulsivo (TOC), quejas de múltiples síntomas físicos (como en el trastorno
entre ellos: 1) Temperamentales: los síntomas de interiori- de somatización) o padecer una enfermedad grave (como
zación (opción 4 correcta), la mayor emotividad negativa en la hipocondría), y la ansiedad y la preocupación no apa-
(opción 2 incorrecta) y la inhibición del comportamiento en recen exclusivamente en el transcurso de un trastorno por
la infancia (opción 3 incorrecta); 2) Ambientales: el maltra- estrés postraumático. E. La ansiedad, la preocupación o los
to físico y sexual en la infancia y otros sucesos estresantes síntomas físicos provocan malestar clínicamente significa-
o traumáticos. Algunos niños pueden desarrollar repentina- tivo o deterioro social, laboral o de otras áreas importantes
mente síntomas obsesivo-compulsivos que se han asociado de la actividad del individuo. F. Estas alteraciones no se
con diferentes factores ambientales, incluidos varios agen- deben a los efectos fisiológicos directos de una sustancia
tes infecciosos y un síndrome autoinmune posinfeccioso; (p. ej., drogas, fármacos) o a una enfermedad médica (p.
3) Genéticos y fisiológicos: la tasa de TOC en los familiares ej., hipertiroidismo) y no aparecen exclusivamente en el

33
transcurso de un trastorno del estado de ánimo, un trastorno piloerección, taquicardia, gestos descoordinados rápidos,
psicótico o un trastorno generalizado del desarrollo. APIR fijación de la mirada en algún punto frontal…). Por lo ge-
Manual de Psicología Clínica, Tomo I, Tema 6. neral, el niño no suele recordar lo ocurrido al día siguiente,
y tampoco lo soñado en el momento del episodio (opción 4
incorrecta) (si acaso alguna imagen o emoción, pero no una
111. Respecto del Trastorno obsesivo compulsivo historia elaborada como en las pesadillas). APIR Manual de
(TOC), cabe afirmar lo siguiente: Psicología Clínica, Tomo II, Tema 17.

1. Los varones presentan una edad más temprana de 113. El diagnóstico de la narcolepsia, aunque
ini cio de los síntomas del trastorno que las esencialmente clínico, idealmente debe ser confirmado
mujeres. en laboratorio de sueño a través de:
2. En general, los estudios muestran que la prevalen-
cia a lo largo de la vida es del 3-4 %. 1. Poligrafía de sueño seguida de tonometría a la ma-
3. La presencia de TOC no es universal. ñana siguiente.
4. El TOC suele comenzar en la infancia. 2. Poligrafía de sueño seguida del Test de Romberg a
la mañana siguiente.
Respuesta correcta:1 3. Polisomnografía nocturna, seguida a la mañana
siguiente del test de latencias múltiples de sueño (TLMS).
4. Polisomnografía nocturna, seguida a la mañana
La prevalencia vital del TOC es de 2,5% (opción 2 inco- siguiente de capnografía.
rrecta) y, además, la presencia del trastorno es universal
(opción 3 incorrecta). En el DSM-IV-TR, y hasta hace re- Respuesta correcta:3
lativamente poco, se consideraba que el TOC afectaba por
igual a hombres y mujeres, sin embargo los últimos estu- Antes de centrarnos en los criterios para la narcolepsia se-
dios y el DSM-5 señalan que las mujeres se ven afectadas gún el DSM-5, es conveniente comentar algunos conceptos
con una tasa ligeramente más alta que los varones en la que aparecen las alternativas de respuesta propuestas. La
edad adulta. Además, es importante señalar que en los va- tonometría (opción 1 incorrecta) es un examen para medir
rones es frecuente que el trastorno se inicie antes (opción la presión dentro de los ojos, utilizado en el diagnóstico
1 correcta). Es por ello que el TOC, durante las primeras del glaucoma. El test de Romberg (opción 2 incorrecta) es
etapas del desarrollo, es más frecuente en varones. Sería una prueba neurológica para valorar la propiocepción cons-
en la adolescencia cuando la prevalencia se igualaría entre ciente, que consiste en pedirle a la persona que se ponga de
hombres o mujeres, o incluso llegaría afectar ligeramente pie con ambos pies juntos con los ojos abiertos, pidiendo
más a estas. La edad de inicio suele ser la adolescencia o seguidamente que los cierre (si en este momento pierde el
edad adulta (opción 4 incorrecta), aunque también puede equilibrio, significaría que existe algún tipo de problema).
aparecer en la infancia, siendo la edad de inicio diferente La capnografía (opción 4 incorrecta) es el estudio de los
según el sexo: entre los 6-15 años en los varones y entre los niveles de dióxido de carbono en el ciclo respiratorio de
20-29 en las mujeres. APIR Manual de Psicología Clínica, la persona. Los criterios diagnósticos de la narcolepsia son
Tomo I, Tema 7. los siguientes: A. Períodos recurrentes de necesidad irrefre-
nable de dormir, de abandonarse al sueño o de echar una
112. Señale la afirmación correcta respecto de los siesta que se producen en un mismo día. Estos episodios se
terrores nocturnos: han de haber producido al menos tres veces por semana du-
rante los últimos tres meses; B. Presencia de al menos una
de las características siguientes: 1. Episodios de cataplejía,
1. Generalmente se producen en el último tercio del
definida por (a) o (b), que se producen como mínimo algu-
periodo principal del sueño.
nas veces al mes: (a). En los individuos con enfermedad de
2. Es una parasomnia del sueño REM. larga duración, episodios breves (segundos o minutos) de
3. Forman parte, junto con el sonambulismo, de una pérdida brusca bilateral del tono muscular con conserva-
mis ma categoría diagnóstica (DSM 5). ción de la consciencia que se desencadenan con la risa o las
4. Suele recordarse el contenido de lo soñado en gran bromas; (b). En los niños o en otros individuos en los seis
me dida. meses posteriores al inicio, episodios espontáneos de mue-
cas o de abrir la boca y sacar la lengua, o hipotonía general
sin un desencadenante emocional evidente; 2. Deficiencia
Respuesta correcta:3 de hipocretina, según el valor de inmunorreactividad de hi-
pocretina-1 en el líquido cefalorraquídeo (LCR) (inferior o
En el DSM-5 aparece el trastorno del despertar del sueño igual a un tercio del valor en individuos sanos analizados
no REM (opción 2 incorrecta) dentro de las parasomnias, con la misma prueba, o inferior o igual a 110 pg/ml). La
que engloba terrores nocturnos y sonambulismo (opción 3 concentración baja de hipocretina-1 en el LCR no se ha de
correcta). Respecto a los terrores nocturnos, suelen produ- observar en el contexto de lesión, inflamación o infección
cirse en el primer tercio de la noche (opción 1 incorrecta), cerebral aguda; 3. Polisomnografía nocturna con latencia
en los que el niño se levanta bruscamente gritando, pu- del sueño REM inferior o igual a 15 minutos, o una prueba
diéndose producir vocalizaciones acompañadas de mani- de latencia múltiple del sueño con un valor medio inferior
festaciones de ansiedad y activación intensa (sudoración, o igual a 8 minutos y dos o más períodos REM al inicio

34
del sueño (opción 3 correcta). APIR Manual de Psicología Tomo II, Tema 17.
Clínica, Tomo II, Tema 17.

115. En el síndrome de la apnea/hipopnea obstructi-


114. En el marco de los trastornos del sueño, res- va del sueño (SAHOS) moderado a grave, con sintoma-
pecto del síndrome de retraso de fase, cabe afirmar tología (somnolencia/diurna) el tratamiento de elección
que: es:

1. Los pacientes presentan un sueño de menor cali- 1. Un fármaco hipnótico.


dad. 2. Un fármaco ansiolítico.
2. Es el problema de ritmo circadiano menos fre- 3. La presión positiva continua en la vía aérea.
cuente. 4. El neurofeedback.
3. Es más frecuente en población general adulta de
edad media que en adolescentes.
4. Existe una forma familiar que representa el 40% Respuesta correcta:3
de los casos.
La presión nasal positiva es un tratamiento ideado por Sulli-
Respuesta correcta:0 van, Berthon-Jones, Issa y Eves. Se trata de una mascarilla
unida a un generador de aire que administra un flujo conti-
ANULADA. En la plantilla de respuestas provisional la nuo de aire a más presión sobre las vías respiratorias supe-
respuesta dada como correcta era la 4, puesto que en el sín- riores, para prevenir la aparición de apneas. Se considera el
drome de retraso de fase suele existir una agregación fami- tratamiento de elección (opción 3 correcta) ya que produce
liar (que representa el 40% de los casos), observándose más mejoría tanto en la sintomatología diurna como nocturna.
mutaciones en el gen Per3 que en los sujetos sanos (factor APIR Manual de Tratamientos Psicológicos, Tema 13.
de riesgo). Por el contrario, variaciones en el gen CK1 ep-
silon ejercen una función protectora (opción 4 correcta). 116. Señale cuál de los siguientes fenómenos es m e -
Además, en un primer momento se consideró la opción 1 nos propio y definitorio del trastorno de identidad d i -
incorrecta, pues según DSM-5 y Caballo, V., Salazar, I. y sociativo (DSM 5):
Carrobles, J. A. (2014) en el Manual de psicopatología y
trastornos psicológicos, los pacientes no presentan un sue- 1. Experiencias de distanciamiento e irrealidad res-
ño de menor calidad en sí mismo, ya que el ciclo sueño-vi- pecto al entorno.
gilia está conservado de manera adecuada, son las deman- 2. Vacíos recurrentes al recordar eventos cotidianos.
das sociales o personales las que provocarían el malestar, si 3. Seria discontinuidad en el sentido de identidad.
no existieran esas demandas y la persona pudiera levantar-
4. Alteraciones en el afecto, conducta, conciencia y
se a la hora deseada no habría disfunción. Sin embargo, la
memoria.
opción 1 también podría ser considerada correcta, teniendo
en cuenta información aportada tanto por DSM-5 como por
el Manual de psicopatología de Belloch, A., Sandín, B. y
Ramos, F. (2008). Según Belloch, la fase de sueño del ritmo Respuesta correcta:1
circadiano aparece más tarde en relación al horario normal
de acostarse, de modo que cuando el sujeto se acuesta no se La opción 1 (experiencias de distanciamiento e irrealidad
duerme, y al levantarse se encuentra con sueño y cansado. respecto al entorno) haría referencia al trastorno de desper-
DSM-5 apunta que son prominentes los síntomas de insom- sonalización/desrealización, que no es propia del trastorno
nio al comienzo del sueño, la dificultad para despertarse de identidad disociativo (opción 1 correcta). Los criterios
por la mañana y la somnolencia excesiva al comienzo del diagnósticos del trastorno de identidad disociativo según el
día. Además, señala que puede desarrollarse un insomnio DSM-5 son los siguientes: A. Perturbación de la identidad
psicofisiológico como consecuencia de las conductas desa- que se caracteriza por dos o más estados de la personali-
daptativas que deterioran el sueño e incrementan la activa- dad bien definidos, que se puede describir en algunas cul-
ción al intentar en repetidas ocasiones quedarse dormidos turas como una experiencia de posesión. La perturbación
a una hora temprana. La información anterior defendería de la identidad implica una discontinuidad importante del
que la calidad del sueño se encontraría deteriorada (opción sentido del yo y del sentido de entidad (opción 3 incorrec-
1 correcta). Por otro lado, DSM-5 aporta información que ta), acompañado de alteraciones relacionadas del afecto,
podría ser incompatible con la opción de respuesta 4: “no el comportamiento, la conciencia, la memoria, la percep-
se ha establecido la prevalencia familiar de tipo de fases ción, el conocimiento y/o el funcionamiento sensitivo-mo-
de sueño retrasada”. El resto de alternativas de respuesta tor (opción 4 incorrecta). Estos signos y síntomas pueden
son incorrectas. Aunque su prevalencia es pequeña (0,17%, ser observados por parte de otras personas o comunicados
pero parece ser mayor del 7% en adolescentes (opción 3 in- por el individuo; B. Lapsos recurrentes en la memoria de
correcta)), no se trata del ritmo circadiano menos frecuente, acontecimientos cotidianos, información personal impor-
pues es de destacar que más del 80% de los diagnósticos de tante, y/o sucesos traumáticos incompatibles con el olvido
trastorno del ritmo circadiano que se realizan en clínica se ordinario (opción 2 incorrecta); C. Los síntomas causan
corresponden precisamente con el tipo de sueño retrasado malestar clínicamente significativo o deterioro en lo social,
(opción 2 incorrecta). APIR Manual de Psicología Clínica, laboral u otras áreas importantes del funcionamiento; D. La

35
alteración no es una parte normal de una práctica cultural Lo que determina que un juego sea patológico es la capa-
o religiosa ampliamente aceptada; E. Los síntomas no se cidad de la persona para controlar voluntariamente su im-
pueden atribuir a los efectos fisiológicos de una sustancia plicación en el juego, pero generalmente la percepción de
(p. ej., laguna mental o comportamiento caótico durante la la persona sobre su propia capacidad puede estar alterada,
intoxicación alcohólica) u otra afección médica (p. ej., epi- sobre todo en los jugadores más patológicos (“ilusión de
lepsia parcial compleja). APIR Manual de Psicología Clíni- control”, que es la creencia de que el azar no influye en el
ca, Tomo II, Tema 11. juego y que uno posee estrategias que le permitirán ganar
[opción 1 correcta]). La negación y la mentira serían más
síntomas/conductas característicos del trastorno (opción 2
incorrecta). Aunque las opciones 3 y 4 también son distor-
117. ¿Cuál de los siguientes síntomas conductuales siones cognitivas, ninguna sería el sesgo principal en este
es más característico de la amnesia disociativa?: trastorno (opciones 3 y 4 incorrectas). APIR Manual de Psi-
cología Clínica, Tomo II, Tema 18.
1. Comportamiento mecánico.
119. ¿Cuáles son los principales efectos de las anfe-
2. Analgesia. taminas?:
3. Autolesiones e intentos de suicidio.
4. Macropsia o micropsia. 1. Elevación del estado de ánimo y disminución de
la sensación de fatiga y del apetito.
2. Después de los iniciales efectos eufóricos en poco
Respuesta correcta:3 tiempo le sigue un claro síndrome amotivacional
junto con supresión del sueño normal durante se-
Sabiendo que la amnesia disociativa, también denominada manas.
amnesia funcional/psicógena/histérica/no orgánica, consti- 3. El consumo produce un sueño excesivo.
tuye una pérdida de memoria retrógrada, episódica, explí- 4. Estado de completo bienestar, euforia, sensación
cita, podemos descartar la analgesia (incapacidad de sentir de flo tar y desaparición de las inhibiciones.
dolor) (opción 2 incorrecta), las macropsias o micropsias
(distorsiones perceptivas en las que objetos se ven de ma-
yor o menor tamaño del real, respectivamente) (opción 4
Respuesta correcta:1
incorrecta), y el comportamiento mecánico, que sería más
propio de algunas alteraciones de la conciencia (opción 1
incorrecta). Acerca del trastorno amnésico, es importante Las anfetaminas son estimulantes del sistema nervioso cen-
saber que es el trastorno disociativo más frecuente, y que tral. Sus efectos son similares a los de la cocaína, pero más
su prevalencia es mayor en mujeres que en hombres. Según duraderos. Producen una elevación del estado de ánimo,
el DSM-5, la duración de este trastorno puede variar desde disminución de la sensación de fatiga y del apetito (opción
minutos hasta décadas. El curso es también variable: remi- 1 correcta; opción 3 incorrecta). La opción 2, menciona el
sión espontánea en la mayoría, o curso crónico. Algunos síndrome amotivacional, que está más relacionado con el
episodios de amnesia disociativa se resuelven rápidamente consumo de cannabis o THC (opción 2 incorrecta). Por úl-
(p.ej., cuando la persona se retira del combate o de alguna timo, la opción 4 (estado completo de bienestar, euforia,
otra situación de estrés), mientras que otros episodios per- sensación de flotar y desaparición de las inhibiciones) hace
sisten durante largos periodos de tiempo. Algunas personas referencia a los efectos que producen los opiáceos (opción
pueden recordar gradualmente los recuerdos disociados 4 incorrecta). APIR Manual de Psicología Clínica, Tomo I,
años más tarde. Las capacidades disociativas pueden dis- Tema 3.
minuir con la edad, pero no siempre. A medida que la am-
nesia remite, puede haber una considerable angustia, con-
ducta suicida (opción 3 correcta) y síntomas de trastorno 120. ¿Cuál de las siguientes drogas produce una
de estrés postraumático, al verse los sujetos invadidos por fuerte dependencia psicológica pero poca dependen-
recuerdos intolerables. Es por ello que las conductas auto- cia física?:
destructivas, entre las que se incluyen las de tipo suicida,
son comunes en estos pacientes. APIR Manual de Psicolo- 1. La cafeína.
gía Clínica, Tomo II, Tema 11. 2. El alcohol.
3. La cocaína. 4. La heroína.
118. ¿Cuál es el principal sesgo cognitivo del juga-
dor patológico?:
Respuesta correcta:3
1. La ilusión de control.
2. La negación y la mentira. La dependencia a una sustancia hace referencia a el patrón
3. Evaluación sesgada de los resultados. de consumo desadaptativo que conlleva un malestar signifi-
4. La idea de que cuanto más se juega hay más posi- cativo a lo largo de un amplio periodo de tiempo (un periodo
bilida des de ganar. de 12 meses según DSM). Existen dos tipos de dependen-
cia: 1) La dependencia física, cuyas manifestaciones son la
Respuesta correcta:1 tolerancia (estado de adaptación caracterizado por la dismi-
nución de la respuesta del organismo a la misma cantidad

36
de droga o por la necesidad de incrementar la dosis para referencia al síndrome amotivacional (descrito por Camí en
conseguir un mismo efecto) y el síndrome de abstinencia 1988) propio del consumo de cannabis (opción 4 incorrec-
(conjunto de síntomas específicos que surgen debido a un ta). APIR Manual de Psicología Clínica, Tomo I, Tema 3.
cese o una disminución del consumo de una sustancia que
se ha producido en grandes cantidades o durante un amplio
periodo de tiempo); y 2) La dependencia psicológica, que
hace referencia al craving (intenso deseo de consumir la 122. La capacidad que tiene una sustancia de producir
sustancia). La cocaína se caracteriza porque produce en sus dependencia en los que la consumen, recibe el nombre
consumidores un intenso deseo por consumir la sustancia de:
(craving/dependencia psicológica) y un menor grado de de-
pendencia física (opción 3 correcta), mientras que el resto 1. Dependencia física.
de sustancias (heroína, cafeína y alcohol) incluidas en las
2. Dependencia psicológica. 3. Au-
opciones 1, 2 y 4, presentan un predominio de dependencia
mento de la tolerancia.
física (opciones 1, 2 y 4 incorrectas). APIR Manual de Psi-
cología Clínica, Tomo I, Tema 3. 4. Potencial adictivo de una droga.

Respuesta correcta:4
121. De las drogas existentes, el consumo regular
de heroína y morfina se caracteriza porque: El potencial adictivo de una droga hace referencia a la
propensión que tiene la sustancia de crear adicción/depen-
dencia (opción 4 correcta). La dependencia a una sustancia
1. Desarrollan rápidamente tolerancia y dependencia hace referencia a el patrón de consumo desadaptativo que
en los consumidores. conlleva un malestar significativo a nivel psicológico (de-
2. Los daños en la salud sólo se producen a largo pendencia psicológica) y a nivel físico (dependencia física)
plazo. a lo largo de un amplio periodo de tiempo (opciones 1 y 2
3. Producen una fuerte dependencia física pero no incorrectas). La tolerancia se refiere al estado de adaptación
psicológica. que se caracteriza por la disminución de la respuesta del
4. Producen un característico estado amotivacional organismo a la misma cantidad de droga o una necesidad de
en sus consumidores. incrementar la dosis para obtener el mismo efecto (opción
3 incorrecta). APIR Manual de Psicología Clínica, Tomo I,
Respuesta correcta:1 Tema 3.

Los sujetos que realizan un consumo regular de heroína y


morfina se caracteriza porque desarrollan rápidamente to-
lerancia y dependencia (opción 1 correcta). Así mismo, el 123. ¿Cuál NO sería una característica de
trastorno por consumo de opiáceos puede comenzar a cual- buen pronóstico en el Trastorno esquizofreniforme?:
quier edad, pero los problemas asociados con el uso de opiá-
ceos se observan con mayor frecuencia en la adolescencia 1. Aparición de síntomas psicóticos notables dentro
tardía o en los primeros años de la veintena. Además, cabe de las primeras cuatro semanas.
tener en cuenta que el aumento de la edad se asocia con
2. Confusión o perplejidad.
una disminución en la prevalencia, como resultado de la
mortalidad inicial (opción 2 incorrecta) y de la remisión de 3. Que el episodio se desencadene con un episodio
los síntomas después de los 40 años. Otros datos curiosos a mania co.
tener en cuenta son: que el curso del trastorno por consumo 4. Ausencia de afecto aplanado o embotado.
de opiáceos tiende a ser abrupto con remisiones y recaídas.
Las tasas de mortalidad a largo plazo pueden ser de hasta Respuesta correcta:3
un 2 % por año y alrededor del 20-30 % de las personas
con este trastorno logra la abstinencia a largo plazo. Por Brevemente, antes de justificar por qué la alternativa de
otro lado, los opiáceos producen dependencia psicológica respuesta correcta es la 3, es importante repasar los concep-
y física. Concretamente, el DSM-5 (Criterio B) exige para tos fundamentales sobre el trastorno esquizofreniforme in-
el diagnóstico de abstinencia a opiáceos tres o más de los cluidos en el DSM5. Concretamente, los síntomas caracte-
siguientes: estado de ánimo disfórico, náuseas o vómitos, rísticos del trastorno esquizofreniforme son idénticos a los
dolores musculares, lagrimeo o rinorrea, dilatación pupilar, de la esquizofrenia (Criterio A), es decir, para diagnosticar
piloerección o aumento de la sudoración, diarrea, bostezos, un trastorno esquizofreniforme es necesaria la presencia de
fiebre o insomnio. Cabe destacar que el DSM-5 señala que 2 o + de los síntomas siguientes: alucinaciones, delirios,
los primeros síntomas de la abstinencia a los opiáceos son discurso desorganizado, comportamiento desorganizado o
subjetivos y consisten en quejas sobre ansiedad, inquietud catatónico y/o síntomas negativos. La diferencia entre la
y una sensación de dolor, junto con irritabilidad y un in- esquizofrenia y el trastorno esquizofreniforme se halla en
cremento de la sensibilidad al dolor. Además también se el criterio B que hace referencia a su duración (entre 1 y
hace mención a que signos como la piloerección y la fie- 6 meses, en el caso del trastorno esquizofreniforme). Otra
bre están asociados a una abstinencia más grave y que no de las características distintivas del trastorno esquizofre-
se ven a menudo en la práctica clínica habitual (opción 3 niforme respecto a la esquizofrenia es que no se requiere
incorrecta). Por último, la alternativa de respuesta 4 hace deterioro en el funcionamiento social y laboral, pudiendo

37
estar presente este deterioro pero sin ser necesariamente un se conoce como “criptolalia” (opción 4 incorrecta). La pa-
criterio obligado para el diagnóstico. El diagnóstico de tras- lilalia consiste en la repetición involuntaria de una palabra
torno esquizofreniforme se realiza en dos situaciones: 1) varias veces seguidas, siendo un trastorno que se engloba
Cuando la duración del episodio de la enfermedad tiene una en el grupo de los trastornos iterativos del lenguaje (opción
duración entre 1 y 6 meses; y 2) Cuando un individuo tie- 2 incorrecta). Vallejo, J. (2015). Introducción a la psicopa-
ne síntomas durante menos de los 6 meses requeridos para tología y psiquiatría. Capítulo 45.
diagnosticar una esquizofrenia pero todavía no se ha recu-
perado. En este caso, el diagnóstico debería ser “trastorno
esquizofreniforme (provisional)”, puesto que no es seguro 125. ¿Cómo se denomina al fenómeno que ocurre
que el individuo vaya a recuperarse del trastorno dentro del si un paciente está convencido de que otra persona
período de 6 meses. Si los síntomas duraran más de 6 me- ha sido transformada físicamente en sí mismo?:
ses se debería cambiar el diagnóstico a esquizofrenia. En
cuanto a los factores de buen pronóstico, tanto el DSM-IV- 1. Síndrome de intermetamorfosis.
TR como el DSM-5 incluyen como características de buen 2. Delirio de los dobles.
pronóstico para el trastorno esquizofreniforme: la aparición 3. Síndrome de Fregolí.
de los síntomas psicóticos en las primeras cuatro semanas
después del cambio apreciable del comportamiento o fun- 4. Síndrome de los dobles subjetivos.
cionamiento habitual (opción 1 incorrecta); confusión o
perplejidad (opción 2 incorrecta); buen funcionamiento so-
cial y laboral antes de la enfermedad; y ausencia de afecto Respuesta correcta:4
embotado y plano (opción 4 incorrecta). Por tanto, el DSM-
5 no recoge que sea un factor de buen pronóstico para el Trastornos delirantes de falsa identidad. Los síndromes de
trastorno esquizofreniforme la presencia de un episodio falsa identificación delirante incluyen una serie de síndro-
maniaco como desencadenante del trastorno (opción 3 co- mes, diferentes pero relacionados, que tienen en común el
rrecta). El resto de alternativas son incorrectas por tratarse denominado concepto del doble. Estos síndromes incluyen
de factores de buen pronóstico incluidos en el DSM. APIR el síndrome de Capgras, el síndrome de Frégoli, el síndro-
Manual de Psicología Clínica, Tomo I, Tema 4. me de intermetamorfosis y el síndrome de los dobles sub-
jetivos. El síndrome de Capgras consiste en la creencia por
parte del sujeto de que una persona, por lo general relacio-
124. ¿Cómo se denomina al fenómeno por el que nada íntimamente con él, ha sido reemplazada por un doble
a una palabra existente se le da un significado diferente exacto. Por ejemplo, un individuo cree que su madre ha
al que en realidad tiene?: sido reemplazada por un impostor. El síndrome de Frégoli
corresponde a la creencia de que una persona desconocida
ha cambiado su apariencia para asemejarse a familiares o
1. Neologismo.
conocidos, con intenciones malignas (opción 3 incorrecta).
2. Palilalia. El síndrome de intermetamorfosis consiste en la creencia
3. Paralogismo. delirante de que son los otros los que experimentan cam-
4. Glosolalia. bios radicales en su identidad física y psicológica, que cul-
minan finalmente en una nueva persona completamente di-
Respuesta correcta:3 ferente, o dicho de otro modo, la creencia de que personas
de su ambiente han intercambiado sus identidades (opción
Los trastornos del lenguaje se pueden dividir en los si- 1 incorrecta). El síndrome de los dobles subjetivos es la
guientes grupos: ausencia del lenguaje, trastornos del rit- creencia delirante en la existencia de duplicados físicos de
mo, trastornos de la iteración, trastornos del significado uno mismo, y la creencia habitual que estas copias poseen
y alteraciones de origen neurológico. Los trastornos de identidades psicológicas diferentes (opción 4 correcta).
la significación afectan al significado del lenguaje, sien- Oyebode, F. (2018). Síntomas mentales. Manual de psico-
do la mayoría de ellos característicos de la esquizofrenia. patología descriptiva. Capítulo 8.
En dicho grupo encontramos: neologismos, paralogismos,
glosomanía, glosolalia, ensalada de palabras y descarrila-
miento. Los neologismos son la creación y uso de palabras 126. ¿Cuál de las siguientes respuestas es correcta
o expresiones sin un significado generalmente aceptado, a con respecto al trastorno facticio?:
las que el paciente les da una significación concreta. El pa-
ciente crea nuevas palabras a partir de la combinación de 1. No hay comportamiento engañoso-fingimiento si
otras, siendo estas palabras incomprensibles para los otros no existe incentivo externo.
(opción 1 incorrecta). Los paralogismos son la confusión
2. La producción de los síntomas no es intencionada.
del concepto que expresa una palabra, de manera que a una
palabra existente se le da un significado diferente al que 3. Lo habitual es que curse en forma de episodios
en realidad tiene, que es la definición que nos pregunta el intermi tentes.
enunciado (opción 3 correcta). La glosomanía es el conjun- 4. En el trastorno facticio aplicado a otro, el diagnós-
to de manifestaciones verbales neoformadas acumuladas tico se aplica tanto al autor como a la víctima.
sin una sintaxis concreta. La glosolalia es el lenguaje perso-
nal del enfermo que utiliza un vocabulario y una sintaxis de Respuesta correcta:3
su propia invención, pero con un sentido propio. También

38
La característica esencial del trastorno facticio es la simu- Algunos individuos describen características disociativas
lación de signos y síntomas médicos o psicológicos, en uno durante o después del atracón. Asimismo, en algunos casos,
mismo o en otros, asociado a un engaño identificado (op- los atracones pueden estar planificados. El tipo de alimen-
ción 1 incorrecta) y en ausencia de una recompensa externa to consumido durante los atracones puede variar entre los
obvia. La producción de síntomas es intencionada (opción distintos individuos y en un mismo individuo. No obstante,
2 incorrecta) para asumir el rol de enfermo, incapacitado en general los individuos tienden a comer alimentos que en
o lesionado. El diagnóstico de trastorno facticio requiere otras circunstancias evitarían (opción 4 incorrecta). APIR
la demostración de que el individuo está cometiendo ac- Manual de Psicología Clínica, Tomo II, Tema 12.
ciones para tergiversar, exagerar, simular, falsificar signos
o síntomas en ausencia de una recompensa externa obvia.
Aunque puede haber una afección médica preexistente, los 128. Ante la sospecha de presencia de depresión en una
afectos de un trastorno facticio muestran el comportamien- persona ¿cuál de los siguientes aspectos relacionados
to engañoso para adoptar el rol de enfermo. Los métodos con el diagnóstico habría que abordar antes?:
para falsificar la enfermedad incluyen la exageración, la fa-
bricación, la simulación y la inducción de síntomas (p. ej., 1. La existencia de una etiología médica u orgánica.
falsificar pruebas de laboratorio, registros médicos, infor- 2. La posible comorbilidad con trastornos de ansie-
mar engañosamente a facultativos, ingerir sustancias,...). dad.
La prevalencia del trastorno facticio es desconocida, pero 3. La especificación de las distorsiones cognitivas
se estima que afecta a un 1% de la población hospitalaria. presen tes en el caso.
El curso suele ser en forma de episodios intermitentes (op-
ción 3 correcta), siendo los episodios únicos y persistentes 4. La realización del análisis funcional para la expli-
menos frecuentes. El inicio del trastorno tiende a ser en la ca ción del origen y mantenimien-
edad adulta y posteriormente a una hospitalización por un to del trastorno.
problema médico o mental. En el caso del trastorno facticio
aplicado a otro, el diagnóstico se aplica al sujeto que induce Respuesta correcta:1
los síntomas y no a la victima y es necesario la demostra-
ción de que causa los síntomas (opción 4 incorrecta). APIR
Manual de Psicología Clínica, Tomo II, Tema 10. Previamente al diagnóstico de cualquier trastorno mental
o patología es necesario realizar un proceso de diagnostico
diferencial. Concretamente, en el caso de una sospecha de
127. ¿Cuál de las siguientes respuestas es correcta depresión en una persona, es relevante, tal y como estable-
con respecto a la bulimia nerviosa?: cen los criterios D y E del DSM-5, descartar que el episodio
depresivo no se explique mejor por un trastorno psicótico
(esquizoafectivo, esquizofreniforme, delirante) y descartar
1. Es más prevalente en mujeres jóvenes que la ano- que no se pueda atribuir a los efectos fisiológicos de una
rexia nerviosa. sustancia o de otra afección médica (opción 1 correcta). Por
2. Los atracones deben ocurrir, al menos, tres veces a otro lado, la presencia de un trastorno de ansiedad no es
la semana, cuatro meses seguidos, para poder diagnosticar excluyente de la posibilidad de diagnosticar un trastorno
el trastorno. depresivo mayor, es una característica altamente asociada,
3. A diferencia de lo que ocurre en la anorexia ner- motivo por el cual se incluye como un posible especificador
viosa, el peso corporal no influye indebidamente en la au- en el DSM-5 y es uno de los trastornos que aparecen fre-
toevaluación de la persona. cuentemente de forma simultanea con la depresión (opción
4. Los atracones suelen ser de alimentos que también 2 incorrecta). Respecto a la alternativa 4, la realización de
consumen, normalmente, entre episodios. un análisis funcional que permita la explicación del origen
y el mantenimiento del trastorno, podría ser una estrategia
Respuesta correcta:1 muy valiosa para la formulación del caso y del tratamiento,
si bien no es una condición previa a abordar ante la sos-
La prevalencia de la anorexia nerviosa en mujeres jóvenes pecha de la presencia de depresión (opción 4 incorrecta).
es de aproximadamente 0,4%, mientras que la prevalencia Finalmente, las distorsiones cognitivas (muchas de ellas
de la bulimia nerviosa entre mujeres jóvenes es del 1-15% descritas por Beck) son frecuentes en la depresión y no ex-
(opción 1 correcta). Las tres características definitorias de cluyen la posibilidad de diagnosticar la misma ni deben de
la bulimia nerviosa son los episodios recurrentes de atra- ser consideradas previamente al diagnóstico de depresión
cones, los comportamientos inapropiados y recurrentes (opción 3 incorrecta). APIR Manual de Psicología Clínica,
para evitar el aumento de peso y la influencia del peso y Tomo I, Tema 5.
la constitución corporal en la autoevaluación. Esta última
característica es común entre bulimia nerviosa y anorexia 129. Según la Teoría de los estilos de respuesta (No-
nerviosa, y se recoge entre los criterios diagnósticos de len-Hoksema), el elemento fundamental para la conso-
ambos trastornos (opción 3 incorrecta). Para diagnosticar lidación de un trastorno depresivo sería:
bulimia nerviosa, según los criterios DSM-5, es necesa-
rio que los atracones y comportamientos compensatorios 1. La vivencia de acontecimientos de pérdida.
inapropiados se produzcan al menos una vez a la semana 2. La limitación en las capacidades de afrontamiento
durante 3 meses (opción 2 incorrecta). En los atracones es de esas pérdidas.
necesario que se produzca la sensación de falta de control.
3. La baja autoestima.

39
4. Las respuestas rumiativas. lleva a la aparición de la depresión.
2. El incremento en la neurotransmisión serotoninér-
gica lleva a la aparición de la depresión.
Respuesta correcta: 4 3. El déficit en la neurotransmisión serotoninérgica
predispone a la aparición de la depresión, presentándose el
La teoría de los estilos de respuesta (Nolen-Hoeksema) estado depresivo si hay además un déficit en la neurotrans-
propuesta en 1991, se centra en los factores que influyen misión catecolaminérgica.
en el curso de la depresión. Esta teoría sugiere que un es- 4. El déficit en la neurotransmisión serotoninérgica
tilo de respuesta con tendencia a la rumiación mantendría predispone a la aparición de la depresión, presentándose el
y agravaría los síntomas del trastorno depresivo (opción 4 estado depresivo si hay además un incremento en la neuro-
correcta), en contraposición con un estilo más activo que transmisión catecolaminérgica.
tiende a la distracción o la solución de problemas. Según
esta autora, el origen del estilo rumiativo estaría en los pri- Respuesta correcta:3
meros años de crianza del sujeto (aprendizaje por modelado
y prácticas de sociabilización). (opciones 1, 2 y 3 incorrec-
tas). APIR Manual de Psicología Clínica, Tomo I, Tema 5. La hipótesis permisiva de la depresión plantea que el dé-
ficit en la neurotransmisión serotoninérgica predispone a
la aparición de la depresión a través de la oscilación de
otros neurotransmisores. Concretamente, promoviendo
130. Según el DSM-5, ¿cuál de las siguientes es una disminución de la neurotransmisión catecolaminérgi-
una característica definitoria de los trastornos depresi- ca en el caso de la depresión (opción 3 correcta; opción
vos melancólicos durante la fase más grave del episodio 2 incorrecta). En contraposición, si el déficit en la neuro-
actual?: transmisión serotoninérgica predispusiera a un incremento
de la neurotransmisión catecolaminérgica, probablemente
1. Reactividad o aumento del estado de ánimo en la sintomatología que observaríamos sería compatible con
respues ta a sucesos positivos reales o potencia un episodio maniaco (opción 4 incorrecta). Finalmente, la
les. opción 1 hace referencia al déficit serotoninérgico hallado
2. Pérdida de placer por todas o casi todas las acti en pacientes con trastornos depresivos por el cual se sugie-
vidades. re que niveles bajos de serotonina desempeñan un papel
relevante en la etiología del trastorno. El dato que ofrece
3. Aparición y remisión de episodios depresivos en
está alternativa, aunque es correcto, no se corresponde con
determinadas épocas del año. el aspecto preguntado en el enunciado (hipótesis permisi-
4. Sensación de nervios o tensión. va de la depresión) (opción 1 incorrecta). APIR Manual de
Psicología Clínica, Tomo I, Tema 5.
Respuesta correcta:2
132. Según el DSM-5, el trastorno de estrés pos-
El DSM-5 incluye en el especificador de “con caracterís- traumático se especifica cómo “con expresión retarda-
ticas melancólicas” aplicable a los trastornos depresivos. da” cuando:
Incluye dos criterios: A. Presencia de una de las siguientes
características durante el período más grave: 1) Pérdida de
1. No va precedido de un trastorno de estrés agudo.
placer por todas o casi todas las actividades (opción 2 co-
rrecta). 2) Falta de reactividad a estímulos generalmente 2. El inicio de los primeros síntomas se produce al
placenteros (opción 1 incorrecta); B. Tres o más de las si- menos un mes después del acontecimiento trau-
guientes características: 1) Estado de ánimo deprimido con mático.
desesperanza y desaliento profundo. 2) Empeoramiento 3. El inicio de los primeros síntomas se produce al
de la depresión por la mañana. 3) Despertar temprano. 4) menos seis meses después del acontecimiento
Agitación o retraso psicomotor. 5) Anorexia o pérdida de traumático.
peso importante. 6) Culpa excesiva o inapropiada. La al- 4. La totalidad de los criterios diagnósticos no se
ternativa de respuesta 3, aparición y remisión de episodios cumplen hasta al menos seis meses después del
depresivos en determinadas épocas del año, hace referencia acontecimiento traumático.
a un síntoma incluido en el especificador “con patrón esta-
cional” (opción 3 incorrecta). Por último, la alternativa de Respuesta correcta:4
respuesta 4, sensación de nervios o tensión, hace referencia
a un síntoma del especificador “con características de an- En esta pregunta se debe responder literalmente la defini-
siedad” (opción 4 incorrecta). APIR Manual de Psicología ción del especificador de trastorno de estrés postraumático
Clínica, Tomo I, Tema 5. (TEPT) “con expresión retardada” (incluida en el DSM-5).
Concretamente, este especificador señala que únicamente
se puede diagnosticar el trastorno de estrés postraumáti-
131. Según la hipótesis permisiva de la depresión: co con expresión retardada si “la totalidad de los criterios
diagnósticos no se cumplen hasta al menos seis meses des-
1. El déficit en la neurotransmisión serotoninérgica pués del acontecimiento aunque el inicio y la expresión de
algunos síntomas puedan ser inmediatos” (opción 4 co-

40
rrecta; opción 3 incorrecta). En la alternativa de respuesta ingresados.
2, podemos observar como el examinador está intentando 4. A un 20-30% de la población clínica con trastor-
confundir el criterio temporal de expresión retardada (6 nos de personalidad.
meses) con el criterio temporal general de duración míni-
ma necesaria para diagnosticar un trastorno de estrés pos- Respuesta correcta:0
traumático, es decir, el criterio F del DSM-5 que refiere
que: “Las alteraciones B, C, D y E (síntomas de intrusión, ANULADA. La respuesta correcta en la plantilla provi-
evitación y las alteraciones en el nivel de alerta y las altera- sional era la 1. El Manual de psicopatología y trastornos
ciones cognitivas) tienen una duración superior a un mes” psicológicos, de Caballo, V., Salazar, I. y Carrobles, J. A.
(opción 2 incorrecta). Finalmente, respecto a la alternativa (2014), de donde se extrae esta pregunta, cita que la pre-
de respuesta 1, podemos decir que ésta sería correcta, pero valencia del trastorno límite de la personalidad (TLP) en la
teniendo en cuenta el resto de alternativas, y concretamen- población general es de un 2%. Sin embargo, esta pregunta
te la alternativa 4, es más correcto señalar como alternati- fue anulada porque DSM-5 (fuente a la que se hace refe-
va correcta la definición literal ofrecida por el DSM-5 de rencia en el enunciado de la pregunta) aporta información
TEPT con expresión retardada que se incluye en la alterna- diferente, indicando que la prevalencia estimada del TLP
tiva de respuesta 4 (opción 4 incorrecta). APIR Manual de es de un 1,6%, pudiendo llegar a afectar incluso a un 5,9%
Psicología Clínica, Tomo I, Tema 8. de la población general (opción 1 incorrecta). El resto de
alternativas de respuesta son incorrectas. En los centros
133. Respecto a las imágenes alucinoides, de Salud mental la prevalencia es más elevada, llegando a
cabe afirmar que: afectar a un 10% de los usuarios (opción 2 incorrecta) y un
20% de los pacientes psiquiátricos hospitalizados (opción
1. Se producen en ausencia de estímulos concretos 3 incorrecta). Respecto a la alternativa de respuesta 4, la
que la activen. prevalencia del TLP, entre la población clínica con trastor-
2. El individuo les otorga juicio de realidad. nos de personalidad, se encuentra entre el 30-60% (opción
3. Se trata de un concepto sinónimo al de imagen ei- 4 incorrecta). Otros datos relevantes que se incluyen en
dética. el manual de Caballo y que caben destacar a nivel epide-
4. Son un tipo de imagen mnémica. miológico sobre el TLP son los siguientes: 1) Es más fácil
encontrar TLP entre personas que buscan ayuda por consu-
mo de sustancias psicoactivas, trastornos de alimentación
y trastornos depresivos. 2) El TLP se diagnostica más en
Respuesta correcta:1
mujeres que en hombres (aproximadamente un 75% de los
casos son mujeres. 3) El patrón familiar que se observa en
Las imágenes alucinoides son un tipo de pseudopercepcio-
el TLP es cinco veces más frecuente en familiares de pri-
nes o imágenes anómalas que se engloban dentro de los
mer rango de quienes ya tienen un TLP. 4) Aproximada-
engaños perceptivos. Se producen en ausencia de estímu-
mente del 8 al 10 % de los sujetos con un TLP consuman un
los desencadenantes (opción 1 correcta) y aparecen en el
suicidio. Son habituales también las automutilaciones. 5)
espacio negro de los ojos cerrados (imágenes de fiebre) o
El deterioro causado por el trastorno y el riesgo de suicidio
en el espacio físico externo, a causa de intoxicaciones o
son mayores en los primeros años de la edad adulta y van
uso de drogas, en cuyo caso se llaman fantasiopsias. Se ca-
desapareciendo gradualmente con la edad, siendo favora-
racterizan por ser subjetivas y autónomas, debido a que el
ble el pronóstico a largo plazo, ya que durante la cuarta o
sujeto no tiene control sobre ellas. Además presentan una
quina década de vida de los supervivientes en general hay
gran plasticidad y el sujeto no les otorga valor de realidad
una mayor estabilidad tanto en las relaciones como en la
(opción 2 incorrecta). Otro tipo de pseudopercepciones son
actividad profesional. APIR Manual de Psicología Clínica,
las imágenes mnémicas que se caracterizan por ser imáge-
Tomo II, Tema 19.
nes de los recuerdos de los sujetos que pueden presentarse
de modo transformado, se experimentan con poca nitidez y
se desvanecen si no se mantienen voluntariamente (opción
4 incorrecta). Una variante de las imágenes mnémicas son 135. La Teoría del procesamiento de la información
las imágenes eidéticas o recordar sensorial (opción 3 in- aplicada al trastorno de estrés postraumático se centra
correcta), que son representaciones exactas de impresiones en la explicación de:
sensoriales vividas en algún momento por el sujeto, donde
el estímulo externo que produjo la percepción inicial ya no 1. La reexperimentación del acontecimiento traumá-
se halla presente. APIR Manual de Psicopatología, Tema 5. tico.
2. Las conductas de evitación de los estímulos que
recuer dan al trauma.
3. Los problemas afectivos, la pérdida de interés y la
134. Según el DSM 5 el trastorno límite de perso- apa tía.
nalidad afecta: 4. La hiperactivación y la ansiedad extrema.

1. A un 2% de la población general. Respuesta correcta:1


2. A un 5% de individuos que acuden a centros am-
bulato rios de salud mental.
3. A un 10% de pacientes de pacientes psiquiátricos La Teoría del procesamiento de la información propuesta

41
inicialmente por Foa y Kozak (1986), intenta ofrecer una suicidio en el trastorno bipolar tipo II y en el tipo I parecen
explicación sobre las estructuras cognitivas del miedo y similares, pero la letalidad es mayor en los pacientes tipo
los síntomas de reexperimentación presentes en el trastor- II. Podría existir asociación entre marcadores genéticos y el
no de estrés postraumático (TEPT). Concretamente, desde aumento del riesgo de suicidio en trastorno bipolar. Manual
esta teoría se plantea que el cuadro clínico surge a partir de Psicología Clínica, Tomo I, Tema 5.
de sucesos aversivos (impredecibles e incontrolables), que
no han sido procesados emocionalmente de forma adecua-
da, generando una red de miedo, y que interfieren en la 137. Según DSM 5, una persona con un familiar
integración cognitiva y emocional de otras experiencias y de primer grado con una fobia específica a los animales
conductas. La exposición a otros estímulos que recuerdan es significativamente más propensa a tener:
el trauma, puede activar la red de miedo y a través de la
propagación de la activación generar síntomas de reexpe- 1. La misma fobia específica.
rimentación del acontecimiento traumático (opción 1 co- 2. Cualquier otra fobia específica.
rrecta). El individuo, en este contexto, realiza una serie de 3. Cualquier otra fobia.
estrategias de afrontamiento de evitación y escape que a
la larga contribuyen al mantenimiento de la red de miedo. 4. Cualquier otro trastorno de ansiedad.
Las conductas de evitación de los estímulos que recuerdan
al trauma se explican mejor desde las teorías del aprendi-
zaje (generalización, condicionamiento de orden superior) Respuesta correcta:1
(opción 2 incorrecta). Del mismo modo, la hiperactivación
y la ansiedad extrema, también podrían explicarse desde el Según DSM-5, podría haber una susceptibilidad genética
condicionamiento clásico (teorías del aprendizaje) (opción a una determinada categoría de fobia específica (siendo,
4 incorrecta). Por último, los problemas afectivos, pérdida por tanto, una persona con un familiar de primer grado con
de interés y apatía, podrían ser explicados desde modelos una fobia específica significativamente más propensa a te-
basados en la teoría de la indefensión aprendida (opción 2 ner la misma fobia específica) (opción 1 correcta; opción 2
incorrecta). APIR Manual de Psicología Clínica, Tomo I, incorrecta). Es cierto que la fobia específica presenta una
Tema 8. comorbilidad elevada con otros trastornos de ansiedad, in-
cluidas las fobias. Sin embargo, a nivel de riesgo genético
en familiares de primer grado, destaca especialmente la
136. Señale la afirmación correcta con respecto a susceptibilidad a un determinado tipo de fobia (opciones
las tasas de suicidio en relación con los trastornos del 3 y 4 incorrectas). Manual de Psicología Clínica, Tomo I,
estado de ánimo: Tema 6.

1. Son mayores en depresión mayor que en trastorno


bi polar. 138. ¿Cuál es la duración mínima necesaria
para poder realizar un diagnóstico de mutismo selectivo
2. Son similares en depresión mayor y en trastorno
según el DSM 5?:
bipo lar.
3. Son mayores en trastorno bipolar que en depre-
sión ma yor. 1. Un mes.
4. Los datos provienen de estudios que no permiten 2. Tres meses.
afir mar nada al respecto. 3. Seis meses.
4. Doce meses.
Respuesta correcta:3
Respuesta correcta:1
Según refieren Belloch, A., Sandín, B. y Ramos, F. (2008)
en su Manual de Psicopatología, el riesgo de suicidio es El mutismo selectivo consiste en la incapacidad persistente
mayor en sujetos con trastornos bipolares que en sujetos por hablar en situaciones sociales específicas (en las que se
con trastornos depresivos (respuesta 3 correcta; respuestas espera que hable, p. ej., en la escuela) a pesar de hacerlo en
1 y 2 incorrectas). Estos datos están extraídos de estudios otras situaciones o situaciones generales ante personas aje-
epidemiológicos de los trastornos afectivos (opción 4 inco- nas al ámbito íntimo. La alteración interfiere el rendimiento
rrecta). Según DSM-5, se estima que el riesgo de suicidio a escolar o laboral o la comunicación social y la duración
lo largo de la vida de los pacientes con trastorno bipolar es es de por lo menos 1 mes (no limitada al primer mes de
15 veces superior al de la población general (siendo, según escuela) (opción 1 correcta, opciones 2, 3 y 4 incorrectas).
Belloch (2008), cuatro veces mayor que el de los pacien- La incapacidad para hablar no se debe a una falta de co-
tes con trastorno depresivo mayor recurrente). De hecho, el nocimiento o de fluidez del lenguaje hablado requerido en
trastorno bipolar supone 1/4 de todos los suicidios consu- la situación social. El trastorno no se explica mejor por la
mados. La historia previa de intentos de suicidio y el por- presencia de un trastorno de la comunicación (p. ej., tarta-
centaje de días con depresión en el último año se asocian a mudeo) y no aparece exclusivamente en el transcurso de un
un mayor riesgo de intentos suicidas o suicidios consuma- trastorno generalizado del desarrollo, esquizofrenia u otro
dos. El riesgo de suicidio es mayor en el trastorno bipolar II trastorno psicótico. Es un trastorno muy poco frecuente.
y alrededor un 1/3 de ellos refiere antecedentes de intento Su prevalencia se sitúa por debajo del 1%. Parece ligera-
de suicidio. Las tasas de prevalencia vital del intento de mente mayor en niñas. Su inicio suele ser anterior a los

42
cinco años, sin embargo puede que no se haga patente hasta 3. Confabulaciones.
que comience la escuela. Suele durar varios meses y lue- 4. Paramnesias reduplicativas.
go remitir de manera espontánea, aunque en algunos casos
puede persistir durante años. APIR Manual de Psicología Respuesta correcta:3
Clínica Infantil, Tema 11.
Todas las opciones de respuesta hacen referencia a distor-
siones de la memoria. La pseudología fantástica y las con-
139. Señale cuál de las siguientes afirmaciones s o - fabulaciones se clasificarían como anomalías del recuerdo.
bre el trastorno de ansiedad por separación es FALSA: En ambas condiciones se hace referencia al mismo fenóme-
no: la fabricación de recuerdos para rellenar lagunas mnési-
1. Se desarrolla con frecuencia después de un estrés cas. El término confabulación se reserva para aquellas fal-
vital (p. ej., la muerte de una mascota fami- sificaciones que se dan en el síndrome amnésico con estado
liar). de conciencia lúcida, en las que el paciente puede inven-
2. Puede ser hereditario. tarse recuerdos, sin intención de mentir, al intentar dar res-
3. Los niños manifiestan una mayor reticencia a asis- puesta a cosas que no recuerda y tener así una continuidad
tir a la escuela que las niñas. mnémica (o narrar recuerdos auténticos pero mal contex-
4. La expresión indirecta del miedo a la separación tualizados). Suele aparecer en el Síndrome de Korsakoff y
pue de ser más común en los varones. en algunas demencias (respuesta 3 correcta). El término de
pseudología fantástica se aplica en aquellos pacientes que
presentan una sintomatología histérica, o al menos no orgá-
Respuesta correcta:3
nica. En este caso, la alteración se refiere a hechos o narra-
ciones completamente inventadas o fantaseadas (que llegan
La ansiedad de separación es un fenómeno primario que
a creerse ellos mismos) a causa de una necesidad afectiva.
sirve para proteger al niño durante los primeros años. Se
Suelen ser experimentadas en su mayor parte por pacientes
trata de un miedo básico, es de los primeros que aparece,
histéricos, con características histriónicas, siendo también
a partir de él pueden ir apareciendo otros problemas más
muy frecuentes en el Síndrome de Münchausen (opción 1
específicos. Se trata de una ansiedad excesiva ante la sepa-
incorrecta). Por otro lado, los falsos reconocimientos y las
ración de individuos con los que el niño está vinculado. Las
paramnesias reduplicativas se clasificarían como anomalías
manifestaciones de este trastorno varían desde los momen-
del reconocimiento. Los falsos reconocimientos podrían
tos previos a la separación en los que se dan manifestacio-
ser positivos o negativos. En el déjà vu o falso reconoci-
nes de ansiedad y temor, a las que se dan una vez se ha pro-
miento positivo, se daría una sensación de familiaridad ina-
ducido la separación, aquí es más frecuente la tristeza. La
propiadamente intensa (es decir, algo nuevo se experimenta
sintomatología de este trastorno sufre ciertas variaciones en
como si ya se hubiera vivido), y en el jamais vu o falso
función de la edad. Los niños más pequeños (5-8 años) sue-
reconocimiento negativo, tal sensación estaría ausente o
len mostrar preocupación porque les ocurra algo malo a sus
atenuada, lo cual llevaría a una ausencia de reconocimiento
figuras de referencia y resistencia a ir al colegio. En niños
(es decir, se reconoce y recuerda una situación, pero no se
más mayores (9-12) predomina el distrés (malestar general
experimentaría sensación de familiaridad) (opción 2 inco-
subjetivo) y en los adolescentes (13-16 años) los síntomas
rrecta). Por último, las paramnesias reduplicativas serían
más prevalentes son las quejas físicas durante los días de
el otro extremo del déjà vu en continuo de experiencias de
colegio. Afecta más a niñas que a niños en una proporción
familiaridad inapropiada, que consistiría en afirmar que se
2:1. Las niñas manifiestan una mayor reticencia o evitación
estuvo en el mismo lugar anteriormente, siendo la primera
a asistir a la escuela que los niños (opción 3 correcta). La
vez que se está. Es decir, un paciente puede afirmar que
expresión indirecta del miedo a la separación puede ser más
estuvo en un hospital exactamente igual al que está ahora,
común en los varones que en las mujeres, por ejemplo, a
o que ya conocía a los enfermeros (un falso reconocimien-
través de una limitación en el funcionamiento independien-
to muy frecuente en el Síndrome de Korsakoff, demencias
te, rechazo a estar fuera de casa solo, rechazo a estar fuera
y estados confusionales) (opción 4 incorrecta). Manual de
de casa solo, angustia cuando el cónyuge o los hijos hacen
Psicología Clínica, Tomo I, Tema 1.
cosas de forma independiente, o cuando el contacto con el
cónyuge o los hijos no es posible (opción 4 incorrecta). Su
inicio temprano se identifica antes de los 6 años, aunque
la edad típica de aparición es a los 9 años. Normalmente 141. Un paciente que tiene una vívida sensación de que
aparece entre 6 y los 11 años, con frecuencia después de un sus venas se están enrollando sobre sí mismas está
estrés vital, en especial una pérdida (opción 1 incorrecta). experimentando una:
Puede ser hereditario (opción 2 incorrecta). APIR Manual
de Psicología Clínica Infantil, Tema 11. 1. Alucinación cenestésica.
2. Parestesia.
140. Las falsificaciones del recuerdo que aparecen 3. Alucinación vascular.
en un estado de lucidez de conciencia asociadas a una 4. Alucinación táctil.
amnesia orgánica y sin que el paciente tenga la inten-
ción de mentir se denomina: Respuesta correcta:1

1. Pseudologías o seudologías fantásticas. Las alucinaciones, según la modalidad sensorial, pue-


2. Falsos reconocimientos. den clasificarse en: 1) Auditivas, 2) Visuales, 3) Táctiles

43
o hápticas, 4) Olfativas, 5) Gustativas, 6) Somáticas, ce- El paciente crea nuevas palabras a partir de la combinación
nestésticas o viscerales, 7) Cinestésicas o de movimiento de otras, siendo estas palabras incomprensibles para los
y 8) Mixtas. El enunciado describe un tipo de alucinación otros, como por ejemplo: “mamérica” y “opecu” (opción
cenestésica, ya que se remite a sensaciones corporales que 3 correcta). La aproximación de palabras se define como
proceden del interior del propio cuerpo o que afectan a sus el uso de palabras de forma no convencional o creación
órganos internos o externos. Si un paciente afirma que las de pseudopalabras siguiendo las reglas de formación de las
venas se le salen, se le enrollan y se le hacen una burbuja, palabras del idioma, como por ejemplo, “formaricar” por
estaríamos ante un tipo de alucinación cenestésica (opción “acto de salir del mar” y también “fornicar homosexual-
1 correcta). Podría confundirnos con la alucinación táctil o mente”. Tanto los neologismos como la aproximación de
háptica, la cual puede manifestarse en cualquier parte del palabras pueden evaluarse mediante la Escala de Trastor-
cuerpo. Pero una alucinación táctil sería más del tipo sen- nos del Pensamiento, el Lenguaje y la Comunicación, ela-
tirse tocado, pellizcado, manoseado, sentir calambres, etc. borada por Andreasen para evaluar el discurso individual a
(opción 4 incorrecta). Dentro de las alucinaciones táctiles o través de la entrevista clínica. Otro concepto a parte es la
hápticas podemos encontrar varias modalidades: térmicas, alexia, que hace referencia a la pérdida de capacidad para
hídricas, parestesias o sensación de hormigueo (opción 2 leer debido a una lesión cerebral, cuando ya fue adquirida
incorrecta), formicación o delirios dermatozoicos. El con- previamente (opción 4 incorrecta). APIR Manual de Psico-
cepto de alucinación vascular no existe (opción 3 incorrec- patología, Tema 7.
ta). APIR Manual de Psicopatología, Tema 5.

144. ¿Cómo se denomina la dificultad de la articu-


142. ¿Cómo se denominan las imágenes anómalas lación y expresión de palabras producida por lesiones
que se pueden percibir en los momentos de transición en el sistema nervioso?:
de la vigila al sueño:
1. Afasia.
1. Imágenes hipnopómpicas. 2. Dislalia.
2. Imágenes mnémicas. 3. Disglosia.
3. Imágenes hipnagógicas. 4. Disartria.
4. Imágenes parásitas.
Respuesta correcta:4

Respuesta correcta:3 La afasia se caracteriza por ser un trastorno del lenguaje


que se produce como consecuencia de una lesión cerebral,
Dentro las pseudopercepciones o imágenes anómalas en- siendo ésta de carácter focal y no difuso. Por tanto, es un
contramos: 1) Alucinaciones fisiológicas, 2) Imágenes trastorno adquirido en personas adultas que anteriormente
alucinoides, 3) Imágenes mnémicas y 4) Imágenes conse- dominaban el lenguaje (opción 1 incorrecta). La dislalia es
cutivas o post-imágenes y 5) Imágenes parásitas. Las alu- la dificultad para articular fonemas, sílabas o palabras que
cinaciones fisiológicas son imágenes que se dan cuando el puede clasificarse en orgánica y funcional, en función de
individuo está en estado de semiconsciencia, entre la vigilia la etiología del trastorno. La dislalia orgánica es produc-
y el sueño. Existen dos subtipos: las hipnagógicas que se to de malformaciones orgánicas, de déficits sensoriales o
producen en fase de adormecimiento (opción 3 correcta), y motores o alteraciones neurológicas. En dicho grupo en-
las hipnopómpicas que se producen en el paso del sueño a contramos por un lado las disglosias, que son alteraciones
la vigilia (opción 1 incorrecta). Las imágenes mnémicas y o defectos en la articulación de los fonemas producido por
las parásitas no se relacionan con el ciclo sueño vigilia, por anomalías congénitas o adquiridas, en los órganos del habla
lo que ambas respuestas serían incorrectas. APIR Manual (opción 3 incorrecta); y por otro lado, las disartrias, que son
de Psicopatología, Tema 5. alteraciones o defectos de la articulación debido a lesiones
en el control muscular, que pueden deberse a una lesión
143. ¿Cómo se denomina la utilización o en el sistema nervioso central, periférico o en ambos (op-
creación de palabras raras cuyo significado sólo conoce ción 4 correcta). En la dislalia funcional no se conoce la
el individuo?: etiología responsable de la alteración en el manejo de los
fonemas. Dentro de dicho grupo encontramos la dislalia,
1. Asintaxia. que es una alteración o defecto en la articulación producido
por un modo incorrecto de articulación, sin lesión orgáni-
2. Aproximación de palabras.
ca (opción 2 incorrecta). APIR Manual de Psicopatología,
3. Neologismos. Tema 8. APIR Manual de Psicopatología y Clínica Infantil,
4. Alexia. Tema 4.
Respuesta correcta:3
145. La afirmación “la presentadora de televisión
El enunciado se corresponde a la definición del concepto de me quiere avisar de que corro un grave peligro y
neologismos. Los neologismos son la creación y uso de pa- por eso se ha puesto un vestido verde” es un ejemplo
labras o expresiones sin un significado generalmente acep- de:
tado, a las que el paciente les da una significación concreta.

44
1. Temple o humor delirante. perceptivas: 1) hiper o hipoestesias (afectan a la percepción
2. Ocurrencia delirante. de la intensidad del estímulo), 2) metamorfopsias (afectan
3. Percepción delirante. a la percepción del tamaño y la forma del estímulo), 3)
4. Representación delirante. anomalías en la percepción de la cualidad del estímulo, 4)
anomalías en la integración perceptiva y 5) ilusiones. De
Respuesta correcta:3 entre las opciones de respuestas, la única que se considera
una distorsión perceptiva es la dismorfopsia, que es un sub-
Jaspers clasificó los delirios en dos tipos desde el punto de tipo de metamorfopsia, consistiendo en la distorsión en la
vista de la forma, distinguiendo los delirios primarios de los percepción visual de la forma (opción 1 correcta). El resto
secundarios. Y a su vez, los delirios primarios los división de opciones se consideran engaños perceptivos (opción 2, 3
en cuatro grupos: atmósfera delirante, intuición delirante, y 4 incorrectas). APIR Manual de Psicopatología, Tema 5.
percepción delirante y recuerdo delirante. La atmósfera
delirante, también denominado temple o humor delirante,
consiste en la experiencia subjetiva de que el mundo ha 147. Cuando una experiencia p e r c e p t i v a s e
cambiado de un modo sutil pero siniestro, inquietante y mantiene y/o se activa a pesar de que el estímulo
difícil o imposible de definir. Se suele acompañar de un inicial que la produjo ya no se encuentra accesible a
estado de humor delirante, ya que el paciente se siente in- los órganos sensoriales, es muy probable que se e x -
cómodo desasosegado o perplejo (opción 1 incorrecta). La perimente:
intuición delirante, también denominada ocurrencia o ins-
piración delirante, se refiere a que el paciente descubre, de 1. Una pareidolia.
repente, a modo de ocurrencia, el significado de las intui- 2. Una alucinación refleja.
ciones o significaciones mal definidas, sin apoyarse en una 3. Una imagen parásita.
percepción o hecho concreto, a diferencia de la percepción 4. Una pseudoalucinación visual.
delirante. El contenido de estas ideas suele ser autorrefe-
rencial y de gran importancia para el paciente. Por ejem- Respuesta correcta:3
plo, un paciente cae en la cuenta de que las siglas de su
nombre, Emilio Elosúa Albéniz de Darco, significan “Eres El enunciado de la pregunta está haciendo alusión a las
El Asesino de Dios” (opción 2 incorrecta). La percepción pseudopercepciones o imágenes anómalas, que son un tipo
delirante consiste en la interpretación delirante de una per- de engaños perceptivos. Las pseudopercepciones son ano-
cepción normal. El paciente se da cuenta de que sabe algo y malías mentales que pueden concebirse como imágenes
comienza a estructurar aquello que antes sólo intuía, y que que, o bien se producen en ausencia de estímulos desen-
ahora empieza a cobrar significado. Por ejemplo, descubre cadenantes concretos (forman parte de este grupo las imá-
que el color rojo de la comida es veneno. La afirmación del genes hipnagógicas, hipnopómpicas y alucinoides), o bien
enunciado sería un ejemplo de percepción delirante, puesto se mantienen a pesar de que el estímulo que los produjo ya
que hay una percepción normal (veo que la presentadora no esté presente (este grupo incluye las imágenes mnémi-
lleva un vestido verde) que se interpreta de manera deliran- cas, las parásitas y las consecutivas). De entre las opciones
te (esto significa que me quiere avisar de que corro un grave de respuesta, la correcta es la imagen parásita (opción 3
peligro), por lo que la opción 3 sería la correcta. Y por úl- correcta), que se produce como consecuencia de un estímu-
timo, el recuerdo delirante, también denominado represen- lo concreto que ya no se halla presente cuando se produce
tación delirante, consiste en la reconstrucción delirante de la imagen. Estas imágenes se denominan parásitas porque
un recuerdo real o recordar algo claramente delirante, por aparecen cuando el individuo no fija su atención en ellas y
ejemplo, recordar que es el hijo de Dios (opción 4 incorrec- desaparecen cuando se concentra en la experiencia. Suelen
ta). APIR Manual de Psicopatología, Tema 7. aparecer en estados de cansancio o fatiga extremos. El resto
de opciones son incorrectas. La pareidolia es una ilusión
que se caracteriza por dar significado a un estímulo con
146. ¿Cuál de las siguientes alteraciones se conside- poca estructuración o ambiguo. Por ejemplo: formas a las
ra una distorsión perceptiva?: nubes (opción 1 incorrecta). En la alucinación refleja, un
estímulo provoca una alucinación en una modalidad sen-
1. Dismorfopsia. sorial diferente. Por ejemplo: sentir dolor cuando alguien
2. Imagen hipnagógica. estornuda (opción 2 incorrecta). Una pseudoalucinación
3. Pseudoalucinación. es una experiencia sensoperceptiva que el sujeto identifi-
4. Imagen autoscópica. ca como subjetiva y perteneciente al mundo interno, con-
Respuesta correcta:1 servándose el juicio de realidad, pudiendo ser auditivas y
visuales (opción 4 incorrecta). APIR Manual de Psicopa-
tología, Tema 5.
Una distorsión perceptiva o sensorial es la percepción dis-
torsionada de un estímulo existente, mientras que un enga-
ño perceptivo es una experiencia perceptiva anómala que 148. ¿Cómo se llama la alteración del pensamiento
no se fundamenta sobre estímulos realmente existentes en el que el paciente para de hablar a mitad de la f r a -
se y cambia de tema en respuesta a estímulos cercanos
fuera del sujeto o que persiste cuando el estímulo que la
(por ejemplo, la corbata del entrevistador)?:
originó ha desaparecido. Las distorsiones perceptivas se
clasifican en función de la cualidad perceptiva que se en-
cuentra afectada, encontrando las siguientes distorsiones 1. Presión del habla.

45
2. Tangencialidad. estupor, que consiste en una actitud de permanecer ajeno o
3. Circunstancialidad. distante del entorno, 5) mutismo, es decir, respuesta verbal
4. Habla distraída. ausente o escasa, 6) estereotipias, que consisten en repe-
tición continuada e innecesaria de movimientos concretos,
Respuesta correcta:4 7) ecosíntomas, que consiste en la repetición automática de
los movimientos (ecopraxia), las palabras (ecolalia) o los
La alteración del pensamiento que plantean en el enuncia- gestos (ecomimia) producidos por el interlocutor. Los tres
do es la definición de habla distraída, consistente en que primeros síntomas enumerados anteriormente se presentan
el paciente se para en medio de una frase o idea y cambia cuando el paciente es inducido a realizar ciertas acciones
el tema en respuesta a estímulos inmediatos. Por ejemplo: o movimientos (opción 1 correcta). En casos extremos, el
“Entonces, dejé San Francisco y me mudé a... ¿Dónde com- mismo individuo puede oscilar entre una actividad motora
pró esa corbata? Parece como sacada de los años 50. Me disminuida y una actividad motora excesiva. Los pacientes
gusta el clima cálido de San Diego. ¿Es una concha lo que catatónicos a menudo salen súbitamente del estado estu-
está sobre su escritorio? ¿Ha ido de buceo alguna vez?” poroso (inmovilidad motora) para entrar en un estado de
(opción 4 correcta). El resto de opciones son alteraciones agitación (raptus), durante el cual se puede producir lo que
del pensamiento que también pueden evaluarse mediante se llama una tempestad de movimientos, como correr, huir,
la Escala de Trastornos del Pensamiento, el Lenguaje y la gritar, golpear o atacar a personas de alrededor. APIR Ma-
Comunicación, elaborada por Andreasen para evaluar el nual de Psicopatología, Tema 10.
discurso individual a través de la entrevista clínica, pero
que no se corresponden con la definición del enunciado.
La presión del habla es el incremento en la cantidad de ha- 150. ¿Cuál de estas perturbaciones de la memoria
bla espontánea comparado con lo que se considera social- constituye, más propiamente, una paramnesia del
mente adecuado. El paciente habla rápidamente y es difícil recuerdo?:
interrumpirle (opción 1 incorrecta). La tangencialidad se
refiere a cuando se le realiza una pregunta al paciente y 1. Agnosia.
éste responde de forma oblicua, tangencial o incluso irrele- 2. Confabulación.
vante. Por ejemplo: - ¿Cuántos años tienes? - Tantos como 3. Amnesia reduplicativa.
las pirámides que se van desintegrando (opción 2 incorrec- 4. Criptomnesia.
ta). La circunstancialidad se refiere a cuando el objetivo del
discurso no se alcanza directamente, puesto que el hablante Respuesta correcta:2
señala detalles tediosos y hace excesivos paréntesis (opción
3 incorrecta). APIR Manual de Psicopatología, Tema 7. El término paramnesia es introducido por Kraepelin para
definir las distorsiones de memoria en grado patológico,
debido a la inclusión de detalles falsos o por una referencia
149. La flexibilidad cérea, el negativismo o la cata- temporal incorrecta. Freud denominó parapraxias a aque-
lepsia que pueden estar presentes como síntomas de ca- llos errores, aparentemente sin importancia, como la punta
tatonia según DSM 5- se manifiestan: de la lengua o el olvido de nombres y fechas. Normalmente
se distingue entre las paramnesias del recuerdo y las del
1. Cuando el paciente es inducido a realizar ciertas reconocimiento. Dentro de las anomalías en el recuerdo
acciones o movimientos. encontramos: 1) el fenómeno de “no puedo ubicarle”, 2)
2. Cuando el paciente decide realizar un movimien- “conozco la cara pero no el nombre”, 3) sensación de co-
to voluntario, ya que estos síntomas están bajo su control nocer, 4) punta de la lengua, 5) laguna temporal, 6) verifi-
voluntario. cación de tareas o checking y 7) pseudomemorias y falsi-
3. Cuando el paciente realiza un movimiento invo- ficación de la memoria (hacen referencia a la elaboración
luntario. de recuerdos para rellenar lagunas mnémicas, donde se
4. Cuando el paciente está en reposo. engloban la confabulación, la pseudología fantástica y los
recuerdos delirantes). De entre las opciones de respuesta,
Respuesta correcta:1 la única que pertenece a una anomalía en el recuerdo es la
confabulación (opción 2 correcta), que consiste en la inven-
La catatonía es un síndrome psicomotor que puede incluir ción de recuerdos por parte del paciente sin la intención de
diversos síntomas como: 1) catalepsia, que consiste en una mentir, al intentar dar respuestas a cosas que no recuerda.
actitud inmóvil con la musculatura rígida que permite co- Dicho fenómeno aparece en el síndrome amnésico. La am-
locar a un individuo en una posición forzada, incómoda o nesia reduplicativa y la criptomnesia son anomalías en el
antigravitatoria y que éste permanezca en la misma pos- reconocimiento (opciones 3 y 4 incorrectas). La agnosia es
tura sin intentar recuperar la posición original durante un una alteración en el reconocimiento de objetos a través de
tiempo indefinido, 2) flexibilidad cérea, que es una espe- los sentidos, debido a una lesión del sistema nervioso cen-
cial forma de resistencia pasiva que se opone a la flexión tral (opción 1 incorrecta). APIR Manual de Psicopatología,
de las articulaciones, 3) negativismo, que consiste en una Tema 6.
actitud de oposición ante cualquier movimiento que se le
ordena al individuo, pudiendo ser pasivo (actitud sistemá-
tica de oposición) o activo (hacer precisamente lo contrario 151. En cuanto a la incidencia del trastorno de
cuando se le intenta obligar a que realice alguna acción), 4) la personalidad obsesivo-compulsivo en población ge-

46
neral: 1. Su presencia en población general es menor
del 1% (DSM 5).
1. Se da más en mujeres que en hombres. 2. Su presencia en población clínica es del 7
2. Se da más en hombres que en mujeres. al
3. Se ha constatado que es coincidente entre hom- 15 % (DSM 5).
bres y mujeres. 3. La mayor comorbilidad se da con las parafi
4. Las debilidades metodológicas de los estudios no lias y las disfunciones sexuales.
per miten conclusiones fiables en este senti- 4. Los factores genéticos parecen explicar el mayor
do. porcentaje de varianza del trastorno.

Respuesta correcta:2 Respuesta correcta:1

El trastorno de la personalidad obsesivo-compulsivo se El trastorno de la personalidad narcisista se caracteriza por


caracteriza por un patrón general de preocupación por el un patrón general de grandiosidad (en la imaginación o en
orden, el perfeccionismo y el control mental e interper- el comportamiento), necesidad de admiración y falta de
sonal, a expensas de la flexibilidad, la espontaneidad y empatía. Según el Manual de Psicopatología de Caballo
la eficiencia. Según DSM5, es uno de los trastornos de la (2014), los porcentajes ofrecidos por el DSM-5 para este
personalidad más prevalentes en la población general, con trastorno van del 2 al 16 % para la población clínica (op-
una estimación de la prevalencia de entre un 2,1 y un 7,9 ción 2 incorrecta) y menos del 1% para la población gene-
%. En los estudios sistemáticos (opción 4 incorrecta), se ral (opción 1 correcta). La mayor comorbilidad se da con
diagnostica aproximadamente con el doble de frecuencia la depresión mayor y la distimia (42-50%), seguidas de los
en el sexo masculino (opción 2 correcta; opciones 1 y 3 trastornos por consumo de sustancias (24-50%) y del tras-
incorrectas). Una mayor prevalencia en hombres, también torno bipolar (5-12%) (opción 3 incorrecta). Aunque son
se ha constatado en el trastorno de personalidad paranoide, importantes las diferencias individuales constitucionales y
el esquizoide, el esquizotípico, el antisocial y el narcisista. temperamentales, tales como la hipersensibilidad, la tole-
Algunos trastornos de personalidad, como el límite o el his- rancia a la frustración y la regulación de las emociones;
triónico, son más prevalentes en mujeres que en hombres. las experiencias evolutivas, más que los factores genéticos,
Por último, el trastorno de personalidad evitativo, tiene una constituyen factores importantes para explicar el desarrollo
prevalencia similar en hombres y mujeres. APIR Manual de del trastorno narcisista de la personalidad (opción 4 inco-
Psicología Clínica, Tomo II, Tema 19. rrecta). APIR Manual de Psicología Clínica, Tomo II, Tema
19.

152. Señale cuál de las siguientes anomalías neu-


roanatómicas o neurofisiológicas NO es propia de la 154. Respecto del trastorno de despersonalización
esquizofrenia: cabe afirmar que:

1. Disminución de los surcos corticales. 1. La duración de los “episodios” es siempre


2. Disminución del volumen talámico. breve (no persistentes).
3. Disminución de lóbulos frontales y tempora 2. El curso del “trastorno” es usualmente episó
les. dico, no crónico.
4. Disminución en el metabolismo de la glucosa 3. Entre sus características clínicas figuran las
en lóbulos frontales. rumiaciones obsesivas y preocupacio
nes somáticas.
Respuesta correcta:1 4. En el transcurso del trastorno se alteran las
pruebas de realidad.
En los pacientes con esquizofrenia se han descrito dife-
rentes alteraciones neurológicas (estructurales y funciona- Respuesta correcta:3
les), entre las que se encuentran: dilatación ventricular (en
ventrículos laterales, tercer y cuarto ventrículo), aumento Los individuos con trastorno de despersonalización/des-
de surcos corticales (opción 1 correcta), atrofia cortical realización pueden tener dificultades para describir sus sín-
(especialmente en zonas frontales), atrofia cerebelosa, tomas y pueden pensar que están “locos” o “volviéndose
disminución del volumen del tálamo (opción 2 incorrec- locos”. Otra experiencia frecuente es el temor a un daño
ta), asimetría hemisférica invertida, disminución cerebral cerebral irreversible. Un síntoma común asociado es una
(especialmente en lóbulos frontales y temporales) (opción alteración subjetiva del sentido del tiempo (p. ej., dema-
3 incorrecta), disminución craneal, hipofrontalidad o dis- siado rápido o demasiado lento), así como una dificultad
minución del metabolismo de glucosa en el lóbulo frontal subjetiva para recordar vívidamente las memorias del pa-
(opción 4 incorrecta). APIR Manual de Psicología Clínica, sado y para ser dueño de las mismas, tanto personal como
Tomo I, Tema 4. emocionalmente. Los síntomas somáticos (opción 3 correc-
ta) tenues, como la saturación de la cabeza, el hormigueo
o la sensación de desmayo, no son infrecuentes. Las per-
153. Señale la afirmación correcta respecto al sonas pueden sufrir una preocupación obsesiva y una ru-
trastorno narcisista de la personalidad: miación marcada (opción 3 correcta) (p. ej., se obsesionan

47
constantemente sobre si realmente existen o controlan sus 1. La menos fiabilidad de los instrumentos y la
percepciones para determinar si parecen reales). También mayor complejidad matemática de los modelos
son características asociadas frecuentes diversos grados de antiguos.
ansiedad y depresión. Se ha observado que las personas con 2. La ausencia de invarianza y la consideración
este trastorno tienden a tener una hiporreactividad fisiológi- de un nivel de precisión uniforme
ca a los estímulos emocionales. Sustratos neuronales de in- 3. La ausencia de invarianza y la ausencia de va
terés son el eje hipotálamo-hipófisis-suprarrenal, el lóbulo lidez predictiva.
parietal inferior y los circuitos de la corteza prefrontal-lím- 4. El grado de dificultad de los ítems y la menor
bica. La duración de los episodios del trastorno de des- validez de contenido.
personalización/desrealización puede variar ampliamente,
desde breves (horas o días) a prolongados (semanas, meses Respuesta correcta:2
o años) (opción 1 incorrecta). El curso de la enfermedad es
a menudo crónico (opción 2 incorrecta). En alrededor de un
tercio de los casos se trata de episodios separados, otro ter-
cio presenta síntomas continuos desde el principio, y otra
tercera parte tiene un curso inicialmente episódico que con 157. La fórmula de Spearman-Brown se usa para esti-
el tiempo se convierte en continuo. Por último, la opción mar el cambio en el coeficiente de fiabilidad de un test
4 es incorrecta ya que el criterio B del DSM-5 refiere lo que se produce cuando aumenta:
siguiente: “B. Durante las experiencias de despersonaliza-
ción o desrealización, las pruebas de realidad se mantienen 1. La variabilidad de una nueva muestra en que se
intactas” (opción 4 incorrecta). APIR Manual de Psicología analiza el test.
Clínica, Tomo II, Tema 11. 2. La longitud de un test añadiendo ítems paralelos
a los del test original.
3. El tamaño de una nueva muestra en que se anali
155. Respecto del trastorno obsesivo compulsivo za el test.
(TOC), señale la opción correcta: 4. La dificultad de los ítems en una nueva versión
del test.
1. Los niños están menos afectados que las ni
ñas. Respuesta correcta:2
2. La remisión espontánea es muy alta.
3. En comparación con trastornos de ansiedad y Como hemos visto, uno de los problemas de la Teoría Clá-
sica de los Test (TCT) es que las estimaciones que realiza
depresión, los pacientes con TOC tienen ma
del coeficiente de fiabilidad NO son independientes de la
yor probabilidad de ser solteros y estar
muestra sobre la que se aplica el test. Esto implica que de-
en paro. terminadas variables de la muestra afectan al cálculo de la
4. El comienzo suele ser brusco/súbito como fiabilidad, entre ellas la variabilidad de las puntuaciones o
respuesta a situaciones de estrés. la longitud del test. Si calculamos la fiabilidad por el méto-
do de dos mitades paralelas hemos de corregir su valor, ya
Respuesta correcta:3 que la correlación entre ambas nos daría el coeficiente de
un test de mitad de tamaño que el original. Es de esperar,
En comparación con trastornos de ansiedad o depresión, por tanto, que el aumento de ítems del test o su mayor lon-
los pacientes con TOC tienen una mayor probabilidad de gitud influyan en el coeficiente de fiabilidad aumentándolo,
ser solteros, estar en paro (opción 3 correcta) y presentar aunque esto solo sucederá si los ítems añadidos son parale-
importantes deterioros en el funcionamiento ocupacional y los a los anteriores. La fórmula con la que se puede calcular
social. El TOC aparece de modo gradual (opción 4 inco- cuál es la fiabilidad resultante de un test al aumentarlo k
rrecta) y tiene un curso crónico con fluctuaciones reactivas veces se denomina fórmula de Spearman-Brown (opción
a acontecimientos estresantes. En ocasiones se relaciona 2 correcta, opciones 1, 3 y 4 incorrectas). APIR Manual de
con cambios vitales. La OMS sitúa el TOC dentro de una Psicología Experimental y Psicometría, Tema 3.
de las diez condiciones más incapacitantes. Son poco fre-
cuentes las remisiones completas (opción 2 incorrecta) y
después de una remisión puede existir recaídas producidas
por ansiedad, depresión (gran comorbilidad), fatiga u otros
158. Para interpretar adecuadamente los resulta-
factores. Además, es importante señalar que en los varones
dos obtenidos mediante un análisis factorial de la va-
es frecuente que el trastorno se inicie antes, es por ello que
rianza debe tenerse en cuenta (indique la afirma-
el TOC, durante las primeras etapas del desarrollo, es más ción INCORRECTA):
frecuente en varones (opción 1 incorrecta). APIR Manual
de Psicología Clínica, Tomo I, Tema 7.
1. Cuando el efecto de interacción es estadísticamen-
te significativo, el interés de los efectos principales se redu-
cen en cierta medida dado que su interpretación depende de
156. Algunas de las limitaciones de la Teoría clá- los niveles del resto de factores.
sica de los test frente a la Teoría de respuesta al ítem
2. El análisis de los efectos simples implica interpre-
vienen por:
tar el efecto de un factor cuando se tiene en cuenta un único

48
nivel del otro factor. dicción que tiene nuestra recta de sobre la variable depen-
3. El análisis de los efectos simples sólo tiene sen- diente. Esta valoración se realiza mediante el coeficiente de
tido cuando la interacción entre los factores es estadística- determinación, que se calcula elevando al cuadrado el coe-
mente significativa. ficiente de correlación de Pearson, e indica qué proporción
4. El efecto de interacción solo será estadísticamente de varianza del criterio (X) queda explicada por el modelo
significativo si los efectos principales también lo son. lineal (opción 3 incorrecta por ser verdadera). La ecuación
de regresión sería la de un plano (para dos variables predic-
Respuesta correcta:4 toras): Y = B0 + B1 X1 + B2 X2. Con B1 representando
la pendiente del plano en el eje 1, B2 representando la pen-
En un análisis factorial de la varianza (ANOVA II) existe diente del plano en el eje 2 y con B0 representando la altura
2 factores o variables independientes manipuladas por el a la que corta el plano el punto Y (el eje de ordenadas). Este
experimentador. Cuando los resultados arrojan diferencias es el valor de la constante, esto es, el valor de Y cuando
estadísticamente significativas, la variabilidad puede ser el predictor/es es/son igual a 0 (opción 2 incorrecta). Aña-
debida: al factor A, al factor B o a la interacción del factor dir variables independientes o predictoras (Y) al modelo
A con el B (opción 4 correcta) (más una variabilidad error), siempre podrá aumentar o mantener el coeficiente de deter-
para lo cual existen unas reglas de jerarquía: si el efecto in- minación, ya que vamos sumando información predictora
teractivo resulta significativo, no se interpretan los efectos a nuestro modelo. En este sentido, para poder obtener el
principales de los factores que conforman tal interacción, mejor modelo con el menor número de variables posibles,
sino que solo se analizará dicho efecto interactivo y los se puede recurrir a los siguientes métodos: 1) jerárquico; 2)
correspondientes efectos simples (es decir, el efecto de un introducir (simultáneo); 3) escalonado (stepwise); 4) hacia
nivel de un factor bajo cada nivel del otro). Es interesante atrás (backward); 5) hacia adelante (forward); y 6) eliminar
conocer que los efectos principales pueden no resultar es- (opción 4 incorrecta por ser verdadera). APIR Manual de
tadísticamente significativos, pero sí alcanzar la significa- Psicología Experimental y Psicometría, Tema 1.
ción estadística un efecto interactivo, al ser éste último el
que mejor explique la varianza de la variable dependiente
(opciones 1, 2 y 3 incorrectas por ser verdaderas). APIR
Manual de Psicología Experimental y Psicometría, Tema 2. 160. En el marco del análisis de la varianza, con
un factor y tres tratamientos, indique la afirmación
INCORRECTA:
159. En el ámbito del análisis estadístico, en con-
creto de la regresión lineal, indique la afirmación IN- 1. La prueba de Dunn-Bonferroni nos permitiría lle-
CORRECTA: var a cabo comparaciones múltiples entre pares de medias
planificadas o a priori.
1. La regresión lineal permite analizar la relación 2. Si el investigador desea realizar todas las posibles
entre uno o más predictores cuantitativos y una variable comparaciones por pares, post hoc o a posteriori, podía uti-
dependiente también cuantitativa, no pudiéndose en ningún lizar la prueba de Tukey.
caso incluir en el análisis predictores categóricos. 3. Para estimar el tamaño del efecto relativo a la pro-
2. En el análisis de regresión, el coeficiente de regre- porción de varianza explicada, sería aconsejable la utiliza-
sión parcial no estandarizado Bo ó b0 ofrece el valor de ción de eta-cuadrado (b2), por ser un estimador no sesgado.
la ecuación de regresión cuando todos los predictores son 4. En el caso de incumplir el supuesto de homoge-
igual a cero. neidad de las varianzas se recomienda utilizar los estadísti-
3. Una estrategia para medir la bondad de ajuste en cos W de Welch o F de Brown y Forsythe, como alternativa
un análisis de regresión simple es utilizar el coeficiente de al estadístico F.
determinación que se calcula elevando al cuadrado el coe-
ficiente de correlación de Pearson. Respuesta correcta:3
4. Si se desea encontrar la ecuación de regresión ca-
paz de ofrecer el mejor ajuste posible con el menos número En un ANOVA de 3 niveles, tendríamos un único factor
de variables, se recomienda utilizar la regresión jerárquica o variable independiente (llamémoslo A) y tres niveles o
o por pasos. tratamientos diferentes, (A1, A2 y A3). Para poder aplicar
un análisis de la varianza (ANOVA) han de cumplirse una
Respuesta correcta:1 serie de supuestos, entre ellos la homocedasticidad o igual-
dad de varianzas. En el caso de que no se cumpla podemos
La regresión es una fórmula matemática que nos permite transformar los datos mediante logaritmo neperiano, míni-
establecer predicciones a partir de las relaciones lineales mos cuadrados, test de Welch o prueba de Brown-Forsythe
observadas entre variables. Los predictores en la regresión (opción 4 incorrecta por ser verdadera). Una vez aplicada la
lineal múltiple pueden ser de un nivel de medida cuanti- prueba y sabiendo que los resultados son estadísticamente
tativo, ordinal o nominal (o categórico) (opción 1 correc- significativos, es decir, que el factor A tiene efecto sobre
ta). Es decir, conociendo la puntuación de un sujeto en una la variable dependiente que queremos medir, habremos
variable (X), se pronostica su puntuación en otra variable de especificar cuáles son los niveles de nuestro factor que
(Y). A menos que la correlación entre las dos variables sea producen dicho efecto. Para comprobar esto se recurre a
perfecta, los dos pares de puntuaciones no coinciden en una distintas pruebas estadísticas. Algunas pruebas de compro-
misma recta. Es decir, hemos de valorar el grado de pre- bación no planeada o a posteriori son la prueba de Scheffé,

49
Tuckey (opción 2 incorrecta por ser verdadera) o Dunnett, nes es INCORRECTA:
o a priori las de Dunn-Sidák o Dunn-Bonferroni (opción 1 1. El diseño de bloques tiene como objetivo, entre
incorrecta por ser verdadera). No obstante, en la actualidad otros, el análisis de la interacción entre la variable de trata-
se emplean otras pruebas, no basadas en la significación miento y la variable de bloqueo.
estadística de decisión dicotómica (rechazo versus man- 2. El diseño de cuatro grupos de Solomon permite
tengo la H0), como son los índices de tamaño de efecto. analizar el posible efecto de sensibilización del pretest, en
En el caso del ANOVA, se recomienda el uso de omega el efecto del tratamiento.
cuadrado, ya que Eta-cuadrado η2 es un estimador sesgado 3. En los diseños de medidas repetidas, el principal
a nivel poblacional (opción 3 correcta por ser falsa). APIR problema es el derivado de la secuencialidad en la que se
Manual de Psicología Experimental y Psicometría, Tema 1. aplican los tratamientos.
4. Los diseños experimentales de covarianza tienen
como objetivo la reducción de la varianza de error, inclu-
161. Un investigador desea analizar el impacto de yendo variables cuantitativas como covariables.
una intervención para mejorar las capacidades de lec-
to-escritura. Para ello, antes de aplicar la intervención Respuesta correcta:1
mide la capacidad de lecto-escritura en la muestra y
establece un punto de corte. Los sujetos con puntuacio- El bloqueo es una técnica de control estadístico que con-
nes por encima del punto de corte constituirán el gru- siste en neutralizar el efecto de una variable contaminadora
po de control y aquéllos por debajo del punto de corte (variable de bloqueo) mediante la formación de conjuntos
recibirán la intervención (grupo experimental).Tras o bloques de sujetos que comparten una característica re-
la intervención, el investigador medirá nuevamente levante al fenómeno estudiado. A continuación se forman
la capacidad en lecto-escritura de todos los sujetos. Se-
grupos que tengan un número similar de sujetos en cada
gún este enunciado, señale la opción INCORRECTA:
uno de los bloques, lo que permite la formación de grupos
homogéneos una vez que se ha eliminado a los sujetos con
1. Se ha utilizado una regla de asignación no aleato- una puntuación extrema en dicha variable. En los diseños
ria pero conocida. intergrupo de bloques, por tanto, es posible interpretar los
2. Se trata de un diseño de discontinuidad en la re- resultados del efecto de la variable independiente sobre la
gresión. variable dependiente con las garantías de haber bloqueado
3. Se trata de un diseño con grupo de control no variables contaminadoras (opción 1 correcta por ser falsa).
equiva lente. Otra herramienta para el control estadístico del error ex-
4. Se ha utilizado un diseño cuasi-experimental pre perimental es el análisis de la covarianza (ANCOVA), que
test-postest con grupo de control. permite eliminar la heterogeneidad causada en la variable
de interés (VD) por la influencia de una o más variables
Respuesta correcta:3 (covariables) (opción 4 incorrecta por ser verdadera).El di-
seño Solomon es un tipo de diseño experimental unifacto-
Dentro de los diseños cuasi-experimentales podemos dis- rial donde la creación de cuatro grupos (grupos con/sin me-
tinguir aquellos con y sin grupo control. Dentro de los di- dida pretratamiento y grupos con/sin variable tratamiento)
seños con grupo control, a su vez, distinguimos diseños de permite mejorar las valideces internas y externas. Una de
grupo control NO equivalente (opción 3 correcta por ser sus ventajas es que permite controlar el efecto de la medida
falsa), y el diseño de discontinuidad en la regresión. En pre-tratamiento al darse una ausencia de interacción entre
este último, la regla de asignación de los sujetos a los tra- esta y la variable independiente (o tratamiento) (opción 2
tamientos se conoce, es decir, NO es aleatoria pero es co- incorrecta por ser verdadera). Los diseños de medidas repe-
nocida (opción 1 incorrecta por ser verdadera). Los sujetos tidas o diseños intragrupo estudian el efecto de una variable
se asignan a la condición experimental o a la condición de independiente o tratamiento dentro de un mismo grupo, por
control en función de las puntuaciones de la medida pretra- lo que aparecen errores progresivos debido a la repetición
tamiento, de forma que aquellos sujetos que puntúen por de tareas por los mismos sujetos (opción 3 incorrecta por
debajo o por encima del punto de corte son asignados a un ser verdadera). APIR Manual de Psicología Experimental y
grupo o a otro (se establece un punto de corte). Se deno- Psicometría, Tema 2.
mina diseño de discontinuidad en la regresión porque se
calcula una recta de regresión a partir de las puntuaciones
obtenidas en el pretest y postest (si el tratamiento no tiene 163. En lo que respecta a la validez de conclusión
efecto los resultados se mostrarán en una sola línea, la línea estadística, indique cuál de las siguientes afirmaciones
iniciada en el grupo control se prolongará en la del experi- es correcta:
mental. En cambio, si el tratamiento tiene efecto se muestra
una discontinuidad en la línea a partir del punto de corte) 1. La potencia es complementaria al error de tipo I y
(opciones 2 y 4 incorrectas por ser verdaderas). Es uno de se define como la capacidad de una prueba estadística para
los diseños cuasiexperimentales con los que puede inferir- detectar una relación entre variables.
se relaciones causales con más garantías. APIR Manual de 2. El error de tipo II se refiere al hecho de detectar
Psicología Experimental y Psicometría, Tema 2. una relación entre variables cuando de hecho no existe tal
relación en la población.
162. En relación con los diseños de investigación 3. La probabilidad de cometer un error de tipo I se
en Psicología, indique cuál de las siguientes afirmacio- denomina alpha y se refiere a rechazar la hipótesis nula

50
cuando dicha hipótesis es verdadera. nismo a la misma cantidad de droga o por la necesidad de
4. El tamaño muestral tiene una relación inversa- una dosis mayor para conseguir el mismo efecto. Si nos
mente proporcional con la potencia de la prueba estadística. preguntan acerca del condicionamiento de la tolerancia, en-
tenderemos que hacen referencia a la tolerancia condicio-
nada (descrito por Siegel), que hablaría sobre las respuestas
Respuesta correcta:3 condicionadas a los estímulos del entorno que se asocian
con la administración de la droga (opción 4 correcta). La
Un contraste de hipótesis es una decisión en la que una hi- opción 1 se referiría al modelo de procesos oponentes de
pótesis formulada en términos estadísticos se pone en rela- Solomon y Corbit, que habla de la tolerancia y abstinen-
ción con unos datos empíricos, y en el que se determina si cia como procesos opuestos que buscan la homeostasis del
existe una compatibilidad entre los dos. El sujeto plantea organismo pero no del proceso de condicionamiento en sí
una hipótesis nula (H0), que supone que no existe relación (opción 1 incorrecta). La opción 2 estaría sugiriendo que
entre las variables, en contraste con una hipótesis alternati- la exposición repetida a la droga elimina el placer, lo que
va (H1), que niega a H0 e incluye todo lo que excluye H0 sería falso (opción 2 incorrecta). La opción 3 no se ajusta-
(es decir, que existe, en una u otra dirección, relación entre ría a ningún modelo concreto (opción 3 incorrecta). APIR
las variables). Para decidir si mantener o rechazar la hipó- Manual de Psicología Clínica, Tomo I, Tema 3.
tesis nula se recurre a la regla de la decisión, que consiste
en cuantificar el nivel de riesgo o significación que se está 165. En la tarea de seguimiento (sombreado)
dispuesto a aceptar. Es decir, define un margen de error a la ideada por Cherry (1953) se utilizó la escucha dicóti-
hora de considerar como falsa una hipótesis que en realidad ca y se presentaron dos mensajes uno por cada oído.
es verdadera (el riesgo que estoy dispuesto a cometer de El participante debía repetir palabra por palabra el
rechazar H0 cuando realmente es verdadera, o lo que es mensaje del oído atendido. Cuando se preguntó por
lo mismo, decir que existe una relación entre las variables el mensaje ignorado (no atendido) los participantes de-
estudiadas cuando realmente no la hay). Este nivel riesgo o tectaron:
significación se representa mediante α, y supone, como he-
mos explicado, cometer un error tipo I (opción 3 correcta). 1. El cambio de idioma.
De forma complementaria tenemos un nivel de confianza, 2. El significado del mensaje.
1-α, que se define como la probabilidad de mantener H0 3. La relación semántica entre el mensaje atendido y
cuando esta es verdadera. Pero ¿Qué sucede si realmente no atendido.
existe una relación entre las variables estudiadas y no la 4. El cambio de voz del hablante.
detectamos? En este caso, mantener la H0 cuando es fal-
sa supone cometer un error tipo II, y se representa con la Respuesta correcta:4
letra β (opción 2 incorrecta). Complementaria al error tipo
II estaría la potencia de la prueba o 1-β, que muestra la En el paradigma de seguimiento de Cherry se realizó un
capacidad de una prueba estadística para rechazar una H0 experimento en el cual se presentaba dos mensajes distintos
cuando realmente es falsa (es decir, para detectar la rela- a los sujetos, uno por cada oído, y se les pedía que prestasen
ción entre variables) (opción 1 incorrecta). La potencia de atención únicamente a uno de ellos. Con este procedimien-
una prueba es directamente proporcional al tamaño de la to se observaba que la atención es selectiva, ya que se re-
muestra, a α y al tamaño del efecto, es decir, cuanto mayor cordaba perfectamente el estímulo atendido, pero también
es α, cuanto mayor tamaño del efecto y cuanto mayor sea que el estímulo no atendido podía mantener alguna de sus
la muestra, mayor potencia estadística tendremos (opción características sensoriales más gruesas en el recuerdo del
4 incorrecta). APIR Manual de Psicología Experimental y sujeto (por ejemplo, el cambio de voz), aunque no su sig-
Psicometría, Tema 1. nificado (opción 1, 2 y 3 incorrectas; opción 4 correcta).
Este paradigma permitió sentar la base de algunos modelos
de filtro precategorial como los de Broadbent y Treisman.
164. El modelo de condicionamiento de la tolerancia APIR Manual de Psicología Básica, Tema 2.
a las drogas atribuye la tolerancia:

1. Al efecto compensatorio de la homeostasis. 166. ¿Qué dos condiciones o pistas (señales) se u t i -


2. A la sobreexposición al estímulo incondicionado lizan en el Test de las redes atencionales (Attentional
(droga) que produce extinción del placer asociado. Network Test, ANT), para calcular la puntuación co-
3. Al contracondicionamiento de las influencias po- rrespondiente a la red de orientación?:
sitivas del entorno familiar y social para que se
deje la droga. 1. La pista (señal) central, y la pista (señal) espacial
4. A las respuestas condicionadas a los estímulos del o periférica.
entorno que se asocian con la administración de 2. La pista (señal) central, y la doble pista (señal).
la droga. 3. La pista (señal) doble, y la no pista (no señal).
4. La no pista (no señal), y la pista (señal) espa
Respuesta correcta:4 cial o periférica.

Entendemos como “tolerancia” al estado de adaptación Respuesta correcta:1


caracterizado por la disminución de la respuesta del orga-

51
El Test de las Redes Atencionales (ANT) fue elaborado con Básica, Tema 3. “
el fin de utilizarse para evaluar las redes atencionales pro-
puestas por Posner: 1) Red de alerta: Encargada de mante-
ner el estado de vigilia y activación del organismo; 2) Red 168. Según el modelo de covariación de Kelley
de orientación: Que dirige la atención a los estímulos, de ¿Qué tipos de información utilizan los sujetos para
forma voluntaria o involuntaria; 3) Red de control ejecu- realizar atribuciones sobre las causas de la conducta de
tivo: Controla procesos ejecutivos como la inhibición de otra persona?:
información distractora o la supervisión de la propia con-
ducta. En el ANT el sujeto debe indicar si una flecha (estí- 1. Locus externo/interno, estabilidad y controlabili-
mulo objetivo) que aparece en pantalla apunta a la izquier- dad.
da o la derecha, midiéndose el tiempo de reacción. Esta 2. Coherencia/consistencia, distintividad y consen-
flecha puede aparecer acompañada de flechas que apuntan so.
en la misma dirección (flanqueo congruente), o de flechas 3. Efectos no comunes, relevancia hedónica y
que apuntan en dirección contraria (flanqueo incongruen- personalismo.
te). Adicionalmente, puede presentarse acompañada o no
4. Correspondencia, agencialidad y causalidad.
de pistas/señales según la red que se esté tratando de eva-
luar: 1) Para evaluar la red de alerta se resta el tiempo de
respuesta medio en la condición de pista doble del tiempo Respuesta correcta:2
de respuesta medio en la condición de no pista (opción 3
incorrecta); 2) Para evaluar la red de orientación se resta el El locus de causalidad, estabilidad y controlabilidad for-
tiempo de respuesta medio en la condición de pista espa- man las tres dimensiones con las que Weiner desarrolla su
cial del tiempo de respuesta medio en la condición de pista Teoría de la atribución (opción 1 incorrecta). Kelley plan-
central (opción 1 correcta); 3) Para evaluar la red de control teó un modelo de covariación en el que una persona atri-
ejecutivo se resta el tiempo de respuesta medio de todas las buye la causa de una conducta o acción a la persona que
situaciones de flanqueo congruente del tiempo de respuesta realiza la acción, a la entidad u objeto a la cual va dirigida
medio en todas las situaciones de flanqueo incongruente. la acción o a las circunstancias que la rodean. Esta atribu-
APIR Manual de Psicología Básica, Tema 2. ción va a depender fundamentalmente de tres factores: Co-
herencia/consistencia, distintividad y consenso (opción 2
167. De acuerdo con los estudios de Tulving y correcta). La opción 3 hace referencia a conceptos propios
otros autores sobre la recuperación de información de de la Teoría de las inferencias correspondientes de Jones y
la memoria, ¿qué claves de recuperación son las más Davis. El Principio de los efectos no comunes dice que el
eficaces para recuperar un determinado contenido de sujeto que percibe realizará la inferencia correspondiente
la memoria episódica?: con mayor confianza cuando la acción elegida tenga pocas
consecuencias únicas o no comunes, es decir, pocas carac-
1. Las claves que están más fuertemente relaciona- terísticas diferenciadoras. La relevancia hedónica se refiere
das semánticamente con el contenido que queremos recu- a que cuanto más nos afecta lo que hace un sujeto, mayor
perar. probabilidad de atribución a factores internos o personales
2. Las claves que, además de al contenido que quere- del mismo. Y el personalismo consiste en la evaluación a
mos recuperar, están asociadas a más contenidos diferentes otro individuo de forma más extrema, sobre todo de forma
de nuestra memoria episódica. negativa, si consideramos que su conducta estaba dirigida
personalmente a nosotros (opción 3 incorrecta). La opción
3. Las claves a las que se presta menos atención du- 4 (incorrecta) habla de conceptos no relacionados con esta
rante la recuperación. teoría. APIR Manual de Psicología Social, Tema 2.
4. Las claves que estuvieron presentes durante la co-
dificación y fueron codificadas junto con el contenido que
ahora queremos recuperar.
169. Según el modelo de la Teoría de la acción ra-
zonada, ¿qué componente determina la actitud general
Respuesta correcta:4 hacia una conducta?:
“En la hipótesis de la especificidad de codificación de
Tulving, se rechaza la idea de que las palabras tienen su 1. La intención conductual.
representación física fija, proponiendo que existe especi- 2. Creencia que la persona tiene sobre las conse-
ficidad de la codificación de un término en cada situación cuencias de la conducta.
particular. Por ello, sostiene que cada vez que se presenta 3. La norma subjetiva.
una palabra se codifica de manera específica en cada si- 4. Las creencias normativas.
tuación. Según defiende esta hipótesis, el rendimiento en
el recuerdo será mejor cuando se utiliza el mismo indicio
en la codificación. Es decir, que nos resultará más sencillo Respuesta correcta:2
recordar claves que estuvieron presentes en la codificación
del recuerdo que tratamos de recuperar, y que no necesa- Según la Teoría de la acción razonada, el determinante in-
riamente deben estar relacionadas semánticamente con el mediato de que el sujeto ejecute o no la “conducta” final
contenido (opción 4 correcta). APIR Manual de Psicología va a depender directamente de la “intención conductual”

52
(opción 1 incorrecta). Ésta a su vez va a depender, por un Psicología Social, Psicología de los grupos, Tema 4.
lado, de la evaluación positiva o negativa que el sujeto hace
sobre la ejecución de la conducta, lo que se conoce como
“actitud hacia la conducta” (opción 2 correcta). Y por otro 171. ¿Cuál de las siguientes opciones NO recoge
lado, va a depender también del juicio que el sujeto realice una de las estrategias de obtención de identidad social
sobre las probabilidades de que otros individuos importan- positiva propuestas por la Teoría de la identidad so-
tes para él deseen que lleve a cabo dicha conducta, o lo que cial?:
es lo mismo, de la “norma subjetiva” (opción 3 incorrec-
ta). Esta norma subjetiva se ve influida a su vez por dos 1. Creatividad social.
aspectos: por un lado las “creencias normativas”, es decir, 2. Movilidad individual.
creencias que el sujeto tiene respecto a lo que las personas 3. Competición social.
relevantes de su entorno piensan que debería hacer; y por
4. Facilitación social.
otro, la “motivación para acomodarse” a tales expectativas,
esto es, en qué medida el individuo se deja llevar por lo
que opinan los seres significativos de su entorno sobre lo Respuesta correcta:4
que debería hacer (opción 4 incorrecta). En este sentido, el
modelo afirma que si las creencias normativas son firmes La Teoría de la identidad social de Tajfel refiere tres estra-
pero la motivación para acomodarse a ellas es inexistente, tegias para enfrentarse a la identidad grupal negativa: 1)
la norma social subjetiva no podrá influir en la intención de Movilidad individual, consiste en dejar el grupo e intentar
la realización de la conducta. La norma subjetiva es consi- pasar al más valorado (opción 2 incorrecta). 2) Creativi-
derada como el producto de los dos factores ya menciona- dad social, consiste en redefinir los términos de la situación
dos, por lo que si uno de ellos falla el producto final sería comparativa adoptando una nueva dimensión de compara-
igual a cero. APIR Manual de Psicología Social, Tema 3. ción, modificando los valores asociados con determinados
atributos de grupo o cambiando el exogrupo con el que se
hace la comparación (opción 1 incorrecta). 3) Competición
social, consiste superar al exogrupo en la misma dimensión
170. Según la Teoría de la categorización del Yo,
en la que era superior (opción 3 incorrecta). La facilitación
la “saliencia” de una categoría social en una situación
concreta es función de dos procesos denominados: social consiste en la modificación de la conducta del in-
dividuo por el hecho de estar siendo observado. Existen
diferentes teorías para explicar este fenómeno, la más co-
1. Prototipicidad y homogeneidad. nocida es la Teoría de la mera presencia de Zajonc que
2. Anclaje y comparación. propone que la presencia de otros llevaba a un incremento
3. Accesibilidad y ajuste. del rendimiento, o facilitación social, si el individuo traba-
4. Atribución y heterogeneidad. jaba en tareas fáciles y bien aprendidas (opción 4 correcta).
APIR Manual de Psicología Social, Psicología de los gru-
Respuesta correcta:3 pos, Tema 4.

Según la Teoría de la categorización del yo (Turner 1982;


Turner y cols. 1987), la saliencia de las categorías hace re- 172. ¿Cuál de los siguientes modelos establece la
ferencia a las condiciones que conducen a las personas a existencia de una “vía central” y una “vía periférica”
categorizarse en un nivel social, con preferencia a uno indi- en los procesos de persuasión:
vidual (o viceversa) en una situación concreta, así como el
nivel social que será más utilizado, esto es, qué categorías 1. El modelo de probabilidad de elaboración.
sociales específicas, entre las diferentes categorías posibles, 2. El modelo de activación automática.
serán psicológicamente significativas en una situación de- 3. El modelo del comportamiento planificado.
terminada. “Accesibilidad y ajuste como determinantes de
4. El modelo de continuidad.
la saliencia” (opción 3 correcta): Accesibilidad; hace refe-
rencia a la disposición de las personas a utilizar categorías
que son centrales, relevantes o útiles, en función de la ex- Respuesta correcta:1
periencia pasada, las expectativas presentes, y los motivos,
valores, metas y necesidades actuales; Ajuste, se refiere a A partir de los planteamientos de McGuire sobre la re-
la relación entre las categorías y la realidad externa, y se levancia de la aceptación del mensaje, Petty y Cacioppo
divide en dos aspectos complementarios: Ajuste compara- (1986) desarrollaron el “Modelo de la Probabilidad de la
tivo, que se basa en el principio según el cual las categorías Elaboración”. Sus estudios sugieren que la evaluación de
que van a ser más salientes son aquellas que maximizan la un mensaje persuasivo se lleva a cabo a través de dos for-
“razón de meta-contraste”, esto es, que hacen que las dife- mas diferentes de procesamiento de la información: la ruta
rencias entre los grupos sean máximas y que las diferencias central y la ruta periférica. A través de la ruta central se ha
dentro del grupo sean mínimas en las dimensiones rele- observado que los receptores gastan bastante tiempo y es-
vantes de comparación. Ajuste normativo, se refiere a que fuerzo en la evaluación crítica del contenido del mensaje. A
esas diferencias intergrupo y semejanzas intragrupo deben través de la ruta periférica el sujeto tiende a usar esquemas
coincidir con las expectativas normativas o estereotipos de simples o reglas de decisión para la evaluación de la vali-
dichas categorías para que éstas sean salientes. Las opcio- dez del argumento y la toma de decisión, usa un procesa-
nes 1, 2 y 4 (incorrectas) son distractoras. APIR Manual de miento heurístico (opción 1 correcta). El modelo MODE,
conocido como “Motivación y Oportunidad como factores

53
Determinantes”, estudia la influencia de la actitud sobre la Respuesta correcta:2
conducta centrándose en la importancia de la accesibilidad
actitudinal. El modelo plantea dos formas fundamentales Janis (1972, 1977) estudió casos de tomas de decisión polí-
de influencia de la actitud en la conducta: 1) Procesamiento ticas y militares catastróficas tomadas en grupos supuesta-
espontáneo, se da una “activación automática de la acti- mente muy inteligentes. Los individuos resultaron víctimas
tud”, lo que presupone una alta accesibilidad de la misma. de manera extrema de polarización grupal, conocida como
2) Proceso deliberativo de larga dración, este proceso su- pensamiento grupal. Se caracteriza por una percepción exa-
pone un análisis detallado y cuidadoso de la información. gerada de la “rectitud moral” de los planteamientos del gru-
(opción 2 incorrecta). El modelo de Comportamiento Pla- po y una “visión homogénea y estereotipada” de los miem-
nificado o Acción Planificada de Ajzen y Madden (1986) bros del exogrupo. Ocurre cuando un grupo muy cohesivo
es una extensión de la Teoría de la Acción Razonada, un o de mentalidad semejante (opción 2 correcta) que ha de
modelo unidimensional centrado en el componente eva- tomar decisiones bajo un contexto de alto estrés (opción 4
luativo de la actitud. Junto a los tres componentes de la incorrecta), está tan condicionado por la búsqueda de con-
teoría anterior, actitud hacia la conducta, norma subjetiva e senso que se deteriora su percepción de la realidad. El pro-
intención, incluye un elemento más, el control conductual ceso se refuerza bajo las siguientes condiciones: 1) Que el
percibido (opción 3 incorrecta). La opción 4 (incorrecta), grupo sea altamente cohesivo (opción 1 incorrecta). 2) Que
no representa ningún modelo relacionado con las actitudes esté privado de otras fuentes de información alternativas
ni con el proceso persuasivo. (opción 4 incorrecta). APIR (opción 3 incorrecta). Que el líder apoye claramente una
Manual de Psicología Social, Tema 3. determinada opción. APIR Manual de Psicología Social,
Psicología de los grupos, Tema 3.

173. Cuando un sujeto se conforma a las expecta-


tivas de una grupo debido a su temor a la desaproba- 175. ¿Cuál de los siguientes NO es un paradigma
ción social, al castigo del grupo, se está produciendo de investigación sobre disonancia cognitiva?:
un proceso de:
1. El paradigma de justificación del esfuerzo.
1. Influencia informativa. 2. El paradigma de complacencia inducida.
2. Conversión. 3. El paradigma adaptación-nivel.
3. Influencia normativa. 4. El paradigma de libre elección
4. Reactancia psicológica.

Respuesta correcta:3
Respuesta correcta:3
Festinger (1957) abre diferentes áreas de investigación con
Cuando una persona se conforma a través de un proceso el fin de comprobar su teoría sobre la disonancia cognitiva.
de influencia normativa, es previsible que sea su conducta Estas áreas son: 1) Paradigma de la complacencia induci-
manifiesta lo único que cambie, y no su convicción previa, da (opción 2 incorrecta), analiza los problemas originados
que continuará manteniéndose en privado; dando lugar a cuando un sujeto se ve obligado a llevar a cabo acciones
un proceso de conformidad pública o sumisión. Tras este que van en contra de sus principios. 2) Paradigma de la
tipo de influencia suele haber gran presión social (opción elección libre (opción 4 incorrecta), se basa en el análisis de
3 correcta).Cuando se conforma en base a la influencia in- la disonancia generada tras decidir entre dos alternativas,
formativa, también se modificará su opinión; dando lugar a también llamada disonancia postdecisional. 3) Exposición
un proceso de conformidad privada o conversión (opciones selectiva a la información, parte de la premisa que tende-
1 y 2 incorrectas). La reactancia psicológica es un concep- mos a buscar las informaciones que reducen la disonancia,
to acuñado por Brehm (1966, 1981). Este autor defiende esto es, llevamos a cabo una exposición selectiva de la in-
que la amenaza o pérdida de alguna libertad suscita en la formación favorable en oposición a la no favorable a nues-
persona un estado motivacional dirigido a la recuperación tras creencias. 4) Paradigma de justificación del esfuerzo
de esa libertad amenazada o perdida (opción 4 incorrecta). (opción 1 incorrecta), se basa en la disonancia generada
APIR Manual de Psicología Social, Psicología de los gru- tras llevar a cabo un esfuerzo que no merece la pena. La
pos, Tema 3. opción 3 (correcta), no se corresponde con ninguna de las
áreas propuestas para la investgación sobre la disonancia
cognitiva. APIR Manual de Psicología Social, Tema 1.
174. ¿Cuál de los siguientes NO es un factor que
favorezca la aparición del denominado “pensamiento
de grupo” según el modelo propuesto por Janis?: 176. La estructura de tres estratos del modelo de
Catell-Horn-Carroll (CHC) sobre la inteligencia esta-
1. La alta cohesión grupal. blece que en el estrato II se sitúan:
2. La heterogeneidad de los miembros en actitu des
e ideología. 1. Las aptitudes intelectuales (por ejem
3. El aislamiento del grupo respecto de la inluencia plo, inte ligencia fluida, cristalizada,
externa. etc.)
4. La existencia de un contexto de alto estrés. 2. El factor G o capacidad general.

54
3. Los factores relacionados con cada aptitud in 178. Señale la respuesta correcta respecto a la disforia
telectual de segundo orden. de género (DG), según el DSM 5:
4. Las tareas que componen la prueba de inteli
gencia. 1. Se trata de un nuevo trastorno introducido en el
DSM 5 dentro de la categoría general de las disfunciones
Respuesta correcta:1 sexuales.
2. La principal diferencia entre la DG en niños y
La principal aportación del modelo de Catell se centra en adolescente es el criterio de duración (6 meses en niños y
desarrollar los factores de segundo orden, entre ellos se 12 en adolescentes).
encuentran la inteligencia fluida, cristalizada, memoria, 3. Si existe otro diagnóstico del desarrollo sexual
aprendizaje... (opción 1 correcta). El factor G o capacidad (por ejemplo, un trastorno androgenital congénito como hi-
general, que en el modelo de Cattel podría equipararse con perplasia adrenal congénita) no debe hacerse el diagnóstico
la Inteligencia fluida histórica, estaría en el tercer nivel de de DG. 4. En niños, el criterio principal (A1) se describe
la pirámide (opción 2 incorrecta). Los factores relaciona- como: “un poderoso deseo de ser del otro sexo o una in-
dos con cada aptitud intelectual de segundo orden hace sistencia de que él o ella es del sexo opuesto (o de un sexo
referencia al primer estrato de la pirámide, o factores pri- alternativo distinto del que se le asigna)”
marios (opción 3 incorrecta). Las tareas que compongan
las pruebas de inteligencia no forman parte del modelo je- Respuesta correcta:4
rárquico de Cattell (opción 4 incorrecta). APIR Manual de
Personalidad y Diferencial, Tema 4. En el DSM 5 la disforia de género supone un capítulo inde-
pendiente (opción 1 incorrecta). La duración del trastorno
en el DSM 5 es la misma tanto para adolescentes/adultos
177. Según el DSM 5, cuando un niño de 10 años como para niños, de 6 meses (opción 2 incorrecta). Puede
alcanza un CI Global de 65 en la evaluación realizada especificarse “con un trastorno de desarrollo sexual” (op-
con un WISC-V, podemos concluir que la gravedad ción 3 incorrecta). Por tanto, la única opción correcta es la
actual del paciente corresponde a: 4, siendo el criterio principal el primero del criterio A: “un
poderoso deseo de ser del otro sexo o una insistencia de que
1. Una discapacidad intelectual Leve. él o ella es del sexo opuesto (o de un sexo alternativo distin-
2. Una discapacidad intelectual Moderada. to del que se le asigna)” (opción 4 correcta). APIR Manual
3. Necesitamos conocer, además del CI Total, de Psicología Clínica, Tomo II, Tema 16.
los índices de inteligencia verbal y manipulativo
para poder especificar la discapacidad y funciona
lidad.
4. No podemos determinar los diferentes niveles 179. ¿Qué tipo de ataque de pánico de la tipología
de gravedad si desconocemos el funcionamiento de Barlow se corresponde con el ataque de pánico limi-
tado situacionalmente propuesto por Klein?:
adaptativo y el nivel de apoyos requeridos.

Respuesta correcta:4 1. Ataque de pánico señalado/no esperado.


2. Ataque de pánico no señalado/no esperado.
La discapacidad intelectual es un trastorno que comienza 3. Ataque de pánico señalado/esperado.
durante el periodo de desarrollo y que incluye limitaciones 4. Ataque de pánico espontáneo.
del funcionamiento intelectual como también del compor-
tamiento adaptativo en los dominios conceptual, social y
práctico (3 incorrecta, opción 4 correcta). Por lo tanto, úni- Respuesta correcta:3
camente conociendo la puntuación del CI, según el DSM
5 no podemos realizar el diagnóstico de Discapacidad In- La correspondencia de la clasificación de Barlow con la de
telectual (opción 1 y 2 incorrectas). El DSM 5 sustituye el Klein es la siguiente: los ataques señalados/esperados de
término Retraso Mental por el de Discapacidad Intelectual. Barlow serían los situaciones de Klein (opción 3 correc-
El DSM IV especificaba la gravedad del retraso mental en ta); los ataques señalados/no esperados de Barlow serían
función del CI, el retraso mental leve se corresponde con los ataques predispuestos de Klein (opción 1 incorrecta);
un CI entre 50-55 y 70, el moderado se corresponde con un los ataques no señalados/esperados de Barlow no tendrían
CI entre 35-40 y 50-55, el grave se corresponde con un CI equivalencia a los de Klein; y los ataques no señalados/no
entre 20-25 y 35-40 y el profundo se corresponde con un esperados se corresponderían con los ataques inesperados
CI inferior a 20-25. La AAIDD prefiere el término discapa- de Klein (opción 2 incorrecta). La opción 4 (ataque de pá-
cidad intelectual frente al de retraso mental. Defienden un nico espontáneo) no se corresponde con ningún ataque for-
modelo funcional, dado que es el funcionamiento social de mulado por este autor (opción 4 incorrecta). APIR Manual
la persona y no su CI, lo que importa a la hora de realizar de Psicología Clínica, Tomo I, Tema 6.
una evaluación y un tratamiento del problema. Entiende el
retraso mental como una forma de ser de cierto tipo de per-
sonas que necesitan apoyos. APIR Manual de Psicología 180. De las diferentes f o b i a s e s p e c í f i c a s ,
Clínica Infantil, Tema 2. ¿cuál presenta una edad de aparición más tardía?:

55
1. Situacionales. Respuesta correcta:2
2. Entorno natural.
3. Animal. “Dada la eficacia de los antidepresivos actuales para el tra-
4. Sangre-inyección-heridas. tamiento de la Depresión, se recomienda la TEC sólo en
los siguientes casos: 1) Riesgos serios de suicidio (opción
Respuesta correcta:1 4 incorrecta); 2) Depresiones con importante inhibición o
agitación psicomotora, e ideas delirantes; 3) Grave com-
Según el DSM 5, la fobia específica generalmente se desa- promiso del estado general (desnutrición, deshidratación)
rrolla en la primera infancia, antes de los 10 años, entre los (opción 1 incorrecta) que requieren de una intervención
7 y los 11 (siendo la edad media de unos 10 años). Las si- rápida y eficaz; 4) Fracaso de los antidepresivos, al menos
tuacionales presentan una edad de inicio más tardía (opción dos, de distinta familia, por tiempo y dosis suficientes (op-
1 correcta) que las fobias específicas del entorno natural, ción 3 incorrecta); 5) Contradicciones para el uso de anti-
animal o a la sangre-inyección-herida (opciones 2, 3 y 4 depresivos por enfermedades (cardiovasculares, hepáticas,
incorrectas). Las que se inician en la infancia y la adoles- etc.), o porque durante el tratamiento aparezcan efectos
cencia suelen sufrir altibajos durante ese periodo. Las que colaterales muy intensos; y 6) Historia personal de buena
persisten en la edad adulta no suelen remitir. A pesar de respuesta a TEC y/o escasa respuesta a los antidepresivos
que se suelen desarrollar en la infancia y la adolescencia, en episodios previos. Castellón, D. S., López, O. F., Jimé-
pueden iniciarse a cualquier edad, normalmente tras expe- nez, J. R. C., Sánchez, I. R., Santana, L. C., Cruz, R. Á. B.
riencias traumáticas. APIR Manual de Psicología Clínica, & González, D. E. M. (2007). Manual para el tratamiento
Tomo I, Tema 6. electroconvulsivo. MediSur, 5(2), 179-186. “

183. Un terapeuta intenta abordar una depresión cró-


nica mediante la Terapia centrada en los esquemas
181. Respecto del tratamiento con antidepresivos para de Young (1990). Señale cuál de estos pasos NO corres-
la bulimia nerviosa, cabe afirmar que: ponde a esta terapia
1. Produce una mayor reducción de los atracones
que los tratamientos psicológicos, a largo plazo. 1. Explorar los orígenes de los modos del esquema
en la niñez y la adolescencia.
2. Muestra peores resultados que la Terapia cogniti-
vo conductual. 2. Usar la imaginería para tener acceso al modo del
esquema del niño vulnerable.
3. Los antagonistas opiáceos son más eficaces que
los antidepresivos. 3. Solicitar un registro diario de pensamientos auto
máticos disfuncionales asociados a los esquemas.
4. Los antidepresivos no han mostrado ser eficaces
para la reducción de atracones y purgas a corto plazo. 4. Propiciar y conducir diálogos entre los diferentes
modos de los esquemas.
Respuesta correcta:2
Respuesta correcta:3
Al contrario que en anorexia, los fármacos pueden jugar un
papel importante en el caso de la bulimia, siendo los más El proceso terapéutico en la terapia centrada en esquemas
destacados los antidepresivos. Diversos estudios confirman se articula en relación a dos fases principales que recogen
que se reducen los atracones y mejora el estado de ánimo, la integración de todo tipo de técnicas, desde técnicas cog-
aunque parece que los efectos antibulímicos serían inde- nitivas a conductuales, pasando por técnicas experienciales
pendientes de los efectos antidepresivos, ya que para que se y por el manejo de la relación paciente-terapeuta. En la pri-
den dichos efectos antibulímicos se requieren dosis más al- mera fase, el objetivo es identificar y activar sus esquemas
tas que para el tratamiento de la depresión (60 mg/día fren- desadaptativos tempranos. Se le explica al cliente el papel
te a 20 mg/día en el caso de la depresión). Si comparamos de los esquemas desadaptativos tempranos en el manteni-
el tratamiento farmacológico con el resto de tratamientos miento de los modos de esquema (opción 1 incorrecta por
para la bulimia, la TCC sigue siendo más eficaz aunque se ser correcta). Para ello, se utilizan varias estrategias: téc-
puede ver potenciada por el uso de antidepresivos (opción nicas experienciales para activar los esquemas como pue-
2 correcta). APIR Manual de Tratamientos Psicológicos, de ser la imaginería (opción 2 incorrecta por ser correcta),
Tema 11. discusión sobre los patrones de conducta que provocan los
esquemas (opción 4 incorrecta por ser correcta), etc. En la
fase de cambio de los esquemas desadaptativos tempranos
detectados anteriormente se utilizan técnicas cognitivas,
182. ¿Cuál de las siguientes características clínicas experienciales, conductuales...No obstante, no se solicita
de un paciente con depresión “endógen” (unipolar o bi- un registro diario de pensamientos automáticos disfuncio-
polar) NO constituye una indicación para iniciar t r a -
nales asociados a los esquemas; esto es más característico
tamiento con terapia electroconvulsiva?:
de la terapia cognitiva de Beck (opción 3 correcta). APIR
1. La presencia de grave desnutrición o deshidrata- Manual de Psicoterapias, Tema 9.
ción importantes.
2. Haber padecido tres recidivas. 184. La “exposición prolongada” propuesta por
3. Antecedentes de fracaso farmacológico en su tra- E. Foa y cols., se desarrolló fundamentalmente para el
tamiento. tratamiento:
4. Riesgo grave de suicidio.

56
1. Del trastorno de pánico. contenidos asociados). Es decir, ninguna de las opciones
2. De la agorafobia. de respuesta hace referencia al fenómeno requerido, lo que
3. Del trastorno de estrés postraumático. contribuyó a la anulación de la pregunta. APIR Manual de
Psicología Básica, Tema 3.
4. Del trastorno obsesivo compulsivo.

Respuesta correcta:3

La exposición prolongada en imaginación de los recuerdos


temidos es el tratamiento de elección y E1 para el TEPT
(opción 3 correcta); para las situaciones que son evitadas,
se utiliza la exposición en vivo. El grupo de Foa y cols.,
desde su modelo teórico, habla de que lo que funciona de
la exposición en imaginación es que posibilita el reprocesa-
miento emocional del trauma, más que la habituación que
se produce a la ansiedad. APIR Manual de Tratamientos
Psicológicos, Tema 7.

185. En el campo de la psicología de la memoria,


el paradigma de la práctica en la recuperación se ha
empleado para estudiar el fenómeno del “olvido indu-
cido por la recuperación”. Señale en qué consiste este
fenómeno:

1. Practicar la recuperación de cierta información a


partir de una determinada clave de recuperación puede ha-
cer que, posteriormente, esa información sea más fácil de
recuperar.
2. Practicar la recuperación de cierta información a
partir de una clave de recuperación puede hacer que, poste-
riormente, otros contenidos asociados a esa misma clave de
recuperación sean más fáciles de recuperar.
3. Practicar la recuperación de una información a
partir de una determinada clave de recuperación puede ha-
cer que, posteriormente, otros contenidos asociados a otras
claves de recuperación sean más difíciles de recordar.
4. Practicar la recuperación de información a partir
de una clave de recuperación puede hacer que, posterior-
mente, el estímulo empleado como clave de recuperación,
sea más difícil de recuperar si se pide su recuerdo.

Respuesta correcta:0

ANULADA. El fenómeno del olvido inducido por la recu-


peración hace referencia al hecho de que al intentar recupe-
rar cierta información a partir de una clave de recuperación
puede ocurrir que, posteriormente, otros contenidos asocia-
dos (semánticamente) a esa misma clave de recuperación
sean más difíciles de recuperar. En la opción de respuesta
1 se expone que la información será más fácil de recordar
gracias a la clave de recuperación, no haciendo referencia a
una mayor dificultad para recuperar contenidos asociados;
en la opción 2 se afirma que otros contenidos asociados
a la clave de recuperación serán más fáciles de recuperar,
proponiendo esta opción lo contrario a lo que ocurre en el
fenómeno (más difíciles); la opción 3 sostiene que otros
contenidos asociados a otras claves de recuperación pue-
den ser más difíciles, pero el fenómeno del olvido inducido
por la recuperación hace referencia a contenidos asociados
a la misma clave de recuperación; la opción 4 refiere una
mayor dificultad de recuperar el estímulo empleado (no

57
EXAMEN PIR 2019 POR ÁREAS

11
25. En la transmisión sináptica, para que se libere con
PSICOBIOLOGÍA éxito el neurotransmisor en el espacio sináptico es ne-
cesario:
24. Las proteínas responsables de producir y mantener
1. Que se transmita un potencial excitatorio post sináptico
estables las concentraciones de sodio y potasio a ambos
a lo largo del axón y llegue al botón terminal.
lados de la membrana y por tanto de asegurar la es-
2. Que se abran los canales de calcio controla dos por vol
tabilidad del potencial de reposo y permitir el futuro
taje, y entre calcio en el botón terminal.
potencial de acción son:
3. Que las vesículas que contienen el neurotrans misor se
fusionen con las zonas activas de la membra- na postsi
1. Los canales de sodio y potasio pasivos.
náptica.
2. Los canales de sodio y potasio controlados
por voltaje. 4. Que se activen los autorreceptores presinápti cos.
3. Las bombas de sodio-potasio.
4. Las proteínas G. Respuesta correcta:2

Respuesta correcta:3 La transmisión de un potencial excitatorio postsináptico


(PEP) permite la despolarización de la neurona, pero úni-
El potencial de membrana es la diferencia de carga eléctrica camente se produce el potencial de acción cuando la suma
que existe entre el interior y el exterior de la célula. Cuando de los PEPs y los PIPs (potenciales inhibidores postsinápti-
la neuro- na está en reposo, dicho potencial es de, aproxi- cos) supera el umbral de excitación (opción 1 incorrecta).
madamente, -70 mV (potencial de reposo), es decir que el La zona de liberación de la membrana presináptica contie-
potencial interior de la neurona es menor que el que hay ne canales de calcio dependientes de voltaje, que se abren
en el exterior de la membrana. Esta diferencia se debe a la cuando la membrana de la terminal ner- viosa se despo-
diferente concentración de iones (sa- les con carga positiva lariza con la ocurrencia de un potencial de acción, permi-
o negativa) en ambos ambientes (intrace- lular y extracelu- tiendo la entrada de calcio y la consecuente liberación de
lar). Los iones que contribuyen al potencial de reposo son: los neurotransmisores (opción 2 correcta). Las vesículas
sodio (Na+), potasio (K+), cloro (Cl-) e iones proteí- nicos liberan por medio de exocitosis el neurotransmisor al es-
de carga negativa. Los iones de Na+ y de Cl- presentan una pacio sinápti- co (opción 3 incorrecta). Los autorreceptores
mayor concentración en el exterior de la célula, mientras presinápticos se suelen encargar de controlar la cantidad de
que los de K+ se concentran más en el interior. La mayoría neurotransmisores presentes en la sinapsis (opción 4 inco-
de los iones proteínicos permanecen en el interior, donde rrecta). APIR Manual de Biopsicología, Tema 1.
son formados. Esta distribución desigual de iones se debe
a tres fenómenos: (a) el gradientes de concentración, o des- 34. Entre los siguientes posibles efectos secundarios de
plazamiento de iones desde zonas de alta concentración la clozapina, ¿cuál es el más grave?:
hacia zonas de menor concentra- ción; (b) la presión elec-
trostática, que es la atracción de cargas opuestas y repul- 1. Agranulocitosis.
sión de cargas iguales y (c) la permeabilidad selectiva de la 2. Sedación.
membrana celular, que permite el paso de deter- minados 3. Aumento de peso.
iones con más facilidad que otros. Estos mecanismos son 4. Taquicardia.
pasivos, por lo que no requieren gasto de energía (opción
1 incorrecta). Las bombas de sodio-potasio son mecanis- Respuesta correcta:1
mos acti- vos que tienen como función expulsar iones de
Na+ al exterior al mismo tiempo que introducen K+ y son La Clozapina es un tipo de antipsicótico atípico. La agranu-
las encargadas de mantener estables las concentraciones loci- tosis es una posible alteración en la síntesis de glóbu-
de ambos iones (opción 3 correcta). Los canales de sodio los blancos de la sangre que aparece en un 2% de pacientes
y potasio controlados por voltaje no intervienen en el man- que hacen uso de Clozapina y que puede ser mortal si no es
tenimiento de las concentraciones de io- nes sino en los detectada a tiempo, por lo que se considera el efecto secun-
procesos de despolarización e hiperpolarización (opción 2 dario más grave que pue- de provocar este fármaco (opción
incorrecta).El potencial de acción se producirá cuando la 1 correcta). Los antipsicóticos atípicos se presentan como
suma de los potenciales excitadores postsinápticos (PEPs) y alternativa a los clásicos y tienen la ventaja de producir me-
los potenciales inhibidores postsinápticos (PIPs) alcanza el nos efectos secundarios. Además de la agranulocitosis que
um- bral de excitación. Las proteínas G generan una serie puede aparecer con el uso de Clozapina, hay otros posibles
de reaccio- nes químicas hasta conseguir que un segundo efectos adversos por el uso de antipsicóticos atípi- cos, como
mensajero pueda provocar la apertura del canal iónico, ade- aumento de peso, taquicardia, hipotensión, xialorrea o ex-
más de otros posibles efectos (opción 4 incorrecta). APIR ceso de salivación o sedación, sin embargo, se consideran
Manual de Biopsicología, Tema 1. efectos secundarios de menor gravedad en comparación

2
con los riesgos que conlleva la agranulocitosis (opciones 2, Oxcarbazepina y Carbamacepi- na (que fue el primer anti-
3 y 4 inco- rrectas). APIR Manual de Psicobiología, Tema 3. convulsivante que mostró su posible utilidad para el Tras-
torno Bipolar). El Litio tiene un estrecho margen terapéuti-
co por lo que es necesario realizar litemias pe- riódicamente
39. ¿Qué clase de fármaco es la quetiapina?: por su riesgo de toxicidad. Los valores de dosis en sangre
adecuados oscilan entre 0,6-0,8 mili/equivalentes por litro.
1. Una benzodiacepina. La litemia se realiza aproximadamente 12 horas después de
2. Un ansiolítico. la última toma del medicamento (opción 2 incorrecta). El
3. Un antidepresivo. Litio es eficaz para el tratamiento agudo de episodios ma-
4. Un antipsicótico. nía- cos del Trastorno Bipolar, sin embargo, una alternativa
eficaz de tratamiento cuando se observa que la persona no
Respuesta correcta:4 responde al Litio es la administración de ácido Valpróico
(Depakine) (op- ción 4 correcta). El tratamiento con antip-
La Quetiapina es un tipo de antipsicótico atípico (opción 4 sicóticos atípicos en fase maníaca se asocia con una reduc-
co- rrecta) (opciones 1, 2 y 3 incorrectas). Los antipsicóti- ción del riesgo de viraje
cos atípicos son fármacos antagonistas tanto de la Dopami- a depresión en comparación con el tratamiento con antipsi-
na (DA) como de la Serotonina (5-HT). Se presentan como cóti- cos típicos como el Haloperidol (opción 1 incorrecta).
alternativa a los antip- sicóticos clásicos y tienen la ventaja Además, el manejo inicial de las alteraciones de conducta y
de producir menos efectos secundarios que estos, además la agitación puede requerir añadir benzodiacepinas a corto
siguen siendo eficaces para los síntomas positivos. No pro- plazo (opción 3 incorrecta). APIR Manual de Psicobiología,
vocan síntomas extrapiramidales ni alteraciones relaciona- Tema 4.
das con la prolactina y además provocan cierta mejoría al
actuar sobre la activación de la vía mesocorti- cal. Entre los 19. La principal vía eferente de la formación hipocam-
posibles efectos adversos por el uso de antipsicóti- cos atí- pal es:
picos tenemos aumento de peso, taquicardia, hipotensión,
xialorrea o exceso de salivación y sedación. Algunos fár- 1. El núcleo supraquiasmático.
macos antipsicóticos atípicos además de la Quetiapina son 2. La circunvolución dentada.
la Clozapina (cuyo posible efecto secundario más grave es 3. El fórnix.
la agranulocitosis), Olanzapina, Risperidona, Ziprasidona, 4. El campo CA3.
Sulpirida, etc. Algunas de las indicaciones de estos fárma-
cos son el tratamiento de Tras- tornos Psicóticos, Trastor- Respuesta correcta:3
nos de conducta y en algunos casos en el Trastorno Bipolar.
APIR Manual de Psicobiología, Tema 3. El núcleo supraquiasmático es el principal regulador de los
ritmos circadianos. Está formado por neuronas de peque-
28. En relación al tratamiento de la fase maníaca en el ño ta- maño, axones cortos y campos dendríticos peque-
trastorno bipolar, ¿cuál de las siguientes afirmaciones ños con una frecuencia espontánea de descarga muy baja,
es co- rrecta?: la cual aumenta proporcionalmente según la intensidad
de la luz que llega a la retina. La información de la retina
1. En caso de ser necesario prescribir un antipsicótico, se proyectada a este núcleo pro- cede de las células ganglio-
preferirá el uso de antipsicóticos atípicos ya que estos ayu- nares con melanopsina y es transpor- tada a través de una
darán a producir un viraje hacia la fase depresiva. vía directa o retino-hipotalámica, mediada por la liberación
2. Se deberá esperar un máximo de 48 horas para determi- de glutamato, y una vía indirecta o genículo-hi- potalámi-
nar los cambios en las concentraciones séricas de litio tras ca mediada por el neuropéptido Y (opción1 incorrec- ta).
una modificación de dosis. La circunvolución dentada y el campo CA3 forman parte
3. La utilización de cualquier benzodiacepina está contrain- de la formación hipocampal y tienen un papel importante
dicada en esta fase del trastorno. en el aprendizaje a largo plazo. Las principales aferencias y
4. El tratamiento con ácido valproico se conside- ra una eferencias neocorticales de la formación hipocampal pasan
alternativa eficaz en pacientes que no respondan al trata- a través de la corteza entorrinal, cuyas neuronas proyectan
miento con litio. a las células granu- losas de la circunvolución dentada, que,
a su vez, conectan con el campo CA3. Las células de CA3
Respuesta correcta:4 tienen receptores NMDA y No NMDA. El NMDA es un
receptor de glutamato de especial importancia en el fenó-
Los estabilizadores del ánimo o eutimizantes son fármacos meno de la Potenciación a Largo Plazo. La circunvolución
espe- cialmente indicados para el tratamiento de los Tras- dentada y el campo CA3 son estructuras que componen la
tornos Afec- tivos, especialmente para el Trastorno Bipo- propia formación hipocampal por lo que no son vías efe-
lar. Los eutimizantes más relevantes son el Litio (Plenur), rentes (opción 2 y 4 incorrectas). El fórnix es una co- mi-
el Ácido Valproico (Depa- kine) y otros como Topiramato, sura que en su máximo grosor lleva un millón de fibras y es

3
la principal vía eferente del hipocampo. Es continuación de mérico, su función más destacable está relacionada con el
las eferencias de CA3, CA1 y subículo y por medio de un reconocimiento de rostros conocidos, su lesión da lugar a
viaje cir- cular, subirá sobre el tálamo y llegará a los cuerpos pro- sopagnosia o déficit en el reconocimiento de rostros
mamilares del hipotálamo. (opción 3 correcta). APIR Ma- familiares (opción 2 incorrecta). La circunvolución angular,
nual de Psicobio- logía, Tema 6. especialmente del hemisferio izquierdo, está involucrada en
aquellas tareas de procesamiento numérico y cálculo que
23. Durante el desarrollo embrionario, la ausencia de requieren un procesa- miento verbal. Al contrario que el
hormona inhibidora del sistema de Müller se relaciona surco intraparietal, su acti- vación es mayor en tareas de
con: cálculo exacto y concretamente en los denominados “he-
chos aritméticos”. Los “hechos aritméti- cos” son operacio-
1. El síndrome de insensibilidad a los andróge nos. nes matemáticas simples y automatizadas que se almacenan
2. La alteración del gen SRY (sex-determining region Y). en la memoria verbal. Un ejemplo podrían ser las tablas de
3. La inhibición en la síntesis de dihidrotestoste rona multiplicar (“2x2=4”) o adiciones de pequeñas canti- da-
4. No se produce desfeminización. des (“5+10=15”) (opción 3 correcta). El surco intraparietal,
que no sintraparietal lo cual es una errata, proporciona la
Respuesta correcta:4 repre- sentación interna de las cantidades y la relación exis-
tente entre estas. Esto incluye el procesamiento abstracto de
El síndrome de insensibilidad a los andrógenos o síndro- magnitudes, ya sean estas o no numéricas, contribuyendo a
me de Morris consiste en la inexistencia de receptores para la cuantificación y a la estimación de magnitudes. Esta re-
los andró- genos, que impide el desarrollo de los órganos gión está involucrada en tareas de cálculo aproximado y en
sexuales inter- nos masculinos, la hormona antimülleriana tareas de cálculo no mediadas
sí provoca su efecto desfeminizante, por lo que tampoco se por el lenguaje (opción 4 incorrecta). APIR Manual de Psi-
desarrollan los órganos internos femeninos (opción 1 in- co- biología, Tema 14.
correcta). El gen SRY produce el factor determinante de
los testículos (una enzima), que provoca la maduración de 21. ¿Qué proteína es necesaria para el establecimien-
las gónadas indiferenciadas hacia testículos. Si el gen SRY to de la potenciación a largo plazo de larga duración
no está presente, se convertirán en ovarios (opción 2 inco- (PLP- LD)?:
rrecta). El déficit en dihidrotestosterona es el causante del
desarrollo del pene retrasado, una condición que impide a 1.El óxido nítrico liberado por la neurona presináptica.
los varones desarrollarse hasta la pubertad, permaneciendo 2.La entrada de iones calcio y activación subsiguiente de
durante la infancia el pene hipodesarrollado y los testículos enzimas como la CaM-KII.
en el interior, dando apariencia de genitales femeninos (op- 3. La activación de los receptores de NMDA ac tivados por
ción 3 incorrecta). Por último, sí es cierto que la ausencia ligando y por voltaje.
de hormona inhibitoria del sistema de Müller implicaría la 4. La enzima PKM-zeta, al facilitar el desplaza miento de los
no desfeminización de los varones XY que presentan esta receptores AMPA a la membrana termi nal.
característica, disponiendo estos de órganos sexuales inter-
nos de ambos sexos (opción 4 correc- ta). APIR Manual de Respuesta correcta:4
Biopsicología, Tema 11.
El proceso de Potenciación a Largo Plazo (PLP) se divide
18. Los “hechos aritméticos” (operaciones automatiza- en dos fases. La PLP inicial (PLP-I) consiste en el siguien-
das que se almacenan en la memoria verbal) son recu- te pro- ceso: primero se produce una despolarización de la
pera- dos de la memoria por: membrana que si es suficientemente fuerte activará los re-
ceptores NMDA permitiendo la entrada de iones de calcio
1. La corteza prefrontal dorsolateral. 2. La circunvolución (opción 3 incorrecta). Esto produce la activación de enzi-
fusiforme. mas como la CaM-KII (calcio calmodulina cinasa tipo II)
3. La circunvolución angular. que influye en el desplazamiento de los receptores de AMPA
4. El surco sintraparietal. a la membrana postsináptica (opción 2 incorrecta). El óxido
nítrico actúa como un mensajero retrógrado a la membrana
Respuesta correcta:3 presináptica que ayudar a la PLP-I (opción 1 in- correcta).
En la segunda fase, o PLP de larga duración (PLP-LD) es ne-
La corteza prefrontal lateral y ventral participa en el pro- cesario la activación de la enzima PKM-zeta. La PKM-zeta
cesa- miento del cálculo proporcionando el mantenimien- activa la enzima NSF (proteína reguladora del tráfico) que
to provisio- nal de los resultados intermedios (memoria causa el traslado de receptores AMPA lateralmente desde el
de trabajo), la plani- ficación y ordenación temporal de tallo den- drítico a la membrana postsináptica de la espina
los componentes de las tareas, la comprobación de resul- dendrítica. Los receptores AMPA controlan los canales de
tados y la corrección de errores (opción 1 incorrecta). El sodio; así, cuando es- tos son activados por el glutamato,
giro fusiforme no toma parte en el procesamien- to nu- producen PEPS en la mem- brana de la espina dendrítica.

4
Con mayor cantidad de receptores AMPA, la liberación de 27. El área preóptica ventrolateral (APOvl):
glutamato por las terminales nerviosas causa un potencial
excitador postsináptico mayor. Así, las sinap- sis se hacen 1. Es una región promotora del sueño de ondas lentas
más fuertes (opción 4 correcta). APIR Manual de Psicobio- 2. Facilita el aprendizaje relacional durante el sueño REM.
logía, Tema 14. 3. Se estimula por las neuronas orexinérgicas del hipotála-
mo lateral.
26. El sueño REM: 4. Inhibe la secreción de GABA en el encéfalo.

1. Está producido por la estimulación colinérgi ca de la for- Respuesta correcta:1


mación reticular pontina.
2. Está producido por la estimulación de las neu ronas sero- El Área Preóptica Ventrolateral (APVL), situada en el hipo-
toninérgicas del núcleo del rafe. tá- lamo anterior, tiene un papel importante en la base neu-
3. Está asociado a una disminución de la tempe ratura cor- ral del sueño No REM o sueño de ondas lentas (opción 1
poral y del consumo de energía. correcta). En ella existen neuronas gabaérgicas que proyec-
4. Presenta un EEG lento y de alto voltaje. tan hacia el núcleo tuberomamilar, la protuberancia dorsal,
los núcleos del rafe y el locus coeruleus, inhibiéndolos (op-
Respuesta correcta:1 ción 4 incorrecta). Además, recibe aferencias inhibitorias
de las mismas regiones que inhi- be, conformándose así un
El sueño REM/MOR o sueño paradójico surge tras 90 mi- circuito de inhibición recíproca lla- mado “flip-flop” que
nutos de sueño y se produce un cambio brusco en varias permite alternar sueño y vigilia y que se estabiliza mediante
medidas fi- siológicas que se registran. Algunas de las ca- un conjunto de neuronas hipocretinérgicas del hipotálamo
racterísticas de esta fase del sueño son: EEG desincroniza- lateral. Las hipocretinas/orexinas son neuropép- tidos sin-
do, ondas Theta acompa- ñadas de movimientos oculares tetizados por un pequeño grupo neuronal localizado en el
verticales, ondas PGO, atonía muscular con EMG aplanado hipotálamo posterolateral. El sistema hipocretinérgico/ore-
por la pérdida de tono muscular y aumento de actividad xi- nérgico muestra una gran actividad durante la vigilia,
Beta. El sueño no REM se caracteriza por la predominan- además la deficiencia parcial o total de estos péptidos o sus
cia de ondas cerebrales lentas, que indican una ac- tividad receptores se asocia a la narcolepsia. Estas neuronas activan
eléctrica disminuida en el SNC (opción 4 incorrecta). En directamente la corteza cerebral, activan grupos neuronales
cuanto a la base neural del sueño REM es importante la que forman parte del Sistema Reticular Ascendente de ac-
función de las neuronas colinérgicas del Área Peribraquial tivación e inhiben la genera- ción de SREM en el tegmento
del tronco en- cefálico. Esta área se prolonga en un núcleo, Pontino Ventral (opción 3 inco- rrecta). La adenosina actúa
que es la Forma- ción Reticular Pontina (FRPM) que par- en el Prosencéfalo Basal como neu- rotransmisor inhibito-
ticipa también en el des- encadenamiento del sueño REM. rio y también conduce al sueño. Mientras que el SNREM
La FRPM envía proyecciones a las neuronas colinérgicas interviene en el adecuado descanso, el SREM interviene en
del prosencéfalo basal para activarlas y generar así la acti- el desarrollo cerebral y consolidación del aprendi- zaje (op-
vación de la corteza. Después el Área Peri- braquial excita ción 2 incorrecta). APIR Manual de Psicobiología, Tema 15.
la vía PGO (protuberancia geniculada occipital) para oca-
sionar ondas PGO y movimientos oculares y la FRPM pro- 20. Resulta esencial para detectar el disgusto en los
voca la atonía propia de esta fase (opción 1 correcta). Por otros y experimentar el propio disgusto:
otra parte, es la inhibición del Locus Coeruleus y el Rafe
la que induce al sueño REM (opción 2 incorrecta). En el 1. La circunvolución fusiforme.
sueño también se producen algunos cambios fisiológicos. 2. La ínsula anterior.
Con respecto al SNA, en SNREM la temperatura corporal 3. La unión temporoparietal.
desciende, mientras que en el SREM la temperatura corpo- 4. El surco temporal superior.
ral se va ajustando a la tempera- tura ambiente. Las tem-
peraturas ambientales extremas provocan interrupción del Respuesta correcta:2
sueño, con frecuentes despertares, reduciéndose la cantidad
de sueño REM. (opción 3 incorrecta). La respiración en la El giro fusiforme o circunvolución fusiforme interviene en
transición vigilia-sueño suele ser irregular, estabilizándose el reconocimiento de rostros conocidos, su lesión da lugar
a medida que el SNREM se hace más profundo, mientras a pro- sopagnosia o déficit en el reconocimiento de rostros
que durante el SREM la frecuencia respiratoria aumenta y familiares, pero no tiene función específica sobre el recono-
la respi- ración vuelve a ser irregular. La frecuencia cardíaca cimiento de las emociones (opción 1 incorrecta). Tanto la
en SNREM disminuye y en el SREM se hace más irregular. ínsula como los ganglios basales son estructuras importan-
APIR Manual de Psicobiología, Tema 15. tes para reconocer la emoción de asco, desagrado o repug-
nancia y experimentar el propio disgusto (opción 2 correc-
ta). La unión temporoparietal es una zona de asociación
encargada de la integración de diferentes tipos de informa-

5
ción tanto externa como interna. Interviene en funciones
relacionadas con la autoconciencia, la Teoría de la Mente y Erik Erikson propone en su teoría psicosocial, desde un en-
experiencias extracorporales (opción 3 incorrecta). El surco foque de estadios, ocho estadios del desarrollo del yo, que
temporal superior interviene en el procesamiento de estí- se desarrollan desde la infancia hasta la vejez. Cada etapa
mulos sociales y está implicada en la percepción social. El incluye una crisis (aspecto primordial del desarrollo) que
área de Wernicke se encuentra en esta zona y se encarga debe ser re- suelta por la persona para un desarrollo sano
en el procesamiento del habla (opción 4 incorrecta). APIR del yo. Además plantea que es necesario que cada etapa se
Manual de Psicobiología, Tema 16. resuelva con éxito para poder avanzar hacia la siguiente de
forma adecuada. Las di- ferentes crisis o desafíos que pro-
22. La extinción de las respuestas emocionales condicio- pone Erikson se asocian a unos rangos de edad aproxima-
nadas se relaciona, fundamentalmente, con la actividad dos en los que resolverlos. El desafío de intimidad vs. ais-
de: lamiento se sitúa en la edad comprendida en- tre los 20-40
años (opción 1 incorrecta). La crisis de integridad del yo vs.
1. La corteza prefrontal ventromedial. desesperación se produce en la última etapa de la vida, a
2. La ínsula. partir de los 65 años. La crisis de generatividad vs. estanca-
3. La corteza cingulada posterior. miento se asocia a la mediana edad, entre los 40-65 años
4. El núcleo accumbens. (opción 3 correcta). La crisis o desafío de productividad
no es una de las crisis propuestas por Erikson en su teoría.
Respuesta correcta:1 APIR Manual de Desarrollo Psicológico, Tema 2.

La corteza prefrontal ventromedial es clave en la memoria 2. Ante dos filas con el mismo número de monedas cada
de ex- tinción en las respuestas de miedo a largo plazo, inhi- una, un niño cree que la fila más larga tiene más mone-
be la expre- sión del miedo (aunque determinadas regiones das que la fila más corta. ¿En qué etapa de Piaget se
de ésta también la favorecen), participa en el condiciona- encuen- tra este niño?:
miento instrumental, planificación y atención (opción 1 co-
rrecta). La ínsula contiene representaciones interoceptivas 1.Operaciones formales.
a modo de representación del es- tado del cuerpo, para lo 2.Operaciones concretas.
que se encuentra bien relacionada con estructuras como la 3.Preoperacional o intuitiva.
amígdala y el estriado. La ínsula ha sido relacionada con 4.Sensoriomotora.
la conciencia interoceptiva y emocional, la em- patía y la
conducta social cooperativa, así como también ejerce un Respuesta correcta:3
papel en la percepción auditiva, gustativa y somatosenso-
rial (opción 2 incorrecta). La corteza cingulada es el lugar Dentro de las diferentes perspectivas teóricas en psi- colo-
donde se procesa en última instancia la información sobre gía del desarrollo encontramos modelos cognitivos, siendo
el dolor (op- ción 3 incorrecta). Por último, el núcleo ac- Piaget uno de sus autores principales. Piaget describe el de-
cumbens forma parte de los ganglios basales y es respon- sarro- llo humano en torno a los conceptos de funciones
sable de la función reforzante de determinados estímulos y estructuras cognitivas. Habla de tres componentes en su
sobre la conducta y también de la función estimulante de teoría (componen- te estructural, funcional y secuencial).
las drogas (opción 4 incorrecta). APIR Manual de Biopsi- Dentro del componente estructural se conceptualizan los
cología, Tema 16. tres estadios generales de la teoría de Piaget: período sen-
soriomotor, período de preparación y organización de las
operaciones concretas y período de opera- ciones formales.
El primer estadio se caracteriza por el logro de la intencio-
DESARROLLO PSICOLÓGICO nalidad y la permanencia del objeto (opción 4 inco- rrecta).
El estadio de preparación y organización de operaciones
concretas se divide en el subperiodo preoperatorio (mar-
cado por la aparición de la función simbólica que se mani-
3. De acuerdo con la propuesta de Erik Erikson, en la
fiesta en el jue- go, imitación, dibujo y lenguaje). Una de las
mediana edad la crisis o desafío que las personas han de
características del pensamiento preoperatorio es la centra-
abordar se denomina:
ción, que se caracteriza por atender de manera selectiva a
un aspecto siendo incapaz de coordinar varias perspectivas
1. Intimidad.
o compensar varias dimensiones de un objeto. Esto se ob-
2. Productividad.
serva en las tareas de conservación como la descrita en la
3. Integridad.
pregunta, en la que ante una tarea de conserva- ción del nú-
4. Generatividad.
mero, el niño se centra únicamente en una dimensión, que
es la longitud de la fila, para dar su respuesta (opción 3 co-
Respuesta correcta:4
rrecta). El subperiodo operacional está caracterizado por la

6
capa- cidad de operar mentalmente con el conocimiento, Respuesta correcta:1
poniéndose de manifiesto en la resolución de problemas de
conservación, seriación y clasificación) (opción 2 incorrec- Kohlberg elaboró una teoría cognitiva sobre el desarro- llo
ta). El período de operaciones formales se caracteriza por moral sustentada en el modelo de Piaget, basada también
alcanzar un pensamiento abstracto y lógico mediante la for- en el uso de dilemas morales que el niño ha de resolver. La
mulación de hipótesis (opción 1 incorrecta). APIR Manual respues- ta a estos dilemas pone en evidencia el nivel de
de Desarrollo Psicológico, Tema 4. razonamiento del niño. Este modelo propone tres niveles
de desarrollo moral (preconvencional, convencional y post-
75. Según el modelo A-B-C en terapia cognitiva aplica- convencional) y divide cada uno de ellos en dos estadios.
da a niños y adolescentes y teniendo en cuenta los esta- En el nivel preconvencional el énfasis está en el control ex-
dios evolutivos postulados por Piaget, señala la opción terno, las normas se cumplen para evitar castigos o para ob-
correcta: tener recompensas (opción 1 correcta). Uno de los estadios
pertenecientes al nivel preconvencional es la orientación
1. La terapia cognitiva no es aplicable a niños en el esta- dio hedonista instrumental e intercambio en la que los niños
de operaciones concretas. se conforman con las reglas por su propio interés, sin em-
2. La terapia cognitiva no es aplicable a niños en el esta- dio bargo no es uno de los tres niveles de desarrollo moral pro-
de operaciones formales. pues- tos por el autor (opción 2 incorrecta). El nivel formal
3. La terapia cognitiva no es aplicable a niños en el esta- dio e intuitivo no son niveles de desarrollo moral propuestos
preoperativo. por este modelo (opciones 3 y 4 incorrectas). APIR Manual
4. La terapia cognitiva no es aplicable en los estadios preope- de Desarrollo Psico- lógico, Tema 6.
rativos y de operaciones concretas, pero sí en el estadio de
operaciones formales.
4. Una técnica de instrucción mediante la que alguien
Respuesta correcta:3 más experto proporciona un apoyo temporal al niño en
aquello que está cercano a ser comprendido o en habi-
“Para Piaget, el niño preoperatorio es incapaz de com- lidades próximas a ser alcanzadas por el niño, se deno-
prender las transformaciones perceptivas de los objetos, minan:
construir series ordenadas de elementos, o clasificar. Es-
tas limitaciones del pen- samiento preoperatorio también 1. Aprendizaje vicario.
condicionan el tipo de psicote- rapia aplicable con niños, 2. Aprendizaje proximal.
teniendo que ajustar las intervenciones 3. Zona de desarrollo próximo.
a la edad del mismo. Algunos autores consideran que de- 4. Andamiaje.
bido a estas limitaciones, ciertos modelos de terapia como
el A-B-C de terapia cognitiva de Ellis no son aplicables du- Respuesta correcta:4
rante el periodo preoperatorio (opción 3 correcta). Estas
intervenciones requie- ren utilizar la lógica, clasificar los El andamiaje es un concepto destacado dentro de la teo-
pensamientos y atender simul- táneamente a varios aspec- ría de la instrucción de Bruner y hace referencia al apoyo
tos de la realidad (como antecedentes, creencias y conse- eficaz que el adulto facilita al niño y que se va adecuando y
cuencias) por lo que no serían aplicables antes del periodo cambiando en la medida en que el niño adquiere más res-
de las operaciones concretas (opción 1, 2 y 4 inco- rrectas). ponsabilidad en la actividad y que tiene, por tanto, carácter
Esta visión de las limitaciones de estadio preoperatorio no transitorio, traspasando progresivamente el control al niño
es unánime, ya que los trabajos de otros autores desde la facilitando así su responsabi- lidad a medida que se progre-
pers- pectiva del procesamiento de la información y desde sa en la tarea (opción 4 correcta). Este concepto es similar al
la teoría de Vigotsky han concluido que en este periodo el de Zona de Desarrollo próximo de Vygotsky, quien también
niño posee una competencia mayor de la que suponía Pia- habla de que los adultos deben ayudar en el aprendizaje del
get. APIR Manual de Desarrollo Psicológico, Tema 4. niño para que luego él pueda internalizar- lo. El término
que recoge esta idea es el de Zona de desarrollo próximo o
1. Un niño, ante la pregunta de por qué no se debe pe- proximal (ZDP) que es la distancia entre el nivel real de de-
gar a un compañero, contesta: “porque si la profesora sarrollo del niño (capacidad para resolver un problema por
te ve, te castiga”. De acuerdo con Kohlberg, ¿en qué ni- sí mismo) y el nivel de desarrollo potencial (capacidad para
vel de desa- rrollo moral se encuentra?: re- solver un problema bajo la guía de un adulto), es decir,
la dife- rencia entre lo que el niño puede hacer solo y lo que
1. Nivel preconvencional. puede hacer con ayuda (opción 2 y 3 incorrectas). El apren-
2. Nivel instrumental. dizaje vicario es un concepto similar al modelado. La dife-
3. Nivel formal. rencia entre ambos tie- ne que ver con el momento del pro-
4. Nivel intuitivo. ceso de aprendizaje en que se pone mayor énfasis, así en el
modelado se pone mayor atención en el aprendizaje de una

7
conducta y cómo dicho aprendizaje se ve afectado por las grupo.
características del modelo y el observador, sin embargo en 3. Consisten en realizar descripciones de lo observado sin
el aprendizaje vicario se pone más énfasis en aspectos mo- existir una estructuración previa que dirija la recogida de
tivacionales relacionados con la percepción de re- fuerzos datos.
para la conducta que se está aprendiendo (opción 1 inco- 4. Se basan en técnicas escalares y sirven para clasificar las
rrecta). APIR Manual de Desarrollo Psicológico, Tema 9. actividades de un sujeto según dimensiones previamente
es- tablecidas.

Respuesta correcta:3
EVALUACIÓN PSICOLÓGICA Las técnicas de observación consisten en una serie de ins-
tru- mentos que empleamos, como su propio nombre in-
dica, para la observación. En relación a la clasificación de
92. Los test psicométricos se pueden clasificar en test de estas técnicas, solemos organizarlas en un continuo en base
rendimiento máximo y test de rendimiento típico según al nivel de estruc- turación . Ordenadas de menor a mayor
la demanda que se le hace al sujeto. ¿En qué consiste un estructuración encon- tramos: 1) los registros narrativos, 2)
test de rendimiento típico?: las escalas de apreciación, 3) los protocolos observaciona-
les de conducta, 4) los códigos o sistemas de categorías, 5)
1. El sujeto tiene que escoger la única respuesta verdadera los registros de productos de conduc- ta y 6) dispositivos
entre una serie de alternativas. electrónicos. En concreto en los registros narrativos, el ob-
2. Se trata de valorar el rendimiento más alto de una per- servador va tomando nota de lo que está suce- diendo en la
sona en un constructo determinado como, por ejemplo, la situación de observación, por lo tanto, no hay una planifica-
inteli- gencia. ción previa de la recogida de datos (Opción 3 correcta). En
3. El objetivo principal es valorar la cantidad de ítems que el empleo de esta técnica destacan dos fuentes de error que
resuelve la persona en un tiempo limitado. afectan a su fiabilidad. Por un lado, los observadores pue-
4. El objetivo es valorar la manera habitual de comportar- se den uti- lizar distintas descripciones verbales para una mis-
de un individuo. ma conducta o patrón de conductas, y por otro, se puede
llegar a categorizar o a dar distinta significación a los even-
Respuesta correcta:4 tos. Esto último puede controlarse previo entrenamiento de
los observadores en el uso de un mismo lenguaje descrip-
Existen múltiples formas de clasificar las distintas técnicas tivo. Por estos motivos estas téc- nicas se consideran útiles
psi- cométricas. A este respecto, Cronbach distingue entre: como trabajo previo a una recogida de datos más precisa o
1) Téc- nicas de rendimiento máximo, objetivas o de ejecu- cuando la conducta a observar es de baja frecuencia, pero
ción, en las cuales el sujeto evaluado debe rendir al máxi- indudablemente no son la técnica de registro observacional
mo de sus posibili- dades, para esto se suele trabajar con que permite un mayor rigor científico (Opción 1 incorrec-
pruebas de tiempo limitado (Opción 3 incorrecta). Dentro ta), siendo estas aquellas que presentan mayor estructu- ra-
de esta clasificación se encuentran los test de inteligencia ción. Respecto al resto de alternativas de respuesta, la nú-
(Opción 2 incorrecta), los test de aptitud general, los test mero 2 hace referencia a los mapas de conducta o formatos
de aptitudes específicas y los test de rendimien- to acadé- de cam- po, que son un tipo de protocolos observacionales
mico. Y 2) Técnicas de rendimiento típico, subjetivas o de de conducta (Opción 2 incorrecta) y la número 4 hace re-
respuesta, las cuales pretenden conocer el comportamiento ferencia a las escalas de apreciación (Opción 4 incorrecta).
cotidiano del sujeto en una determinada situación o en va- APIR Manual Evaluación Psicológica, Tema 3.
rias circunstancias diferentes (Opción 4 correcta). Implican
la esta- bilidad de la conducta a lo largo del tiempo. Entre 89. Cuando estamos evaluando la conducta de un sujeto
estas se inclu- yen las técnicas de evaluación de la persona- a través de la observación, ¿qué se recomienda para re-
lidad o de variables emocionales, Inventarios autodescripti- ducir el sesgo de reactividad?:
vos, Test situacionales y Técnicas Proyectivas. APIR Manual
de Evaluación Psicológica, Tema 2. 1. Maximizar la interacción entre sujeto y observador para
que el sujeto se sienta más cómodo cuando es observado.
88. Respecto a las técnicas de registro de la conducta de 2. Utilizar un amplio periodo de observación para que el
un sujeto al que estamos observando, ¿qué característi- sujeto se habitúe a la situación de observación.
cas tiene el registro narrativo?: 3. Utilizar un único observador para que la recogida de da-
tos sea más homogénea.
1. Es la técnica de registro observacional que permite un 4. No utilizar dispositivos ocultos o a distancia que po-
mayor rigor científico respecto a las demás técnicas. drían disminuir la validez ecológica de la observación.
2. Está dirigido a evaluar las interacciones que se produ-
cen entre el ambiente y la conducta, o entre un dividuo y un Respuesta correcta:2

8
La observación es la técnica de investigación básica, que su- Dentro del muestreo de intervalo nos encontramos con los
pone la conducta voluntaria de recogida de datos con el fin siguientes tipos: 1) Muestreo de intervalo total o comple-
de for- mular y posteriormente comprobar hipótesis. Esta to: En este tipo de muestreo para registrar la conducta ésta
técnica, del mismo modo que cualquier otra, no está 100% debe estar presente durante todo el intervalo y resulta útil
libre de fallos, y podemos encontrar ciertos tipos de ses- en conductas que queremos que se den durante largo tiem-
go propios de las técnicas de observación. Estos sesgos se po, (p. ej., la aten- ción de un niño en clase) (Opción 3 co-
pueden clasificar en tres grupos según la fuente de error, rrecta). 2) Muestreo de intervalo parcial: En este caso, para
encontrándonos con sesgos procedentes registrar la conducta ésta debe aparecer en algún momento
del sujeto observado, sesgos procedentes del observador y del intervalo y resulta útil para conductas breves de alta fre-
ses- gos procedentes de la modalidad de observación. En cuencia (p. ej., tics o decir tacos) (Opción 1 incorrecta). Y
concreto la reactividad es un sesgo propio del sujeto obser- 3) Muestras de intervalo momentáneas: En este subgrupo,
vado que consis- te en la modificación de la propia con- para registrar la conducta ésta debe aparecer en un momen-
ducta como consecuencia de saberse observado. Es directa- to predeterminado del intervalo de observación, al inicio
mente proporcional al grado de modificación ambiental de o al final del mismo (Opción 2 incorrecta). APIR Manual
la observación y puede reducirse, entre otras técnicas, mini- Evaluación Psicológica, Tema 3.
mizando la interacción entre sujeto y observador para que
el sujeto se sienta más cómodo cuando es observado (Op- 105. ¿Qué nivel de muestreo intrasesional estamos uti-
ción 1 incorrecta), esto se puede conseguir con dispositivos li- zando si en una sesión de clase se muestrea cada 30
ocultos o a distancia (Opción 4 incorrecta). Otra forma de segundos a un alumno para ver si se mantiene en la po-
reducir la reactividad es utilizando un amplio periodo de sición correcta?:
observación para que el sujeto se habitúe a la situación de
observación (Opción 2 correcta). Por otra parte, utilizar un 1. Continúo de toda la sesión.
único observador para que la recogida de datos sea más ho- 2. De eventos.
mogénea se- ría una forma de aumentar la fiabilidad, pero 3. Temporal.
no de disminuir la reactividad (Opción 3 incorrecta). APIR 4. Focal.
Manual de Evaluación Psicológica, Tema 3.
Respuesta correcta:3
98. Cuando el comportamiento a observar es no discre-
to y por tanto resulta difícil determinar cuándo empieza Podemos distinguir dos tipos de muestreo de tiempo, en-
y cuándo acaba, el método de medición más útil es el contrán- donos por un lado con el muestreo intersesional,
método de intervalos, por ejemplo, el muestreo de inter- que consiste en analizar el periodo de observación, perio-
valo completo o total. En este muestreo: dicidad de las sesiones, número mínimo de sesiones y cri-
terios de inicio y fin de sesión. Y por otro lado, el muestreo
1. Se requiere que la conducta aparezca al menos una vez a intrasesional, en el que se analiza- ría la información regis-
lo largo del intervalo. trada dentro de cada sesión. Esta última forma podemos di-
2. Se registra la conducta que ocurre en el momento en el vidirla en cuatro subtipos de muestreo, que son: 1) Registro
que termina el intervalo. continuo de toda la sesión: En este modo se registra toda
3. La conducta no se registra si no se produce durante la la información relevante de la sesión según nues- tro obje-
totalidad del intervalo de observación establecido. tivo. Por ejemplo: los desplazamientos mediante una acti-
4. Se registra la frecuencia de las conductas que aparecen vidad (Opción 1 incorrecta). 2) Muestreo de eventos: Sólo
durante el intervalo. se registran determinados tipos de conductas. Por ejemplo,
si únicamente nos interesan las posturas de equilibrio en
Respuesta correcta:3 sesiones de actividad física para personas de tercera edad
(Opción 2 in- correcta). 3) Muestreo temporal: Se elige la
El muestreo consiste en las distintas técnicas utilizadas para información que se registra en función del tiempo real. Este
obtener muestras representativas de aquello que es objeto a su vez puede dividir- se en muestreo instantáneo o de
de observación. Dentro de los distintos tipos de muestreos puntos de tiempo, muestreo de intervalo total y muestreo
nos en- contramos con el muestreo de tiempo y dentro de de intervalo parcial. Un ejemplo lo encontramos si en una
este subtipo nos encontramos con el muestreo de intervalos sesión de clase se muestrea cada 30 segundos a un alumno
de observación, que consiste en dividir el tiempo total de para saber si se mantiene bien sentado (Opción 3 correcta).
observación en perio- dos temporales iguales. En cada in- Y por último, 4) Muestreo focal: Se utiliza cuando se obser-
tervalo, el observador señala si la conducta ha aparecido o van varios sujetos, observando a cada uno de ellos mediante
no, con independencia del núme- ro de veces que haya ocu- rondas sucesivas. Por ejemplo, en la evaluación de un en-
rrido (Opción 4 incorrecta). Este tipo de muestreo resulta trenamiento deportivo, cinco minutos de focalización con
útil en conductas no discretas, situaciones donde es difícil cada uno de los participantes de forma sucesiva (Opción 4
determinar cuándo empieza y cuándo acaba la conducta incorrecta). APIR Manual de Evaluación Psicológica, Tema
y en conductas con una tasa de frecuencia muy eleva- da. 3.

9
87. A partir de qué edad mínima (aproximada) es fiable tancias de la entrevista. Son especialmente útiles cuando se
la recogida de información mediante autoinformes en desea tener un diagnóstico rápi- do, fiable y válido. Algunos
los ni- ños: ejemplos son la SCID I (Structured Clinical Interview for
DSM-IV Axis I) y la SCID II (Structured Clinical Inter-
1. 5 años. view for DSM-IV Axis II), la SCAN, la CIDI de la OMS y la
2. 7 años. IPDE. Finalmente las entrevistas no estructuradas o libres
3. 10 años. son aquellas en las que el entrevistado habla según sus ne-
4. 12 años. cesidades, se formulan preguntas abiertas y el entrevistador
tiene que evitar ser directivo. APIR Manual de Evaluación
Respuesta correcta:2 Psi- cológica, Tema 5.

Los autoinformes son manifestaciones verbales que emite 100. Entre las técnicas de entrevista verbal no directivas
un sujeto sobre cualquier manifestación propia, bien sea se encuentra:
cogniti- va, fisiológica o motora. Por lo tanto, el observador
es el propio sujeto. Es la técnica de evaluación más utiliza- 1. La proxemia.
da y es empleada por todos los modelos teóricos, y abarca 2. La paráfrasis.
una amplia variedad temática. En cuanto a sus limitaciones, 3. La Kinesia.
una de las fundamenta- les que debemos tener en cuenta 4. La sincronía.
de cara a la evaluación infantil es el nivel de desarrollo cog-
nitivo del niño o niña que estamos evaluando, no siendo Respuesta correcta:2
fiable la información recogida con estas técnicas en edades
inferiores a los 7 años (Opción 2 correcta; Opciones 1, 3 y En esta pregunta se busca una técnica de entrevista verbal
4 incorrectas). Esto podemos relacionarlo con que previa- no directiva y se nos presentan una serie de alternativas
mente a los 7 años, el niño o la niña se encuentran en el es- de respues- ta, de las cuales la Paráfrasis es el única que se
tadio preoperacional del desarrollo cognitivo, y no es hasta considera como una técnica de entrevista y la única con un
aproximadamente los 7 años que se alcanza el estadio de las componente verbal (Opción 2 correcta). En cualquier caso
operaciones concretas, en el cual comienzan a desarrollar- procedemos a analizar una por una la definición de las cua-
se múltiples habilidades entre ellas las relacionadas con la tro alternativas de respues- ta. En primer lugar, la proxémi-
identi- ficación de las propias características psicológicas y ca hace referencia a la conducta espacial. Su observación se
la diferen- ciación de los aspectos físicos y mentales de uno centra en el análisis de la conducta en relación a aspectos
mismo. APIR Manual Evaluación Psicológica, Tema 4. estáticos como el lugar en el espacio y dinámicos como el
conjunto de desplazamientos de un sujeto, la realización de
96. La Entrevista diagnóstica DIS (Diagnostic Inter- trayectorias... (Opción 1 incorrecta). Por otra parte, la Ki-
view Schedule), que proporciona información sobre nesia hace referencia al lenguaje corporal y se cen- tra en el
el inicio, duración y actualidad de los síntomas, es un significado expresivo, apelativo o comunicativo de los mo-
ejemplo de en- trevista: vimientos corporales y de los gestos no verbales (Opción 3
incorrecta). Respecto a la cuarta alternativa de respuesta,
1. Estructurada. la sin- cronía puede hacer referencia a múltiples conceptos
2. Semiestructurada. relaciona- dos con el análisis temporal de los sucesos pero
3. Motivacional. en ningún caso se refiere a una conducta verbal (Opción 4
4. Abierta. incorrecta). De este modo concluimos que la paráfrasis, que
consiste en que el entre- vistador expresa con sus propias
Respuesta correcta:1 palabras el contenido de lo que ha explicado el entrevista-
do, para ver si ha comprendido bien lo que le ha querido
Existen múltiples formas de clasificación de las entrevistas, decir, es la única alternativa de respuesta correcta (Opción
una de las fundamentales es según su nivel de estructura- 2 correcta). APIR Manual Evaluación Psico- lógica, Tema 5.
ción. En esta clasificación nos encontramos en primer lugar
con las entrevistas estructuradas, que son entrevistas con 104. Atendiendo a la “finalidad” de la entrevista, y en
preguntas prefijadas y sistema para codificar las respuestas re- lación a la entrevista consultiva:
y analizarlas posteriormente. los principales ejemplos son
la DIS (Diagnostic Interview Schedule) (Opción 1 correcta; 1. Suele tener un objetivo focal, con una finalidad muy con-
Opciones 2, 3 y 4 inco- creta, en la que el entrevistado no va a proseguir con un
rrectas), DISC-R (Diagnostic interview Structure For Chil- tra- bajo clínico posterior.
dren) PSE, PRIME-MD, DICA, DICA-R y CHIPS. Por otra
parte, las entrevistas semiestructuradas consisten en un 2. Tiene un objetivo no terapéutico y de consejo.
guión previo que le permita al entrevistador cierta flexibi- 3. Está orientada a establecer un diagnóstico en un pa- cien-
lidad y capacidad de adaptación al cliente y a las circuns- te.

10
4. Tiene por finalidad operar un cambio en una dirección 108. En la evaluación psicofisiológica de la actividad
acordada por ambos, entrevistador y entrevistado. electrodérmica, los parámetros más frecuentes para
medir la conductancia de la piel son la amplitud de res-
Respuesta correcta:1 puesta y:

Existen múltiples formas de clasificación de las entrevistas y 1. El periodo de recuperación


una de las fundamentales es según los objetivos planteados. 2. El tiempo de latencia
A este respecto, Moreno Rosset plantea la siguiente clasi- 3. La resistencia
ficación: 1) Entrevistas diagnósticas: Se llevan a cabo con 4. La variabilidad de la respuesta a lo largo de todo el pro-
el fin de evaluar y diagnosticar (Opción 3 incorrecta). 2) ceso de medida.
Entrevistas consultivas: Pretenden dar una respuesta a una
pregunta en relación con un tema específico. Suele tener un Respuesta correcta:2
objetivo focal, y no se va a pro- seguir con un trabajo clínico
posterior (Opción 1 correcta). 3) Entrevistas de orientación Definimos la actividad electrodérmica de un organismo
vocacional: Según las capacidades o intereses del sujeto, se como la facilitación u oposición diferencial al paso de la co-
le pretende orientar hacia qué estudios o qué ámbito profe- rriente eléc- trica. Es decir, podemos entenderla como la va-
sional es el que mejor se adecua a él. 4) Entre- vistas tera- riación en carac- terísticas como la conductancia de la piel,
péuticas: Su objetivo es llevar a cabo el tratamiento median- mediadas por la su- doración. La actividad electrodérmica
te entrevistas con el fin de conseguir un cambio acordado es un índice de activación general, que posee un carácter
con el paciente (Opción 4 incorrecta). 5) Entrevistas de in- más bien inespecífico, indicando el grado de movilización
vesti- gación: En estas se recoge información con el fin de si del organismo ante un estímulo o ante una situación deter-
el sujeto se puede adscribir o no a una investigación. APIR minada, siendo el resultado de la actividad simpática del
Manual de Evaluación Psicológica, Tema 5. sistema nervioso autónomo. Los parámetros más frecuen-
temente estudiados en las respuestas de conductancia son:
106. Desde los “sistemas de expertos”, propiciados por 1) La amplitud de la respuesta, que consiste en la conduc-
el desarrollo de la Inteligencia artificial, se han creado tancia máxima en comparación con el nivel de conductan-
progra- mas destinados a: cia anterior; y 2) El tiempo de latencia, que se define como
el perio- do transcurrido desde el inicio de la presentación
1. Desarrollar baterías neuropsicológicas. del estímulo hasta el comienzo de la respuesta (Opción 2
2. La construcción de test adaptativos informatizados. correcta). El resto de alternativas de respuesta, no se corres-
3. La utilización de la “realidad virtual”. ponden con parámetros evaluados de forma especialmente
4. Emular el proceso que sigue el clínico en la evalua frecuente en relación con la actividad electrodérmica (Op-
ción para producir conclusiones diagnósticas. ciones 1, 3 y 4 incorrectas). APIR Manual Evaluación Psi-
cológica, Tema 6.
Respuesta correcta:4

Un sistema experto, es un sistema informático que emu- 90. ¿Cuál de las siguientes es una técnica de evaluación
la el ra- zonamiento humano actuando tal y como lo ha- proyectiva?:
ría un experto en un área de conocimiento. De este modo,
podemos establecer que los sistemas expertos son una de 1. El Diferencial semántico de Osgood.
las aplicaciones de la inteli- gencia artificial que pretende 2. El Test de aptitudes perceptivas.
simular el razonamiento humano, de la misma manera que 3. La Técnica de la rejilla.
lo haría un experto en un área de espe- cialización. En base 4. El Test de apercepción temática de Murray.
a esto no nos queda duda de que la única alternativa de
respuesta apropiada es la número 4, ya que con- sideramos Respuesta correcta:4
al clínico como el experto cuya función debe emular este
sistema, más concretamente en el proceso de evaluación y Las técnicas proyectivas se definen como técnicas espe-
diagnostico (Opción 4 correcta). Por otra parte, el resto de cialmente sensibles para revelar aspectos inconscientes
alter- nativas de respuesta quedan claramente descartadas de la conducta, y en las que generalmente el material esti-
ya que en ninguna de ellas encontramos esta característica mular que se presenta es ambiguo, por lo que la interpre-
de “emular el razonamiento humano actuando tal y como lo tación depende de un análisis glo- bal. Son técnicas muy
haría un experto” sino que todas ellas hablan del desarrollo influenciadas por la teoría Psicoanalítica y se basan en que
de herramientas de evaluación o intervención (Opciones 1, las respuestas que da el sujeto ante los estí- mulos que se
2 y 3 incorrectas). APIR Manual de Evaluación Psicológica, le presentan son proyecciones de los sentimientos, emocio-
Tema 1. nes, conflictos, etc., más inconscientes del sujeto. Las alter-
nativas de respuesta que nos proponen, la única que hace
referencia a una técnica proyectiva es el Test de apercepción

11
te- mática (TAT) de Murray (Opción 4 correcta). Esta téc- de palabras y las Frases incompletas. APIR Manual Evalua-
nica se engloba dentro de las técnicas temáticas o interpre- ción Psicológica, Tema 8.
tativas para adultos y consiste en 31 láminas, de las cuales
30 son ilustracio- nes estructuradas donde se representa 103. ¿Qué tipo de técnica proyectiva es el Test del Árbol
una escena determinada (como temas laborales, de peligro de Koch?:
y miedo, paterno-filiales) y 1 lámina es en blanco (para que
el sujeto imagine una escena). La tarea del sujeto consiste 1. Asociativa.
en ir narrando una historia para cada lámina presentada 2. Catártica.
que contenga un principio, un desarrollo y un final, y don- 3. Expresiva.
de se muestren los pensamientos y los sentimientos de los 4. Estructural.
personajes. Respecto al resto de alternativas de respuesta, el
Diferencial semántico de Osgood y la Técnica de la rejilla Respuesta correcta:3
de Kelly son técnicas subjetivas (Opciones 1 y 3 incorrectas)
mientras que las pruebas de aptitudes perceptivas se cen- Las técnicas proyectivas se definen como técnicas muy in-
tran en evaluar rapidez y precisión perceptiva mediante test fluen- ciadas por la teoría Psicoanalítica y especialmente
en los que el sujeto ha de localizar rápidamente una figura sensibles para revelar aspectos inconscientes de la conducta,
en base a los de- talles visuales, lo cual no forma parte de las en las que generalmente el material estimular que se presen-
técnicas proyectivas (Opción 2 incorrecta). APIR Manual ta es ambiguo, por lo que la interpretación depende de un
Evaluación Psicológica, Tema 8. análisis global. Una de las principales clasificaciones de las
técnicas proyectivas es la propuesta por Fernández Balleste-
97. ¿Qué tipo de técnica proyectiva es el test “Dibujo de ros, que divide las técnicas proyectivas en: 1) Estructurales:
la figura humana”?: Su principal característica es la presentación de estímulos
poco estructurados los cuales el sujeto ha de estructurar.
1. Estructural. Entre estas técnicas destacan el Test de Ror- schach, el HIT
2. Temática. de Holtzman y la Z de Zulliger (Opción 4 inco- rrecta). 2)
3. Constructiva. Temáticas: Estas pruebas se caracterizan por presen- tar lá-
4. Expresiva. minas sobre las que el sujeto debe narrar una historia. Las
pruebas principales de este grupo son el TAT, el TRO, el
Respuesta correcta:4 PFT y el CAT. 3) Expresivas: En estas pruebas se le dice al
sujeto que ha de hacer un dibujo. Las técnicas principales de
Las técnicas proyectivas se definen como técnicas muy este subgrupo son el Test de la familia de Corman, el Test de
influen- ciadas por la teoría Psicoanalítica y especialmen- la figura humana de Machover y el Test del árbol de Koch
te sensibles para revelar aspectos inconscientes de la con- (Opción 3 correcta). 4) Constructivas: En estas pruebas se
ducta, en las que generalmente el material estimular que se dan unas instrucciones y el sujeto ha de organizar el mate-
presenta es ambiguo, por lo que la interpretación depende rial que se le da, siendo las pruebas principales el Test del
de un análisis global. Una de las principales clasificaciones pueblo, el Test de la aldea imaginaria y el Juego diagnóstico.
de las técnicas proyectivas es la propuesta por Fernández 5) Asociativas: En estas técnicas el sujeto ha de responder
Ballesteros, que divide las técnicas proyectivas en: 1) Es- a los estímulos que se le presentan y sus respuestas se aso-
tructurales: Su principal característica es la presentación cian a ellos. Entre estas pruebas encontramos las Fábulas
de estímulos poco estructurados los cuales el su- jeto ha de de Düss, la Asociación de palabras y las Frases incompletas
estructurar. Entre estas técnicas destacan el Test de Rors- (Opción 1 incorrecta). Finalmente, la opción de respuesta
chach, el HIT de Holtzman y la Z de Zulliger. 2) Temáticas: núme- ro 2 (Incorrecta) se corresponde con la clasificación
Estas pruebas se caracterizan por presentar láminas sobre de Frank de las técnicas proyectivas y se relaciona con las
las que el sujeto debe narrar una historia. Las pruebas prin- técnicas construc- tivas de la clasificación de Fernández Ba-
cipales de este grupo son el TAT, el TRO, el PFT y el CAT. llesteros. APIR Manual Evaluación Psicológica, Tema 8.
3) Expresivas: En estas pruebas se le dice al sujeto que ha de
hacer un dibujo. Las técnicas principales de este subgrupo
son el Test de la familia de Corman, el Test de la figura hu- 91. En las Escalas de inteligencia de Wechsler para
mana de Machover (Opción 4 co- rrecta; Opciones 1, 2 y 3 adultos (WAIS-III, WAIS-IV), ¿cuáles de estas escalas
incorrectas) y el Test del árbol de Koch. 4) Constructivas: for- man parte del Índice de memoria de trabajo?:
En estas pruebas se dan unas instrucciones y el sujeto ha de
organizar el material que se le da, siendo las pruebas prin- 1. Clave de números y Búsqueda de Símbolos.
cipales el Test del pueblo, el Test de la aldea imaginaria y 2. Vocabulario y Semejanzas.
el Juego diagnóstico. Y finalmente, 5) Asociativas: En estas 3. Aritmética y Dígitos.
técni- cas el sujeto ha de responder a los estímulos que se 4. Cubos y Matrices.
le presentan y sus respuestas se asocian a ellos. Entre estas
pruebas encontra- mos las Fábulas de Düss, la Asociación Respuesta correcta:3

12
Las Escalas de inteligencia de Wechsler son escalas basadas 3. Escalas de capacidad diferencial.
en metodología racional y arrojan puntuaciones de CI. En 4. Test de matrices progresivas de Raven.
concreto el WAIS se corresponde con la versión para adul-
tos y tiene como finalidad el evaluar la inteligencia general. Respuesta correcta:4
Uno de los cambios que introduce el WAIS-III es la inclu-
sión de los índices de Com- prensión verbal, Memoria de Desde un enfoque clásico las pruebas de inteligencia se divi-
trabajo, Organización perceptiva y Velocidad de procesa- den en test basados en el método racional y test basados en
miento. Las pruebas que se engloban en cada uno de estos el método factorial. Test basados en la metodología racional
índices son los siguientes: 1) Comprensión verbal: Seme- o enfoques clínicos son pruebas que están elaboradas según
janzas, Vocabulario e Información (Opción 2 in- correc- la teoría a la que pertenezca el autor. El máximo exponente
ta); 2) Memoria de trabajo: Aritmética, Dígitos y Letras y de esta orienta- ción es Wechsler, con sus escalas (Opción
Números (Opción 3 correcta); 3) Organización perceptiva: 1 incorrecta). Tam- bién podemos englobar en este grupo
Cubos, Matrices, Figuras Incompletas (Opción 4 incorrec- las pruebas desarrolladas siguiendo los trabajos de Binet,
ta); y 4) Velocidad de procesamiento: Clave de Números y donde encontramos la escala de inteligencia Stanford-Binet
Búsqueda de Símbolos (Opción 1 incorrecta). APIR Ma- (Opción 2 incorrecta). Por otra parte, los test basados en la
nual Evaluación Psicológica, Tema 9. metodología factorial son pruebas elabo- radas a partir del
análisis factorial, siendo Spearman el pionero de esta for-
93. Las Escalas McCarthy de aptitudes y psicomotrici- ma de entender la inteligencia. Dentro de estas prue- bas
dad par niños (MSCA): podemos encontrar dos tipos: 1) los test de factor G, en los
cuales se realiza una única medida relacionada con el fac-
1. Evalúan las aptitudes del niño a través de dos escalas tor G para conocer el nivel intelectual del sujeto y siendo el
principales: la de procesamiento secuencial y la de procesa- ejemplo más característico el test de Matrices Progresivas
mien- to simultáneo. de Raven (Op- ción 4 correcta); y 2) los test de aptitudes,
2. Son adecuadas para evaluar a niños muy pequeños y con que evalúan una serie de capacidades o habilidades especí-
dificultades de aprendizaje. ficas que posee el sujeto. El ejemplo más característico es el
3. Se utilizan para valorar la inteligencia desde la perspec- PMA de Thurstone. APIR Ma- nual de Evaluación Psicoló-
tiva de la evaluación dinámica, es decir, evalúan el potencial gica, Tema 9.
de aprendizaje del niño.
4. Son útiles para valorar la motricidad y las aptitudes de 99. Respecto a la evaluación de la inteligencia infantil,
niños con inteligencia dentro de la media, ya que son poco ¿qué prueba permite medir la inteligencia general se-
sen- sibles para evaluar a niños con retraso intelectual. gún dos subescalas: verbal y no verbal?:

Respuesta correcta:2 1. El K-Bit de Kaufman.


2. Las Escalas McCarthy (MSCA).
Las Escalas McCarthy de aptitudes y psicomotricidad para 3. El TONI-2.
niños tienen como objetivo la evaluación de las capacidades 4. El K-ABC de Kaufman.
cognos- citivas de niños pequeños (2 años y 6 meses - 8
años y 6 meses) o con dificultades de aprendizaje (Opción 2 Respuesta correcta:1
correcta; Opción 4 incorrecta). Estas escalas proporcionan
una medida general del funcionamiento intelectual a tra- Dentro de las escalas de inteligencia infantil nos encontra-
vés del Índice Cognoscitivo Ge- neral (ICG) y muestran un moscon las escalas de inteligencia de Kaufman, entre las
perfil de capacidades. Esto lo hace empleando 6 escalas, que que ubica-mos el K-ABC y el K-BIT. La primera de estas
son: Escala verbal, Escala perceptivo manipulativa, Escala pruebas es una batería elaborada con el objetivo de evaluar
numérica, Escala de memoria, Escala de motricidad y Esca- de forma diferen- ciada inteligencia y rendimiento acadé-
la general cognoscitiva. Respecto al resto de alternativas de mico, centrándose en la forma de procesamiento, más que
respuesta, la número 1 hace referencia la escala K-ABC de en el resultado final. Resulta
Kaufman (Opción 1 incorrecta) y la número 3 al en- foque útil para la evaluación de niños con trastornos del apren-
de Feuerstein de evaluación del potencial de aprendizaje dizaje, 1. retraso del lenguaje o sordos, y cuenta con escasa
(Opción 3 incorrecta). APIR Manual Evaluación Psicoló- pruebas ver- 2. bales. Resulta poco adecuada para sujetos
gica, Tema 9. brillantes o superdo- tados (Opción 4 incorrecta). En rela-
ción al K-BIT, es un test de 3. screening (no diagnóstico),
95. ¿Qué instrumento para la medida de la inteligencia que se aplica en 15-30 minutos y 4. es de fácil corrección.
se diseñó para evaluar el factor “g” tal y como fue defi- Esta prueba evalúa la inteligencia verbal y no verbal (Op-
nido por Spearman?: ción 1 correcta) y está considerado como una gran medida
de la Inteligencia General. Está compuesto por dos sub-
1. Escalas de inteligencia de Weschler. pruebas: 1) Vocabulario: Mide la inteligencia verbal y cons-
2. Escala de inteligencia de Stanford-Binet. ta de Vocabulario expresivo y Definiciones; y 2) Matrices:

13
Mide la inteligencia no verbal y está formado por Dibujos y 1. Memoria de trabajo.
Figuras abstractas con el fin de eliminar la influencia cultu- 2. Comprensión verbal.
ral. Respecto al resto de alternativas de respuesta, el MSCA 3. Velocidad de procesamiento.
tienen como ob jetivo la evaluación de las capacidades cog- 4. Organización perceptiva.
noscitivas de niños pequeños o con dificultades de apren-
dizaje. Estas escalas pro- porcionan una medida general del Respuesta correcta:3
funcionamiento intelectual a través del Índice Cognoscitivo
General (ICG) y muestran un per- fil de capacidades: ca- Las Escalas de inteligencia de Wechsler son escalas basadas
pacidad verbal, de razonamiento no verbal, aptitud numé- en metodología racional y arrojan puntuaciones de CI. En
rica, memoria a corto plazo y coordinación (Opción 2 in- concreto el WAIS se corresponde con la versión para adultos
correcta). Finalmente, el TONI-2 permite la estimación del y tiene como finalidad el evaluar la inteligencia general. Uno
funcionamiento intelectual general mediante la evaluación de los cambios que introduce el WAIS-III es la inclusión de
de la capacidad para resolver problemas abstractos de tipo los índices de Com- prensión verbal, Memoria de trabajo,
gráfico y no verbal (Opción 3 incorrecta). APIR Manual Organización perceptiva y Velocidad de procesamiento. Las
Evaluación Psi- cológica, Tema 9 pruebas que se engloban en cada uno de estos índices son
los siguientes: 1) Comprensión verbal: Semejanzas, Voca-
101. En general, la evaluación dinámica del potencial bulario e Información (Opción 2 in- correcta); 2) Memoria
de aprendizaje es apropiada para determinar el trata- de trabajo: Aritmética, Dígitos y Letras y Números (Opción
miento en casos de: 1 incorrecta); 3) Organización perceptiva: Cubos, Matrices,
Figuras Incompletas (Opción 4 incorrecta); y 4) Velocidad
1.Sujetos de bajo nivel cognitivo. de procesamiento: Clave de Números (Opción 3 correcta)
2.Sujetos que presentan rendimiento académico adecua do. y Búsqueda de Símbolos . APIR Manual Evaluación Psico-
3.Sujetos con dificultades para aprendizajes generales. lógica, Tema 9.
4.En aquellos casos en que sea preciso clarificar los pro ce-
sos automáticos implícitos en los aprendizajes. 107. Cuando se aplica la escala de inteligencia Wechsler
(WPPSI-IV) en niños menores de cuatro años se inclu-
Respuesta correcta:1 ye:

El modelo de evaluación dinámica se aplica fundamental- 1.Test de dibujos.


mente en el ámbito clínico, educativo y social. En concreto 2. Aritmética.
en el ám- bito clínico se ha empleado para evaluar el po- 3. Búsqueda de animales.
tencial de apren dizaje de sujetos con retraso mental, di- 4. Test de semejanzas.
ficultades de aprendiza- je lector, ciegos o con dificultades
de lenguaje. Por otra parte, en el ámbito educativo ha sido Respuesta correcta:1
empleado para la identificación de superdotación entre
grupos sociocultural o lingüísticamente desfavorecidos. En Las Escalas de inteligencia de Wechsler son escalas basadas
base a esto no es de extrañar que sea en el dominio socio- en metodología racional y arrojan puntuaciones de CI. En
cultural donde la evaluación dinámica alcanza sus máximas concreto el WPPSI-IV es la versión del WPPSI más actual
aplicaciones, especialmente en la evaluación cogni- tiva de y evalúa ni- ños de preescolar y primaria con edades com-
grupos pertenecientes a minorías étnicas (en casos de in- prendidas entre 2,6 y 7,7 años. Es más flexible y presenta
migración, o grupos especiales). También se ha aplicado en una estructura ampliada y conformada por una escala to-
el ámbito laboral y en el de la tercera edad. A modo gene- tal, escalas primarias y secun- darias para los dos posibles
ral se establece que la evaluación dinámica del potencial de rangos de edad de aplicación (2,6- 3,11 y 4-7,7 años) de la
aprendi- zaje es apropiada para determinar el tratamiento prueba. En concreto el enunciado nos está preguntando por
en casos de: 1) sujetos de bajo nivel cognitivo (Opción 1 las pruebas diseñadas para el primer rango de edad, en el
correcta); 2) sujetos que presentan rendimiento académi- que podemos encontrar en la escala de Lenguaje General la
co inadecuado que no puede ser justificado mediante las prueba nombres, en la escala CI verbal las pruebas Dibujos
medidas cognitivas psicométricas clásicas (Opción 2 inco- (Opción 1 correcta) e información y en la escala CI Ma-
rrecta); 3) sujetos con dificultades para aprendizajes espe- nipulativo las pruebas cubos y rompecabezas. Respecto al
cíficos (Opción 3 incorrecta); y 4) en aquellos casos en que resto de alternativas de respuesta, Semejanzas y Búsqueda
sea preciso clarificar los procesos motivacionales implícitos de anima- les constituyen pruebas complementarias para el
en los aprendizajes (Opción 4 incorrecta). APIR Ma- nual segundo rango de edad de WPPSI-IV (Opciones 3 y 4 in-
Evaluación Psicológica, Tema 9. correctas) y Aritmética es una prueba propia del WISC, no
del WPPSI. APIR Manual Evaluación Psicológica, Tema 9.
102. En las Escalas de Inteligencia de Wechsler para
adultos (WAIS-III y WAIS-IV), ¿de qué índice forma
parte la prueba “Clave de números”?:

14
94. La evaluación de la personalidad se ha hecho des- de ESTADÍSTECA, MÉTODO EXPE-
diversos modelos y teorías. Desde las teorías factoriales,
el test más representativo que ha inspirado el desarrollo RIMENTAL Y PSICOMETRÍA
de otros, es:

1. Minnesota multiphasic personality inventory, (MMPI).


2. Sixteen personality factor questionnaire, (16PF). 159. En el ámbito del análisis estadístico, en concreto de
3. Gordon personal profile (GPP). la regresión lineal, indique la afirmación INCORREC-
4. Tennessee Self Concept Scale (TSCS). TA:

Respuesta correcta:2 1. La regresión lineal permite analizar la relación entre uno


o más predictores cuantitativos y una variable dependiente
El modelo psicométrico de evaluación de la personalidad se también cuantitativa, no pudiéndose en ningún caso incluir
en- marca dentro de los modelos de rasgos, enfatizan los as- en el análisis predictores categóricos.
pectos nomotéticos y cuantitativos. Sin embargo, los crite- 2. En el análisis de regresión, el coeficiente de regresión par-
rios desde los que se han desarrollado las distintas pruebas cial no estandarizado Bo ó0 ofrece el valor de la ecuación
psicométricas difie- ren, pudiendo clasificarse en criterio de regresión cuando todos los predictores son igual a cero.
racionales, criterios empí- ricos, criterios factoriales y cri- 3. Una estrategia para medir la bondad de ajuste en un aná-
terios combinados. La evaluación desde criterios racionales lisis de regresión simple es utilizar el coeficiente de deter-
fue la primera en aparecer y se basa en la idea de que existe minación que se calcula elevando al cuadrado el coeficiente
una correspondencia entre la respuesta que el sujeto da al de correlación de Pearson.
ítem presentado y determinados estados internos hipoté- 4. Si se desea encontrar la ecuación de regresión capaz de
ticos postulados desde una teoría. La primera prueba en ofrecer el mejor ajuste posible con el menos número de va-
aparecer que se basa en estos criterios es la Hoja de Datos ria-bles, se recomienda utilizar la regresión jerárquica o por
Per- sonales (PDS) de Woodworth (1914). Las pruebas de pasos.
evaluación desde criterios empíricos son pruebas que están
orientadas por un criterio externo, dando nula importancia Respuesta correcta:1
a la unión entre la respuesta emitida por un individuo y el
comportamiento que pre- dice. Los cuestionarios más re- La regresión es una fórmula matemática que nos permite
presentativos de esta categoría son el MMPI (Opción 1 in- esta- blecer predicciones a partir de las relaciones lineales
correcta) y el CPI. La evaluación desde criterios factoriales observadas entre variables. Los predictores en la regresión
consiste en que con el análisis factorial de los ítems se consi- lineal múltiple pueden ser de un nivel de medida cuantita-
gue agruparlos en factores, siendo estos fac- tores conside- tivo, ordinal o nomi- nal (o categórico) (opción 1 correc-
rados desde esta perspectiva como rasgos dimen- sionales. ta). Es decir, conociendo la puntuación de un sujeto en una
Esta estrategia es la más común en la construcción de los variable (X), se pronostica su puntuación en otra variable
cuestionarios para la evaluación de rasgos y dimensiones de (Y). A menos que la correlación en- tre las dos variables sea
personalidad, siendo el test más representativo y que sirvió perfecta, los dos pares de puntuaciones no coinciden en una
de base para el resto el 16PF de Cattel (Opción 2 correcta). misma recta. Es decir, hemos de valorar el grado de pre-
Fi- nalmente, respecto a la evaluación desde criterios com- dicción que tiene nuestra recta de sobre la variable depen-
binados, estas pruebas personalidad han sido creadas a par- diente. Esta valoración se realiza mediante el coeficiente de
tir de la com- binación de criterios racionales, empíricos y determinación, que se calcula elevando al cuadrado el coe-
factoriales, siendo las más conocidas en este ámbito las rea- ficiente de correlación de Pearson, e indica qué proporción
lizadas por Millon y Cloninger. Respecto al resto de opcio- de varianza del criterio (X) queda explicada por el modelo
nes de respuesta, el Perfil personal de Gordon (GPP) es una lineal (opción 3 incorrecta por ser verdadera). La ecuación
prueba factorial basada en el 16PF que evalúa Ascendencia, de regre- sión sería la de un plano (para dos variables pre-
Responsabilidad, Estabilidad y Sociabilidad (Opción 3 in- dictoras): Y = B0 + B1 X1 + B2 X2. Con B1 representando
correcta). La Tennessee self concept scale (TSCS) también la pendiente del plano en el eje 1, B2 representando la pen-
es una prueba factorial, pero de menor importancia que el diente del plano en el eje 2 y con B0 representando la altura
16PF (Opción 4 incorrecta). APIR Manual de Evaluación a la que corta el plano el punto Y (el eje de ordenadas). Este
Psicológica, Tema 11. es el valor de la constante, esto es, el valor de Y cuando el
predictor/es es/son igual a 0 (opción 2 incorrecta). Añadir
variables independientes o predictoras (Y) al modelo siem-
pre podrá aumentar o mantener el coeficiente de determi-
nación, ya que vamos sumando información predictora a
nuestro modelo. En este sentido, para poder obtener el me-
jor modelo con el menor número de variables posibles, se
puede recurrir a los siguientes métodos: 1) jerárquico; 2) in-

15
troducir (si- multáneo); 3) escalonado (stepwise); 4) hacia ne como la capacidad de una prueba estadística para detec-
atrás (backward); 5) hacia adelante (forward); y 6) eliminar tar una relación entre variables.
(opción 4 incorrecta por ser verdadera). APIR Manual de 2. El error de tipo II se refiere al hecho de detectar una rela-
Psicología Experimental y Psicometría, Tema 1. ción entre variables cuando de hecho no existe tal relación
en la población.
160. En el marco del análisis de la varianza, con un fac- 3. La probabilidad de cometer un error de tipo I se deno-
tor y tres tratamientos, indique la afirmación INCO- mina alpha y se refiere a rechazar la hipótesis nula cuando
RREC- TA: dicha hipótesis es verdadera.
4. El tamaño muestral tiene una relación inversamente pro-
1. La prueba de Dunn-Bonferroni nos permitiría llevar a porcional con la potencia de la prueba estadística.
cabo comparaciones múltiples entre pares de medias plani-
fica- das o a priori. Respuesta correcta:3
2. Si el investigador desea realizar todas las posibles com-
paraciones por pares, post hoc o a posteriori, podía utilizar Un contraste de hipótesis es una decisión en la que una hi-
la prueba de Tukey. pótesis formulada en términos estadísticos se pone en rela-
3. Para estimar el tamaño del efecto relativo a la propor- ción con unos datos empíricos, y en el que se determina si
ción de varianza explicada, sería aconsejable la utilización existe una com- patibilidad entre los dos. El sujeto plantea
de eta-cuadrado (2), por ser un estimador no sesgado. una hipótesis nula (H0), que supone que no existe relación
4. En el caso de incumplir el supuesto de homogeneidad de entre las variables, en contraste con una hipótesis alternati-
las varianzas se recomienda utilizar los estadísticos W de va (H1), que niega a H0 e in- cluye todo lo que excluye H0
Welch o F de Brown y Forsythe, como alternativa al esta- (es decir, que existe, en una u otra dirección, relación entre
dístico F. las variables). Para decidir si mantener o rechazar la hipó-
tesis nula se recurre a la regla de la decisión, que consiste
Respuesta correcta:3 en cuantificar el nivel de riesgo o significación que se está
dispuesto a aceptar. Es decir, define un margen de error a la
En un ANOVA de 3 niveles, tendríamos un único factor o hora de considerar como falsa una hipótesis que en realidad
varia- ble independiente (llamémoslo A) y tres niveles o tra- es verdadera (el riesgo que estoy dispuesto a cometer de re-
tamientos diferentes, (A1, A2 y A3). Para poder aplicar un cha- zar H0 cuando realmente es verdadera, o lo que es lo
análisis de la varianza (ANOVA) han de cumplirse una se- mismo, decir que existe una relación entre las variables es-
rie de supuestos, en- tre ellos la homocedasticidad o igual- tudiadas cuan- do realmente no la hay). Este nivel riesgo o
dad de varianzas. En el caso de que no se cumpla podemos significación se re- presenta mediante α, y supone, como he-
transformar los datos mediantelogaritmo neperiano, míni- mos explicado, cometer un error tipo I (opción 3 correcta).
mos cuadrados, test de Welch o prueba de Brown-Forsythe De forma complementaria tenemos un nivel de confianza,
(opción 4 incorrecta por ser verdadera). Una vez aplicada la 1-α, que se define como la pro- babilidad de mantener H0
prueba y sabiendo que los resultados son esta- dísticamen- cuando esta es verdadera. Pero ¿Qué sucede si realmente
te significativos, es decir, que el factor A tiene efecto sobre existe una relación entre las variables estu- diadas y no la
la variable dependiente que queremos medir, habremos de detectamos? En este caso, mantener la H0 cuando es falsa
especificar cuáles son los niveles de nuestro factor que pro- supone cometer un error tipo II, y se representa con la le-
ducen dicho efecto. Para comprobar esto se recurre a distin- tra β (opción 2 incorrecta). Complementaria al error tipo
tas pruebas estadísticas. Algunas pruebas de comprobación II es- taría la potencia de la prueba o 1-β, que muestra la
no planeada o a posteriori son la prueba de Scheffé, Tuckey capacidad de una prueba estadística para rechazar una H0
(opción 2 incorrec- ta por ser verdadera) o Dunnett, o a cuando realmente es falsa (es decir, para detectar la rela-
priori las de Dunn-Sidák o Dunn-Bonferroni (opción 1 in- ción entre variables) (op- ción 1 incorrecta). La potencia de
correcta por ser verdadera). No obstante, en la actualidad se una prueba es directamente proporcional al tamaño de la
emplean otras pruebas, no basadas en la significación esta- muestra, a α y al tamaño del efecto, es decir, cuanto mayor
dística de decisión dicotómica (rechazo versus mantengo la es α, cuanto mayor tamaño del efecto y cuanto mayor sea la
H0), como son los índices de tamaño de efecto. En el caso muestra, mayor potencia estadística tendre- mos (opción 4
del ANOVA, se recomienda el uso de omega cuadrado, ya incorrecta). APIR Manual de Psicología Experi- mental y
que Eta-cuadrado η2 es un estimador sesgado a nivel po- Psicometría, Tema 1.
blacional (opción 3 correcta por ser falsa). APIR Manual de
Psicología Experimental y Psicometría, Tema 1. 158. Para interpretar adecuadamente los resultados
ob- tenidos mediante un análisis factorial de la varianza
163. En lo que respecta a la validez de conclusión esta- debe tenerse en cuenta (indique la afirmación INCO-
dís- tica, indique cuál de las siguientes afirmaciones es RRECTA):
correcta:
1. Cuando el efecto de interacción es estadísticamente sig-
1. La potencia es complementaria al error de tipo I y se defi- nificativo, el interés de los efectos principales se reducen en

16
cierta medida dado que su interpretación depende de los grupo control NO equivalente (opción 3 correcta por ser
niveles del resto de factores. falsa), y el diseño de discontinuidad en la regresión. En este
2. El análisis de los efectos simples implica interpretar el último, la regla de asignación de los sujetos a los tratamien-
efecto de un factor cuando se tiene en cuenta un único ni- tos se conoce, es decir, NO es aleatoria pero es conocida
vel del otro factor. (opción 1 incorrecta por ser verdadera). Los sujetos se asig-
3. El análisis de los efectos simples sólo tiene sentido cuan- nan a la condición experimental o a la condición de control
do la interacción entre los factores es estadísticamente sig- en función de las puntuaciones de la medida pretratamien-
nificativa. to, de forma que aquellos sujetos que pun- túen por debajo
4. El efecto de interacción solo será estadísticamente sig- ni- o por encima del punto de corte son asignados a un grupo
ficativo si los efectos principales también lo son. o a otro (se establece un punto de corte). Se denomina dise-
ño de discontinuidad en la regresión porque se calcula una
Respuesta correcta:4 recta de regresión a partir de las puntuaciones obtenidas
en el pretest y postest (si el tratamiento no tiene efecto los
En un análisis factorial de la varianza (ANOVA II) existe 2 resultados se mostrarán en una sola línea, la línea iniciada
fac- tores o variables independientes manipuladas por el ex- en el grupo con- trol se prolongará en la del experimen-
perimen- tador. Cuando los resultados arrojan diferencias tal. En cambio, si el tra- tamiento tiene efecto se muestra
estadísticamen- te significativas, la variabilidad puede ser una discontinuidad en la línea a partir del punto de corte)
debida: al factor A, al factor B o a la interacción del factor (opciones 2 y 4 incorrectas por ser verdaderas). Es uno de
A con el B (opción 4 correcta) (más una variabilidad error), los diseños cuasiexperimentales con los que puede inferirse
para lo cual existen unas reglas de jerarquía: si el efecto in- relaciones causales con más garantías. APIR Manual de Psi-
teractivo resulta significativo, no se interpretan los efectos cología Experimental y Psicometría, Tema 2.
principales de los factores que con- forman tal interacción,
sino que solo se analizará dicho efecto interactivo y los 162. En relación con los diseños de investigación en Psi-
correspondientes efectos simples (es decir, el efecto de un cología, indique cuál de las siguientes afirmaciones es
nivel de un factor bajo cada nivel del otro). Es in- teresan- INCO- RRECTA:
te conocer que los efectos principales pueden no resultar
estadísticamente significativos, pero sí alcanzar la significa- 1. El diseño de bloques tiene como objetivo, entre otros, el
ción estadística un efecto interactivo, al ser éste último el análisis de la interacción entre la variable de tratamiento y
que mejor explique la varianza de la variable dependiente la variable de bloqueo.
(opciones 1, 2 y 3 incorrectas por ser verdaderas). APIR 2. El diseño de cuatro grupos de Solomon permite anali- zar
Manual de Psicología Experimental y Psicometría, Tema 2. el posible efecto de sensibilización del pretest, en el efecto
del tratamiento.
161. Un investigador desea analizar el impacto de una 3. En los diseños de medidas repetidas, el principal pro-
in- tervención para mejorar las capacidades de lecto-es- blema es el derivado de la secuencialidad en la que se apli-
critura. Para ello, antes de aplicar la intervención mide can los tratamientos.
la capacidad de lecto-escritura en la muestra y establece 4. Los diseños experimentales de covarianza tienen como
un punto de cor- te. Los sujetos con puntuaciones por objetivo la reducción de la varianza de error, incluyendo va-
encima del punto de corte constituirán el grupo de con- ria- bles cuantitativas como covariables.
trol y aquéllos por debajo del punto de corte recibirán
la intervención (grupo experi- mental).Tras la interven- Respuesta correcta:1
ción, el investigador medirá nueva- mente la capacidad
en lecto-escritura de todos los sujetos. Según este enun- El bloqueo es una técnica de control estadístico que consis-
ciado, señale la opción INCORRECTA: te en neutralizar el efecto de una variable contaminadora
(variable de bloqueo) mediante la formación de conjuntos
1. Se ha utilizado una regla de asignación no aleatoria pero o bloques de suje- tos que comparten una característica re-
conocida. levante al fenómeno es- tudiado. A continuación se forman
2. Se trata de un diseño de discontinuidad en la regresión. grupos que tengan un número similar de sujetos en cada
3. Se trata de un diseño con grupo de control no equiva uno de los bloques, lo que permite la formación de grupos
lente. homogéneos una vez que se ha eliminado a los sujetos con
4. Se ha utilizado un diseño cuasi-experimental pre test-pos- una puntuación extrema en dicha variable. En los diseños
test con grupo de control. intergrupo de bloques, por tanto, es posible interpretar los
resultados del efecto de la variable independiente sobre la
Respuesta correcta:3 variable dependiente con las garantías de haber bloqueado
va- riables contaminadoras (opción 1 correcta por ser fal-
Dentro de los diseños cuasi-experimentales podemos dis- sa). Otra herramienta para el control estadístico del error
tinguir aquellos con y sin grupo control. Dentro de los di- experimental es el análisis de la covarianza (ANCOVA),
seños con grupo control, a su vez, distinguimos diseños de que permite eliminar la heterogeneidad causada en la varia-

17
ble de interés (VD) por la influencia de una o más variables Esto implica que se obtiene un indicador de la precisión de
(covariables) (opción 4 inco- rrecta por ser verdadera).El las estimaciones para niveles concretos (opción 2 correcta,
diseño Solomon es un tipo de dise- ño experimental uni- opciones 1, 3 y 4 incorrectas). APIR Manual de Psicología
factorial donde la creación de cuatro grupos (grupos con/ Experimental y Psicometría, Tema 3.
sin medida pretratamiento y grupos con/sin variable trata-
miento) permite mejorar las valideces internas y externas. 157. La fórmula de Spearman-Brown se usa para esti-
Una de sus ventajas es que permite controlar el efecto de la mar el cambio en el coeficiente de fiabilidad de un test
me- dida pre-tratamiento al darse una ausencia de interac- que se produce cuando aumenta:
ción entre esta y la variable independiente (o tratamiento)
(opción 2 inco- rrecta por ser verdadera). Los diseños de 1.La variabilidad de una nueva muestra en que se analiza
medidas repetidas o diseños intragrupo estudian el efecto el test.
de una variable indepen- diente o tratamiento dentro de 2.La longitud de un test añadiendo ítems parale los a los del
un mismo grupo, por lo que apa- recen errores progresi- test original.
vos debido a la repetición de tareas por los mismos sujetos 3.El tamaño de una nueva muestra en que se analiza el test.
(opción 3 incorrecta por ser verdadera). APIR Manual de 4.La dificultad de los ítems en una nueva ver sión del test.
Psicología Experimental y Psicometría, Tema 2.
Respuesta correcta:2
156. Algunas de las limitaciones de la Teoría clásica de
los test frente a la Teoría de respuesta al ítem vienen Como hemos visto, uno de los problemas de la Teoría Clá-
por: sica de los Test (TCT) es que las estimaciones que realiza
del coeficien- te de fiabilidad NO son independientes de la
1. La menos fiabilidad de los instrumentos y la mayor com- muestra sobre la que se aplica el test. Esto implica que de-
plejidad matemática de los modelos antiguos. terminadas variables de la muestra afectan al cálculo de la
2. La ausencia de invarianza y la consideración de un nivel fiabilidad, entre ellas la varia- bilidad de las puntuaciones o
de precisión uniforme la longitud del test. Si calculamos la fiabilidad por el méto-
3. La ausencia de invarianza y la ausencia de va lidez pre- do de dos mitades paralelas hemos de corregir su valor, ya
dictiva. que la correlación entre ambas nos daría el coeficiente de un
4. El grado de dificultad de los ítems y la menor validez de test de mitad de tamaño que el original. Es de esperar, por
contenido. tanto, que el aumento de ítems del test o su mayor longitud
influyan en el coeficiente de fiabilidad aumentándolo, aun-
Respuesta correcta:2 que esto solo sucederá si los ítems añadidos son paralelos
a los anteriores. La fórmula con la que se puede calcular
La Teoría de Respuesta al Ítem (TRI) o Teoría del rasgo la- cuál es la fiabilidad resultante de un test al aumentarlo k
tente, surge de la mano de Lord, Rasch y Lazarsfeld como veces se denomi- na fórmula de Spearman-Brown (opción
un modelo psicométrico alternativo a la Teoría Clásica de 2 correcta, opciones 1, 3 y 4 incorrectas). APIR Manual de
los Test (TCT). Su punto de partida es que todos los sujetos Psicología Experimental y Psicometría, Tema 3.
evaluados poseen una cierta aptitud en un rasgo psicoló-
gico que no es observable y que no tiene porque mantener
una relación lineal con la puntuación de los sujetos en el
test. Una de las características que define a este modelo es la
PSICOLOGÍA BÁSICA
utilización de modelos NO lineales de mayor complejidad
165. En la tarea de seguimiento (sombreado) ideada por
que los de la TCT (opción 1 incorrecta), como los modelos
Cherry (1953) se utilizó la escucha dicótica y se presen-
de Ojiva Normal o los modelos logísticos de 1, 2 o 3 pará-
ta- ron dos mensajes uno por cada oído. El participante
metros. La TCT presenta una serie de problemas, superados
debía repetir palabra por palabra el mensaje del oído
por la TRI, que se pueden resumir en 2: el resultado obteni-
atendido. Cuando se preguntó por el mensaje ignorado
do al medir una variable es inseparable del test usado (que
(no atendido) los participantes detectaron:
sería como pensar que el peso de un objeto depende de la
balanza que se utilice), y las propiedades de los ítems y el
1. El cambio de idioma.
test están determinadas por las características de los sujetos
2. El significado del mensaje.
evaluados (por ejemplo, esto equivaldría a pensar que un
3. La relación semántica entre el mensaje atendi
kilo de acero no pesa lo mismo que un kilo de plumas). La
do y no atendido.
TRI, por el contrario se caracteriza por su invarianza res-
4. El cambio de voz del hablante.
pecto a la muestra, es decir, los parámetros del ítem no cam-
bian aunque las personas que contesten sean distintas, y por
Respuesta correcta:4
invarianza con respecto al instrumento utilizado, por lo que
el nivel de habilidad de la persona que evaluamos no de-
En el paradigma de seguimiento de Cherry se realizó un
pende del test (es decir, de que sea muy fácil o muy difícil).

18
expe- rimento en el cual se presentaba dos mensajes distin- 167. De acuerdo con los estudios de Tulving y otros au-
tos a los sujetos, uno por cada oído, y se les pedía que pres- tores sobre la recuperación de información de la me-
tasen atención únicamente a uno de ellos. Con este proce- moria, ¿qué claves de recuperación son las más eficaces
dimiento se observaba que la atención es selectiva, ya que se para recu- perar un determinado contenido de la me-
recordaba perfectamen- te el estímulo atendido, pero tam- moria episódica?:
bién que el estímulo no aten- dido podía mantener alguna
de sus características sensoriales más gruesas en el recuerdo 1. Las claves que están más fuertemente relacionadas se-
del sujeto (por ejemplo, el cambio de voz), aunque no su mánticamente con el contenido que queremos recuperar.
significado (opción 1, 2 y 3 incorrectas; opción 4 correcta). 2. Las claves que, además de al contenido que queremos
Este paradigma permitió sentar la base de algunos modelos recuperar, están asociadas a más contenidos diferentes de
de filtro precategorial como los de Broadbent y Treisman. nues- tra memoria episódica.
APIR Manual de Psicología Básica, Tema 2. 3. Las claves a las que se presta menos atención durante la
recuperación.
166. ¿Qué dos condiciones o pistas (señales) se utilizan 4. Las claves que estuvieron presentes durante la codifi- ca-
en el Test de las redes atencionales (Attentional Network ción y fueron codificadas junto con el contenido que ahora
Test, ANT), para calcular la puntuación correspondien- queremos recuperar.
te a la red de orientación?:
Respuesta correcta:4
1.La pista (señal) central, y la pista (señal) espa cial o peri-
férica. “En la hipótesis de la especificidad de codificación de Tul-
2.La pista (señal) central, y la doble pista (se ñal). ving, se rechaza la idea de que las palabras tienen su repre-
3.La pista (señal) doble, y la no pista (no señal). sentación fí- sica fija, proponiendo que existe especificidad
4. La no pista (no señal), y la pista (señal) espa cial o peri- de la codificación de un término en cada situación parti-
férica. cular. Por ello, sostiene que cada vez que se presenta una
palabra se codifica de manera es- pecífica en cada situa-
Respuesta correcta:1 ción. Según defiende esta hipótesis, el ren- dimiento en el
recuerdo será mejor cuando se utiliza el mismo indicio en
El Test de las Redes Atencionales (ANT) fue elaborado con la codificación. Es decir, que nos resultará más sen- cillo
el fin de utilizarse para evaluar las redes atencionales pro- recordar claves que estuvieron presentes en la codificación
puestas por Posner: 1) Red de alerta: Encargada de mante- del recuerdo que tratamos de recuperar, y que no necesa-
ner el estado de vigilia y activación del organismo; 2) Red riamente deben estar relacionadas semánticamente con el
de orientación: Que dirige la atención a los estímulos, de contenido (op- ción 4 correcta). APIR Manual de Psicolo-
forma voluntaria o involun- taria; 3) Red de control ejecuti- gía Básica, Tema 3. “
vo: Controla procesos ejecutivos como la inhibición de in-
formación distractora o la supervisión de la propia conduc- 185. En el campo de la psicología de la memoria, el para-
ta. En el ANT el sujeto debe indicar si una flecha (estímulo digma de la práctica en la recuperación se ha empleado
objetivo) que aparece en pantalla apunta a la izquierda o para estudiar el fenómeno del “olvido inducido por la
la derecha, midiéndose el tiempo de reacción. Esta flecha recupera- ción”. Señale en qué consiste este fenómeno:
puede aparecer acompañada de flechas que apuntan en la
misma dirección (flanqueo congruente), o de flechas que 1. Practicar la recuperación de cierta información a partir
apun- tan en dirección contraria (flanqueo incongruente). de una determinada clave de recuperación puede hacer que,
Adicional- mente, puede presentarse acompañada o no de pos- teriormente, esa información sea más fácil de recupe-
pistas/señales se- gún la red que se esté tratando de eva- rar.
luar: 1) Para evaluar la red de alerta se resta el tiempo de 2. Practicar la recuperación de cierta información a partir
respuesta medio en la condición de pista doble del tiempo de una clave de recuperación puede hacer que, posterior-
de respuesta medio en la condición de no pista (opción 3 mente, otros contenidos asociados a esa misma clave de re-
incorrecta); 2) Para evaluar la red de orien- tación se resta cuperación sean más fáciles de recuperar.
el tiempo de respuesta medio en la condición de pista espa- 3. Practicar la recuperación de una información a partir de
cial del tiempo de respuesta medio en la condición de pista una determinada clave de recuperación puede hacer que,
central (opción 1 correcta); 3) Para evaluar la red de control poste- riormente, otros contenidos asociados a otras claves
ejecutivo se resta el tiempo de respuesta medio de todas las de recupe- ración sean más difíciles de recordar.
si- tuaciones de flanqueo congruente del tiempo de respues- 4. Practicar la recuperación de información a partir de una
ta medio en todas las situaciones de flanqueo incongruente. clave de recuperación puede hacer que, posteriormente, el
APIR Manual de Psicología Básica, Tema 2. estí- mulo empleado como clave de recuperación, sea más
difícil de recuperar si se pide su recuerdo.

Respuesta correcta:0

19
ANULADA. El fenómeno del olvido inducido por la recu- PSICOLOGÍA CLÍNICA
pera- ción hace referencia al hecho de que al intentar recu-
perar cierta información a partir de una clave de recupera-
ción puede ocurrir que, posteriormente, otros contenidos 55. Según el Modelo transteórico del cambio de Pro-
asociados (semánticamen- te) a esa misma clave de recu- chaska y Di Clemente (1984) ¿en qué fase estaría una
peración sean más difíciles de recu- perar. En la opción de perso- na que evalúa la posibilidad de cambiar?:
respuesta 1 se expone que la información será más fácil de
recordar gracias a la clave de recuperación, no haciendo re- 1. Precontemplación.
ferencia a una mayor dificultad para recuperar contenidos 2. Contemplación.
asociados; en la opción 2 se afirma que otros conte- nidos 3. Preparación.
asociados a la clave de recuperación serán más fáciles de 4. Acción.
recuperar, proponiendo esta opción lo contrario a lo que
ocurre en el fenómeno (más difíciles); la opción 3 sostiene Respuesta correcta:2
que otros contenidos asociados a otras claves de recupera-
ción pueden ser más difíciles, pero el fenómeno del olvido El Modelo transteórico del cambio de Prochaska y Di Cle-
inducido por la re- cuperación hace referencia a contenidos men- te (1984) permite entender la conducta adictiva como
asociados a la misma clave de recuperación; la opción 4 re- un conti- nuo en la intención de abandonar una adicción.
fiere una mayor dificultad de recuperar el estímulo emplea- Es un modelo tridimensional integrado por los estadios
do (no contenidos asociados). Es decir, ninguna de las op- (“¿qué se cambia?”), los procesos (“¿cómo se cambia?”) y
ciones de respuesta hace referencia al fenómeno requerido, los niveles (“¿cuándo se cambia?”) de cambio. En concreto,
lo que contribuyó a la anulación de la el enunciado nos pregunta por el estadio en el que se en-
pregunta. APIR Manual de Psicología Básica, Tema 3. cuentra una persona que evalúa la posibilidad de cambiar.
Esta persona, que empieza a darse cuen- ta de que tiene un
6. Nos enfadamos cuando nuestro equipo de fútbol pier- problema, y comienza a buscar información y se plantea un
de una liga, nos alegramos cuando nuestro mejor amigo cambio (que suele ser en los próximos 6 meses), se encuen-
consigue el trabajo que anhelaba y sufrimos cuando al- tra en el estadio de contemplación (opción 2 correcta). En
guien querido nos deja. Para Frijda, la razón que expli- este estadio, los sujetos son conscientes pero no han reali-
ca estas emociones es la: za- do aún ningún intento de abandono de la conducta y
tampoco hay compromiso con ningún tratamiento. El resto
1. Ley del Interés. de opciones de respuesta serían incorrectas. El estadio de
2. Ley de la realidad aparente. precontemplación se caracteriza por el hecho de que la con-
3. Ley de la conservación del momento emocional. ducta no se ve como un problema, por lo que la persona
4. Ley del cierre manifiesta escasos deseos de cambiarla en los próximos 6
meses. En este estadio los pacientes no son conscientes del
Respuesta correcta:1 problema y acuden de forma obligada a consulta. La perso-
na descrita en el enunciado no se encontraría
Frijda propuso 11 leyes para poder estudiar la estructura en este punto, pues ya evalúa la posibilidad de cambiar
de la experiencia emocional. Cuatro de ellas aparecen en (porque empieza a considerar que su conducta puede ser un
las opciones de respuesta: Ley del interés, Ley de la realidad problema) (opción 1 incorrecta). El estadio de preparación
aparente, Ley de la conservación del momento emocional y para la acción se caracteriza por la decisión y el compromi-
Ley del cierre. La ley del interés se refiere al hecho de que las so. El objetivo de la persona es modificar su conducta en los
emociones surgen en respuesta a acontecimientos que son próximos 30 días. Ade- más, ha intentado dejarlo durante
importantes para la per- sona (opción 1 correcta); la ley de al menos 24 horas en el último año (opción 3 incorrecta).
la realidad aparente sostie- ne que las emociones aparecen La persona descrita en el anunciado se plantea la posibili-
ante acontecimientos evaluados como reales, con mayor in- dad de cambiar, pero todavía no se ha de- cidido a hacer
tensidad cuanto mayor sea el grado de realidad (opción 2 un cambio a corto plazo. En el estadio de acción, la perso-
incorrecta); la ley de la conservación del momento emocio- na inicia activamente la modificación de la conducta con
nal hace referencia a que los fenómenos emo- cionales en éxito (opción 4 incorrecta). Para poder pasar a una etapa
forma de recuerdo permiten elicitar la emoción a lo largo de man- tenimiento han de transcurrir al menos 6 meses.
del tiempo, a menos que una exposición repetida ocasione En este estadio podría encontrarse un sujeto que acude por
habituación o extinción (opción 3 incorrecta); por último, primera vez a las se- siones con el psicólogo clínico para
la ley del cierre afirma que las emociones tienden a formar dejar de beber y cumple con las recomendaciones del tera-
una idea absolutista de la realidad (opción 4 incorrecta). peuta (PIR 17, 113). APIR Manual Psicología Clínica, Tomo
APIR Manual de Psicología Básica, Tema 7. I, Tema 3.

20
119. ¿Cuáles son los principales efectos de las anfetami- gica y física. Concretamente, el DSM-5 (Criterio B) exige
nas?: para el diagnóstico de abstinencia a opiáceos tres o más de
los siguientes: estado de ánimo disfórico, náuseas o vómi-
1. Elevación del estado de ánimo y disminución de la sen- tos, dolores musculares, lagri- meo o rinorrea, dilatación
sación de fatiga y del apetito. pupilar, piloerección o aumento de la sudoración, diarrea,
2. Después de los iniciales efectos eufóricos en poco tiem- bostezos, fiebre o insomnio. Cabe destacar que el DSM-5
po le sigue un claro síndrome amotivacional junto con su- señala que los primeros síntomas de la abstinencia a los
presión del sueño normal durante semanas. opiáceos son subjetivos y consisten en quejas sobre ansie-
3. El consumo produce un sueño excesivo. dad, inquietud y una sensación de dolor, junto con irritabi-
4. Estado de completo bienestar, euforia, sensación de flo lidad y un incremento de la sensibilidad al dolor. Además
tar y desaparición de las inhibiciones. también se hace mención a que signos como la piloerección
y la fiebre están asociados a una abstinencia más grave y que
Respuesta correcta:1 no se ven a me- nudo en la práctica clínica habitual (opción
3 incorrecta). Por último, la alternativa de respuesta 4 hace
Las anfetaminas son estimulantes del sistema nervioso cen- referencia al síndrome amotivacional (descrito por Camí en
tral. Sus efectos son similares a los de la cocaína, pero más 1988) propio del consumo de cannabis (opción 4 incorrec-
durade- ros. Producen una elevación del estado de ánimo, ta). APIR Manual de Psicología Clínica, Tomo I, Tema 3.
disminución de la sensación de fatiga y del apetito (opción
1 correcta; opción 3 incorrecta). La opción 2, menciona el 122. La capacidad que tiene una sustancia de producir
síndrome amotivacional, que está más relacionado con el dependencia en los que la consumen, recibe el nombre
consumo de cannabis o THC (opción 2 incorrecta). Por úl- de:
timo, la opción 4 (estado comple- to de bienestar, euforia,
sensación de flotar y desaparición de las inhibiciones) hace 1. Dependencia física.
referencia a los efectos que producen los opiáceos (opción 2. Dependencia psicológica.
4 incorrecta). APIR Manual de Psicología Clí- nica, Tomo 3. Aumento de la tolerancia.
I, Tema 3. 4. Potencial adictivo de una droga.

121. De las drogas existentes, el consumo regular de he- Respuesta correcta:4


roína y morfina se caracteriza porque:
El potencial adictivo de una droga hace referencia a la pro-
1. Desarrollan rápidamente tolerancia y dependencia en pen- sión que tiene la sustancia de crear adicción/depen-
los consumidores. dencia (op- ción 4 correcta). La dependencia a una sustan-
2. Los daños en la salud sólo se producen a largo plazo. 3. cia hace referencia a el patrón de consumo desadaptativo
Producen una fuerte dependencia física pero no psico- ló- que conlleva un malestar significativo a nivel psicológico
gica. (dependencia psicológica) y a nivel físico (dependencia fí-
4. Producen un característico estado amotivacional en sus sica) a lo largo de un amplio periodo de tiempo (opciones
consumidores. 1 y 2 incorrectas). La tolerancia se refiere al estado de adap-
tación que se caracteriza por la disminución de la respuesta
Respuesta correcta:1 del organismo a la misma cantidad de droga o una necesi-
dad de incrementar la dosis para obtener el mismo efecto
Los sujetos que realizan un consumo regular de heroína y (opción 3 incorrecta). APIR Manual de Psicología Clínica,
mor- fina se caracteriza porque desarrollan rápidamente Tomo I, Tema 3.
tolerancia y dependencia (opción 1 correcta). Así mismo,
el trastorno por consumo de opiáceos puede comenzar a 164. El modelo de condicionamiento de la tolerancia a
cualquier edad, pero los problemas asociados con el uso de las drogas atribuye la tolerancia:
opiáceos se observan con mayor frecuencia en la adolescen-
cia tardía o en los primeros años de la veintena. Además, 1. Al efecto compensatorio de la homeostasis. 2
cabe tener en cuenta que el aumen- to de la edad se asocia 2. A la sobreexposición al estímulo incondicionado (dro-
con una disminución en la prevalencia, como resultado de ga) que produce extinción del placer asociado.
la mortalidad inicial (opción 2 incorrecta) y de la remisión 3. Al contracondicionamiento de las influencias positivas
de los síntomas después de los 40 años. Otros datos curio- del entorno familiar y social para que se deje la droga.
sos a tener en cuenta son: que el curso del trastorno por 4. A las respuestas condicionadas a los estímulos del en-
consumo de opiáceos tiende a ser abrupto con remisiones y torno que se asocian con la administración de la droga.
recaídas. Las tasas de mortalidad a largo plazo pueden ser
de hasta un 2 % por año y alrededor del 20-30 % de las per- Respuesta correcta:4
sonas con este trastorno logra la abstinencia a largo plazo.
Por otro lado, los opiáceos producen dependencia psicoló- Entendemos como “tolerancia” al estado de adaptación ca-

21
rac- terizado por la disminución de la respuesta del orga- los factores de buen pronóstico, tanto el DSM-IV-TR como
nismo a la misma cantidad de droga o por la necesidad de el DSM-5 inclu- yen como características de buen pro-
una dosis ma- yor para conseguir el mismo efecto. Si nos nóstico para el trastorno esquizofreniforme: la aparición
preguntan acerca del condicionamiento de la tolerancia, en- de los síntomas psicóticos en las primeras cuatro semanas
tenderemos que hacen re- ferencia a la tolerancia condicio- después del cambio apreciable del comportamiento o fun-
nada (descrito por Siegel), que hablaría sobre las respuestas cionamiento habitual (opción 1 incorrec- ta); confusión o
condicionadas a los estímulos del entorno que se asocian perplejidad (opción 2 incorrecta); buen funcio- namiento
con la administración de la droga (op- ción 4 correcta). La social y laboral antes de la enfermedad; y ausencia de afecto
opción 1 se referiría al modelo de procesos oponentes de embotado y plano (opción 4 incorrecta). Por tanto, el DSM-
Solomon y Corbit, que habla de la tolerancia y abstinen- 5 no recoge que sea un factor de buen pronóstico para el
cia como procesos opuestos que buscan la homeostasis del trastorno esquizofreniforme la presencia de un episodio
organismo pero no del proceso de condicionamiento en sí maniaco como desencadenante del trastorno (opción 3 co-
(opción 1 incorrecta). La opción 2 estaría sugiriendo que rrecta). El resto de alternativas son incorrectas por tratarse
la ex- posición repetida a la droga elimina el placer, lo que de factores de buen pronóstico incluidos en el DSM. APIR
sería falso (opción 2 incorrecta). La opción 3 no se ajustaría Manual de Psicología Clínica, Tomo I, Tema 4.
a ningún mo- delo concreto (opción 3 incorrecta). APIR
Manual de Psicología Clínica, Tomo I, Tema 3. 152. Señale cuál de las siguientes anomalías neuroana-
tó- micas o neurofisiológicas NO es propia de la esqui-
123. ¿Cuál NO sería una característica de buen pronós- zofrenia:
ti- co en el Trastorno esquizofreniforme?:
1. Disminución de los surcos corticales.
1. Aparición de síntomas psicóticos notables dentro de las 2. Disminución del volumen talámico.
primeras cuatro semanas. 3. Disminución de lóbulos frontales y temporales.
2. Confusión o perplejidad. 4. Disminución en el metabolismo de la glucosa en lóbulos
3. Que el episodio se desencadene con un episodio manico. frontales.
4. Ausencia de afecto aplanado o embotado.
Respuesta correcta:1
Respuesta correcta:3
En los pacientes con esquizofrenia se han descrito diferen-
Brevemente, antes de justificar por qué la alternativa de res- tes alte- raciones neurológicas (estructurales y funciona-
puesta correcta es la 3, es importante repasar los conceptos les), entre las que se encuentran: dilatación ventricular (en
fundamen- tales sobre el trastorno esquizofreniforme in- ventrículos laterales, tercer y cuarto ventrículo), aumento
cluidos en el DSM- 5. Concretamente, los síntomas caracte- de surcos corticales (opción 1 correcta), atrofia cortical
rísticos del trastorno es- quizofreniforme son idénticos a los (especialmente en zonas frontales), atrofia cerebelosa, dis-
de la esquizofrenia (Criterio A), es decir, para diagnosticar minución del volumen del tálamo (opción 2 incorrecta),
un trastorno esquizofreniforme es necesaria la presencia asimetría hemisférica invertida, disminución cere- bral
de 2 o + de los síntomas siguientes: alucinaciones, delirios, (especialmente en lóbulos frontales y temporales) (opción
discurso desorganizado, comportamien- to desorganizado 3 incorrecta), disminución craneal, hipofrontalidad o dis-
o catatónico y/o síntomas negativos. La dife- rencia entre la minución del metabolismo de glucosa en el lóbulo frontal
esquizofrenia y el trastorno esquizofreniforme se halla en el (opción 4 inco- rrecta). APIR Manual de Psicología Clínica,
criterio B que hace referencia a su duración (entre 1 y 6 me- Tomo I, Tema 4.
ses, en el caso del trastorno esquizofreniforme). Otra de las
características distintivas del trastorno esquizofreniforme 53. El tratamiento psicológico de trastornos como la de-
respecto a la esquizofrenia es que no se requiere deterioro presión, en casos en los que puede ir acompañada de
en el funcionamiento social y laboral, pudiendo estar pre- una disfunción sexual, es especialmente importante tra-
sente este deterioro pero sin ser necesariamente un criterio tar los problemas que tienen que ver con:
obligado para el diagnóstico. El diagnóstico de trastorno es-
quizofreniforme se realiza en dos situaciones: 1) Cuando la 1. Bajo deseo sexual.
duración del episodio de la enfermedad tiene una duración 2. Problemas de excitación.
entre 1 y 6 meses; y 2) Cuando un individuo tiene síntomas 3. Eyaculación precoz.
durante menos de los 6 meses reque- ridos para diagnos- 4. Anorgasmia.
ticar una esquizofrenia pero todavía no se ha recuperado.
En este caso, el diagnóstico debería ser “trastorno esquizo- Respuesta correcta:1
freniforme (provisional)”, puesto que no es seguro que el
individuo vaya a recuperarse del trastorno dentro del pe- En los trastornos depresivos, la aparición de cambios físicos
ríodo de 6 meses. Si los síntomas duraran más de 6 meses se es habitual y suele ser uno de los motivos principales por
debería cambiar el diagnóstico a esquizofrenia. En cuanto a los que se solicita la ayuda de un profesional. Un síntoma

22
típico, que afecta a un 70-80% de los pacientes depresivos, malmente y persistentemente elevado, expansivo o irrita-
es los problemas de sueño. Otros síntomas físicos comunes ble, y un aumento anormal y persistente de la actividad o la
son la fatiga, la pérdida de apetito y una disminución de energía junto a, por ejemplo, disminución de la necesidad
la actividad y el deseo sexuales (Opción 1). Es decir, como de dormir, sentimientos de grandeza, verborrea,...) (opción
parte del estado general de anhedo- nia, anergia, etc., las 2 incorrecta). La opción 3 es incorrecta porque el trastorno
conductas y deseos sexuales se ven casi siempre afectados explosivo intermitente se caracte- riza más por conductas
negativamente. BELLOCH, A., SANDÍN, B., RAMOS, F. agresivas como problemas de control de impulsos que por
(2009): Manual de psicopatología (2.a ed., Vol. 2, Pág. 254). irritabilidad constante más propia del trastorno de disregu-
Madrid, España: McGraw-Hill. APIR Manual de Psi- colo- lación disruptiva de estado de ánimo (opción 3 inco- rrec-
gía Clínica, Tema ta). APIR Manual de Psicología Clínica, Tomo I, Tema 5.

86. Un niño de 8 años presenta accesos de cólera fre- 128. Ante la sospecha de presencia de depresión en una
cuentes, de tres a cuatro por semana, rompe cosas y se persona ¿cuál de los siguientes aspectos relacionados
en- fada mucho, está irritado en general, y estas carac- con el diagnóstico habría que abordar antes?:
terísticas son más frecuentes en casa, aunque típicas
también en el co- legio. ¿Qué trastorno presenta?: 1. La existencia de una etiología médica u orgánica.
2. La posible comorbilidad con trastornos de ansiedad.
1. Trastorno depresivo mayor. 3. La especificación de las distorsiones cognitivas presentes
2. Trastorno bipolar. en el caso.
3. Trastorno explosivo intermitente. 4. La realización del análisis funcional para la explicación
4. Trastorno de desregulación disruptiva del estado de áni- del origen y mantenimiento del trastorno.
mo.
Respuesta correcta:1
Respuesta correcta:4
Previamente al diagnóstico de cualquier trastorno mental o
El caso descrito en el enunciado se podría corresponder pato- logía es necesario realizar un proceso de diagnostico
con el trastorno de desregulación disruptiva del estado de diferencial. Concretamente, en el caso de una sospecha de
ánimo (op- ción 4 correcta). Señalemos los criterios de di- depresión en una persona, es relevante, tal y como estable-
cho trastorno que se cumplen en este enunciado: Un niño cen los criterios D y E del DSM-5, descartar que el episodio
de 8 años [CRITERIO G. El primer diagnóstico no se debe depresivo no se explique mejor por un trastorno psicótico
hacer antes de los 6 años o después de los 18 años; CRI- (esquizoafectivo, esquizofre- niforme, delirante) y descartar
TERIO H. Por la historia o la obser- vación, los Criterios que no se pueda atribuir a los efectos fisiológicos de una
A-E comienzan antes de los 10 años] pre- senta accesos de sustancia o de otra afección médica (opción 1 correcta). Por
cólera frecuentes [CRITERIO A. Accesos de cólera graves y otro lado, la presencia de un trastorno de ansiedad no es
recurrentes que se manifiestan verbalmente (p. ej., rabietas excluyente de la posibilidad de diagnosticar un trastorno
verbales) y/o con el comportamiento (p. ej., agresión física depresivo mayor, es una característica altamente asociada,
a personas o propiedades) cuya intensidad o duración son motivo por el cual se incluye como un posible espe- cifi-
desproporcionadas a la situación o provocación], de tres a cador en el DSM-5 y es uno de los trastornos que apare-
cuatro por semana [CRITERIO C. Los accesos de cólera se cen frecuentemente de forma simultanea con la depresión
producen, en término medio, tres o más veces por sema- (opción 2 incorrecta). Respecto a la alternativa 4, la reali-
na], rompe cosas y se enfada mucho, está irritado en gene- zación de un análisis funcional que permita la explicación
ral [CRITERIO D. El estado de ánimo entre los accesos de del origen y el mantenimiento del trastorno, podría ser una
cólera es persistentemente estrategia muy va- liosa para la formulación del caso y del
irritable o irascible la mayor parte del día, casi todos los tratamiento, si bien no es una condición previa a abordar
días, y es observable por parte de otras personas (p. ej., pa- ante la sospecha de la presencia de depresión (opción 4 in-
dres, maes- tros, compañeros)], y estas características son correcta). Finalmente, las distorsiones cognitivas (muchas
más frecuentes en casa, aunque típicas también en el co- de ellas descritas por Beck) son frecuentes en la depresión y
legio [CRITERIO F. Los Criterios A y D están presentes al no excluyen la posibilidad de diagnosticar la misma ni de-
menos en dos de tres con- textos (es decir, en casa, en la ben de ser consideradas previamente al diagnóstico de de-
escuela, con los compañeros) y son graves al menos en uno presión (opción 3 incorrecta). APIR Manual de Psicología
de ellos]. La opción 1 es incorrecta porque no aparece irri- Clínica, Tomo I, Tema 5.
tabilidad (propia en niños) dentro del resto de síntomas de-
presivos (ej.: disminución o aumento de apetito, insomnio o 129. Según la Teoría de los estilos de respuesta (No-
hipersomnia, baja autoestima,...) característicos del trastor- len-Hoksema), el elemento fundamental para la consoli-
no depresivo mayor (opción 1 incorrecta). La opción 2 es da- ción de un trastorno depresivo sería:
incorrecta porque no se cumplen criterios para un episodio
ma- níaco (período bien definido de estado de ánimo anor- 1.La vivencia de acontecimientos de pérdida.

23
2. La limitación en las capacidades de afrontamiento de la aparición de la depresión.
esas pérdidas. 2. El incremento en la neurotransmisión serotoninérgica
3.La baja autoestima. lleva a la aparición de la depresión.
4. Las respuestas rumiativas. 3. El déficit en la neurotransmisión serotoninérgica pre-
dispone a la aparición de la depresión, presentándose el
Respuesta correcta:4 estado depresivo si hay además un déficit en la neurotrans-
misión cate- colaminérgica.
La teoría de los estilos de respuesta (Nolen-Hoeksema) pro- 4. El déficit en la neurotransmisión serotoninérgica pre-
pues- ta en 1991, se centra en los factores que influyen en dispone a la aparición de la depresión, presentándose el
el curso de la depresión. Esta teoría sugiere que un estilo de estado depresivo si hay además un incremento en la neuro-
respuesta con tendencia a la rumiación mantendría y agra- transmisión catecolaminérgica.
varía los síntomas del trastorno depresivo (opción 4 correc-
ta), en contraposición con un estilo más activo que tiende a Respuesta correcta:3
la distracción o la solución de problemas. Según esta autora,
el origen del estilo rumiativo es- taría en los primeros años La hipótesis permisiva de la depresión plantea que el défi-
de crianza del sujeto (aprendizaje por modelado y prácticas cit en la neurotransmisión serotoninérgica predispone a la
de sociabilización). (opciones 1, 2 y 3 in- correctas). APIR aparición de la depresión a través de la oscilación de otros
Manual de Psicología Clínica, Tomo I, Tema 5. neurotransmisores. Concretamente, promoviendo una dis-
minución de la neurotrans- misión catecolaminérgica en
130. Según el DSM-5, ¿cuál de las siguientes es una el caso de la depresión (opción 3 correcta; opción 2 inco-
característica definitoria de los trastornos depresivos rrecta). En contraposición, si el déficit en la neurotransmi-
melancó- licos durante la fase más grave del episodio sión serotoninérgica predispusiera a un incre- mento de la
actual?: neurotransmisión catecolaminérgica, probablemen- te la
sintomatología que observaríamos sería compatible con un
1. Reactividad o aumento del estado de ánimo en respues ta episodio maniaco (opción 4 incorrecta). Finalmente, la op-
a sucesos positivos reales o potenciales. ción 1 hace referencia al déficit serotoninérgico hallado en
2. Pérdida de placer por todas o casi todas las actividades. pacientes con trastornos depresivos por el cual se sugiere
3. Aparición y remisión de episodios depresivos en deter que niveles bajos de serotonina desempeñan un papel re-
minadas épocas del año. levante en la etiología del trastorno. El dato que ofrece está
4. Sensación de nervios o tensión. alternativa, aunque es correc- to, no se corresponde con el
aspecto preguntado en el enuncia- do (hipótesis permisi-
Respuesta correcta:2 va de la depresión) (opción 1 incorrecta). APIR Manual de
Psicología Clínica, Tomo I, Tema 5.
El DSM-5 incluye en el especificador de “con características
melancólicas” aplicable a los trastornos depresivos. Incluye 136. Señale la afirmación correcta con respecto a las ta-
dos criterios: A. Presencia de una de las siguientes caracte- sas de suicidio en relación con los trastornos del estado
rísticas du- rante el período más grave: 1) Pérdida de placer de ánimo:
por todas o casi todas las actividades (opción 2 correcta).
2) Falta de reactividad a estímulos generalmente placente- 1. Son mayores en depresión mayor que en trastorno bi-
ros (opción 1 incorrecta); B. Tres o más de las siguientes polar.
características: 1) Estado de ánimo deprimido con deses- 2. Son similares en depresión mayor y en trastorno bipo lar.
peranza y desaliento profundo. 2) Empeo- ramiento de la 3. Son mayores en trastorno bipolar que en depresión ma
depresión por la mañana. 3) Despertar temprano. 4) Agi- yor.
tación o retraso psicomotor. 5) Anorexia o pérdida de peso 4. Los datos provienen de estudios que no permiten afir mar
importante. 6) Culpa excesiva o inapropiada. La alternativa nada al respecto.
de respuesta 3, aparición y remisión de episodios depresi-
vos en de- terminadas épocas del año, hace referencia a un Respuesta correcta:3
síntoma incluido en el especificador “con patrón estacional”
(opción 3 incorrecta). Por último, la alternativa de respues- Según refieren Belloch, A., Sandín, B. y Ramos, F. (2008) en
ta 4, sensación de nervios o tensión, hace referencia a un su Manual de Psicopatología, el riesgo de suicidio es ma-
síntoma del especificador “con características de ansiedad” yor en sujetos con trastornos bipolares que en sujetos con
(opción 4 incorrecta). APIR Manual de Psicología Clínica, trastornos depresivos (respuesta 3 correcta; respuestas 1 y 2
Tomo I, Tema 5. incorrectas). Estos datos están extraídos de estudios epide-
miológicos de los trastornos afectivos (opción 4 incorrec-
131. Según la hipótesis permisiva de la depresión: ta). Según DSM-5, se es- tima que el riesgo de suicidio a
lo largo de la vida de los pacientes con trastorno bipolar es
1. El déficit en la neurotransmisión serotoninérgica lleva a 15 veces superior al de la población general (siendo, según

24
Belloch (2008), cuatro veces mayor que el de los pacientes tes de la actividad del individuo. F. Estas alteraciones no se
con trastorno depresivo mayor recurrente). deben a los efectos fisiológicos directos de una sustancia (p.
De hecho, el trastorno bipolar supone 1/4 de todos los suici- ej., drogas, fármacos) o a una enfermedad mé- dica (p. ej.,
dios consumados. La historia previa de intentos de suicidio hipertiroidismo) y no aparecen exclusivamente en el trans-
y el por- centaje de días con depresión en el último año se curso de un trastorno del estado de ánimo, un trastorno
asocian a un mayor riesgo de intentos suicidas o suicidios psi- cótico o un trastorno generalizado del desarrollo. APIR
consumados. El riesgo de suicidio es mayor en el trastorno Manual de Psicología Clínica, Tomo I, Tema 6.
bipolar II y alrededor un 1/3 de ellos refiere antecedentes de
intento de suicidio. Las tasas de prevalencia vital del intento 137. Según DSM 5, una persona con un familiar de pri
de suicidio en el trastorno bipolar tipo II y en el tipo I pa- mer grado con una fobia específica a los animales es sig-
recen similares, pero la letalidad es mayor en los pacientes nifi- cativamente más propensa a tener:
tipo II. Podría existir asociación entre marcadores genéti-
cos y el aumento del riesgo de suicidio en trastorno bipolar. 1. La misma fobia específica.
Manual de Psicología Clínica, Tomo I, Tema 5. 2. Cualquier otra fobia específica.
3. Cualquier otra fobia.
110. Respecto de los criterios diagnósticos para el tras- 4. Cualquier otro trastorno de ansiedad.
torno de ansiedad generalizada, el DSM-5 establece
como duración mínima de los síntomas nucleares (seña- Respuesta correcta:1
le la res- puesta correcta):
Según DSM-5, podría haber una susceptibilidad genética a
1. Dos semanas. una determinada categoría de fobia específica (siendo, por
2. Un mes. tanto, una persona con un familiar de primer grado con
3. Tres meses. una fobia espe- cífica significativamente más propensa a
4. Seis meses. tener la misma fobia específica) (opción 1 correcta; opción
2 incorrecta). Es cierto que la fobia específica presenta una
Respuesta correcta:4 comorbilidad elevada con otros trastornos de ansiedad, in-
cluidas las fobias. Sin embargo, a nivel de riesgo genético en
Los criterios del trastorno de ansiedad generalizada en el familiares de primer grado, destaca especialmente la sus-
DSM-5 son los siguientes: A. Ansiedad y preocupación ex- ceptibilidad a un determinado tipo de fo bia (opciones 3 y 4
cesivas (anti- cipación aprensiva) sobre una amplia gama incorrectas). Manual de Psicología Clínica, Tomo I, Tema 6.
de acontecimientos o actividades (como el rendimiento la-
boral o escolar), que se prolonga más de 6 meses (opción 4 140. Las falsificaciones del recuerdo que aparecen en un
correcta; opciones 1, 2 y 3 incorrectas). B. Al individuo le estado de lucidez de conciencia asociadas a una amnesia
resulta difícil controlar este es- tado de constante preocu- or- gánica y sin que el paciente tenga la intención de
pación. C. La ansiedad y preocupación se asocian a tres (o mentir se denomina:
más) de los seis síntomas siguientes (algunos de los cuales
han persistido más de 6 meses): 1. Inquietud o im- pacien- 1. Pseudologías o seudologías fantásticas.
cia; 2. Fatigabilidad fácil; 3. Dificultad para concentrarse o 2. Falsos reconocimientos.
tener la mente en blanco; 4. Irritabilidad; 5. Tensión mus- 3. Confabulaciones.
cular; 6. Alteraciones del sueño (dificultad para conciliar o 4. Paramnesias reduplicativas.
mantener el sueño, o sensación al despertarse de sueño no
reparador). D. El centro de la ansiedad y de la preocupación Respuesta correcta:3
no se limita a los síntomas de un trastorno del Eje I; por
ejemplo, la ansiedad o preocupación no hacen referencia Todas las opciones de respuesta hacen referencia a distor-
a la posibilidad de presentar una crisis de angustia (como siones de la memoria. La pseudología fantástica y las con-
en el trastorno de angustia), pasarlo mal en público (como fabulaciones se clasificarían como anomalías del recuerdo.
en la fobia social), contraer una enferme- dad (como en el En ambas condi- ciones se hace referencia al mismo fe-
trastorno obsesivo-compulsivo), estar lejos de casa o de los nómeno: la fabricación de recuerdos para rellenar lagu-
seres queridos (como en el trastorno de ansiedad por se- nas mnésicas. El término confabu- lación se reserva para
paración), engordar (como en la anorexia nerviosa), tener aquellas falsificaciones que se dan en el síndrome amnési-
quejas de múltiples síntomas físicos (como en el trastorno co con estado de conciencia lúcida, en las que el pacien-
de so- matización) o padecer una enfermedad grave (como te puede inventarse recuerdos, sin intención de mentir, al
en la hipo- condría), y la ansiedad y la preocupación no intentar dar respuesta a cosas que no recuerda y tener así
aparecen exclusiva- mente en el transcurso de un trastorno una continuidad mnémica (o narrar recuerdos auténticos
por estrés postraumático. E. La ansiedad, la preocupación o pero mal contextualizados). Suele aparecer en el Síndrome
los síntomas físicos provocan malestar clínicamente signifi- de Korsakoff y en algunas demencias (respuesta 3 correcta).
cativo o deterioro social, laboral o de otras áreas importan- El término de pseudología fantástica se aplica en aquellos

25
pacientes que pre- sentan una sintomatología histérica, o 1. Situacionales.
al menos no orgánica. En este caso, la alteración se refie- 2. Entorno natural.
re a hechos o narraciones com- pletamente inventadas o 3. Animal.
fantaseadas (que llegan a creerse ellos mismos) a causa de 4. Sangre-inyección-heridas.
una necesidad afectiva. Suelen ser experi- mentadas en su Respuesta correcta:1
mayor parte por pacientes histéricos, con carac- terísticas
histriónicas, siendo también muy frecuentes en el Sín- dro- Según el DSM 5, la fobia específica generalmente se desa-
me de Münchausen (opción 1 incorrecta). Por otro lado, rrolla en la primera infancia, antes de los 10 años, entre los
los falsos reconocimientos y las paramnesias reduplicativas 7 y los 11 (siendo la edad media de unos 10 años). Las situa-
se clasi- ficarían como anomalías del reconocimiento. Los cionales pre- sentan una edad de inicio más tardía (opción
falsos recono- cimientos podrían ser positivos o negativos. 1 correcta) que las fobias específicas del entorno natural,
En el déjà vu o falso reconocimiento positivo, se daría una animal o a la sangre-in- yección-herida (opciones 2, 3 y 4
sensación de familiaridad inapropiadamente intensa (es de- incorrectas). Las que se inician en la infancia y la adoles-
cir, algo nuevo se experimenta como si ya se hubiera vivi- cencia suelen sufrir altibajos durante ese periodo. Las que
do), y en el jamais vu o falso reconoci- miento negativo, tal persisten en la edad adulta no suelen remi- tir. A pesar de
sensación estaría ausente o atenuada, lo cual llevaría a una que se suelen desarrollar en la infancia y la ado- lescencia,
ausencia de reconocimiento (es decir, se reconoce y recuer- pueden iniciarse a cualquier edad, normalmente tras expe-
da una situación, pero no se experimentaría sensación de riencias traumáticas. APIR Manual de Psicología Clínica,
familiaridad) (opción 2 incorrecta). Por último, las param- Tomo I, Tema 6.
ne- sias reduplicativas serían el otro extremo del déjà vu en
continuo de experiencias de familiaridad inapropiada, que 109. ¿Qué factor de riesgo temperamental se relaciona
consistiría en afirmar que se estuvo en el mismo lugar an- con el trastorno obsesivo compulsivo (DSM-5)?:
teriormente, siendo la primera vez que se está. Es decir, un
paciente puede afirmar que 1.Extraversión.
estuvo en un hospital exactamente igual al que está ahora, 2. Reducida emotividad positiva.
o que ya conocía a los enfermeros (un falso reconocimiento 3. Comportamiento perturbador.
muy fre- cuente en el Síndrome de Korsakoff, demencias y 4. Síntomas de interiorización.
estados con- fusionales) (opción 4 incorrecta). Manual de
Psicología Clínica, Tomo I, Tema 1. Respuesta correcta:4

179. ¿Qué tipo de ataque de pánico de la tipología de Según el DSM-5, hay diferentes factores de riesgo y pronós-
Barlow se corresponde con el ataque de pánico limitado tico para padecer trastorno obsesivo compulsivo (TOC),
si- tuacionalmente propuesto por Klein?: entre ellos: 1) Temperamentales: los síntomas de interiori-
zación (opción 4 correcta), la mayor emotividad negativa
1. Ataque de pánico señalado/no esperado. (opción 2 incorrecta) y la inhibición del comportamiento
2. Ataque de pánico no señalado/no esperado. en la infancia (opción 3 inco- rrecta); 2) Ambientales: el
3. Ataque de pánico señalado/esperado. maltrato físico y sexual en la infancia y otros sucesos es-
4. Ataque de pánico espontáneo. tresantes o traumáticos. Algunos niños pueden desarrollar
repentinamente síntomas obsesivo-compulsivos que se han
Respuesta correcta:3 asociado con diferentes factores ambientales, incluidos va-
rios agentes infecciosos y un síndrome autoinmune posin-
La correspondencia de la clasificación de Barlow con la de fec- cioso; 3) Genéticos y fisiológicos: la tasa de TOC en
Klein es la siguiente: los ataques señalados/esperados de los familia- res de primer grado de los adultos con TOC es
Barlow serían los situaciones de Klein (opción 3 correcta); aproximadamente 2 veces mayor que entre los familiares de
los ataques señala- dos/no esperados de Barlow serían los primer grado de los que no tienen el trastorno; sin embargo,
ataques predispuestos de Klein (opción 1 incorrecta); los entre los familiares de primer grado de los individuos con
ataques no señalados/esperados de Barlow no tendrían un inicio del TOC en la niñez o en la adolescencia, la tasa
equivalencia a los de Klein; y los ataques no señalados/no se incrementa 10 veces. Esta trans- misión familiar se debe
esperados se corresponderían con los ataques inesperados en parte a factores genéticos (p. ej., una tasa de concordan-
de Klein (opción 2 incorrecta). La opción 4 (ataque 2. de cia de 0,57 para los gemelos monocigóticos frente a 0,22
pánico espontáneo) no se corresponde con ningún ataque para los gemelos dicigóticos). La disfunción de la corteza
for- 3. mulado por este autor (opción 4 incorrecta). APIR orbitofrontal, de la corteza cingulada anterior y del cuer- po
Manual de Psicología Clínica, Tomo I, Tema 6. estriado está fuertemente implicada. La extraversión no se
considera un factor de riesgo temperamental relacionado
180. De las diferentes fobias específicas, ¿cuál presenta con el TOC (opción 1 incorrecta). APIR Manual de Psico-
una edad de aparición más tardía?: logía Clínica, Tomo I, Tema 7.

26
111. Respecto del Trastorno obsesivo compulsivo (TOC), factores. Además, es importante señalar que en los varones
cabe afirmar lo siguiente: es frecuente que el trastorno se inicie antes, es por ello que
el TOC, durante las primeras etapas del desarrollo, es más
1. Los varones presentan una edad más temprana de ini cio frecuente en varones (opción 1 incorrecta). APIR Manual
de los síntomas del trastorno que las mujeres. de Psicología Clí- nica, Tomo I, Tema 7.
2. En general, los estudios muestran que la prevalencia a lo
largo de la vida es del 3-4 %. 132. Según el DSM-5, el trastorno de estrés postrau-
3. La presencia de TOC no es universal. 4. El TOC suele máti- co se especifica cómo “con expresión retardada”
comenzar en la infancia. cuando:

Respuesta correcta:1 1. No va precedido de un trastorno de estrés agudo.


2. El inicio de los primeros síntomas se produce al menos
La prevalencia vital del TOC es de 2,5% (opción 2 inco- un mes después del acontecimiento traumático.
rrecta) y, además, la presencia del trastorno es universal 3. El inicio de los primeros síntomas se produce al menos
(opción 3 in- correcta). En el DSM-IV-TR, y hasta hace seis meses después del acontecimiento traumático.
relativamente poco, se consideraba que el TOC afectaba 4. La totalidad de los criterios diagnósticos no se cumplen
por igual a hombres y mu- jeres, sin embargo los últimos hasta al menos seis meses después del acontecimiento trau-
estudios y el DSM-5 señalan que las mujeres se ven afec- mático.
tadas con una tasa ligeramente más alta que los varones en
la edad adulta. Además, es importante señalar que en los Respuesta correcta:4
varones es frecuente que el trastorno se inicie antes (opción
1 correcta). Es por ello que el TOC, durante las primeras En esta pregunta se debe responder literalmente la defini-
etapas del desarrollo, es más frecuente en varones. Sería ción del especificador de trastorno de estrés postraumático
en la adolescencia cuando la prevalencia se igualaría entre (TEPT) “con expresión retardada” (incluida en el DSM-5).
hombres o mujeres, o incluso llegaría afectar ligeramente Concretamente, este especificador señala que únicamente
más a estas. La edad de inicio suele ser la adolescencia o se puede diagnosticar el trastorno de estrés postraumáti-
edad adulta (opción 4 incorrecta), aunque también puede co con expresión retardada si “la totalidad de los criterios
aparecer en la infancia, sien- do la edad de inicio diferente diagnósticos no se cumplen hasta al menos seis meses des-
según el sexo: entre los 6-15 años en los varones y entre los pués del acontecimiento aunque el ini- cio y la expresión
20-29 en las mujeres. APIR Manual de Psicología Clínica, de algunos síntomas puedan ser inmediatos” (opción 4 co-
Tomo I, Tema 7. rrecta; opción 3 incorrecta). En la alternativa de res- puesta
2, podemos observar como el examinador está intentando
155. Respecto del trastorno obsesivo compulsivo (TOC), confundir el criterio temporal de expresión retardada (6
señale la opción correcta: meses) con el criterio temporal general de duración míni-
ma necesaria para diagnosticar un trastorno de estrés pos-
1. Los niños están menos afectados que las niñas. traumático, es de- cir, el criterio F del DSM-5 que refiere
2. La remisión espontánea es muy alta. que: “Las alteraciones B, C, D y E (síntomas de intrusión,
3. En comparación con trastornos de ansiedad y depresión, evitación y las alteracio- nes en el nivel de alerta y las altera-
los pacientes con TOC tienen ma yor probabilidad de ser ciones cognitivas) tienen una duración superior a un mes”
solteros y estar en paro. (opción 2 incorrecta). Finalmente, respecto a la alternativa
4. El comienzo suele ser brusco/súbito como respuesta a si- de respuesta 1, podemos decir que ésta sería correcta, pero
tuaciones de estrés. teniendo en cuenta el resto de alternativas, y concretamente
la alternativa 4, es más correcto señalar como alternativa
Respuesta correcta:3 correcta la definición literal ofrecida por el DSM-5 de TEPT
con expresión retardada que se incluye en la alternativa de
En comparación con trastornos de ansiedad o depresión, respuesta 4 (opción 4 incorrecta). APIR Manual de Psicolo-
los pa- cientes con TOC tienen una mayor probabilidad de gía Clínica, Tomo I, Tema 8.
ser solteros, estar en paro (opción 3 correcta) y presentar
importantes deterio- ros en el funcionamiento ocupacional 135. La Teoría del procesamiento de la información
y social. El TOC aparece de modo gradual (opción 4 inco- apli- cada al trastorno de estrés postraumático se centra
rrecta) y tiene un curso crónico con fluctuaciones reactivas en la ex- plicación de:
a acontecimientos estresantes. En ocasiones se relaciona
con cambios vitales. La OMS sitúa el TOC dentro de una de 1. La reexperimentación del acontecimiento traumático. 2.
las diez condiciones más incapacitantes. Son poco frecuen- 2.Las conductas de evitación de los estímulos que recuer-
tes las remisiones completas (opción 2 inco- rrecta) y des- dan al trauma.
pués de una remisión puede existir recaídas produ- cidas 3. Los problemas afectivos, la pérdida de interés y la apa
por ansiedad, depresión (gran comorbilidad), fatiga u otros tía.

27
4. La hiperactivación y la ansiedad extrema. afectos de un trastorno facticio muestran el comportamien-
to engañoso para adoptar el rol de enfermo. Los métodos
Respuesta correcta:1 para falsificar la enfermedad incluyen la exageración, la fa-
bricación, la simulación y la inducción de síntomas (p. ej.,
La Teoría del procesamiento de la información propuesta falsificar prue- bas de laboratorio, registros médicos, infor-
inicial- mente por Foa y Kozak (1986), intenta ofrecer una mar engañosamente a facultativos, ingerir sustancias,...).
explicación sobre las estructuras cognitivas del miedo y los La prevalencia del trastorno facticio es desconocida, pero
síntomas de re- experimentación presentes en el trastorno se estima que afecta a un 1% de la población hospitalaria.
de estrés postraumá- tico (TEPT). Concretamente, desde El curso suele ser en forma de episodios intermitentes (op-
esta teoría se plantea que el cuadro clínico surge a partir ción 3 correcta), siendo los episodios únicos y persistentes
de sucesos aversivos (impredecibles e incontrolables), que menos frecuentes. El inicio del trastorno tiende a ser en la
no han sido procesados emocionalmente de forma adecua- edad adulta y posteriormente a una hospitalización por un
da, generando una red de miedo, y que interfie- ren en la problema médico o mental. En el caso del trastorno facticio
integración cognitiva y emocional de otras experiencias y aplicado a otro, el diagnóstico se aplica al sujeto que induce
conductas. La exposición a otros estímulos que recuerdan los síntomas y no a la victima y es necesario la demostra-
el trauma, puede activar la red de miedo y a través de la ción de que causa los síntomas (opción 4 incorrecta). APIR
propaga- ción de la activación generar síntomas de reex- Manual de Psicología Clínica, Tomo II, Tema 10.
perimentación del acontecimiento traumático (opción 1
correcta). El individuo, en este contexto, realiza una serie 116. Señale cuál de los siguientes fenómenos es menos
de estrategias de afrontamiento de evitación y escape que a propio y definitorio del trastorno de identidad disocia-
la larga contribuyen al mantenimien- to de la red de miedo. tivo (DSM 5):
Las conductas de evitación de los estímu- los que recuerdan
al trauma se explican mejor desde las teorías del aprendiza- 1. Experiencias de distanciamiento e irrealidad respecto al
je (generalización, condicionamiento de orden su- perior) entorno.
(opción 2 incorrecta). Del mismo modo, la hiperactiva- 2. Vacíos recurrentes al recordar eventos cotidianos.
ción y la ansiedad extrema, también podrían explicarse 3. Seria discontinuidad en el sentido de identidad.
desde el condicionamiento clásico (teorías del aprendizaje) 4. Alteraciones en el afecto, conducta, conciencia y me
(opción 4 incorrecta). Por último, los problemas afectivos, moria.
pérdida de interés y apatía, podrían ser explicados desde
modelos basados en la teoría de la indefensión aprendida Respuesta correcta:1
(opción 2 incorrecta). APIR Manual de Psicología Clínica,
Tomo I, Tema 8. La opción 1 (experiencias de distanciamiento e irreali-
dad res- pecto al entorno) haría referencia al trastorno de
126. ¿Cuál de las siguientes respuestas es correcta con despersonaliza- ción/desrealización, que no es propia del
respecto al trastorno facticio?: trastorno de identidad disociativo (opción 1 correcta). Los
criterios diagnósticos del trastorno de identidad disociativo
1. No hay comportamiento engañoso-fingimiento si no según el DSM-5 son los si- 1. guientes: A. Perturbación de
existe incentivo externo. la identidad que se caracteriza por 2. dos o más estados de
2. La producción de los síntomas no es intencionada. la personalidad bien definidos, que se pue- 3. de describir
3. Lo habitual es que curse en forma de episodios intermi- en algunas culturas como una experiencia de pose- 4. sión.
tentes. La perturbación de la identidad implica una discontinuidad
4. En el trastorno facticio aplicado a otro, el diagnóstico se importante del sentido del yo y del sentido de entidad (op-
aplica tanto al autor como a la víctima. ción 3 incorrecta), acompañado de alteraciones relaciona-
das del afecto, el comportamiento, la conciencia, la memo-
Respuesta correcta:3 ria, la percepción, el conocimiento y/o el funcionamiento
sensitivo-motor (opción 4 incorrecta). Estos signos y sín-
La característica esencial del trastorno facticio es la simu- tomas pueden ser observados por parte de otras personas
lación de signos y síntomas médicos o psicológicos, en uno o comunicados por el individuo; B. Lapsos recurrentes en
mismo o en otros, asociado a un engaño identificado (op- la memoria de acontecimientos cotidia- nos, información
ción 1 incorrecta) y en ausencia de una recompensa externa personal importante, y/o sucesos traumáticos incompati-
obvia. La producción de síntomas es intencionada (opción bles con el olvido ordinario (opción 2 incorrecta); C. Los
2 incorrecta) para asumir el rol de enfermo, incapacitado síntomas causan malestar clínicamente significativo o dete-
o lesionado. El diagnóstico de trastorno facticio requiere rioro en lo social, laboral u otras áreas importantes del fun-
la demostración de que el individuo está cometiendo ac- ciona- miento; D. La alteración no es una parte normal de
ciones para tergiversar, exagerar, simular, fal- sificar signos una práctica cultural o religiosa ampliamente aceptada; E.
o síntomas en ausencia de una recompensa externa obvia. Los síntomas no se pueden atribuir a los efectos fisiológicos
Aunque puede haber una afección médica preexistente, los de una sustancia (p. ej., laguna mental o comportamiento

28
caótico durante la intoxicación alcohólica) u otra afección Respuesta correcta:3
médica (p. ej., epilepsia parcial com- pleja). APIR Manual
de Psicología Clínica, Tomo II, Tema 11. Los individuos con trastorno de despersonalización/des-
reali- zación pueden tener dificultades para describir sus
117. ¿Cuál de los siguientes síntomas conductuales es síntomas y pueden pensar que están “locos” o “volviéndo-
más característico de la amnesia disociativa?: se locos”. Otra experiencia frecuente es el temor a un daño
cerebral irrever- sible. Un síntoma común asociado es una
1. Comportamiento mecánico. alteración subjetiva del sentido del tiempo (p. ej., dema-
2. Analgesia. siado rápido o demasiado lento), así como una dificultad
3.Autolesiones e intentos de suicidio. subjetiva para recordar vívida- mente las memorias del pa-
4.Macropsia o micropsia. sado y para ser dueño de las mismas, tanto personal como
emocionalmente. Los síntomas somáticos (opción 3 correc-
Respuesta correcta:3 ta) tenues, como la saturación de la cabeza, el hormigueo
o la sensación de desmayo, no son infrecuentes. Las per-
Sabiendo que la amnesia disociativa, también denominada sonas pueden sufrir una preocupación obsesiva y una ru-
am- nesia funcional/psicógena/histérica/no orgánica, cons- mia- ción marcada (opción 3 correcta) (p. ej., se obsesionan
tituye una pérdida de memoria retrógrada, episódica, explí- constan- temente sobre si realmente existen o controlan sus
cita, podemos descartar la analgesia (incapacidad de sentir percepciones para determinar si parecen reales). También
dolor) (opción 2 in- correcta), las macropsias o micropsias son características asociadas frecuentes diversos grados de
(distorsiones perceptivas en las que objetos se ven de ma- ansiedad y depresión.
yor o menor tamaño del real, respectivamente) (opción 4 Se ha observado que las personas con este trastorno tien-
incorrecta), y el comportamiento mecánico, que sería más den a tener una hiporreactividad fisiológica a los estímulos
propio de algunas alteraciones de la conciencia (opción 1 emocio- nales. Sustratos neuronales de interés son el eje hi-
incorrecta). Acerca del trastorno amnésico, es importante potálamo-hi- pófisis-suprarrenal, el lóbulo parietal inferior
saber que es el trastorno disociativo más frecuen- te, y que y los circuitos de la corteza prefrontal-límbica. La duración
su prevalencia es mayor en mujeres que en hombres. Según de los episodios del trastorno de despersonalización/des-
el DSM-5, la duración de este trastorno puede variar des- de realización puede variar am- pliamente, desde breves (ho-
minutos hasta décadas. El curso es también variable: remi- ras o días) a prolongados (semanas, meses o años) (opción
sión espontánea en la mayoría, o curso crónico. Algunos 1 incorrecta). El curso de la enfermedad es a menudo cró-
episodios de amnesia disociativa se resuelven rápidamente nico (opción 2 incorrecta). En alrededor de un tercio de los
(p.ej., cuando la persona se retira del combate o de alguna casos se trata de episodios separados, otro tercio 1. presenta
otra situación de estrés), mientras que otros episodios per- síntomas continuos desde el principio, y otra tercera parte
sisten durante largos periodos de tiempo. Algunas personas tiene un curso inicialmente episódico que con el tiempo se
pueden recordar gradual- mente los recuerdos disociados 2. convierte en continuo. Por último, la opción 4 es inco-
años más tarde. Las capacidades disociativas pueden dis- rrecta ya que el criterio B del DSM-5 refiere lo siguiente: “B.
minuir con la edad, pero no siempre. A medida que la am- Durante las 3. experiencias de despersonalización o desrea-
nesia remite, puede haber una considerable angustia, con- lización, las pruebas de realidad se mantienen intactas” (op-
ducta suicida (opción 3 correcta) y síntomas de trastorno ción 4 incorrecta). APIR 4. Manual de Psicología Clínica,
de estrés postraumático, al verse los sujetos invadidos por Tomo II, Tema 11.
recuerdos intolerables. Es por ello que las conductas auto-
destructivas, entre las que se incluyen las de tipo suicida, 127. ¿Cuál de las siguientes respuestas es correcta con
son co- munes en estos pacientes. APIR Manual de Psicolo- respecto a la bulimia nerviosa?:
gía Clínica, Tomo II, Tema 11.
1. Es más prevalente en mujeres jóvenes que la anorexia
154. Respecto del trastorno de despersonalización cabe nerviosa.
afirmar que: 2. Los atracones deben ocurrir, al menos, tres veces a la se-
mana, cuatro meses seguidos, para poder diagnosticar el
1. La duración de los “episodios” es siempre breve (no per- tras- torno.
sistentes). 3. A diferencia de lo que ocurre en la anorexia nerviosa, el
2. El curso del “trastorno” es usualmente episó dico, no cró- peso corporal no influye indebidamente en la autoevalua-
nico. ción de la persona.
3. Entre sus características clínicas figuran las rumiaciones 4. Los atracones suelen ser de alimentos que también con-
obsesivas y preocupacio sumen, normalmente, entre episodios.
nes somáticas.
4. En el transcurso del trastorno se alteran las pruebas de Respuesta correcta:1
realidad.
La prevalencia de la anorexia nerviosa en mujeres jóvenes

29
es de aproximadamente 0,4%, mientras que la prevalencia sia, terrores nocturnos y sonambulismo.
de la buli- mia nerviosa entre mujeres jóvenes es del 1-15% 2. Trastornos primarios del sueño, e incluyen pesadillas, te-
(opción 1 co- rrecta). Las tres características definitorias de rrores nocturnos y sonambulismo.
la bulimia nerviosa son los episodios recurrentes de atra- 3. Trastornos del sueño y de la motricidad, e incluyen sín-
cones, los comportamientos inapropiados y recurrentes drome de piernas inquietas y sonambulismo.
para evitar el aumento de peso y la influencia del peso y 4. Un tipo de insomnio que incluye la apnea obstructiva y
la constitución corporal en la autoevalua- ción. Esta última las pesadillas.
característica es común entre bulimia nerviosa y anorexia
nerviosa, y se recoge entre los criterios diagnósticos de am- Respuesta correcta:2
bos trastornos (opción 3 incorrecta). Para diagnosticar bu-
limia nerviosa, según los criterios DSM-5, es necesario que Al preguntarnos por parasomnias lo primero que debe-
los atracones y comportamientos compensatorios inapro- mos de hacer es conocer su definición y qué trastornos lo
piados se produzcan al menos una vez a la semana durante conforman. Se trata de trastornos que se caracterizan por
3 meses (op- ción 2 incorrecta). En los atracones es necesa- comportamientos o fenómenos fisiológicos anormales que
rio que se produzca la sensación de falta de control. Algu- tienen lugar durante el sueño o en las fases de transición
nos individuos describen características disociativas duran- del sueño-vigilia (trastorno por pesadillas, del despertar del
te o después del atracón. Asi- mismo, en algunos casos, los sueño no REM (que aúna sonambu- lismo y terrores), com-
atracones pueden estar planifica- dos. El tipo de alimento portamental del sueño REM y síndrome de piernas inquie-
consumido durante los atracones puede variar entre los dis- tas) (opción 2 correcta). En contraposición, las disomnias
tintos individuos y en un mismo individuo. No obstante, en son alteraciones de la cantidad, calidad u horario del sueño
general los individuos tienden a comer alimen- tos que en (insomnio, hipersomnia, narcolepsia (opción 1 incorrec-
otras circunstancias evitarían (opción 4 incorrecta). APIR ta), trastornos del sueño relacionados con la respiración
Manual de Psicología Clínica, Tomo II, Tema 12. como las apneas (opción 4 incorrecta) y trastornos del rit-
mo circa- diano). La opción 3 no puede ser correcta ya que
178. Señale la respuesta correcta respecto a la disforia no se pueden considerar trastornos puramente motrices o
de género (DG), según el DSM 5: motores, que haría referencia a aquellos trastornos con pro-
blemas en las habilida- des motoras (opción 3 incorrecta).
1. Se trata de un nuevo trastorno introducido en el DSM 5 APIR Manual de Psicología Clínica, Tomo II, Tema 17.
dentro de la categoría general de las disfunciones sexuales.
2. La principal diferencia entre la DG en niños y adoles- 112. Señale la afirmación correcta respecto de los terro-
cente es el criterio de duración (6 meses en niños y 12 en res nocturnos:
adoles- centes).
3. Si existe otro diagnóstico del desarrollo sexual (por ejem- 1. Generalmente se producen en el último tercio del perio-
plo, un trastorno androgenital congénito como hiperplasia do principal del sueño.
adrenal congénita) no debe hacerse el diagnóstico de DG. 2. Es una parasomnia del sueño REM.
4. En niños, el criterio principal (A1) se describe como: “un 3. Forman parte, junto con el sonambulismo, de una mis
poderoso deseo de ser del otro sexo o una insistencia de ma categoría diagnóstica (DSM 5).
que él o ella es del sexo opuesto (o de un sexo alternativo 4. Suele recordarse el contenido de lo soñado en gran me
distinto del que se le asigna)” dida.

Respuesta correcta:4 Respuesta correcta:3

En el DSM 5 la disforia de género supone un capítulo inde- En el DSM-5 aparece el trastorno del despertar del sueño
pen- diente (opción 1 incorrecta). La duración del trastorno no REM (opción 2 incorrecta) dentro de las parasomnias,
en el DSM 5 es la misma tanto para adolescentes/adultos que eng- loba terrores nocturnos y sonambulismo (opción
como para niños, de 6 meses (opción 2 incorrecta). Puede 3 correcta). Respecto a los terrores nocturnos, suelen pro-
especificarse “con un trastorno de desarrollo sexual” (op- ducirse en el primer tercio de la noche (opción 1 incorrec-
ción 3 incorrecta). Por tanto, la única opción correcta es la ta), en los que el niño se levanta bruscamente gritando,
4, siendo el criterio principal el primero del criterio A: “un pudiéndose producir vocalizacio- nes acompañadas de ma-
poderoso deseo de ser del otro sexo o una insistencia de que nifestaciones de ansiedad y activación intensa (sudoración,
él o ella es del sexo opuesto (o de un sexo alternativo distin- piloerección, taquicardia, gestos descoordi- nados rápidos,
to del que se le asigna)” (opción 4 correcta). APIR Manual fijación de la mirada en algún punto frontal...). Por lo gene-
de Psicología Clínica, Tomo II, Tema 16. ral, el niño no suele recordar lo ocurrido al día si- guiente, y
tampoco lo soñado en el momento del episodio (op- ción 4
84. ¿En la infancia, las parasomnias se consideran?: incorrecta) (si acaso alguna imagen o emoción, pero no una
historia elaborada como en las pesadillas). APIR Manual de
1. Trastornos secundarios del sueño, e incluyen narcolep- Psicología Clínica, Tomo II, Tema 17.

30
113. El diagnóstico de la narcolepsia, aunque esencial- 114. En el marco de los trastornos del sueño, respecto
mente clínico, idealmente debe ser confirmado en labo- del síndrome de retraso de fase, cabe afirmar que:
rato- rio de sueño a través de:
1. Los pacientes presentan un sueño de menor calidad.
1. Poligrafía de sueño seguida de tonometría a la mañana 2. Es el problema de ritmo circadiano menos frecuente.
siguiente. 3. Es más frecuente en población general adulta de edad
2. Poligrafía de sueño seguida del Test de Romberg a la ma- media que en adolescentes.
ñana siguiente. 4. Existe una forma familiar que representa el 40% de los
3. Polisomnografía nocturna, seguida a la mañana si- casos.
guiente del test de latencias múltiples de sueño (TLMS).
4. Polisomnografía nocturna, seguida a la mañana si- guien- Respuesta correcta:0
te de capnografía.
ANULADA. En la plantilla de respuestas pro- visional la
Respuesta correcta:3 respuesta dada como correcta era la 4, puesto que en el sín-
drome de retraso de fase suele existir una agregación fami-
Antes de centrarnos en los criterios para la narcolepsia se- liar (que representa el 40% de los casos), observándose más
gún el DSM-5, es conveniente comentar algunos conceptos mutaciones en el gen Per3 que en los sujetos sanos (factor
que apa- recen las alternativas de respuesta propuestas. La de riesgo). Por el contrario, variaciones en el gen CK1 ep-
tonometría (opción 1 incorrecta) es un examen para medir silon ejer- cen una función protectora (opción 4 correcta).
la presión dentro de los ojos, utilizado en el diagnóstico del Además, en un primer momento se consideró la opción 1
glaucoma. El test de Romberg (opción 2 incorrecta) es una incorrecta, pues según DSM-5 y Caballo, V., Salazar, I. y Ca-
prueba neurológica para valorar la propiocepción cons- rrobles, J. A. (2014) en el Manual de psicopatología y tras-
ciente, que consiste en pedirle a la persona que se ponga de tornos psicológicos, los pa- cientes no presentan un sueño
pie con ambos pies juntos con los ojos abiertos, pidiendo de menor calidad en sí mismo, ya que el ciclo sueño-vigilia
seguidamente que los cierre (si en este momento pierde el está conservado de manera adecuada, son las demandas
equilibrio, significaría que existe algún tipo de problema). sociales o personales las que provocarían el malestar, si no
La capnografía (opción 4 incorrecta) es el estu- dio de los existieran esas demandas y la persona pudiera levantarse a
niveles de dióxido de carbono en el ciclo respiratorio de la la hora deseada no habría disfunción. Sin embargo, la op-
persona. Los criterios diagnósticos de la narcolepsia son los ción 1 también podría ser considerada correcta, teniendo
siguientes: A. Períodos recurrentes de necesidad irrefrena- en cuenta información aportada tanto por DSM-5 como
ble de dormir, de abandonarse al sueño o de echar una sies- por el Manual de psicopatología de Belloch, A., Sandín, B. y
ta que se producen en un mismo día. Estos episodios se han Ramos, F. (2008). Según Belloch, la fase de sueño del ritmo
de haber producido al menos tres veces por semana durante circadiano aparece más tarde en relación al horario normal
los últimos tres meses; B. Presencia de al menos una de las de acostarse, de modo que cuando el sujeto se acuesta no se
características siguientes: 1. Episodios de cataplejía, defini- duerme, y al levan- tarse se encuentra con sueño y cansado.
da por (a) o (b), que se producen como mínimo algunas DSM-5 apunta que son prominentes los síntomas de in-
veces al mes: (a). En los in- dividuos con enfermedad de somnio al comienzo del sueño, la dificultad para despertar-
larga duración, episodios breves (segundos o minutos) de se por la mañana y la somnolencia excesiva al comienzo del
pérdida brusca bilateral del tono mus- cular con conserva- día. Además, señala que puede desa- rrollarse un insomnio
ción de la consciencia que se desencadenan con la risa o las psicofisiológico como consecuencia de las conductas desa-
bromas; (b). En los niños o en otros individuos en los seis daptativas que deterioran el sueño e incrementan la activa-
meses posteriores al inicio, episodios espontáneos de mue- ción al intentar en repetidas ocasiones quedarse dormi- dos
cas o de abrir la boca y sacar la lengua, o hipotonía general a una hora temprana. La información anterior defendería
sin un desencadenante emocional evidente; 2. Deficiencia que la calidad del sueño se encontraría deteriorada (opción
de hi- pocretina, según el valor de inmunorreactividad de 1 correcta). Por otro lado, DSM-5 aporta información que
hipocretina-1 podría ser incompatible con la opción de respuesta 4: “no
en el líquido cefalorraquídeo (LCR) (inferior o igual a un se ha estable- cido la prevalencia familiar de tipo de fases
tercio del valor en individuos sanos analizados con la mis- de sueño retrasada”. El resto de alternativas de respuesta
ma prueba, o inferior o igual a 110 pg/ml). La concentra- son incorrectas. Aunque su prevalencia es pequeña (0,17%,
ción baja de hipocre- tina-1 en el LCR no se ha de observar pero parece ser mayor del 7% en adolescentes (opción 3
en el contexto de lesión, inflamación o infección cerebral incorrecta)), no se trata del ritmo circa- diano menos fre-
aguda; 3. Polisomnografía noc- turna con latencia del sueño cuente, pues es de destacar que más del 80% de los diagnós-
REM inferior o igual a 15 minutos, o una prueba de latencia ticos de trastorno del ritmo circadiano que se realizan en
múltiple del sueño con un valor medio inferior o igual a clínica se corresponden precisamente con el tipo de sueño
8 minutos y dos o más períodos REM al inicio del sueño retrasado (opción 2 incorrecta). APIR Manual de Psicolo-
(opción 3 correcta). APIR Manual de Psicología Clí- nica, gía Clínica, Tomo II, Tema 17.
Tomo II, Tema 17.

31
120. ¿Cuál de las siguientes drogas produce una fuerte individuo tiene una edad cronológica de seis años por lo
dependencia psicológica pero poca dependencia física?: menos (o un grado de desarrollo equivalente)] se caracteri-
za por presentar fuertes y muy frecuentes agresiones verba-
1. La cafeína. les y arrebatos agresivos físicos esporádicos (pero más de 2
2. El alcohol. al mes), que producen destrucción de propiedades, daño a
3. La cocaína. personas y animales [CRITERIO A. Arre- batos recurrentes
4. La heroína. en el comportamiento que reflejan una falta de control de
los impulsos de agresividad, manifestada por una de las si-
Respuesta correcta:3 guientes: 1. Agresión verbal o física sin daños en los últimos
3 meses; o 2. Tres arrebatos que provoquen daños o destruc-
La dependencia a una sustancia hace referencia a el patrón ción en los últimos doce meses], que no son premeditados
de consumo desadaptativo que conlleva un malestar signi- [CRITE- RIO C: Los arrebatos agresivos recurrentes no son
ficativo a lo largo de un amplio periodo de tiempo (un pe- premeditados (es decir, son impulsivos o provocados por
riodo de 12 meses según DSM). Existen dos tipos de depen- la ira) ni persiguen ningún objetivo tangible (p. ej., dinero,
dencia: 1) La de- pendencia física, cuyas manifestaciones poder, intimidación)], y suelen durar unos 20 o 30 minutos,
son la tolerancia (estado de adaptación caracterizado por la echándole la culpa siempre a lo que ocurre a su alrededor,
disminución de la respuesta del organismo a la misma can- no existiendo un periodo previo prodrómico. La opción 1
tidad de droga o por la necesidad de incrementar la dosis es incorrecta, ya que el caso presentado en el enunciado no
para conseguir un mismo efecto) y el síndrome de abstinen- presenta ni problemas de impulsividad (ej.: con frecuencia
cia (conjunto de síntomas específicos que surgen debido a juguetea con o golpea las manos o los pies o se retuerce en
un cese o una disminución del consumo de una sustancia el asiento, con frecuencia corretea o trepa en situa- ciones
que se ha producido en grandes cantidades o durante un en las que no resulta apropiado,...) ni atencionales (ej.: con
amplio periodo de tiempo); y 2) La dependencia psicológi- frecuencia falla en prestar la debida atención a detalles o
ca, que hace referencia al craving (intenso deseo de consu- por descuido se cometen errores en las tareas, con frecuen-
mir la sus- tancia). La cocaína se caracteriza porque produ- cia parece no escuchar cuando se le habla directamente,... )
ce en sus consu- midores un intenso deseo por consumir la propios del TDAH, ni un comportamiento en el que no se
sustancia (craving/de- pendencia psicológica) y un menor respetan los dere- chos básicos de otros, las normas o reglas
grado de dependencia física (opción 3 correcta), mientras sociales, característico del trastorno de conducta (opción 1
que el resto de sustancias (heroína, cafeína y alcohol) in- incorrecta). La opción 3 es incorrecta porque para el tras-
cluidas en las opciones 1, 2 y 4, presentan un predominio torno de disregulación disruptiva del estado de ánimo no
de dependencia física (opciones 1, 2 y 4 incorrec- tas). APIR se contempla la condición de que coexis- tan con los sínto-
Manual de Psicología Clínica, Tomo I, Tema 3. mas períodos de bienestar o euforia (opción 3 incorrecta).
La opción 4 es incorrecta porque no se cumplen cri- terios
85. Una niña de 9 años se caracteriza por presentar para un episodio maníaco (período bien definido de estado
fuertes y muy frecuentes agresiones verbales y arreba- de ánimo anormalmente y persistentemente elevado, ex-
tos agresivos físicos esporádicos (pero más de 2 al mes), pansivo o irritable, y un aumento anormal y persistente de
que pro- ducen destrucción de propiedades, daño a per- la actividad o la energía junto a, por ejemplo, disminución
sonas y ani- males, que no son premeditados, y suelen de la necesidad de dormir, sentimientos de grandeza, ver-
durar unos 20 o 30 minutos, echándole la culpa siempre borrea,...) (opción 4 inco- rrecta). APIR Manual de Psicolo-
a lo que ocurre a su alrededor, no existiendo un periodo gía Clínica, Tomo II, Tema 18.
previo prodrómico. ¿Cuál de los siguientes trastornos
podría presentar?: 118. ¿Cuál es el principal sesgo cognitivo del jugador pa
tológico?:
1. Trastorno por déficit de atención e hiperactividad, y se
podría diagnosticar también trastorno de conducta. La ilusión de control.
2. Trastorno explosivo intermitente. La negación y la mentira.
3. Trastorno de desregulación disruptiva del estado de áni- Evaluación sesgada de los resultados.
mo, siempre que se acompañe de periodos estables La idea de que cuanto más se juega hay más posibilida des
de bienestar y euforia. de ganar.
4. Trastorno bipolar de inicio en la infancia.
Respuesta correcta:1
Respuesta correcta:2
Lo que determina que un juego sea patológico es la capa-
El caso descrito en el enunciado se podría corresponder cidad de la persona para controlar voluntariamente su im-
con el trastorno explosivo intermitente (opción 2 correcta). plicación en el juego, pero generalmente la percepción de
Señalemos los criterios de dicho trastorno que se cumplen la persona sobre su propia capacidad puede estar alterada,
en este enun- ciado: Una niña de 9 años [CRITERIO E: El sobre todo en los jugado- res más patológicos (“ilusión de

32
control”, que es la creencia de que el azar no influye en el tanto en las relaciones como en la actividad profe- sional.
juego y que uno posee estrategias que le permitirán ganar APIR Manual de Psicología Clínica, Tomo II, Tema 19.
[opción 1 correcta]). La negación y la men- tira serían más
síntomas/conductas característicos del trastorno (opción 2 151. En cuanto a la incidencia del trastorno de la perso-
incorrecta). Aunque las opciones 3 y 4 también son distor- nalidad obsesivo-compulsivo en población general:
siones cognitivas, ninguna sería el sesgo principal en este
trastorno (opciones 3 y 4 incorrectas). APIR Manual de Psi- 1. Se da más en mujeres que en hombres.
colo- gía Clínica, Tomo II, Tema 18. 2. Se da más en hombres que en mujeres.
3. Se ha constatado que es coincidente entre hombres y mu-
134. Según el DSM 5 el trastorno límite de personalidad jeres.
afecta: 4. Las debilidades metodológicas de los estudios no per
miten conclusiones fiables en este sentido.
1. A un 2% de la población general.
2. A un 5% de individuos que acuden a centros ambulato Respuesta correcta:2
rios de salud mental.
3. A un 10% de pacientes de pacientes psiquiátricos El trastorno de la personalidad obsesivo-compulsivo se ca-
ingresados. rac- teriza por un patrón general de preocupación por el or-
4. A un 20-30% de la población clínica con trastornos de den, el perfeccionismo y el control mental e interpersonal, a
personalidad. expensas de la flexibilidad, la espontaneidad y la eficiencia.
Según DSM- 5, es uno de los trastornos de la personalidad
Respuesta correcta:0 más prevalentes en la población general, con una estima-
ción de la prevalencia de entre un 2,1 y un 7,9 %. En los
ANULADA. La respuesta correcta en la plantilla provisio- estudios sistemáticos (opción 4 incorrecta), se diagnosti-
nal era la 1. El Manual de psicopatología y trastornos psico- ca aproximadamente con el doble de frecuencia en el sexo
lógicos, de Caballo, V., Salazar, I. y Carrobles, J. A. (2014), masculino (opción 2 correcta; opciones 1 y 3 incorrectas).
de donde se ex- trae esta pregunta, cita que la prevalencia Una mayor prevalencia en hombres, también se ha consta-
del trastorno límite de la personalidad (TLP) en la pobla- tado en el trastorno de personalidad paranoide, el es- qui-
ción general es de un 2%. Sin embargo, esta pregunta fue zoide, el esquizotípico, el antisocial y el narcisista. Algunos
anulada porque DSM-5 (fuente a la que se hace referencia trastornos de personalidad, como el límite o el histrióni-
en el enunciado de la pregunta) aporta in- formación dife- co, son más prevalentes en mujeres que en hombres. Por
rente, indicando que la prevalencia estimada del TLP es de último, el tras- torno de personalidad evitativo, tiene una
un 1,6%, pudiendo llegar a afectar incluso a un 5,9% de la prevalencia similar en hombres y mujeres. APIR Manual de
población general (opción 1 incorrecta). El resto de alterna- Psicología Clínica, Tomo II, Tema 19.
tivas de respuesta son incorrectas. En los centros de Salud
men- tal la prevalencia es más elevada, llegando a afectar a 153. Señale la afirmación correcta respecto al trastorno
un 10% de los usuarios (opción 2 incorrecta) y un 20% de narcisista de la personalidad:
los pacientes psiquiátricos hospitalizados (opción 3 inco-
rrecta). Respecto a la alternativa de respuesta 4, la preva- 1. Su presencia en población general es menor del 1% (DSM
lencia del TLP, entre la pobla- ción clínica con trastornos de 5).
personalidad, se encuentra entre el 30-60% (opción 4 inco- 2. Su presencia en población clínica es del 7 al 15 % (DSM
rrecta). Otros datos relevantes que se incluyen en el manual 5).
de Caballo y que caben destacar a nivel epidemiológico so- 3. La mayor comorbilidad se da con las parafi lias y las dis-
bre el TLP son los siguientes: 1) Es más fácil encontrar TLP funciones sexuales.
entre personas que buscan ayuda por consumo de sustan- 4. Los factores genéticos parecen explicar el ma yor porcen-
cias psicoactivas, trastornos de alimentación y trastor- nos taje de varianza del trastorno.
depresivos. 2) El TLP se diagnostica más en mujeres que en
hombres (aproximadamente un 75% de los casos son muje- Respuesta correcta:1
res. 3) El patrón familiar que se observa en el TLP es cinco
veces más frecuente en familiares de primer rango de quie- El trastorno de la personalidad narcisista se caracteriza por
nes ya tienen un TLP. 4) Aproximadamente del 8 al 10 % de un patrón general de grandiosidad (en la imaginación o en
los sujetos con un TLP consuman un suicidio. Son habitua- el com- portamiento), necesidad de admiración y falta de
les también las automu- tilaciones. 5) El deterioro causado empatía. Se- gún el Manual de Psicopatología de Caballo
por el trastorno y el riesgo de suicidio son mayores en los (2014), los porcen- tajes ofrecidos por el DSM-5 para este
primeros años de la edad adulta y van desapareciendo gra- trastorno van del 2 al 16 % para la población clínica (op-
dualmente con la edad, siendo favora- ble el pronóstico a ción 2 incorrecta) y menos del 1% para la población gene-
largo plazo, ya que durante la cuarta o quina década de vida ral (opción 1 correcta). La mayor co- morbilidad se da con
de los supervivientes en general hay una mayor estabilidad la depresión mayor y la distimia (42-50%), seguidas de los

33
trastornos por consumo de sustancias (24-50%) y del tras- 81.Los delirios en los niños:
torno bipolar (5-12%) (opción 3 incorrecta). Aunque son
importantes las diferencias individuales constitucionales 1.Son más frecuentes que en los adultos.
y temperamentales, tales como la hipersensibilidad, la to- 2.Tienden a ser muy sistematizados.
lerancia a la frustración y la regulación de las emociones; 3.Son, habitualmente, de tipo persecutorio, grandeza o hi-
las experiencias evolutivas, más que los factores genéticos, pocondría.
constituyen factores importantes para explicar el desarrollo 4.No se dan delirios en los niños sólo ideas sobrevaloradas.
del trastorno narcisista de la personalidad (opción 4 inco-
rrecta). APIR Manual de Psicología Clínica, Tomo II, Tema Respuesta correcta:3
19.
La bibliografía recoge casos y características de esquizo-
frenia y delirios en la población infantil (opción 4 inco-
rrecta). Los datos con respecto a la prevalencia, son muy
escasos y pobres. Se estima que es muy poco frecuente y
PSICOPATOLOGÍA Y CLÍNICA que su prevalencia va aumentando hasta llegar al 1% en la
INFANTIL adolescencia (opción 1 incorrecta). Las características de la
esquizofrenia en la infancia son iguales que en los adultos.
Los síntomas positivos son los más característicos del cua-
177. Según el DSM 5, cuando un niño de 10 años alcan-
dro, aunque en este caso suelen ser de menor elaboración y
za un CI Global de 65 en la evaluación realizada con un
estructuración (opción 2 incorrecta). Pueden ser más ha-
WISC-V, podemos concluir que la gravedad actual del
bituales las alucinaciones visuales. APIR Manual de Psico-
pa- ciente corresponde a:
logía Clínica Infantil, Tema 3.En Wicks-Nelson, (1997) se
recoge lo siguiente, “las ideas delirantes, creencias falsas que
1. Una discapacidad intelectual Leve.
se mantienen incluso ante la evidencia de una contradic-
2. Una discapacidad intelectual Moderada.
ción real, se encontraron con relativa frecuencia y de forma
3. Necesitamos conocer, además del CI Total,
consistente entre las muestras de Kolvin et al., 1971, Green
los índices de inteligencia verbal y manipulativo para poder
et al., 1996, Russell et al., 1989 y Volkmar et al., 1988. A con-
especificar la discapacidad y funcionalidad.
tinuación se presentan algunos ejemplos. Persecutoria: Un
4. No podemos determinar los diferentes niveles de gra
niño creía que su padre se había fugado de la cárcel y que
vedad si desconocemos el funcionamiento adaptativo y el
iba a matarle. Somática: Un niño creía que el espíritu de un
nivel de apoyos requeridos.
niño y el de una niña vivían dentro de su cabeza. Extrava-
gante: Un niño estaba convencido de que era un perro y que
Respuesta correcta:4
le estaba creciendo el pelo. Una vez se negó a abandonar la
consulta del veterinario si no le pegaban un tiro. Grandeza:
La discapacidad intelectual es un trastorno que comienza
Un niño tenía la firme creencia de que era diferente y capaz
du- rante el periodo de desarrollo y que incluye limitacio-
de matar a otras personas. Sintió que se volvió muy fuerte
nes del funcionamiento intelectual como también del com-
cuando Dios “zumbó” a través suyo.”(Opción 3 correcta).
portamiento adaptativo en los dominios conceptual, social
Wicks-Nelson, R,(1997): Psicopatología del niño y del ado-
y práctico (3 inco- rrecta, opción 4 correcta). Por lo tan-
lescente. Prentice Hall.
to, únicamente conociendo la puntuación del CI, según el
DSM 5 no podemos realizar el diagnóstico de Discapacidad
78. En la Hipótesis de la coerción (Patterson, 1982),
Intelectual (opción 1 y 2 incorrec- tas). El DSM 5 sustituye
cuando el padre retira la orden por no oír llorar o gritar
el término Retraso Mental por el de Discapacidad Intelec-
al niño porque no obedece, el mantenimiento de la con-
tual. El DSM IV especificaba la gravedad del retraso mental
ducta coercitiva se explica por:
en función del CI, el retraso mental leve se corresponde con
un CI entre 50-55 y 70, el moderado se corres- ponde con
1. Refuerzo positivo.
un CI entre 35-40 y 50-55, el grave se corresponde con un
2. Refuerzo negativo.
CI entre 20-25 y 35-40 y el profundo se corresponde con un
3. Castigo positivo.
CI inferior a 20-25. La AAIDD prefiere el término discapa-
4. Castigo negativo.
ci- dad intelectual frente al de retraso mental. Defienden un
modelo funcional, dado que es el funcionamiento social de
Respuesta correcta:2
la persona y no su CI, lo que importa a la hora de realizar
una evaluación y un tratamiento del problema. Entiende
Patterson elabora la teoría explicativa más ampliamen-
el retraso mental como una forma de ser de cierto tipo de
te aceptada y estudiada sobre las posibles contribuciones
personas que necesitan apoyos. APIR Manual de Psicología
ambientalesa las conductas agresivas, la “Teoría de la coac-
Clínica Infantil, Tema 2.
ción”, a través del estudio observacional en vídeo del niño
en la escuela y en interacción con sus padres en el hogar. La

34
teoría de la coacción explica cómo se dan patrones disocia- den manifestar una expresión inhibida de las emociones
les en las dinámicas fami- liares. Los niños utilizan conduc- positivas (opción 3 incorrecta), retrasos cognitivos y del
tas agresivas (rabietas, lloros...) para controlar la conducta lenguaje, y alteraciones en la reciprocidad social (opción 4
de sus padres. Estas conductas se re- fuerzan positivamente incorrecta). Por ello, sería necesario diferenciar el trastorno
(el niño consigue lo que quiere) o nega- tivamente (el niño de apego reactivo del trastorno del espectro autista. Estos
evita situaciones que no quiere). El proceso por el cual el dos trastornos se pueden distinguir basándose en las dife-
niño utiliza conductas aversivas en su beneficio se denomi- rentes his- torias de abandono y por la presencia de intere-
na coacción. En los procesos de coacción presentes en las ses restringidos o comportamientos ritualizados, un déficit
relaciones entre padres e hijos destaca el papel explicativo específico en la comu- nicación social y comportamientos
del reforzamiento. Los padres ante las conductas del niño de apego selectivos (opción 2 correcta).Los niños con tras-
no actúan correctamente y ceden ante sus deseos. A corto torno de apego reactivo han ex- perimentado una historia
plazo resulta agradable para ambas partes, pero a largo pla- de abandono social grave, aunque no siempre es posible ob-
zo se convierte en un grave problema. Los padres al ceder tener historias detalladas sobre la natura- leza precisa de sus
ante sus hijos caen en la “trampa del reforzamiento”, forta- experiencias, sobre todo en las evaluaciones iniciales. No
leciendo este tipo de con- ductas negativas y coercitivas, es es común que los niños con trastorno del espectro autista
por esto último por lo que se nos pregunta. En términos tengan una historia de abandono social. Los intereses res-
de psicología del aprendizaje, refuer- 81. zos serían aquellas tringidos y los comportamientos repetitivos, característicos
consecuencias o estímulos que aumentarían la probabilidad del trastorno del espectro autista, no son una particularidad
de que repitiéramos la conducta, siendo positivo 1. cuando del trastorno de apego reactivo. Estas características clíni-
se presenta un estímulo apetitivo contingente a la con- 2. cas mani- fiestan una fijación excesiva a rituales y rutinas,
ducta (opción 1 incorrecta) y negativo cuando es la desa- con intereses restringidos, obsesivos y reacciones sensoria-
parición 3. de un estímulo aversivo, en este caso los gritos les inusuales. Sin embargo, es importante destacar que los
del niño, lo que aumentan la probabilidad de que repitamos niños con cualquiera de las dos afecciones pueden exhibir
la conducta, retirar 4. la orden, ceder, lo que nos provoca comportamientos estereoti- pados, tales como mecerse o
alivio (opción 2 correcta). moverse. Los niños con cualquiera de ambos trastornos
Por su parte, castigos serían aquellas contingencias que dis- también pueden mostrar una variedad de funcionamiento
mi- nuyen la probabilidad de conducta, siendo positivo, si intelectual, pero sólo los niños con trastorno del espectro
es por la aparición de una consecuencia aversiva (opción 3 autista muestran alteraciones selectivas en los compor- ta-
incorrecta) o negativo si es por la eliminación de una con- mientos sociales de comunicación tales como la comuni-
secuencia o estímulo apetitivo (opción 4 incorrecta). APIR cación intencional (p. ej., dificultades en la comunicación
Manual de Psicología Clí- nica Infantil, Tema 8. deliberada, dirigida a una meta y con objeto de influir en el
comportamien- to del destinatario). Los niños con trastor-
82. Según el DSM 5, señale a partir de qué característi- no de apego reactivo muestran un funcionamiento de co-
cas pueden diferenciarse, en niños pequeños, el Trastor- municación social compara- ble a su nivel general de fun-
no de apego reactivo del Trastorno del espectro autista: cionamiento intelectual. Por último, los niños con trastorno
del espectro autista normalmente mues- tran una conducta
1. Analizar los síntomas nucleares que presenta ya que no de apego que es característica de su nivel de desarrollo. Por
hay semejanzas entre un trastorno y otro. el contrario, los niños con trastorno de apego re- activo lo
2. La historia de cuidados previos al desarrollo del trastor- hacen de una manera rara o inconsecuente”. American Psy-
no, así como observar la presencia de comportamientos re- chiatric Association,( 2014), DSM - 5: Manual diagnóstico
petiti- vos e intereses restringidos. y estadístico de los trastornos mentales, 5a edición, Madrid:
3. La manifestación de expresiones inhibidas de las emocio- Edi- torial Médica Panamericana.
nes positivas.
4. La presencia de retraso cognitivo del lenguaje y de al- te- 138. ¿Cuál es la duración mínima necesaria para poder
raciones de la reciprocidad social. realizar un diagnóstico de mutismo selectivo según el
DSM 5?:
Respuesta correcta:2
1. Un mes.
Es importante el diagnóstico diferencial entre el Trastorno 2. Tres meses.
de apego reactivo y el Trastorno del espectro autista ya que 3. Seis meses.
am- bos comparten algunas de sus características y mani- 4. Doce meses.
festaciones (opción 1 incorrecta). Con respecto a esto, el
DSM 5 recoge lo siguiente; “Los comportamientos sociales Respuesta correcta:1
aberrantes no sólo se manifiestan en los niños pequeños
con trastorno de apego reacti- vo, sino que también son la El mutismo selectivo consiste en la incapacidad persistente
principal característica del trastorno del espectro autista. por hablar en situaciones sociales específicas (en las que se
En concreto, los niños pequeños de ambas afecciones pue- espera que hable, p. ej., en la escuela) a pesar de hacerlo

35
en otras situa- ciones o situaciones generales ante personas zo a estar fuera de casa solo, angustia cuando el cónyuge o
ajenas al ámbito íntimo. La alteración interfiere el rendi- los hijos hacen cosas de forma independiente, o cuando el
miento escolar o laboral o la comunicación social y la dura- contacto con el cón- yuge o los hijos no es posible (opción
ción es de por lo menos 1 mes (no limitada al primer mes de 4 incorrecta). Su inicio temprano se identifica antes de los
escuela) (opción 1 correcta, opcio- nes 2, 3 y 4 incorrectas). 6 años, aunque la edad típica de aparición es a los 9 años.
La incapacidad para hablar no se debe a una falta de co- Normalmente aparece entre 6 y los 11 años, con frecuencia
nocimiento o de fluidez del lenguaje hablado requerido en después de un estrés vital, en especial una pérdida (opción
la situación social. El trastorno no se explica mejor por la 1 incorrecta). Puede ser hereditario (opción 2 incorrecta).
presencia de un trastorno de la comunicación (p. ej., tarta- APIR Manual de Psicología Clínica Infantil, Tema 11.
mudeo) y no aparece exclusivamente en el transcurso de un
tras- torno generalizado del desarrollo, esquizofrenia u otro 69. Dentro de los miedos normales en el desarrollo evo-
trastorno psicótico. Es un trastorno muy poco frecuente. Su lutivo, señale cuál de las siguientes opciones NO descri-
prevalencia se sitúa por debajo del 1%. Parece ligeramente be los miedos propios de la edad que se indica:
mayor en niñas. Su inicio suele ser anterior a los cinco años,
sin embargo puede que no se haga patente hasta que co- 1. A los 2 años: ruidos fuertes, oscuridad, separación de los
mience la escuela. Suele du- rar varios meses y luego remi- padres, objetos o máquinas grandes, cambios en el ambien-
tir de manera espontánea, aunque en algunos casos puede te personal.
persistir durante años. APIR Manual de Psicología Clínica 2. De los 3-4 años: miedo a las máscaras, oscuridad, ani-
Infantil, Tema 11. males, separación de los padres, ruidos inesperados (inclu-
yendo los nocturnos).
139. Señale cuál de las siguientes afirmaciones sobre el 3. De los 7-8 años: seres sobrenaturales, oscuridad, per-
trastorno de ansiedad por separación es FALSA: manecer solos, miedos basados en sucesos informados por
me- dios de comunicación, daño físico, ridículo.
1.Se desarrolla con frecuencia después de un estrés vital (p. 4. De los 9-12 años: relaciones entre iguales, aspecto físi- co
ej., la muerte de una mascota familiar). y pérdida de autoestima.
2.Puede ser hereditario.
3.Los niños manifiestan una mayor reticencia a asistir a la Respuesta correcta:0
escuela que las niñas.
4.La expresión indirecta del miedo a la separación pue de ANULADA. El miedo es una reacción normal, con gran
ser más común en los varones. valor funcional y adaptativo. Forma parte del desarrollo
normal del niño y sus contenidos suelen ser cambiantes y
Respuesta correcta:3 evolucionar a la vez que el niño se va desarrollando. Esta
pregunta está plan- teada para contestarla con la clasifica-
La ansiedad de separación es un fenómeno primario que ción que realizan Bragado y Morris y Kratochwill (1994).
sirve para proteger al niño durante los primeros años. Se Según la clasificación de estos autores, las opciones 1, 2 y
trata de un miedo básico, es de los primeros que aparece, 3 son incorrectas por contener en los enunciados miedos
a partir de él pue- den ir apareciendo otros problemas más propios de la edad indicada, y la opción 4 correcta. Entre
específicos. Se trata de una ansiedad excesiva ante la sepa- los 9-12 años, señalan que disminuye el miedo a la oscu-
ración de individuos con los que el niño está vinculado. ridad y a estar solo, se mantienen el miedo a las lesiones
Las manifestaciones de este trastorno varían desde los mo- corporales, los fenómenos naturales y aparecen escolares
mentos previos a la separación en los que se dan manifes- como exámenes o rendimiento académico y la muerte. Se-
taciones de ansiedad y temor, a las que se dan una vez se ha gún Eche- burua, en cambio, entre los 12 y los 18 años, son
producido la separación, aquí es más frecuente la tristeza. miedos evolu- tivos normales los miedos a las relaciones in-
La sintomatología de este trastorno sufre ciertas varia- cio- terpersonales y a la pérdida de autoestima y según la clasi-
nes en función de la edad. Los niños más pequeños (5-8 ficación de Sandín en la adolescencia son miedos comunes
años) suelen mostrar preocupación porque les ocurra algo los sexuales, los relacionados con la autoidentidad, el rendi-
malo a sus figuras de referencia y resistencia a ir al colegio. miento personal, los sociales, los académicos, los políticos y
En niños más mayores (9-12) predomina el distrés (males- los económicos. Por lo tanto, la pre- gunta ha sido anulada,
tar general subjetivo) y en los adolescentes (13-16 años) ya que no mencionan los autores en ella, y se solapan los
los síntomas más prevalentes son las quejas físicas durante miedos en función de las distintas clasificaciones. Bragado,
los días de colegio. Afecta más a niñas que a niños en una C. “Terapia de conducta en la infancia: trastornos de ansie-
proporción 2:1. Las niñas manifiestan una mayor reticen- dad”. Fundación Universidad-Empresa. Madrid. 1994
cia o evitación a asistir a la escuela que los niños (opción 3
correcta). La expresión indirecta del miedo a la separación
puede ser más común en los varones que en las muje- res,
por ejemplo, a través de una limitación en el funcionamien-
to independiente, rechazo a estar fuera de casa solo, recha-

36
PSICOLOGÍA DIFERENCIAL Y 10. Atendiendo al modelo cúbico de la estructura del in-
telecto de Guilford, señale qué facetas de las dimensio-
DE PERSONALIDAD nes “operación mental” y “contenido” intervienen si se
le pide a un niño que diga todos los animales mamíferos
9. ¿A qué tipo de covariación entre genes y ambiente que conoce:
nos referimos si son los propios padres los que trasmiten
tan- to el genotipo como el ambiente propicio para el 1. Convergente semántico.
desarrollo de un rasgo?: 2. Divergente semántico.
3. Convergente simbólico.
1.Activa. 4. Divergente simbólico.
2.Específica.
3.Pasiva. Respuesta correcta:1
4.Reactiva.
En cuanto a las operaciones mentales, la producción con-
Respuesta correcta:3 vergen- te implica la elaboración de respuestas o conclu-
siones lógicas para alcanzar la mejor respuesta para la
Hablamos de correlación pasiva cuando un rasgo se ve fa- información de la que disponemos (la lista de mamíferos
cilita- do tanto por la predisposición genética como por la existentes es finita); mientras que la producción divergente
influencia del ambiente (Opción 3 correcta). Un ejemplo de implica ofrecer soluciones nuevas y alternativas, relaciona-
este tipo se- ría cuando los progenitores, que son quienes do con la creatividad. Por otra parte, los contenidos semán-
aportan los genes que predisponen a un niño a un rasgo, ticos hacen referencia a constructos mentales a los que se
también suministran el entorno que estimula el desarrollo aplican palabras (los nombres de animales mamíferos son
de ese rasgo. Las correlacio- nes activas implican cuando contenidos semánticos); mientras que los contenidos sim-
el sujeto busca de forma activa el entorno adecuado a su bó- licos utilizan información en forma de signos que no
contenido genético (opción 1 incorrecta). Las correlaciones tienen sig- nificado por sí mismos, como serían letras, mo-
reactivas implican cuando los sujetos, como reacción a su nedas, números... Teniendo en cuenta lo anterior la única
genotipo, proporcionan un ambiente adecuado al mismo opción correcta es la nú- mero 1. APIR Manual de Persona-
(opción 4 incorrecta). La correlación específica es un dis- lidad y Diferencial, Tema 4.
tractor (opción 2 incorrecta). APIR Manual de Personali-
dad y Diferencial, Tema 2. 13. Mientras estudias con la ventana abierta oyes una
canción y tu mente se traslada al verano y la última
17. Con relación a las causas de las diferencias indivi- fiesta en la que conociste a alguien muy interesante, de
duales, ¿qué es la heredabilidad amplia?: quien no has vuelto a saber. Tu mente va de una imagen
a otra y cuando te das cuenta han pasado 10 minutos.
1. La proporción de varianza fenotípica explica da por la Este proceso refleja, según el modelo de R. J. Sternberg:
varianza genotípica.
2. La proporción de varianza genotípica explica da por la 1. Cómo operan los “componentes” o procesos elementales
varianza fenotípica. de información.
3. La proporción de varianza fenotípica aditiva explicada 2. El funcionamiento de los correlatos de la inte ligencia.
por la varianza genotípica. 3. La gran asociación entre capacidad auditiva y memoria,
4. La proporción de varianza fenotípica explica da por la base de las diferencias individuales
varianza genotípica aditiva. en inteligencia.
4. La asociación entre capacidad auditiva y aten ción focal.
Respuesta correcta:1
Respuesta correcta:1
Heredabilidad amplia: explica la parte del fenotipo que pro-
vie- ne del genotipo. El resultado será entre 0 y 1. El valor Dentro de la teoría Triárquica de Sternberg encontramos la
máximo implicaría que la variabilidad total del comporta- Sub- teoría componencial para la inteligencia analítica, la
miento viene determinado por los genes. (opción 1 correc- cual estudia la relación entre la inteligencia y el mundo in-
ta). La opción de respuesta 2 es un distractor y no tendría terno del sujeto, poniendo el énfasis sobre los procesos que
sentido puesto que el fenotipo es el resultado de la interac- intervienen en el pen- samiento. La unidad de análisis es
ción genes-ambiente. La op- ción de respuesta 3 también el “componente”, entendido como proceso básico que actúa
es un distractor, ya que el concepto de varianza fenotípica sobre las representaciones inter- nas de los datos recogidos
aditiva no existe. El concepto de Varianza genotípica aditiva del exterior. Se encarga de traducir los datos externos en
significa: parte de la variabilidad debida a un gen determi- representaciones conceptuales (opción 1 correcta). La teo-
nado en un cromosoma concreto (opción 4 incorrec- ta). ría de Sternberg está claramente centrada en el paradigma
APIR Manual de Personalidad y Diferencial, Tema 2. del procesamiento de la información y pone por ello mucho

37
énfasis en el medio externo, el medio interno y la rela- ción 12. Señale qué se puede afirmar, en general, sobre la es-
entre ambos, pero no indaga en los correlatos anatómicos tabilidad de la personalidad a lo largo del tiempo:
de los procesos cognitivos superiores (Opción 2 incorrec-
ta). Las opciones de respuesta 3 y 4 son distractores. APIR 1. La personalidad es menos estable en periodos cortos de
Manual de Personalidad y Diferencial, Tema 4. tiempo que considerando periodos más largos.
2. Es más clara la estabilidad de la inteligencia que la de la
14. Respecto de la relación entre la inteligencia (crista- personalidad.
lizada y fluida) y la edad, señale la opción correcta: 3. Las puntuaciones en los test de personalidad son muy
estables a lo largo del ciclo vital.
1. A partir de la edad en que alcanzan el máximo nivel, la 4. No hay diferencias individuales en la estabili dad durante
fluida experimenta un declive más pronunciado que la cris- el desarrollo.
talizada.
2. A partir de la edad en que alcanzan el máximo nivel, Respuesta correcta:2
ambos tipos de inteligencia se mantienen en el tiempo sin
variación. Como patrón general, la estabilidad de la inteligencia está
3. El curso de ambos tipos de inteligencias es de tipo fluc- más demostrada que la estabilidad de la personalidad (op-
tuante, sin que exista un patrón definido de incremento o ción 2 co- rrecta). Los estudios de rasgos muestran ligeros
decremento asociado a la edad. cambios en do- minancia, mayores en jóvenes, y en cuanto
4. El curso de ambos tipos de inteligencia es es- table a par- a los cinco grandes, los consabidos aumentos de amabilidad
tir de la adolescencia. y responsabilidad, y descensos en extroversión, neuroticis-
mo y apertura a la expe- riencia (opción 3 incorrecta). En
Respuesta correcta:1 pequeñas medidas temporales acontecerán menores o casi
ningún cambio en la personalidad, mientras que con un ho-
La inteligencia fluida alcanza el punto máximo a los 14-15 rizonte temporal de años o décadas (op- ción 1 incorrecta).
años y sufre un mayor declive con la edad, mientras que la Por supuesto, siempre hay que tener en cuen- ta que la esta-
inteligen- cia cristalizada alcanza su punto máximo en tor- bilidad/cambio en parámetros de personalidad está sujeto a
no a los 20 años y se conserva mejor con la edad (Opción 1 la variabilidad individual, (opción 4 incorrecta). APIR Ma-
correcta, opciones 2,3,4 incorrectas). APIR Manual de Per- nual de Personalidad y Diferencial, Tema 7.
sonalidad y Diferencial, Tema 4.
11. Según el modelo jerárquico de H.J. Eysenck, seña-
176. La estructura de tres estratos del modelo de Cate- le en qué nivel se sitúa el hecho de que una persona se
ll-Horn-Carroll (CHC) sobre la inteligencia establece enfade habitualmente y tenga sentimientos de culpabili-
que en el estrato II se sitúan: dad y ansie- dad tanto en casa como en el trabajo:

1. Las aptitudes intelectuales (por ejemplo, inte ligencia flui- 1. Respuesta estereotipada.
da, cristalizada, etc.) 2. Patrón de respuesta.
2. El factor G o capacidad general. 3. Hábito.
3. Los factores relacionados con cada aptitud in 4. Rasgo.
telectual de segundo orden.
4. Las tareas que componen la prueba de inteli Respuesta correcta:4
gencia.
En el modelo jerárquico de Eysenck, los rasgos hacen refe-
Respuesta correcta:1 rencia a constructos que tienen su base en respuestas ha-
bituales y que tienden a darse en diferentes situaciones o
La principal aportación del modelo de Catell se centra en situaciones que no son percibidas como similares (Opción
desa- rrollar los factores de segundo orden, entre ellos se 4 correcta). Por otro lado, los hábitos o respuestas habitua-
encuentran la inteligencia fluida, cristalizada, memoria, les implican respuestas especí- ficas que sólo se repiten ante
aprendizaje... (opción 1 correcta). El factor G o capacidad circunstancias similares (opción 3 incorrecta). Las opciones
general, que en el modelo de Cattel podría equipararse con de respuesta 1 y 2 son distractores, ya que son conceptos
la Inteligencia fluida histórica, es- taría en el tercer nivel de que no pertenecen a la teoría de Eysenck. APIR Manual de
la pirámide (opción 2 incorrecta). Los factores relacionados Personalidad y Diferencial, Tema 9.
con cada aptitud intelectual de segundo orden hace refe-
rencia al primer estrato de la pirámide, o factores primarios 15. En el Modelo de los Cinco Grandes, de McCrae y
(opción 3 incorrecta). Las tareas que compongan las prue- Costa, ¿a qué dimensión pertenece la faceta de impul-
bas de inteligencia no forman parte del modelo jerárquico sividad?:
de Cattell (opción 4 incorrecta). APIR Manual de Persona-
lidad y Diferencial, Tema 4. 1. Neuroticismo.

38
2. Extraversión sos a experimentar estrés.
3. Búsqueda de sensaciones. 3. Los sujetos idiocéntricos son más afables mientras que
4. Responsabilidad. los sujetos alocéntricos son más competitivos.
4. Los sujetos idiocéntricos son más afables y los alocéntri-
Respuesta correcta:1 cos más optimistas.

Según el modelo de los Big Five, los rasgos que componen Respuesta correcta:1
el factor Neuroticismo son: ansiedad, depresión, hostilidad,
im- pulsividad, timidez y vulnerabilidad (opción 1 correc- Las personas individualistas se orientan más a conseguir
ta); y los rasgos que componen el factor Extraversión: aser- el éxito, mientras que las colectivistas a la evitación del
tividad, afecto, búsqueda de emociones, actividad, emocio- fracaso. Por otro lado, las variables más frecuentes en la
nes positivas y gre- garismo (Opciones 2, 3 incorrectas). construcción del self en las culturas individualistas son la
Los rasgos que componen el factores Responsabilidad son: percepción de sí mismos como personas independientes,
Deliberación, autodisciplina, competencia, necesidad de autónomas y completas; acentúan los límites entre el yo y
logro, sentido del deber y orden (Op- ción 4 incorrecta). los otros, percibiéndose como agentes se- parados que ac-
APIR Manual de Personalidad y Diferencial, Tema 10. túan para conseguir sus propias metas; su propio self actúa
como fuente de acción y motivación. Por otra parte, en las
16. En el modelo de los Cinco Grandes, hay una dimen- culturas colectivistas se perciben como interdependientes
sión de personalidad cuyos niveles medios aumentan de los demás, sienten conexión con los miembros del grupo
consis- tentemente a lo largo del ciclo vital. ¿De cuál se a los que pertenecen y actúan en sintonía con las metas y
trata?: los deseos de los otros cercanos; son los otros la fuente de
acción y motiva- ción; se describen a sí mismas mucho más
1. La responsabilidad. mediante relaciones y roles que con atributos internos (Op-
2. La extraversión. ción 1 correcta). No está demostrado que los sujetos alocén-
3. La emocionalidad negativa. tricos sean más propensos a experimentar estrés (Opción 2
4. La apertura a la experiencia. incorrecta), aunque sí pueden ser las situaciones ansiógenas
de diferente índole frente a los sujetos idiocéntricos. Los su-
Respuesta correcta:1 jetos idiocéntricos son más competitivos u orientados al lo-
gro, mientras que los alocéntricos se rigen más por valores
Los estudios sobre estabilidad de los factores que compo- comunes en su grupo social, relacionado con la afa- bilidad
nen el modelo Big Five concluyen que se trata de elementos y compromiso común (Opción 3 incorrecta). La variable
relativa- mente estables de la personalidad que apenas se ve Optimismo no presenta valores diferenciales por pertene-
modificados con la edad. Dentro de los cambios que sí se cer a culturas individualistas frente a colectivistas (Opción
pudieron captar, se aprecia que el Neuroticismo, la Apertura 4 incorrec- ta). Bermúdez, J. (2011). Psicología de la Perso-
a la experiencia y la Extraversión aparecen menos elevados nalidad. Madrid: UNED.
en grupos de mayor edad; mientras que Responsabilidad y
Cordialidad tienden a au- mentar en las etapas de edad más 8. Markus (1977) propone el concepto de “esquemas del
avanzada (opción 1 correcta, opciones 2 y 4 incorrectas). La self” que indica que, del mismo modo que formamos
opción 3 hace referencia al térmi- no “emocionalidad nega- es- tructuras cognitivas sobre otros fenómenos, también
tiva” que es un concepto perteneciente al Modelo de los Sie- forma- mos estas estructuras en la relación con el self.
te Grandes de Tellegen y Waller. Si bien se correspondería Respecto a estos esquemas del self podemos afirmar
teóricamente con el factor Neuroticismo del Big Five, sigue que:
siendo una opción incorrecta puesto que el Neuroti- cismo
hemos visto que decrece en los grupos de edades avanza 1. Afectan a la rapidez con que procesamos in- formación
das. APIR Manual de Personalidad y Diferencial, Tema 10. relativa a nosotros mismos pero no influyen en la me- mo-
ria o recuerdo de esta información.
5. Triandis (1995) propone los términos idiocentrismo 2. Son generalizaciones cognitivas sobre noso- tros mismos
y alocentrismo para designar a las tendencias persona- por lo que no están afectados por las influencias culturales.
les que constituyen reflejos de la cultura. Algunas di- 3. Como estructuras cognitivas no están relacio- nadas con
ferencias entre los sujetos idiocéntricos y alocéntricos variables afectivas o motivacionales.
son que: 4. Representan no solo la consideración actual de nuestro
self sino nuestros “selves posibles” (como lo que creemos
1. Los sujetos idiocéntricos tienden hacia la do minancia que llegaremos a ser o lo que nos gustaría llegar a ser).
mientras que los alocéntricos muestran nive les más altos
de conformidad. Respuesta correcta:4
2. Los sujetos idiocéntricos están más motiva dos por el lo-
gro mientras que los sujetos alocéntri cos son más propen- Teniendo en cuenta el marco de referencia temporal de los

39
con- tenidos del autoconcepto, Markus propone una po- 141. Un paciente que tiene una vívida sensación de que
sible diferen- ciación entre los diversos tipos de «yoes» o sus venas se están enrollando sobre sí mismas está expe-
autoconceptos. Así, en la persona, puede coexistir un au- ri- mentando una:
toconcepto ligado al pasado («cómo era yo cuando mis
hijos eran pequeños»), otro referido al momento presente 1. Alucinación cenestésica.
(«cómo soy cuando estoy con mi familia»), y otro relativo 2. Parestesia.
al futuro («cómo sería mi parte más optimista»). El tercer 3.Alucinación vascular.
tipo de autoconcepto, el que hace referencia al futuro, es de- 4.Alucinación táctil.
nominado por Markus como los posibles yoes. Este tipo de
conocimiento engloba el conjunto de esperanzas, temores y Respuesta correcta:1
deseos que resultan relevantes para el individuo; así como
las diferentes cualidades que a la persona le gustaría tener o Las alucinaciones, según la modalidad sensorial, pueden
podría tener, y aquellas otras ante las que se sentiría mal en clasifi- carse en: 1) Auditivas, 2) Visuales, 3) Táctiles o hápti-
el caso de que las tuviera (Opción 4 correcta). Las opciones cas, 4) Ol- fativas, 5) Gustativas, 6) Somáticas, cenestésticas
1, 2 y 3 se ofrecen como distractores. APIR Manual de Per- o viscerales, 7) Cinestésicas o de movimiento y 8) Mixtas.
sonalidad y Diferencial, Tema 14. El enunciado des- cribe un tipo de alucinación cenestésica,
ya que se remite a sen- saciones corporales que proceden
del interior del propio cuerpo o que afectan a sus órganos
internos o externos. Si un paciente afirma que las venas se
le salen, se le enrollan y se le hacen una burbuja, estaríamos
PSICOPATOLOGÍA ante un tipo de alucinación cenestésica (op- ción 1 correc-
ta). Podría confundirnos con la alucinación táctil o háptica,
la cual puede manifestarse en cualquier parte del cuer- po.
133. Respecto a las imágenes alucinoides, cabe afirmar
Pero una alucinación táctil sería más del tipo sentirse toca-
que:
do, pellizcado, manoseado, sentir calambres, etc. (opción 4
inco- rrecta). Dentro de las alucinaciones táctiles o hápticas
1. Se producen en ausencia de estímulos concretos que la
podemos encontrar varias modalidades: térmicas, hídricas,
activen.
parestesias o sensación de hormigueo (opción 2 incorrec-
2. El individuo les otorga juicio de realidad.
ta), formicación o delirios dermatozoicos. El concepto de
3. Se trata de un concepto sinónimo al de imagen eidética.
alucinación vascular no existe (opción 3 incorrecta). APIR
4. Son un tipo de imagen mnémica.
Manual de Psicopatología, Tema 5.
Respuesta correcta:1
142. ¿Cómo se denominan las imágenes anómalas que
se pueden percibir en los momentos de transición de la
Las imágenes alucinoides son un tipo de pseudopercep-
vigila al sueño:
ciones o imágenes anómalas que se engloban dentro de los
engaños per- ceptivos. Se producen en ausencia de estímu-
1. Imágenes hipnopómpicas.
los desencadenantes (opción 1 correcta) y aparecen en el
2. Imágenes mnémicas.
espacio negro de los ojos cerrados (imágenes de fiebre) o en
3. Imágenes hipnagógicas.
el espacio físico externo, a causa de intoxicaciones o uso de
4. Imágenes parásitas.
drogas, en cuyo caso se llaman fantasiopsias. Se caracteri-
zan por ser subjetivas y autónomas, debido a que el sujeto
Respuesta correcta:3
no tiene control sobre ellas. Además pre- sentan una gran
plasticidad y el sujeto no les otorga valor de realidad (op-
Dentro las pseudopercepciones o imágenes anómalas en-
ción 2 incorrecta). Otro tipo de pseudopercepciones son las
contra- mos: 1) Alucinaciones fisiológicas, 2) Imágenes
imágenes mnémicas que se caracterizan por ser imágenes
alucinoides, 3) Imágenes mnémicas y 4) Imágenes conse-
de los recuerdos de los sujetos que pueden presentarse de
cutivas o post-imáge- nes y 5) Imágenes parásitas. Las alu-
modo transformado, se experimentan con poca nitidez y se
cinaciones fisiológicas son imágenes que se dan cuando el
desvane- cen si no se mantienen voluntariamente (opción
individuo está en estado de se- miconsciencia, entre la vigi-
4 incorrecta). Una variante de las imágenes mnémicas son
lia y el sueño. Existen dos subtipos: las hipnagógicas que se
las imágenes eidé- ticas o recordar sensorial (opción 3 inco-
producen en fase de adormecimiento (opción 3 correcta), y
rrecta), que son repre- sentaciones exactas de impresiones
las hipnopómpicas que se producen en el paso del sueño a
sensoriales vividas en algún momento por el sujeto, donde
la vigilia (opción 1 incorrecta). Las imágenes mnémicas y
el estímulo externo que produjo la percepción inicial ya no
las parásitas no se relacionan con el ciclo sueño vigilia, por
se halla presente. APIR Manual de Psicopatología, Tema 5.
lo que ambas respuestas serían incorrectas. APIR Manual
de Psicopatología, Tema 5.

40
146. ¿Cuál de las siguientes alteraciones se considera apa- recen cuando el individuo no fija su atención en ellas y
una distorsión perceptiva?: desapa- recen cuando se concentra en la experiencia. Sue-
len aparecer en estados de cansancio o fatiga extremos. El
1. Dismorfopsia. resto de opciones son incorrectas. La pareidolia es una ilu-
2. Imagen hipnagógica. sión que se caracteriza por dar significado a un estímulo
3. Pseudoalucinación. con poca estructuración o ambi- guo. Por ejemplo: formas a
4. Imagen autoscópica. las nubes (opción 1 incorrecta). En la alucinación refleja, un
estímulo provoca una alucinación en una modalidad sen-
Respuesta correcta:1 sorial diferente. Por ejemplo: sentir dolor cuando alguien
estornuda (opción 2 incorrecta). Una pseudoalucinación
Una distorsión perceptiva o sensorial es la percepción dis- es una experiencia sensoperceptiva que el sujeto identifi-
torsio- nada de un estímulo existente, mientras que un en- ca como subjetiva y perteneciente al mundo interno, con-
gaño percepti- vo es una experiencia perceptiva anómala servándose el juicio de realidad, pudiendo ser auditivas y
que no se fundamen- ta sobre estímulos realmente existen- visuales (opción 4 incorrecta). APIR Manual de Psicopato-
tes fuera del sujeto o que persiste cuando el estímulo que logía, Tema 5
la originó ha desaparecido. Las distorsiones perceptivas se
clasifican en función de la cualidad perceptiva que se en- 150. ¿Cuál de estas perturbaciones de la memoria cons-
cuentra afectada, encontrando las siguien- tes distorsiones tituye, más propiamente, una paramnesia del recuer-
perceptivas: 1) hiper o hipoestesias (afectan a la percepción do?:
de la intensidad del estímulo), 2) metamorfopsias (afectan
a la percepción del tamaño y la forma del estímulo), 3) ano- 1. Agnosia.
malías en la percepción de la cualidad del estímulo, 4) ano- 2. Confabulación.
malías en la integración perceptiva y 5) ilusiones. De entre 3. Amnesia reduplicativa.
las opciones de respuestas, la única que se considera una 4. Criptomnesia.
distorsión perceptiva es la dismorfopsia, que es un subti-
po de metamor- fopsia, consistiendo en la distorsión en la Respuesta correcta:2
percepción visual de la forma (opción 1 correcta). El resto
de opciones se consideran engaños perceptivos (opción 2, 3 El término paramnesia es introducido por Kraepelin para
y 4 incorrectas). APIR Manual de Psicopatología, Tema 5. definir las distorsiones de memoria en grado patológico,
debido a la inclusión de detalles falsos o por una referen-
cia temporal inco- rrecta. Freud denominó parapraxias a
147. Cuando una experiencia perceptiva se mantiene aquellos errores, aparen- temente sin importancia, como la
y/o se activa a pesar de que el estímulo inicial que la punta de la lengua o el olvido de nombres y fechas. Nor-
produjo ya no se encuentra accesible a los órganos sen- malmente se distingue entre las param- nesias del recuerdo
soriales, es muy probable que se experimente: y las del reconocimiento. Dentro de las ano- malías en el
recuerdo encontramos: 1) el fenómeno de “no puedo ubi-
1. Una pareidolia. carle”, 2) “conozco la cara pero no el nombre”, 3) sensación
2. Una alucinación refleja. de conocer, 4) punta de la lengua, 5) laguna temporal, 6)
3. Una imagen parásita. verifi- cación de tareas o checking y 7) pseudomemorias y
4. Una pseudoalucinación visual. falsificación de la memoria (hacen referencia a la elabora-
ción de recuerdos para rellenar lagunas mnémicas, donde
Respuesta correcta:3 se engloban la confabu- lación, la pseudología fantástica y
los recuerdos delirantes). De entre las opciones de respues-
El enunciado de la pregunta está haciendo alusión a las ta, la única que pertenece a una anomalía en el recuerdo es
pseu- dopercepciones o imágenes anómalas, que son un la confabulación (opción 2 correcta), que consiste en la in-
tipo de en- gaños perceptivos. Las pseudopercepciones son vención de recuerdos por parte del paciente sin la intención
anomalías mentales que pueden concebirse como imágenes de mentir, al intentar dar respuestas a cosas que no recuer-
que, o bien se producen en ausencia de estímulos desenca- da. Dicho fenómeno aparece en el síndrome amnésico. La
denantes concretos (forman parte de este grupo las imáge- amnesia reduplicativa y la criptomnesia son anomalías en
nes hipnagógicas, hipno- pómpicas y alucinoides), o bien se el reconocimiento (opciones 3 y 4 incorrectas). La agnosia
mantienen a pesar de que el estímulo que los produjo ya no es una alteración en el reconocimiento de objetos a través
esté presente (este grupo incluye las imágenes mnémicas, de los senti- dos, debido a una lesión del sistema nervioso
las parásitas y las consecutivas). De en- tre las opciones de central (opción 1 incorrecta). APIR Manual de Psicopato-
respuesta, la correcta es la imagen parásita (opción 3 co- logía, Tema 6.
rrecta), que se produce como consecuencia de un estímu-
lo concreto que ya no se halla presente cuando se produce 124. ¿Cómo se denomina al fenómeno por el que a una
la imagen. Estas imágenes se denominan parásitas porque palabra existente se le da un significado diferente al que

41
en realidad tiene?: lacionada íntimamente con él, ha sido reempla- zada por
un doble exacto. Por ejemplo, un individuo cree que su
1. Neologismo. madre ha sido reemplazada por un impostor. El síndrome
2. Palilalia. de Frégoli corresponde a la creencia de que una persona
3. Paralogismo. desconoci- da ha cambiado su apariencia para asemejarse a
4. Glosolalia. familiares o co- nocidos, con intenciones malignas (opción
3 incorrecta). El sín- drome de intermetamorfosis consiste
Respuesta correcta: 3 en la creencia delirante de que son los otros los que experi-
mentan cambios radicales en su identidad física y psicológi-
Los trastornos del lenguaje se pueden dividir en los siguien- ca, que culminan finalmente en una nueva persona comple-
tes grupos: ausencia del lenguaje, trastornos del ritmo, tras- tamente diferente, o dicho de otro modo, la creencia de que
tornos de la iteración, trastornos del significado y alteracio- personas de su ambiente han intercambiado sus identidades
nes de origen neurológico. Los trastornos de la significación (opción 1 incorrecta). El síndrome de los dobles subjetivos
afectan al signi- ficado del lenguaje, siendo la mayoría de es la creencia delirante en la existencia de duplicados físi-
ellos característicos de la esquizofrenia. En dicho grupo cos de uno mismo, y la creencia habitual que estas copias
encontramos: neologismos, paralogismos, glosomanía, glo- poseen identidades psicológicas diferentes (opción 4 co-
solalia, ensalada de palabras y descarrilamiento. Los neolo- rrecta). Oyebode, F. (2018). Síntomas mentales. Manual de
gismos son la creación y uso de pa- labras o expresiones sin psicopatolo- gía descriptiva. Capítulo 8.
un significado generalmente aceptado, a las que el paciente
les da una significación concreta. El paciente crea nuevas 143. ¿Cómo se denomina la utilización o creación de pa-
palabras a partir de la combinación de otras, siendo estas labras raras cuyo significado sólo conoce el individuo?:
palabras incomprensibles para los otros (opción 1 incorrec-
ta). Los paralogismos son la confusión del concepto que 1. Asintaxia.
expresa una palabra, de manera que a una palabra existente 2. Aproximación de palabras.
se le da un significado diferente al que en realidad tiene, 3. Neologismos.
que es la definición que nos pregunta el enunciado (opción 4. Alexia.
3 correcta). La gloso- manía es el conjunto de manifesta-
ciones verbales neoformadas acumuladas sin una sintaxis Respuesta correcta:3
concreta. La glosolalia es el lengua- je personal del enfer-
mo que utiliza un vocabulario y una sintaxis de su propia El enunciado se corresponde a la definición del concepto
invención, pero con un sentido propio. También se conoce de neo- logismos. Los neologismos son la creación y uso
como “criptolalia” (opción 4 incorrecta). La palilalia consis- de palabras o expresiones sin un significado generalmente
te en la repetición involuntaria de una palabra varias veces aceptado, a las que el paciente les da una significación con-
seguidas, siendo un trastorno que se engloba en el grupo de creta. El paciente crea nuevas palabras a partir de la com-
los trastornos iterativos del lenguaje (opción 2 incorrecta). binación de otras, siendo es- tas palabras incomprensibles
Vallejo, J. (2015). Introducción a la psicopatología y psi- para los otros, como por ejemplo: “mamérica” y “opecu”
quiatría. Capítulo 45. (opción 3 correcta). La aproximación de palabras se define
como el uso de palabras de forma no con- vencional o crea-
125. ¿Cómo se denomina al fenómeno que ocurre si un ción de pseudopalabras siguiendo las reglas de formación
paciente está convencido de que otra persona ha sido de las palabras del idioma, como por ejemplo, “for- mari-
trans- formada físicamente en sí mismo?: car” por “acto de salir del mar” y también “fornicar homo-
sexualmente”. Tanto los neologismos como la aproximación
1. Síndrome de intermetamorfosis. de palabras pueden evaluarse mediante la Escala de Tras-
2. Delirio de los dobles. tornos del Pensamiento, el Lenguaje y la Comunicación,
3. Síndrome de Fregolí. elaborada por Andreasen para evaluar el discurso indivi-
4. Síndrome de los dobles subjetivos. dual a través de la en- trevista clínica. Otro concepto a parte
es la alexia, que hace re- ferencia a la pérdida de capacidad
Respuesta correcta:4 para leer debido a una lesión cerebral, cuando ya fue adqui-
rida previamente (opción 4 inco- rrecta). APIR Manual de
Trastornos delirantes de falsa identidad. Los síndromes de Psicopatología, Tema 7.
falsa identificación delirante incluyen una serie de síndro-
mes, diferen- tes pero relacionados, que tienen en común 145. La afirmación “la presentadora de televisión me
el denominado con- cepto del doble. Estos síndromes in- quiere avisar de que corro un grave peligro y por eso se
cluyen el síndrome de Cap- gras, el síndrome de Frégoli, el ha puesto un vestido verde” es un ejemplo de:
síndrome de intermetamorfosis y el síndrome de los dobles
subjetivos. El síndrome de Capgras consiste en la creencia 1. Temple o humor delirante.
por parte del sujeto de que una persona, por lo general re- 2. Ocurrencia delirante.

42
3. Percepción delirante. el paciente se para en medio de una frase o idea y cambia
4. Representación delirante. el tema en respuesta a estímulos inmediatos. Por ejemplo:
“Entonces, dejé San Fran- cisco y me mudé a... ¿Dónde
Respuesta correcta:3 compró esa corbata? Parece como sacada de los años 50. Me
gusta el clima cálido de San Diego. ¿Es una concha lo que
Jaspers clasificó los delirios en dos tipos desde el punto de está sobre su escritorio? ¿Ha ido de bu- ceo alguna vez?”
vista de la forma, distinguiendo los delirios primarios de los (opción 4 correcta). El resto de opciones son alteraciones
secun- darios. Y a su vez, los delirios primarios los división del pensamiento que también pueden evaluarse me- diante
en cuatro grupos: atmósfera delirante, intuición delirante, la Escala de Trastornos del Pensamiento, el Lenguaje y la
percepción de- lirante y recuerdo delirante. La atmósfera Comunicación, elaborada por Andreasen para evaluar el
delirante, también de- nominado temple o humor deliran- discur- so individual a través de la entrevista clínica, pero
te, consiste en la experiencia subjetiva de que el mundo que no se co- rresponden con la definición del enunciado.
ha cambiado de un modo sutil pero siniestro, inquietante La presión del habla es el incremento en la cantidad de ha-
y difícil o imposible de definir. Se suele acompañar de un bla espontánea comparado con lo que se considera social-
estado de humor delirante, ya que el pacien- te se siente in- mente adecuado. El paciente habla rápidamente y es difícil
cómodo desasosegado o perplejo (opción 1 inco- rrecta). interrumpirle (opción 1 incorrecta). La tangencialidad se
La intuición delirante, también denominada ocurrencia o refiere a cuando se le realiza una pregunta al paciente y éste
inspiración delirante, se refiere a que el paciente descubre, responde de forma oblicua, tangencial o incluso irrelevan-
de repente, a modo de ocurrencia, el significado de las in- te. Por ejemplo: - ¿Cuántos años tienes? - Tantos como las
tuiciones o significaciones mal definidas, sin apoyarse en pirámides que se van desintegrando (opción 2 incorrecta).
una percepción o hecho concreto, a diferencia de la percep- La circunstancialidad se refiere a cuando el objetivo del dis-
ción delirante. El con- tenido de estas ideas suele ser auto- curso no se alcanza directamente, puesto que el hablante
rreferencial y de gran impor- tancia para el paciente. Por señala detalles tediosos y hace excesivos paréntesis (opción
ejemplo, un paciente cae en la cuenta de que las siglas de su 3 incorrecta). APIR Manual de Psicopatología, Tema 7.
nombre, Emilio Elosúa Albéniz de Darco, significan “Eres
El Asesino de Dios” (opción 2 incorrecta). La percepción 144. ¿Cómo se denomina la dificultad de la articulación
delirante consiste en la interpretación delirante de una per- y expresión de palabras producida por lesiones en el sis-
cepción normal. El paciente se da cuenta de que sabe algo y tema nervioso?:
comienza a estructurar aquello que antes sólo intuía, y que
ahora empieza a cobrar significado. Por ejemplo, descubre 1. Afasia.
que el color rojo de la comida es veneno. La afirmación 2. Dislalia.
del enuncia- do sería un ejemplo de percepción delirante, 3. Disglosia.
puesto que hay una percepción normal (veo que la presen- 4. Disartria.
tadora lleva un vestido ver- de) que se interpreta de manera
delirante (esto significa que me quiere avisar de que corro Respuesta correcta:4
un grave peligro), por lo que la opción 3 sería la correcta.
Y por último, el recuerdo delirante, también denominado La afasia se caracteriza por ser un trastorno del lenguaje
representación delirante, consiste en la reconstruc- ción de- que se produce como consecuencia de una lesión cerebral,
lirante de un recuerdo real o recordar algo claramente deli- siendo ésta de carácter focal y no difuso. Por tanto, es un
rante, por ejemplo, recordar que es el hijo de Dios (opción 4 trastorno adquirido en personas adultas que anteriormente
incorrecta). APIR Manual de Psicopatología, Tema 7. dominaban el lenguaje (opción 1 incorrecta). La dislalia es
la dificultad para articular fonemas, sílabas o palabras que
puede clasificarse en orgánica y funcional, en función de la
148. ¿Cómo se llama la alteración del pensamiento en etiología del trastorno. La dislalia orgánica es producto de
el que el paciente para de hablar a mitad de la frase y malformaciones orgánicas, de déficits sensoriales o motores
cambia de tema en respuesta a estímulos cercanos (por o alteraciones neurológicas. En dicho grupo encontramos
ejemplo, la corbata del entrevistador)?: por un lado las disglosias, que son altera- ciones o defectos
en la articulación de los fonemas producido por anomalías
1. Presión del habla. congénitas o adquiridas, en los órganos del habla (opción
2. Tangencialidad. 3 incorrecta); y por otro lado, las disartrias, que son al- te-
3. Circunstancialidad. raciones o defectos de la articulación debido a lesiones en
4. Habla distraída. el control muscular, que pueden deberse a una lesión en el
sistema nervioso central, periférico o en ambos (opción 4
Respuesta correcta:4 correcta). En la dislalia funcional no se conoce la etiología
responsable de la alteración en el manejo de los fonemas.
La alteración del pensamiento que plantean en el enuncia- Dentro de dicho grupo encontramos la dislalia, que es una
do es la definición de habla distraída, consistente en que alteración o defecto en la articulación producido por un

43
modo incorrecto de articulación, sin lesión orgánica (op- en- fermo propuesto por Parsons (1951), ¿cuál de los
ción 2 incorrecta). APIR Manual de Psi- copatología, Tema siguientes requisitos es necesario cumplir?
8. APIR Manual de Psicopatología y Clínica
Infantil, Tema 4. 1. Mostrar baja adherencia al tratamiento
2. Ser responsable de los síntomas
149. La flexibilidad cérea, el negativismo o la catalepsia 3. Implicarse en la recuperación de la salud
que pueden estar presentes como síntomas de catatonia 4. Continuar asumiendo las mismas responsabilidades
se- gúnDSM5-semanifiestan:
Respuesta correcta:3
1. Cuando el paciente es inducido a realizar ciertas accio
nes o movimientos. El estar enfermo, dice Parsons, no es sólo experimentar la
2. Cuando el paciente decide realizar un movimiento vo- condi- ción física de enfermedad. Estar enfermo más bien
luntario, ya que estos síntomas están bajo su control volun- constituye un rol social porque implica conductas basadas
tario. en expectativas ins- titucionales y, con frecuencia, ese rol
3. Cuando el paciente realiza un movimiento involuntario. social se ve reforzado por las normas de la sociedad que
4. Cuando el paciente está en reposo. corresponden a esas expectativas. El concepto de “rol de en-
fermo” de Parsons (1951) se basa en el supuesto de que estar
Respuesta correcta:1 enfermo no es una elección consciente y deliberada, aunque
una enfermedad pueda producirse como consecuencia de
La catatonía es un síndrome psicomotor que puede incluir una exposición intencionada a una infección o una lesión.
di- versos síntomas como: 1) catalepsia, que consiste en una Los requisitos que el enfoque de Parsons señala que deben
actitud inmóvil con la musculatura rígida que permite co- cumplirse para asumir el rol de enfermo son: a la perso-
locar a un in- dividuo en una posición forzada, incómoda na que enferma se le eximen de sus roles sociales normales
o antigravitatoria y que éste permanezca en la misma pos- (opción 4 incorrecta), la persona enferma no es responsable
tura sin intentar recuperar la posición original durante un de su condi- ción (opción 2 incorrecta), la persona enferma
tiempo indefinido, 2) flexibilidad cérea, que es una especial debe recuperar su estado de salud (opción 1 incorrecta) y la
forma de resistencia pasiva que se opone a la flexión de las persona que enferma debe buscar ayuda de expertos com-
articulaciones, 3) negativismo, que con- siste en una actitud petentes y colaborar en la recuperación (opción 3 correcta).
de oposición ante cualquier movimiento que se le ordena Marín, J. R., López, M. D. C. N. (2008). Manual de psicolo-
al individuo, pudiendo ser pasivo (actitud sistemáti- ca de gía social de la salud. Síntesis.
oposición) o activo (hacer precisamente lo contrario cuan-
do se le intenta obligar a que realice alguna acción), 4) es- 54. De las siguientes estrategias ¿cuál se dirige a detec-
tupor, que consiste en una actitud de permanecer ajeno o tar precozmente la existencia de una enfermedad?
distante del entorno, 5) mutismo, es decir, respuesta verbal
ausente o escasa, 6) estereotipias, que consisten en repeti- 1. Promoción de salud
ción continuada e inne- cesaria de movimientos concretos, 2. Prevención primaria
7) ecosíntomas, que consiste en la repetición automática de 3. Prevención secundaria
los movimientos (ecopraxia), las palabras (ecolalia) o los 4. Prevención terciaria
gestos (ecomimia) producidos por el in- terlocutor. Los tres
primeros síntomas enumerados anteriormen- te se presen- Respuesta correcta:3
tan cuando el paciente es inducido a realizar ciertas accio-
nes o movimientos (opción 1 correcta). En casos extremos, La prevención secundaria se dirige a la detección temprana
el mismo individuo puede oscilar entre una actividad mo- de los problemas de salud y la facilitación de cambios de
tora disminuida y una actividad motora excesiva. Los pa- conducta que permitan el tratamiento de las enfermedades
cientes ca- tatónicos a menudo salen súbitamente del estado (opción 3 co- rrecta). La prevención primaria se dirige a
estuporoso (in movilidad motora) para entrar en un estado prevenir la aparición de la enfermedad en pacientes sanos,
de agitación (raptus), durante el cual se puede producir lo reduciendo el riesgo de nuevos casos al intervenir sobre fac-
que se llama una tempestad de movimientos, como correr, tores de riesgo y protección (opción 2 incorrecta). La pre-
huir, gritar, golpear o atacar a per- sonas de alrededor. APIR vención terciaria se dirige a la re- cuperación de la salud y a
Manual de Psicopatología, Tema 10. la prevención de recaídas (opción 4 incorrecta). Por último,
la promoción de la salud se dirige a la adquisición y mante-
nimiento de hábitos saludables, es decir, a promover inmu-
nógenos conductuales (opción 1 incorrecta). APIR Manual
PSICOLOGÍA DE LA SALUD de Psicología de la Salud, Tema 2.

59. Para que se identifique a una persona en el rol de


60. ¿En cuál de las siguientes propuestas teóricas se in-

44
cluye la intención de conducta como un determinante PSICOTERAPIAS
proximal de la conducta?

1. Modelo creencias de salud (Rosenstock, 1974) 41. En la técnica del desafío de la Terapia estructural
2. Teoría del aprendizaje social (Bandura, 1986) (Minuchin):
3.Teoría de la acción razonada (Ajzen y Fishbein, 1970)
4.Modelo transteórico (Prochaska y Di Clemete, 1984) 1. Se necesita una fuerte alianza terapéutica, por lo que el
terapeuta mantiene una posición de cercanía.
Respuesta correcta:3 2. Se busca establecer unos límites rígidos entre los subsis-
temas familiares para revertir una triangulación de desvia-
En la teoría de la acción razonada, la intención conductual ción.
es el determinante inmediato de la conducta. La intención 3. El terapeuta da a escoger entre dos alternati vas, una de
conduc- tual, a su vez, está determinada por nuestras actitu- ellas es una tarea directiva y la otra una tarea mucho más
des hacia la conducta y nuestras normas subjetivas (opción onerosa que aquella.
3 correcta). El Modelo de Creencias en Salud en su versión 4. Es necesario intensificar y repetir el mensaje cuando la
original (Rosens- took, 1960), identifica básicamente cuatro inercia de los sistemas familiares les hace resistirse al cam-
variables que parecen influir en el sujeto a la hora de llevar a bio.
cabo una determinada acción preventiva. Estas variables se
agrupan en torno a dos di- mensiones generales de creen- Respuesta correcta:4
cias: La primera dimensión se re- fiere al grado de prepara-
ción o disposición psicológica del sujeto para llevar a cabo importante y la intensificación del mensaje del terapeuta.
una determinada acción preventiva que viene determinada Nor- malmente es necesario repetir muchas veces el men-
por la vulnerabilidad percibida y la gravedad per- cibida. La saje debido a la inercia de los sistemas familiares que les
segunda dimensión hace referencia a los beneficios/ costes hace resistirse al cambio (opción 4 correcta). La opción 3 se
percibidos de la acción preventiva. La idea principal que de- refiere a la ordalía (opción 3 incorrecta). La opción 1 se re-
fiende el modelo es que las conductas de una persona están fiere al procedimiento de mimetismo en el cual el terapeuta
determinadas por la amenaza percibida a su salud. Esta vul- pretende potenciar elementos de similitud con la familia
ne- rabilidad que se experimenta, a su vez, está condiciona- para favorecer la alianza terapéutica (opción 1 incorrecta).
da por la percepción acerca de la susceptibilidad personal a La opción 2 hace referencia a la técnica de reestructuración
la enfermedad y la severidad percibida de las consecuencias mediante la cual se fijan límites entre los sub- sistemas fa-
de adquirirlas. Un incremento en la percepción de vulnera- miliares para alterar la estructura familiar (opción 2 inco-
bilidad ante una amenaza para la salud produce una moti- rrecta). APIR Manual de Psicoterapias, Tema 6..
vación y un fortalecimiento de las conductas de protección
hacia esa amenaza. Si hay una dispo- sición alta para emitir 65. En un tratamiento psicológico orientado a la supe-
las conductas de protección y los costes de las mismas son ración de miedos, fobias y ansiedad en general, la técni-
bajos, es probable que se realice la conducta preventiva o ca de modelado más recomendable es:
de salud, al contrario, se omitirá la acción (opción 1 inco-
rrecta). La teoría de la autoeficacia de Bandura plantea que 1. De afrontamiento.
el factor principal de adhesión al tratamiento son las ex- 2. Asertivo.
pectati- vas de autoeficacia. La teoría del aprendizaje social 3. Con reproducción forzada.
de Bandura se apoya en los conceptos de expectativa de re- 4. Competente.
fuerzo, creencia de que con una determinada conducta en
un momento concreto puede obtener el refuerzo, y lugar de Respuesta correcta:1
control, expectativa que tie- ne el sujeto sobre el control que
tiene sobre su conducta o sobre que ésta dependa de factores Las técnicas de modelado tienen como objetivo enseñar o
externos a él (opción 2 incorrecta). El modelo transteórico cam- biar una conducta a partir de la observación de la con-
del cambio entiende la conducta adictiva como un continuo ducta de los demás y las consecuencias que le siguen. Exis-
en la intención de abandonar una adicción. Se trata de un ten variantes del modelado, en función de la competencia
modelo tridimensional integrado por estadios, proce- sos y del modelado en- contramos: 1)Modelado mastery/domi-
niveles de cambio. Enfatiza factores de tratamiento como la nio: el modelo posee las habilidades precisas para desempe-
motivación al cambio (opción 4 incorrecta). APIR Manual ñarse correctamente desde el principio. Realiza la conducta
de Psicología de la Salud, Tema 2. de una forma relajada. Se utili- za para aprender conduc-
tas motoras (opción 4 incorrecta). 2) Modelado coping/de
afrontamiento: el modelo empieza con un nivel similar al
del observador y, poco a poco, va mostrando las habilidades
necesarias para resolver la situación. Se muestra ansioso al
principio y relajado al final. Se utiliza en problemas de an-

45
siedad (opción 1 correcta). Otras variantes del modelado, Las técnicas de modelado tienen como objetivo enseñar
según el grado de dificultad de la conducta a modelar: 1) o cam- biar una conducta a partir de la observación de la
Mo- delado gradual: el modelo va realizando aproximacio- conducta de un modelo y las consecuencias que le siguen.
nes a la conducta meta. 2)Modelado con reproducción re- Existen variantes en función del objetivo que se tenga: Si el
forzada: se van reforzando las conductas progresivamente objetivo del procedi- miento del modelado es que el obser-
más difíciles (opción 3 incorrecta). Por otro lado, el mode- vador aprenda que tras rea- lizar una determinada conducta
lado asertivo es aquel que se utiliza en el contexto de adqui- no le siguen consecuencias ne- gativas, es decir, que existe
sición de habilidades sociales (opción 2 incorrecta). APIR un efecto desinhibitorio, estaremos hablando de extinción
Manual de Psicoterapias, Tema 8. vicaria (opción 1 correcta). Hablaremos de modelado ne-
gativo cuando el modelo realiza una conducta inadecuada
66. Señale cómo se denomina la intervención que con- (opción 4 incorrecta). El moldeamiento vicario es un pro-
siste en la repetición mental de fórmulas verbales (fra- ceso mediante el cual, el sujeto a través de la observación de
ses ela- boradas) sobre sensaciones psicofisiológicas un modelo, aprende a realizar una conducta nueva a partir
durante breves periodos de tiempo, además de una con- del refuerzo sucesivo de aproximaciones a esa conducta y la
centración pasiva en dichas sensaciones: extin- ción de tales aproximaciones a medida que se va con-
figurando la conducta final deseada (opción 2 incorrecta).
1. Autorregulación emocional. El castigo negativo vicario es un procedimiento mediante el
2. Entrenamiento autógeno. cual se observa la reti- rada de un estimulo reforzante tras
3. Entrenamiento en autoinstrucciones y respiración. la emisión de una conducta que se desea reducir (opción 4
4. Entrenamiento en consciencia plena (mindfulness). incorrecta). APIR Manual de Psi- coterapias, Tema 8.

Respuesta correcta:2 68. La desensibilización por contacto es una variante de


la desensibilización sistemática, en la cual:
Existen varios procedimientos mediante los cuales la per-
sona aprende a controlar su propio nivel de activación, 1. El terapeuta utiliza instrucciones verbales y guía física
entre ellos en- contramos técnicas como: Entrenamiento para que el cliente realice la exposición con mayor facilidad.
autógeno, mediante el cual la persona aprende a asociar 2. El terapeuta utiliza, en el proceso de desensibilización,
determinadas frases con re- presentaciones de sensaciones elementos adicionales para favorecer la imaginación de es-
corporales. Se realizan dos ciclos de ejercicios, el primero se cenas, como fotografías, diapositivas o grabaciones en au-
dirige a la desactivación fisiológica y el segundo se dirige a dio.
controlar los estados mentales (opción 2 correcta). Respi- 3. El terapeuta actúa como modelo usando, incluso, el con-
ración, mediante el cual se pretende enseñar un adecuado tacto físico con el cliente (sosteniendo la mano del cliente o
control voluntario de la respiración, para después automa- colocando su mano en su espalda) mientras éste realiza la
tizar este control y generalizarlo a situaciones estresantes exposición.
(opción 3 incorrecta). El mindfulness o entrenamiento en 4. El terapeuta facilita que el cliente realice contacto físi- co
cons- ciencia puede concebirse como una terapia en sí mis- con los estímulos fóbicos (tocándolos con sus propias ma-
ma o den- tro de un programa multicomponente. Se deriva nos) para así aumentar la eficacia de la exposición.
de las prácticas budistas y busca modificar la forma en la
que el individuo se relaciona con su experiencia y reacciona Respuesta correcta:3
ante los pensamientos o sentimientos desagradables (op-
ción 4 incorrecta). La autorregu- lación emocional es la ca- La desensibilización sistemática es una técnica que se utiliza
pacidad para experimentar emociones de forma moderada para reducir respuestas de ansiedad y eliminar las conduc-
y flexible, así como la capacidad para ma- nejarlas (opción 1 tas de evitación asociadas a dicha ansiedad. Se lleva a cabo a
incorrecta). APIR Manual de Psicoterapias, Tema 8. través del entrenamiento en el uso de respuestas incompati-
bles con la ansiedad de forma que se impide que ésta se de-
67. Cuando en la técnica de modelado, el modelo mues- sarrolle ante es- tímulos fóbicos y que se pongan en marcha
tra conductas evocadoras de ansiedad que no conllevan conductas de evi- tación. Existen variaciones técnicas que
con- secuencias negativas, el terapeuta intenta conse- comparten los mismos principios que la desensibilización
guir: pero difieren en el modo de aplicación: 1) La desensibiliza-
ción por contacto combina la des- ensibilización sistemática
1. Extinción vicaria. con el modelado. El terapeuta actúa como modelo usando,
2. Moldeamiento vicario. incluso, el contacto físico con el cliente mientras éste realiza
3. Castigo negativo vicario. la exposición. Se le llama también mode- lado participati-
4.Modelado negativo. vo o participante y se utiliza en niños. La carac- terística
fundamental de la DS por contacto es la combinación de
Respuesta correcta: 1 DS con el Modelado. Las definiciones de los manuales de
re- ferencia hacen hincapié en esta característica, hasta el

46
hecho de subrayar que esta técnica estaría a medio camino 3. El uso de técnicas experienciales para evocar experien
entre la DS y el modelado. Según Olivares, el terapeuta se cias infantiles traumáticas.
expondría (actuando como modelo) a los estímulos que ge- 4. Reelaborar las experiencias de transferencia negativa.
neran ansiedad y luego lo haría el paciente. Según Labrador
el terapeuta funciona como modelo exponiéndose a los es- Respuesta correcta:1
tímulos antes que el sujeto, sien do el terapeuta muy activo,
pudiendo incluso ayudar al paciente con guía física cuando La terapia cognitiva de Young se deriva de la TC de Beck
éste realiza la exposición. En todo caso, y se aplica en personas con trastornos de personalidad. El
la característica esencial de esta técnica es la combinación objetivo de ésta es lograr una relación terapéutica colabo-
de la 3. DS con el modelado, tal y como aparece en la opción rativa, modificar creencias rígidas y lograr reconocer las
de respuesta (opción 3 correcta). 2) La desensibilización emociones y cogniciones que se derivan de estas creencias
enriquecida es aquella en la que el terapeuta utiliza, en el para desarrollar un modo adul- to saludable del paciente
proceso de desensibi- lización, elementos adicionales para logrando la remisión o el control de
favorecer la imaginación de escenas, como fotografías, dia- los modos disfuncionales. Esto implica que el trabajo se
positivas o grabaciones de audio (opción 2 incorrecta). 3) centre más en la reestructuración cognitiva de los ETD con
La práctica reforzada consiste en ir reforzando al cliente en la poste- rior implicación que esto tendría a nivel conduc-
las conductas de aproximación gradual a los objetos temi- tual y emocional (opción 1 correcta; opción 2 incorrecta).
dos hasta que pueda permanecer en contacto con ellos sin Young, en su terapia, describe diferentes esquemas disfun-
escapar y sin evitarlos, no actuando el terapeuta como mo- cionales tempranos (ETD) que se agrupan en 5 dimensio-
delo. Se apoya en una serie de incitadores como son las ins- nes relacionadas con la no satisfac- ción de las necesidades
trucciones verbales, guía física, extinción de las conductas emocionales centrales. En función del esquema disfuncio-
fóbicas y feedback de ejecución (opciones 1 y 4 incorrectas). nal que se active en cada momento, estaremos hablando de
APIR Manual de Psicoterapias, Tema 8. un modo de esquema que va a determinar el fun- ciona-
miento del paciente (a nivel de emociones, pensamiento y
71. Las escenificaciones emotivas, como procedimiento conductas). Para llevar a cabo el proceso de cambio de los
de tratamiento de la ansiedad en niños, constan de: esquemas tempranos disfuncionales utiliza técnicas cogni-
tivas, experienciales, conductuales y técnicas que trabajan
1. Tres componentes: exposición en vivo, modelado parti- con la rela- ción paciente-terapeuta. No se centra sólo en el
cipante y práctica reforzada. uso de técnicas experienciales (opción 3 incorrecta). APIR
2. Tres componentes: terapia de juego, exposición en vivo y Manual de Psicote- rapias, Tema 9.
reforzamiento diferencial.
3. Un componente fundamental basado en la exposición 183. Un terapeuta intenta abordar una depresión cró-
con prevención de respuesta. ni- ca mediante la Terapia centrada en los esquemas de
4. Tres componentes: desensibilización, reforzamiento di- Young (1990). Señale cuál de estos pasos NO correspon-
ferencial y moldeamiento. de a esta terapia:

Respuesta correcta:1 1. Explorar los orígenes de los modos del esquema en la ni-
ñez y la adolescencia.
La técnica de escenificaciones emotivas fue creada por 2. Usar la imaginería para tener acceso al modo del esque-
Méndez (1986). Consiste en la exposición gradual en vivo ma del niño vulnerable.
con el estímu- lo fóbico en el transcurso de un juego de rol o 3. Solicitar un registro diario de pensamientos automáti cos
representación de papeles. La relación con el estímulo fóbi- disfuncionales asociados a los esquemas.
co se facilita mediante instrucciones, guía física, modelado 4. Propiciar y conducir diálogos entre los diferentes mo dos
y autoinstrucciones, y se consolida mediante reforzamiento de los esquemas.
social y material suministra- do en el marco de un sistema
de economía de fichas. Se des- criben como componentes Respuesta correcta:3
principales la exposición en vivo, el modelado participante
y la práctica reforzada (opción 1 correcta; opciones 2, 3 y 4 El proceso terapéutico en la terapia centrada en esquemas
incorrectas). APIR Manual de Psicoterapias, Tema 8. se articula en relación a dos fases principales que recogen la
inte- gración de todo tipo de técnicas, desde técnicas cogni-
51. El tratamiento de los trastornos de la personalidad tivas a con- ductuales, pasando por técnicas experienciales
basado en el modelo de J.E.Young, se basa en: y por el manejo de la relación paciente-terapeuta. En la pri-
mera fase, el objetivo es identificar y activar sus esquemas
1. Promover cambios en los esquemas tempranos disfun desadaptativos tempranos. Se le explica al cliente el papel
cionales o desadaptativos (ETD). de los esquemas desadaptativos tempranos en el manteni-
2. Promover cambios en las conductas que se derivan de miento de los modos de esquema (op- ción 1 incorrecta por
los ETD. ser correcta). Para ello, se utilizan varias estrategias: técni-

47
cas experienciales para activar los esquemas como puede 2. La aceptación.
ser la imaginería (opción 2 incorrecta por ser co- rrecta), 3. El contacto con el momento presente.
discusión sobre los patrones de conducta que provocan los 4. El yo como contexto.
esquemas (opción 4 incorrecta por ser correcta), etc. En la
fase de cambio de los esquemas desadaptativos tempranos Respuesta correcta:0
de- tectados anteriormente se utilizan técnicas cognitivas,
experien- ciales, conductuales...No obstante, no se solicita ANULADA. Esta pregunta aparece anulada en la plantilla
un registro dia- rio de pensamientos automáticos disfun- pro- visional, manteniéndose así en la definitiva también.
cionales asociados a los esquemas; esto es más característi- La opción que podría ser correcta es la 1, la defusión cogni-
co de la terapia cognitiva de Beck (opción 3 correcta). APIR tiva. Tenien- do en cuenta la bibliografía existente referen-
Manual de Psicoterapias, Tema 9. te a la Terapia de Aceptación y Compromiso (ACT), en el
manual de Tratamien- tos psicológicos para la psicosis de
31. En un tratamiento psicológico el terapeuta pide al Fonseca (2019) se hace refe- rencia a lo siguiente: “ Ejem-
cliente que analice cómo han sido las soluciones que ha plos de ejercicios de defusión serían la desliteralización de
veni- do aplicando a su trastorno hasta el momento. Le palabras, el etiquetado de pensamientos (“estoy teniendo el
pide valo- rar si le han resultado exitosas, ineficientes, o pensamiento de que nada me sale bien” o “estoy oyendo vo-
incluso perju- diciales. Está aplicando: ces”) y la contradicción del pensamiento con la acción (de-
cir “no puedo caminar cuando de hecho lo haces)”. Hay que
1. La reconstrucción del estilo afectivo de la te rapia cons- tener en cuenta que en el enunciado de la pregunta fi- gura
tructivista. el término “contracción” en lugar de “contradicción” como
2. La fase de desesperanza creativa de la terapia de acepta- aparece en dicho manual, razón por la cual se anula la pre-
ción y compromiso. gun- ta. Siguiendo también a Marino Pérez en su manual de
3. La teoría del guión del análisis transaccional. Terapias de Tercera generación encontramos lo siguiente:
4. La proyección temporal de la terapia cogniti va. “La defusión trata de crear contextos de desliteralización
que disminuyan las funciones regulatorias innecesarias del
Respuesta correcta:2 lenguaje y de facilitar el contacto con los eventos en curso”.
La idea es disminuir la credi- bilidad de los propios even-
La fase de desesperanza creativa es un concepto nuclear en tos privados y la “fusión” de uno con ellos”. FONSECA, E.
la terapia de aceptación y compromiso y refleja la inutilidad (2019): Tratamientos psicológicos para la psicosis. Madrid:
de los intentos que ha realizado el paciente para solucio- Pirámide, pág.15. Pérez Álvarez, M. (2014): Las terapias
nar sus pro- blemas de forma que éste busque otras alter- de tercera generación como terapias contextuales. Síntesis.
nativas para ello (opción 2 correcta). Teniendo en cuenta Madrid.
que el enunciado de la pregunta se refiere al componente
de desesperanza creativa, de la ACT, terapia perteneciente 57. En la Terapia de aceptación y compromiso aplicada
a las terapias de tercera genera- ción, descartamos la op- al dolor crónico ¿a qué se refiere la fase de “desesperan-
ción 1 referente a la reconstrucción del estilo afectivo. Esto za creativa”?:
es propio de las terapias constructivistas y consiste en que
a través de la relación terapéutica se lleve a cabo una re- 1. A la toma de conciencia de la falta de eficacia que han
construcción del sistema cognitivo del individuo (opción 1 tenido las estrategias que el paciente ha aplicado hasta el
incorrecta). La teoría del guión hace referencia a las pau- momento para el control del dolor.
tas de comportamiento arraigadas y juegos que se repiten, 2. A ayudar al paciente a ver sus pensamientos y senti-
relacionados con una especie de programación paterna en mientos desde la perspectiva de un observador.
la infancia. Pertene- ce al Análisis Transaccional de Berne, 3. A enseñar al paciente a identificar los valores importan-
terapia de corte humanis- ta (opción 3 incorrecta). La pro- tes de su vida.
yección temporal es una técnica cognitiva destinada a que 4. A ayudar al paciente a desarrollar planes de conducta
el paciente consiga cierta objetividad, situando las imágenes para cambiar su vida.
de gran contenido emocional en un mo- mento distinto al
actual (opción 4 incorrecta). APIR Manual de Psicotera- Respuesta correcta:1
pias, Tema 10.
La fase de desesperanza creativa es uno de los componen-
35. Dentro de la Terapia de aceptación y compromiso tes de la terapia de aceptación y compromiso y refleja la in-
para la psicosis, ¿qué proceso psicológico y terapéutico utilidad de los intentos que ha realizado el paciente para
se trabaja cuando se realizan ejercicios como la deslite- solucionar sus problemas con el objetivo de buscar otras
raliza- ción de palabras, el etiquetado de pensamientos alternativas para so- lucionarlo (opción 1 correcta). Algu-
y la contrac- ción del pensamiento con la acción?: nos de los objetivos en los que se centra la ACT son la bús-
queda e identificación de los valores importantes de la vida
1. La defusión. del paciente (opción 3 incorrecta) y a ayudar al paciente a

48
desarrollar planes de conducta en com- promiso con sus va- Respuesta correcta:1
lores personales (opción 4 incorrecta). Otro eje fundamen-
tal de la ACT es la defusión cognitiva y consiste en ayudar La TIP es una terapia breve, focalizada en el terreno de lo
al paciente a ver sus pensamientos y sentimientos desde la inter- personal, cuyo principal objetivo es la mejora de las
perspectiva de un observador (opción 2 incorrecta). APIR relaciones interpersonales centrándose en 4 áreas proble-
Ma- nual de Psicoterapias, Tema 10. máticas: 1)Duelo: pérdidas de personas significativas. 2)
Disputas interpersonales. 3)Transición de rol: renunciar al
63. Si un psicólogo/a clínico emplea las técnicas de “min- rol actual, expresar sentimien- tos de culpa, enfado o pérdi-
dfulness desapegado”, “entrenamiento en atención” y da, adquirir habilidades y establecer nuevos vínculos. 4)Dé-
“reorientación” (o “refocalización”) de la atención si- ficits interpersonales. En el manual origi- nal de la Terapia
tuacio- nal”, en qué modelo de psicoterapia se estaría Interpersonal de Klerman y Weissman los con- ceptos uti-
basando: lizados para nombrar las cuatro áreas problema son: Grief,
Role dispute, Role transition, Interpersonal deficits; y en la
1. Terapia metacognitiva (Wells). versión traducida del manual original los conceptos utiliza-
2. Terapia de aceptación y compromiso (Hayes y cols.). dos son: Duelo, disputas interpersonales de rol, transiciones
3. Psicoterapia analítico funcional (Kholenberg y Tsai). de rol y déficit interpersonales. El desarrollo de la TIP cons-
4. Terapia cognitiva basada en mindfulness para la depre- ta de 3 mo- mentos: 1. Fases iniciales: durante estas sesiones
sión se realiza una revisión minuciosa de los síntomas, se le da
nombre al síndrome y se instruye sobre la depresión, po-
Respuesta correcta:1 niéndola en relación con su contexto interpersonal. En este
momento se puede otorgar al pa- ciente el rol de enfermo
La terapia metacognitiva (TMC) de Wells y la TC (terapia y valorar la necesidad de medicación. 2. Fases intermedias:
cogni- tiva) comparten la importancia que le dan al procesa- se trabajan las 4 áreas problemáticas en función de lo eva-
miento men- tal en los trastornos psicológicos. Sin embar- luado en las fases iniciales. 3. Fases de termi- nación: la fi-
go, la TMC se dirige más a los procesos que a los contenidos nalización de la terapia debe ser abordada explíci- tamente,
cognitivos. La TMC conserva el uso de estrategias típicas reconociéndola como un momento de posible duelo. APIR
de la TC (cuestionamiento socrático, experimentos cogni- Manual de Psicoterapias, Tema 10
tivos...) dirigidas a modificar me- tacreencias con el tras-
torno mientras que la TC estándar lo hace con las creencias 1. J. Frank es uno de los autores más importantes en el
ordinarias. La TMC se focaliza en el estilo del pensamiento estudio de los “factores comunes” en psicoterapia. Se-
y formula el síndrome cognitivo atencional. Inclu- ye es- ñale cuál de los siguientes es uno de los cuatro “factores
trategias de entrenamiento atencional y conciencia plena comu- nes” que propone como características de todas
“desapegada”. Dado que para el modelo metacognitivo, la (o muchas) las formas de psicoterapia (Frank y Frank,
pato- logía psicológica está relacionada con la activación de 1991):
un estilo de pensar tóxico, el síndrome cognitivo atencional,
que consiste en un estilo perseverativo de pensamiento que 1. “Un ritual” o procedimiento que requiere la participa-
toma la forma de preocupación o rumiación, foco atencio- ción activa del terapeuta y del cliente.
nal en la amenaza y estrategias de afrontamiento contrapro- 2. Un “Condicionamiento operante” a través del cual el te-
ducentes, se propone la atención plena como un modo de rapeuta expresa aprobación o desaprobación de la conducta
impedir la activación del sín- drome cognitivo atencional del paciente.
(opción 1 correcta). APIR Manual de Psicoterapias, Tema 3. Una “regulación conductual” o “experiencia correcti- va”
10. basada en la capacidad de sugestión y persuasión del tera-
peuta.
32. El tratamiento con la Terapia interpersonal (Kler- 4. “Factores del terapeuta” como directividad, empatía, au-
man, Weissman y cols. 1979) se enfoca en resolver las toridad o aceptación positiva incondicional.
si- guientes cuatro áreas de problema interpersonal re-
laciona- das con el inicio o mantenimiento del episodio Respuesta correcta:1
depresivo del paciente:
Desde los años 90 está ganando terreno una visión meta-te-
1. Duelo, disputa de roles, transiciones de roles, déficits in- ra- péutica y surgió la pregunta: “Si funcionan las terapias,
terpersonales. pero no tiene que ver con sus idiosincrasias ¿cuáles son los
2. Duelo, culpa, pérdida, déficits interpersonales. factores tera- péuticos comunes?”. La obra de Frank sentó
3. Pérdida, conflicto de roles, transición de roles, déficits la base de un análi- sis continuado de los factores comu-
interpersonales. nes en toda la familia de pro- yectos orientados a producir
4. Duelo, culpa, transición de roles, déficits interpersonales. curación (“healing”) de trastornos o problemas psicológi-
cos. Comparó psicoterapia tradicional, tera- pias de grupo
y familiar, tratamiento intraclínico, drogoterapia, medicina,

49
curación religiomágica en sociedades no-industriali- zadas, 4. Control.
cultos y renacimientos. Su conclusión fue que los aspec- tos
compartidos por todas las terapias eficaces son cuatro: 1) Respuesta correcta:2
Una relación de confianza y con contenido emocional con
la persona que quiere ayudar. 2)Un entorno “sanador”. 3) El conservadurismo cognitivo hace referencia a la tenden-
Un esquema con- ceptual, razón o mito que ofrece una ex- cia de la cognición humana a ser conservadora, ya que in-
plicación plausible de los síntomas del paciente y prescribe tentamos preservar aquello que ya está establecido, a man-
un ritual o procedimiento para su solución. 4) Un ritual o tener nuestros conocimientos, actitudes e hipótesis previas
procedimiento que requiere la partici- pación activa tanto (Opción 1 incorrec- ta). El heurístico de Disponibilidad se
del paciente como del terapeuta y que ambos consideran basa en la accesibilidad de cierta información a la memoria
el medio para restaurar la salud (opción 1 correcta). APIR y supone la priorización de características más fácilmente
Manual de Psicoterapias, Tema 11. accesibles ante la evaluación de una situación de tipo social.
En este caso, la alarma creada por el brote de listeriosis llevó
a la disminución del consumo de ciertos tipos de productos
por la facilidad con la que aparecía la infor- mación sobre la
PSICOLOGÍA SOCIAL posibilidad de enfermar (opción 2 correcta). La actitud se
define como una tendencia psicológica que implica la eva-
175. ¿Cuál de los siguientes NO es un paradigma de in- luación de favorabilidad o desfavorabilidad hacia un objeto
vestigación sobre disonancia cognitiva?: (opción 3 incorrecta). En cuanto a la opción 4 (incorrec-
ta), el concepto de control puede ser muy amplio; el control
1. El paradigma de justificación del esfuerzo. cognitivo, que sería el concepto más relacionado con las
2. El paradigma de complacencia inducida. otras opciones de respuesta, es una entidad de naturaleza
3. El paradigma adaptación-nivel. psicodinámica que regula la expresión de las necesidades
4. El paradigma de libre elección en modos socialmente adaptati- vos. Es decir, los controles
cognitivos actúan en función de los requerimientos de la
Respuesta correcta:3 situación (variables intermedias entre las necesidades y la
percepción). Surgirían a lo largo del desarrollo, fruto de la
Festinger (1957) abre diferentes áreas de investigación con interacción entre los genes y el medio ambiente. APIR Ma-
el fin de comprobar su teoría sobre la disonancia cognitiva. nual de Psicología Social, Tema 2.
Estas áreas son: 1) Paradigma de la complacencia induci-
da (opción 2 incorrecta), analiza los problemas originados 168. Según el modelo de covariación de Kelley ¿Qué
cuando un sujeto se ve obligado a llevar a cabo acciones que ti- pos de información utilizan los sujetos para realizar
van en contra de sus principios. 2) Paradigma de la elec- atribu- ciones sobre las causas de la conducta de otra
ción libre (opción 4 incorrec- ta), se basa en el análisis de persona?:
la disonancia generada tras decidir entre dos alternativas,
también llamada disonancia postdecisio- nal. 3) Exposición 1. Locus externo/interno, estabilidad y controla bilidad.
selectiva a la información, parte de la premisa que tende- 2. Coherencia/consistencia, distintividad y con senso.
mos a buscar las informaciones que reducen la diso- nancia, 3. Efectos no comunes, relevancia hedónica y personalismo.
esto es, llevamos a cabo una exposición selectiva de la infor- 4. Correspondencia, agencialidad y causalidad.
mación favorable en oposición a la no favorable a nuestras
creencias. 4) Paradigma de justificación del esfuerzo (op- Respuesta correcta:2
ción 1 incorrecta), se basa en la disonancia generada tras
llevar a cabo un esfuerzo que no merece la pena. La opción El locus de causalidad, estabilidad y controlabilidad forman
3 (correcta), no se corresponde con ninguna de las áreas las tres dimensiones con las que Weiner desarrolla su Teo-
propuestas para la investgación sobre la disonancia cogniti- ría de la atribución (opción 1 incorrecta). Kelley planteó un
va. APIR Manual de Psicolo- gía Social, Tema 1. modelo de covariación en el que una persona atribuye la
causa de una con- ducta o acción a la persona que realiza la
7. La alerta alimentaria producida tras un brote de lis- acción, a la entidad u objeto a la cual va dirigida la acción
teriosis asociada a una marca de carne mechada produ- o a las circunstancias que la rodean. Esta atribución va a
jo una drástica disminución del consumo no solo de este depender fundamentalmente de tres factores: Coherencia/
tipo de productos sino de otros similares. El juicio del consistencia, distintividad y consenso (op- ción 2 correc-
consumidor estuvo afectado por uno de los heurísticos ta). La opción 3 hace referencia a conceptos pro- pios de
descritos por Kah-neman y Tversky, el heurístico de: la Teoría de las inferencias correspondientes de Jones y
Davis. El Principio de los efectos no comunes dice que el
1. Conservadurismo cognitivo. sujeto que percibe realizará la inferencia correspondiente
2. Conservadurismo cognitivo. con mayor confianza cuando la acción elegida tenga pocas
3. Actitud. consecuencias únicas o no comunes, es decir, pocas carac-

50
terísticas diferencia- doras. La relevancia hedónica se refie- Respuesta correcta:1
re a que cuanto más nos afecta lo que hace un sujeto, mayor
probabilidad de atribución a factores internos o personales A partir de los planteamientos de McGuire sobre la relevan-
del mismo. Y el personalismo consiste en la evaluación a cia de la aceptación del mensaje, Petty y Cacioppo (1986)
otro individuo de forma más extrema, sobre todo de forma desarro- llaron el “Modelo de la Probabilidad de la Elabora-
negativa, si consideramos que su conducta estaba dirigida ción”. Sus estudios sugieren que la evaluación de un mensaje
personalmente a nosotros (opción 3 incorrecta). La opción persuasivo se lleva a cabo a través de dos formas diferen-
4 (incorrecta) habla de conceptos no relacionados con esta tes de procesamiento de la información: la ruta central y la
teoría. APIR Manual de Psicología Social, Tema 2. ruta periférica. A través de la ruta central se ha observado
que los receptores gastan bas- tante tiempo y esfuerzo en
169. Según el modelo de la Teoría de la acción razona- la evaluación crítica del contenido del mensaje. A través
da, ¿qué componente determina la actitud general ha- de la ruta periférica el sujeto tiende a usar esquemas sim-
cia una conducta?: ples o reglas de decisión para la evaluación de la validez del
argumento y la toma de decisión, usa un procesa- miento
1. La intención conductual. heurístico (opción 1 correcta). El modelo MODE, co- no-
2. Creencia que la persona tiene sobre las consecuencias de cido como “Motivación y Oportunidad como factores De-
la conducta. ter- minantes”, estudia la influencia de la actitud sobre la
3. La norma subjetiva. conducta centrándose en la importancia de la accesibilidad
4. Las creencias normativas. actitudinal. El modelo plantea dos formas fundamentales
de influencia de la actitud en la conducta: 1) Procesamiento
Respuesta correcta:2 espontáneo, se da una “activación automática de la actitud”,
lo que presupone una alta accesibilidad de la misma. 2) Pro-
Según la Teoría de la acción razonada, el determinante in- ceso deliberativo de larga dración, este proceso supone un
media- to de que el sujeto ejecute o no la “conducta” final análisis detallado y cuidadoso de la información. (opción
va a depender directamente de la “intención conductual” 2 incorrecta). El modelo de Compor- tamiento Planifica-
(opción 1 incorrecta). Ésta a su vez va a depender, por un do o Acción Planificada de Ajzen y Madden (1986) es una
lado, de la evaluación positi- va o negativa que el sujeto extensión de la Teoría de la Acción Razonada, un modelo
hace sobre la ejecución de la conduc- ta, lo que se conoce unidimensional centrado en el componente evaluativo de
como “actitud hacia la conducta” (opción 2 correcta). Y por la actitud. Junto a los tres componentes de la teoría ante-
otro lado, va a depender también del juicio que el sujeto rior, actitud hacia la conducta, norma subjetiva e intención,
realice sobre las probabilidades de que otros individuos im- incluye un elemento más, el control conductual percibido
portantes para él deseen que lleve a cabo dicha conducta, o (opción 3 in- correcta). La opción 4 (incorrecta), no repre-
lo que es lo mismo, de la “norma subjetiva” (opción 3 inco- senta ningún modelo relacionado con las actitudes ni con el
rrecta). Esta norma subjetiva se ve influida a su vez por dos proceso persuasivo. (opción 4 incorrecta). APIR Manual de
aspectos: por un lado las “creencias normativas”, es decir, Psicología Social, Tema 3.
creencias que el sujeto tiene respecto a lo que las personas
relevantes de su entorno piensan que debería hacer; y por 173. Cuando un sujeto se conforma a las expectativas de
otro, la “motivación para acomodarse” a tales expectativas, una grupo debido a su temor a la desaprobación social,
esto es, en qué medida el individuo se deja llevar por lo que al castigo del grupo, se está produciendo un proceso de:
opinan los seres significativos de su entorno sobre lo que
debería hacer (opción 4 incorrecta). En este sentido, el mo- 1. Influencia informativa.
delo afirma que si las creencias normativas son firmes pero 2. Conversión.
la motivación para acomodarse a ellas es inexis- tente, la 3.Influencia normativa.
norma social subjetiva no podrá influir en la intención de la 4.Reactancia psicológica.
realización de la conducta. La norma subjetiva es conside-
rada como el producto de los dos factores ya mencionados, Respuesta correcta: 3
por lo que si uno de ellos falla el producto final sería igual a
cero. APIR Manual de Psicología Social, Tema 3. Cuando una persona se conforma a través de un proceso
de in- fluencia normativa, es previsible que sea su conducta
172. ¿Cuál de los siguientes modelos establece la existen manifiesta lo único que cambie, y no su convicción previa,
cia de una “vía central” y una “vía periférica” en los que continuará manteniéndose en privado; dando lugar a
proce- sos de persuasión: un proceso de con- formidad pública o sumisión. Tras este
tipo de influencia suele haber gran presión social (opción 3
1. El modelo de probabilidad de elaboración. correcta).Cuando se confor- ma en base a la influencia in-
2. El modelo de activación automática. formativa, también se modificará su opinión; dando lugar a
3. El modelo del comportamiento planificado. un proceso de conformidad privada o conversión (opciones
4. El modelo de continuidad. 1 y 2 incorrectas). La reactancia psicológi- ca es un con-

51
cepto acuñado por Brehm (1966, 1981). Este autor defiende serán psicológicamente signifi- cativas en una situación de-
que la amenaza o pérdida de alguna libertad suscita en la terminada. “Accesibilidad y ajuste como determinantes de
persona un estado motivacional dirigido a la recuperación la saliencia” (opción 3 correcta): Accesi- bilidad; hace refe-
de esa libertad amenazada o perdida (opción 4 incorrecta). rencia a la disposición de las personas a utilizar categorías
APIR Manual de Psicología Social, Psicología de los grupos, que son centrales, relevantes o útiles, en función de la ex-
Tema 3. periencia pasada, las expectativas presentes, y los motivos,
valores, metas y necesidades actuales; Ajuste, se refiere a la
174. ¿Cuál de los siguientes NO es un factor que favo- relación entre las categorías y la realidad externa, y se divide
rezca la aparición del denominado “pensamiento de en dos aspectos complementarios: Ajuste comparativo, que
grupo” según el modelo propuesto por Janis?: se basa en el principio según el cual las categorías que van
a ser más salientes son aquellas que maximizan la “razón
1. La alta cohesión grupal de meta-contras- te”, esto es, que hacen que las diferencias
2. La heterogeneidad de los miembros en actitudes e ideo- entre los grupos sean máximas y que las diferencias dentro
logía. del grupo sean mínimas en las dimensiones relevantes de
3. El aislamiento del grupo respecto de la in comparación. Ajuste norma- tivo, se refiere a que esas dife-
fluencia externa. rencias intergrupo y semejanzas intragrupo deben coincidir
4. La existencia de un contexto de alto estrés. con las expectativas normativas o es- tereotipos de dichas
categorías para que éstas sean salientes. Las opciones 1, 2 y
Respuesta correcta:2 4 (incorrectas) son distractoras. APIR Manual de Psicología
Social, Psicología de los grupos, Tema 4.
Janis (1972, 1977) estudió casos de tomas de decisión po-
líticas y militares catastróficas tomadas en grupos supues- 171. ¿Cuál de las siguientes opciones NO recoge una de
tamente muy inteligentes. Los individuos resultaron vícti- las estrategias de obtención de identidad social positiva
mas de manera extre- ma de polarización grupal, conocida pro- puestas por la Teoría de la identidad social?:
como pensamiento grupal. Se caracteriza por una percep-
ción exagerada de la “rectitud mo- ral” de los planteamien- 1. Creatividad social.
tos del grupo y una “visión homogénea y estereotipada” de 2. Movilidad individual.
los miembros del exogrupo. Ocurre cuando un grupo muy 3. Competición social.
cohesivo o de mentalidad semejante (opción 2 correcta) que 4. Facilitación social.
ha de tomar decisiones bajo un contexto de alto estrés (op-
ción 4 incorrecta), está tan condicionado por la bús- queda Respuesta correcta:4
de consenso que se deteriora su percepción de la realidad.
El proceso se refuerza bajo las siguientes condiciones: 1) La Teoría de la identidad social de Tajfel refiere tres estra-
Que el grupo sea altamente cohesivo (opción 1 incorrecta). tegias para enfrentarse a la identidad grupal negativa: 1)
2) Que esté privado de otras fuentes de información alter- Movilidad individual, consiste en dejar el grupo e intentar
nativas (opción 3 incorrecta). Que el líder apoye claramente pasar al más valorado (opción 2 incorrecta). 2) Creatividad
una determinada opción. APIR Manual de Psicología So- social, consiste en redefinir los términos de la situación
cial, Psicología de los grupos, Tema 3. comparativa adoptando una nueva dimensión de compara-
ción, modificando los valores asociados con determinados
170. Según la Teoría de la categorización del Yo, la “sa- atributos de grupo o cambiando el exogrupo con el que se
liencia” de una categoría social en una situación concre- hace la comparación (opción 1 incorrec- ta). 3) Competi-
ta es función de dos procesos denominados: ción social, consiste superar al exogrupo en la misma di-
mensión en la que era superior (opción 3 incorrecta). La fa-
1. Prototipicidad y homogeneidad. cilitación social consiste en la modificación de la conducta
2. Anclaje y comparación. del individuo por el hecho de estar siendo observado. Exis-
3. Accesibilidad y ajuste. ten diferentes teorías para explicar este fenómeno, la más
4. Atribución y heterogeneidad. conocida es la Teoría de la mera presencia de Zajonc que
propone que la presencia de otros llevaba a un incremento
Respuesta correcta: 3 del rendimiento, o facilitación social, si el individuo traba-
jaba en tareas fáciles y bien aprendidas (opción 4 correcta).
Según la Teoría de la categorización del yo (Turner 1982; APIR Manual de Psicología Social, Psicología de los grupos,
Turner y cols. 1987), la saliencia de las categorías hace re- Tema 4.
ferencia a las condiciones que conducen a las personas a
categorizarse en un nivel social, con preferencia a uno in-
dividual (o viceversa) en una situación concreta, así como
el nivel social que será más utilizado, esto es, qué categorías
sociales específicas, entre las diferentes categorías posibles,

52
TRATAMIENTO PSICOLÓGICO ción pausado) ya que era el componente que se había mos-
trado menos efectivo del programa (Opción 1 correcta). El
entrena- miento en Mindfulness y el cuestionamiento de
64. Las guías clínicas del Sistema Nacional de Salud las metapreo- cupaciones son técnicas que corresponden a
(guíasalud.es) establecen 8 niveles de evidencia (de 1++ terapias de tercera generación, que son usadas últimamen-
a 4) y 4 grados de recomendación (de “A” a “D”). ¿Qué te para múltiples trastornos, pero no ha sido incorporada
grado de recomendación suponen las “opiniones de los dentro de la terapia del control de pánico de Barlow (op-
expertos” y las “series de casos clínicos”?: ción 3 y 4 incorrectas). Por su parte, el trabajo con exposi-
ción en imaginación no es característico del tratamiento del
1. Nivel “B”. trastorno de pánico, que suele trabajarse con ex- posición
2. Nivel “C”. interoceptiva (opción 2 incorrecta). APIR Manual de Trata-
3. Nivel “D”. mientos Psicológicos, Tema 5.
4. No se consideran como evidencia científica, por lo que
esos aspectos están excluidos como criterios de cara a la
asigna- ción de niveles de recomendación. 40. Según Marlatt y Gordon (1985), en el efecto de vio-
lación de la abstinencia (EVA) se produce un fenómeno
Respuesta correcta:3 de:

“La Guía de Práctica Clínica sobre el manejo de la Depre- 1. Disonancia cognitiva


sión en el adulto del Sistema Nacional de Salud establece la 2. Atribución de responsabilidad a facto res externos incon-
propuesta de SIGN (Scottish Intercollegiate Guidelines Ne- trolables
twork) como mo- delo de clasificación de los niveles de evi- 3. Restablecimiento del consumo adicti vo hasta sus niveles
dencia clínica y de gra- dos de recomendación. El grado D iniciales.
se corresponde con evidencia científica de nivel 3 (estudios 4. Generación de expectativas de resulta do positivas.
no analíticos, como informes de casos y series de casos) o
4 (opinión de expertos) (opción 3 co- rrecta); o evidencia Respuesta correcta:1
científica extrapolada desde estudios clasifi- cados como 2+
(estudios de cohortes o de casos y controles bien realizados Desde el postulado de la prevención de recaídas de Marlatt
con bajo riesgo de sesgo y con una moderada proba- bili- y Gordon, se considera una clara diferenciación entre los
dad de establecer una relación causal). Biblioteca de Guías concep- tos caída y recaída. El efecto de violación de la abs-
de Práctica del Sistema Nacional de Salud. (2019). Niveles tinencia o EVA, estaría ligado a la recaída, mientras la caída
de evidencia y grados de recomendación del SIGN. Recu- sería un con- sumo aislado en un momento determinado.
perado de https://portal.guiasalud.es/egpc/depresion-adul- El EVA explica ese paso de la caída a la recaída. La inten-
to-niveles/ “ sidad del EVA dependerá del grado de compromiso que el
paciente tenga en mantener la abstinencia, la duración de la
42. ¿Qué componente se introdujo en las últimas ver- misma (cuanta más, mayor será el efecto) y el valor subje-
sio- nes del programa de tratamiento para el control del tivo que se le dé a la conducta. Por con- tra, una atribución
pánico de Barlow y Craske (2007)?: externa, inestable y controlable disminuye el EVA (opción
2 incorrecta). Se compone de dos elementos: 1) Disonancia
1. Re-entrenamiento en respiración. cognitiva, que entra en contradicción con el auto- concepto
2. Exposición en imaginación. de abstinente (opción 1 correcta); 2) Atribución perso- nal
3. Entrenamiento en mindfulness. interna, estable y global de la conducta de beber (opción 3
4. Cuestionamiento de las metapreocupaciones incorrecta). La opción 4 es un mero distractor. APIR Ma-
nual de Tratamientos psicológicos, Tema 3.
Respuesta correcta:1
62. La entrevista motivacional (EM), terapia desarro-
El programa de Barlow se centra más en la parte conduc- llada en sus orígenes para el tratamiento de las adiccio-
tual (exposición a las sensaciones interoceptivas temidas), nes, ha sido reformulada recientemente por sus autores
mientras que el programa de Clark tiene una orientación (Miller y Rollnick, 2013), quienes proponen que son
más cognitiva, y la inducción de sensaciones interoceptivas cuatro procesos los que componen la EM; estos son:
es utilizada como experimento conductual para la descon-
firmación de creencias erróneas posterior. Algunas de las 1. Vincular, enfocar, evocar y planificar.
técnicas para facilitar la expo- sición interoceptiva son la 2. Precontemplación, contemplación, acción y manteni
práctica de ejercicios cardiovascula- res, la hiperventilación miento.
y las inhalaciones de dióxido de carbono. En la última ver- 3. Empatía, discrepancia, discusión y autoeficacia.
sión del programa de Barlow, la relajación se ha sustituido 4. Evaluar, dialogar, escuchar, convencer.
por re-entrenamiento en respiración (patrón de respi- ra-

53
Respuesta correcta:1 poner a prueba la creencia generalizada de «No puedo con-
trolar mis voces». En primer lugar, esto se reformula como
Actualmente Miller y Rollnick (2013) proponen que el cómo «No puedo hacer que las voces aparezcan y desaparezcan».
y el por qué las personas cambian, sucede en un proceso re- Luego el terapeuta plantea si- tuaciones para aumentar y
cursivo en el que se superponen y confluyen cuatro tipos de luego disminuir la probabilidad de oír voces. Una evalua-
procesos: 1) Comprometiéndose: El primer proceso de la ción cognitiva inicial debería identificar las señales que
Entrevista Mo- tivacional es enganchar al paciente en una sirven para provocar las voces y una técnica que tiene una
relación de trabajo colaborativo. Es un proceso por el cual elevada probabilidad de eliminar las voces de forma du-
ambas partes establecen una conexión útil y una relación radera es la verbalización concurrente (Birchwood, 1986)
de trabajo; 2) Enfocándose: Es el proceso mediante el cual (opción 3 correcta). La persona aviva y reprime las voces
el terapeuta desarrolla y mantiene un curso específico en varias veces con el fin de proporcionar una prueba comple-
la conversación acerca del cambio. Se trata de ayudar a los ta. Caballo, V. (2007). Manual para el tratamiento cogniti-
pacientes a clarificar sus objetivos; 3) Evocando: Este pro- vo-conductual de los trastornos psicológicos. Madrid: Siglo
ceso por el cual el terapeuta induce la motivación del cliente XXI. Vol. 1. p 670.
hacia el cambio; y 4) Planeando: Es un proceso mediante
el cual el terapeuta ayuda al paciente a formular planes de 36. Dentro de las intervenciones psicológicas basadas
acción específicos para alcanzar el cambio deseado (opción en internet para personas con psicosis, ¿en qué mode-
1 correcta). Miller, W. y Rollnick, S. (2015). La entrevista lo de tera- pia se fundamenta el sistema online HORY-
motivacional (3a ed.). Paidós Ibérica. sión (Segal y cols.) ZONS?:

83. Señale cuál de los siguientes elementos no está pre- 1. Terapia web de aceptación y compromiso.
sente en el programa para el tratamiento de las adic- 2. Terapia online “afrontando voces”.
ciones comportamentales de Echeburúa (Echeburúa y 3. Terapia mindfulness basada en la web para alucinaciones
cols., 2005 y Echeburúa, 2009): auditivas.
4. Terapia social online moderada.
1. Análisis de los estímulos asociados a la adicción.
2. Control de estímulos asociados a la adicción. Respuesta correcta:4
3. Atención plena a los estímulos asociados a la adicción.
4. Exposición a estímulos asociados a la adicción. El proyecto HORYZONS nace con el propósito de facilitar
el acceso a intervenciones basadas en la evidencia a los jóve-
Respuesta correcta:3 nes con psicosis, haciendo uso de una página web interac-
tiva. Se trata de utilizar la tecnología digital para extender
El programa de Echeburúa y Báez (1994) para el para el tra- los benefi- cios de las tradicionales intervenciones psicoso-
tamiento de las adicciones consta de: 1) Control de estímu- ciales cara a cara. Este sistema comprende una plataforma
los (opciones 1 y 2 incorrectas); 2) Exposición con preven- que facilita el acceso a una amplia variedad de intervencio-
ción de respuesta o EPR (opción 4 incorrecta); y 3) Terapia nes psicosociales interacti- vas, potenciadas por un entorno
de grupo cognitivo-conductual (simultánea al tratamiento de red social online moderado por profesionales y usuarios
individual, 5-6 pacientes). APIR Manual de Tratamientos expertos que procura la interacción social y el apoyo mutuo
Psicológicos, Tema 2. (opción 4 correcta). Fonseca, E. (2019). Tratamientos psico-
lógicos para la psicosis. Madrid: Pirámide.
33. En la terapia cognitiva para las psicosis se propone
una evaluación inicial donde se deberían identificar las 37. ¿Cuál es el primer módulo que se administra cuan-
seña- les que sirven para disparar las alucinaciones au- do se realiza una intervención según la Terapia psicoló-
ditivas. Si el terapeuta pide a su cliente que intente avi- gica integrada para personas con esquizofrenia?:
var y reprimir las voces varias veces al día ¿qué técnica
está utilizando?: 1. Solución de problemas.
2. Diferenciación cognitiva.
1. Autoinstrucciones. 3. Memoria diferida.
2. Evaluación analítica funcional. 4. Entrenamiento en habilidades sociales.
3. Verbalización concurrente.
4. Condicionamiento encubierto. Respuesta correcta:2

Respuesta correcta:3 La IPT de Roder y Brenner (2007) es un programa de trata-


miento conductual para la esquizofrenia, grupal y jerarqui-
En la terapia cognitiva para la psicosis se emplean dos enfo- zado, que se compone de 5 módulos en los que se incluye:
ques para comprobar las creencias de forma empírica. Por la rehabili- tación cognitiva (módulos: 1. Diferenciación
una parte, tenemos un conjunto de procedimientos para cognitiva (opción 1 correcta), 2. Percepción social y 3. Co-

54
municación verbal) y el entrenamiento en competencia so- 1. La fototerapia es más eficaz si es administrada por la ma-
cial (módulos: 4. Entrenamien- to en habilidades sociales ñana.
(opción 4 incorrecta) y 5. Solución de problemas interper- 2. La hipersomnia y la avidez por hidratos de carbono son
sonales): Diferenciación cognitiva, intervi- niendo en las predictores de buena respuesta a la fototerapia.
habilidades de atención y formación de conceptos (opción 3. La intensidad de la luz en la fototerapia deberá estar
2 incorrecta). Percepción social, analizando los estímu- los siempre por debajo de los 8500 lux.
sociales. Comunicación verbal, mediante el entrenamien- 4. Es importante que la luz contenga poca ener- gía del es-
to en habilidades de comunicación. Habilidades sociales. pectro ultravioleta para minimizar los efectos secunda- rios
Solución de problemas interpersonales. La opción 3 es un de la fototerapia.
mero distractor. Esta pregunta es un clásico de los últimos
años dentro de las preguntas sobre los tratamientos de la Respuesta correcta:4
esquizofrenia. Concreta- mente, es esencial memorizar sus
etapas, el orden y los nombres. Así como no mezclar los El mecanismo de acción de la fototerapia consiste, en par-
contenidos que se abordan en la terapia con el título de sus te, en disminuir la depleción de triptófano presente en los
diferentes módulos. APIR Manual de Trata- mientos Psico- pacientes con TAE. La dosis inicial de administración debe
lógicos, Tema 3. de ser de 15- 20 min al día y gradualmente ir aumentando
hasta 30-45 min al día a 10000 lux (unidad photométrica)
38. Cuando se utilizan técnicas de focalización para el (opción 3 correcta). Los estudios sobre la hora de admi-
tratamiento cognitivo-conductual de las alucinaciones, nistración de la luminoterapia han encontrado resultados
la primera fase de intervención consiste en que el pa- contradictorios. Parece que las sesiones por la mañana pro-
ciente: ducen mejores resultados que las adminis- tradas por las
tardes, aunque estas sí son superiores al placebo. Gatón Mo-
1. Analice las creencias y pensamientos respect o a las voces. reno, M. A., González Torres, M. Á., & Gaviria, M. (2015).
2. Preste atención al contenido de las voces. Trastornos afectivos estacionales,” winter blues”. Re- vista
3. Dirija la atención a la forma y características físicas de de la Asociación Española de Neuropsiquiatría, 35(126),
las voces. 367-380.
4. Registre los sucesos que acontecen antes de las voces, así
como los pensamientos y voces que le siguen. 30. La Terapia interpersonal ha sido ampliamente apli-
cadaeneltratamientodeladepresión.¿Quécaracterizaala
Respuesta correcta:3 Terapia Interpersonal?:

La terapia cognitiva o Terapia de Focalización de Bentall, 1. La terapia interpersonal hace hincapié en de- tectar pen-
Had- dock y Slade (1994) Bentall afirma que la causa de las samientos automáticos interpersonales.
alucina- ciones es un fallo de la metacognición (teoría del 2. Enfatiza el papel de las experiencias interpe sonales de
déficit en la ha- bilidad metacognitiva de discriminación de la niñez.
la realidad de Slade y Bentall), por lo que se atribuyen suce- 3. Focalizarse en cambiar la personalidad que está generan-
sos internos autogenerados a una fuente externa. Consta de do los conflictos interpersonales.
tres fases: 1) Prestar atención a la forma y aspectos físicos de 4. Se basa en el modelo médico de la enferme- dad depre-
las voces, 2) Prestar atención al contenido de las voces (que siva.
suele reflejar preocupaciones que el paciente pueda tener)
y 3) Identificar las creencias y pensamien- tos tanto antece- Respuesta correcta:4
dentes como consecuentes, que están asociados a esas vo-
ces. También se añade frecuentemente una cuarta fase que de la Asociación Española de Neuropsiquiatría, 35(126),
se caracteriza por la búsqueda del significado que tienen las 367- 380.
voces para el paciente, teniendo en cuenta su historia pa- La terapia interpersonal de Klerman y Weissman (1984) se
sada y los sucesos que están ocurriendo en el presente. Se plan- teó inicialmente desde una postura eminentemen-
trata de otra pregunta también calló el año pasado en el exa- te psiquiátrica, con la intención de prevenir las recaídas
men PIR 2018. Además, esta teoría lleva siendo objeto de en trastornos afectivos depresivos tras el tratamiento con
preguntas PIR desde el año 2012 de forma recurrente, por antidepresivos (opción 4 co- rrecta). Sin embargo, dada la
lo que su contenido debe ser temario a machacar durante eficacia del tratamiento (demos- trado por estudios tales
el estudio PIR. Además, es bas- tante útil manejar los dife- como el conocido estudio NIMH) se planteó la posibilidad
rentes sinónimos de los nombres de las terapias para iden- de usarlo como tratamiento psicológico para los trastor-
tificarlas durante el examen. APIR Manual de Tratamientos nos afectivos, y numerosos profesionales lo uti- lizan en su
Psicológicos, Tema 3. práctica clínica. El impacto del tratamiento ha sido tal ha
sido extrapolado a otras aplicaciones, como su uso en ado
29. En relación al tratamiento del trastorno afectivo es- lescentes o en Trastorno Bipolar. El principal objetivo de
tacional (TAE), señale la opción FALSA: esta terapia es la mejora de las relaciones interpersonales

55
(opción 3 incorrecta) centrándose en 4 áreas problemáti- que am- plía la propuesta de Heimberg en cuanto a compo-
cas (opción 1 in- correcta): Duelo, disputas interpersonales, nentes de la terapia. De forma que mantiene los tres pilares
transición de rol y déficits interpersonales. Los problemas característicos del Gold Standard de Heimberg: Reestructu-
que aborda esta terapia profundizan en la infancia del pa- ración Cognitiva, Exposición en vivo y Tareas para casa, a
ciente, sino que se aborda con técnicas centradas en proble- lo que le añade tres nue- vos componentes: Psicoeducación,
mas tangibles del momento actual (opción 2 incorrecta). El Videofeedback y Desviación de la atención. En este caso,
contenido de este ítem fue preguntado en el examen PIR la traducción al castellano de Vi- deofeedback es retroali-
2018, por lo que lleva siendo preguntado dos años seguidos, mentación en vídeo (opción 3 correcta). El entrenamiento
es un dato muy relevante a tener en cuenta. APIR Manual en habilidades sociales es un componente que añaden otros
de Tratamientos Psicológicos, Tema 4. tratamientos cognitivo-conductuales para la fobia social,
como la TCCC de Davidson, el Set de Turner o la IJAFS
182.¿Cuál de las siguientes características clínicas de un de Olivaries, pero no es un componente del tratamiento de
paciente con depresión “endógena” (unipolar o bipolar) McE- voy (opción 1 incorrecta). El entrenamiento en re-
NO constituye una indicación para iniciar tratamiento lajación no es un componente que añada ninguno de los
con tera- pia electroconvulsiva?: tratamientos para la fobia social, siendo más típico de los
tratamientos del trastorno de pánico (no exentos de crítica)
1.La presencia de grave desnutrición o deshidratación im- (opción 2 incorrecta). El bio- feedback podría ser un dis-
portantes. tractor si leemos la pregunta con pri- sa, ya que claramente
2.Haber padecido tres recidivas. intentan confundirnos, pretendiendo hacer pasar la opción
3.Antecedentes de fracaso farmacológico en su trata mien- por “videofeedback”, pero no lo es (opción 4 incorrecta). El
to. biofeedback es una técnica conductual que no se usa para
4.Riesgo grave de suicidio. Fobias Sociales en las TCC más usadas. APIR Manual de
Tratamientos psicológicos, Tema 5.
Respuesta correcta:2
44. En el programa de tratamiento del trastorno de an-
“Dada la eficacia de los antidepresivos actuales para el tra- siedad generalizada del grupo de Dugas, ¿cuándo se
tamien- to de la Depresión, se recomienda la TEC sólo en emplea la exposición funcional cognitiva?:
los siguientes casos: 1) Riesgos serios de suicidio (opción
4 incorrecta); 2) De- presiones con importante inhibición 1. Ante preocupaciones que se refieren a acontecimientos
o agitación psicomotora, e ideas delirantes; 3) Grave com- basados en la realidad o altamente probables.
promiso del estado general (des- nutrición, deshidratación) 2. Ante preocupaciones que se refieren a acontecimientos
(opción 1 incorrecta) que requieren de una intervención rá- altamente improbables.
pida y eficaz; 4) Fracaso de los antidepresivos, al menos dos, 3. Ante preocupaciones que se refieren a acontecimientos
de distinta familia, por tiempo y dosis suficientes (opción 3 modificables.
incorrecta); 5) Contradicciones para el uso de antide- presi- 4. Ante preocupaciones sobre la generación de soluciones
vos por enfermedades (cardiovasculares, hepáticas, etc.), o alternativas.
porque durante el tratamiento aparezcan efectos colaterales
muy intensos; y 6) Historia personal de buena respuesta a Respuesta correcta:2
TEC y/o escasa respuesta a los antidepresivos en episodios
previos. Cas- tellón, D. S., López, O. F., Jiménez, J. R. C., El tratamiento cognitivo conductual de Dugas y Ladoceur
Sánchez, I. R., San- tana, L. C., Cruz, R. Á. B. & González, (1997) tiene una evidencia E3 de acuerdo con Pérez (2003).
D. E. M. (2007). Manual para el tratamiento electroconvul- Su trata- miento propone una primera fase de análisis con-
sivo. MediSur, 5(2), 179-186. “ ductual del pro- blema que se trata (real y soluble, real e
irresoluble o irreal) y el trabajo en darse cuenta de ello. En
43. En el tratamiento para el trastorno de ansiedad so- una segunda fase se plantean las diferentes intervenciones
cial, ¿qué incluye la Terapia cognitivo-conductual en sobre los diferentes tipos de preo- cupaciones: En el caso de
grupo de McEvoy?: que sean problemas reales y solubles, nos centraremos en el
problema (opción 3 incorrecta), en caso de que sean reales
1. Entrenamiento en habilidades sociales. e irresolubles en la emoción y, finalmente, si son problemas
2. Entrenamiento en relajación. irreales nos centraremos en el uso de la exposición funcio-
3. Retroalimentación en vídeo. nal cognitiva (opción 1 incorrecta, opción 2 correcta), que
4. Biofeedback. consiste en aplicar una flecha descendente y exposición con
pre- vención de respuesta encubierta. En su última ver-
Respuesta correcta:3 sión (2007), este tratamiento añade el reconocimiento de
la incertidumbre, la exposición y la prevención de recaídas
La TCCG de McEvoy es un tratamiento cognitivo conduc- como componentes de su terapia. La opción 4 es un distrac-
tual eminentemente grupal con una duración de 7 sesiones tor. APIR Manual de Trata- mientos Psicológicos, Tema 5.

56
45. En el tratamiento cognitivo-conductual del trastor- 3. Tratamiento de la ansiedad por la salud de Warwick y
no de ansiedad generalizada, se “enseña” a los pacientes Salkovskis (1990, 2001).
que la preocupación patológica o improductiva consiste 4. Tratamiento cognitivo de las ideas delirantes (de tipo so-
en: mático) de Chadwick (2009)

1. Pensamientos, imágenes, impulsos o sensaciones físi- cas Respuesta correcta:3


desagradables, de carácter intruso, involuntario e indesea-
do. El tratamiento de la ansiedad por la salud de Warwick y
2. Una cadena de pensamientos sobre problemas inmedia- Salko- vskis (1990, 2001) (Opción 3 correcta) es un progra-
tos y realistas, centrada en la resolución de estos o de la si- ma cog- nitivo-conductual estructurado y creado para los
tua- ción. trastornos de ansiedad por la enfermedad (hipocondría).
3. Pensamientos negativos, autorreferenciales, vinculados Dicho programa se basa en las interpretaciones catastrófi-
habitualmente con eventos del pasado y que aparecen de cas y los supuestos disfun- cionales, consiste en ayudar al
forma automática. paciente a detectar y cambiar los pensamientos automáticos
4. Una cadena de pensamientos sobre problemas distales o negativos sobre sus síntomas físicos, las creencias desadap-
abstractos, que se centra en la emoción negativa asociada tativas sobre la salud y enfermedad y las conductas proble-
con la situación objeto de preocupación. máticas. El resto de opciones están creadas para otro tipo
de trastornos. El tratamiento del trastorno de pánico de D.
Respuesta correcta:4 Barlow (2001) (opción 1) está diseñada para el trastorno de
pánico. El tratamiento cognitivo conductual de la somati-
Las últimas investigaciones muestran que las TCC son su- zación de Woolfolk y Allen (2006) (Opción 2) para el tras-
perio- res a la farmacoterapia con benzodiacepinas mien- torno de sín- tomas somáticos. El tratamiento cognitivo de
tras que és- tas no parecen aportar beneficios considerables las ideas delirantes (de tipo somático) de Chadwick (2009)
cuando se las usa junto a la TCC. Barlow específica que las (opción 4) está destinado a la reducción de las creencias de-
más eficaces son aquellas que combinan las TCC con re- lirantes. No obstante, esta pre- gunta puede ser controver-
lajación y cuyo objetivo es colocar el proceso de preocu- tida ya que no queda claro el diagnós- tico de hipocondría
pación bajo control del cliente, enseñando al paciente que del caso expuesto en el enunciado, el criterio B del DSM
la preocupación patológica improduc- tiva es una cadena descarta el diagnóstico si existen síntomas como los descri-
de pensamientos sobre problemas distales o abstractos, que tos, por lo que podría ser acertado el diagnóstico de T. de
se centra en la emoción negativa asociada con la situación síntomas somáticos (siendo la opción correcta posiblemen-
objeto de preocupación (opción 4 correcta). APIR Manual te co- rrecta). APIR Manual de Tratamientos Psicológicos,
de Tratamientos Psicológicos, Tema 5. Tema 8.

46. Andrea, de 35 años de edad, presenta una elevada 47. ¿Cuál es el componente fundamental de la Terapia
ansiedad cada vez que nota presión en la cabeza y una cognitiva específica para el tratamiento del trastorno
leve sensación de mareo, lo que la ha llevado en varias obse- sivo-compulsivo (como, por ejemplo, en la pro-
ocasio- nes al servicio de urgencias por miedo de sufrir puesta de Be- lloch et al., 2011)?:
un “ictus o un infarto”, donde le han descartado orga-
nicidad, diagnos- ticándola de ansiedad y “somatiza- 1. La exposición y prevención de respuesta a las situacio-
ciones” y le han pautado ansiolíticos. Desde la primera nes temidas por el paciente.
ocasión, vigila cada vez más su cuerpo, ha evitado hacer 2. La exposición y prevención de respuesta a los conteni-
ejercicio para no marearse y, cada vez que lee o escucha dos de los pensamientos u obsesiones principales.
hablar de enfermedades, apaga la TV o sale de la con- 3. El entrenamiento en técnicas de control de la activación
versación porque refiere que “noto todos los síntomas (por ejemplo, relajación).
que escucho, como si los tuviera”, de hecho, en la última 4. El trabajo con las valoraciones y creencias disfunciona-
semana ha ido al médico por notarse los ojos enroje- ci- les asociadas a las obsesiones.
dos y un dolor fluctuante pero
molesto en el costado. A pesar de consultar sus sínto- Respuesta correcta:4
mas des- confía de los médicos porque su madre falle-
ció cuando ella tenía 14 años, “porque los médicos no “El componente fundamental de la Terapia cognitiva espe-
supieron lo que le pa- saba”. Según la sintomatología cífica para el tratamiento del trastorno obsesivo-compulsi-
de Andrea, ¿qué programa de tratamiento sería el más vo es el tra- bajo con las valoraciones y creencias disfun-
indicado?: cionales asociadas a las obsesiones (Opción 4 Correcta).
Tanto la opción de respuesta 1 como la 2 nos mencionan
1. Tratamiento del trastorno de pánico de D. Barlow (2001). la exposición con prevención de respuesta, aunque la EPR
2. Tratamiento cognitivo conductual de la somatización de se considera eficaz (E1) y constituye el tratamiento de elec-
Woolfolk y Allen (2006). ción, la terapia cognitiva en el contexto del tratamiento del

57
TOC aparece bien como un suplemento que potencia-faci- 184. La “exposición prolongada” propuesta por E. Foa
lita las técnicas conductuales como la EPR, bien como trata- y cols., se desarrolló fundamentalmente para el trata-
miento principal que incluye como experimento con- duc- miento:
tual la EPR con el objetivo de trabajar con las valoraciones
y creencias disfuncionales asociadas a las obsesiones. Por 1. Del trastorno de pánico.
otro lado, el entrenamiento en técnicas de control de la ac- 2. De la agorafobia.
tivación como la relajación (Opción 3) no sería el objetivo 3. Del trastorno de estrés postraumático.
principal en este tiempo de tratamiento. APIR Manual de 4. Del trastorno obsesivo compulsivo.
Tratamientos Psi- cológicos, Tema 6.
Respuesta correcta:3
48. En pacientes con diagnóstico de trastorno obsesivo
compulsivo ¿en qué caso estaría más indicado aplicar La exposición prolongada en imaginación de los recuerdos
un componente de exposición con prevención de res- te- midos es el tratamiento de elección y E1 para el TEPT
puesta?: (opción 3 correcta); para las situaciones que son evitadas,
se utiliza la ex- posición en vivo. El grupo de Foa y cols.,
1. Paciente con obsesiones de tipo sexual (por ejemplo, desde su modelo teóri- co, habla de que lo que funciona de
prácticas sexuales no deseadas o inmorales). la exposición en imaginación es que posibilita el reprocesa-
2. Paciente con obsesiones agresivas hacia sí mismo o los miento emocional del trauma, más que la habituación que
otros. se produce a la ansiedad. APIR Manual de Tratamientos
3. Paciente con obsesiones sobre la contaminación o sucie- Psicológicos, Tema 7.
dad y/o compulsiones de lavado y/o limpieza.
4. Paciente con obsesiones blasfemas (por ejemplo, insul- 181. Respecto del tratamiento con antidepresivos para
tar a Dios). la bulimia nerviosa, cabe afirmar que:

Respuesta correcta:3 1. Produce una mayor reducción de los atracones que los
tratamientos psicológicos, a largo plazo.
Podemos encontrarnos obsesiones con o sin conductas 2. Muestra peores resultados que la Terapia cognitivo con-
compulsi- vas manifiestas. Con respecto a las obsesiones ductual.
sin compulsiones (Opción 1, 2 y 4), Salkovskis y Westbrook 3. Los antagonistas opiáceos son más eficaces que los an-
proponen la técnica de habituación ante los pensamientos tidepresivos.
obsesivos, siendo el ele- mento clave del tratamiento la pre- 4. Los antidepresivos no han mostrado ser eficaces para la
dictibilidad de los estímulos a los que se expone el sujeto. reducción de atracones y purgas a corto plazo.
Con respecto a las obsesiones con conductas compulsivas
(opción 3) si estaría más indicado aplicar un componente Respuesta correcta:2
de exposición con prevención de respuesta que consiste en
la exposición a los estímulos externos o internos que pro- Al contrario que en anorexia, los fármacos pueden jugar
vocan las obsesiones bloqueando la realización de los ritua- un papel importante en el caso de la bulimia, siendo los
les. APIR Manual de Tratamientos Psicológicos, Tema 6. más destacados los antidepresivos. Diversos estudios con-
firman que se reducen los atracones y mejora el estado de
70. La Desensibilización y reprocesamiento por movi- ánimo, aunque parece que los efectos antibulímicos serían
mientos oculares (EMDR) en su aplicación a niños: independientes de los efectos anti- depresivos, ya que para
que se den dichos efectos antibulímicos se requieren do-
1. Consta de 8 fases, igual que en adultos. sis más altas que para el tratamiento de la depre- sión (60
2.Sólo utiliza la fase de desensibilización o reprocesa mien- mg/día frente a 20 mg/día en el caso de la depresión). Si
to. comparamos el tratamiento farmacológico con el resto de
3.Reduce las 8 fases a cuatro. trata- mientos para la bulimia, la TCC sigue siendo más
4.Prescinde de la intervención sobre aspectos cognitivos. eficaz aunque se puede ver potenciada por el uso de antide-
presivos (opción 2 correcta). APIR Manual de Tratamientos
Respuesta correcta:1 Psicológicos, Tema 11.

La EMDR es una técnica que consta de 8 fases, tanto para 52. La técnica de parada-arranque en la que el pacien-
ni- ños como para adultos, (opción 1 correcta) en las que te estimula su pene hasta conseguir la erección, después
el sujeto se expone al recuerdo traumático mientras recibe se detiene hasta que vuelva al estado de flaccidez y de
estimulación bilateral. APIR Manual de Tratamientos Psi- nuevo vuelve a estimularlo, está indicada ante proble-
cológicos, Tema 7. mas de :

1. Bajo deseo sexual.

58
2. Aversión al sexo. antihistamínicos). Con respecto al TOC (Opción 1), la ex-
3. Excitación (en el hombre). posición con prevención de respuesta, se considera eficaz
4. Anorgasmia. (E1) y de elección. Para el tra- tamiento para el trastorno de
personalidad obsesivo-compulsivo una de las técnicas que
Respuesta correcta:3 más se usan al principio es la solución de problemas para
empezar a trabajar y modificar algunos de los comporta-
La técnica de parada-arranque ha sido ampliamente utili- mientos rígidos del pacientes, así como su pensamien- to
zada para el tratamiento de la eyaculación precoz. El DSM- dicotómico. Por último, no existen hoy en día tratamientos
IV cla- sifica las disfunciones sexuales basándose (con al- empíricamente establecidos para el trastorno dismórfico
gunas modi- ficaciones) en la diferenciación de fases en la corpo- ral (opción 4), la TCC de Rosen es el único trata-
respuesta sexual establecida por Masters y Johnson (1966), miento recogido para este trastorno y cuenta con una efi-
diferenciando cuatro grupos fundamentales de trastornos: cacia E3. APIR Manual de Tratamientos Psicológicos, Tema
del deseo sexual (TDS) (TDS hipoactivo y Trast. aversión 14.
al sexo), de la excitación se- xual (TES) (TES mujer y Trast.
erección (hombre)), del orgasmo (Disf. orgásmica feme- 50. Entre los principios en los que se sustenta la psicote-
nina, Disf. orgásmica masculina y Eya- culación precoz) y rapia de los trastornos de la personalidad se encuentra
trastornos sexuales por dolor (Dispareunia y Vaginismo). el de “gradualidad”, que hace referencia a:
La opción dada por correcta pero que finalmente fue anu-
lada es la 3, la eyaculación precoz no está clasificado dentro 1. Promover cambios en los aspectos o manifestaciones que
de los trastornos de la excitación sexual sino dentro de los conllevan un mayor nivel de perturbación e interferencia en
trastornos del orgasmo. APIR Manual de Tratamientos Psi- la vida del paciente, para ir progresando paulatinamente en
cológicos, Tema 12. la mejora de su calidad de vida.
2. Establecer una jerarquía de principios y necesidades de
115. En el síndrome de la apnea/hipopnea obstructiva cambio y acordar con el paciente la dificultad de cada uno
del sueño (SAHOS) moderado a grave, con sintomatolo- de ellos para ir avanzando en función de ese gradiente.
gía (somnolencia/diurna) el tratamiento de elección es: 3. Exponerse a las situaciones que provocan malestar, co-
menzando por las que generan un malestar intermedio.
1. Un fármaco hipnótico. 4. Promover cambios en los aspectos o manifestaciones más
2. Un fármaco ansiolítico. periféricas del trastorno para ir progresando paulatinamen-
3. La presión positiva continua en la vía aérea. te hacia la consecución de cambios en los patrones más bá-
4. El neurofeedback. sicos de la organización personal.

Respuesta correcta:3 Respuesta correcta:4

La presión nasal positiva es un tratamiento ideado por Su- Los objetivos de la psicoterapia ante un TP deben enmar-
llivan, Berthon-Jones, Issa y Eves. Se trata de una mascarilla carse dentro de un plan de confirmación de la experiencia
unida a un generador de aire que administra un flujo conti- disfuncio- nal, respetando el modo de ser del paciente. Des-
nuo de aire a más presión sobre las vías respiratorias supe- de esta pers- pectiva, el principio de gradualidad hace refe-
riores, para prevenir la aparición de apneas. Se considera el rencia a que los cambios deben ser graduales, progresando
tratamiento de elección (opción 3 correcta) ya que produce desde las manifesta- ciones periféricas hacia los patrones
mejoría tanto en la sinto- matología diurna como nocturna. básicos de la organización personal (opción 4 correcta). Be-
APIR Manual de Tratamientos Psicológicos, Tema 13. lloch, A. (2010). Tratado de los trastornos de la personali-
dad. Editorial Síntesis. pp 297.
49. Señale para cuál de los siguientes trastornos está
más indicado el entrenamiento en reversión del hábito: 73. La técnica de Inversión del hábito (Azrin y Nunn):

1. Trastorno obsesivo-compulsivo. 1. Ha demostrado su eficacia en los trastornos por tics en


2. Tricoticolomanía. niños, tanto a corto como a largo plazo.
3. Trastorno de la personalidad obsesivo-compulsivo. 2. No es una técnica psicológica eficaz, ya que tiende a in-
4. Trastorno dismórfico corporal. crementar los tics a largo plazo.
3. Está indicada en el tratamiento del trastorno obsesi-
Respuesta correcta:2 vo-compulsivo infantil.
4. Se basa fundamentalmente en la relajación de Jacob- son.
El procedimiento más recomendado para la tricotilomanía
(Opción 2) es la terapia de conducta e inversión del hábito, Respuesta correcta:1
a lo que se pueden sumar medidas farmacológicas (ISRS,
antipsicóticos o litio) y medidas dermatológicas (esteroides, El único tratamiento empíricamente validado para el tras-

59
torno por tics es la inversión o reversión del hábito (opción cionamiento operante.
1 correcta), y es el que se ha demostrado más eficaz. APIR 3. Se basa, fundamentalmente, en los principios del bio-
Manual de Trata- mientos Psicológicos, Tema 20. feedback.
4. No existe un tratamiento exclusivamente conductual de
74. ¿Cuál de las siguientes técnicas está dirigida a tra- la encopresis funcional.
bajar las conductas impulsivas en niños?:
Respuesta correcta:2
1. Técnica del árbol.
2. Técnica de la alfombra mágica. El entrenamiento en el uso del baño es un procedimiento
3. Técnica del águila. mul- ticomponente en el que intervienen el reforzamien-
4. Técnica de la tortuga. to positivo, el control de estímulos, instigadores, desvane-
cimiento y manejo del ambiente. El objetivo es enseñar al
Respuesta correcta:4 niño a defecar de forma regular en el inodoro. Bragado,
basándose fundamentalmente en los principios del condi-
La técnica de la tortuga de López Soler y Martínez Núñez cionamiento operante (opción 2 correc- ta), recomienda
(1999), que consiste en enseñar a los niños a relajarse a tra- no sólo reforzar al niño cuando se produzca la defecación,
vés de la analogía de “meterse en su caparazón” como hacen sino también reforzar la conducta de sentarse en el inodoro.
las tor- tugas cada vez que sientan frustración, ira o enfado APIR Manual de Tratamientos Psicológicos, Tema 24.
(opción 4 correcta), y una vez aprendan esto se realiza un
entrenamien- to en resolución de problemas interpersona- 77. Señale la respuesta correcta con respecto al progra-
les. Esta técnica se utiliza con niños más pequeños (Edu- ma ACCIÓN (Stark y cols., 2010) para el tratamiento
cación Infantil y primeros cursos de Educación Primaria). grupal de la depresión mayor:
APIR Manual de Tratamientos Psicológicos, Tema 21.
1. La población diana son adolescentes de entre 13 y 18
76. En el tratamiento de la ansiedad por separación y años.
ansiedad generalizada en la infancia, para poder aplicar 2. Utiliza planificación de actividades placenteras y de do-
la técnica de relajación progresiva, Cautela y Groden minio y el entrenamiento en habilidades de autocontrol de
(1989) se- ñalan unas habilidades mínimas para realizar la ira.
los ejercicios de relajación. Entre estas se encuentran: 3. No tiene en cuenta el uso de técnicas de reestructura-
ción cognitiva adaptadas a niños.
1. Permanecer quieto durante cinco segundos. 4. Las habilidades de solución de problemas no forman par-
2. Mantener el contacto ocular durante diez segundos. 3. te del programa.
Cumplir instrucciones complejas.
4. Imitación de habilidades motoras complejas. Respuesta correcta:2

Respuesta correcta:1 El programa ACTION de STARK Y KENDALL fue creado


para niños de 8 a 13 años (opción 1 incorrecta) e incluye
Para poder aplicar la técnica de relajación progresiva, Cau- las siguien- tes técnicas: 1) Actividades agradables; 2) Auto-
tela y Groden (1989) señalan unas habilidades mínimas que control (opción 2 correcta); 3) Relajación; 4) Reestructura-
deberá ad- quirir el niño antes de comenzar la enseñanza de ción cognitiva (opción 3 incorrecta); 5) Entrenamiento en
la relajación: 1) Permanecer quieto durante cinco segundos asertividad; 6) Solución de problemas (opción 4 incorrec-
(opción 1 correcta); 2) Mantener la mirada durante cinco ta); 7) Solución de problemas in- terpersonales; 8) Educa-
segundos (opción 2 inco- rrecta); 3) Ser capaz de imitar ac- ción emocional; y 9) Autoinstrucciones. APIR Manual de
ciones sencillas como levantar su mano por encima de la Tratamientos Psicológicos, Tema 27.
cabeza, tocar la mesa y tocarse el pe- cho (opción 4 inco-
rrecta); y 4) Obedecer órdenes sencillas como “levántate”, 79. “Imagina que eres un gato que se despereza, esti-
“siéntate” y “ven aquí” (opción 3 incorrecta). Cau- tela, J. R. rando sus patas y arqueando el lomo”. ¿A qué corres-
& Groden, J. (1989). Técnicas de relajación (Manual prác- ponde esta instrucción?:
tico para adultos, niños y educación especial). Barcelona:
Ediciones Martínez Roca, S. A., p.68. 1. Ésta es una instrucción que forma parte del entrena-
miento en respiración profunda, para el control de la acti-
72. El tratamiento conductual de la encopresis funcio- vación fisiológica excesiva en niños.
nal (por ejemplo, Bragado, 2001): 2. Forma parte del entrenamiento en autoinstrucciones
(Meichembaum y Goodman, 1971).
1. Se basa fundamentalmente en los principios del com- 3. Forma parte de las instrucciones del entrenamiento en
portamiento clásico. relajación de Koeppen (1974), una variante de la relajación
2. Se basa fundamentalmente en los principios del condi- pro- gresiva.

60
4. Forma parte de las autoinstrucciones utilizadas en el pro- 56. El entrenamiento para el uso del “peak flow meter”
grama Coping Cat de Kendall (1990). forma parte de los programas de automanejo de:

Respuesta correcta:3 1. El colon irritable.


2. El asma.
Una variante de la relajación progresiva que incluye el em- 3. La diabetes.
pleo de imágenes que ayudan al niño a tensar cada grupo 4. Los problemas cardiovasculares.
muscular y a mantenerlo motivado hacia el ejercicio, es el
entrenamiento en imaginación de Koeppen (1974). Se na- Respuesta correcta:2
rran varias historias que el niño va escuchando, y en un
momento de ellas debe tensar una zona de su cuerpo. Por Un programa de automanejo para pacientes con asma sería
ejemplo, para relajar brazos y hombros, la instrucción sería: el medidor de Peak-flow (opción 2 correcta), una técnica de
«imagina que eres un gato que se despereza, estirando sus ma- nejo utilizada en atención primaria. El flujo espiratorio
patas y arqueando el lomo» (opción 3 correcta). Koeppen, máximo (FEM) o peak expiratory flow (PEF) en termino-
A. S. (1974). Relaxation training for children. Ele- mentary logía anglosa- jona, es el mayor flujo que se alcanza durante
School Guidance & Counseling, 9(1), 14-21. una maniobra de espiración forzada. Se consigue al haber
espirado el 75-80% de la capacidad pulmonar total. Entre
80. ¿Cuál de los siguientes NO es un programa de tra- sus ventajas, obvian la ne- cesidad de registro manual de
tamiento para el trastorno de ansiedad generalizada en los valores por parte del paciente, aumentan la exactitud del
la infancia?: registro y graban el momento del día en que se realiza la
maniobra. Su principal inconveniente es su alto precio. Me-
1. El programa de tratamiento Coping Cat. didor de Peak-flow: técnica de manejo y utilidad en Aten-
2. El programa FRIENDS. ción Primaria. Medifam vol.12 no.3 mar. 2002.
3. El programa de tratamiento Coping Bear.
4. El programa de intervención de Herbert.

Respuesta correcta:4

El programa de intervención de Herbert está diseñado para


pa- dres (opción 4 correcta) que tienen dificultades para
manejar eficazmente el comportamiento de sus hijos, sien-
do el principal objetivo dotarles de información y habilida-
des para conseguir de ellos un estilo educativo democráti-
co. Coping Cat, Coping Bear y FRIENDS sí son programas
para el tratamiento de la ansiedad generalizada en la infan-
cia (opciones 1, 2 y 3 incorrectas). APIR Manual de Trata-
mientos Psicológicos, Tema 27.

58. ¿Para qué tipo de enfermedad se diseñó la Terapia


psicológica adyuvante de Moorey y Greer (1989)?:

1. Cáncer.
2. VIH.
3. Diabetes.
4. Fibromialgia.

Respuesta correcta:1

“La terapia psicológica adyuvante (TPA) de Greer ha sido


dise- ñada específicamente para pacientes con cáncer. Se
trata de una adaptación de la terapia cognitiva de Beck y
parte de la idea que el afrontamiento de la enfermedad de-
pende del significado que la persona le dé a ésta y de su
estilo de afrontamiento. Se trata de un programa flexible de
alrededor de 6 sesiones que puede divi- dirse en tres fases,
cada una de ellas con objetivos diferenciados. APIR Manual
de Tratamientos Psicológicos, Tema 32.

61

También podría gustarte